Determinants Class 12 Maths Important Questions Chapter 4

Determinants Exercise 4.1

Question 1. \(\left|\begin{array}{cc}
2 & 4 \\
-5 & -1
\end{array}\right|\)

Solution:

Let A=\(\left|\begin{array}{cc}2 & 4 \\ -5 & -1\end{array}\right| \Rightarrow|A|=2 \times(-1)-(-5) \times\) 4=-2+20=18

Question 2. 1.\(\left|\begin{array}{cc}\cos \theta & -\sin \theta \\ \sin \theta & \cos \theta\end{array}\right|\)

2. \(\left|\begin{array}{cc}x^2-x+1 & x-1 \\ x+1 & x+1\end{array}\right|\)

Solution:

⇒ \(\left|\begin{array}{cc}
\cos \theta & -\sin \theta \\
\sin \theta & \cos \theta
\end{array}\right|\)

=\((\cos \theta)(\cos \theta)-(\sin \theta)(-\sin \theta) \)

=\(\cos ^2 \theta+\sin ^2 \theta=1\left (\sin ^2 \theta+\cos ^2 \theta=1\right)\)

⇒ \(\left|\begin{array}{cc}
x^2-x+1 & x-1 \\
x+1 & x+1
\end{array}\right|=\left(x^2-x+1\right)(x+1)-(x+1)(x-1)\)

= \(x^3-x^2+x+x^2-x+1-\left(x^2-1\right)=x^3+1-\left(x^2-1\right)=x^3-x^2+2\)

Question 3. If A= \(\left[\begin{array}{ll}
1 & 2 \\
4 & 2
\end{array}\right] \)
, then show that |2 A|=4|A|

Solution:

Given, A=\(\left[\begin{array}{ll}
1 & 2 \\
4 & 2
\end{array}\right]\)

2 A=2\(\left[\begin{array}{ll}
1 & 2 \\
4 & 2
\end{array}\right]=\left[\begin{array}{ll}
2 \times 1 & 2 \times 2 \\
2 \times 4 & 2 \times 2
\end{array}\right]=\left[\begin{array}{ll}
2 & 4 \\
8 & 4
\end{array}\right] \)

LHS =|2 A|=\(\left[\begin{array}{ll}
2 & 4 \\
8 & 4
\end{array}\right]=(2 \times 4)-(8 \times 4)\)=-24

Read and Learn More Class 12 Maths Chapter Wise with Solutions

Now, |A|=\(\left|\begin{array}{ll}
1 & 2 \\
4 & 2
\end{array}\right|=(1 \times 2)-(4 \times 2)\)=-6

RHS =4|A|=4 \(\times(-6)\)=-24

LHS = RHS

Hence; |2A|=4|A|

CBSE Class 12 Maths Chapter 4 Determinants Important Question And Answers

Question 4. If A=\(\left[\begin{array}{lll}
1 & 0 & 1 \\
0 & 1 & 2 \\
0 & 0 & 4
\end{array}\right]\)
 , then show that |3 A|=27|A|

Solution:

Given, A=\(\left[\begin{array}{lll}
1 & 0 & 1 \\
0 & 1 & 2 \\
0 & 0 & 4
\end{array}\right]\)

then 3 A=\(\left[\begin{array}{ccc}
3 & 0 & 3 \\
0 & 3 & 6 \\
0 & 0 & 12
\end{array}\right]\)

Now; \(|\mathrm{A}|=\left|\begin{array}{lll}
1 & 0 & 1 \\
0 & 1 & 2 \\
0 & 0 & 4
\end{array}\right|\)=1(4-0)-0(0)+1(0)=4

⇒ \(|3 \mathrm{~A}|=\left|\begin{array}{lll}
3 & 0 & 3 \\
0 & 3 & 6 \\
0 & 0 & 12
\end{array}\right|\)=3(36-0)-0(0)+3(0)=108

⇒ \(|3 \mathrm{~A}|=108=27 \times 4=27|\mathrm{~A}|\),Hence proved.

Question 5. Evaluate the determinants.

  1. \(\left|\begin{array}{ccc}3 & -1 & -2 \\ 0 & 0 & -1 \\ 3 & -5 & 0\end{array}\right|\)
  2. \(\left|\begin{array}{ccc}3 & -4 & 5 \\ 1 & 1 & -2 \\ 2 & 3 & 1\end{array}\right|\)
  3. \(\left|\begin{array}{ccc}0 & 1 & 2 \\ -1 & 0 & -3 \\ -2 & 3 & 0\end{array}\right|\)
  4. \(\left|\begin{array}{ccc}2 & -1 & -2 \\ 0 & 2 & -1 \\ 3 & -5 & 0\end{array}\right|\)

Solution:

1. Let |A|=\(\left|\begin{array}{ccc}3 & -1 & -2 \\ 0 & 0 & -1 \\ 3 & -5 & 0\end{array}\right|\)

By expanding along \(R_2\)

⇒ \(|\mathrm{A}|=-0\left|\begin{array}{cc}
-1 & -2 \\
-5 & 0
\end{array}\right|+0\left|\begin{array}{cc}
3 & -2 \\
3 & 0
\end{array}\right|-(-1)\left|\begin{array}{cc}
3 & -1 \\
3 & -5
\end{array}\right|=\{3 \times(-5)-3 \times(-1)\}\)=-15+3=-12

2. Let |A|=\(\left|\begin{array}{ccc}3 & -4 & 5 \\ 1 & 1 & -2 \\ 2 & 3 & 1\end{array}\right|\)

By expanding along \(\mathrm{R}_1\) (first row), we get

⇒ \(|A| =3\left|\begin{array}{cc}
1 & -2 \\
3 & 1
\end{array}\right|-(-4)\left|\begin{array}{cc}
1 & -2 \\
2 & 1
\end{array}\right|+5\left|\begin{array}{ll}
1 & 1 \\
2 & 3
\end{array}\right|\)

=3(1+6)+4(1+4)+5(3-2)=3(7)+4(5)+5(1)=21+20+5=46

3. Let |A|=\(\left|\begin{array}{ccc}0 & 1 & 2 \\ -1 & 0 & -3 \\ -2 & 3 & 0\end{array}\right|\)

By expanding along \(R_1\) (first row), we get

⇒ \(|A|=0\left|\begin{array}{cc}
0 & -3 \\
3 & 0
\end{array}\right|-1\left|\begin{array}{cc}
-1 & -3 \\
-2 & 0
\end{array}\right|+2\left|\begin{array}{ll}
-1 & 0 \\
-2 & 3
\end{array}\right|\)

=-1(0-6)+2(-3-0)=-1(-6)+2(-3)=6-6=0

4. Let \(|A|=\left|\begin{array}{ccc}2 & -1 & -2 \\ 0 & 2 & -1 \\ 3 & -5 & 0\end{array}\right|\)

By expanding along \(R_2\) (second row), we get

|A|=\(-0\left|\begin{array}{cc}
-1 & -2 \\
-5 & 0
\end{array}\right|+2\left|\begin{array}{cc}
2 & -2 \\
3 & 0
\end{array}\right|-(-1)\left|\begin{array}{cc}
2 & -1 \\
3 & -5
\end{array}\right|\)

=0+2(0+6)+(-10+3)=12-7=5

Question 6. If A=\(\left[\begin{array}{lll}
1 & 1 & -2 \\
2 & 1 & -3 \\
5 & 4 & -9
\end{array}\right]\)
, find|A| ?

Solution:

Given, A=\(\left[\begin{array}{lll}1 & 1 & -2 \\ 2 & 1 & -3 \\ 5 & 4 & -9\end{array}\right]\)

By expanding along R_1 (first row), we get

⇒ \(|A|=1\left|\begin{array}{ll}
1 & -3 \\
4 & -9
\end{array}\right|-1\left|\begin{array}{ll}
2 & -3 \\
5 & -9
\end{array}\right|+(-2)\left|\begin{array}{ll}
2 & 1 \\
5 & 4
\end{array}\right|\)

=1(-9+12)-1(-18+15)-2(8-5)

=1(3)-1(-3)-2(3)=3+3-6=0

Question 7. Find the values of x, if:

1. \(\left|\begin{array}{cc}2 & 4 \\ 5 & 1\end{array}\right|=\left|\begin{array}{cc}2 x & 4 \\ 6 & x\end{array}\right|\)

2. \(\left|\begin{array}{ll}2 & 3 \\ 4 & 5\end{array}\right|=\left|\begin{array}{cc}x & 3 \\ 2 x & 5\end{array}\right|\)

Solution:

1. Given, \(\left|\begin{array}{cc}2 & 4 \\ 5 & 1\end{array}\right|=\left|\begin{array}{cc}2 \mathrm{x} & 4 \\ 6 & \mathrm{x}\end{array}\right|\)

On expanding both determinants, we get

⇒ \((2 \times 1)-(5 \times 4)=(2 x \times x)-(6 \times 4) \Rightarrow 2-20=2 x^2-24\)

⇒ \(2 x^2=-18+24 \Rightarrow x^2=\frac{6}{2}=3 \Rightarrow x= \pm \sqrt{3}\)

2. Given, \(\left|\begin{array}{ll}2 & 3 \\ 4 & 5\end{array}\right|=\left|\begin{array}{cc}x & 3 \\ 2 x & 5\end{array}\right|\)

On expanding both determinants, we get

⇒ \((2 \times 5)-(4 \times 3)=(5 \times x)-(3 \times 2 x)\)

10-12=5 x-6 x \(\Rightarrow-2=-x \Rightarrow\) x=2

Question 8. If \(\left|\begin{array}{cc}
x & 2 \\
18 & x
\end{array}\right|=\left|\begin{array}{cc}
6 & 2 \\
18 & 6
\end{array}\right|\), then x is equal to ?

  1. 6
  2. \(\pm\) 6
  3. -6
  4. 0

Solution: 2. \(\pm\) 6

On expanding both determinants, we get

⇒ \((\mathrm{x} \times \mathrm{x})-(18 \times 2)=(6 \times 6)-(18 \times 2) \Rightarrow \mathrm{x}^2-36=36-36\)

\(\mathrm{x}^2-36=0 \Rightarrow \mathrm{x}^2=36 \Rightarrow \mathrm{x}= \pm 6\)

So, (B) is the correct option.

Determinants Exercise 4.2

Question 1. Find the area of the triangle with vertices at the points in each of the following:

(1,0), (6,0), (4,3)

(2,7). (1,1), (10,8)

(-2,-3), (3,2), (-1,-8)

Solution:

Area of triangle =\(\frac{1}{2}\left|\begin{array}{lll}
x_1 & y_1 & 1 \\
x_2 & y_2 & 1 \\
x_3 & y_3 & 1
\end{array}\right|\)

1. Required area =\(\frac{1}{2}\left|\begin{array}{lll}
1 & 0 & 1 \\
6 & 0 & 1 \\
4 & 3 & 1
\end{array}\right|=\frac{1}{2}|1(0-3)-0(6-4)+1(18-0)|\)

⇒ \(\frac{1}{2}|{-3+18}|=\frac{15}{2}\)..sq. units

2. Required area =\(\frac{1}{2}\left|\begin{array}{lll}
2 & 7 & 1 \\
1 & 1 & 1 \\
10 & 8 & 1
\end{array}\right|\)

= \(\frac{1}{2}|2(1-8)-7(1-10)+1(8-10)|\)

= \(\frac{1}{2}|2(-7)-7(-9)+1(-2)|=\frac{1}{2}|-14+63-2|=\frac{47}{2}\) sq. units

3. Required area =\(\frac{1}{2}\left|\begin{array}{ccc}
-2 & -3 & 1 \\
3 & 2 & 1 \\
-1 & -8 & 1
\end{array}\right|\)

= \(\frac{1}{2}|-2(2+8)+3(3+1)+1(-24+2)|\)

= \(\frac{1}{2}|-20+12-22|=\frac{1}{2}|-30|\)=15 sq. units

(Since the area of the triangle is always positive)

Question 2. Show that the points A(a,b + c), B(b,c +a), C(e, a + b) are collinear

Solution:

Area of \(\triangle A B C =\frac{1}{2}\left|\begin{array}{lll}
a & b+c & 1 \\
b & c+a & 1 \\
c & a+b & 1
\end{array}\right|\)

=\(\frac{1}{2}|a\{(c+a) \times 1-(a+b) \times 1\}-(b+c)\{b \times 1-1 \times c\}+1\{b \times(a+b)-(c+a) \times c\}|\)

=\(\frac{1}{2}\left|a(c+a-a-b)-(b+c)(b-c)+1\left(a b+b^2-c^2-a c\right)\right|\)

=\(\frac{1}{2}\left|a c-a b-b^2+c^2+a b+b^2-c^2-a c\right|=\frac{1}{2} \times \)0=0

Since, the area of AABC = 0.

Hence, points A(a, b + c), … C(c, a + b) are collinear,

Question 3. Find the value of k, the area of a triangle is 4 sq. units and the vertices are:

1. (k,0), (4,0),(0,2)

2. (-2,0),(0,4),(0, k)

Solution:

Area of triangle =\(\frac{1}{2}\left|\begin{array}{lll}x_1 & y_1 & 1 \\ x_2 & y_2 & 1 \\ x_3 & y_3 & 1\end{array}\right|\)

1. Given, \(\frac{1}{2}\left|\begin{array}{lll}
\mathrm{k} & 0 & 1 \\
4 & 0 & 1 \\
0 & 2 & 1
\end{array}\right|=4\)

⇒ \(|\mathrm{k}(0-2)+1(8-0)|=8 \Rightarrow \mathrm{k}(0-2)+1(8-0)= \pm 8\)

On taking positive sign;

-2k + 8 = 8 ⇒ -2k = 0 ⇒ k = 0

On taking negative sign;

-2k + 8 = 8 ⇒-2k = -16⇒ k = 8

k = 0,8

2. Given, \(\frac{1}{2}\left|\begin{array}{ccc}-2 & 0 & 1 \\ 0 & 4 & 1 \\ 0 & \mathrm{k} & 1\end{array}\right|=4 \Rightarrow|-2(4-\mathrm{k})+1(0-0)|\)=8

⇒ –\(2(4-\mathrm{k})+1(0-0)= \pm 8 \Rightarrow(-8+2 \mathrm{k})= \pm\) 8

On taking positive sign, 2 k-8=8 \(\Rightarrow\) 2 \(\mathrm{k}=16 \Rightarrow \)k=8

On taking negative sign, 2 k-8=-8 \(\Rightarrow 2 \mathrm{k}\)=0 \(\Rightarrow\) k=0

k =0,8

Question 4. 1. Find the equation of the line joining (1, 2) and (3, 6) using determinants.

2. Find the equation of the line joining (3, 1) and (9, 3) using determinants.

Solution.

1. Let P(x,y) beany point on the line joining A(1,2)andB(3,6) .

⇒ Area of triangle =\(\frac{1}{2}\left|\begin{array}{lll}x_1 & y_1 & 1 \\ x_2 & y_2 & 1 \\ x_3 & y_3 & 1\end{array}\right|\)=0

⇒ \(\frac{1}{2}\left|\begin{array}{lll}
1 & 2 & 1 \\
3 & 6 & 1 \\
x & y & 1
\end{array}\right|=0 \Rightarrow \frac{1}{2}[1(6-y)-2(3-x)+1(3 y-6 x)]\)=0

6-y-6+2x+3y-6x = 0 ⇒ 2y-4x = 0 ⇒y = 2x

Fluence, the equation of the line joining the given points is y = 2x.

2. Let P(x, y) be any point on the line joining A(3,i)and B(9.3)

Area of triangle =\(\frac{1}{2}\left|\begin{array}{lll}
x_1 & y_1 & 1 \\
x_2 & y_2 & 1 \\
x_3 & y_3 & 1
\end{array}\right|\)=0

⇒ \(\frac{1}{2}\left|\begin{array}{lll}
3 & 1 & 1 \\
9 & 3 & 1 \\
x & y & 1
\end{array}\right|\)=0

⇒ \(\frac{1}{2}|3(3-y)-1(9-x)+1(9 y-3 x)\)|=0

9-3 y-9+x+9 y-3 x=0 \(\Rightarrow 6 y-2 x=0 \Rightarrow\) x-3 y=0

Hence, the equation of the line joining the given points is x-3 y=0.

Question 5. If the area of a triangle is 35 sq. units with vertices (2,-6),(5,4), and (k,4), then k is.

  1. 12
  2. -2
  3. -1 2, -2
  4. 12,-2

Solution: 4. 12,-2

Given, \(\frac{1}{2}\left|\begin{array}{ccc}
2 & -6 & 1 \\
5 & 4 & 1 \\
k & 4 & 1
\end{array}\right|\)=35

|2(4-4)+6(5-k)+1(20-4 k)|=70

2(4-4)+6(5-k)+1(20-4 k)= \(\pm\) 70

30-6 k+20-4 k= \(\pm 70\)

On taking positive sign, -10 k +50=70 \(\Rightarrow\)-10 k =20 \(\Rightarrow\) k=-2

On taking negative sign, -10 k +50=-70 \(\Rightarrow\)-10 \(\mathrm{k}=-120 \Rightarrow\) k =12

k=12,-2 Hence, the correct option is 4.

Determinants Exercise 4.3

Question 1. \(\left|\begin{array}{cc}2 & -4 \\ 0 & 3\end{array}\right|\)

2. \(\left|\begin{array}{ll}a & c \\ b & d\end{array}\right|\)

Solution:

Here, \(\left|\begin{array}{cc}2 & -4 \\ 0 & 3\end{array}\right|\)is given

Minors, \(M_{11}=3, M_{12}=0, M_{21}=-4 and M_{22}\)=2

Also cofactors, \(A_{11}=(-1)^{1+1} \mathrm{M}_{11}=1 \times 3=3\)

⇒ \(A_{12}=(-1)^{1+2} M_{12}=(-1) \times\) 0=0

⇒ \(A_{21}=(-1)^{2+1} M_{21}=(-1) \times(-4)\)=4

and \(\mathrm{A}_{22}=(-1)^{2+2} \mathrm{M}_{22}=1 \times \)2=2

2. Here, \(\left|\begin{array}{ll}\mathrm{a} & \mathrm{c} \\ \mathrm{b} & \mathrm{d}\end{array}\right|\) is given

Minors, \(\mathrm{M}_{11}=\mathrm{d}, \mathrm{M}_{12}=\mathrm{b}, \mathrm{M}_{21}=\mathrm{c}\) and \(\mathrm{M}_{22}=\mathrm{a}\)

Also cofactors, \(A_{11}=(-1)^{1+1} M_{11}=1 \times\) d=d

⇒ \(A_{12}=(-1)^{1+2} M_{12}=(-1) \times \)b=-b

⇒ \(A_{21}=(-1)^{2+1} M_{21}=(-1) \times c\)=-c

and \(A_{22}=(-1)^{2+2} \mathrm{M}_{22}=1 \times \mathrm{a}=\mathrm{a}\)

Question 2. \(\left|\begin{array}{lll}1 & 0 & 0 \\ 0 & 1 & 0 \\ 0 & 0 & 1\end{array}\right|\)

\(\left|\begin{array}{ccc}1 & 0 & 4 \\ 3 & 5 & -1 \\ 0 & 1 & 2\end{array}\right|\)

Solution:

1. Here, \(\left|\begin{array}{lll}
1 & 0 & 0 \\
0 & 1 & 0 \\
0 & 0 & 1
\end{array}\right|\) is given

Minors of elements of the first row are:

⇒ \(M_{11}=\left|\begin{array}{ll}
1 & 0 \\
0 & 1
\end{array}\right|=1-0=1, M_{12}=\left|\begin{array}{ll}
0 & 0 \\
0 & 1
\end{array}\right|\)=0-0=0

and \(M_{13}=\left|\begin{array}{ll}
0 & 1 \\
0 & 0
\end{array}\right|\)=0-0=0

Minors of elements of the second row are:

\(M_{21}=\left|\begin{array}{ll}
0 & 0 \\
0 & 1
\end{array}\right|=0-0=0, M_{22}=\left|\begin{array}{ll}
1 & 0 \\
0 & 1
\end{array}\right|\)=1-0=1

and  \(M_{23}=\left|\begin{array}{ll}
1 & 0 \\
0 & 0
\end{array}\right|\)=0-0=0

Minors of elements of the third row are:

⇒ \(M_{31}=\left|\begin{array}{ll}
0 & 0 \\
1 & 0
\end{array}\right|=0-0=0, M_{32}=\left|\begin{array}{ll}
1 & 0 \\
0 & 0
\end{array}\right|\)=0-0=0

and \(M_{33}=\left|\begin{array}{ll}
1 & 0 \\
0 & 1
\end{array}\right|\)=1-0=1

Hence, the cofactors of elements of the first row are:

⇒ \(A_{11}=(-1)^{1+1} M_{11}=1 \times 1=1, A_{12}\)

=\((-1)^{162} M_{12}=(-1) \times 0=0, A_{13}=(-1)^{1+1} M_{13}=1 \times\) 0=0

The cofactors of elements of the second row are:

⇒ \(A_{21}=(-1)^{2+1} M_{21}=(-1) \times 0=0, A_{22}=(-1)^{2+2} M_{21}=1 \times \)1=1,

⇒ \(A_{23}=(-1)^{2 \times 3} M_3=(-1) \times \)0=0

The cofactors of elements of the third row are:

⇒ \(A_{31}=(-1)^{3+1} M_{34}=1 \times 0=0, A_{32}=(-1)^{3.2} M_{12}=(-1) \times\) 0=0

⇒ \(A_{33}=(-1)^{3,3} M_{33}=1 \times \)1=1

2. Here, \(\left|\begin{array}{ccc}
1 & 0 & 4 \\
3 & 5 & -1 \\
0 & 1 & 2
\end{array}\right|\) is given

Minors of elements of the first row are:

⇒ \(\mathrm{M}_{11}=\left|\begin{array}{cc}
5 & -1 \\
1 & 2
\end{array}\right|=10+1=11, \mathrm{M}_{13}=\left|\begin{array}{cc}
3 & -1 \\
0 & 2
\end{array}\right|=6-0=6\) and

⇒ \(\mathrm{M}_{13}=\left|\begin{array}{ll}
3 & 5 \\
⇒ 0 & 1
\end{array}\right|\)=3-0=3

Minors of elements of the second row are:

⇒ \(M_{31}=\left|\begin{array}{ll}
0 & 4 \\
1 & 2
\end{array}\right|\)=0-4=-4,

⇒ \(M_{23}=\left|\begin{array}{ll}
1 & 4 \\
0 & 2
\end{array}\right|\)=2-0=2 and

\(M_{23}=\left|\begin{array}{ll}
1 & 0 \\
0 & 1
\end{array}\right|\)=1-0=1

Minors of elements of the third row are:

⇒ \(M_{31}=\left|\begin{array}{cc}
0 & 4 \\
5 & -1
\end{array}\right|\)=0-20=-20,

⇒ \(M_{32}=\left|\begin{array}{cc}
1 & 4 \\
3 & -1
\end{array}\right|\)=-1-12=-13 and

⇒ \(M_{33}=\left|\begin{array}{ll}
1 & 0 \\
3 & 5
\end{array}\right|\)=5-0=5

Hence, the cofactors of elements of the first row are:

⇒ \(A_{11}=(-1)^{1+1} M_{14}=1 \times 11=11, A_{12}=(-1)^{1+2} M_{12}=(-1) \times 6=-6, A_{13}\)

=\((-1)^{1+3} M_{13}=1 \times\) 3=3

The cofactors of elements of the second row are:

\(A_{21}=(-1)^{2+1} M_{21}=(-1) \times(-4)=4, A_{22}=(-1)^{2+2} M_{22}=1 \times \)2=2

⇒ \(A_{21}=(-1)^{2+3} M_{21}=(-1) \times\) 1=-1

The cofactors of elements of the third row are:

⇒ \(A_{11}=(-1)^{3+1} M_{31}=1 \times(-20)=-20, A_{12}=(-1)^{3+2} M_{12}=(-1) \times(-13)=13\)

⇒ \(A_{31}=(-1)^{3-3} M_{33}=1 \times\) 5=5

Question 3. Using cofactors of the elements of the second row, evaluate \(\Delta=\left|\begin{array}{lll}
5 & 3 & 8 \\
2 & 0 & 1 \\
1 & 2 & 3
\end{array}\right|\)

Solution:

Given, \(\Delta=\left|\begin{array}{lll}5 & 3 & 8 \\ 2 & 0 & 1 \\ 1 & 2 & 3\end{array}\right|\)

The cofactors of the elements of the second row are:

⇒ \(\mathrm{A}_{21}=(-1)^{2+1}\left|\begin{array}{ll}
3 & 8 \\
2 & 3
\end{array}\right|\)=-(9-16)=7,

⇒ \([\mathrm{A}_4=(-1)^{* / 1} \mathrm{M}_4]\)

⇒ \(\mathrm{A}_{22}=(-1)^{2+2}\left|\begin{array}{ll}
5 & 8 \\
1 & 3
\end{array}\right|\)=(15-8)=7

and \(A_y=(-1)^{2+3}\left|\begin{array}{ll}5 & 3 \\ 1 & 2\end{array}\right|=-(10-3)=-7\)

Now, expansion of \(\Delta\) using cofactors of elements of second row is given by

⇒ \(\Delta=a_{y 1} A_{y t}+a_{y 2} A_{y z}+a_y A_{y y}=(2 \times 7)+(0 \times 7)+(1) \times(-7)=14-7=7\)

Question 4. Using cofactors of elements of the third column, evaluate \(\Delta=\left|\begin{array}{lll}
1 & x & y z \\
1 & y & z x \\
1 & z & x y
\end{array}\right|\)

Solution:

Given, \(\Delta=\left|\begin{array}{lll}1 & x & y z \\ 1 & y & x \\ 1 & z & x y\end{array}\right|\)

The cofactors of the elements of the third column are:

⇒ \(A_{13}=(-1)^{1+3}\left|\begin{array}{ll}
1 & y \\
1 & z
\end{array}\right|=1(z-y)\)=z-y,

⇒ \(A_{23}=(-1)^{2+3}\left|\begin{array}{ll}
1 & x \\
1 & z
\end{array}\right|\)=-1(z-x)=x-z,

⇒ \(A_{x 3}=(-1)^{3+3}\left|\begin{array}{ll}
1 & x \\
1 & y
\end{array}\right|\)=1(y-x)=y-x

Now, the expansion of A using cofactors of elements of the third column is given by

\(\Delta =a_{13} A_{13}+a_{39} A_{23}+a_{33} A_{31}=y z(z-y)+z x(x-z)+x y(y-x)\)

=\(y z^2-y^2 z+z x^2-z^2 x+x y^2-x^2 \)

y=\(x^2(z-y)+x\left(y^2-z^2\right)+y z(z-y)\)

= \((z-y)\left\{x^2-x(y+z)+y z\right\}=(z-y)\left\{x^2-x y-x z+y z\right\}\)

= (z-y)[x(x-y)-z(x-y)]=(y-z)(x-y)(z-x)=(x-y)(y-z)(z-x)

Question 5. If \(\Delta=\left|\begin{array}{lll}
a_{11} & a_{12} & a_{13} \\
a_{21} & a_{22} & a_{23} \\
a_{31} & a_{32} & a_{33}
\end{array}\right|\)
 and \(A_1\) is cofactor of \(a_{i j}\), then value of \(\Delta\) is given by ?

  1. \(a_{11} A_{31}+a_{12} A_{32}+a_{19} A_{30}\)
  2. \(a_{11} A_{11}+a_{12} A_{21}+a_{10} A_n\)
  3. \(a_{31} A_{11}+a_{23} A_{12}+a_{2 y} A_{13}\)
  4. \(a_{11} \mathbf{A}_{11}+a_{2 /} A_{24}+a_n A_3\)

Solution: 4. \(a_{11} \mathbf{A}_{11}+a_{2 /} A_{24}+a_n A_3\)

⇒ \(\Delta\) is equal to the sum of the products of the elements of a row (or a column) with their corresponding cofactors.

⇒ \(\Delta=a_{11} A_{11}+a_{12} A_{12}+a_{13} A_{13} \text { or } a_{21} A_{21}+a_{22} A_{22}+a_{23} A_{23}\)

or \(a_{12} A_{12}+a_{22} A_{22}+a_{12} A_{32}\) or \(a_{13} A_{13}+a_{23} A_{23}+a_{33} A_{33}\)

Hence, the sum of the products of the elements of the first column with their corresponding cofactors is \(\Delta=a_{11} A_{11}+a_{21} A_{21}+a_{31} A_{31}\)

Hence, the correct option is 4.

Determinants Exercise 4.4

Question 1. \(\left[\begin{array}{ll}1 & 2 \\ 3 & 4\end{array}\right]\)

Solution :

Let A=\(\left[\begin{array}{ll}
1 & 2 \\
3 & 4
\end{array}\right]\)

⇒  \(A_{11}=4, A_{12}=-3, A_{21}\)=-2 and \(A_{22}\)=1

⇒ \({adj} A=\left[A_4\right]=\left[\begin{array}{ll}
A_{11} & A_{22} \\
A_{21} & A_{22}
\end{array}\right]^{\prime}\)

= \(\left[\begin{array}{cc}
4 & -3 \\
-2 & 1
\end{array}\right]=\left[\begin{array}{cc}
4 & -2 \\
-3 & 1
\end{array}\right]\)

Question 2. \(\left[\begin{array}{ccc}
1 & -1 & 2 \\
2 & 3 & 5 \\
-2 & 0 & 1
\end{array}\right]\)

Solution:

Let A=\(\left[\begin{array}{ccc}
1 & -1 & 2 \\
2 & 3 & 5 \\
-2 & 0 & 1
\end{array}\right]\)

The cofactors of elements of the first row are:

\(A_{11}=\left|\begin{array}{ll}
3 & 5 \\
0 & 1
\end{array}\right|\)=3-0=3,

⇒  \(A_{12}=-\left|\begin{array}{cc}
2 & 5 \\
-2 & 1
\end{array}\right|=-(2+10)=-12 \text { and } A_{13}=\left|\begin{array}{cc}
2 & 3 \\
-2 & 0
\end{array}\right|\)=0-(-6)=6

The cofactors of elements of the second row are:

⇒ \(A_{21}=-\left|\begin{array}{cc}
-1 & 2 \\
0 & 1
\end{array}\right|=-(-1-0)=1, A_2=\left|\begin{array}{cc}
1 & 2 \\
-2 & 1
\end{array}\right|\)

= \((1+4)=5 and A_{23}=-\left|\begin{array}{cc}
1 & -1 \\
-2 & 0
\end{array}\right|\)=-(0-2)=2

The cofactors of elements of the third row are:

⇒  \(A_{11}=\left|\begin{array}{cc}
-1 & 2 \\
3 & 5
\end{array}\right|=(-5-6)=-11,\)

⇒ \(A_{93}=-\left|\begin{array}{ll}
1 & 2 \\
2 & 5
\end{array}\right|\)=-(5-4)=-1

and \(A_{33}=\left|\begin{array}{cc}
1 & -1 \\
2 & 3
\end{array}\right|\)=3+2=5

Hence,\({adj}(A)=\left[A_0\right]^{\prime}=\left[\begin{array}{ccc}3 & -12 & 6 \\ 1 & 5 & 2 \\ -11 & -1 & 5\end{array}\right]^{\prime}=\left[\begin{array}{ccc}3 & 1 & -11 \\ -12 & 5 & -1 \\ 6 & 2 & 5\end{array}\right]\)

Question 3. \(\left[\begin{array}{cc}
2 & 3 \\
-4 & -6
\end{array}\right]\)

Solution:

Let A=\(\left[\begin{array}{cc}2 & 3 \\ -4 & -6\end{array}\right],|A|=\left|\begin{array}{cc}2 & 3 \\ -4 & -6\end{array}\right|=-12-(-12)\)=-12+12=0

⇒ \(|\mathrm{A}| \mathrm{I}=0\left[\begin{array}{ll}
1 & 0 \\
0 & 1
\end{array}\right]=\left[\begin{array}{ll}
0 & 0 \\
0 & 0
\end{array}\right]=\mathrm{O}\)

Cofactors of A are \(A_{11}=-6, A_{12}=4, A_{21}=-3, A_2=2\)

⇒ \({adj}(A)=\left[A_{i 1}\right]=\left[\begin{array}{ll}
-6 & 4 \\
-3 & 2
\end{array}\right]=\left[\begin{array}{cc}
-6 & -3 \\
4 & 2
\end{array}\right]\)

Now, \((adj A) \mathrm{A}=\left[\begin{array}{cc}-6 & -3 \\ 4 & 2\end{array}\right]\left[\begin{array}{cc}2 & 3 \\ -4 & -6\end{array}\right]=\left[\begin{array}{cc}-12+12 & -18+18 \\ 8-8 & 12-12\end{array}\right]=\left[\begin{array}{ll}0 & 0 \\ 0 & 0\end{array}\right]=\mathrm{0}\)

Also, \(A(adj A)=\left[\begin{array}{cc}2 & 3 \\ -4 & -6\end{array}\right]\left[\begin{array}{cc}-6 & -3 \\ 4 & 2\end{array}\right]=\left[\begin{array}{cc}-12+12 & -6+6 \\ 24-24 & 12-12\end{array}\right]=\left[\begin{array}{ll}0 & 0 \\ 0 & 0\end{array}\right]\)=0

Hence, \(\mathrm{A}({adj} \mathrm{A})=({adj} \mathrm{A}) \mathrm{A}=|\mathrm{A}| \mathrm{I}_2\).

Question 4. \(\left[\begin{array}{ccc}
1 & -1 & 2 \\
3 & 0 & -2 \\
1 & 0 & 3
\end{array}\right]\)

Solution:

Let A=\(\left[\begin{array}{ccc}1 & -1 & 2 \\ 3 & 0 & -2 \\ 1 & 0 & 3\end{array}\right]\)

Now, \(|A|=\left|\begin{array}{ccc}1 & -1 & 2 \\ 3 & 0 & -2 \\ 1 & 0 & 3\end{array}\right|\)=1(0-0)-(-1)(9+2)+2(0-0)=0+11+0=11

⇒ \(|A| I=11\left[\begin{array}{lll}
1 & 0 & 0 \\
0 & 1 & 0 \\
0 & 0 & 1
\end{array}\right]\)

= \(\left[\begin{array}{ccc}
11 & 0 & 0 \\
0 & 11 & 0 \\
0 & 0 & 11
\end{array}\right]\)

Cofactors of A are:

⇒ \(A_{11}=0, A_{12}=-(9+2)=-11, A_{13}=0, A_{21}=-(-3-0)=3, A_{22}=3-2=1, A_{33}\)

=-(0+1)=-1,

⇒ \(A_{31}=2-0=2, A_{32}=-(-2-6)=8, A_{39}=0+3=3 \)

⇒ \({adj}(A)=\left[A_7\right]^{\prime}=\left[\begin{array}{ccc}
0 & -11 & 0 \\
3 & 1 & -1 \\
2 & 8 & 3
\end{array}\right]=\left[\begin{array}{ccc}
0 & 3 & 2 \\
-11 & 1 & 8 \\
0 & -1 & 3
\end{array}\right]\)

Now, \(A({adj} A)=\left[\begin{array}{ccc}
1 & -1 & 2 \\
3 & 0 & -2 \\
1 & 0 & 3
\end{array}\right]\left[\begin{array}{ccc}
0 & 3 & 2 \\
-11 & 1 & 8 \\
0 & -1 & 3
\end{array}\right]\)

= \(\left[\begin{array}{lll}
0+11+0 & 3-1-2 & 2-8+6 \\
0+0+0 & 9+0+2 & 6+0-6 \\
0+0+0 & 3+0-3 & 2+0+9
\end{array}\right]\)

= \(\left[\begin{array}{ccc}
11 & 0 & 0 \\
0 & 11 & 0 \\
0 & 0 & 11
\end{array}\right]\)

Also, \(({adj} A) A=\left[\begin{array}{ccc}0 & 3 & 2 \\ -11 & 1 & 8 \\ 0 & -1 & 3\end{array}\right]\left[\begin{array}{ccc}1 & -1 & 2 \\ 3 & 0 & -2 \\ 1 & 0 & 3\end{array}\right]\)

= \(\left[\begin{array}{ccc}
0+9+2 & 0+0+0 & 0-6+6 \\
-11+3+8 & 11+0+0 & -22-2+24 \\
0-3+3 & 0+0+0 & 0+2+9
\end{array}\right]\)

= \(\left[\begin{array}{ccc}
11 & 0 & 0 \\
0 & 11 & 0 \\
0 & 0 & 11
\end{array}\right]\)

=\(11\left[\begin{array}{lll}
1 & 0 & 0 \\
0 & 1 & 0 \\
0 & 0 & 1
\end{array}\right]\)

Hence, \(\mathrm{A}({adj} \mathrm{A})=({adj} \mathrm{A}) \mathrm{A}=|\mathrm{A}| \mathrm{I}\)

Question 5. \(\left[\begin{array}{cc}
2 & -2 \\
4 & 3
\end{array}\right]\)

Solution:

Let A=\(\left[\begin{array}{cc}2 & -2 \\ 4 & 3\end{array}\right].\)

We have, \(|\mathrm{A}|=\left|\begin{array}{cc}2 & -2 \\ 4 & 3\end{array}\right|=6-(-8)=14 \neq 0 \mathrm{A}^{-1}\) exists

Cofactors of \(\mathrm{A}_{11} are \mathrm{A}_{11}\)=3,

⇒ \(\mathrm{~A}_{12}=-4, \mathrm{~A}_{21}=2, \mathrm{~A}_{22}=2\)

⇒ \({adj}(\mathrm{A})=\left[\mathrm{A}_{i j}\right]^{\prime}=\left[\begin{array}{cc}
3 & -4 \\
2 & 2
\end{array}\right]^{\prime}=\left[\begin{array}{cc}
3 & 2 \\
-4 & 2
\end{array}\right]\)

Now, \(A^{-1}=\frac{1}{|A|}({adj} A)=\frac{1}{14}\left[\begin{array}{cc}
3 & 2 \\
-4 & 2
\end{array}\right]\)

= \(\left[\begin{array}{cc}
\frac{3}{14} & \frac{2}{14} \\
-\frac{4}{14} & \frac{2}{14}
\end{array}\right]\)

= \(\left[\begin{array}{cc}
\frac{3}{14} & \frac{1}{7} \\
-\frac{2}{7} & \frac{1}{7}
\end{array}\right]\)

Question 6. \(\left[\begin{array}{ll}
-1 & 5 \\
-3 & 2
\end{array}\right]\)

Solution:

Let A=\(\left[\begin{array}{ll}-1 & 5 \\ -3 & 2\end{array}\right]\).

We have, \(|A|==-2-(-15)=13 \neq\) 0

\(\mathrm{A}^{-1}\) exists

Now, cofactors of A are \(A_{11}=2, A_{12}=3, A_{21}=-5, A_{21}\)=-1

⇒ \({adj}(A)=\left[A_0\right]^{\prime}=\left[\begin{array}{cc}
2 & 3 \\
-5 & -1
\end{array}\right]^{\prime}=\left[\begin{array}{ll}
2 & -5 \\
3 & -1
\end{array}\right]\)

Now, \(A^{-1}=\frac{1}{|A|}({adj} A)=\frac{1}{13}\left[\begin{array}{cc}2 & -5 \\ 3 & -1\end{array}\right]\)

= \(\left[\begin{array}{cc}\frac{2}{13} & -\frac{5}{13} \\ \frac{3}{13} & -\frac{1}{13}\end{array}\right]\)

Question 7. \(\left[\begin{array}{lll}
1 & 2 & 3 \\
0 & 2 & 4 \\
0 & 0 & 5
\end{array}\right]\)

Solution:

Let A=\(\left[\begin{array}{lll}1 & 2 & 3 \\ 0 & 2 & 4 \\ 0 & 0 & 5\end{array}\right]\)

We have, |\(\mathrm{A}|=1(10-0)-2(0-0)+3(0-0)=10 \neq 0\)

⇒ \(\mathrm{A}^{-1}\) exists

Now, cofactors of A are

⇒ \(A_{11}\)=10-0=10, \(A_{12}\)=-(0-0)=0, \(A_{13}\)=0-0=0,

⇒ \(A_{21}\)=-(10-0)=-10, \(A_{22}\)=5-0=5, \(A_{23}\)=-(0-0)=0 ,

⇒ \(\mathrm{A}_{31}=8-6=2, \mathrm{~A}_{32}=-(4-0)=-4, \quad \mathrm{~A}_{35}=2-0=2 \)

⇒ \({adj}(\mathrm{A})=\left[\mathrm{A}_{i j}\right]^{\prime}=\left[\begin{array}{ccc}
10 & 0 & 0 \\
-10 & 5 & 0 \\
2 & -4 & 2
\end{array}\right]^{\prime}\)

= \(\left[\begin{array}{ccc}
10 & -10 & 2 \\
0 & 5 & -4 \\
0 & 0 & 2
\end{array}\right] \)

Now, \(A^{-1}=\frac{1}{|A|}({adj} A)=\frac{1}{10}\left[\begin{array}{ccc}
10 & -10 & 2 \\
0 & 5 & -4 \\
0 & 0 & 2
\end{array}\right]\)

= \(\left[\begin{array}{ccc}
1 & -1 & \frac{1}{5} \\
0 & \frac{1}{2} & -\frac{2}{5} \\
0 & 0 & \frac{1}{5}
\end{array}\right]\).

Question 8. \(\left[\begin{array}{ccc}
1 & 0 & 0 \\
3 & 3 & 0 \\
5 & 2 & -1
\end{array}\right]\)

Solution:

Let A=\(\left[\begin{array}{ccc}1 & 0 & 0 \\ 3 & 3 & 0 \\ 5 & 2 & -1\end{array}\right]\)

⇒ \(|A|=1(-3-0)-0+0=-3 \neq\) 0

⇒ \(\mathrm{A}^{-1}\) exists

Cofactors of A are:

⇒ \(A_{11}=-3-0=-3, A_{12}=-(-3-0)=3, A_{13}=6-15=-9\),

⇒ \(A_{21}=-(0-0)=0, A_{22}=-1-0=-1, A_{23}=-(2-0)=-2\),

⇒ \(A_{31}=0-0=0, A_{12}=-(0-0)=0, A_{33}\)=3-0=3

⇒ \({adj}(A)=\left[A_{i 1}\right]^{\prime}=\left[\begin{array}{ccc}
-3 & 3 & -9 \\
0 & -1 & -2 \\
0 & 0 & 3
\end{array}\right]^{\prime}\)

= \(\left[\begin{array}{ccc}
-3 & 0 & 0 \\
3 & -1 & 0 \\
-9 & -2 & 3
\end{array}\right]\)

Now, \(A^{-1}=\frac{1}{|A|}({adj} A)=\frac{1}{-3}\left[\begin{array}{ccc}
-3 & 0 & 0 \\
3 & -1 & 0 \\
-9 & -2 & 3
\end{array}\right]\)

= \(\left[\begin{array}{ccc}
1 & 0 & 0 \\
-1 & \frac{1}{3} & 0 \\
3 & \frac{2}{3} & -1
\end{array}\right]\)

Question 9. \([\left[\begin{array}{ccc}
2 & 1 & 3 \\
4 & -1 & 0 \\
-7 & 2 & 1
\end{array}\right]\)

Solution:

Let A=\(\left[\begin{array}{ccc}2 & 1 & 3 \\ 4 & -1 & 0 \\ -7 & 2 & 1\end{array}\right]\)

|A|=2(-1-0)-1(4-0)+3(8-7)=-2-4+3=-3 \(\neq\) 0

⇒ \(\mathrm{A}^{-1}\) exists

Cofactors of A are:

⇒ \(A_{11}=-1-0=-1, A_{12}=-(4-0)=-4, A_{13}=8-7=1\)

⇒ \(A_{21}=-(1-6)=5, A_{22}=2+21=23, \quad A_{33}\)=-(4+7)=-11,

⇒ \(A_{31}=0+3=3, A_{32}=-(0-12), A_{33}\)=-2-4=-6

⇒ \({adj}(A)=\left[A_{61}\right]^{\prime}=\left[\begin{array}{ccc}
-1 & -4 & 1 \\
5 & 23 & -11 \\
3 & 12 & -6
\end{array}\right]\)

=\(\left[\begin{array}{ccc}
-1 & 5 & 3 \\
-4 & 23 & 12 \\
1 & -11 & -6
\end{array}\right]\)

Now, \(A^{-1}=\frac{1}{|A|}({adj}A)\)

= \(\frac{1}{-3}\left[\begin{array}{ccc}
-1 & 5 & 3 \\
-4 & 23 & 12 \\
1 & -11 & -6
\end{array}\right]\)

= \(\left[\begin{array}{ccc}
\frac{1}{3} & -\frac{5}{3} & -1 \\
\frac{4}{3} & -\frac{23}{3} & -4 \\
-\frac{1}{3} & \frac{11}{3} & 2
\end{array}\right]\)

Question 10. \(\left[\begin{array}{ccc}
1 & -1 & 2 \\
0 & 2 & -3 \\
3 & -2 & 4
\end{array}\right]\)

Solution:

Let A=\(\left[\begin{array}{ccc}1 & -1 & 2 \\ 0 & 2 & -3 \\ 3 & -2 & 4\end{array}\right]\)

We have, \(|A|=\left|\begin{array}{ccc}1 & -1 & 2 \\ 0 & 2 & -3 \\ 3 & -2 & 4\end{array}\right|\)

=1(8-6)-(-1)(0+9)+2(0-6)=2+9-12=-1 \(\neq\) 0

⇒ \(\mathrm{A}^{-1}\) exists

Cofactors of A are:

⇒ \(A_{11}=8-6=2, A_{12}=-(0+9)=-9, A_{12}=0-6=-6\),

⇒ \(A_{21}=-(-4+4)=0, A_{22}=4-6=-2, A_{23}=-(-2+3)=-1\)

⇒ \(A_{31}=3-4=-1, A_{22}=-(-3-0)=3, A_{33}=2-0=2\)

⇒ \({adj}(A)=\left[A_4\right]^{+}=\left[\begin{array}{ccc}
2 & -9 & -6 \\
0 & -2 & -1 \\
-1 & 3 & 2
\end{array}\right]^{\prime}=\left[\begin{array}{ccc}
2 & 0 & -1 \\
-9 & -2 & 3 \\
-6 & -1 & 2
\end{array}\right]\)

Now; \(A^{-1}=\frac{1}{|A|}({adj} A)=\frac{1}{-1}\left[\begin{array}{ccc}
2 & 0 & -1 \\
-9 & -2 & 3 \\
-6 & -1 & 2
\end{array}\right]\)

= \(\left[\begin{array}{ccc}
-2 & 0 & 1 \\
9 & 2 & -3 \\
6 & 1 & -2
\end{array}\right]\)

Question 11. \(\left[\begin{array}{ccc}
1 & 0 & 0 \\
0 & \cos \alpha & \sin \alpha \\
0 & \sin \alpha & -\cos \alpha
\end{array}\right]\)

Solution:

Let A=\(\left[\begin{array}{ccc}1 & 0 & 0 \\ 0 & \cos \alpha & \sin \alpha \\ 0 & \sin \alpha & -\cos \alpha\end{array}\right] \Rightarrow|A|\)

= \(\left|\begin{array}{ccc}1 & 0 & 0 \\ 0 & \cos \alpha & \sin \alpha \\ 0 & \sin \alpha & -\cos \alpha\end{array}\right|=1\left(-\cos ^2 \alpha-\sin ^2 \alpha\right)\)

=-\(\left(\cos ^2 \alpha+\sin ^2 \alpha\right)=-1 \neq 0 \left[ \cos ^2 \theta+\sin ^2 \theta=1\right]\)

⇒ \(\mathrm{A}^{-1}\) exists

Cofactors of A are:

⇒ \(\mathrm{A}_{11}=-\cos ^2 \alpha-\sin ^2 \alpha\)=-1,

⇒ \(\mathrm{~A}_{12}=-(0-0)=0, \mathrm{~A}_{13}\)=0-0=0

⇒ \(\mathrm{~A}_{21}=-(0-0)=0, \mathrm{~A}_{22}=-\cos \alpha-0=-\cos \alpha, \mathrm{A}_{23}=-(\sin \alpha-0)=-\sin \alpha,\)

⇒ \(\mathrm{A}_{21}=0-0=0, \mathrm{~A}_{32}=-(\sin \alpha-0)=-\sin \alpha, \mathrm{A}_{35}=\cos \alpha-0=\cos \alpha \)

⇒ \({adj}(\mathrm{A})=\left[\mathrm{A}_{1 j}\right]^{\prime}=\left[\begin{array}{ccc}
-1 & 0 & 0 \\
0 & -\cos \alpha & -\sin \alpha \\
0 & -\sin \alpha & \cos \alpha
\end{array}\right]^{\prime}\)

= \(\left[\begin{array}{ccc}
-1 & 0 & 0 \\
0 & -\cos \alpha & -\sin \alpha \\
0 & -\sin \alpha & \cos \alpha
\end{array}\right]\)

Now, \(\mathrm{A}^{-1}=\frac{1}{|\mathrm{~A}|}({adj} \mathrm{A})=\frac{1}{-1}\left[\begin{array}{ccc}
-1 & 0 & 0 \\
0 & -\cos \alpha & -\sin \alpha \\
0 & -\sin \alpha & \cos \alpha
\end{array}\right]\)

= \(\left[\begin{array}{ccc}
1 & 0 & 0 \\
0 & \cos \alpha & \sin \alpha \\
0 & \sin \alpha & -\cos \alpha
\end{array}\right]\)

Question 12. Let A=\(\left[\begin{array}{ll}
3 & 7 \\
2 & 5
\end{array}\right]\) and\( \mathrm{B}=\left[\begin{array}{ll}
6 & 8 \\
7 & 9
\end{array}\right]\). Verify that \((A B)^{-1}=B^{-1} A^{-1}\).

Solution:

Given, A=\(\left[\begin{array}{ll}3 & 7 \\ 2 & 5\end{array}\right] ;|A|=\left|\begin{array}{ll}3 & 7 \\ 2 & 5\end{array}\right|=15-14=1 \neq 0\)

⇒ \(\mathrm{A}^{-1}\) exists

Cofactors of A are \(\mathrm{A}_{11}\)=5,

⇒ \(\mathrm{~A}_{12}=-2, \mathrm{~A}_{21}=-7, \mathrm{~A}_{27}\)=3

⇒ \({adj}(A)=\left[A_{i j}\right]^{\prime}=\left[\begin{array}{cc}5 & -2 \\ -7 & 3\end{array}\right]^{\prime}=\left[\begin{array}{cc}5 & -7 \\ -2 & 3\end{array}\right]\)

Now, \(A^{-1}=\frac{1}{|A|}({adj} A)=\frac{1}{1}\left[\begin{array}{cc}5 & -7 \\ -2 & 3\end{array}\right]=\left[\begin{array}{cc}5 & -7 \\ -2 & 3\end{array}\right] \)

Here, B=\(\left[\begin{array}{ll}6 & 8 \\ 7 & 9\end{array}\right] |B|=\left|\begin{array}{ll}6 & 8 \\ 7 & 9\end{array}\right|=54-56=-2 \neq \)

⇒ \(\mathrm{B}^{-1}\) exists

Cofactors of B are \(\mathrm{B}_{11}=9, \mathrm{~B}_{22}=-7, \mathrm{~B}_{21}=-8, \mathrm{~B}_{22}\)=6

⇒ \({adj}(B)=\left[B_{i j}\right]^{\prime}=\left[\begin{array}{cc}
9 & -7 \\
-8 & 6
\end{array}\right]^{\prime}=\left[\begin{array}{cc}
9 & -8 \\
-7 & 6
\end{array}\right]\)

⇒ \(B^{-1}=\frac{1}{|B|}(\text { adj } B)=\frac{1}{-2}\left[\begin{array}{cc}
9 & -8 \\
-7 & 6
\end{array}\right]\)

Now,\(B^{-1} A^{-1} =\frac{1}{-2}\left[\begin{array}{cc}
9 & -8 \\
-7 & 6
\end{array}\right]\left[\begin{array}{cc}
5 & -7 \\
-2 & 3
\end{array}\right]\)

=\(\frac{1}{-2}\left[\begin{array}{cc}
45+16 & -63-24 \\
-35-12 & 49+18
\end{array}\right] \)

=\(\frac{1}{-2}\left[\begin{array}{cc}
61 & -87 \\
-47 & 67
\end{array}\right]\)

=\(\left[\begin{array}{cc}
-\frac{61}{2} & \frac{87}{2} \\
\frac{47}{2} & -\frac{67}{2}
\end{array}\right]\) Equation 1

Now, let \(\mathrm{C}=\mathrm{AB}=\left[\begin{array}{ll}3 & 7 \\ 2 & 5\end{array}\right]\left[\begin{array}{ll}6 & 8 \\ 7 & 9\end{array}\right]\)

=\(\left[\begin{array}{ll}18+49 & 24+63 \\ 12+35 & 16+45\end{array}\right]\)

=\(\left[\begin{array}{ll}67 & 87 \\ 47 & 61\end{array}\right]\)

⇒ \(|\mathrm{AB}|=\left|\begin{array}{ll}
67 & 87 \\
47 & 61
\end{array}\right|\)

=\((67 \times 61)-(47 \times 87)=4087-4089=-2 \neq 0\)

∴ \(\mathrm{C}^{-1} exists\),

Cofactors of C are \(C_{11}=61, C_{12}=-47, C_{21}=-87, C_{22}=67\)

⇒ \({adj}(\mathrm{AB})=\left[\mathrm{C}_{\mathrm{ii}}\right]^{+}=\left[\begin{array}{cc}
61 & -47 \\
-87 & 67
\end{array}\right]^{\prime}=\left[\begin{array}{cc}
61 & -87 \\
-47 & 67
\end{array}\right]\)

⇒ \((\mathrm{AB})^{-1}=\frac{1}{|\mathrm{AB}|}(\mathrm{adj} \mathrm{AB})=\frac{1}{-2}\left[\begin{array}{cc}
61 & -87 \\
-47 & 67
\end{array}\right]\)

= \(\left[\begin{array}{cc}
-\frac{61}{2} & \frac{87}{2} \\
\frac{47}{2} & -\frac{67}{2}
\end{array}\right]\)  → Equation 2

From Eq. (1) and (2), we get \((A B)^{-1}=B^{-1} A^{-1}\)

Hence, the given result is proved.

Question 13. If A=\(\left[\begin{array}{cc}
3 & 1 \\
-1 & 2
\end{array}\right]\), show that \(A^2-5 A+7 I=0\). Hence, find \(A^{-1}\).

Solution:

Given, A=\(\left[\begin{array}{cc}3 & 1 \\ -1 & 2\end{array}\right]\)

⇒ \(A^2=A \cdot A=\left[\begin{array}{cc}
3 & 1 \\
-1 & 2
\end{array}\right]\left[\begin{array}{cc}
3 & 1 \\
-1 & 2
\end{array}\right]\)

= \(\left[\begin{array}{cc}
9-1 & 3+2 \\
-3-2 & -1+4
\end{array}\right]=\left[\begin{array}{cc}
8 & 5 \\
-5 & 3
\end{array}\right]\)

Now,\( A^2-5 A+7 I\)

= \(\left[\begin{array}{cc}
8 & 5 \\
-5 & 3
\end{array}\right]-5\left[\begin{array}{cc}
3 & 1 \\
-1 & 2
\end{array}\right]+7\left[\begin{array}{ll}
1 & 0 \\
0 & 1
\end{array}\right]\)

= \(\left[\begin{array}{cc}
8 & 5 \\
-5 & 3
\end{array}\right]-\left[\begin{array}{cc}
15 & 5 \\
-5 & 10
\end{array}\right]+\left[\begin{array}{ll}
7 & 0 \\
0 & 7
\end{array}\right]\)

= \(\left[\begin{array}{cc}
8-15+7 & 5-5+0 \\
-5+5+0 & 3-10+7
\end{array}\right]\)

=\(\left[\begin{array}{ll}
0 & 0 \\
0 & 0
\end{array}\right]\)=0

\(A^2-5 A+7 I\)=0

⇒ \(|A|=\left|\begin{array}{cc}
3 & 1 \\
-1 & 2
\end{array}\right|=6+1=7 \neq \)0, \(A^{-1}\) exists.

Now, A \(\cdot\) A-5 A=-71

Post multiplying by \(\mathrm{A}^{-1}\) on both sides, we get

A \(\cdot A\left(A^{-1}\right)-5 A^{-1}=-7 A^{-1} \Rightarrow A I-5 I=-7 A^{-1}\)

Using \(A^{-1}=I\) and \(I^{-1}=A^{-1}\)

⇒ \(A^{-1}=-\frac{1}{7}(A-5 I) \Rightarrow A^{-1}=\frac{1}{7}(5 I-A)=\frac{1}{7}\left(\left[\begin{array}{ll}
5 & 0 \\
0 & 5
\end{array}\right]-\left[\begin{array}{cc}
3 & -1 \\
-1 & 2
\end{array}\right]\right)\)

=\(\frac{1}{7}\left[\begin{array}{cc}
2 & -1 \\
1 & 3
\end{array}\right] \)

⇒  \(A^{-1}=\frac{1}{7}\left[\begin{array}{cc}
2 & -1 \\
1 & 3
\end{array}\right]\)

Question 14. For the matrix A=\(\left[\begin{array}{ll}
3 & 2 \\
1 & 1
\end{array}\right]\), find the numbers ‘ a ‘ and ‘ b ‘ such that \(A^2+a A+b I\)=0.

Solution:

Given, A=\(\left[\begin{array}{ll}3 & 2 \\ 1 & 1\end{array}\right]\)

⇒ \(A^2=A \cdot A=\left[\begin{array}{ll}
3 & 2 \\
1 & 1
\end{array}\right]\left[\begin{array}{ll}
3 & 2 \\
1 & 1
\end{array}\right]\)

=\(\left[\begin{array}{ll}
9+2 & 6+2 \\
3+1 & 2+1
\end{array}\right]=\left[\begin{array}{cc}
11 & 8 \\
4 & 3
\end{array}\right]\)

Given, \(\mathrm{A}^2+\mathrm{aA}+\mathrm{bI}\)=0

On putting the values of \(\mathrm{A}^2, \mathrm{~A}\) and 1 ; we get

⇒ \(\left[\begin{array}{ll}
11 & 8 \\
4 & 3
\end{array}\right]+\mathrm{a}\left[\begin{array}{ll}
3 & 2 \\
1 & 1
\end{array}\right]+\mathrm{b}\left[\begin{array}{ll}
1 & 0 \\
0 & 1
\end{array}\right]\)=0

⇒ \(\left[\begin{array}{cc}
11+3 a+b & 8+2 a+0 \\
4+a+0 & 3+a+b
\end{array}\right]=0 \)

⇒ \({\left[\begin{array}{cc}
11+3 a+b & 8+2 a \\
4+a & 3+a+b
\end{array}\right]=\left[\begin{array}{ll}
0 & 0 \\
0 & 0
\end{array}\right] }\)

If two matrices are equal, then their corresponding elements are also equal.

11+3a+b=0  → Equation 1

8+2 a=0  → Equation 2

4+a=0 → Equation 3

and 3+a+b=0  → Equation 4

Solving Eq. (3) and (4), we get 4+\(\mathrm{a}=0 \Rightarrow \mathrm{a}\)=-4

And 3+a+b=0 \(\Rightarrow 3-4+b=0 \Rightarrow\) b=1

Thus, a=-4 and b=1

Question 15. For the matrix A=\(\left[\begin{array}{ccc}
1 & 1 & 1 \\
1 & 2 & -3 \\
2 & -1 & 3
\end{array}\right]\), show that \(A^3-6 A^2+5 A+111\)=0. Hence, find \(A^{-1}\),

Solution:

Given, A=\(\left[\begin{array}{ccc}1 & 1 & 1 \\ 1 & 2 & -3 \\ 2 & -1 & 3\end{array}\right] ;|A|=1(6-3)-1(3+6)+1(-1-4)-3-9-5=-11 \neq 0\)

⇒ \(\mathrm{A}^{-1}\) exists

Now,\(\mathbf{A}^2=\mathbf{A} \cdot \mathrm{A}  =\left[\begin{array}{ccc}
1 & 1 & 1 \\
1 & 2 & -3 \\
2 & -1 & 3
\end{array}\right]\left[\begin{array}{ccc}
1 & 1 & 1 \\
1 & 2 & -3 \\
2 & -1 & 3
\end{array}\right]\)

=\(\left[\begin{array}{ccc}
1+1+2 & 1+2-1 & 1-3+3 \\
1+2-6 & 1+4+3 & 1-6-9 \\
2-1+6 & 2-2-3 & 2+3+9
\end{array}\right]=\left[\begin{array}{ccc}
4 & 2 & -1 \\
-3 & 8 & -14 \\
7 & -3 & 14
\end{array}\right]\)

And \(A^3=A^2 \cdot A =\left[\begin{array}{ccc}
4 & 2 & 1 \\
-3 & 8 & -14 \\
7 & -3 & 14
\end{array}\right]\left[\begin{array}{ccc}
1 & 1 & 1 \\
1 & 2 & -3 \\
2 & -1 & 3
\end{array}\right]\)

=\(\left[\begin{array}{ccc}
4+2+2 & 4+4-1 & 4-6+3 \\
-3+8-28 & -3+16+14 & -3-24-42 \\
7-3+28 & 7-6-14 & 7+9+42
\end{array}\right]\)

=\(\left[\begin{array}{ccc}
8 & 7 & 1 \\
-23 & 27 & -69 \\
32 & -13 & 58
\end{array}\right]\)

⇒ \(A^3-6 A^2+  \text { A + } 111 \\
=\left[\begin{array}{ccc}
8 & 7 & 1 \\
-23 & 27 & -69 \\
32 & -13 & 58
\end{array}\right]-6\left[\begin{array}{ccc}
4 & 2 & 1 \\
-3 & 8 & -14 \\
7 & -3 & 14
\end{array}\right]+5\left[\begin{array}{ccc}
1 & 1 & 1 \\
1 & 2 & -3 \\
2 & -1 & 3
\end{array}\right]+11\left[\begin{array}{ccc}
1 & 0 & 0 \\
0 & 1 & 0 \\
0 & 0 & 1
\end{array}\right]\)

=\(\left[\begin{array}{ccc}
8 & 7 & 1 \\
-23 & 27 & -69 \\
32 & -13 & 58
\end{array}\right]-\left[\begin{array}{ccc}
24 & 12 & 6 \\
-18 & 48 & -84 \\
42 & -18 & 84
\end{array}\right]+\left[\begin{array}{ccc}
5 & 5 & 5 \\
5 & 10 & -15 \\
10 & -5 & 15
\end{array}\right]+\left[\begin{array}{ccc}
11 & 0 & 0 \\
0 & 11 & 0 \\
0 & 0 & 11
\end{array}\right]\)

=\(\left[\begin{array}{ccc}
8-24+5+11 & 7-12+5+0 & 1-6+5+0 \\
-23+18+5+0 & 27-48+10+11 & -69+84-15+0 \\
32-42+10+0 & -13+18-5+0 & 58-84+15+11
\end{array}\right]=\left[\begin{array}{lll}
0 & 0 & 0 \\
0 & 0 & 0 \\
0 & 0 & 0
\end{array}\right]\)=0

Now, \(A^3-6 A^2+5 A+111\)=0

⇒ \((A A A) A^{-1}-6(A A) A^{-1}+5 A A^{-1}+111 A^{-1}\)=0 (Post-multiplying by }\( A^{-t} as |A| \neq 0)\)

⇒ \(A A\left(A A^{-1}\right)-6 A\left(A^{-1}\right)+5\left(A^{-1}\right)+11\left(\mathrm{IA}^{-1}\right)\)=0

⇒ \(\mathrm{AAI}-6 \mathrm{AI}+5 \mathrm{I}+11 \mathrm{~A}^{-1}=0 (Using \mathrm{AA}^{-1}=\mathrm{I} and\mathrm{IA}^{-1}=\mathrm{A}^{-1}\)

⇒ \(\mathrm{A}^2-6 \mathrm{~A}+5 \mathrm{I}=-11 \mathrm{~A}^{-1} \)

⇒ \((Using \mathrm{AAI}=\mathrm{A}^2 and \mathrm{AI}=\mathrm{A}\)

⇒ \(A^{-1}=-\frac{1}{11}\left(A^2-6 A+51\right) \Rightarrow A^{-1}=\frac{1}{11}\left(-A^2+6 A-5 I\right)\)

=\(\frac{1}{11}\left\{-\left[\begin{array}{ccc}
4 & 2 & 1 \\
-3 & 8 & -14 \\
7 & -3 & 14
\end{array}\right]+6\left[\begin{array}{ccc}
1 & 1 & 1 \\
1 & 2 & -3 \\
2 & -1 & 3
\end{array}\right]-5\left[\begin{array}{lll}
1 & 0 & 0 \\
0 & 1 & 0 \\
0 & 0 & 1
\end{array}\right]\right\} \)

= \(\frac{1}{11}\left\{\left[\begin{array}{ccc}
-4 & -2 & -1 \\
3 & -8 & 14 \\
-7 & 3 & -14
\end{array}\right]+\left[\begin{array}{ccc}
6 & 6 & 6 \\
6 & 12 & -18 \\
12 & -6 & 18
\end{array}\right]-\left[\begin{array}{ccc}
5 & 0 & 0 \\
0 & 5 & 0 \\
0 & 0 & 5
\end{array}\right]\right\}\)

=\(\frac{1}{11}\left[\begin{array}{ccc}
-4+6-5 & -2+6-0 & -1+6-0 \\
3+6-0 & -8+12-5 & 14-18-0 \\
-7+12-0 & 3-6-0 & -14+18-5
\end{array}\right]\)

=\(\frac{1}{11}\left[\begin{array}{ccc}
-3 & 4 & 5 \\
9 & -1 & -4 \\
5 & -3 & -1
\end{array}\right]\)

Question 16. If A=\(\left[\begin{array}{ccc}
2 & -1 & 1 \\
-1 & 2 & -1 \\
1 & -1 & 2
\end{array}\right]\)
 , verify that \(A^3-6 A^2+9 A-4\) I=O and hence, find \(A^{-1}\)

Solution:

Given, A=\(\left[\begin{array}{ccc}
2 & -1 & 1 \\
-1 & 2 & -1 \\
1 & -1 & 2
\end{array}\right] \)

\(A^2=A \cdot A=\left[\begin{array}{ccc}
2 & -1 & 1 \\
-1 & 2 & -1 \\
1 & -1 & 2
\end{array}\right]\left[\begin{array}{ccc}
2 & -1 & 1 \\
-1 & 2 & -1 \\
1 & -1 & 2
\end{array}\right]\)

= \(\left[\begin{array}{ccc}
4+1+1 & -2-2-1 & 2+1+2 \\
-2-2-1 & 1+4+1 & -1-2-2 \\
2+1+2 & -1-2-2 & 1+1+4
\end{array}\right]\)

= \(\left[\begin{array}{ccc}
6 & -5 & 5 \\
-5 & 6 & -5 \\
5 & -5 & 6
\end{array}\right]\)

and \(A^3=A^2 \cdot A =\left[\begin{array}{ccc}
6 & -5 & 5 \\
-5 & 6 & -5 \\
5 & -5 & 6
\end{array}\right]\left[\begin{array}{ccc}
2 & -1 & 1 \\
-1 & 2 & -1 \\
1 & -1 & 2
\end{array}\right]\)

= \(\left[\begin{array}{ccc}
12+5+5 & -6-10-5 & 6+5+10 \\
-10-6-5 & 5+12+5 & -5-6-10 \\
10+5+6 & -5-10-6 & 5+5+12
\end{array}\right]\)

=\(\left[\begin{array}{ccc}
22 & -21 & 21 \\
-21 & 22 & -21 \\
21 & -21 & 22
\end{array}\right]\)

⇒ \(A^3-6 A^2 +9 A-41\)

= \(\left[\begin{array}{ccc}
22 & -21 & 21 \\
-21 & 22 & -21 \\
21 & -21 & 22
\end{array}\right]-6\left[\begin{array}{ccc}
6 & -5 & 5 \\
-5 & 6 & -5 \\
5 & -5 & 6
\end{array}\right]+9\left[\begin{array}{ccc}
2 & -1 & 1 \\
-1 & 2 & -1 \\
1 & -1 & 2
\end{array}\right]-4\left[\begin{array}{ccc}
1 & 0 & 0 \\
0 & 1 & 0 \\
0 & 0 & 1
\end{array}\right]\)

= \(\left[\begin{array}{ccc}
22 & -21 & 21 \\
-21 & 22 & -21 \\
21 & -21 & 22
\end{array}\right]-\left[\begin{array}{ccc}
36 & -30 & 30 \\
-30 & 36 & -30 \\
30 & -30 & 36
\end{array}\right]+\left[\begin{array}{ccc}
18 & -9 & 9 \\
-9 & 18 & -9 \\
9 & -9 & 18
\end{array}\right]-\left[\begin{array}{ccc}
4 & 0 & 0 \\
0 & 4 & 0 \\
0 & 0 & 4
\end{array}\right]\)

=\(\left[\begin{array}{ccc}
22-36+18-4 & -21+30-9-0 & 21-30+9-0 \\
-21+30-9-0 & 22-36+18-4 & -21+30-9-0 \\
21-30+9-0 & -21+30-9-0 & 22-36+18-4
\end{array}\right]\)

= \(\left[\begin{array}{lll}
0 & 0 & 0 \\
0 & 0 & 0 \\
0 & 0 & 0
\end{array}\right]\)=0

⇒ \(A^3-6 A^2+9 A-4 I=0 \Rightarrow(A A A) A^{-1}-6(A A) A^{-1}+9 A^{-1}-4 I A^{-1}\)=0

(Post-multiplying by } \(\mathrm{A}^{-1} \text { as }|\mathrm{A}| \neq 0 \)

⇒ \([|A|=|\begin{array}{ccc}
2 & -1 & 1 \\
-1 & 2 & -1 \\
1 & -1 & 2
\end{array}|\)=2(4-1)+1(-2+1)+1(1-2)=6-1-1=4 \(\neq 0\)

⇒ \(\mathrm{AA}\left(\mathrm{AA}^{-1}\right)-6 \mathrm{~A}\left(\mathrm{AA}^{-1}\right)+9\left(\mathrm{AA}^{-1}\right)-4\left(\mathrm{IA}^{-1}\right)\)=0

⇒ \(\mathrm{AAI}-6 \mathrm{AI}+9 \mathrm{I}-4 \mathrm{~A}^{-1}=0\)

(Using }\(\mathrm{AA}^{-1}= I\) and  \(1 \mathrm{~A}^{-1}=\mathrm{A}^{-1}\)

⇒ \(A^2-6 A+9 I=4 A^{-1}\)

(Using \(A^2 T=A^2\) and A\(\mathrm{~A}^{-1}\)=A )

⇒ \(A^{-1}=\frac{1}{4}\left(A^2-6 A+91\right)\)

⇒ \(\frac{1}{4}\left\{\left[\begin{array}{ccc}
6 & -5 & 5 \\
-5 & 6 & -5 \\
5 & -5 & 6
\end{array}\right]-6\left[\begin{array}{ccc}
2 & -1 & 1 \\
-1 & 2 & -1 \\
1 & -1 & 2
\end{array}\right]+9\left[\begin{array}{ccc}
1 & 0 & 0 \\
0 & 1 & 0 \\
0 & 0 & 1
\end{array}\right]\right\}\)

⇒ \(\frac{1}{4}\left\{\left[\begin{array}{ccc}
6 & -5 & 5 \\
-5 & 6 & -5 \\
5 & -5 & 6
\end{array}\right]-\left[\begin{array}{ccc}
12 & -6 & 6 \\
-6 & 12 & -6 \\
6 & -6 & 12
\end{array}\right]+\left[\begin{array}{ccc}
9 & 0 & 0 \\
0 & 9 & 0 \\
0 & 0 & 9
\end{array}\right]\right\}\)

=\(\frac{1}{4}\left[\begin{array}{ccc}
6-12+9 & -5+6+0 & 5-6+0 \\
-5+6+0 & 6-12+9 & -5+6+0 \\
5-6+0 & -5+6+0 & 6-12+9
\end{array}\right]\)

=\(\frac{1}{4}\left[\begin{array}{ccc}
3 & 1 & -1 \\
1 & 3 & 1 \\
-1 & 1 & 3
\end{array}\right]\)

Question 17. Let A be anon-singular square matrix of order 3×3, then |adj A| equal to?

  1. \(|\mathrm{A}|\)
  2. \(|\mathrm{A}|^2\)
  3. \(|\mathrm{A}|\)
  4. 3\(|\mathrm{~A}|\)

Solution: 2. \(|\mathrm{A}|^2\)

We know that \(({adj} \mathrm{A}) \mathrm{A}=|\mathrm{A}| \mathrm{I}\)

= \(|A|\left[\begin{array}{ccc}
1 & 0 & 0 \\
0 & 1 & 0 \\
0 & 0 & 1
\end{array}\right]=\left[\begin{array}{ccc}
|A| & 0 & 0 \\
0 & |A| & 0 \\
0 & 0 & |A|
\end{array}\right]\)

⇒ \(|({adj} A) A|=\left|\begin{array}{ccc}
A & 0 & 0 \\
0 & |A| & 0 \\
0 & 0 & |A|
\end{array}\right|\)

=\(|A|^3\left|\begin{array}{lll}
1 & 0 & 0 \\
0 & 1 & 0 \\
0 & 0 & 1
\end{array}\right|=|A|^3|\mathrm{I}|\)

⇒ \(|{adj} A| A|=| A|^3 (|I|=1)\)

⇒ \(|{adj} A|=|A|^2\)

Hence, the correct option is 2

Question 18. If A is an invertible matrix of order 2, then det \(\left(\mathrm{A}^{-1}\right)\) is equal to?

  1. \({det}(\mathrm{A})\)
  2. \(\frac{1}{det}(\mathrm{A}\)
  3. 1
  4. zero

Solution:

We know that \(\mathrm{AA}^{-1}=\mathrm{I}\)

⇒ \(|\mathrm{AA}^{-1}|=|\mathrm{I}| \Rightarrow|\mathrm{A}|\mathrm{A}^{-1}|\)=1

⇒ {Using\(|\mathrm{AA}^{-1}|=|\mathrm{A}||\mathrm{A}^{-1}|\) and \(|\mathrm{I}|=1)\)

⇒ \(|\mathrm{A}^{-1}|=\frac{1}{|\mathrm{~A}|}=\frac{1}{{det}(\mathrm{A})}\) .

Hence, the correct option is (B).

Determinants Exercise 4.5

Question 1. x+2 y=2,2 x+3 y=3.

Solution:

The given system can be written as AX= B, where

⇒ \(\mathrm{A}=\left[\begin{array}{ll}
1 & 2 \\
2 & 3
\end{array}\right], \mathrm{X}=\left[\begin{array}{l}
\mathrm{x} \\
\mathrm{y}
\end{array}\right] \)

and \(\mathrm{B}=\left[\begin{array}{l}
2 \\
3
\end{array}\right]\)

Here, \(|\mathrm{A}|=\left|\begin{array}{ll}1 & 2 \\ 2 & 3\end{array}\right|=1(3)-2(2)=3-4=-1 \neq\) 0

A is non-singular. Therefore, \(\mathrm{A}^{-1}\) exists.

Hence, the given system of equations is consistent.

Question 2. 2 x-y=5, x+y=4

Solution:

The given system can be written as A X=B, where

A=\(\left[\begin{array}{cc}
2 & -1 \\
1 & 1
\end{array}\right], X=\left[\begin{array}{l}
x \\
y
\end{array}\right] \text { and } B=\left[\begin{array}{l}
5 \\
4
\end{array}\right]\)

Here, \(|A|=\left|\begin{array}{cc}2 & -1 \\ 1 & 1\end{array}\right|=2(1)-(-1)(1)=2+1=3 \neq \)

A is non-singular. Therefore, \(\mathrm{A}^{-1}\) exists.

Hence, the given system of equations is consistent.

Question 3. x+3 y=5,2 x+6 y=8

Solution:

The given system can be written as AX = B, where

A=\(\left[\begin{array}{ll}
1 & 3 \\
2 & 6
\end{array}\right], X=\left[\begin{array}{l}
x \\
y
\end{array}\right] \text { and } B=\left[\begin{array}{l}
5 \\
8
\end{array}\right]\)

Here, \(|A|=\left|\begin{array}{ll}1 & 3 \\ 2 & 6\end{array}\right|=1(6)-3(2)=6-6=0\)

A is a singular matrix.

Nothing can be said about consistency as yet. We compute

⇒ \(({adj} \mathrm{A}) \mathrm{B} =\left[\begin{array}{cc}
6 & -2 \\
-3 & 1
\end{array}\right]\left[\begin{array}{l}
5 \\
8
\end{array}\right]\)

=\(\left[\begin{array}{cc}
6 & -3 \\
-2 & 1
\end{array}\right]\left[\begin{array}{l}
5 \\
8
\end{array}\right]=\left[\begin{array}{c}
30-24 \\
-10+8
\end{array}\right]\)

= \(\left[\begin{array}{c}
6 \\
-2
\end{array}\right] \neq 0\)

Thus, the solution of the given system of equations does not exist. Hence, the system of equations is inconsistent.

Question 4. x+y+z=1,2 x+3 y+2 z=2, a x+a y+2 a z=4

Solution:

The given system can be written as AX – B, where

A=\(\left[\begin{array}{ccc}
1 & 1 & 1 \\
2 & 3 & 2 \\
a & a & 2 a
\end{array}\right], X=\left[\begin{array}{l}
x \\
y \\
z
\end{array}\right]\) and

B=\(\left[\begin{array}{l}
1 \\
2 \\
4
\end{array}\right]\)

Here,|A| =\(\left|\begin{array}{lll}
1 & 1 & 1 \\
2 & 3 & 2 \\
a & a & 2 a
\end{array}\right|=1(6 a-2 a)-1(4 a-2 a)+1(2 a-3 a)\)

=4 a-2 a-a=4 a-3 a=a \(\neq 0\)

A is non-singular. Therefore, \(\mathrm{A}^{-1}\) exists.

Hence, the given system of equations is consistent.

Question 5. 3 x-y-2 z=2,2 y-z=-1,3 x-5 y=3

Solution:

The given system is 3 x-y-2 z=2,0 x+2 y-z=-1 and 3 x-5 y+0 z=3 which can be written as AX =B, where

A=\(\left[\begin{array}{ccc}
3 & -1 & -2 \\
0 & 2 & -1 \\
3 & -5 & 0
\end{array}\right], X=\left[\begin{array}{l}
x \\
y \\
z
\end{array}\right]\) and

B=\(\left[\begin{array}{c}
2 \\
-1 \\
3
\end{array}\right]\)

Here, \(|A|=\left|\begin{array}{ccc}3 & -1 & -2 \\ 0 & 2 & -1 \\ 3 & -5 & 0\end{array}\right|=3(0-5)+1(0+3)-2(0-6)\)=-15+3+12=0

A is a singular matrix.

Therefore, nothing can be said about consistency as yet. So, we compute (adj A)B.

Cofactors of A are:

⇒ \(A_{11}=-5, A_{12}=-3, A_{13}\)=-6,

⇒ \(A_{21}=10, A_{22}=6, A_{23}=12\),

⇒ \(A_{91}=5, A_{32}=3, A_{33}\)=6

⇒ \({adj}(A)=\left[A_4\right]^{\prime}=\left[\begin{array}{ccc}
-5 & -3 & -6 \\
10 & 6 & 12 \\
5 & 3 & 6
\end{array}\right]=\left[\begin{array}{ccc}
-5 & 10 & 5 \\
-3 & 6 & 3 \\
-6 & 12 & 6
\end{array}\right]\)

⇒ \(({adj} A) B=\left[\begin{array}{ccc}
-5 & 10 & 5 \\
-3 & 6 & 3 \\
-6 & 12 & 6
\end{array}\right] \cdot\left[\begin{array}{c}
2 \\
-1 \\
3
\end{array}\right]\)

= \(\left[\begin{array}{c}
-10-10+15 \\
-6-6+9 \\
-12-12+18
\end{array}\right]=\left[\begin{array}{c}
-5 \\
-3 \\
-6
\end{array}\right] \neq 0\)

Thus, the solution of the given system of equations does not exist.

Hence, the system of equations is inconsistent.

Question 6. 5 x-y+4 z=5,2 x+3 y+5 z=2,5 x-2 y+6 z=-1

Solution:

The given system can be written as A X=B, where

⇒ \(\mathbf{A}=\left[\begin{array}{ccc}
5 & -1 & 4 \\
2 & 3 & 5 \\
5 & -2 & 6
\end{array}\right]\),

X=\(\left[\begin{array}{l}
x \\
y \\
z
\end{array}\right]\)

and \(B=\left[\begin{array}{c}
5 \\
2 \\
-1
\end{array}\right]\)

Here, \(|A|=\left|\begin{array}{ccc}5 & -1 & 4 \\ 2 & 3 & 5 \\ 5 & -2 & 6\end{array}\right|\)

=5(18+10)-(-1)(12-25)+4(-4-15)=140-13-76=51 \(\neq 0\)

A is non-singular.

Therefore, \(\mathrm{A}^{-1}\) exists.

Hence, the given system of equations is consistent.

Question 7. 5 x+2 y=4,7 x+3 y=5

Solution:

The given system can be written as AX = B, where

⇒ \(\mathrm{A}=\left[\begin{array}{ll}5 & 2 \\ 7 & 3\end{array}\right], X=\left[\begin{array}{l}\mathrm{x} \\ \mathrm{y}\end{array}\right] \)

and \(\mathrm{B}=\left[\begin{array}{l}4 \\ 5\end{array}\right]\)

Here, \(|\mathrm{A}|=\left|\begin{array}{ll}5 & 2 \\ 7 & 3\end{array}\right|=15-14=1 \neq 0\)

Thus, A is non-singular.

Therefore, its \(\mathrm{A}^{-1}\) exists.

Therefore, the given system is consistent and has a unique solution given by

⇒ \(\mathrm{A}^{-1}(\mathrm{AX})=\mathrm{A}^{-1} \mathrm{~B} \Rightarrow \mathrm{X}=\mathrm{A}^{-1} \mathrm{~B}\)

Cofactors of A are, \(A_{11}=3, A_{12}=-7, A_{21}=-2, A_{22}=5\)

⇒ \({adj}(A)=\left[A_{i j}\right]^{\prime}=\left[\begin{array}{cc}
3 & -7 \\
-2 & 3
\end{array}\right]^{\prime}=\left[\begin{array}{cc}
3 & -2 \\
-7 & 5
\end{array}\right]\)

Now, \(A^{-1}=\frac{1}{|A|}({adj} A)=\frac{1}{1}\left[\begin{array}{cc}3 & -2 \\ -7 & 5\end{array}\right]=\left[\begin{array}{cc}3 & -2 \\ -7 & 5\end{array}\right]\)

⇒ \(\mathrm{X}=\mathrm{A}^{-1} \mathrm{~B}=\left[\begin{array}{cc}
3 & -2 \\
-7 & 5
\end{array}\right]\left[\begin{array}{l}
4 \\
5
\end{array}\right]=\left[\begin{array}{l}
3 \times 4+(-2) \times 5 \\
(-7) \times 4+5 \times 5
\end{array}\right]\)

= \(\left[\begin{array}{c}
2 \\
-3
\end{array}\right] \Rightarrow\left[\begin{array}{l}
x \\
y
\end{array}\right]=\left[\begin{array}{c}
2 \\
-3
\end{array}\right]\)

Hence, x=2 and y=-3.

Question 8. 2 x-y=-2,3 x+4 y=3

Solution:

The given system can be written as AX = B, where

A=\(\left[\begin{array}{cc}
2 & -1 \\
3 & 4
\end{array}\right], X=\left[\begin{array}{l}
x \\
y
\end{array}\right]\)

and \(B=\left[\begin{array}{c}
-2 \\
3
\end{array}\right]\)

Here, |A|=\(\left|\begin{array}{cc}2 & -1 \\ 3 & 4\end{array}\right|=2 \times 4-(-3)=11 \neq 0\)

Thus, A is non-singular.

Therefore, its \(\mathrm{A}^{-1}\) exists.

Therefore, the given system is consistent and has a unique solution given by

⇒ \(\mathrm{X}=\mathrm{A}^{-1} \mathrm{~B}\)

Cofactors of A are, \(A_{11}=4, A_{12}=-3, A_{21}=1, A_{21}=2\)

⇒ \({adj}(A)=\left[A_i\right]^{\prime}=\left[\begin{array}{cc}
4 & -3 \\
1 & 2
\end{array}\right]^{\prime}-\left[\begin{array}{cc}
4 & 1 \\
-3 & 2
\end{array}\right]\)

⇒ \(A^{-1}=\frac{1}{|A|}({adj} A)=\frac{1}{11}\left[\begin{array}{cc}
4 & 1 \\
-3 & 2
\end{array}\right]\)

Now, \(X=A^{-1} B=\frac{1}{11}\left[\begin{array}{cc}4 & 1 \\ -3 & 2\end{array}\right]\left[\begin{array}{c}-2 \\ 3\end{array}\right]=\frac{1}{11}\left[\begin{array}{c}-8+3 \\ 6+6\end{array}\right]\)

= \(\frac{1}{11}\left[\begin{array}{c}-5 \\ 12\end{array}\right]=\left[\begin{array}{c}-\frac{5}{11} \\ \frac{12}{11}\end{array}\right]\)

⇒ \(\left[\begin{array}{l}x \\ y\end{array}\right]=\left[\begin{array}{c}\frac{-5}{11} \\ \frac{12}{11}\end{array}\right]\)

Hence, x=\(\frac{-5}{11}\) and y=\(\frac{12}{11}\).

Question 9. 4 x-3 y=3,3 x-5 y=7

Solution:

The given system can be written as AX = B, where

A=\(\left[\begin{array}{ll}
4 & -3 \\
3 & -5
\end{array}\right]\),

X=\(\left[\begin{array}{l}
x \\
y
\end{array}\right] \)and B=\(\left[\begin{array}{l}
3 \\
7
\end{array}\right]\)

Here, \(|A|=\left|\begin{array}{ll}4 & -3 \\ 3 & -5\end{array}\right|=4(-5)-3(-3)=-20+9=-11 \neq 0\)

Thus, A is non-singular.

Therefore, its \(\mathrm{A}^{-1}\) exists,

Therefore, the given system is consistent and has a unique solution given by

⇒ \(\mathrm{X}=\mathrm{A}^{-1} \mathrm{~B}\)

Cofactors of A are, \(A_{11}=-5, A_{12}=-3, A_{21}=3, A_{22}=4\)

⇒ \({adj}(A)=\left[A_{i j}\right]^{\prime}=\left[\begin{array}{cc}
-5 & -3 \\
3 & 4
\end{array}\right]^{\prime}=\left[\begin{array}{ll}
-5 & 3 \\
-3 & 4
\end{array}\right]\)

⇒ \(A^{-1}=\frac{1}{|A|}({adj} A)=\frac{1}{-11}\left[\begin{array}{ll}
-5 & 3 \\
-3 & 4
\end{array}\right] \)

Now, \(X=A^{-1} B=\frac{1}{-11}\left[\begin{array}{ll}
-5 & 3 \\
-3 & 4
\end{array}\right]\left[\begin{array}{l}
3 \\
7
\end{array}\right]\)

=\(\frac{1}{-11}\left[\begin{array}{c}
-15+21 \\
-9+28
\end{array}\right]=\frac{1}{-11}\left[\begin{array}{c}
6 \\
19
\end{array}\right] \)

⇒ \(\left[\begin{array}{l}
x \\
y
\end{array}\right]=\left[\begin{array}{c}
\frac{-6}{11} \\
\frac{-19}{11}
\end{array}\right]\)

Hence, x=-\(\frac{6}{11}\) and y=-\(\frac{19}{11}\)

Question 10. 5 x+2 y=3,3 x+2 y=5

Solution:

The given system can be written as AX = B, where

⇒ \(\mathrm{A}=\left[\begin{array}{ll}5 & 2 \\ 3 & 2\end{array}\right], \mathrm{X}=\left[\begin{array}{l}\mathrm{x} \\ \mathrm{y}\end{array}\right] and \mathrm{B}=\left[\begin{array}{l}3 \\ 5\end{array}\right]\)

Here, |A|=\(\left|\begin{array}{ll}5 & 2 \\ 3 & 2\end{array}\right|=10-6=4 \neq 0\)

Thus, A is non-singular.

Therefore, its \(\mathrm{A}^{-1}\) exists.

Therefore, the given system is consistent and has a unique solution given by \(X=A^{-1} B\)

Cofactors of A are, \(A_{11}=2, \mathrm{~A}_{12}=-3, \mathrm{~A}_{21}=-2, \mathrm{~A}_{22}\)=5

adj \((A)=\left[A_4\right]^{\prime}=\left[\begin{array}{cc}2 & -3 \\ -2 & 5\end{array}\right]^*=\left[\begin{array}{cc}2 & -2 \\ -3 & 5\end{array}\right]\)

⇒ \(\mathrm{A}^{-1}=\frac{1}{\mid \mathrm{A}}({adj} \mathrm{A})=\frac{1}{4}\left[\begin{array}{cc}2 & -2 \\ -3 & 5\end{array}\right]\)

Now, X=\(\mathrm{A}^{-1}
\mathrm{~B}=\frac{1}{4}\left[\begin{array}{cc}2 & -2 \\ -3 & 5\end{array}\right]\left[\begin{array}{l}3 \\ 5\end{array}\right]\)

=\(\frac{1}{4}\left[\begin{array}{c}6-10 \\ -9+25\end{array}\right]=\frac{1}{4}\left[\begin{array}{c}-4 \\ 16\end{array}\right]\)

=\(\left[\begin{array}{c}-1 \\ 4\end{array}\right] \Rightarrow\left[\begin{array}{l}x \\ y\end{array}\right]=\left[\begin{array}{c}-1 \\ 4\end{array}\right]\)

Hence, x=-1 and y=4

Question 11. 2 x+y+z=1, x-2 y-z=\(\frac{3}{2}\), 3 y-5 z=9

Solution:

The given system can be written as AX=B where

A=\(\left[\begin{array}{ccc}
2 & 1 & 1 \\
2 & -4 & -2 \\
0 & 3 & -5
\end{array}\right], X=\left[\begin{array}{l}
x \\
y \\
z
\end{array}\right]\)

and B=\(\left[\begin{array}{l}
1 \\
3 \\
9
\end{array}\right]\)

Here, \(|A|=\left|\begin{array}{ccc}2 & 1 & 1 \\ 2 & -4 & -2 \\ 0 & 3 & -5\end{array}\right|=2(20+6)-1(-10-0)+1(6-0)=52+10+6=68 \neq 0\)

Thus, A is non-singular.

Therefore, its \(\mathrm{A}^{-1}\) exists.

Therefore, the given system is consistent and has a unique solution given by X=\(\mathrm{A}^{-1} \mathrm{~B}\)

Cofactors of A are,

⇒ \(A_{11}=20+6=26, A_{12}=-(-10-0)=10, A_{13}\)=6-0=6

⇒ \(A_{21}=-(-5-3)=8, A_{22}=-10-0=-10, A_{23}=-(6-0)\)=-6

⇒ \(A_{31}=(-2+4)=2, A_{32}=-(-4-2)=6, A_{33}\)=-8-2=-10

⇒ \({adj}(A)=\left[A_1\right]^{\prime}=\left[\begin{array}{ccc}
26 & 10 & 6 \\
8 & -10 & -6 \\
2 & 6 & -10
\end{array}\right]=\left[\begin{array}{ccc}
26 & 8 & 2 \\
10 & -10 & 6 \\
6 & -6 & -10
\end{array}\right]\)

⇒ \(A^{-1}=\frac{1}{|A|}({adj} A)=\frac{1}{68}\left[\begin{array}{ccc}
26 & 8 & 2 \\
10 & -10 & 6 \\
6 & -6 & -10
\end{array}\right]\)

Now, \(X=A^{-1} B \Rightarrow\left[\begin{array}{l}x \\ y \\ z\end{array}\right]=\frac{1}{68}\left[\begin{array}{ccc}26 & 8 & 2 \\ 10 & -10 & 6 \\ 6 & -6 & -10\end{array}\right]\left[\begin{array}{l}1 \\ 3 \\ 9\end{array}\right]\)

⇒ \(\left[\begin{array}{l}
x \\
y \\
z
\end{array}\right]\)

=\(\frac{1}{68}\left[\begin{array}{c}
26+24+18 \\
10-30+54 \\
6-18-90
\end{array}\right]=\frac{1}{68}\left[\begin{array}{c}
68 \\
34 \\
-102
\end{array}\right]\)

⇒ \(\Rightarrow\left[\begin{array}{l}
x \\
y \\
z
\end{array}\right]=\left[\begin{array}{c}
1 \\
\frac{1}{2} \\
\frac{-3}{2}
\end{array}\right]\)

Hence; x=1, y=\(\frac{1}{2}\) and z=\(\frac{-3}{2}\)

Question 12. x-y+z=4, 2 x+y-3 z=0, x+y+z=2

Solution:

The given system can be written as AX=B, where

A=\(\left[\begin{array}{ccc}
1 & -1 & 1 \\
2 & 1 & -3 \\
1 & 1 & 1
\end{array}\right], X=\left[\begin{array}{l}
x \\
y \\
z
\end{array}\right]\)

and B=\(\left[\begin{array}{l}
4 \\
0 \\
2
\end{array}\right]\)

Here, \(|\mathrm{A}|=\left|\begin{array}{ccc}1 & -1 & 1 \\ 2 & 1 & -3 \\ 1 & 1 & 1\end{array}\right|=1(1+3)-(-1)(2+3)+1(2-1)=4+5+1=10 \neq 0\)

Thus, A is non-singular.

Therefore, its \(\mathrm{A}^{-1}\) exists.

Therefore, the given system is consistent and has a unique solution given by \(\mathrm{X}=\mathrm{A}^{-1} \mathrm{~B}\)

Cofactors of A are.

⇒ \(A_{11}=1+3=4, A_{13}=-(2+3)=-5, A_{13}=2-1=1 \)

⇒ \(A_{21}=-(-1-1)=2, A_{22}=1-1=0, A_{25}=-(1+1)=-2\)

⇒ \(A_{71}=3-1=2, A_{22}=-(-3-2)=5, A_{23}=1+2=3\)

⇒ \({adj}(A)=\left[A_{i j}\right]^{\prime}=\left[\begin{array}{ccc}
4 & -5 & 1 \\
2 & 0 & -2 \\
2 & 5 & 3
\end{array}\right]=\left[\begin{array}{ccc}
4 & 2 & 2 \\
-5 & 0 & 5 \\
1 & -2 & 3
\end{array}\right]\)

⇒ \(A^{-1}=\frac{1}{|A|}(\text { adj } A)=\frac{1}{10}\left[\begin{array}{ccc}
4 & 2 & 2 \\
-5 & 0 & 5 \\
1 & -2 & 3
\end{array}\right]\)

Now, \(X=A^{-1} B \Rightarrow\left[\begin{array}{l}x \\ y \\ z\end{array}\right]=\frac{1}{10}\left[\begin{array}{ccc}4 & 2 & 2 \\ -5 & 0 & 5 \\ 1 & -2 & 3\end{array}\right]\left[\begin{array}{l}4 \\ 0 \\ 2\end{array}\right]\)

⇒ \(\left[\begin{array}{l}
x \\
y \\
z
\end{array}\right]=\frac{1}{10}\left[\begin{array}{c}
16+0+4 \\
-20+0+10 \\
4+0+6
\end{array}\right] \Rightarrow\left[\begin{array}{l}
x \\
y \\
z
\end{array}\right]\)

=\(\frac{1}{10}\left[\begin{array}{c}
20 \\
-10 \\
10
\end{array}\right]=\left[\begin{array}{c}
2 \\
-1 \\
1
\end{array}\right]\)

Hence; x=2, y=-1 and z=1

Question 13. 2 x+3 y+3 z=5, x-2 y+z=-4, 3 x-y-2 z=3

Solution:

The given system can be written as AX = B, where

A=\(\left[\begin{array}{ccc}
2 & 3 & 3 \\
1 & -2 & 1 \\
3 & -1 & -2
\end{array}\right], X=\left[\begin{array}{l}
x \\
y \\
z
\end{array}\right]\)

and B=\(\left[\begin{array}{c}
5 \\
-4 \\
3
\end{array}\right]\)

Here, \(|\mathrm{A}|=\left|\begin{array}{ccc}2 & 3 & 3 \\ 1 & -2 & 1 \\ 3 & -1 & -2\end{array}\right|=2(4+1)-3(-2-3)+3(-1+6)=10+15+15=40 \neq 0\)

Thus, A is non-singular.

Therefore, its \(\mathrm{A}^{-4}\) exists.

Therefore, the given system is consistent and has a unique solution given by X =\(\mathrm{A}^{-1}\) B Cofictors of A are,

⇒ \(A_{11}=4+1=5, A_{12}=-(-2-3)=5, A_{13}=(-1+6)=5\)

⇒ \(A_{21}=-(-6+3)=3, A_{27}=(-4-9)=-13, A_{33}=-(-2-9)=11\)

⇒ \(A_{31}=3+6=9, A_{21}=-(2-3)=1, A_{32}=-4-3=-7\)

⇒ \({adj}(\mathrm{A})=\left[\mathrm{A}_1\right]^{\prime}=\left[\begin{array}{ccc}
5 & 5 & 5 \\
3 & -13 & 11 \\
9 & 1 & -7
\end{array}\right]^{\prime}=\left[\begin{array}{ccc}
5 & 3 & 9 \\
5 & -13 & 1 \\
5 & 11 & -7
\end{array}\right]\)

⇒ \(\mathrm{A}^{-1}=\frac{1}{|\mathrm{~A}|}({adj} \mathrm{A})=\frac{1}{40}\left[\begin{array}{ccc}
5 & 3 & 9 \\
5 & -13 & 1 \\
5 & 11 & -7
\end{array}\right]\)

⇒ \(\mathrm{X}=\mathrm{A}^{-1} \mathrm{~B} \Rightarrow\left[\begin{array}{l}
\mathrm{x} \\
\mathrm{y} \\
\mathrm{z}
\end{array}\right]=\frac{1}{40}\left[\begin{array}{ccc}
5 & 3 & 9 \\
5 & -13 & 1 \\
5 & 11 & -7
\end{array}\right] \cdot\left[\begin{array}{c}
5 \\
-4 \\
3
\end{array}\right]\)

= \(\frac{1}{40}\left[\begin{array}{c}
25-12+27 \\
25+52+3 \\
25-44-21
\end{array}\right]=\frac{1}{40}\left[\begin{array}{c}
40 \\
80 \\
-40
\end{array}\right]\)

= \(\left[\begin{array}{c}
1 \\
2 \\
-1
\end{array}\right]\)

Hence, x=1, y=2 and z=-1

Question 14. x-y+2 z=7,3 x+4 y-5 z=-5,2 x-y+3 z=12

Solution:

The given system can be written as A X=B, where

⇒ \(\mathrm{A}=\left[\begin{array}{ccc}
1 & -1 & 2 \\
3 & 4 & -5 \\
2 & -1 & 3
\end{array}\right]\),

X=\(\left[\begin{array}{l}
x \\
y \\
z
\end{array}\right],\)

and B=\(\left[\begin{array}{c}
7 \\
-5 \\
12
\end{array}\right]\)

Here, \(|A|=\left|\begin{array}{ccc}1 & -1 & 2 \\ 3 & 4 & -5 \\ 2 & -1 & 3\end{array}\right|=1(12-5)-(-1)(9+10)+2(-3-8)=7+19-22=4 \neq 0\)

Thus, A is non-singular.

Therefore, its \(\mathrm{A}^{-1}\) exists.

Therefore, the given system is consistent and has a unique solution given by

⇒ \(\mathrm{X}=\mathrm{A}^{-1} \mathrm{~B}\)

Cofactors of A are,

⇒ \(A_{11}=12-5=7, A_{22}=-(9+10)=-19, A_{13}\)=-3-8=-11

⇒ \(A_{21}=-(-3+2)=1, A_{22}=3-4=-1, A_{23}=-(-1+2)\)=-1

⇒ \(A_{31}=5-8=-3, A_{32}=-(-5-6)=11, A_{39}\)=4+3=7

⇒ \({adj}(\mathrm{A})=\left[\mathrm{A}_{\mathrm{ij}}\right]^{\prime}=\left[\begin{array}{ccc}
7 & -19 & -11 \\
1 & -1 & -1 \\
-3 & 11 & 7
\end{array}\right]^{\prime}=\left[\begin{array}{ccc}
7 & 1 & -3 \\
-19 & -1 & 11 \\
-11 & -1 & 7
\end{array}\right] \)

⇒ \(\mathrm{A}^{-1}=\frac{1}{|\mathrm{~A}|}({adj} \mathrm{A})=\frac{1}{4}\left[\begin{array}{ccc}
7 & 1 & -3 \\
-19 & -1 & 11 \\
-11 & -1 & 7
\end{array}\right]\)

⇒ \(\mathrm{X}=\mathrm{A}^{-1} \mathrm{~B} \Rightarrow\left[\begin{array}{l}
\mathrm{x} \\
\mathrm{y} \\
\mathrm{z}
\end{array}\right]\)

=\(\frac{1}{4}\left[\begin{array}{ccc}
7 & 1 & -3 \\
-19 & -1 & 11 \\
-11 & -1 & 7
\end{array}\right] \cdot\left[\begin{array}{c}
7 \\
-5 \\
12
\end{array}\right]\)

= \(\frac{1}{4}\left[\begin{array}{c}
49-5-36 \\
-133+5+132 \\
-77+5+84
\end{array}\right] \Rightarrow\left[\begin{array}{l}
x \\
y \\
z
\end{array}\right]\)

= \(\frac{1}{4}\left[\begin{array}{l}
8 \\
4 \\
12
\end{array}\right]=\left[\begin{array}{l}
2 \\
1 \\
3
\end{array}\right]\)

Hence, x=2, y=1 and z=3

 Question 15. If A=\(\left[\begin{array}{ccc}2 & -3 & 5 \\ 3 & 2 & -4 \\ 1 & 1 & -2\end{array}\right]\), find \(A^{-1}\). Using \(A^{-1}\), solve the system of equations 2 x-3 y+5 z=11, 3 x+2 y-4 z=-5, x+y-2 z=-3.

Solution:

The given system can written as AX = R,where A = =\(\left[\begin{array}{ccc}
2 & -3 & 5 \\
3 & 2 & -4 \\
1 & 1 & -2
\end{array}\right]\),

X=\(\left[\begin{array}{l}
x \\
y \\
z
\end{array}\right] and B=\left[\begin{array}{c}
11 \\
-5 \\
-3
\end{array}\right]\)

Here, \(|A|=\left|\begin{array}{ccc}
2 & -3 & 5 \\
3 & 2 & -4 \\
1 & 1 & -2
\end{array}\right|\)

=2(-4+4)-(-3)(-6+4)+5(3-2)=0-6+5=-1 \(\neq 0\)

Thus, A is non-singular.

Therefore, its \(\mathrm{A}^{-1}\) exists.

Therefore, the given system is consistent and has a unique solution given by X=\(A^{-1}\) B Cofactors of A are,

⇒ \(A_{11}=-4+4=0, A_{12}=-(-6+4)=2 \cdot A_{13}=3-2=1\)

⇒ \(A_{21}=-(6-5)=-1, A_{22}=-4-5=-9 \cdot A_{33}=-(2+3)=-5\)

⇒ \(A_{13}=(12-10)=2, A_{12}=-(-8-15)=23, A_{73}=4+9=13\)

⇒ \({adj}(\mathrm{A})=\left[\mathrm{A}_{\mathrm{ii}}\right]^{\prime}=\left[\begin{array}{ccc}
0 & 2 & 1 \\
-1 & -9 & -5 \\
2 & 23 & 13
\end{array}\right]=\left[\begin{array}{ccc}
0 & -1 & 2 \\
2 & -9 & 23 \\
1 & -5 & 13
\end{array}\right]\)

⇒ \(A^{-1}=\frac{1}{|A|}({adj} \mathrm{A})=\frac{1}{-1}\left[\begin{array}{ccc}
0 & -1 & 2 \\
2 & -9 & 23 \\
1 & -5 & 13
\end{array}\right]=\left[\begin{array}{ccc}
0 & 1 & -2 \\
-2 & 9 & -23 \\
-1 & 5 & -13
\end{array}\right]\)

Now, \(X=A^{-1} \mathrm{~B} \Rightarrow\left[\begin{array}{l}
\mathrm{x} \\
\mathrm{y} \\
\mathrm{z}
\end{array}\right]=\left[\begin{array}{ccc}
0 & 1 & -2 \\
-2 & 9 & -23 \\
-1 & 5 & -13
\end{array}\right]\left[\begin{array}{c}
11 \\
-5 \\
-3
\end{array}\right]\)

⇒ \(\left[\begin{array}{l}
x \\
y \\
z
\end{array}\right]=\left[\begin{array}{c}
0-5+6 \\
-22-45+69 \\
-11-25+39
\end{array}\right]=\left[\begin{array}{l}
1 \\
2 \\
3
\end{array}\right]\)

Hence, x=1, y=2 and z=3

Question 16. The cost of 4 kg onion, 3 kg wheat, and 2kg rice is ₹ 60. The cost of 2 kg} onion, 4 kg wheat, and 6kg rice is ₹ 90. The cost of 6kg onion, 2 kg wheat, and 3 kg rice is ₹ 70. Find the cost of each item per kg by matrix method.

Solution:

Let the prices (per kg ) of onion, wheat, and rice be ₹ x,₹ y, and ₹ z respectively. Then 4 x+3 y+2 z=60,2 x+4 y+6 z=90,6 x+2 y+3 z=70

The given system can be written as AX = B, where A =\(\left[\begin{array}{lll}4 & 3 & 2 \\ 2 & 4 & 6 \\ 6 & 2 & 3\end{array}\right], \mathrm{X}=\left[\begin{array}{l}\mathrm{x} \\ \mathrm{y} \\ \mathrm{z}\end{array}\right]\)

and B =\(\left[\begin{array}{l}60 \\ 90 \\ 70\end{array}\right]\)

Here, \(|A|=\left|\begin{array}{lll}4 & 3 & 2 \\ 2 & 4 & 6 \\ 6 & 2 & 3\end{array}\right|=4(12-12)-3(6-36)+2(4-24)=0+90-40=50 \neq 0\)

Thus, A is non-singular.

Therefore, its \(A^{-1}\) exists.

Therefore, the given system is consistent and has a unique solution given by X=\(\mathrm{A}^{-1} B\)

Cofactors of A are,

⇒ \(A_{11}=12-12=0, A_{12}=-(6-36)=30, A_{13}=4-24=-20,\)

⇒ \(A_{21}=-(9-4)=-5, A_{22}=12-12=0, A_{23}=-(8-18)=10\),

⇒ \(A_{31}=(18-8)=10, A_{92}=-(24-4)=-20, A_{31}=16-6\)=10

⇒ \({adj}(A)=\left[A_{11}\right]^{\prime}=\left[\begin{array}{ccc}
0 & 30 & -20 \\
-5 & 0 & 10 \\
10 & -20 & 10
\end{array}\right]=\left[\begin{array}{ccc}
0 & -5 & 10 \\
30 & 0 & -20 \\
-20 & 10 & 10
\end{array}\right]\)

⇒ \(A^{-1}=\frac{1}{|A|}(\text { adj } A)=\frac{1}{50}\left[\begin{array}{ccc}
0 & -5 & 10 \\
30 & 0 & -20 \\
-20 & 10 & 10
\end{array}\right]\)

Now, \(X=A^{-1} B \Rightarrow\left[\begin{array}{l}
x \\
y \\
z
\end{array}\right]=\frac{1}{50}\left[\begin{array}{ccc}
0 & -5 & 10 \\
30 & 0 & -20 \\
-20 & 10 & 10
\end{array}\right]\)

=\(\left[\begin{array}{l}
60 \\
90 \\
70
\end{array}\right]=\frac{1}{50}\left[\begin{array}{c}
\sigma-450+700 \\
1800+0-1400 \\
-1200+900+700
\end{array}\right]\)

⇒ \(\left[\begin{array}{l}
x \\
y \\
z
\end{array}\right]=\frac{1}{50}\left[\begin{array}{l}
250 \\
400 \\
400
\end{array}\right]\)

= \(\left[\begin{array}{l}
5 \\
8 \\
8
\end{array}\right] \Rightarrow\left[\begin{array}{l}
x \\
y \\
z
\end{array}\right]=\left[\begin{array}{l}
5 \\
8 \\
8
\end{array}\right]\)

x=5, y=8 and z=8 .

Hence, the price of onion per kg is ₹ 5, the price of wheat per kg is ₹ 8 and that of rice per kg is ₹ 8.

Determinants Miscellaneous Exercise

Question 1. Prove that the determinant \(\left|\begin{array}{ccc}x & \sin \theta & \cos \theta \\ -\sin \theta & -x & 1 \\ \cos \theta & 1 & x\end{array}\right|\) is independent of \theta.

Solution:

Let \(|A|=\left|\begin{array}{ccc}x & \sin \theta & \cos \theta \\ -\sin \theta & -x & 1 \\ \cos \theta & 1 & x\end{array}\right|\)

Expanding to the corresponding first row, we get

⇒ \(|A| =x\left|\begin{array}{cc}
-x & 1 \\
1 & x
\end{array}\right|-\sin \theta\left|\begin{array}{cc}
-\sin \theta & 1 \\
\cos \theta & x
\end{array}\right|+\cos \theta\left|\begin{array}{cc}
-\sin \theta & -x \\
\cos \theta & 1
\end{array}\right|\)

= x\(\left(-x^2-1\right)-\sin \theta(-x \sin \theta-\cos \theta)+\cos \theta(-\sin \theta+x \cos \theta)\)

=-\(x^3-x+x \sin ^2 \theta+\sin \theta \cos \theta-\sin \theta \cos \theta+x \cos ^2 \theta \quad\left(\sin ^2 \theta+\cos ^2 \theta=1\right)\)

=-\(x^3-x+x\left(\sin ^2 \theta+\cos ^2 \theta\right)=-x^3-x+x\)

= \(-x^3\)

Hence, A is independent of \(\theta\).

Question 2. Evaluate \(\left|\begin{array}{ccc}
\cos \alpha \cos \beta & \cos \alpha \sin \beta & -\sin \alpha \\
-\sin \beta & \cos \beta & 0 \\
\sin \alpha \cos \beta & \sin \alpha \sin \beta & \cos \alpha
\end{array}\right|\).

Solution:

Given, \(|A|=\left|\begin{array}{ccc}\cos \alpha \cos \beta & \cos \alpha \sin \beta & -\sin \alpha \\ -\sin \beta & \cos \beta & 0 \\ \sin \alpha \cos \beta & \sin \alpha \sin \beta & \cos \alpha\end{array}\right|\)

Expanding corresponding to \(R_1\), we get

|A| =\(\cos \alpha \cos \beta(\cos \alpha \cos \beta-0)-\cos \alpha \sin \beta(-\cos \alpha \sin \beta-0)\)

–\(\sin \alpha\left(-\sin ^2 \beta \sin \alpha-\cos ^2 \beta \sin \alpha\right)\)

= \(\cos ^2 \alpha(\cos ^2 \beta+\sin ^2 \beta)+\sin ^2 \alpha(\sin ^2 \beta+\cos ^2 \beta) [\sin ^2 \theta+\cos ^2 \theta=1]\)

= \(\cos ^2 \alpha(1)+\sin ^2 \alpha(1)=\cos ^2 \alpha+\sin ^2 \alpha=1\) .

Question 3. If \(A^{-1}=\left[\begin{array}{ccc}3 & -1 & 1 \\ -15 & 6 & -5 \\ 5 & -2 & 2\end{array}\right]\) and

B=\(\left[\begin{array}{ccc}1 & 2 & -2 \\ -1 & 3 & 0 \\ 0 & -2 & 1\end{array}\right]\), find \((A B)^{-1}\).

Solution:

We know that \((\mathrm{AB})^{-1}=\mathrm{B}^{-1} \mathrm{~A}^{-1}\) and \(\mathrm{A}^{-1}\) is known, therefore we proceed to find \(\mathrm{B}^{-1}\).

Here, \(|B|=\left|\begin{array}{ccc}1 & 2 & -2 \\ -1 & 3 & 0 \\ 0 & -2 & 1\end{array}\right|=1(3-0)-2(-1-0)-2(2-0)=3+2-4=1 \neq 0\)

⇒ \(\mathrm{B}^{-1}\) exists. Cofactors of B are:

⇒ \(\mathrm{B}_{11}=(3-0)=3, \mathrm{~B}_{12}=-(-1-0)=1\),

⇒ \(\mathrm{~B}_{13}\)=(2-0)=2,

⇒ \(\mathrm{~B}_{11}=-(2-4)=2, \mathrm{~B}_{21}=(1-0)\)=1,

⇒ \(\mathrm{~B}_{23}\)=-(-2-0)=2,

⇒ \(\mathrm{~B}_{31}\)=(0+6)=6,

⇒ \(\mathrm{~B}_{32}\)=-(0-2)=2,

⇒ \(\mathrm{~B}_{33}=(3+2)\)=5,

⇒ \({adj}(\mathrm{B})=\left[\mathrm{B}_6\right]^{+}=\left[\begin{array}{lll}
3 & 1 & 2 \\
2 & 1 & 2 \\
6 & 2 & 5
\end{array}\right]=\left[\begin{array}{lll}
3 & 2 & 6 \\
1 & 1 & 2 \\
2 & 2 & 5
\end{array}\right]\)

⇒ \(B^{-1}=\frac{1}{|B|} {adj}(B)=\frac{1}{1}\left[\begin{array}{lll}
3 & 2 & 6 \\
1 & 1 & 2 \\
2 & 2 & 5
\end{array}\right]\)

= \(\left[\begin{array}{lll}
3 & 2 & 6 \\
1 & 1 & 2 \\
2 & 2 & 5
\end{array}\right]\)

Now,\((A B)^{-1}=B^{-1} A^{-1}=\left[\begin{array}{lll}
3 & 2 & 6 \\
1 & 1 & 2 \\
2 & 2 & 5
\end{array}\right]\left[\begin{array}{ccc}
3 & -1 & 1 \\
-15 & 6 & -5 \\
5 & -2 & 2
\end{array}\right]\)

= \(\left[\begin{array}{ccc}
9-30+30 & -3+12-12 & 3-10+12 \\
3-15+10 & -1+6-4 & 1-5+4 \\
6-30+25 & -2+12-10 & 2-10+10
\end{array}\right]\)

= \(\left[\begin{array}{ccc}
9 & -3 & 5 \\
-2 & 1 & 0 \\
1 & 0 & 2
\end{array}\right]\)

Question 4. Evaluate \(\left|\begin{array}{ccc}x & y & x+y \\ y & x+y & x \\ x+y & x & y\end{array}\right|\)

Solution:

Let \(\Delta=\left|\begin{array}{ccc}x & y & x+y \\ y & x+y & x \\ x+y & x & y\end{array}\right|\)

Expand along \(R_1\)

⇒ \(\Delta=x\left(x y+y^2-x^2\right)-y\left(y^2-x^2-x y\right)+(x+y)\left(x y-x^2-2 x y-y^2\right)\)

⇒ \(\Delta=x^2 y+x y^2-x^3-y^3+x^2 y+x y^2+x^2 y-x^3-2 x^2 y-x y^2+x y^2-x^2 y-2 x y^2-y^2\)

⇒ \(\Delta=-2\left(x^3+y^3\right)\)

Question 5. Evaluate \(\left|\begin{array}{ccc}1 & x & y \\ 1 & x+y & y \\ 1 & x & x+y\end{array}\right|\),

Solution:

Let \(\Delta=\left|\begin{array}{ccc}1 & x & y \\ 1 & x+y & y \\ 1 & x & x+y\end{array}\right|\)

Expand along C

⇒ \(\Delta=1\left(x^2+2 x y+y^2-x y\right)-1\left(x^2+x y-x y\right)+1\left(x y-x y-y^2\right)\)

⇒ \(\Delta-x^2+x y+y^2-x^2-y^2\)=x y

Question 6. If x, y, z are non-zero real numbers, then the inverse of matrix A=\(\left[\begin{array}{lll}x & 0 & 0 \\ 0 & y & 0 \\ 0 & 0 & z\end{array}\right]\) is ?

  1. \(\left[\begin{array}{ccc}x^{-1} & 0 & 0 \\ 0 & y^{-1} & 0 \\ 0 & 0 & z^{-1}\end{array}\right]\)
  2. \(x y z\left[\begin{array}{ccc}x^{-1} & 0 & 0 \\ 0 & y^{-1} & 0 \\ 0 & 0 & z^{-1}\end{array}\right]\)
  3. \(\frac{1}{x y z}\left[\begin{array}{ccc}x & 0 & 0 \\ 0 & y & 0 \\ 0 & 0 & z\end{array}\right]\)
  4. \(\frac{1}{x y z}\left[\begin{array}{lll}1 & 0 & 0 \\ 0 & 1 & 0 \\ 0 & 0 & 1\end{array}\right]\)

Solution: 1. \(\left[\begin{array}{ccc}x^{-1} & 0 & 0 \\ 0 & y^{-1} & 0 \\ 0 & 0 & z^{-1}\end{array}\right]\)

Given, A=\(\left[\begin{array}{lll}x & 0 & 0 \\ 0 & y & 0 \\ 0 & 0 & z\end{array}\right]\)

⇒ \(|A|=\left[\begin{array}{lll}x & 0 & 0 \\ 0 & y & 0 \\ 0 & 0 & z\end{array}\right]=x(y z-0)=x y z \neq\) 0(because x, y and z are non-zero )

⇒\(\mathrm{A}^{-1}\) exists

Cofactors of A are:

⇒ \(A_{11}=(y z-0)=y z_1 A_{12}=-(0-0)=0, A_{13}=0-0=0\),

⇒ \(A_{21}=-(0-0)=0, A_{22}=x z-0=x z_{23}\)=-(0-0)=0,

⇒ \(A_{31}=0-0=0, A_{31}=-(0-0)=0, A_{13}=(x y-0)\)=x y

⇒ \({adj}(A)=\left[A_{i t}\right]^{\prime}=\left[\begin{array}{ccc}
y z & 0 & 0 \\
0 & x z & 0 \\
0 & 0 & x y
\end{array}\right]^{\prime}=\left[\begin{array}{ccc}
y z & 0 & 0 \\
0 & x z & 0 \\
0 & 0 & x y
\end{array}\right] \)

Now, \(A^{-1}=\frac{1}{|A|}({adj} A) \Rightarrow A^{-1}=\frac{1}{x y z}\left[\begin{array}{ccc}
y z & 0 & 0 \\
0 & x z & 0 \\
0 & 0 & x y
\end{array}\right]=\left[\begin{array}{ccc}
\frac{1}{x} & 0 & 0 \\
0 & \frac{1}{y} & 0 \\
0 & 0 & \frac{1}{z}
\end{array}\right]\)

= \(\left[\begin{array}{ccc}
x^{-1} & 0 & 0 \\
0 & y & 0 \\
0 & 0 & z^{-1}
\end{array}\right]\)

Hence, the correct option is 1.

Question 7. Let A=\(\left[\begin{array}{ccc}1 & \sin \theta & 1 \\ -\sin \theta & 1 & \sin \theta \\ -1 & -\sin \theta & 1\end{array}\right]\), where 0 \(\leq \theta \leq 2 \pi\), then:

  1. \({\det} \mathrm{A}\)=0
  2. \({\det} \mathrm{A} \in(2, \infty)\)
  3. \({\det} \mathrm{A} \in(2,4)\)
  4. \({\det} \mathrm{A} \in[2,4]\)

Solution: 4. \({det} \mathrm{A} \in[2,4]\)

Given, A=\(\left[\begin{array}{ccc}1 & \sin \theta & 1 \\ -\sin \theta & 1 & \sin \theta \\ -1 & -\sin \theta & 1\end{array}\right]\)

⇒ \(|A|=\left|\begin{array}{ccc}
1 & \sin \theta & 1 \\
-\sin \theta & 1 & \sin \theta \\
-1 & -\sin \theta & 1
\end{array}\right|\)

=1\(\left(1+\sin ^2 \theta\right)-\sin \theta(-\sin \theta+\sin \theta)+1\left(\sin ^2 \theta+1\right)\)

⇒ \(|A|=2+2 \sin ^2 \theta\)

For 0 \(\leq \theta \leq 2 \pi,-1 \leq \sin \theta \leq 1 \Rightarrow 0 \leq \sin ^2 \theta \leq 1\)

1 \(\leq 1+\sin ^2 \theta \leq 2 \Rightarrow 2 \leq 2\left(1+\sin ^2 \theta\right) \leq 4 {det}(A) \in[2,4]\)

Hence, the correct option is 4.

Probability Class 12 Maths Important Questions Chapter 13

Probability

Question 1. Given that E and F are events such that P(E)=0.6, P(F)=0.3 and \(\mathrm{P}(\mathrm{E} \cap \mathrm{F})=0.2\), find \(\mathrm{P}(\mathrm{E} \mid \mathrm{F})\) and \(P(F \mid E)\).
Solution:

It is given that P(E)=0.6, P(F)=0.3, and \(P(E \cap F)=0.2\)

⇒ \(P(E \mid F)=\frac{P(E \cap F)}{P(F)}=\frac{0.2}{0.3}=\frac{2}{3}\)

⇒ \(P(F \mid E)=\frac{P(F \cap E)}{P(E)}=\frac{0.2}{0.6}=\frac{1}{3}\)

Question 2. Compute \(\mathrm{P}(\mathrm{A} \mid \mathrm{B})\), if \(\mathrm{P}(\mathrm{B})=0.5\) and \(\mathrm{P}(\mathrm{A} \cap \mathrm{B})=0.32\)
Solution:

It is given that P(B)=0.5 and \(P(A \cap B)=0.32 \Rightarrow P\left(\frac{A}{B}\right)=\frac{P(A \cap B)}{P(B)}=\frac{0.32}{0.5}=\frac{16}{25}\)

Question 3. If P(A)=0.8, P(B)=0.5 and \(P(B \mid A)=0.4\), find

  1. \(\mathrm{P}(\mathrm{A} \cap \mathrm{B})\)
  2. \(\mathrm{P}(\mathrm{A} \mid \mathrm{B})\)
  3. \(\mathrm{P}(\mathrm{A} \cup \mathrm{B})\)

Solution:

It is given that P(A)=0.8, P(B)=0.5, and \(P(B \mid A)=0.4\)

1. \(\mathrm{P}(\mathrm{B} \mid \mathrm{A})=0.4\)

∴ \(\frac{P(B \cap A)}{P(A)}=0.4\)

(because \(\mathrm{P}\left(\frac{\mathrm{B}}{\mathrm{A}}\right)=\frac{\mathrm{P}(\mathrm{B} \cap \mathrm{A})}{\mathrm{P}(\mathrm{A})}\))

∴ \(\frac{\mathrm{P}(\mathrm{A} \cap \mathrm{B})}{0.8}=0.4 \Rightarrow \mathrm{P}(\mathrm{A} \cap \mathrm{B})=0.32\)

(because \(B \cap A=A \cap B\))

2. \(\mathrm{P}(\mathrm{A} \mid \mathrm{B})=\frac{\mathrm{P}(\mathrm{A} \cap \mathrm{B})}{\mathrm{P}(\mathrm{B})}\)

(because \(\mathrm{P}\left(\frac{\mathrm{A}}{\mathrm{B}}\right)=\frac{\mathrm{P}(\mathrm{A} \cap \mathrm{B})}{\mathrm{P}(\mathrm{B})}\))

⇒ \(\mathrm{P}(\mathrm{A} \mid \mathrm{B})=\frac{0.32}{0.5}=0.64\)

3. \(\mathrm{P}(\mathrm{A} \cup \mathrm{B})=\mathrm{P}(\mathrm{A})+\mathrm{P}(\mathrm{B})-\mathrm{P}(\mathrm{A} \cap \mathrm{B}) \Rightarrow \mathrm{P}(\mathrm{A} \cup \mathrm{B})=0.8+0.5-0.32=0.98\)

Read and Learn More Class 12 Maths Chapter Wise with Solutions

Question 4. Evaluate \(\mathrm{P}(\mathrm{A} \cup \mathrm{B})\), if 2P(A)=P(B)=\(\frac{5}{13}\) and \(\mathrm{P}(\mathrm{A} \mid \mathrm{B})=\frac{2}{5}\)
Solution:

It is given that, \(2 P(A)=P(B)=\frac{5}{13} \Rightarrow P(A)=\frac{5}{26}\) and \(P(B)=\frac{5}{13}\)

Now, \(\mathrm{P}(\mathrm{A} \mid \mathrm{B})=\frac{2}{5} \Rightarrow \frac{\mathrm{P}(\mathrm{A} \cap \mathrm{B})}{\mathrm{P}(\mathrm{B})}=\frac{2}{5}\)

(because \(\mathrm{P}\left(\frac{\mathrm{A}}{\mathrm{B}}\right)=\frac{\mathrm{P}(\mathrm{A} \cap \mathrm{B})}{\mathrm{P}(\mathrm{B})}\))

⇒ \(\mathrm{P}(\mathrm{A} \cap \mathrm{B})=\frac{2}{5} \times \mathrm{P}(\mathrm{B})=\frac{2}{5} \times \frac{5}{13}=\frac{2}{13}\)

It is known that, \(\mathrm{P}(\mathrm{A} \cup \mathrm{B})=\mathrm{P}(\mathrm{A})+\mathrm{P}(\mathrm{B})-\mathrm{P}(\mathrm{A} \cap \mathrm{B})\)

⇒ \(\mathrm{P}(\mathrm{A} \cup \mathrm{B})=\frac{5}{26}+\frac{5}{13}-\frac{2}{13} \Rightarrow \mathrm{P}(\mathrm{A} \cup \mathrm{B})=\frac{5+10-4}{26} \Rightarrow \mathrm{P}(\mathrm{A} \cup \mathrm{B})=\frac{11}{26}\)

Question 5. If \(P(A)=\frac{6}{11}, P(B)=\frac{5}{11}\) and \(P(A \cup B)=\frac{7}{11}\), find

  1. \(\mathrm{P}(\mathrm{A} \cap \mathrm{B})\)
  2. \(\mathrm{P}(\mathrm{A} \mid \mathrm{B})\)
  3. \(\mathrm{P}(\mathrm{B} \mid \mathrm{A})\)

Solution:

It is given that \(P(A)=\frac{6}{11}, P(B)=\frac{5}{11}\) and \(P(A \cup B)=\frac{7}{11}\)

1. P(A ∪ B)=\(\frac{7}{11}\) (because \(P(A \cup B)=P(A)+P(B)-P(A \cap B)\))

∴ \(P(A)+P(B)-P(A \cap B)=\frac{7}{11}\)

⇒ \(\frac{6}{11}+\frac{5}{11}-P(A \cap B)=\frac{7}{11}\)

⇒ \(P(A \cap B)=\frac{11}{11}-\frac{7}{11}=\frac{4}{11}\)

2. It is known that, \(\mathrm{P}(\mathrm{A} \mid \mathrm{B})=\frac{\mathrm{P}(\mathrm{A} \cap \mathrm{B})}{\mathrm{P}(\mathrm{B})} \Rightarrow \mathrm{P}(\mathrm{A} \mid \mathrm{B})=\frac{\frac{4}{11}}{\frac{5}{11}}=\frac{4}{5}\)

3. It is known that, \(\mathrm{P}(\mathrm{B} \mid \mathrm{A})=\frac{\mathrm{P}(\mathrm{B} \cap \mathrm{A})}{\mathrm{P}(\mathrm{A})} \Rightarrow \mathrm{P}(\mathrm{B} \mid \mathrm{A})=\frac{\frac{4}{11}}{\frac{6}{11}}=\frac{4}{6}=\frac{2}{3}\)

Determine P(E | F)

Question 6. A coin is tossed three times, where

  1. E: head on third toss, F: heads on first two tosses
  2. E: at least two heads, F: at most two heads
  3. E: at most two tails, F: at least one tail

Solution:

If a coin is tossed three times, then the sample space S is S = {HHH, HHT, HTH, HTT, THH, THT, TTH, TTT}

⇒ n(S) = 8

1. E = {HHH, HTH, THH, TTH}

F = {HHH, HHT}

∴ E ∩ F = {HHH}

∴ P(F) = \(\frac{2}{8}=\frac{1}{4} \text { and } P(E \cap F)=\frac{1}{8}\)

∴ \(P(E \mid F)=\frac{P(E \cap F)}{P(F)}=\frac{\frac{1}{8}}{\frac{1}{4}}=\frac{4}{8}=\frac{1}{2}\)

2. E = {HHH, HHT, HTH, THH}

F = {HHT, HTH, HTT, THH, THT, TTH, TTT}

∴ E ∩ F = {HHT, HTH, THH}

Clearly, \(\mathrm{P}(\mathrm{E} \cap \mathrm{F})=\frac{3}{8}\) and \(\mathrm{P}(\mathrm{F})=\frac{7}{8}\)

∴ \(\mathrm{P}(\mathrm{E} \mid \mathrm{F})=\frac{\mathrm{P}(\mathrm{E} \cap \mathrm{F})}{\mathrm{P}(\mathrm{F})}=\frac{\frac{3}{8}}{\frac{7}{8}}=\frac{3}{7}\)

3. E = {HHH, HHT, HTT, HTH, THH, THT, TTH}

F = {HHT, HTT, HTH, THH, THT, TTH, TTT}

∴ E ∩ F = {HHT, HTT, HTH, THH, THT, TTH}

⇒ \(\mathrm{P}(\mathrm{F})=\frac{7}{8}\) and \(\mathrm{P}(\mathrm{E} \cap \mathrm{F})=\frac{6}{8}\)

Therefore, \(\mathrm{P}(\mathrm{E} \mid \mathrm{F})=\frac{\mathrm{P}(\mathrm{E} \cap \mathrm{F})}{\mathrm{P}(\mathrm{F})}=\frac{\frac{6}{8}}{\frac{8}{8}}=\frac{6}{7}\)

CBSE Class 12 Maths Chapter 13 Probability Important Question And Answers

Question 7. Two coins are tossed once, where:

  1. E: tail appears on one coin, F: one coin shows the head
  2. E: no tail appears, F: no head appears

Solution:

If two coins are tossed once, then the sample space S is S = {HH, HT, TH, TT}

1. E = {HT, TH}, F = {HT, TH}

∴ E ∩ F = {HT, TH}

⇒ \(\mathrm{P}(\mathrm{F})=\frac{2}{4}=\frac{1}{2}\)

∴ \(\mathrm{P}(\mathrm{E} \cap \mathrm{F})=\frac{2}{4}=\frac{1}{2} \Rightarrow \mathrm{P}(\mathrm{E} \mid \mathrm{F})=\frac{\mathrm{P}(\mathrm{E} \cap \mathrm{F})}{\mathrm{P}(\mathrm{F})}=\frac{\frac{1}{2}}{\frac{1}{2}}=1\)

2. E = {HH} [Set of events having no tail]

F = {TT} [Set of events having no head]

∴ \(E \cap F=\phi\)

P(E) = \(\frac{1}{4}, P(F)=\frac{1}{4}\) and \(P(E \cap F)=0\)

∴ \(P(E \mid F)=\frac{P(E \cap F)}{P(F)}=\frac{0}{1 / 4}=0\)

Question 8. A die is thrown three times, E: 4 appears on the third toss, F: 6 and 5 appear respectively on the first two tosses
Solution:

If a die is thrown three times, then the number of elements in the sample space will be 6x6x6 = 216 The sample space is S = {(x, y, z): x, y, z ∈ 1,2, 3, 4, 5, 6}

E = \(\left\{\begin{array}{l}
(1,1,4),(1,2,4), \ldots . .(1,6,4) \\
(2,1,4),(2,2,4), \ldots \ldots(2,6,4) \\
(3,1,4),(3,2,4), \ldots \ldots(3,6,4) \\
(4,1,4),(4,2,4), \ldots \ldots(4,6,4) \\
(5,1,4),(5,2,4), \ldots \ldots(5,6,4) \\
(6,1,4),(6,2,4), \ldots \ldots . .(6,6,4)
\end{array}\right\} \Rightarrow n(E)=36\)

F = {(6, 5, 1), (6, 5, 2), (6, 5, 3), (6, 5, 4), (6, 5, 5), (6, 5, 6)}

∴ \(E \cap F=\{(6,5,4)\} \Rightarrow n(E \cap F)=1\)

P(F) = \(\frac{6}{216} \text { and } P(E \cap F)=\frac{1}{216}\)

⇒ \(P(E \mid F)=\frac{P(E \cap F)}{P(F)}=\frac{\frac{1}{6}}{\frac{216}{216}}=\frac{1}{6}\)

Question 9. Mother, father, and son line up at random for a family picture, E: son on one end, F: father in middle.
Solution:

If the mother (M), father (F), and son (S) line up for the family picture, then the sample space will be S = {MFS, MSF, FMS, FSM, SMF, SFM}

⇒ E = (MFS, FMS, SMF, SFM}, F = (MFS, SFM} => E n F = (MFS, SFM} = 2

⇒ \(P(E \cap F)=\frac{2}{6}=\frac{1}{3} \text { and } P(F)=\frac{2}{6}=\frac{1}{3} \Rightarrow P(E \mid F)=\frac{P(E \cap F)}{P(F)}=\frac{\frac{1}{3}}{\frac{1}{3}}=1\)

 

Question 10. A black and a red dice are rolled.

  1. Find the conditional probability of obtaining a sum greater than 9, given that the black die resulted in a 5.
  2. Find the conditional probability of obtaining the sum 8, given that the red die resulted in a number less than 4.

Solution:

Let the first observation be from the black die and the second from the red die. When two dice (one black and another red) are rolled, the sample space S = 6 x 6 = 36 number of elements.

1. Let A: Obtaining a sum greater than 9 = {(4, 6), (5. 5), (5, 6), (6, 4), (6, 5), (6, 6)}

B: Black die results in a 5 = {(5, 1), (5,2), (5, 3), (5,4), (5, 5), (5, 6)}

∴ A ∩ B= {(5, 5), (5, 6)}

The conditional probability of obtaining a sum greater than 9, given that the black die resulted in a 5, is given by P (A|B).

∴ P(A|B) = \(\frac{\mathrm{P}(\mathrm{A} \cap \mathrm{B})}{\mathrm{P}(\mathrm{B})}=\frac{\frac{2}{36}}{\frac{6}{36}}=\frac{2}{6}=\frac{1}{3}\)

2. E: Sum of the observations is 8 = {(2, 6), (3, 5), (4, 4), (5, 3), (6, 2)}

F: Red die resulted in a number less than 4 = \(\left\{\begin{array}{l}
(1,1),(1,2),(1,3),(2,1),(2,2),(2,3), \\
(3,1),(3,2),(3,3),(4,1),(4,2),(4,3), \\
(5,1),(5,2),(5,3),(6,1),(6,2),(6,3)
\end{array}\right\}\)

∴ E ∩ F = {(5, 3),(6, 2)}

P(F) = 18/36 and P(E n F) = 2/36

The conditional probability of obtaining the sum equal to 8, given that the red die resulted in a number less than 4, is given by P (E | F).

Therefore, \(P(E \mid F)=\frac{P(E \cap F)}{P(F)}=\frac{\frac{2}{36}}{\frac{18}{36}}=\frac{2}{18}=\frac{1}{9}\)

Question 11. A fair die is rolled. Consider events E = {1, 3, 5}, F = {2, 3} and G = (2, 3,4, 5}. Find

  1. P(E | F) and P(F | E)
  2. P(E | G) and P(G | E)
  3. P((E ∪ F) | G) and P ((E ∩ F) | G)

Solution:

When a fair die is rolled, the sample space S will be S = {1,2,3, 4, 5, 6)

It is given that E = {1, 3, 5}, F = z[2, 3}, and G = {2, 3, 4, 5}

∴ P(E) = \(\frac{3}{6}=\frac{1}{2}, P(F)=\frac{2}{6}=\frac{1}{3}, P(G)=\frac{4}{6}=\frac{2}{3}\)

1. \(\mathrm{E} \cap \mathrm{F}=\{3\} \Rightarrow \mathrm{P}(\mathrm{E} \cap \mathrm{F})=\frac{1}{6}\)

∴ \(\mathrm{P}(\mathrm{E} \mid \mathrm{F})=\frac{\mathrm{P}(\mathrm{E} \cap \mathrm{F})}{\mathrm{P}(\mathrm{F})}=\frac{\frac{1}{6}}{\frac{1}{3}}=\frac{1}{2} ; \mathrm{P}(\mathrm{F} \mid \mathrm{E})=\frac{\mathrm{P}(\mathrm{F} \cap \mathrm{E})}{\mathrm{P}(\mathrm{E})}=\frac{\frac{1}{6}}{\frac{1}{2}}=\frac{1}{3}\)

2. \(\mathrm{E} \cap \mathrm{G}=\{3,5\}\)

∴ \(\mathrm{P}(\mathrm{E} \cap \mathrm{G})=\frac{2}{6}=\frac{1}{3}\)

∴ \(P(E \mid G)=\frac{P(E \cap G)}{P(G)}=\frac{\frac{1}{3}}{\frac{2}{3}}=\frac{1}{2} ; P(G \mid E)=\frac{P(G \cap E)}{P(E)}=\frac{\frac{1}{3}}{\frac{1}{2}}=\frac{2}{3}\)

3. \(\mathrm{P} \cup \mathrm{F}=\{1,2,3,5\}\)

⇒ \((\mathrm{E} \cup \mathrm{F}) \cap \mathrm{G}=\{1,2,3,5\} \cap\{2,3,4,5\}=\{2,3,5\}\)

⇒ \(\mathrm{E} \cap \mathrm{F}=\{3\}\)

⇒ \((\mathrm{E} \cap \mathrm{F}) \cap \mathrm{G}=\{3\} \cap\{2,3,4,5\}=\{3\}\)

⇒ \(P((\mathrm{E} \cup \mathrm{F}) \cap \mathrm{G})=\frac{3}{6}=\frac{1}{2}, \mathrm{P}((\mathrm{E} \cap \mathrm{F}) \cap \mathrm{G})=\frac{1}{6}\)

∴ \(\mathrm{P}((\mathrm{E} \cap \mathrm{F}) \mid \mathrm{G})=\frac{\mathrm{P}((\mathrm{E} \cap \mathrm{F}) \cap \mathrm{G})}{\mathrm{P}(\mathrm{G})}=\frac{\frac{1}{6}}{\frac{2}{3}}=\frac{1}{6} \times \frac{3}{2}=\frac{1}{4}\)

Question 12. Assume that each bom child is equally likely to be a boy or a girl. If a family has two children, what is the conditional probability that both are girls given that

  1. The youngest is a girl,
  2. At least one is a girl?

Solution:

Let b and g represent the boy and the girl child respectively. If a family has two children, the sample space will be

S = {(b, b), (b, g), (g, b), (g, g)}

Let A be the event that both children are girls.

∴ A={(g,g)}

1. Let B be the event that the youngest child is a girl.

∴ B = [(b, g), (g, g)] ⇒ A ∩ B = {(g, g)}

∴ \(P(B)=\frac{2}{4}=\frac{1}{2}, \quad P(A \cap B)=\frac{1}{4}\)

The conditional probability that both are girls, given that the youngest child is a girl, is given by P (A | B).

⇒ \(\mathrm{P}(\mathrm{A} \mid \mathrm{B})=\frac{\mathrm{P}(\mathrm{A} \cap \mathrm{B})}{\mathrm{P}(\mathrm{B})}=\frac{\frac{1}{4}}{\frac{1}{2}}=\frac{1}{2}\)

Therefore, the required probability is 1/2.

2. Let C be the event that at least one child is a girl.

∴ C = {(b, g), (g,b), (g, g)} ⇒ A ∩ C = {g, g} ⇒ P(C) = 3/4

⇒ P(A ∩ C) = 1/4

The conditional probability that both are girls, given that at least one child is a girl, is given by P(A|C).

Therefore. P(A C) = \(P(A \mid C)=\frac{P(A \cap C)}{P(C)}=\frac{\frac{1}{4}}{\frac{3}{4}}=\frac{1}{3}\)

Question 13. An instructor has a question bank consisting of 300 easy True/False questions, 200 difficult True/False questions, 500 easy multiple choice questions, and 400 difficult multiple choice questions. If a question is selected at random from the question bank, what is the probability that it will be an easy question given that it is a multiple-choice question?
Solution:

The given data can be tabulated as

Probability Instructor Has A Question bank

Let us denote E = easy questions, M = multiple choice questions, D = difficult questions, and T = True/False questions

Total number of questions = 1400

Total number of multiple choice questions = 900

Therefore, the probability of selecting an easy multiple choice question is \(P(E \cap M)=\frac{500}{1400}=\frac{5}{14}\)

Probability of selecting a multiple choice question, \(P(M)=\frac{900}{1400}=\frac{9}{14}\)

P (E | M) represents the probability that a randomly selected question will be an easy question, given that it is a multiple-choice question.

∴ \(\mathrm{P}(\mathrm{E} \mid \mathrm{M})=\frac{\mathrm{P}(\mathrm{E} \cap \mathrm{M})}{\mathrm{P}(\mathrm{M})}=\frac{\frac{-5}{14}}{\frac{9}{14}}=\frac{5}{9}\)

Therefore, the required probability is 5/9

Question 14. Given that the two numbers appearing on throwing the two dice are different. Find the probability of the event ‘the sum of numbers on the dice is 4.
Solution:

When two dice are thrown, the number of observations in the sample space = 6 x 6 = 36

Let A be the event that the sum of the numbers on both dice is 4 and B be the event that the two numbers appearing on throwing the two dice are different.

∴ A ={(1,3), (2, 2), (3,1)}

⇒ n(A) = 3

B = \(\left\{\begin{array}{l}
(1,2),(1,3),(1,4),(1,5),(1,6) \\
(2,1),(2,3),(2,4),(2,5),(2,6) \\
(3,1),(3,2),(3,4),(3,5),(3,6) \\
(4,1),(4,2),(4,3),(4,5),(4,6) \\
(5,1),(5,2),(5,3),(5,4),(5,6) \\
(6,1),(6,2),(6,3),(6,4),(6,5)
\end{array}\right\}=n(B)=30\)

∴ \(A \cap B=\{(1,3),(3,1)\}\)

⇒ P(B) = \(\frac{30}{36}=\frac{5}{6} \text { and } P(A \cap B)=\frac{2}{36}=\frac{1}{18}\)

Let \(\mathrm{P}(\mathrm{A} \mid \mathrm{B})\) represent the probability that the sum of the numbers on both dice is 4, given that the two numbers appearing on throwing the two dice are different.

∴ \(\mathrm{P}(\mathrm{A} \mid \mathrm{B})=\frac{\mathrm{P}(\mathrm{A} \cap \mathrm{B})}{\mathrm{P}(\mathrm{B})}=\frac{\frac{1}{18}}{\frac{5}{6}}=\frac{1}{15}\)

Therefore, the required probability is \(\frac{1}{15}\).

Question 15. Consider the experiment of throwing a die, if a multiple of 3 comes up, throw the die again and if any other number comes, toss a coin. Find the conditional probability of the event ‘the coin shows a tail’, given that at least one die shows a 3.
Solution:

The sample space of the experiment is,

S = \(\left\{\begin{array}{l}
(1, H),(1, T),(2, H),(2, T),(3,1),(3,2),(3,3),(3,4),(3,5),(3,6) \\
(4, H),(4, T),(5, H),(5, T),(6,1),(6,2),(6,3),(6,4),(6,5),(6,6)
\end{array}\right\}\)

Let A be the event that the coin shows a tail and B be the event that at least one die shows 3.

∴ A = {(1, T), (2, T), (4, T), (5, T)}

B = {(3,1), (3, 2), (3, 3), (3, 4), (3, 5), (3, 6), (6, 3)}

⇒ A ∩ B = ø

∴ P(A ∩ B) = 0

because there are no common elements.

Then, P(B) =P({3,1}) + P({3,2}) + P({3,3}) + P({3,4}) + P({3,5}) + P({3,6}) + P({6,3})

= \(\frac{1}{36}+\frac{1}{36}+\frac{1}{36}+\frac{1}{36}+\frac{1}{36}+\frac{1}{36}+\frac{1}{36}=\frac{7}{36}\)

The probability of the event that the coin shows a tail, given that at least one die shows 3, is given by P(A | B).

Therefore. \(P(A \mid B)=\frac{P(A \cap B)}{P(B)}=\frac{0}{\frac{7}{36}}=0\)

Choose The Correct Answer

Question 16. If P(A) = \(\frac{1}{2}\), P(B) = 0, then P(A | B) is

  1. 0
  2. \(\frac{1}{2}\)
  3. Not Defined
  4. 1

Solution: 3. Not defined

It is given that P(A) = \(\frac{1}{2}\) and P(B) = 0

P\((A \mid B)=\frac{P(A \cap B)}{P(B)}=\frac{P(A \cap B)}{0}\)

Therefore, P (A | B) is not defined.

Thus, the correct Answer is 3.

Question 17. If A and B are events such that P (A | B) = P(B | A), then

  1. A ⊂ B but A ≠ B
  2. A = B
  3. A ∩ B = ø
  4. P(A) = P(B)

Solution: 4. P(A) = P(B)

It is given that, P(A | B) = P(B | A)

⇒ \(\frac{P(A \cap B)}{P(B)}=\frac{P(B \cap A)}{P(A)}\)

⇒ P(A)=P(B)

Thus, the correct Answer is 4.

Probability Exercise 13.2

Question 1. If P(A) = \(\frac{3}{5}\) and P(B) = \(\frac{1}{5}\), find P (A ∩ B) if A and B are independent events.
Solution:

It is given that P(A) = \(\frac{3}{5}\) and P(B) = \(\frac{1}{5}\)

A and B are independent events. Therefore, P(A ∩ B) = P(A)P(B) = \(\frac{3}{5}\) \(\frac{1}{5}\) = \(\frac{3}{25}\)

Question 2. Two cards are drawn at random and without replacement from a pack of 52 playing cards. Find the probability that both the cards are black.
Solution:

There are 26 black cards in a deck of 52 cards.

Let A: event that 1st card is black, B: event that 2nd card is black.

P(getting a black card in the first draw) = P(A) = \(\frac{26}{52}\) = \(\frac{1}{2}\)

P (getting a black card on the second draw) – P(B/A) = \(\frac{25}{51}\) (v card is not replaced)

Thus, P(getting both the cards black) = P(A ∩ B) = P(A).P(B/A) = \(\frac{1}{2}\) x \(\frac{25}{51}\) = \(\frac{25}{102}\)

Question 3. A box of oranges is inspected by examining three randomly selected oranges drawn without replacement. If all three oranges are good, the box is approved for sale, otherwise, it is rejected. Find the probability that a box containing 15 oranges out of which 12 are good and 3 are bad ones will be approved for sale.
Solution:

Let A, B, and C be the events respectively that the first, second, and third drawn orange is good.

Therefore, probability that lust drawn orange is good, P(A) = \(\frac{12}{15}\)

Since the second orange is drawn without replacement (now the total number of good oranges will be 11 and the total oranges will be 14

∴ The conditional probability of B, given that A has already occurred is P(B/A) ⇒ P(B/A) = \(\frac{11}{14}\)

Again, the third orange is drawn without replacement (now the total number of good oranges will be 10, and the total number of oranges will be 13).

∴ The conditional probability of C, given that A and B have already occurred is P(C/AB)

⇒ P(C/AB)= \(\frac{10}{13}\)

The box is approved for sale if all three oranges are good.

Thus, the probability of getting all the oranges good

= P(A ∩ B ∩ C) = P(A). P(B/A). P(C/AB) = \(\frac{12}{15}\) x \(\frac{11}{14}\) x \(\frac{10}{3}\) = \(\frac{44}{91}\)

Therefore, the probability that the box is approved for sale is \(\frac{44}{91}\)

Question 4. A fair coin and an unbiased die are tossed. Let A be the event ‘head appears on the coin’ and B be the event ‘3 on the die’. Check whether A and B are independent events or not.
Solution:

If a fair coin and an unbiased die are tossed, then the sample space S is given by,

S = \(\left\{\begin{array}{l}
(\mathrm{H}, 1),(\mathrm{H}, 2),(\mathrm{H}, 3),(\mathrm{H}, 4),(\mathrm{H}, 5),(\mathrm{H}, 6) \\
(\mathrm{T}, 1),(\mathrm{T}, 2),(\mathrm{T}, 3),(\mathrm{T}, 4),(\mathrm{T}, 5),(\mathrm{T}, 6)
\end{array}\right\} \Rightarrow \mathrm{n}(\mathrm{S})=12\)

Let A: Head appears on the coin

A = {(H, 1), (H, 2), (H, 3), (H, 4), (H, 5), (H, 6)}

⇒ P(A) = \(\frac{6}{12}\) = \(\frac{1}{2}\)

B: 3 on die = {(H, 3), (T, 3)}

P(B) = \(\frac{2}{12}\) = \(\frac{1}{6}\)

∴ A∩B = {(H, 3)}

P(A∩B) = \(\frac{1}{12}\)

Also, P(A)·P(B) =\(\frac{1}{2}\) x \(\frac{1}{6}\) =\(\frac{1}{12}\) = P(A∩B)

Therefore, A and B are independent events.

Question 5. A die marked 1, 2, 3 in red and 4, 5, 6 in green is tossed. Let A be the event, ‘the number is even,’ and B be the event, ‘the number is red’. Are A and B independent?
Solution:

When a die is thrown, the sample space (S) is S = {1, 2, 3, 4, 5, 6}

Let A: the number is even = {2, 4, 6} ⇒ P(A) = \(\frac{3}{6}\) = \(\frac{1}{2}\)

B: the number is red = {1,2, 3} ⇒ P(B) = \(\frac{3}{6}\) = \(\frac{1}{2}\)

∴ A∩B = {2} ⇒ P(A∩B) = \(\frac{1}{6}\)

Also, P(A)·P(B) = \(\frac{1}{2}\) x \(\frac{1}{2}\) = \(\frac{1}{4}\)≠\(\frac{1}{6}\)≠P(A∩B)

⇒ P(A) · P(B) ≠ P(A∩B)

Therefore, A and B are not independent.

Question 6. Let E and F be events withP(E) = \(\frac{3}{5}\), P(F) = \(\frac{3}{10}\) and P(E∩F) = \(\frac{1}{5}\). Are E and F independent?
Solution:

It is given that P(E) = \(\frac{3}{5}\), P(F) = \(\frac{3}{10}\) and P(E∩F) = \(\frac{1}{5}\)

Now, P(E) · P(F) = \(\frac{3}{5} \cdot \frac{3}{10}=\frac{9}{50} \neq \frac{1}{5} \Rightarrow \mathrm{P}(\mathrm{E} \cap \mathrm{F}) \Rightarrow \mathrm{P}(\mathrm{E}) \cdot \mathrm{P}(\mathrm{F}) \neq \mathrm{P}(\mathrm{E} \cap \mathrm{F})\)

Therefore, E and F are not independent.

Question 7. Given that the events A and B are such that P(A) = \(\frac{1}{2}\), P(A∪B) = \(\frac{3}{5}\) and P(B) = p. Find p if they are

  1. Mutually exclusive
  2. Independent

Solution:

It is given that P(A) = \(\frac{1}{2}\), P(A∪B) = \(\frac{3}{5}\) and P(B) = p

1. When A and B are mutually exclusive, A∩B = ø

∴ P(A∩B) = 0

It is known that, P(A∪B) = P(A) + P(B) – P(A∩B)

2. When A and B are independent, events then, P(A∩B) = P(A)·P(B) = \(\frac{1}{2}\) p

It is known that, P(A∩B) = P(A) + P(B) – P(A∩B)

⇒ \(\frac{3}{5}=\frac{1}{2}+\mathrm{p}-\frac{1}{2} \mathrm{p} \Rightarrow \frac{3}{5}=\frac{1}{2}+\frac{\mathrm{p}}{2}\)

⇒ \(\frac{\mathrm{p}}{2}=\frac{3}{5}-\frac{1}{2}=\frac{1}{10} \Rightarrow \mathrm{p}=\frac{2}{10}=\frac{1}{5}\)

Question 8. Let A and B be independent events with P(A) = 0.3 and P(B) = 0.4. Find

  1. (P(A∩B),
  2. P(A∪B),
  3. P(A|B),
  4. P(B|A)

Solution:

It is given that P(A) = 0.3 and P(B) = 0.4

  1. If A and B are independent events, then P(A∩B) = P(A)·P(B) = 0.3 x 0.4 = 0.12
  2. P(A∪B) = P(A) + P(B)-P(A∩B) ⇒ P(A∪B) = 0.3 + 0.4 – 0.12 = 0.58
  3. It is known that, P(A | B) = \(\frac{P(A \cap B)}{P(B)} \Rightarrow P(A \mid B)=\frac{0.12}{0.4}=0.3\)
  4. It is known that, P(B | A) = \(\frac{\mathrm{P}(\mathrm{B} \cap \mathrm{A})}{\mathrm{P}(\mathrm{A})} \Rightarrow \mathrm{P}(\mathrm{B} \mid \mathrm{A})=\frac{0.12}{0.3}=0.4\)

Question 9. If A and B are two events such that P(A) = \(\frac{1}{2}\), P(B) = \(\frac{1}{2}\) and P(A∩B) = \(\frac{1}{8}\), find P(not A and not B).
Solution:

It is given that P(A) = \(\frac{1}{4}\), P(B) = \(\frac{1}{2}\) and P(A∩B) = \(\frac{1}{8}\)

P(not A and not B) = P(A’∩B’)

P(not A and not B) = p((A∪B)’) [A’∩B’ = (A∪B)’]

= \(1-P(A \cup B)=1-[P(A)+P(B)-P(A \cap B)]\)

= \(1-\left[\frac{1}{4}+\frac{1}{2}-\frac{1}{8}\right]=1-\frac{5}{8}=\frac{3}{8}\)

Question 10. Events A and B are such that P(A) = \(\frac{1}{2}\), P(B) = \(\frac{7}{12}\) and P(not A or not B) = \(\frac{1}{4}\). State whether A and B are independent.
solution:

It is given that \(P(A)=\frac{1}{2}, P(B)=\frac{7}{12}\) and P(not A or not B)= \(\frac{1}{4}\).

⇒ \(\mathrm{P}\left(\mathrm{A}^{\prime} \cup \mathrm{B}^{\prime}\right)=\frac{1}{4}\)

⇒ \(\mathrm{P}\left((\mathrm{A} \cap \mathrm{B})^{\prime}\right)=\frac{1}{4}\)

(because \(\mathrm{A}^{\prime} \cup \mathrm{B}^{\prime}=(\mathrm{A} \cap \mathrm{B})^{\prime}\))

⇒ 1-\(\mathrm{P}(\mathrm{A} \cap \mathrm{B})=\frac{1}{4} \Rightarrow \mathrm{P}(\mathrm{A} \cap \mathrm{B})=\frac{3}{4}\)….(1)

However, \(\mathrm{P}(\mathrm{A}) \cdot \mathrm{P}(\mathrm{B})=\frac{1}{2} \cdot \frac{7}{12}=\frac{7}{24}\)

Here, \(\frac{3}{4} \neq \frac{7}{24}\)…..(2)

∴ \(P(A \cap B) \neq P(A) \cdot P(B)\)

Therefore, A and B are not independent events.

Question 11. Given two independent events A and B such that P (A) = 0.3, P (B) = 0.6. Find

  1. P(A and B)
  2. P(A and not B)
  3. P(A or B)
  4. P(neither A nor B)

Solution:

It is given that P (A) = 0.3 and P (B) = 0.6

Also, A and B are independent events.

  1. P(A and B) = P(A)·P(B) ⇒ P(A∩B) = 0.3 x 0.6 = 0.18
  2. P(A and not B) – P(A∩B’) = P(A) – P(A∩B) = 0.3 – 0.18 = 0.12
  3. P(A or B) = P(A∪B) = P(A) + P(B) – P(A∩B) = 0.3 + 0.6 – 0.18 = 0.72
  4. P(neither A nor B) = P(A’∩B’) = P((A∪B)’) = 1 – P(A∪B) = 1- 0.72 = 0.28

Question 12. A die is tossed thrice. Find the probability of getting an odd number at least once.
Solution:

Probability of getting an odd number in a single throw of a die = \(\frac{3}{6}\) = \(\frac{1}{2}\)

Similarly, probability of getting an even number = \(\frac{3}{6}\) = \(\frac{1}{2}\)

Probability of getting an even number three times = \(\frac{1}{2}\) x \(\frac{1}{2}\) x \(\frac{1}{2}\) = \(\frac{1}{8}\)

Therefore, the probability of getting an odd number at least once

= 1 – Probability of getting an odd number in none of the throws

= 1 – Probability of getting an even number thrice

= 1- \(\frac{1}{8}\) = \(\frac{7}{8}\)

Question 13. Two balls are drawn at random with replacement from a box containing 10 black and 8 red balls. Find the probability that

  1. Both balls are red.
  2. The first ball is black and the second is red.
  3. One of them is black and the other is red.

Solution:

Let R: event that drawn ball is red

B: event that drawn ball is black

∴ P(R) = \(\frac{8}{18}\) = \(\frac{4}{9}\) and P(B) = \(\frac{10}{18}\) = \(\frac{5}{9}\)

1. P(Both balls are red)

= \(P(R \cap R)=P(R) \cdot P(R)\) (Ball is replaced)

= \(\frac{4}{9} \cdot \frac{4}{9}=\frac{16}{81}\)

2. P(first ball is black and second is red) = P(B∩R)

= P(B). P(R) (Ball is replaced)

= \(\frac{5}{9} \cdot \frac{4}{9}=\frac{20}{81}\)

3. P(one of them is black and other red)

= P [(R∩B)∪(B∩R)] = P(R∩B) + P(B∩R)

= P(R). P(B) + P(B). P(R) (Ball is replaced)

= \(\frac{4}{9} \cdot \frac{5}{9}+\frac{5}{9} \cdot \frac{4}{9}=\frac{20}{81}+\frac{20}{81}=\frac{40}{81}\)

Question 14. The probability of solving specific problems independently by A and B are \(\frac{1}{2}\) and \(\frac{1}{3}\) respectively. If both try to solve the problem independently, find the probability that

  1. The problem is solved
  2. Exactly one of them solves the problem.

Solution:

Let E1: an event that A solves the problem

E2: an event that B solves the problem

Then \(P\left(E_1\right)=\frac{1}{2}\) and \(P\left(E_1\right)=\frac{1}{3} \Rightarrow P\left(\bar{E}_1\right)=1-\frac{1}{2}=\frac{1}{2}\) and \(P\left(\bar{E}_2\right)=1-\frac{1}{3}=\frac{2}{3}\)

Clearly, \(\mathrm{E}_1\) and \(\mathrm{E}_2\) are independent events :

1. P (the problem is solved)

= \(P(\text { at least one of } A \text { and } B \text { solves the problem })\)

= \(P\left(E_1 \cup E_2\right)=P\left(E_1\right)+P\left(E_2\right)-P\left(E_1 \cap E_2\right)=P\left(E_1\right)+P\left(E_2\right)-P\left(E_1\right), P\left(E_2\right)\)

= \(\frac{1}{2}+\frac{1}{3}-\frac{1}{2} \cdot \frac{1}{3}=\frac{2}{3}\)

2. P(exactly one of them solves the problem)

= \(P\left[\left(E_1 \cap \bar{E}_2\right) \cup\left(\bar{E}_1 \cap E_2\right)\right]\)

= \(P\left(E_1 \cap \bar{E}_2\right)+P\left(\bar{E}_1 \cap E_2\right)\)

= \(P\left(E_1\right) \cdot P\left(\bar{E}_2\right)+P\left(\bar{E}_1\right) \cdot P\left(E_2\right)\)

= \(\frac{1}{2} \times \frac{2}{3}+\frac{1}{3} \times \frac{1}{2}=\frac{1}{2}\)

Question 15. One card is drawn at random from a well-shuffled deck of 52 cards. In which of the following cases are the events E and F independent?

  1. E: ‘The card drawn is a spade’, F: ‘The card drawn is an ace’
  2. E: ‘The card drawn is black’, F: ‘The card drawn is a king’
  3. E: ‘The card drawn is a king or queen’, F: ‘The card drawn is a queen or jack’

Solution:

1. In a deck of 52 cards, 13 cards are spades and 4 cards are aces.

∴ P(E) = P(the card drawn is a spade) = \(\frac{13}{52}\) = \(\frac{1}{4}\)

∴ P(F) = P(the card drawn is an ace) = \(\frac{4}{52}\) = \(\frac{1}{3}\)

In the deck of cards, only 1 card is an ace of spades.

P(E∩F) = P(the card drawn is spade and an ace) = \(\frac{1}{52}\)

⇒ \(\mathrm{P}(\mathrm{E}) \cdot \mathrm{P}(\mathrm{F})=\frac{1}{4} \cdot \frac{1}{13}=\frac{1}{52}=\mathrm{P}(\mathrm{E} \cap \mathrm{F})\)

⇒ \(\mathrm{P}(\mathrm{E}) \cdot \mathrm{P}(\mathrm{F})=\mathrm{P}(\mathrm{E} \cap \mathrm{F})\)

Therefore, the events E and F are independent.

2. In a deck of 52 cards, 26 cards are black and 4 cards are kings.

∴ P(E) = P(the card drawn is black) = \(\frac{26}{52}\) = \(\frac{1}{2}\)

∴ P(F) = P(the card drawn is black) = \(\frac{4}{52}\) = \(\frac{1}{13}\)

In the pack of 52 cards, 2 cards are black as well as kings.

∴ P(E∩F) = P(the card drawn is a black king) = \(\frac{2}{52}\) = \(\frac{1}{26}\)

Also, \(P(E) \times P(F)=\frac{1}{2} \cdot \frac{1}{13}=\frac{1}{26}=P(E \cap F)\)

Therefore, the given events E and F are independent.

3. In a deck of 52 cards, 4 cards are kings, 4 cards are queens, and 4 cards are jacks.

∴ P(E) = P(the card drawn is a king or a queen) = \(\frac{8}{52}\) = \(\frac{2}{13}\)

∴ P(F) = P(the card drawn is a queen or a jack) = \(\frac{8}{52}\) = \(\frac{2}{13}\)

There are 4 cards which are either king or queen and either queen or jack.

∴P(E∩F) = P(the card drawn is either king or queen and either queen or jack) = \(\frac{4}{52}\) = \(\frac{1}{13}\)

Now, \(\mathrm{P}(\mathrm{E}) \times \mathrm{P}(\mathrm{F})=\frac{2}{13} \cdot \frac{2}{13}=\frac{4}{169} \neq \frac{1}{13}\)

⇒ P(E)·P(F) ≠ P(E∩ F)

Therefore, the given events E and F are not independent.

Question 16. In a hostel, 60% of the students read Hindi newspapers, 40% read English newspapers and 20% read both Hindi and English newspapers. A student is selected at random.

  1. Find the probability that she reads neither Hindi nor English newspapers.
  2. If she reads a Hindi newspaper, find the probability that she reads an English newspaper.
  3. If she reads an English newspaper, find the probability that she reads a Hindi newspaper.

Solution:

Let H denote the students who read Hindi newspapers and E denote the students who read English newspapers.

It is given that, P(H)=60 \(\%=\frac{6}{10}=\frac{3}{5} ; \mathrm{P}(\mathrm{E})=40 \%=\frac{40}{100}=\frac{2}{5} ; \mathrm{P}(\mathrm{H} \cap \mathrm{E})=20 \%=\frac{20}{100}=\frac{1}{5}\)

1. The probability that a student reads neither Hindi nor English newspapers is,

⇒ \(\mathrm{P}_{(}\left(\mathrm{H}^{\prime} \cap \mathrm{E}^{\prime}\right)=\mathrm{P}\left(\mathrm{H} \cup \mathrm{E}^{\prime}\right.=1-\mathrm{P}(\mathrm{H} \cup \mathrm{E})=1-\{\mathrm{P}(\mathrm{H})+\mathrm{P}(\mathrm{E})-\mathrm{P}(\mathrm{H} \cap \mathrm{E})\}\)

= \(1-\left(\frac{3}{5}+\frac{2}{5}-\frac{1}{5}\right)=1-\frac{4}{5}=\frac{1}{5}\)

2. The probability that a randomly chosen student reads an English newspaper if she reads a Hindi newspaper, is given by \(\mathrm{P}(\mathrm{E} \mid \mathrm{H})\).

P(E | H) = \(\frac{P(E \cap H)}{P(H)}=\frac{\frac{1}{5}}{\frac{3}{5}}=\frac{1}{3}\)

3. The probability that a randomly chosen student reads a Hindi newspaper if she reads an English newspaper, is given by \(\mathrm{P}(\mathrm{H} \mid \mathrm{E})\).

P(H | E) = \(\frac{P(H \cap E)}{P(E)}=\frac{\frac{1}{5}}{\frac{2}{5}}=\frac{1}{2}\)

Choose The Correct Answer

Question 17. The probability of obtaining an even prime number on each die. when a pair of dice is rolled is

  1. 0
  2. 1/3
  3. 1/12
  4. 1/36

Solution: 4. 1/36

When two dice are rolled, the number of outcomes is 36. i.e. ⇒ n(S) = 36

The only even prime number is 2.

i.e. ⇒ n(E) = 1 Let E be the event of getting an even prime number on each die.

∴ E = {(2, 2)} ⇒ P(E) = 1/36

Therefore, the correct Answer is 4.

Question 18. Two events A and B will be independent, if

  1. A and B are mutually exclusive
  2. P(A)=P(B)
  3. P(A’B’) = [1 – P(A)] [1 – P(B)]
  4. P(A) = P(B) (D) P(A) + P(B) – 1

Solution: 2. P(A)=P(B)

Two events A and B are said to be independent if P(A∩B) = P(A) x P(B)

Consider the result given in alternative B.

P(A’B’) = [1 – P(A)] [1 – P(B)]

⇒ P(A’∩B’) = l-P(A)-P(B) + P(A)P(B)

⇒ 1-P(A∪B)=l- P(A) – P(B) + P(A)P(B)

⇒ P(A∪B) = P(A) + P(B)-P(A)P(B)

⇒ P(A) + P(B) – P(AB) = P(A) + P(B) – P(A)P(B)

⇒ P(AB) = P(A)P(B)

This implies that A and B are independent, if P(A’B’) = [1 – P(A)] [1 – P(B)]

Probability Exercise 13.3

Question 1. An urn contains 5 red and 5 black balls. A ball is drawn at random, its color is noted and is returned to the urn. Moreover, 2 additional balls of the color drawn are put in the urn and then a ball is drawn at random. What is the probability that the second ball is red?
Solution:

The urn contains 5 red and 5 black balls.

Let a red ball be drawn in the first attempt.

∴ P (drawing a red ball) = \(\frac{5}{10}\) = \(\frac{1}{2}\)

If two red balls are added to the urn, then the urn contains 7 red and 5 black balls.

P(drawing a red ball) = \(\frac{7}{12}\)

Let a black ball be drawn in the first attempt.

∴ P (drawing a black ball in the first attempt) = \(\frac{5}{10}\) = \(\frac{1}{2}\)

If two black balls are added to the urn, then the urn contains 5 red and 7 black balls.

P(drawing a red all) = \(\frac{5}{12}\)

Therefore, probability of drawing second ball as red is = \(\frac{1}{2} \times \frac{7}{12}+\frac{1}{2} \times \frac{5}{12}=\frac{1}{2}\left(\frac{7}{12}+\frac{5}{12}\right)=\frac{1}{2} \times 1=\frac{1}{2}\)

Question 2. A bag contains 4 red and 4 black balls, another bag contains 2 red and 6 black balls. One of the two bags is selected at random and a ball is drawn from the bag which is found to be red. Find the probability that the ball is drawn from the first bag.
Solution:

Let E1 and E2 be the events of selecting the first bag and second bag respectively.

∴ \(\mathrm{P}\left(\mathrm{E}_1\right)=\mathrm{P}\left(\mathrm{E}_2\right)=\frac{1}{2}\)

Let A be the event of getting a red bail.

⇒ P(A | E1) = P(drawing a red ball from first bag) = \(\frac{4}{8}\) = \(\frac{1}{2}\)

⇒ P(A | E2) = P(drawing a red ball from second bag) = \(\frac{2}{8}\) = \(\frac{1}{4}\)

The probability of drawing a ball from the first bag, given that it is red, is given by P (E1/A). By using Bayes’ theorem, we obtain

⇒ \(P\left(E_1 \mid A\right)=\frac{P\left(E_1\right) \cdot P\left(A \mid E_1\right)}{P\left(E_1\right) \cdot P\left(A \mid E_1\right)+P\left(E_2\right) \cdot P\left(A \mid E_2\right)}\)

= \(\frac{\frac{1}{2} \cdot \frac{1}{2}}{\frac{1}{2} \cdot \frac{1}{2}+\frac{1}{2} \cdot \frac{1}{4}}=\frac{\frac{1}{4}}{\frac{1}{4}+\frac{1}{8}}=\frac{\frac{1}{4}}{\frac{3}{8}}=\frac{2}{3}\)

Question 3. Of the students in a college, it is known that 60% reside in a hostel and 40% are day scholars (not residing in the hostel). Previous year results report that 30% of all students who reside in hostel attain A grades and 20% of day scholars attain A grades in their annual examination. At the end of the year, one student is chosen at random from the college and he has an A grade, what is the probability that the student is hostlier?
Solution:

Let E1 and E2 be the events in which the student is a hostlier and a day scholar respectively and A be the event in which the chosen student gets a grade A.

Then, E1 and E2 are naturally exclusive and exhaustive.

∴ \(\mathrm{P}\left(\mathrm{E}_1\right)=60 \%=\frac{60}{100}=0.6 ; \quad \mathrm{P}\left(\mathrm{E}_2\right)=40 \%=\frac{40}{100}=0.4\)

P(A|E1) = P(student getting an A grade is a costlier) = 30% = 0.3

P(A|E2) = P(student getting an A grade is a day scholar) = 20% = 0.2

The probability that a randomly chosen student is a hostlier, given that he has an A grade, is given by P(E1|A).

By using Bayes’ theorem, we obtained

⇒ \(P\left(E_1 \mid A\right)=\frac{P\left(E_1\right) \cdot P\left(A \mid E_1\right)}{P\left(E_1\right) \cdot P\left(A \mid E_1\right)+P\left(E_2\right) \cdot P\left(A \mid E_2\right)}=\frac{0.6 \times 0.3}{0.6 \times 0.3+0.4 \times 0.2}=\frac{0.18}{0.26}=\frac{18}{26}=\frac{9}{13}\)

Question 4. In answering a question on a multiple-choice test, a student either knows the answer or guesses. Let \(\frac{3}{4}\) be the probability that he knows the answer and \(\frac{1}{4}\)  be the probability that he guesses.

Assuming that a student who guesses the answer will be correct with probability \(\frac{1}{4}\), What is the probability that the student knows the answer given that he answered it correctly?

Solution:

Let E1 and E2 be the events which the student knows the answer and guesses the answer respectively.

Let A be the event that the answer is correct.

∴ \(\mathrm{P}\left(\mathrm{E}_1\right)=\frac{3}{4} ; \quad \mathrm{P}\left(\mathrm{E}_2\right)=\frac{1}{4}\)

The probability that the student answered correctly, given that he knows the answer, is 1.

∴ \(\mathrm{P}\left(\mathrm{A} \mid \mathrm{E}_1\right)=1\)

The probability that the student answered correctly, given that he guessed, is

∴ \(\mathrm{P}\left(\mathrm{A} \mid \mathrm{E}_2\right)=1\)

The probability that the student knows the answer, given that he answered it correctly, is given by P(E1|A).

By using Bayes’ theorem, we obtain

⇒ \(P\left(E_1 \mid A\right)=\frac{P\left(E_1\right) \cdot P\left(A \mid E_1\right)}{P\left(E_1\right) \cdot P\left(A \mid E_1\right)+P\left(E_2\right) \cdot P\left(A \mid E_2\right)}=\frac{\frac{3}{4} \cdot 1}{\frac{3}{4} \cdot 1+\frac{1}{4} \cdot \frac{1}{4}}=\frac{\frac{3}{4}}{\frac{3}{4}+\frac{1}{16}}=\frac{\frac{3}{4}}{\frac{13}{16}}=\frac{12}{13}\)

Question 5. A laboratory blood test is 99% effective in detecting a certain disease when it is in fact, present. However, the test also yields a false positive result for 0.5% of the healthy person tested (i.e., if a healthy person is tested, then, with a probability of 0.005, the test will imply he has the disease). If 0.1 percent of the population actually has the disease, what is the probability that a person has the disease given that his test result is positive?
Solution:

Let E1 and E2 be the events that a person has a disease and a person is healthy respectively.

Since E1 and E2 are pairwise disjoint and exhaustive events.

P(E1) + P(E2) = 1 ⇒ P(E2) = 1 – P(E1) = 1 – 0.001 = 0.999 [P(E1) = 0.1% = 0.001]

Let A be the event that the blood test result is positive.

P(A|E1) = P(result is positive given the person has disease) = 99% = 0.99

P(A|E2) = Pfresult is positive given that the person is healthy) = 0.5% = 0.005

The probability that a person has a disease, given that his test result is positive, is given by P (E1|A).

By using Bayes’ theorem, we obtain

⇒ \(\mathrm{P}\left(\mathrm{E}_1 \mid \mathrm{A}\right) =\frac{\mathrm{P}\left(\mathrm{E}_1\right) \cdot \mathrm{P}\left(\mathrm{A} \mid \mathrm{E}_1\right)}{\mathrm{P}\left(\mathrm{E}_1\right) \cdot \mathrm{P}\left(\mathrm{A} \mid \mathrm{E}_1\right)+\mathrm{P}\left(\mathrm{E}_2\right) \cdot \mathrm{P}\left(\mathrm{A} \mid \mathrm{E}_2\right)}\)

= \(\frac{0.001 \times 0.99}{0.001 \times 0.99+0.999 \times 0.005}=\frac{0.00099}{0.00099+0.004995}\)

= \(\frac{0.00099}{0.005985}=\frac{990}{5985}=\frac{110}{665}=\frac{22}{133}\)

Question 6. There are three coins. One is a two-headed coin (having heads on both faces), another is a biased coin that comes up heads 75% of the time and the third is an unbiased coin. One of the three coins is chosen at random and tossed, it shows heads, what is the probability that it was the two-headed coin?
Solution:

Let E1, E2,  and E3 be the respective events of choosing a two-headed coin, a biased coin, and an unbiased coin.

Then, E1, E2, and E3 are mutually exclusive and exhaustive events.

∴ P(E1) = P(E2) = P(E3) = 1/3

Let A be the event that the coin shows heads.

A two-headed coin will always show heads.

∴ P(A|E1) = P(coin showing heads, given that it is a two-headed coin) = 1

Probability of heads coming up, given that it is a biased coin= 75%

∴ P(A|E2) = P(coin showing heads, given that it is a biased coin) = \(\frac{75}{1000}\) = \(\frac{3}{4}\)

Since the third coin is unbiased, the probability that it shows heads is always  \(\frac{1}{2}\)

∴ P(A|E3) = P(coin showing heads, given that it is a biased coin) = \(\frac{1}{2}\)

The probability that the coin is two-headed. given that it shows heads, is given by P (E1lA)

By using Bayes’ theorem, we obtain

⇒ \(P\left(E_1 \mid A\right)=\frac{P\left(E_1\right) \cdot P\left(A \mid E_1\right)}{P\left(E_1\right) \cdot P\left(A \mid E_1\right)+P\left(E_2\right) \cdot P\left(A \mid E_2\right)+P\left(E_3\right) \cdot P\left(A \mid E_3\right)}\)

= \(\frac{\frac{1}{3} \cdot 1}{\frac{1}{3} \cdot 1+\frac{1}{3} \cdot \frac{3}{4}+\frac{1}{3} \cdot \frac{1}{2}}=\frac{\frac{1}{3}}{\frac{1}{3}\left(1+\frac{3}{4}+\frac{1}{2}\right)}=\frac{1}{\frac{9}{4}}=\frac{4}{9}\)

Question 7. An insurance company insured 2000 scooter drivers, 4000 car drivers, and 6000 truck drivers. The probability of accidents is 0.01,0.03 and 0.15 respectively. One of the insured persons meets with an accident. What is the probability that he is a scooter driver?
Solution:

Let E1, E2, and E3 be the events in which the driver is a scooter driver, a car driver, and a truck driver respectively.

Let A be the event that the person meets with an accident.

There are 2000 scooter drivers, 4000 car drivers, and 6000 truck drivers.

Total number of drivers = 2000 + 4000 + 6000 = 12000

P\(\left(E_1\right)\)=P(driver is a scooter driver)=\(\frac{2000}{12000}=\frac{1}{6}\)

P\(\left(\mathrm{E}_2\right)=\mathrm{P}\)(driver is a car driver) = \(\frac{4000}{12000}=\frac{1}{3}\)

P\(\left(\mathrm{E}_3\right)=\mathrm{P}\)(driver is a truck driver )=\(\frac{6000}{12000}=\frac{1}{2}\)

P\(\left(\mathrm{A} \mid \mathrm{E}_1\right)=\mathrm{P}\)(scooter driver met with an accident) =0.01 = \(\frac{1}{100}\)

P\(\left(\mathrm{A} \mid \mathrm{E}_2\right)=\mathrm{P}\)(car driver met with an accident)[/latex] = \(0.03=\frac{3}{100}\)

P\(\left(\mathrm{A} \mid \mathrm{E}_3\right)=\mathrm{P}\)(truck driver met with an accident) = \(0.15=\frac{15}{100}\)

The probability that the driver is a scooter driver, given that he met with an accident, is given by \(\mathrm{P}\left(\mathrm{E}_1 \mid \mathrm{A}\right)\).

By using Bayes’ theorem, we obtain

⇒ \(P\left(E_1 \mid A\right)=\frac{P\left(E_1\right) \cdot P\left(A \mid E_1\right)}{P\left(E_1\right) \cdot P\left(A \mid E_1\right)+P\left(E_2\right) \cdot P\left(A \mid E_2\right)+P\left(E_3\right) \cdot P\left(A \mid E_3\right)}\)

= \(\frac{\frac{1}{6} \cdot \frac{1}{100}}{\frac{1}{6} \cdot \frac{1}{100}+\frac{1}{3} \cdot \frac{3}{100}+\frac{1}{2} \cdot \frac{15}{100}}=\frac{\frac{1}{6} \cdot \frac{1}{100}}{\frac{1}{100}\left(\frac{1}{6}+1+\frac{15}{2}\right)}=\frac{\frac{1}{6}}{\frac{104}{12}}=\frac{1}{6} \times \frac{12}{104}=\frac{1}{52}\)

Question 8. A factory has two machines A and B. Past records show that machine A produced 60% of the items of output and machine B produced 40% of the items. Further, 2% of the items produced by machine A and 1% produced by machine B were defective. All the items are put into one stockpile and then one item is chosen at random from this and is found to be defective. What is the probability that was produced by machine B?
Solution:

Let E1 and E2 be the events of items produced by machines A and B respectively. Let X be the event that the produced item was found to be defective.

∴ Probability of items produced by machine A, P (E1) = 60% = \(\frac{3}{4}\)

Probability of items produced by machine B, P (E2) = 40% = \(\frac{2}{5}\)

Probability that machine A produced defective items, P(X|E1)= 2% = \(\frac{2}{100}\)

Probability that machine B produced defective items, P(X|E2) = 1% = \(\frac{1}{100}\)

The probability that the randomly selected item was from machine B, given that it is defective, is given by P (E2|X).

By using Bayes’ theorem, we obtain

⇒ \(P\left(E_2 \mid X\right)=\frac{P\left(E_2\right) \cdot P\left(X \mid E_2\right)}{P\left(E_1\right) \cdot P\left(X \mid E_1\right)+P\left(E_2\right) \cdot P\left(X \mid E_2\right)}=\frac{\frac{2}{5} \cdot \frac{1}{100}}{\frac{3}{5} \cdot \frac{2}{100}+\frac{2}{5} \cdot \frac{1}{100}}=\frac{\frac{2}{500}}{\frac{6}{500}+\frac{2}{500}}=\frac{2}{8}=\frac{1}{4}\)

Question 9. Two groups are competing for the position on the board of directors of a corporation. The probabilities that the first and the second groups will win are 0.6 and 0.4 respectively. Further, if the first group wins, the probability of introducing a new product is 0.7 and the corresponding probability is 0.3 if the second group wins. Find the probability that the new product introduced was by the second group.
Solution:

Let E1 and E2 be the events that the first group and the second group win the competition respectively. Let A be the event of introducing a new product.

P (E1) = Probability that the first group wins the competition = 0.6

P (E2) = Probability that the second group wins the competition = 0.4

P (A|E1) = Probability of introducing a new product if the first group wins = 0.7

P (A|E2) = Probability of introducing a new product if the second group wins = 0.3

The probability that the new product is introduced by the second group is given by P (E2|A).

By using Bayes’ theorem, we obtain

⇒ \(P\left(E_2 \mid A\right)=\frac{P\left(E_2\right) \cdot P\left(A \mid E_2\right)}{P\left(E_1\right) \cdot P\left(A \mid E_1\right)+P\left(E_2\right) \cdot P\left(A \mid E_2\right)}=\frac{0.4 \times 0.3}{0.6 \times 0.7+0.4 \times 0.3}=\frac{0.12}{0.42+0.12}=\frac{0.12}{0.54}=\frac{12}{54}=\frac{2}{9}\)

Question 10. Suppose a girl throws a die. If she gets a 5 or 6, she tosses a coin three times and notes the number of heads. If she gets 1, 2, 3, or 4, she tosses a coin once and notes whether a head or tail is obtained. If she obtained exactly one head, what is the probability that she threw 1, 2, 3, or 4 with the die?
Solution:

Let E1 be the event that the outcome on the die is 5 or 6 and E2 be the event that the outcome on the die is 1,2, 3, or 4.

∴ \(P\left(E_1\right)=\frac{2}{6}=\frac{1}{3} \text { and }\left(E_2\right)=\frac{4}{6}=\frac{2}{3}\)

Let A be the event of getting exactly one head.

P(A|E1) = Probability of getting exactly one head by tossing the coin three times if she gets 5 or 6 = \(\frac{3}{8}\)

P(A|E2) = Probability of getting exactly one head in a single throw of the coin if she gets 1, 2, 3, or 4 = \(\frac{1}{2}\)

The probability that the girl took 1,2, 3, or 4 with the die, if she obtained exactly one head, is given by P(E2|A). By using Bayes’ theorem, we obtain

∴ \(P\left(E_2 \mid A\right)=\frac{P\left(E_2\right) \cdot P\left(A \mid E_2\right)}{P\left(E_1\right) \cdot P\left(A \mid E_1\right)+P\left(E_2\right) \cdot P\left(A \mid E_2\right)}=\frac{\frac{2}{3} \cdot \frac{1}{2}}{\frac{1}{3} \cdot \frac{3}{8}+\frac{2}{3} \cdot \frac{1}{2}}=\frac{\frac{1}{3}}{\frac{1}{3}\left(\frac{3}{8}+1\right)}=\frac{1}{\frac{11}{8}}=\frac{8}{11}\)

Question 11. A manufacturer has three machine operators A, B, and C. The first operator A produces 1% defective items, whereas the other two operators B and C produce 5% and 7% defective items respectively. A is on the job for 50% of the time, B is on the job for 30% of the time and C is on the job for 20% of the time. A defective item is produced, what is the probability that was produced by A?
Solution:

Let E1, E2, and E3 be the events of the time consumed by machine operators A, B, and C for the job respectively.

⇒ \(\mathrm{P}\left(\mathrm{E}_1\right)=50 \%=\frac{50}{100}=\frac{1}{2} ; \mathrm{P}\left(\mathrm{E}_2\right)=30 \%=\frac{30}{100}=\frac{3}{10} ; \mathrm{P}\left(\mathrm{E}_3\right)=20 \%=\frac{20}{100}=\frac{1}{5}\)

Let X be the event of producing defective items.

⇒ \(\mathrm{P}\left(\mathrm{X} \mid \mathrm{E}_1\right)=1 \%=\frac{1}{100} ; \mathrm{P}\left(\mathrm{X} \mid \mathrm{E}_2\right)=5 \%=\frac{5}{100} ; \mathrm{P}\left(\mathrm{X} \mid \mathrm{E}_3\right)=7 \%=\frac{7}{100}\)

The probability that the defective item was produced by A is given by \(\mathrm{P}(\mathrm{E} \mid \mathrm{A})\).

By using Bayes’ theorem, we obtain

⇒ \(P\left(E_1 \mid X\right)=\frac{P\left(E_1\right) \cdot P\left(X \mid E_1\right)}{P\left(E_1\right) \cdot P\left(X \mid E_1\right)+P\left(E_2\right) \cdot P\left(X \mid E_2\right)+P\left(E_3\right) \cdot P\left(X \mid E_3\right)}\)

= \(\frac{\frac{1}{2} \cdot \frac{1}{100}}{\frac{1}{2} \cdot \frac{1}{100}+\frac{3}{10} \cdot \frac{5}{100}+\frac{1}{5} \cdot \frac{7}{100}}=\frac{\frac{1}{100} \cdot \frac{1}{2}}{\frac{1}{100}\left(\frac{1}{2}+\frac{3}{2}+\frac{7}{5}\right)}=\frac{\frac{1}{17}}{\frac{17}{5}}=\frac{5}{34}\)

Question 12. A card from a pack of 52 cards is lost. From the remaining cards of the pack, two cards are drawn and are found to be both diamonds. Find the probability of the lost card being a diamond.
Solution:

Let E1 and E2 be the events that the lost card is a diamond card and a card that is not a diamond respectively.

Let A be the event that two diamond cards are drawn.

Out of 52 cards, 13 cards are diamond and 39 cards are not diamond.

∴ \(\mathrm{P}\left(\mathrm{E}_1\right)=\frac{13}{52}=\frac{1}{4} ; \quad \mathrm{P}\left(\mathrm{E}_2\right)=\frac{39}{52}=\frac{3}{4}\)

When one diamond card is lost, there are 12 diamond cards out of 51 cards.

Two diamond cards can be drawn out of 12 diamond cards in 12C2 ways and 2 cards can be drawn out of 51 cards in 51C2, ways. The probability of getting two diamond cards, when one diamond card is lost, is given by P (A|E1).

⇒ \(\mathrm{P}\left(\mathrm{A} \mid \mathrm{E}_1\right)=\frac{{ }^{12} \mathrm{C}_2}{{ }^{51} \mathrm{C}_2}=\frac{12 !}{2 ! \times 10 !} \times \frac{2 ! \times 49 !}{51 !}=\frac{11 \times 12}{50 \times 51}=\frac{22}{425}\)

When the lost card is not a diamond, there are 13 diamond cards out of 51 cards.

Two diamond cards can be drawn out of 13 diamond cards in 13C2 ways whereas 2 cards can be drawn out of 51 cards in 51C2, ways.

The probability of getting two diamond cards, when one card is lost which is not a diamond, is given by P (A|E2).

∴ \(P\left(A \mid E_2\right)=\frac{{ }^{13} C_2}{{ }^{51} C_2}=\frac{13 !}{2 ! \times 11 !} \times \frac{2 ! \times 49 !}{51 !}=\frac{12 \times 13}{50 \times 51}=\frac{26}{425}\)

The probability of getting two diamond cards, when one card is lost which is not a diamond, is given by P (A|E2)

∴ \(P\left(A \mid E_2\right)=\frac{{ }^{13} C_2}{{ }^{51} C_2}=\frac{13 !}{2 ! \times 11 !} \times \frac{2 ! \times 49 !}{51 !}=\frac{12 \times 13}{50 \times 51}=\frac{26}{425}\)

The probability that the lost card is a diamond if drawn cards are found to be both diamonds is given by P(E1lA),

By using Bayes’ theorem, we obtain \(P\left(E_1 \mid A\right)=\frac{P\left(E_1\right) \cdot P\left(A \mid E_1\right)}{P\left(E_1\right) \cdot P\left(A \mid E_1\right)+P\left(E_2\right) \cdot P\left(A \mid E_2\right)}\)

= \(\frac{\frac{1}{4} \cdot \frac{22}{425}}{\frac{1}{4} \cdot \frac{22}{425}+\frac{3}{4} \cdot \frac{26}{425}}=\frac{\frac{1}{425}\left(\frac{22}{4}\right)}{\frac{1}{425}\left(\frac{22}{4}+\frac{26 \times 3}{4}\right)}\)

= \(\frac{\frac{11}{25}}{50}\)

Question 13. The probability that A speaks truth is \(\frac{4}{5}\). A coin is tossed. Reports that a head appears. The probability that actually there was a head is

  1. \(\frac{4}{5}\)
  2. \(\frac{1}{2}\)
  3. \(\frac{1}{5}\)
  4. \(\frac{2}{5}\)

Solution: 1. \(\frac{4}{5}\)

Let E1 and E2 be the events such that

E1: A speaks truth, E2: A speaks lie

Let X be the event that a head appears.

⇒ \(P\left(E_1\right)=\frac{4}{5}\)

∴ \(P\left(E_2\right)=1-P\left(E_1\right)=1-\frac{4}{5}=\frac{1}{5}\)

If a coin is tossed, then it may result in either head (H) or tail (T).

The probability of getting a head is \(\frac{1}{2}\) and the probability of not getting a head is also \(\frac{1}{2}\)

∴ \(P\left(X \mid E_1\right)=P\left(X \mid E_2\right)=\frac{1}{2}\)

The probability that there is actually a head is given by \(\mathrm{P}\left(\mathrm{E}_{\mid} \mathrm{X}\right)\).

⇒ \(P\left(E_1 \mid X\right)=\frac{P\left(E_1\right) \cdot P\left(X \mid E_1\right)}{P\left(E_1\right) \cdot P\left(X \mid E_1\right)+P\left(E_2\right) \cdot P\left(X \mid E_2\right)}=\frac{\frac{4}{5} \cdot \frac{1}{2}}{\frac{4}{5} \cdot \frac{1}{2}+\frac{1}{5}+\frac{1}{2}}=\frac{\frac{1}{2} \cdot \frac{4}{5}}{\frac{1}{2}\left(\frac{4}{5}+\frac{1}{5}\right)}=\frac{\frac{4}{5}}{1}=\frac{4}{5}\)

Therefore, the correct answer is 1.

Question 14. If A and B are two events such that \(A \subset B\) and \(P(B) \neq 0\), then which of the following is correct?

  1. \(P(A \mid B)=\frac{P(B)}{P(A)}\)
  2. \(\mathrm{P}(\mathrm{A} \mid \mathrm{B})<\mathrm{P}(\mathrm{A})\)
  3. \(\mathrm{P}(\mathrm{A} \mid \mathrm{B}) \geq \mathrm{P}(\mathrm{A})\)
  4. None of these

Solution: 3. \(\mathrm{P}(\mathrm{A} \mid \mathrm{B}) \geq \mathrm{P}(\mathrm{A})\)

If \(A \subset B\), then \(A \cap B=A\)

⇒ \(\mathrm{P}(\mathrm{A} \cap \mathrm{B})=\mathrm{P}(\mathrm{A})\)

Also, \(\mathrm{P}(\mathrm{A})<\mathrm{P}(\mathrm{B})\)

Consider \(\mathrm{P}(\mathrm{A} \mid \mathrm{B})=\frac{\mathrm{P}(\mathrm{A} \cap \mathrm{B})}{\mathrm{P}(\mathrm{B})}=\frac{\mathrm{P}(\mathrm{A})}{\mathrm{P}(\mathrm{B})} \neq \frac{\mathrm{P}(\mathrm{B})}{\mathrm{P}(\mathrm{A})}\)…..(1)

Again, consider \(P(A \mid B)=\frac{P(A \cap B)}{P(B)}=\frac{P(A)}{P(B)}\)….(2)

It is known that, \(\mathrm{P}(\mathrm{B}) \leq 1 \Rightarrow \frac{1}{\mathrm{P}(\mathrm{B})} \geq 1 \Rightarrow \frac{\mathrm{P}(\mathrm{A})}{\mathrm{P}(\mathrm{B})} \geq \mathrm{P}(\mathrm{A})\)

From (2), we obtain

∴ \(\mathrm{P}(\mathrm{A} \mid \mathrm{B}) \geq \mathrm{P}(\mathrm{A})\)…….(3)

∴ \(P(A \mid B)\) is not less than P(A).

Thus, from (3), it can be concluded that the relation given in alternative 3 is correct.

Probability Exercise 13.4

Random Variable And Its Probability Distribution, Mean Of Random Variable

Question 1. Slate which of the following are not the probability distributions of a random variable? Give reasons for your answer:

Probability Probability Distribution Of A Random Variable

Solution:

It is known that the sum of all the probabilities in a probability distribution is one.

1. Sum of the probabilities = 0.4 + 0.4 + 0.2 = 1

Therefore, the given table is a probability distribution of random variables.

2. It can be seen that for X = 3, P (X) = -0.1

It is known that the probability of any observation is not negative. Therefore, the given table is not a probability distribution of random variables.

3. Sum of the probabilities .= 0,6 + 0.1 + 0.2 = 0.9 ≠ 1

Therefore, the given table is not a probability distribution of random variables,

4. Sum of the probabilities = 0.3 + 0,2 + 0.4 + 0.1 + 0.05 = 1.05 ≠ 1

Therefore, the given table is not a probability distribution of random variables.

Question 2. An urn contains 5 red and 2 black balls. Two balls are randomly drawn. Let X represent the number of black balls. What are the possible values of X? Is X a random variable?
Solution:

The two balls selected can be represented as BB, BR, RB, and RR, where B represents a black ball and R represents a red ball.

X represents the number of black balls.

∴ X(BB) = 2; X (BR) = 1; X (RB) = 1; X (RR) = 0

Therefore, the possible values of X are 0, 1, or 2.

Yes, X is a random variable.

Question 3. Let X represent the difference between the number of heads and the number of tails obtained when a coin is tossed 6 times. What are possible values of X?
Solution:

A coin is tossed six times and X represents the difference between the number of heads and the number of tails.

∴ X (6H, 0T) = |6-0| = 6; X (5H, IT) = |5 – 1| = 4

X (4H, 2T) = |4 – 2| = 2; X (3H, 3T) = |3 – 3| = 0

X (2H, 4T) = |2 – 4| = 2; X (1H, 5T) = |1 – 5| = 4

X (0H, 6T) = |0 – 6| = 6

Thus, the possible values of X are 6,4, 2 or 0.

Question 4. Find the probability distribution of

  1. Number of heads in two tosses of a coin
  2. Number of tails in the simultaneous tosses of three coins
  3. Number of heads in four tosses of a coin

Solution:

1. When one coin is tossed twice, the sample space is {HH, HT, TH, TT}

Let X represent the number of heads.

∴ X (HH) = 2, X (HT) = 1, X (TH) = 1, X (TT) = 0

Therefore, X can take the value of 0, 1, or 2.

It is known that,

P(HH) = P(HT) = P(TH) = P(TT} = \(\frac{1}{4}\)

P(X = 0) = P(TT) = \(\frac{1}{4}\)

P(X = 1) = P (HT) + P(TH) = \(\frac{1}{4}\) + \(\frac{1}{4}\) = \(\frac{1}{2}\)

P(X = 2) = P (HH) = \(\frac{1}{4}\)

Thus, the required probability distribution is as follows,

Probability Probability Distribution Of A value 0 1 Or 2

2. When three coins are tossed simultaneously, the sample space is {HHH, HHT, HTH HTT, THH, THT, TTH, TTT}

Let X represent the number of tails.

It can be seen that X can take the value of 0, 1, 2, or 3.

P (X = 0) = P (HHH) = \(\frac{1}{8}\)

P (X = 1) = P (HHT) + P (HTH) + P (THH) = \(\frac{1}{8}\) + \(\frac{1}{8}\) + \(\frac{1}{8}\) = \(\frac{3}{8}\)

P (X = 2) = P (HTT) + P (THT) + P (TTH) = \(\frac{1}{8}\) + \(\frac{1}{8}\) + \(\frac{1}{8}\) = \(\frac{3}{8}\)

P (X = 3) = P (TTT) = \(\frac{1}{8}\)

Thus, the probability distribution is as follows.

Probability Number Of Heads In Three Tosses Of A Fair Coin

3. When a coin is tossed four times, the sample space is

S = {HHHH, HHHT, HHTH, HHTT, HTHT, HTHH, HTTH, HTTT, THHH, THHT, THTH, THTT, TTHH, TTHT, TTTH, TTTT}

Let X be the random variable, which represents the number of heads.

It can be seen that X can take the value of 0,1,2, 3, or 4,

P(X = 0) = P(TTTT) = \(\frac{1}{16}\)

p (X = 1) = p (TTTH) + p (TTHT) + P (THTT) + P (HTTT) = \(\frac{1}{16}\) + \(\frac{1}{16}\) + \(\frac{1}{16}\) + \(\frac{1}{16}\) = \(\frac{1}{4}\)

P(X = 2) = P (HHTT) + P (THHT) + P (TTHH) + p (IITTH) + p (HTHT) + P (THTH)

= \(\frac{1}{16}\) + \(\frac{1}{16}\) + \(\frac{1}{16}\) + \(\frac{1}{16}\) + \(\frac{1}{16}\) = \(\frac{1}{16}\) = \(\frac{6}{16}\) = \(\frac{3}{8}\)

P (X = 3) = P (HHHT) + P (HHTH) + P (HTHH) P (THHH)

= \(\frac{1}{16}\) + \(\frac{1}{16}\) + \(\frac{1}{16}\) = \(\frac{4}{16}\) = \(\frac{1}{4}\)

P (X = 4) = P(HHHH)’ = \(\frac{1}{16}\)

Thus, the probability distribution is as follows.

Probability Probability Distribution Of A value 0 1 2 3 Or 4

Question 5. Find the probability distribution of the number of successes in two tosses of a die, where success is defined as

  1. Number greater than 4
  2. Six appear on at least one die

Solution:

When a die is tossed two times, we obtain (6×6) = 36 number of observations.

Let X be the random variable, which represents the number of successes,

1. Here, success refers to the number greater than 4, X can take the value 0, 1 or 2.

P (X = 0) = P (a number less than or equal to 4 on both the tosses) = \(\frac{4}{6}\) x \(\frac{4}{6}\) = \(\frac{4}{9}\)

P (X = 1) = P (number less than or equal to 4 on first toss and greater than 4 on second toss) + P (number greater than 4 on first toss and less than or equal to 4 on second toss)

= \(\frac{4}{6}\) x \(\frac{2}{6}\) + \(\frac{4}{6}\) x \(\frac{2}{6}\) = \(\frac{4}{9}\)

P (X = 2) = P (number greater than 4 on both the tosses) = \(\frac{2}{6}\) x \(\frac{2}{6}\) = \(\frac{1}{9}\)

Thus, the probability distribution is as follows.

Probability Distribution Of Point At x Is 2

2. Here, success means six appears on at least one die, X can take the value 0 or 1.

P (X = 0) = P (six does not appear on any of the dice) = \(\frac{5}{6}\) x \(\frac{5}{6}\) = \(\frac{25}{36}\)

P (X = 1) = P (six appears on at least one of the dice).

Thus, the required probability distribution is as follows.

Probability Distribution Of Point At x Is 1

Question 6. From a lot of 30 bulbs which include 6 defectives, a sample of 4 bulbs is drawn at random with replacement. Find the probability distribution of the number of defective bulbs.
Solution:

It is given that out of 30 bulbs, 6 are defective.

⇒ Number of non-defective bulbs = 30 – 6 = 24

4 bulbs are drawn from the lot with replacement.

Let X be the random variable that denotes the number of defective bulbs In the selected bulbs, X can take the value 0, 1, 2, 3 or 4.

∴ P (X = 0) = P (4 non-defective and 0 defective) = \({ }^4 \mathrm{C}_0 \cdot \frac{4}{5} \cdot \frac{4}{5} \cdot \frac{4}{5} \frac{4}{5}=\frac{256}{625}\)

P (X = 1) = P (3 non-defective and 1 defective) = \({ }^4 \mathrm{C}_1 \cdot\left(\frac{1}{5}\right) \cdot\left(\frac{4}{5}\right)^3=\frac{256}{625}\)

P (x = 2) = P (2 non-defective and 2 detective) = \({ }^4 C_2 \cdot\left(\frac{1}{5}\right)^2 \cdot\left(\frac{4}{5}\right)^2=\frac{96}{625}\)

p (X = 3) = P(1 non-defective and 3 defective) = \({ }^4 \mathrm{C}_3 \cdot\left(\frac{1}{5}\right)^3 \cdot\left(\frac{4}{5}\right)=\frac{16}{625}\)

P (x = 4) = P (0 non-defective and 4 defective)= \({ }^4 \mathrm{C}_4 \cdot\left(\frac{1}{5}\right)^4 \cdot\left(\frac{4}{5}\right)^0=\frac{1}{625}\)

Therefore, the required probability distribution is as follows

Probability Distribution Of Number Of Defective Balls

Question 7. A coin is biased so that the head is 3 times as likely to occur as the tail. If the coin is tossed twice, find the probability distribution of a number of tails.
Solution:

Let the probability of getting a tail in tossing one biased coin be x.

P (T) = x

⇒ P(H) = 3x

Now, P (T) + P (H) = 1

⇒ x + 3x = 1

⇒ 4x = l

x = \(\frac{1}{4}\)

∴ P (T) = \(\frac{1}{4}\) and P (H) = \(\frac{3}{4}\)

When the coin is tossed twice, the sample space is {HH, TT, HT, TH}.

Let X be the random variable representing the number of tails, Here ‘X’ can be 0, 1 or 2

∴ P(X = 0) – P (no tail) = P(H) x P(H) = \(\frac{3}{4} \times \frac{3}{4}=\frac{9}{16}\)

p (X= 1) = P (one tail) = P (HT) + P (TH) = \(\frac{3}{4} \cdot \frac{1}{4}+\frac{1}{4} \cdot \frac{3}{4}=\frac{3}{8}\)

P (X = 2) = P (two tails) = P (TT) = \(\frac{1}{4} \times \frac{1}{4}=\frac{1}{16}\)

Therefore, the required probability distribution is as follows.

Probability Distribution Of Number Of Tails

Question 8. A random variable X has the following probability distribution. Determine

Probability Random Variable X Has Probability Distribution

  1. k
  2. P (X < 3)
  3. P (X > 6)
  4. P (0 < X < 3)

Solution:

1. It is known that the sum of probabilities of a probability distribution of random variables is one.

∴ 0 + k + 2k + 2k + 3k + k² + 2k² + (7k² + k) = 1

⇒ 10k² + 9k – 1 = 0 ⇒(l0k-1)(k + 1) = 0

⇒ k = -1, \(\frac{1}{10}\)

k = -1 is not possible as the probability of an event is never negative.

∴ k = \(\frac{1}{10}\)

2. P (X < 3) = P (X – 0) + P (X = 1) + P (X = 2)

= \(0+\mathrm{k}+2 \mathrm{k}=3 \mathrm{k}=3 \times \frac{1}{10}=\frac{3}{10}\) (because \(\mathrm{k}=\frac{1}{10}\))

3. \(\mathrm{P}(\mathrm{X}>6)=\mathrm{P}(\mathrm{X}=7)=7 \mathrm{k}^2+\mathrm{k}=7 \times\left(\frac{1}{10}\right)^2+\frac{1}{10}=\frac{7}{100}+\frac{1}{10}=\frac{17}{100}\)

4. \(\mathrm{P}(0<\mathrm{X}<3)=\mathrm{P}(\mathrm{X}=1)+\mathrm{P}(\mathrm{X}=2)=\mathrm{k}+2 \mathrm{k}=3 \mathrm{k}=3 \times \frac{1}{10}=\frac{3}{10}\) (because \(\mathrm{k}=\frac{1}{10})\)

Question 9. The random variable X has probability distribution P(X) of the following form, where k is some number:

P(X) = \(=\left\{\begin{array}{cc}
k, & \text { if } x=0 \\
2 k, & \text { if } x=1 \\
3 k & \text { if } x=2 \\
0 & \text { otherwise }
\end{array}\right.\)

  1. Determine the value of k.
  2. Find P(X < 2), P(X ≤ 2), P(X ≥ 2).

Solution:

1. It is known that the sum of probabilities of a probability distribution of random variables is one.

∴ k + 2k + 3k + 0 = 1 6k = 1 ⇒ k = \(\frac{1}{6}\)

2. P(X < 2) = P(X = 0) + P(X = l) = k + 2k = 3k = \(\frac{3}{6}\) = \(\frac{1}{2}\)

P(X ≤ 2) = P(X = 0) + P(X = 1) + P (X = 2) = k + 2k + 3k + 6k = \(\frac{6}{6}\) = 1

P(X ≥ 2) = P(X = 2) + P (X > 2) = 3k + 0 = 3k = \(\frac{3}{6}\) = \(\frac{1}{2}\)

Question 10. Find the mean number of heads in three tosses of a fair coin.
Solution:

Let X denote the success of getting heads.

Therefore, the sample space is S = {HHH, HHT, HTH, HTT, THH, THT, TTH, TTT} It can be seen that X can take the value of 0, 1, 2 or 3.

P(X = 0) = P (TTT) = \(\frac{1}{8}\)

P(X = 1) = P(getting one head and two tails)

= P(TTH} + P{THT} + P{HTT} = \(\frac{1}{8}\) + \(\frac{1}{8}\) + \(\frac{1}{8}\) = \(\frac{3}{8}\)

P(X = 2) = P (getting 2 head and 1 tail)

= P(HHT) + P (HTH) + P(THH) = \(\frac{1}{8}\) + \(\frac{1}{8}\) + \(\frac{1}{8}\) = \(\frac{3}{8}\)

P(X = 3) = P(HHH) = \(\frac{1}{8}\)

Therefore, the required probability distribution is as follows.

Probability Probability Distribution Of A value 0 1 2 Or 3

Mean \(\mu=\Sigma X_i P\left(X_i\right)=0 \times \frac{1}{8}+1 \times \frac{3}{8}+2 \times \frac{3}{8}+3 \times \frac{1}{8}=\frac{3}{8}+\frac{3}{4}+\frac{3}{8}=\frac{3}{2}=1.5\)

Question 11. Two dice are thrown simultaneously. If X denotes the number of sixes, find the expectation of X.
Solution:

Here, X represents the number of sixes obtained when two dice are thrown simultaneously.

Therefore, X can take the value of 0, 1 or 2.

∴ P(X = 0) = P (not getting six on any of the dice) = \(\frac{25}{36}\)

P(X = 1) = P (six on first die and no six on second die) + P (no six on first die and six on second die) \(\frac{1}{6} \times \frac{5}{6}+\frac{1}{6} \times \frac{5}{6}=2\left(\frac{1}{6} \times \frac{5}{6}\right)=\frac{10}{36}\)

P (X = 2) = P (six on both the dice) = \(\frac{1}{36}\)

Therefore, the required probability distribution is as follows.

Probability Two dice Are Thrown Simutaneously

Then, expectation of X = \(E(X)=\sum X_i P\left(X_i\right)=0 \times \frac{25}{36}+1 \times \frac{10}{36}+2 \times \frac{1}{36}=\frac{1}{3}\)

Question 12. Two numbers are selected at random (without replacement) from the first six positive integers. Let X denote the larger of the two numbers obtained. Find E(X).
Solution:

The two positive integers can be selected from the first six positive integers without replacement in 6 x 5 = 30 ways.

X represents the larger of the two numbers obtained. Therefore, X can hike the value of 2,3,4, 5 or 6.

For X = 2, the possible observations are (1, 2) and (2, 1).

∴ P(X = 2) = \(\frac{2}{30}\) = \(\frac{1}{15}\)

For X = 3, the possible observations are (1,3), (2, 3), (3,1) and (3,2).

∴ P(X = 3) = \(\frac{4}{30}\) = \(\frac{2}{15}\)

For X = 4, the possible observations are (1, 4), (2, 4), (3, 4), (4, 3), (4, 2) and (4, 1).

∴ P(X = 4) = \(\frac{6}{30}\) = \(\frac{3}{15}\)

For X = 5, the possible observations are (1, 5), (2, 5), (3, 5), (4, 5), (5,4), (5, 3), (5,2) and (5,1).

∴ P(X = 5) = \(\frac{8}{30}\) = \(\frac{4}{15}\)

For X = 6, the possible observations are (1, 6), (2, 6), (3, 6), (4, 6), (5, 6), (6, 5), (6, 4), (6, 3), (6, 2) and (6. 1).

∴ P(X = 6) = \(\frac{10}{30}\) = \(\frac{1}{3}\)

Therefore, the required probability distribution is as follows.

Probability Two Numbers Are Selected At Random Variables

Then, \(E(X)=\Sigma X_i P\left(X_i\right)=2 \cdot \frac{1}{15}+3 \cdot \frac{2}{15}+4 \cdot \frac{1}{5}+5 \cdot \frac{4}{15}+6 \cdot \frac{1}{3}\)

= \(\frac{2}{15}+\frac{2}{5}+\frac{4}{5}+\frac{4}{3}+2=\frac{70}{15}=\frac{14}{3}\)

Choose The Correct Answer

Question 13. The mean of the numbers obtained on throwing a die having written 1 on three faces, 2 on two faces and 5 on one face is

  1. 1
  2. 2
  3. 5
  4. 8/3

Solution:

Let X be the random variable representing a number on the die. Here X can be 1,2 or 5

The total number of observations is six.

∴ P(X=1) = \(\frac{3}{6}=\frac{1}{2}\)

P(X=2) = \(\frac{2}{6}=\frac{1}{3}\)

P(X=5) = \(\frac{1}{6}\)

Therefore, the probability distribution is as follows:

Probability X Be The Random Varible Representing A Number On Die

Mean = \(E(X)=\Sigma X_i P\left(X_i\right)=1 \times \frac{1}{2}+2 \times \frac{1}{3}+5 \times \frac{1}{6}=\frac{1}{2}+\frac{2}{3}+\frac{5}{6}=\frac{3+4+5}{6}=\frac{12}{6}=2\)

Question 14. Suppose that two cards are drawn at random from a deck of cards. Let X be the number of aces obtained. Then the value of E(X) is

  1. \(\frac{37}{221}\)
  2. \(\frac{5}{13}\)
  3. \(\frac{1}{13}\)
  4. \(\frac{2}{13}\)

Solution:

Let X denote the number of aces obtained. Therefore, X can take any of the values of 0, 1, or 2.

In a deck of 52 cards, 4 cards are aces. Therefore, there are 48 non-ace cards.

∴ P (X = 0) = P (0 ace and 2 non-ace cards)

= \(\frac{{ }^4 \mathrm{C}_0 \times{ }^{48} \mathrm{C}_2}{{ }^{52} \mathrm{C}_2}=\frac{1128}{1326}\)

P(X=1) = P(1 ace and 1 non-ace cards)

= \(\frac{{ }^4 \mathrm{C}_1 \times{ }^{48} \mathrm{C}_1}{{ }^{52} \mathrm{C}_2}=\frac{192}{1326}\)

P(X=2)=P(2 ace and 0 non- ace cards)

= \(\frac{{ }^4 \mathrm{C}_2 \times{ }^{48} \mathrm{C}_6}{{ }^{57} \mathrm{C}_2}=\frac{6}{1326}\)

Thus, the probability distribution is as follows.

Probability Two Cards Are Drawn At random From A Deck Of cards

Then, \(E(X)=\Sigma P\left(X_i\right) \cdot X_i=0 \times \frac{1128}{1326}+1 \times \frac{192}{1326}+2 \times \frac{6}{1326}=\frac{204}{1326}=\frac{2}{13}\)

Therefore, the correct answer is (4).

Probability Miscellaneous Exercise

Question 1. A and B are two events such that P(A)≠0.Find P(B | A), If

  1. A is a subset of B
  2. A∩B = ø

Solution:

It is given that. P (A) ≠ 0

1. Given A is a subset of B

⇒ \(A \cap B=A \text { i.e. }(A \subset B)\)

∴ \(P(A \cap B)=P(B \cap A)=P(A)\)

∴ \(P(B \mid A)=\frac{P(B \cap A)}{P(A)}=\frac{P(A)}{P(A)}=1\)

2. \(\mathrm{A} \cap \mathrm{B}=\phi \Rightarrow \mathrm{P}(\mathrm{A} \cap \mathrm{B})=0\)

∴ \(P(B \mid A)=\frac{P(A \cap B)}{P(A)}=0\)

Question 2. A couple has two children,

  1. Find the probability that both children are males, if it is known that at least one of the children is male,
  2. Find the probability that both children are females if it is known that the elder child is a female.

Solution:

If a couple has two children, then the sample space is S = |(b,b),(b,g),(g.b),(g,g)|

1. Let E and F respectively denote the events that both children are males and at least one of the children is a male.

∴ \(\mathrm{E} \cap \mathrm{F}=\{(\mathrm{b}, \mathrm{b})\} \Rightarrow \mathrm{P}(\mathrm{E} \cap \mathrm{F})=\frac{1}{4}\)

∴ \(\mathrm{P}(\mathrm{E})=\frac{1}{4} ; \mathrm{P}(\mathrm{F})=\frac{3}{4} \Rightarrow \mathrm{P}(\mathrm{E} \mid \mathrm{F})=\frac{\mathrm{P}(\mathrm{E} \cap \mathrm{F})}{\mathrm{P}(\mathrm{F})}=\frac{\frac{1}{4}}{\frac{3}{4}}=\frac{1}{3}\)

2. Let A and B respectively denote the events that both children are females and the elder child is a female.

A = \(\{(g, g)\} \Rightarrow P(A)=\frac{1}{4}, B=\{(g, b),(g, g)\} \Rightarrow P(B)=\frac{2}{4}\)

A \(\cap B=\{(g, g)\} \Rightarrow P(A \cap B)=\frac{1}{4}\)

P\((A \mid B)=\frac{P(A \cap B)}{P(B)}=\frac{\frac{1}{4}}{\frac{2}{4}}=\frac{1}{2}\)

Question 3. Suppose that 5% of men and 0.25% of women have grey hair. A grey-haired person is selected at random. What is the probability of this person being male? Assume that there are equal numbers of males and females.
Solution:

Let E1 and E2 be the events of male and female persons respectively.

∴ P(E1) = 0.5 and P(E2) = 0.5

Let A be the event of a grey-hair person

⇒ P(A | E1) = Probability of selected a grey-haired male = 5% = 0.05

⇒ P(A | E2) = Probability of selected a grey-haired female = 0.25% = 0.0025

The probability of the person selected is male if the person is grey-haired

i.e. \(P\left(E_1 | A\right)=\frac{P\left(E_1\right) \cdot P\left(A | E_1\right)}{P\left(E_1\right) \cdot P\left(A \mid E_1\right)+P\left(E_2\right) \cdot P\left(A | E_2\right)}\)

= \(\frac{0.5 \times 0.05}{0.5 \times 0.0025+0.5 \times 0.05}\)

= \(\frac{0.5 \times 0.05}{0.5[0.0025+0.05]}\)

= \(\frac{0.05}{0.0525}=\frac{500}{525}=\frac{20}{21}\)

Question 4. Suppose that 90% of people are right-handed. What is the probability that at most 6 of a random sample of 10 people are right-handed?
Solution:

A person can be either right-handed or left-handed.

It is given that 90% of the people are right-handed.

∴ p = P (Right-handed) = \(\frac{9}{10}\)

q = P (Left-handed) = 1 – \(\frac{9}{10}\) = \(\frac{1}{10}\)

Using the binomial distribution, the probability that more than 6 people are right-handed is given by,

⇒ \(\sum_{\mathrm{r}=7}^{10}{ }^{10} \mathrm{C}_{\mathrm{r}} \mathrm{p}^{\mathrm{T}} \mathrm{q}^{\mathrm{n}-\mathrm{r}}=\sum_{\mathrm{r}=7}^{10}{ }^{10} \mathrm{C}_{\mathrm{r}}\left(\frac{9}{10}\right)^{\mathrm{r}}\left(\frac{1}{10}\right)^{10-\mathrm{r}}\)

Therefore, the probability that at most 6 people are right-handed

= 1-P More than 6 are right-handed

= \(1-\sum_{r=7}^{10}{ }^{10} \mathrm{C}_r(0.9)^r(0.1)^{10-r}\)

Question 5. If a leap year is selected at random, what is the chance that it will contain 53 Tuesdays?
Solution:

In a leap year, there are 366 days, i.e., 52 weeks and 2 days.

In 52 weeks, there are 52 Tuesdays.

Therefore, the probability that the leap year will contain 53

Tuesdays is equal to the probability that one of the remaining 2 days will be Tuesdays.

The remaining 2 days can be S = {Monday Tuesday, Tuesday Wednesday, Wednesday Thursday,

Thursday Friday, Friday Saturday, Saturday Sunday, Sunday Monday}

⇒ n(S) = 7

Favourable cases F = {(Monday, Tuesday), (Tuesday, Wednesday)}

⇒ n(F) = 2

Probability that a leap year will have 53 Tuesdays = \(\frac{2}{7}\)

Question 6. Suppose we have four boxes. A, B, C and D containing coloured marbles as given below:

Probability Four Boxes Containing Coloured Maribles

One of the boxes has been selected at random and a single marble is drawn from it. If the marble is red, what is the probability that it was drawn from box A?, box B?, or box C?

Solution:

Let R be the event of drawing the red marble

Let EA, EB and EC respectively denote the events of selecting the boxes A, B and C.

Total number of marbles = 40

Number of red marbles = 15

∴ P(R) = \(\frac{15}{40}\) = \(\frac{3}{8}\)

The probability of drawing the red marble from box A is given by P(EA|R).

∴ \(P\left(E_A \mid R\right)=\frac{P\left(E_A \cap R\right)}{P(R)}=\frac{\frac{1}{40}}{\frac{3}{8}}=\frac{1}{15}\)

The probability that the red marble is from box B is P(EB|R)

∴ \(P\left(E_B \mid R\right)=\frac{P\left(E_B \cap R\right)}{P(R)}=\frac{\frac{6}{40}}{\frac{3}{8}}=\frac{2}{5}\)

The probability that the red marble is from box C is P(EC|R)

∴ \(P\left(E_C \mid R\right)=\frac{P\left(E_C \cap R\right)}{P(R)}=\frac{\frac{8}{40}}{\frac{3}{8}}=\frac{8}{15}\)

Question 7. Assume that the chances of the patient having a heart attack are 40%. It is also assumed that a meditation and yoga course reduces the risk of heart attack by 30% and prescription of certain drugs reduces its chances by 25%. At a time a patient can choose any one of the two options with equal probabilities. It is given that after going through one of the two options the patient selected at random suffers a heart attack. Find the probability that the patient followed a course of meditation and yoga.
Solution:

Let E1 and E2 denote the events that the selected person followed the course of yoga and meditation, and the person adopted the drug prescription, respectively

∴ P(E1) = P(E2) = \(\frac{1}{2}\)

Let A be the event that person has a heart attack

∴ P(A) = 0.40

P(A|E1) = 0.40×0.70 = 0.28

P(A|E2) = 0.40X0.75 = 0.30

The probability that the patient suffering a heart attack followed a course of meditation and yoga is given by P(E1|A).

⇒ \(\mathrm{P}\left(\mathrm{E}_1 \mid \mathrm{A}\right)=\frac{\mathrm{P}\left(\mathrm{E}_1\right) \mathrm{P}\left(\mathrm{A} \mid \mathrm{E}_1\right)}{\mathrm{P}\left(\mathrm{E}_1\right) \mathrm{P}\left(\mathrm{A} \mid \mathrm{E}_1\right)+\mathrm{P}\left(\mathrm{E}_2\right) \mathrm{P}\left(\mathrm{A} \mid \mathrm{E}_2\right)}\)

= \(\frac{\frac{1}{2} \times 0.28}{\frac{1}{2} \times 0.28+\frac{1}{2} \times 0.30}=\frac{14}{29}\)

Question 8. If each element of a second-order determinant is either zero or one, what is the probability that the value of the determinant is positive? (Assume that the individual entries of the determinant are chosen independently, each value being assumed with probability \(\frac{1}{2}\)).
Solution:

The total number of determinants of second order with each element being 0 or 1 is (2)4 =16

The value of the determinant is positive in the following cases, \(\left\{\left|\begin{array}{ll}
1 & 0 \\
0 & 1
\end{array}\right|,\left|\begin{array}{ll}
1 & 1 \\
0 & 1
\end{array}\right|,\left|\begin{array}{ll}
1 & 0 \\
1 & 1
\end{array}\right|\right\}\)

Required probability = \(\frac{3}{16}\)

Question 9. An electronic assembly consists of two subsystems, say A and B. From previous testing procedures, the following probabilities are assumed to be known:

  1. P (A fails) = 0.2
  2. P (B fails alone) = 0,15
  3. P (A and B fail) = 0.15

Evaluate the following probabilities

  1. P (A fails | B has failed)
  2. P (A fails alone)

Solution:

Let the event in which A fails and B fails to be denoted by EA and EB.

P(EA)=0.2

P(EA∩EB) = 0.15

P (B fails alone) = P (EB) – P(EA∩EB)

∴ 0.15 = P(EB)-0.15

⇒ P(EB) = 0.3

  1. \(P\left(E_A \mid E_B\right)=\frac{P\left(E_A \cap E_B\right)}{P\left(E_B\right)}\) = \(\frac{0.15}{0.3}=0.5\)
  2. \(\mathrm{P}(\mathrm{A} \text { fails alone })=\mathrm{P}\left(\mathrm{E}_{\mathrm{A}}\right)-\mathrm{P}\left(\mathrm{E}_{\mathrm{A}} \cap \mathrm{E}_{\mathrm{B}}\right)\) =0.2-0.15=0.05

Question 10. Bag 1 contains 3 red and 4 black balls and Bag 2 contains 4 red and 5 black balls. One ball is transferred from Bag 1 to Bag 2 and then a ball is drawn from Bag 2. The ball so drawn is found to be red in colour. Find the probability that the transferred ball is black.
Solution:

Let E1 and E2 respectively denote the events that a red ball is transferred from bag 1 to 2 and a black ball is transferred from bag 1 to 2.

⇒ \(\mathrm{P}\left(\mathrm{E}_1\right)=\frac{3}{7} \text { and } \mathrm{P}\left(\mathrm{E}_2\right)=\frac{4}{7}\)

Let A be the event that the ball drawn is red.

When a red ball is transferred from bag 1 to 2, \(\mathrm{P}\left(\mathrm{A} \mid \mathrm{E}_1\right)=\frac{5}{10}=\frac{1}{2}\)

When a black ball Is transferred from bag 1 to 2, \(\mathrm{P}\left(\mathrm{A} \mid \mathrm{E}_2\right)=\frac{4}{10}=\frac{2}{5}\)

The probability that the transferred ball is black in colour when the ball is drawn is red in colour is given by P(E2|A)

By using Baye’s theorem, we obtain

∴ \(P\left(E_2 \mid A\right)=\frac{P\left(E_2\right) P\left(A \mid E_2\right)}{P\left(E_1\right) P\left(A \mid E_1\right)+P\left(E_2\right) P\left(A \mid E_2\right)}\)

= \(\frac{\frac{4}{7} \times \frac{2}{5}}{\frac{3}{7} \times \frac{1}{2}+\frac{4}{7} \times \frac{2}{5}}=\frac{16}{31}\)

Choose The Correct Answer

Question 11. If A and B are two events such that \(P(A) \neq 0\) and \(P(B \mid A)=1\), then.

  1. \(\mathrm{A} \subset \mathrm{B}\)
  2. \(\mathrm{B} \subset \mathrm{A}\)
  3. \(\mathrm{B}=\phi\)
  4. \(\mathrm{A}=\phi\)

Solution: 1. \(\mathrm{A} \subset \mathrm{B}\)

P\((\mathrm{A}) \neq 0\) and \(\mathrm{P}(\mathrm{B} \mid \mathrm{A})=1\)

Now, \(\mathrm{P}(\mathrm{B} \mid \mathrm{A})=\frac{\mathrm{P}(\mathrm{B} \cap \mathrm{A})}{\mathrm{P}(\mathrm{A})}\)

1 = \(\frac{P(B \cap A)}{P(A)}\)

⇒ \(P(A)=P(B \cap A) \Rightarrow A \subset B\)

Thus, the correct answer is 1.

Question 12. If P\((\mathrm{A} \mid \mathrm{B})>\mathrm{P}(\mathrm{A})\), then which of the following is correct:

  1. \(\mathrm{P}(\mathrm{B} \mid \mathrm{A})<\mathrm{P}(\mathrm{B})\)
  2. \(\mathrm{P}(\mathrm{A} \cap \mathrm{B})<\mathrm{P}(\mathrm{A}) \cdot \mathrm{P}\)
  3. \(\mathrm{P}(\mathrm{B} \mid \mathrm{A})>\mathrm{P}(\mathrm{B})\)
  4. \(\mathrm{P}(\mathrm{B} \mid \mathrm{A})=\mathrm{P}(\mathrm{B})\)

Solution: 3. \(\mathrm{P}(\mathrm{B} \mid \mathrm{A})>\mathrm{P}(\mathrm{B})\)

⇒ \(\mathrm{P}(\mathrm{A} \mid \mathrm{B})>\mathrm{P}(\mathrm{A})\)

⇒ \(\frac{P(A \cap B)}{P(B)}>P(A)\)

⇒ \(P(A \cap B)>P(A) \cdot P(B)\)

⇒ \(\frac{P(A \cap B)}{P(A)}>P(B)\)

⇒ \(P(B \mid A)>P(B)\)

Thus, the correct answer is (3).

Question 13. If A and B are any two events such that P (A) + P (B) – P (A and B) = P (A), then

  1. P (B|A) = 1
  2. P (A]B) = 1
  3. P (B|A) = 0
  4. P (A|B) = 0

Solution:

P (A) + P (B) – P (A and B) = P(A)

⇒ P(A) + P(B)-P(A∩B)=P(A)

⇒ P(B)-P(A∩B) = 0

⇒ P(A∩B)=P(B)

Thus, the correct answer is (2).

 

 

Linear Programming Class 12 Maths Important Questions Chapter 12

Linear Programming Exercise 12.1

Solve the following Linear Programming problems graphically.

Question 1. Maximize Z = 3x+4y,

Subject to the constraints x+y≤4, x≥0 and y≥0.

Solution:

We have to

Maximize Z = 3x + 4y

Subject to constraints x + y≤4, x≥0, y≥0

Firstly, draw the graph of the line x+y = 4

Then, putting (0, 0) in the inequality x+y≤4 we have 0 + 0≤4

⇒ ≤4 (Which is true)

So, the half-plane is towards the origin.

Since, x,y≥0

So, the feasible region lies in the first quadrant.

Linear Programming Feasible Region Lies In The First Quadrant

∴ The feasible region is OABO.

The comer points of the feasible region are 0(0,0), A(4,0), and B(0,4), The values of Z at these points are as follows:

Linear Programming Maximum Value Of Z Is 16

Therefore, the maximum value of Z is 16 at the point B(0,4).

Question 2. Minimize Z = -3x + Ay, subject to constraints x + 2y≤8,3x + 2y≤1 2,x≥0 and y≥0.
Solution:

We have to

Minimize Z = -3x+4y

Subject to constraints x+2y≤8, 3x + 2y≤12, x≥0, y≥0

Firstly, draw the graph of the line, x + 2y = 8

Putting (0, 0) in the inequality x + 2y≤8, we have 0 + 0≤8

⇒ 0≤8 (Which is true)

Read and Learn More Class 12 Maths Chapter Wise with Solutions

So, the half-plane is towards the origin.

Linear Programming Half Plane Is Towards The Origin

Since, x,y≥0

So, the feasible region lies in the first quadrant.

Secondly, draw the graph of the line, 3x + 2y = 12

Linear Programming Graph Of The Line

Putting (0, 0) in the inequality 3x + 2y≤12 we have 3 x 0 + 2 x 0 ≤ 12

⇒ 0 ≤12 (Which is true)

So, the half-plane is towards the origin.

∴ Feasible region is OABCO.

On solving equations x + 2y = 8 and 3x+2y = 12, we get x = 2 and y = 3

∴ Intersection point B is (2,3)

The corner points of the feasible region are 0(0,0), A(4,0), B(2,3)and C(0,4).

The values of Z at these points are as follows.

Linear Programming Minimum Value Of Z is -12

Therefore, the minimum value of Z is -12 at the point A(4,0).

Question 3. Maximize Z = 5x + 3y, subject to constraints 3x + 5y≤ 15. 5x + 2y≤10. x≥0 and y≥0.
Solution:

We have to

Maximize Z = 5x + 3y

Subject to constraints 3x + 5y≤15, 5x + 2y≤10, x≥0, y≥0

Firstly, draw the graph of the line. 3x + 5y = 15

Putting (0, 0) in the inequality 3x + 5y≤15. we have 3 x 0 + 5 x 0≤15

⇒ 0≤15 (Which is true)

So, the half-plane is towards the origin.

Since, x,y ≥ 0 So, the feasible region lies in the first quadrant.

Secondly, draw the graph of the line, 5x + 2y≤10

Putting (0, 0) in the inequality 5x + 2y≤10 we have 5 x 0 + 2 x 0 ≤ 10

⇒ 0≤10 (Which is true)

So, the half-plane is towards the origin.

On solving equations, 3x + 5y = 15 and 5x + 2y = 10, we get x = 20/19 and y = 45/19

Linear Programming Coordinatates Of Points

Coordinates of point B is \(\left(\frac{20}{19}, \frac{45}{19}\right)\)

∴ The feasible region is OABCO

The corner points of the feasible region are 0(0,0), A(2,0), B\(\left(\frac{20}{19}, \frac{45}{19}\right)\) and C(0,3)

The values of Z at these points are as follows:

Linear Programming Maximum Value Of Z Is 235 By 19

Therefore, the maximum value of Z is \(\frac{235}{19}\) at the point B\(\left(\frac{20}{19}, \frac{45}{19}\right)\)

Question 4. Minimize Z = 3x + 5y subject to constraints x + 3y≥3, x+y≥2 and x,y≥0.
Solution:

We have to

Minimize Z = 3x + 5y

Subject to constraints x + 3y≥3, x+y≥2, x≥0, y≥0

Firstly, draw the graph of the line, x + 3y = 3

Putting (0, 0) in the inequality x + 3y≥3, we have 0 + 3 x 0 ≥ 3

⇒ 0≥3 (Which is false)

So, the half-plane is away from the origin. Since, x,y≥0 So, the feasible region lies in the first quadrant.

Secondly, draw the graph of the line, x+y = 2

Putting (0, 0) in the inequality x + y≥2 we have 0 + 0≥2

⇒ G≥2 (Which is false)

So, the half-plane is away from the origin. It can be seen that the feasible region is unbounded.

On solving equations x+y = 2 and x + 3y = 3, we get x = 3/2 and y = 1/2

∴ Intersection point is B\(\left(\frac{3}{2}, \frac{1}{2}\right)\)

Linear Programming Intersection Of The Point

The corner points of the feasible region axe A(3,0), B\(\left(\frac{3}{2}, \frac{1}{2}\right)\) and C(0,2).

The values of Z at these points are as follows:

Linear Programming Maximum Value Of Z Is 3 By 2 And 1 By 2

As the feasible region is unbounded, therefore, 7 may or may not be the minimum value of Z.

For this, we draw the graph of the inequality, 3x + 5y < 7, and check whether the resulting half-plane has points in common with the feasible region or not.

It can be seen that the feasible region has no common point with 3x + 5y < 7.

Therefore, the maximum value of Z is 7 at point B\(\left(\frac{3}{2}, \frac{1}{2}\right)\)

CBSE Class 12 Maths Chapter 12 Linear Programming Important Question And Answers

Question 5. Maximize Z = 3x + 2y, subject to constraints x + 2y≤10, 3x + y≤15and x,y≥0.
Solution:

We have to

Maximize Z = 3x + 2y

Subject to constraints x + 2y≤10, 3x + y≤15, x≥0, y≥0

Firstly, draw the graph of the line, x + 2y = 10

Putting (0, 0) in the inequality x + 2y≤10, we have 0 + 2 x 0≤10

⇒ 0 ≤10 (Which is true)

So, the half-plane is towards the origin.

Since, x,y≥0

So, the feasible region lies in the first quadrant.

Secondly, draw the graph of the line, 3x+y = 15

Putting (0, 0) in the inequality 3x+y≤15 we have 3 x 0 + 0 ≤ 15

⇒ 0 ≤ 15 (Which is true)

So, the half-plane is towards the origin.

On solving equations x + 2y = 10 and 3x + y = 15, we get x = 4 and y = 3

∴ Intersection point B is (4,3)

∴ The feasible region is OABCO.

Linear Programming Corner Points Of The Feasible Region

The corner points of the feasible region are 0(0,0), A(5,0), B(4,3)and C(0,5). The values of Z at these points are as follows:

Linear Programming Maximum Value Of Z Is 18

Therefore, the maximum value of Z is 18 at the point B(4,3).

Question 6. Minimize Z = x + 2y, subject to constraints are 2x+y≥3, x + 2y≥6 and x,y≥0. Show that the minimum of Z occurs at more than two points.
Solution:

We have to

Minimize Z = x + 2y

Subject to constraints 2x+y≥3, x + 2y≥6 , x≥0, y≥ 0

Firstly, draw the graph of the line, 2x+y = 3

Putting (0, 0) in the inequality 2x+y≥3, we have 2 x 0 + 0≥3

⇒ 0≥3 (Which is false)

So, the half-plane is away from the origin.

Since, x, y≥0 So, the feasible region lies in the first quadrant.

Secondly, draw the graph of the line, x + 2y = 6

Putting (0, 0) in the inequality x+2y≥6 we have 0 + 2 x 0 ≥ 6

⇒ 0≥6 (Which is false) So. the half plane is away from the origin.

The intersection point of the lines x + 2y = 6 and 2x + y = 3 is B (0,3)

The corner points of the feasible region are A(6,0), and B(0,3). The values of Z at these points are as follows:

Linear Programming Feasible Region Is Unbounded

Linear Programming Maximum Value Of Z At Every Point On The Line

As the feasible region is unbounded, therefore, Z = 6 may or may not be the minimum value. For this, we draw the inequality, x+2y<6, and check whether the resulting half-plane has points in common with the feasible region or not.

Here there is no common point between the unbounded feasible region and the open half plane Therefore, the value of Z is minimum at every point on the line, x+2y = 6.

Question 7. Minimize and maximize Z = 5x + 10y subject to constraints are x+2y≤120, x+y≥60, x-2y≥0and x,y≥0.
Solution:

We have to

Minimize and maximize Z = 5x +10y

Subject to constraints x+2y≤120, x+y≥60, x,y≥0, x-2y≥0

Firstly, draw the graph of the line, x+2y = 120

Putting (0, 0) in the inequality x + 2y≤120, we have 0 + 2 x 0≤120 ⇒ 0≤120 (Which is true) So, the half-plane is towards the origin.

Secondly, draw the graph of the line, x+y = 60

Linear Programming Corner Points Of The Feasible Region And Coordinates

Putting (0, 0) in the inequality x+y≥60, we have 0 + 0≥60

⇒ 0≥60 (Which is false)

So, the half-plane is away from the origin.

Thirdly, draw the graph of the line x- 2y = 0

Putting (5, 0) in the inequality x-2y≥0 we have 5 – 2 x 0 ≥ 0

⇒ 5≥0 (Which is true)

So, the half-plane is towards the A-axis. Since, x,y≥0

So, the feasible region lies in the first quadrant.

∴ The feasible region is ABCDA.

On solving equations x – 2y = 0 and x + y = 60, we get D(40,20)

And on solving equations x-2v = 0 and x+2y = 120 , we get C(60,30)

The corner points of the feasible region are, A(60,0), B(120,0), C(60,30), and D(40,20).

The values of Z at these points are as follows:

Linear Programming Minimum Value Of Z is 300

The minimum value of Z is 300 at A (60,0) and the maximum value of Z is 600 at all the points on the line segment joining the points B (120, 0) and C (60, 30).

Question 8. Minimize and maximize Z = x + 2y subject to constraints are x + 2y≥100, 2x -y≤ 0,
2x + y≤200 and x,y≥0.
Solution:

We have to

Minimize and maximize Z = x+2y

Subject to constraints x+2y≥100, 2x-y≤ 0, 2x + y≤200, x≥0, y≥ 0

Firstly, draw the graph of the line, x + 2y = 100

Putting (0, 0) in the inequality x + 2y≥100, we have 0 + 2×0≥100

⇒ 0≥100 (Which is false)

So, the half-plane is away from the origin.

Secondly, draw the graph of the line, 2x -y = 0

Putting (5, 0) in the inequality 2x -y≤0

we have 2 x 5 – 0 ≤ 0

⇒ 10≤0 (Which is false)

So, the half-plane is towards the Y-axis.

Thirdly, draw the graph of the line 2x+y = 200

Putting (0, 0) in the inequality 2x+y≤200 we have 2 x 0 + 0 ≤ 200 ⇒ 0 < 200 (Which is true)

So, the half-plane is towards the origin. Since, x,y ≥ 0 So, the feasible region lies in the first quadrant.

Linear Programming Feasible Region Of ABCDA

On solving equations 2x-y = 0 and x + 2y = 100 , we get B(20,40)

And on solving equations2x-y = 0and 2x+y = 200 , we get C(50, l00)

∴ The feasible region is ABCDA.

The corner points of the feasible region are, A (0,50), B(20,40), C(50,100), and (0,200).

The values of Z at these points are as follows:

Linear Programming Maximum Value Of Z Is 400

The maximum value of Z is 400 at D(0,200)and the minimum value of Z is 100 at all the points on the line segment joining A(0,50)and B(20,40).

Question 9. Maximize Z = -x + 2y, subject to the constraints x≥3, x + y≥5,x + 2y≥6 and y≥0.
Solution:

We have to

Maximize Z = -x+2y

Subject to constraints x≥3, x + y≥5, x+2y≥6, x≥0, y≥0

Firstly, draw the graph of the line, x+y = 5

Putting (0, 0) in the inequality x+y≥5, we have 0 + 0≥5

⇒ 0≥5 (Which is false)

So, the half-plane is away from the origin.

Secondly, draw the graph of the line, x+2y = 6

Putting (0, 0) in the inequality x + 2y≥6, we have 0 + 2 x 0≥6

⇒ 0≥6 (Which is false)

So, the half-plane is away from the origin.

It can be seen that the feasible region is unbounded.

The corner points of the feasible region are A(6,0), B(4,1) and C(3,2).

The values of Z at these points are as follows;

Linear Programming Z Has No Maximum Value

As the feasible region is unbounded, therefore, Z = 1 may or may not be the maximum value.

For this, we draw the inequality, -x + 2y > 1, and check whether the resulting half-plane has points in common with the feasible region or not.

The resulting feasible region has points in common with the feasible region.

Therefore, Z = 1, is not the maximum value.

Hence, Z has no maximum value.

Question 10. Maximize Z = x+y, subject to constraints are x-y≤-1, -x + y≤0 and x, y≥0.
Solution:

We have to

Maximize Z = x+y

Subject to constraints x-y≤-1, -x + y≤0, x ≥0, y ≥ 0

Firstly, draw the graph of the line, x-y = -1

Putting (0, 0) in the inequality x-y ≤ -1, we have 0-0≤-1

⇒ 0≤-1 (Which is false)

So, the half-plane is away from the origin.

Secondly, draw the graph of the line, -x + y = 0

Putting (2, 0) in the inequality-x+y ≤ 0 we have -2 + 0≤0

⇒ -2≤0 (Which is true)

So, the half-plane is towards the X-axis.

Since, x,y≥0

So, the feasible region lies in the first quadrant.

Linear Programming No Common Region

From the above graph, it is clearly shown that there is no common region. Hence, there is no feasible region and thus Z has no maximum value.

 

 

 

Three Dimensional Geometry Class 12 Maths Important Questions Chapter 11

Three-Dimensional Geometry Exercise 11.1

Question 1. If a line makes angles 90°, 135°, and 45° with x, y, and z-axes respectively, find its direction cosines. Sol. Let the direction cosines of the line be l, m, and n.
Solution:

Let the direction cosines of the line be l, m, and n.

l = \(\cos 90^{\circ}=0 ; m=\cos \left(180^{\circ}-45^{\circ}\right)=-\cos 45^{\circ}=-\frac{1}{\sqrt{2}} ; \mathrm{n}=\cos 45^{\circ}=\frac{1}{\sqrt{2}}\)

Therefore, the direction cosines of the line are \(0,-\frac{1}{\sqrt{2}}\) and \(\frac{1}{\sqrt{2}}\)

Therefore, the direction cosines of the line are 0,\(-\frac{1}{\sqrt{2}}\) and \(\frac{1}{\sqrt{2}}\)

Question 2. Find the direction cosines of a line which makes equal angles with the coordinate axes.
Solution:

Let the line make an angle ‘α’ with each of the coordinate axes.

∴ l = \(\cos \alpha, \mathrm{m}=\cos \alpha, \mathrm{n}=\cos \alpha\)

∴ \(l^2+\mathrm{m}^2+\mathrm{n}^2=1\)

⇒ \(\cos ^2 \alpha+\cos ^2 \alpha+\cos ^2 \alpha=1 \Rightarrow 3 \cos ^2 \alpha=1 \Rightarrow \cos ^2 \alpha=\frac{1}{3} \Rightarrow \cos \alpha= \pm \frac{1}{\sqrt{3}}\)

Thus, the direction cosines of the line, which is equally inclined to the coordinate axes, are \(\pm \frac{1}{\sqrt{3}}, \pm \frac{1}{\sqrt{3}}\), and \(\pm \frac{1}{\sqrt{3}}\)

Question 3. If a line has the direction ratios -18, 12, – 4, then what are its direction cosines?
Solution:

If a line has direction ratios of -18, 12, and -4, then its direction cosines are \(\frac{-18}{\sqrt{(-18)^2+(12)^2+(-4)^2}}, \frac{12}{\sqrt{(-18)^2+(12)^2+(-4)^2}}, \frac{-4}{\sqrt{(-18)^2+(12)^2+(-4)^2}}\)

i.e., \(\frac{-18}{22}, \frac{12}{22}, \frac{-4}{22} \Rightarrow \frac{-9}{11}, \frac{6}{11}, \frac{-2}{11}\)

Thus, the direction cosines are \(\frac{-9}{11}, \frac{6}{11}\), and \(\frac{-2}{11}\)

Read and Learn More Class 12 Maths Chapter Wise with Solutions

Question 4. Show that the points (2, 3, 4), (-1, -2, 1), (5, 8, 7) are collinear.
Solution:

The given points are A (2, 3, 4), B (- 1, – 2, 1), and C (5, 8, 7).

It is known that the direction ratios of lines joining the points, (x1, y1, z1) and (x2, y2, z2), are given by, (x2-x1, y2-y1, z2-z1).

The direction ratios of AB are a1= (-1 – 2), b1= (-2 – 3), and c1= (1 -4) i.e., a1 = -3, b1= -5, and c1= – 3.

The direction ratios of BC are a2= (5 – (- 1)), b2 = (8 – (- 2)), and c2= (7-1) i.e., a2 = 6, b2= 10, and c2= 6.

∴ \(\frac{\mathrm{a}_1}{\mathrm{a}_2}=\frac{\mathrm{b}_1}{\mathrm{~b}_2}=\frac{\mathrm{c}_1}{\mathrm{c}_2}=-\frac{1}{2}\) i.e. they are proportional.

Therefore, AB is parallel to BC. Since point B is common to both AB and BC, points A, B, and C are collinear.

Question 5. Find the direction cosines of the sides of the triangle whose vertices are (3, 5, – 4), (-1,1,2), and (-5,-5,-2)
Solution:

The vertices of ΔABC are A (3, 5, -4), B (-1, 1,2), and C (-5, -5, -2)

The direction ratios of side AB are (-1 -3), (1 -5), and (2 -(-4)) i.e., (- 4, -4, 6).

Three Dimensional Geometry Direction Cosines Of The Sides Of The Triangle

Therefore, the direction cosines of AB are \(\frac{-4}{\sqrt{(-4)^2+(-4)^2+(6)^2}}, \frac{-4}{\sqrt{(-4)^2+(-4)^2+(6)^2}}, \frac{6}{\sqrt{(-4)^2+(-4)^2+(6)^2}}\)

⇒ \(\frac{-4}{2 \sqrt{17}}, \frac{-4}{2 \sqrt{17}}, \frac{6}{2 \sqrt{17}}=\frac{-2}{\sqrt{17}}, \frac{-2}{\sqrt{17}}, \frac{3}{\sqrt{17}}\)

The direction ratios of BC are (-5-(-1)), (-5-1), and (-2-2) i.e., (-4,-6,-4). Therefore, the direction cosines of BC are \(\frac{-4}{\sqrt{(-4)^2+(-6)^2+(-4)^2}}, \frac{-6}{\sqrt{(-4)^2+(-6)^2+(-4)^2}}, \frac{-4}{\sqrt{(-4)^2+(-6)^2+(-4)^2}}\)

= \(\frac{-4}{2 \sqrt{17}}, \frac{-6}{2 \sqrt{17}}, \frac{-4}{2 \sqrt{17}}=\frac{-2}{\sqrt{17}}, \frac{-3}{\sqrt{17}}, \frac{-2}{\sqrt{17}}\)

The direction ratios of AC are (-5-3),(-5-5), and (-2-(-4)) i.e., (-8,-10 and 2). Therefore, the direction cosines of AC are \(\frac{-8}{\sqrt{(-8)^2+(-10)^2+(2)^2}}, \frac{-10}{\sqrt{(-8)^2+(-10)^2+(2)^2}}, \frac{2}{\sqrt{(-8)^2+(-10)^2+(2)^2}}\)

= \(\frac{-8}{2 \sqrt{42}}, \frac{-10}{2 \sqrt{42}}, \frac{2}{2 \sqrt{42}}=\frac{-4}{\sqrt{42}}, \frac{-5}{\sqrt{42}}, \frac{1}{\sqrt{42}}\)

CBSE Class 12 Maths Chapter 11 Three Dimensional Geometry Question And Answers

Three-Dimensional Geometry Exercise 11.2

Question 1. Show that the three lines with direction cosines \(\frac{12}{13}, \frac{-3}{13}, \frac{-4}{13}; \frac{4}{13}, \frac{12}{13}, \frac{3}{13}; \frac{3}{13}, \frac{-4}{13}, \frac{12}{13}\) are mutually perpendicular.
Solution:

Two lines with direction cosines, l1, m1, n1, and l2, m2, n2, are perpendicular to each other, if l1l2 + m1m2 + n1n2 = 0

1. For the lines with direction cosines, \(\frac{12}{13}, \frac{-3}{13}, \frac{-4}{13}\) and \(\frac{4}{13}, \frac{12}{13}, \frac{3}{13}\), we obtain

∴ \(l_1 l_2+m_1 m_2+n_1 n_2=\left(\frac{12}{13}\right) \times\left(\frac{4}{13}\right)+\left(\frac{-3}{13}\right) \times\left(\frac{12}{13}\right)+\left(\frac{-4}{13}\right) \times\left(\frac{3}{13}\right)\)

= \(\frac{48}{169}-\frac{36}{169}-\frac{12}{169}=0\)

Therefore, the lines are perpendicular

2. For the lines with direction cosines, \(\frac{4}{13}, \frac{12}{13}, \frac{3}{13}\) and \(\frac{3}{13}=\frac{-4}{13}, \frac{12}{13}\), we obtain

∴ \(l_1 l_2+m_1 m_2+n_1 n_2=\left(\frac{4}{13}\right) \times\left(\frac{3}{13}\right)+\left(\frac{12}{13}\right) \times\left(\frac{-4}{13}\right)+\left(\frac{3}{13}\right) \times\left(\frac{12}{13}\right)\)

= \(\frac{12}{169}-\frac{48}{169}+\frac{36}{169}=0\)

Therefore, the lines are perpendicular,

3. For the lines with direction cosines, \(\frac{3}{13}, \frac{-4}{13}, \frac{12}{13}\) and \(\frac{12}{13}, \frac{-3}{13}, \frac{-4}{13}\), we obtain

∴ \(l_1 l_2+\mathrm{m}_1 \mathrm{~m}_2+\mathrm{n}_1 \mathrm{n}_2=\left(\frac{3}{13}\right) \times\left(\frac{12}{13}\right)+\left(\frac{-4}{13}\right) \times\left(\frac{-3}{13}\right)+\left(\frac{12}{13}\right) \times\left(\frac{-4}{13}\right)\)

= \(\frac{36}{169}+\frac{12}{169}-\frac{48}{169}=0\)

Therefore, the lines are perpendicular.

Thus, all lines are mutually perpendicular.

Question 2. Show that the line through the points (1, -1,2) and (3, 4, -2) is perpendicular to the line through the points (0, 3, 2) and (3, 5, 6).
Solution:

Let AB be the line joining the points, (1, -1, 2) and (3, 4, -2), and CD be the line joining the points, (0, 3, 2) and (3, 5, 6).

The direction ratios, a1, b∴ 1, c1, of AB are (3 -1), (4 – (-1)), and (-2 -2) i.e., (2, 5, – 4).

The direction ratios, a2, b2, c2, of CD are (3 – 0), (5 -3), and (6 -2) i.e., (3, 2, 4).

AB and CD will be perpendicular to each other if a1a2 + b1b2 + c1c2 = 0 ⇒ a1a2 + b1b2 + c1c2 = 2 x 3 + 5 x 2 + (-4) x 4= 6 + 10 -16 = 0

Therefore, AB and CD are perpendicular to each other.

Question 3. Show that the line through the points (4, 7, 8), (2, 3, 4) is parallel to the line through the points (-1,-2, 1), (1,2, 5).
Solution:

Let AB be the line through the points, (4, 7, 8) and (2, 3, 4), and CD be the line through the points, (-1, -2, 1) and (1, 2, 5).

The directions ratios, a1, b1, c1 of AB are (2 – 4), (3 -7), and (4 -8) i.e., (- 2, – 4, – 4).

The direction ratios, a2, b2, c2 of CD are (1 – (-1)), (2 – (-2)), and (5 – 1) i.e., (2, 4, 4).

AB will be parallel to CD, if \(\frac{a_1}{a_2}=\frac{b_1}{b_2}=\frac{c_1}{c_2}\)

⇒ \(\frac{\mathrm{a}_1}{\mathrm{a}_2}=\frac{-2}{2}=-1, \frac{\mathrm{b}_1}{\mathrm{~b}_2}=\frac{-4}{4}=-1\)

and \(\frac{\mathrm{c}_1}{\mathrm{c}_2}=\frac{-4}{4}=-1\)

∴ \(\frac{\mathrm{a}_1}{\mathrm{a}_2}=\frac{\mathrm{b}_1}{\mathrm{~b}_2}=\frac{\mathrm{c}_1}{\mathrm{c}_2}=-1\)

Thus, AB is parallel to CD.

Question 4. Find the equation of the line that passes through the point (1, 2, 3) and is parallel to the vector \(3 \hat{i}+2 \hat{j}-2 \hat{k}\)
Solution:

It is given that the line passes through the point A(1,2,3). Therefore, the position vector point A is \(\overrightarrow{\mathrm{a}}=\hat{\mathrm{i}}+2 \hat{\mathrm{j}}+3 \hat{\mathrm{k}}\).

Let, \(\overrightarrow{\mathrm{b}}=3 \hat{\mathrm{i}}+2 \hat{\mathrm{j}}-2 \hat{\mathrm{k}}\)

It is known that the line which passes through point A and parallel to \(\vec{b}\) is given by \(\vec{r}=\vec{a}+\lambda \vec{b}\), where \(\lambda\) is a constant \(\vec{r}=\hat{i}+2 \hat{j}+3 \hat{k}+\lambda(3 \hat{i}+2 \hat{j}-2 \hat{k})\).

This is the required equation of the line.

Question 5. Find the equation of the line in vector and in Cartesian form’ that passes through the point with position vector \(2 \hat{\mathrm{i}}-\hat{\mathrm{j}}+4 \hat{\mathrm{k}}\) and is in the direction \(\hat{\mathrm{i}}+2 \hat{\mathrm{j}}-\hat{\mathrm{k}}\).
Solution:

⇒ \(\vec{a}=2 \hat{i}-\hat{j}+4 \hat{k}\) and \(\vec{b}=\hat{i}+2 \hat{j}-\hat{k}\)

It is known that a line through a point with position vector \(\vec{a}\) and parallel to \(\vec{b}\) is given by the equation \(\overrightarrow{\mathrm{r}}=\overrightarrow{\mathrm{a}}+\lambda \overrightarrow{\mathrm{b}} \Rightarrow \overrightarrow{\mathrm{i}}=2 \hat{\mathrm{i}}-\hat{\mathrm{j}}+4 \hat{\mathrm{k}}+\lambda(\hat{\mathrm{i}}+2 \hat{\mathrm{j}}-\hat{\mathrm{k}})\)

This is the required equation of the line in vector form.

⇒ \(\vec{r}=x \hat{i}+y \hat{j}+z \hat{k} \Rightarrow x \hat{i}+y \hat{j}+z \hat{k}=(\lambda+2) \hat{i}+(2 \lambda-1) \hat{j}+(-\lambda+4) \hat{k}\)

Eliminating \(\lambda\), we obtain the Cartesian form equation as \(\frac{x-2}{1}=\frac{y+1}{2}=\frac{z-4}{-1}\)

This is the required equation of the given time in Cartesian form.

Question 6. Find the Cartesian equation of the line which passes through the point (-2, 4, -5) and parallel to \(\frac{x+3}{3}=\frac{y-4}{5}=\frac{z+8}{6}\)
Solution:

It is given that the line passes through the point (-2, 4, -5) and is parallel to \(\frac{x+3}{3}=\frac{y-4}{5}=\frac{z+8}{6}\)

The direction rations of the line, \(\frac{x+3}{3}=\frac{y-4}{5}=\frac{z+8}{6}\) are (3, 5, 6).

The required line is parallel to \(\frac{x+3}{3}=\frac{y-4}{5}=\frac{z+8}{6}\)

Therefore, its direction ratios are 3k, 5k, and 6k, where k ≠ 0

It is known that the equation of the line through the point (x1, y1, z1) and with direction ratios, a, b, c is given by \(\frac{x-x_1}{a}=\frac{y-y_1}{b}=\frac{z-z_1}{c}\)

Therefore the equation of the required line is \(\frac{x+2}{3 k}=\frac{y-4}{5 k}=\frac{z+5}{6 k} \Rightarrow \frac{x+2}{3}=\frac{y-4}{5}=\frac{z+5}{6}=k\)

Question 7. The Cartesian equation of a line is \(\frac{x-5}{3}=\frac{y+4}{7}=\frac{z-6}{2}\). Write its Vector form.
Solution:

The Cartesian equation of the line is \(\frac{x-5}{3}=\frac{y+4}{7}=\frac{z-6}{2}\)

The given line passes through the point (5, -4, 6), The position vector of this point is \(\overrightarrow{\mathrm{a}}=5 \hat{\mathrm{i}}-4 \hat{\mathrm{j}}+6 \hat{\mathrm{k}}\)

Also, the direction ratios of the given line are 3, 7, and 2.

This means that the line is in the direction of the vector, \(\overrightarrow{\mathrm{b}}=3 \hat{\mathrm{i}}+7 \hat{\mathrm{j}}+2 \hat{\mathrm{k}}\)

It is known that the line through position vector \(\vec{a}\) and in the direction of the vector \(\vec{b}\) is given by the equation, \(\overrightarrow{\mathrm{r}}=\overrightarrow{\mathrm{a}}+\lambda \overrightarrow{\mathrm{b}}, \lambda \in \mathrm{R} \Rightarrow \overrightarrow{\mathrm{r}}=(5 \hat{\mathrm{i}}-4 \hat{\mathrm{j}}+6 \hat{\mathrm{k}})+\lambda(3 \hat{\mathrm{i}}+7 \hat{\mathrm{j}}+2 \hat{\mathrm{k}})\)

This is the required equation of the given line in vector form.

Question 8. Find the angle between the following pairs of lines:

  1. \(\overrightarrow{\mathrm{r}}=2 \hat{\mathrm{i}}-5 \hat{\mathrm{j}}+\hat{\mathrm{k}}+\lambda(3 \hat{\mathrm{i}}+2 \hat{\mathrm{j}}+6 \hat{\mathrm{k}})\) and \({\overrightarrow{\mathrm{r}}}=7 \hat{\mathrm{i}}-6 \hat{\mathrm{k}}+\mu(\hat{\mathrm{i}}+2 \hat{\mathrm{j}}+2 \hat{\mathrm{k}})\)
  2. \(\overrightarrow{\mathrm{r}}=3 \hat{\mathrm{i}}+\hat{\mathrm{j}}-2 \hat{\mathrm{k}}+\lambda(\hat{\mathrm{i}}-\hat{\mathrm{j}}-2 \hat{\mathrm{k}})\) and \(\overrightarrow{\mathrm{r}}=2 \hat{\mathrm{i}}-\hat{\mathrm{j}}-56 \hat{\mathrm{k}}+\mu(3 \hat{\mathrm{i}}-5 \hat{\mathrm{j}}-4 \hat{\mathrm{k}})\)

Solution:

1. Let θ be the angle between the given lines.

The angle between the given pairs of lines is given by, \(\cos \theta=\left|\frac{\vec{b}_1 \cdot \vec{b}_2}{\left|\vec{b}_1\right|\left|\vec{b}_2\right|}\right|\)

The given lines are parallel to the vectors, \(\vec{b}_1=3 \hat{i}+2 \hat{j}+6 \hat{k}\) and \(\vec{b}_2=\hat{i}+2 \hat{j}+2 \hat{k}\) respectively.

∴ \(\left|\vec{b}_1\right|=\sqrt{3^2+2^2+6^2}=7\)

∴ \(\left|\vec{b}_2\right|=\sqrt{(1)^2+(2)^2+(2)^2}=3\)

⇒ \(\vec{b}_1 \cdot \vec{b}_2=(3 \hat{i}+2 \hat{j}+6 \hat{k})(\hat{i}+2 \hat{j}+2 \hat{k})=3 \times 1+2 \times 2+6 \times 2=3+4+12=19\)

⇒ \(\cos \theta=\frac{19}{7 \times 3} \Rightarrow \theta=\cos ^{-1}\left(\frac{19}{21}\right)\)

2. The given lines are parallel to the vectors, \(\vec{b}_1=\hat{i}-\hat{j}-2 \hat{k}\) and \(\vec{b}_2=3 \hat{i}-5 \hat{j}-4 \hat{k}\) respectively

∴ \(\left|\vec{b}_1\right|=\sqrt{(1)^2+(-1)^2+(-2)^2}=\sqrt{6}\)

∴ \(\left|\vec{b}_2\right|=\sqrt{(3)^2+(-5)^2+(-4)^2}=\sqrt{50}=5 \sqrt{2}\)

∴ \(\vec{b}_1 \vec{b}_2=(\hat{i}-\hat{j}-2 \hat{k})(3 \hat{i}-5 \hat{j}-4 \hat{k})=1.3-1(-5)-2(-4)=3+5+8=16\)

⇒ \(\cos \theta=\left|\frac{\vec{b}_1, \vec{b}_2}{\left|\vec{b}_1\right|\left|\vec{b}_2\right|}\right|\)

⇒ \(\cos \theta=\frac{16}{\sqrt{6} \cdot 5 \sqrt{2}}=\frac{16}{\sqrt{2} \cdot \sqrt{3} \cdot 5 \sqrt{2}}=\frac{16}{10 \sqrt{3}} \Rightarrow \cos \theta=\frac{8}{5 \sqrt{3}} \Rightarrow \theta=\cos ^{-1}\left(\frac{8}{5 \sqrt{3}}\right)\)

Question 9. Find the angle between the following pairs of lines :

  1. \(\frac{x-2}{2}=\frac{y-1}{5}=\frac{z+3}{-3}\) and \(\frac{x+2}{-1}=\frac{y-4}{8}=\frac{z-5}{4}\)
  2. \(\frac{x}{2}=\frac{y}{2}=\frac{z}{1}\) and \(\frac{x-5}{4}=\frac{y-2}{1}=\frac{z-3}{8}\)

Solution:

1. Let \(\vec{b}_1\) and \(\vec{b}_2\) be the vectors parallel to the pair of lines, \(\frac{x-2}{2}=\frac{y-1}{5}=\frac{z+3}{-3}\) and \(\frac{x+2}{-1}=\frac{y-4}{8}=\frac{z-5}{4}\), respectively

⇒ \(\vec{b}_1=2 \hat{i}+5 \hat{j}-3 \hat{k}\) and \(\vec{b}_2=-\hat{i}+8 \hat{j}+4 \hat{k}\)

∴ \(\left|\vec{b}_1\right|=\sqrt{(2)^2+(5)^2+(-3)^2}=\sqrt{38}\)

∴ \(\left|\vec{b}_2\right|=\sqrt{(-1)^2+(8)^2+(4)^2}=\sqrt{81}=9\)

∴ \(\vec{b}_1 \vec{b}_2=(2 \hat{i}+5 \hat{j}-3 \hat{k}) \cdot(-\hat{i}+8 \hat{j}+4 \hat{k})=2(-1)+5 \times 8+(-3) \cdot 4=-2+40-12=26\)

The angle \(\theta\) between the given pair of lines is given by the relation,

⇒ \(\cos \theta=\left|\frac{\vec{b}_1, \vec{b}_2}{\left|\overrightarrow{\mathrm{b}}_1\right|\left|\overrightarrow{\mathrm{b}}_2\right|}\right| \Rightarrow \cos \theta=\frac{26}{9 \sqrt{38}} \Rightarrow \theta=\cos ^{-1}\left(\frac{26}{9 \sqrt{38}}\right)\)

2. Let \(\vec{b}_1\) and \(\vec{b}_2\) be the vectors parallel to the given pair of lines, \(\frac{x}{2}=\frac{y}{2}=\frac{z}{1}\) and \(\frac{x-5}{4}=\frac{y-2}{1}=\frac{z-3}{8}\), respectively.

⇒ \(\vec{b}_1=2 \hat{i}+2 \hat{j}+\hat{k} \text { and } \vec{b}_2=4 \hat{i}+\hat{j}+8 \hat{k}\)

⇒ \(\left|\vec{b}_1\right|=\sqrt{(2)^2+(2)^2+(1)^2}=\sqrt{9}=3\)

⇒ \(\left|\vec{b}_2\right|=\sqrt{4^2+1^2+8^2}=\sqrt{81}=9\)

⇒ \(\vec{b}_1, \vec{b}_2=(2 \hat{i}+2 \hat{j}+\hat{k}) \cdot(4 \hat{i}+\hat{j}+8 \hat{k})=2 \times 4+2 \times 1+1 \times 8=8+2+8=18\)

If \(\theta\) is the angle between the given pair of lines, then

∴ \(\cos \theta=\left|\frac{\vec{b}_1 \cdot \vec{b}_2}{\left|\vec{b}_1\right| \mid \vec{b}_2}\right| \Rightarrow \cos \theta=\frac{18}{3 \times 9}=\frac{2}{3} \Rightarrow \theta=\cos ^{-1}\left(\frac{2}{3}\right)\)

Question 10. Find the values of p so that the lines \(\frac{1-x}{3}=\frac{7 y-14}{2 p}=\frac{z-3}{2} \text { and } \frac{7-7 x}{3 p}=\frac{y-5}{1}=\frac{6-z}{5}\) are at right angles.
Solution:

The given equations can be written in the standard form as \(\frac{x-1}{-3}=\frac{y-2}{\frac{2 p}{7}}=\frac{z-3}{2} \text { and } \frac{x-1}{\frac{-3 p}{7}}=\frac{y-5}{1}=\frac{z-6}{-5}\)

The direction ratios of the lines are – 3, 2p/7, 2, and -3p/7,1,-5 respectively.

Two lines with direction ratios, a1, b1, c1 and a2, b2, c2 are perpendicular to each other, if a1a2 + b1b2 + c1c2 = 0

∴ (-3) \(\cdot\left(\frac{-3 p}{7}\right)+\left(\frac{2 p}{7}\right) \cdot(1)+2(-5)=0\)

⇒ \(\frac{9 p}{7}+\frac{2 p}{7}=10 \quad \Rightarrow 11 p=70 \quad \Rightarrow p=\frac{70}{11}\)

Thus, the value of p is \(\frac{70}{11}\)

Question 11. Show that the lines \(\frac{x-5}{7}=\frac{y+2}{-5}=\frac{z}{1} \text { and } \frac{x}{1}=\frac{y}{2}=\frac{z}{3}\) are perpendicular to each other.
Solution:

The equations of the given lines are \(\frac{x-5}{7}=\frac{y+2}{-5}=\frac{z}{1} \text { and } \frac{x}{1}=\frac{y}{2}=\frac{z}{3}\)

The direction ratios of the given lines are 7, -5, 1, and 1, 2, and 3 respectively.

Two lines with direction ratios, a1, b1, c1 and a2, b2, c2 are perpendicular to each other, if a1a2+ b1b2+ c1c2= 0

∴ 7 x 1 + (-5) x 2+1 x 3 = 7- 10 + 3 = 0

Therefore, the given lines are perpendicular to each other.

Question 12. Find the shortest distance between the lines: \(\overrightarrow{\mathrm{r}}=(\hat{\mathrm{i}}+2 \hat{\mathrm{j}}+\hat{\mathrm{k}})+\lambda(\hat{\mathrm{i}}-\hat{\mathrm{j}}+\hat{\mathrm{k}}) \text { and } \overrightarrow{\mathrm{r}}=(2 \hat{\mathrm{i}}-\hat{\mathrm{j}}-\hat{\mathrm{k}})+\mu(2 \hat{\mathrm{i}}+\hat{\mathrm{j}}+2 \hat{\mathrm{k}})\)
Solution:

The equations of the given lines are \(\overrightarrow{\mathrm{r}}=(\hat{\mathrm{i}}+2 \hat{\mathrm{j}}+\hat{\mathrm{k}})+\lambda(\hat{\mathrm{i}}-\hat{\mathrm{j}}+\hat{\mathrm{k}}) \text { and } \overrightarrow{\mathrm{r}}=(2 \hat{\mathrm{i}}-\hat{\mathrm{j}}-\hat{\mathrm{k}})+\mu(2 \hat{\mathrm{i}}+\hat{\mathrm{j}}+2 \hat{\mathrm{k}})\)

It is known that the shortest distance between the lines, \(\vec{r}=\vec{a}_1+\lambda \vec{b}_1\) and \(\vec{r}=\vec{a}_2+\mu \vec{b}_2\) is given by

d = \(\left|\frac{\left(\vec{b}_1 \times \vec{b}_2\right) \cdot\left(\vec{a}_2-\vec{a}_1\right)}{\left|\vec{b}_1 \times \vec{b}_2\right|}\right|\)……(1)

Comparing the given equations, we obtain

∴ \(\vec{a}_1=\hat{i}+2 \hat{j}+\hat{k}, \vec{b}_1=\hat{i}-\hat{j}+\hat{k}, \vec{a}_2=2 \hat{i}-\hat{j}-\hat{k}, \vec{b}_2=2 \hat{i}+\hat{j}+2 \hat{k}\)

⇒ \(\vec{a}_2-\vec{a}_1=(2 \hat{i}-\hat{j}-\hat{k})-(\hat{i}+2 \hat{j}+\hat{k})=\hat{i}-3 \hat{j}-2 \hat{k}\)

⇒ \(\overrightarrow{\mathrm{b}}_1 \times \overrightarrow{\mathrm{b}}_2\)

= \(\left|\begin{array}{ccc}
\hat{\mathrm{i}} & \hat{\mathrm{j}} & \hat{\mathrm{k}} \\
1 & -1 & 1 \\
2 & 1 & 2
\end{array}\right|\)

⇒ \(\overrightarrow{\mathrm{b}}_1 \times \overrightarrow{\mathrm{b}}_2=(-2-1) \hat{\mathrm{i}}-(2-2) \hat{\mathrm{j}}+(1+2) \hat{\mathrm{k}}=-3 \hat{\mathrm{i}}+3 \hat{\mathrm{k}}\)

⇒ \(\left|\overrightarrow{\mathrm{b}}_1 \times \overrightarrow{\mathrm{b}}_2\right|=\sqrt{(-3)^2+(3)^2}=\sqrt{9+9}=\sqrt{18}=3 \sqrt{2}\)

Substituting all the values in equation (1), we obtain

d = \(\left|\frac{(-3 \hat{i}+3 \hat{k}) \cdot(\hat{i}-3 \hat{j}-2 \hat{k})}{3 \sqrt{2}}\right| \Rightarrow d=\left|\frac{-3 \cdot 1+3(-2)}{3 \sqrt{2}}\right| \Rightarrow d=\left|\frac{-9}{3 \sqrt{2}}\right|\)

⇒ d = \(\frac{3}{\sqrt{2}}=\frac{3 \times \sqrt{2}}{\sqrt{2} \times \sqrt{2}}=\frac{3 \sqrt{2}}{2}\)

Therefore, the shortest distance between the two lines is \(\frac{3 \sqrt{2}_2^{-}}{2}\) units.

Question 13. Find the shortest distance between the lines \(\frac{x+1}{7}=\frac{y+1}{-6}=\frac{z+1}{1}\) and \(\frac{x-3}{1}=\frac{y-5}{-2}=\frac{z-7}{1}\)
Solution:

The given lines are \(\frac{x+1}{7}=\frac{y+1}{-6}=\frac{z+1}{1}\) and \(\frac{x-3}{1}=\frac{y-5}{-2}=\frac{z-7}{1}\)

It is known that the shortest distance between the two lines,
\(\frac{x-x_1}{a_1}=\frac{y-y_1}{b_1}=\frac{z-z_1}{c_1}\) and \(\frac{x-x_2}{a_2}=\frac{y-y_2}{b_2}=\frac{z-z_2}{c_2}\) is given as :

d = \(\frac{\left|\begin{array}{ccc}
x_2-x_1 & y_2-y_1 & z_2-z_1 \\
a_1 & b_1 & c_1 \\
a_2 & b_2 & c_2
\end{array}\right|}{\sqrt{\left(b_1 c_2-b_2 c_1\right)^2+\left(c_1 a_2-c_2 a_1\right)^2+\left(a_1 b_2-a_2 b_1\right)^2}}\)…..(1)

Comparing the given equations, we obtain

⇒ \(x_1=-1, y_1=-1, z_1=-1 ; x_2=3, y_2=5, z_2=7\)

⇒ \(a_1=7, b_1=-6, c_1=1 ; a_2=1, b_2=-2, c_2=1\)

Then, \(\left|\begin{array}{ccc}x_2-x_1 & y_2-y_1 & z_2-z_1 \\ a_1 & b_1 & c_1 \\ a_2 & b_2 & c_2\end{array}\right|=\left|\begin{array}{ccc}4 & 6 & 8 \\ 7 & -6 & 1 \\ 1 & -2 & 1\end{array}\right|\)

= \(4(-6+2)-6(7-1)+8(-14+6)=-16 -36-64=-116\)

⇒ \(\sqrt{\left(b_1 c_2-b_2 c_1\right)^2+\left(c_1 a_2-c_2 a_1\right)^2+\left(a_1 b_2-a_2 b_1\right)^2}=\sqrt{(-6+2)^2+(1-7)^2+(-14+6)^2}\)

= \(\sqrt{16+36+64}=\sqrt{116}=2 \sqrt{29}\)

Substituting all the values in equation (1), we obtain

d = \(\left|\frac{-116}{\sqrt{116}}\right|=\frac{116}{\sqrt{116}}=2 \sqrt{29}\)

Since distance is always non-negative, the distance between the given lines is \(2 \sqrt{29}\) units.

Question 14. Find the shortest distance between the lines whose vector equations are: \(\vec{r}=\hat{i}+2 \hat{j}+3 \hat{k}+\lambda(\hat{i}-3 \hat{j}+2 \hat{k})\) and \(\vec{r}=4 \hat{i}+5 \hat{j}+6 \hat{k}+\mu(2 \hat{i}+3 \hat{j}+\hat{k})\)
Solution:

The given lines are \(\vec{r}=\hat{i}+2 \hat{j}+3 \hat{k}+\lambda(\hat{i}-3 \hat{j}+2 \hat{k})\) and \(\vec{r}=4 \hat{i}+5 \hat{j}+6 \hat{k}+\mu(2 \hat{i}+3 \hat{j}+\hat{k})\)

It is known that the shortest distance between the lines, \(\vec{r}=\vec{a}_1+\lambda \vec{b}_1\) and \(\vec{r}=\vec{a}_2+\mu \vec{b}_2\) is given by,

d = \(\left|\frac{\left.\overrightarrow{\mathrm{b}}_1 \times \overrightarrow{\mathrm{b}}_2\right) \cdot\left(\overrightarrow{\mathrm{a}}_2-\overrightarrow{\mathrm{a}}_1\right)}{\left|\overrightarrow{\mathrm{b}}_1 \times \overrightarrow{\mathrm{b}}_2\right|}\right|\)…….(1)

Comparing the given equations with \(\overrightarrow{\mathrm{r}}=\overrightarrow{\mathrm{a}}_1+\lambda \overrightarrow{\mathrm{b}}_1\) and \(\overrightarrow{\mathrm{r}}=\overrightarrow{\mathrm{a}}_2+\mu \overrightarrow{\mathrm{b}}_2\); we have:

⇒ \(\vec{a}_1=\hat{i}+2 \hat{j}+3 \hat{k} ; \vec{a}_2=4 \hat{i}+5 \hat{j}+6 \hat{k}\)

⇒ \(\vec{b}_1=\hat{i}-3 \hat{j}+2 \hat{k} ; \vec{b}_2=2 \hat{i}+3 \hat{j}+\hat{k}\)

⇒ \(\vec{a}_2-\vec{a}_1=(4 \hat{i}+5 \hat{j}+6 \hat{k})-(\hat{i}+2 \hat{j}+3 \hat{k})=3 \hat{i}+3 \hat{j}+3 \hat{k}\)

⇒ \(\vec{b}_1 \times \vec{b}_2\)

= \(\left|\begin{array}{ccc}
\hat{i} & \hat{j} & \hat{k} \\
1 & -3 & 2 \\
2 & 3 & 1
\end{array}\right|\)

= \((-3-6 \hat{i}-(1-4) \hat{j}+(3+6) \hat{k}=-9 \hat{i}+3 \hat{j}+9 \hat{k}\)

⇒ \(\left|\vec{b}_1 \times \vec{b}_2\right|=\sqrt{(-9)^2+(3)^2+(9)^2}=\sqrt{81+9+81}=\sqrt{171}=3 \sqrt{19}\)

⇒ \(\left(\vec{b}_1 \times \vec{b}_2\right) \cdot\left(\vec{a}_2-\vec{a}_1\right)=(-9 \hat{i}+3 \hat{j}+9 \hat{k}) \cdot(3 \hat{i}+3 \hat{j}+3 \hat{k})=-9 \times 3+3 \times 3+9 \times 3=9\)

Substituting all the values in equation (1), we obtain

d = \(\left|\frac{9}{3 \sqrt{19}}\right|=\frac{3}{\sqrt{19}}\)

Therefore, the shortest distance between the two given lines is \(\frac{3}{\sqrt{19}}\) units.

Question 15. Find the shortest distance between the lines whose vector equations are \(\vec{r}=(1-t) \hat{i}+(t-2) \hat{j}+(3-2 t) \hat{k}\) and \(\vec{r}=(s+1) \hat{i}+(2 s-1) \hat{j}-(2 s+1) \hat{k}\)
Solution:

The given lines are

⇒ \(\vec{r}=(1-t) \hat{i}+(t-2) \hat{j}+(3-2 t) \hat{k}\)

⇒ \(\vec{r}=(\hat{i}-2 \hat{j}+3 \hat{k})+t(-\hat{i}+\hat{j}-2 \hat{k})\)….(1)

⇒ \(\vec{r}=(s+1) \hat{i}+(2 s-1) \hat{j}-(2 s+1) \hat{k}\)

⇒ \(\vec{r}=(\hat{i}-\hat{j}-\hat{k})+s(\hat{i}+2 \hat{j}-2 \hat{k})\)…..(2)

It is known that the shortest distance between the lines, \(\vec{r}=\vec{a}_1+\lambda \vec{b}_1\) and \(\vec{r}=\vec{a}_2+\mu \vec{b}_2\) is given by

d = \(\left|\frac{\left(\vec{b}_1 \times \vec{b}_2\right) \cdot\left(\vec{a}_2-\vec{a}_1\right)}{\left|\vec{b}_1 \times \vec{b}_2\right|}\right|\)……..(3)

For the given equations,

⇒ \(\vec{a}_1=\hat{i}-2 \hat{j}+3 \hat{k} ; \vec{a}_2=\hat{i}-\hat{j}-\hat{k}\)

⇒ \(\vec{b}_1=-\hat{i}+\hat{j}-2 \hat{k} ; \vec{b}_2=\hat{i}+2 \hat{j}-2 \hat{k}\)

⇒ \(\vec{a}_2-\vec{a}_1=(\hat{i}-\hat{j}-\hat{k})-(\hat{i}-2 \hat{j}+3 \hat{k})=\hat{j}-4 \hat{k}\)

⇒ \(\vec{b}_1 \times \vec{b}_2\)

= \(\left|\begin{array}{ccc}
\hat{i} & \hat{j} & \hat{k} \\
-1 & 1 & -2 \\
1 & 2 & -2
\end{array}\right|\)

= \((-2+4) \hat{i}-(2+2) \hat{j}+(-2-1) \hat{k}=2 \hat{i}-4 \hat{j}-3 \hat{k}\)

⇒ \(\left|\vec{b}_1 \times \vec{b}_2\right|=\sqrt{(2)^2+(-4)^2+(-3)^2}=\sqrt{4+16+9}=\sqrt{29}\)

∴ \(\left(\vec{b}_1 \times \vec{b}_2\right) \cdot\left(\vec{a}_2-\vec{a}_1\right)=(2 \hat{i}-4 \hat{j}-3 \hat{k}) \cdot(\hat{j}-4 \hat{k})=-4+12=8\)

Substituting all the values in equation (3), we, obtain \(\mathrm{d}=\left|\frac{8}{\sqrt{29}}\right|=\frac{8}{\sqrt{29}}\)

Therefore, the shortest distance between the lines is \(\frac{8}{\sqrt{29}}\) units.

Three-Dimensional Geometry Miscellaneous Exercise

Question 1. Find the angle between the lines whose direction ratios are (a, b, c) and (b – c, c – a, a -b).
Solution:

The angle θ between the lines with direction cosines, (a, b, c) and (b – c, c – a, a -b), is given by,

cos \(\theta=\left|\frac{a_1 a_2+b_1 b_2+c_1 c_2}{\sqrt{a_1^2+b_1^2+c_1^2}+\sqrt{a_2^2+b_2^2+c_2^2}}\right| \Rightarrow \cos \theta=\left|\frac{a(b-c)+b(c-a)+c(a-b)}{\sqrt{a^2+b^2+c^2}+\sqrt{(b-c)^2+(c-a)^2+(a-b)^2}}\right|\)

⇒cos \(\theta=0 \Rightarrow \theta=\cos ^{-1}(0) \Rightarrow \theta=90^{\circ}\)

Thus, the angle between the lines is 90°.

Question 2. Find the equation of a line parallel to the x-axis and passing through the origin.
Solution:

The line parallel to the x-axis and passing through the origin is the x-axis itself.

Let A be a point on the x-axis. Therefore, the coordinates of A are given by (a, 0, 0), where a ∈ R.

The direction ratios of OA are (a – 0), (0 – 0), (0 – 0) i.e. a, 0, 0

The equation of OA is given by \(\frac{x-0}{a}=\frac{y-0}{0}=\frac{z-0}{0} \Rightarrow \frac{x}{1}=\frac{y}{0}=\frac{z}{0}=a\)

Thus, the equation of the line parallel to the x-axis and passing through the origin is \(\frac{x}{1}=\frac{y}{0}=\frac{z}{0}\)

Question 3. If the lines \(\frac{x-1}{-3}=\frac{y-2}{2 k}=\frac{z-3}{2}\) and \(\frac{x-1}{3 k}=\frac{y-1}{1}=\frac{z-6}{-5}\) are perpendicular, find the value of k.
Solution:

The direction ratios of the lines, \(\frac{x-1}{-3}=\frac{y-2}{2 k}=\frac{z-3}{2}\) and \(\frac{x-1}{3 k}=\frac{y-1}{1}=\frac{z-6}{-5}\), are -3,2 k, 2 and 3 k, 1,-5 respectively.

It is known that two lines with direction ratios, \(\mathrm{a}_1, \mathrm{~b}_1, \mathrm{c}_1\) and \(\mathrm{a}_2, \mathrm{~b}_2, \mathrm{c}_2\), are perpendicular if \(a_1 a_2+b_1 b_2+c_1 c_2=0\)

∴ -3(3k) + 2kx 1 + 2(-5) = 0 => ~9k + 2k-10 = 0 => 7k=-10 => k = -10/7

Question 4. Find the shortest distance between lines \(\overrightarrow{\mathrm{r}}=6 \hat{i}+2 \hat{j}+2 \hat{k}+\lambda(\hat{i}-2 \hat{j}+2 \hat{k})\) and \(\overrightarrow{\mathrm{r}}=-4 \hat{\mathrm{i}}-\hat{\mathrm{k}}+\mu(3 \hat{\mathrm{i}}-2 \hat{\mathrm{j}}-2 \hat{\mathrm{k}})\)
Solution:

The given lines are \(\vec{r}=6 \hat{i}+2 \hat{j}+2 \hat{k}+\lambda(\hat{i}-2 \hat{j}+2 \hat{k})\)…..(1)

⇒ \(\vec{r}=-4 \hat{i}-\hat{k}+\mu(3 \hat{i}-2 \hat{j}-2 \hat{k})\)….(2)

It is known that the shortest distance between two lines, \(\vec{r}=\vec{a}_1+\lambda \vec{b}_1\), and \(\vec{r}=\vec{a}_2+\mu \vec{b}_2\) is given by

d = \(\left|\frac{\left(\overrightarrow{\mathrm{b}}_1 \times \overrightarrow{\mathrm{b}}_2\right) \cdot\left(\overrightarrow{\mathrm{a}}_2-\overrightarrow{\mathrm{a}}_1\right)}{\left|\overrightarrow{\mathrm{b}}_1 \times \overrightarrow{\mathrm{b}}_2\right|}\right|\)……(3)

Comparing \(\overrightarrow{\mathrm{r}}=\overrightarrow{\mathrm{a}}_1+\lambda \overrightarrow{\mathrm{b}}_1\) and \(\overrightarrow{\mathrm{r}}=\overrightarrow{\mathrm{a}}_2+\mu \overrightarrow{\mathrm{b}}_2\) to equations (1) and (2), we obtain

⇒ \(\vec{a}_1=6 \hat{i}+2 \hat{j}+2 \hat{k}, \vec{b}_1=\hat{i}-2 \hat{j}+2 \hat{k}, \vec{a}_2=-4 \hat{i}-\hat{k}, \vec{b}_2=3 \hat{i}-2 \hat{j}-2 \hat{k}\)

⇒ \(\vec{a}_2-\vec{a}_1=(-4 \hat{i}-\hat{k})-(6 \hat{i}+2 \hat{j}+2 \hat{k})=-10 \hat{i}-2 \hat{j}-3 \hat{k}\)

⇒ \(\vec{b}_1 \times \vec{b}_2=\left|\begin{array}{ccc}
i & \hat{j} & \hat{k} \\
1 & -2 & 2 \\
3 & -2 & -2
\end{array}\right|\)

= \((4+4) \hat{i}-(-2-6) \hat{j}+(-2+6) \hat{k}=8 \hat{i}+8 \hat{j}+4 \hat{k}\)

⇒ \(\left|\vec{b}_1 \times \vec{b}_2\right|=\sqrt{(8)^2+(8)^2+(4)^2}=12\)

⇒ \(\left(\vec{b}_1 \times \vec{b}_2\right) \cdot\left(\vec{a}_2-\vec{a}_1\right)=(8 \hat{i}+8 \hat{j}+4 \hat{k}) \cdot(-10 \hat{i}-2 \hat{j}-3 \hat{k})=-80-16-12=-108\)

Substituting all the values in equation (3), we obtain \(\mathrm{d}=\left|\frac{-108}{12}\right|=9\) units

Therefore, the shortest distance between the two given lines is 9 units.

Question 5. Find the vector equation of the line passing through the point (1, 2, -4) and perpendicular to the two lines: \(\frac{x-8}{3}=\frac{y+19}{-16}=\frac{z-10}{7}\) and \(\frac{x-15}{3}=\frac{y-29}{8}=\frac{z-5}{-5}\)
Solution:

Let the required line be parallel to the vector \(\vec{b}\) given by, \(\vec{b}=b_1 \hat{i}+b_2 \hat{j}+b_3 \hat{k}\)

The position vector of the point (1,2,-4) is \(\vec{a}=\hat{i}+2 \hat{j}-4 \hat{k}\)

The equation of the line passing through (1,2,-4) and parallel to vector \(\vec{b}\) is \(\overrightarrow{\mathrm{r}}=\overrightarrow{\mathrm{a}}+\lambda \overrightarrow{\mathrm{b}} \Rightarrow \overrightarrow{\mathrm{r}}=(\hat{\mathrm{i}}+2 \hat{\mathrm{j}}-4 \hat{\mathrm{k}})+\lambda\left(\mathrm{b}_1 \hat{\mathrm{i}}+\mathrm{b}_2 \hat{\mathrm{j}}+\mathrm{b}_3 \hat{\mathrm{k}}\right)\)……..(1)

The equations of the lines are \(\frac{x-8}{3}=\frac{y+19}{-16}=\frac{z-10}{7}\)…..(2)

and \(\frac{x-15}{3}=\frac{y-29}{8}=\frac{z-5}{-5}\)…..(3)

Lines (1) and line (2) are perpendicular to each other \(3 b_1-16 b_2+7 b_3=0\)…..(4)

Also, lines (1) and line (3) are perpendicular to each other. \(3 b_1+8 b_2-5 b_3=0\)…..(5)

From equations (4) and (5), we obtain

\(\frac{b_1}{(-16)(-5)-8 \times 7}=\frac{b_2}{7 \times 3-3(-5)}=\frac{b_3}{3 \times 8-3(-16)} \Rightarrow \frac{b_1}{24}=\frac{b_2}{36}=\frac{b_3}{72} \Rightarrow \frac{b_1}{2}=\frac{b_2}{3}=\frac{b_3}{6}\)

∴ Direction ratios of \(\overrightarrow{\mathrm{b}}\) are 2,3, and 6.

∴ \(\vec{b}=2 \hat{i}+3 \hat{j}+6 \hat{k}\)

Substituting \(\vec{b}=2 \hat{i}+3 \hat{j}+6 \hat{k}\) in equation (1), we obtain \(\vec{r}=(\hat{i}+2 \hat{j}-4 \hat{k})+\lambda(2 \hat{i}+3 \hat{j}+6 \hat{k})\)

This is the equation of the required line.

 

Vector Algebra Class 12 Maths Important Questions Chapter 10

Vector Algebra Exercise 10.1

Question 1. Represent graphically a displacement of 40 km, 30° east of north.
Solution:

Vector Algebra Gaphicall Representation Of Distance Of 40 km

Here, \(\overrightarrow{\mathrm{OP}}\) vector represents the displacement of 40 km, 30° East of North.

Question 2. Classify the following measures as scalars and vectors.

  1. 10kg
  2. 2 metres north-west
  3. 40°
  4. 40 watt
  5. 10-19
  6.  coulomb 20 m/s²

Solution:

  1. 10 kg is a scalar quantity because it involves only magnitude.
  2. 2 meters northwest is a vector quantity as it involves both magnitude and direction.
  3. 40° is a scalar quantity as it involves only magnitude.
  4. 40 watts is a scalar quantity as it involves only magnitude.
  5. 10-19 coulomb is a scalar quantity as it involves only magnitude.
  6. 20 m/s² is a vector quantity as it involves magnitude as well as direction.

Question 3. Classify the following as scalar and vector quantities.

  1. Time period
  2. Distance
  3. Force
  4. Velocity
  5. Work done

Solution:

  1. Time period is a scalar quantity as it involves only magnitude.
  2. Distance is a scalar quantity as it involves only magnitude.
  3. Force is a vector quantity as it involves both magnitude and direction.
  4. Velocity is a vector quantity as it involves both magnitude as well as direction.
  5. Work done is a scalar quantity as it involves only magnitude.

Question 4. Identify the following vectors,

  1. Coinitial
  2. Equal
  3. Collinear but not equal

Solution:

Vector Algebra

  1. Vectors \(\vec{a}\) and \(\vec{d}\) are coinitial because they have the same initial point.
  2. Vectors \(\vec{b}\) and \(\vec{d}\) are equal because they have the same magnitude and direction.
  3. Vectors \(\vec{a}\) and \(\vec{c}\) are collinear but not equal. This is because although they are parallel, their directions are not the same.

Read and Learn More Class 12 Maths Chapter Wise with Solutions

Question 5. Answer the following as true or false.

  1. \(\vec{\mathrm{a}}\) and –\(\vec{\mathrm{a}}\) are collinear.
  2. Two collinear vectors are always equal in magnitude.
  3. Two vectors having the same magnitude are collinear.
  4. Two collinear vectors having the same magnitude are equal.

Solution:

  1. True, Vectors \(\vec{\mathrm{a}}\) and –\(\vec{\mathrm{a}}\) are parallel to the same line.
  2. False, Collinear vectors are those vectors that are parallel to the same line.
  3. False, Two vectors having the same magnitude need not necessarily be parallel to the same line,
  4. False, Only if the magnitude and direction of two vectors are the same regardless of the positions of their initial points, the two vectors are said to be equal.

Vector Algebra Exercise 10.2

Question 1. Compute the magnitude of the following vectors: \(\vec{a}=\hat{i}+\hat{j}+\hat{k} ; \vec{b}=2 \hat{i}-7 \hat{j}-3 \hat{k} ; \vec{c}=\frac{1}{\sqrt{3}} \hat{i}+\frac{1}{\sqrt{3}} \hat{j}-\frac{1}{\sqrt{3}} \hat{k}\)
Solution:

The given vectors are: \(\vec{a}=\hat{i}+\hat{j}+\hat{k} ; \vec{b}=2 \hat{i}-7 \hat{j}-3 \hat{k} ; \vec{c}=\frac{1}{\sqrt{3}} \hat{i}+\frac{1}{\sqrt{3}} \hat{j}-\frac{1}{\sqrt{3}} \hat{k}\)

⇒ \(|\vec{a}|=\sqrt{(1)^2+(1)^2+(1)^2}=\sqrt{3}\)

⇒ \(|\vec{b}|=\sqrt{(2)^2+(-7)^2+(-3)^2}=\sqrt{4+49+9}=\sqrt{62}\)

⇒ \(|\vec{c}|=\sqrt{\left(\frac{1}{\sqrt{3}}\right)^2+\left(\frac{1}{\sqrt{3}}\right)^2+\left(-\frac{1}{\sqrt{3}}\right)^2}=\sqrt{\frac{1}{3}+\frac{1}{3}+\frac{1}{3}}=1\)

Question 2. Write two different vectors having the same magnitude.
Solution:

Consider \(\overrightarrow{\mathrm{a}}=(\hat{\mathrm{i}}-2 \hat{\mathrm{j}}+3 \hat{\mathrm{k}})\) and \(\overrightarrow{\mathrm{b}}=(2 \hat{\mathrm{i}}+\hat{\mathrm{j}}-3 \hat{\mathrm{k}})\).

It can be observed that \(|\overrightarrow{\mathrm{a}}|=\sqrt{1^2+(-2)^2+3^2}=\sqrt{1+4+9}=\sqrt{14}\)

and \(|\vec{b}|=\sqrt{2^2+1^2+(-3)^2}=\sqrt{4+1+9}=\sqrt{14}\)

Hence, \(\vec{a}\) and \(\vec{b}\) are two different vectors having the same magnitude. The vectors are different because they have different directions.

Question 3. Write two different vectors having the same direction.
Solution:

Consider \(\vec{a}=(\hat{i}+\hat{j}+\hat{k})\) and \(\vec{b}=(2 \hat{i}+2 \hat{j}+2 \hat{k})\).

The direction consines of \(\vec{a}\) are given by,

l = \(\frac{1}{\sqrt{1^2+1^2+1^2}}=\frac{1}{\sqrt{3}}, \mathrm{~m}=\frac{1}{\sqrt{1^2+1^2+1^2}}=\frac{1}{\sqrt{3}}, \mathrm{n}=\frac{1}{\sqrt{1^2+1^2+1^2}}=\frac{1}{\sqrt{3}}\).

The direction cosines of \(\vec{b}\) are given by

l = \(\frac{2}{\sqrt{2^2+2^2+2^2}}=\frac{2}{2 \sqrt{3}}=\frac{1}{\sqrt{3}}, \mathrm{~m}=\frac{2}{\sqrt{2^2+2^2+2^2}}=\frac{2}{2 \sqrt{3}}=\frac{1}{\sqrt{3}}, \mathrm{n}=\frac{2}{\sqrt{2^2+2^2+2^2}}=\frac{2}{2 \sqrt{3}}=\frac{1}{\sqrt{3}}\)

The direction cosines of \(\vec{a}\) and \(\vec{b}\) are the same. Hence, the two vectors have the same direction.

Question 4. Find the values of x and y so that the vectors \(2 \hat{\mathrm{i}}+3 \hat{\mathrm{j}} \text { and } x \hat{\mathrm{i}}+y \hat{\mathrm{j}}\) are equal.
Solution:

The two vectors \(2 \hat{\mathrm{i}}+3 \hat{\mathrm{j}} \text { and } x \hat{\mathrm{i}}+y \hat{\mathrm{j}}\) will be equal if their corresponding scalar components are equal. Hence, the required values of x and y are 2 and 3 respectively.

Question 5. Find the scalar and vector components of the vector with initial point (2,1) and terminal point (-5, 7).
Solution:

The vector with the initial point P (2, 1) and terminal point Q (-5, 7) can be given by, \(\overrightarrow{\mathrm{PQ}}=(-5-2) \hat{\mathrm{i}}+(7-1) \hat{\mathrm{j}} \Rightarrow \overrightarrow{\mathrm{PQ}}=-7 \hat{\mathrm{i}}+6 \hat{\mathrm{j}}\)

Hence, the required scalar components are -7 and 6 while the vector components are \(-7 \hat{\mathrm{i}}+6 \hat{\mathrm{j}}\)

CBSE Class 12 Maths Chapter 10 Vector Algebra Question And Answers

Question 6. Find the sum of the vectors \(\vec{a}=\hat{i}-2 \hat{j}+\hat{k}, \hat{b}=-2 \hat{i}+4 \hat{j}+5 \hat{k}\) and \(\vec{c}=\hat{i}-6 \hat{j}-7 \hat{k}\).
Solution:

The given vectors are \(\vec{a}=\hat{i}-2 \hat{j}+\hat{k}, \hat{b}=-2 \hat{i}+4 \hat{j}+5 \hat{k}\) and \(\vec{c}=\hat{i}-6 \hat{j}-7 \hat{k}\).

∴ \(\vec{a}+\vec{b}+\vec{c}=(1-2+1) \hat{i}+(-2+4-6) \hat{j}+(1+5-7)) \hat{k}=0 \hat{i}-4 \hat{j}-1 \hat{k}=-4 \hat{j}-\hat{k}\)

Question 7. Find the unit vector in the direction of the vector \(\overrightarrow{\mathrm{a}}=\hat{\mathrm{i}}+\hat{\mathrm{j}}+2 \hat{\mathrm{k}}\).
Solution:

The unit vector in the direction of the vector \(\vec{a}=\hat{i}+\hat{j}+2 \hat{k}\) is given by \(\hat{a}=\frac{\vec{a}}{|\vec{a}|}\).

⇒ \(|\vec{a}|=\sqrt{1^2+1^2+2^2}=\sqrt{1+1+4}=\sqrt{6}\)

∴ \(\hat{a}=\frac{\vec{a}}{|\vec{a}|}=\frac{\hat{i}+\hat{j}+2 \hat{k}}{\sqrt{6}}=\frac{1}{\sqrt{6}} \hat{i}+\frac{1}{\sqrt{6}} \hat{j}+\frac{2}{\sqrt{6}} \hat{k}\)

Question 8. Find the unit vector in the direction of the vector \(\overrightarrow{\mathrm{PQ}}\), where P and Q are the points (1, 2, 3) and (4, 5, 6), respectively.
Solution:

The given points are P(l, 2, 3) and Q (4, 5, 6).

∴ \(\overrightarrow{P Q}=(4-1) \hat{i}+(5-2) \hat{j}+(6-3) \hat{k}=3 \hat{i}+3 \hat{j}+3 \hat{k}\)

∴ Magnitude of given vector, \(|\overrightarrow{\mathrm{PQ}}|=\sqrt{3^2+3^2+3^2}=\sqrt{9+9+9}=\sqrt{27}=3 \sqrt{3}\)

Hence, the unit vector in the direction of \(\overrightarrow{\mathrm{PQ}}\) is \(\frac{\overrightarrow{\mathrm{PQ}}}{|\overrightarrow{\mathrm{PQ}}|}=\frac{3 \hat{\mathrm{i}}+3 \hat{\mathrm{j}}+3 \hat{\mathrm{k}}}{3 \sqrt{3}}=\frac{1}{\sqrt{3}} \hat{\mathrm{i}}+\frac{1}{\sqrt{3}} \hat{\mathrm{j}}+\frac{1}{\sqrt{3}} \hat{\mathrm{k}}\)

Question 9. For given vectors, \(\vec{a}=2 \hat{i}-\hat{j}+2 \hat{k}\) and \(\vec{b}=-\hat{i}+\hat{j}-\hat{k}\), find the unit vector in the direction of the vector \(\vec{a}\) + \(\vec{b}\).
Solution:

The given vectors are \(\vec{a}=2 \hat{i}-\hat{j}+2 \hat{k}\) and \(\vec{b}=-\hat{i}+\hat{j}-\hat{k}\).

∴ \(\vec{a}+\vec{b}=(2-1) \hat{i}+(-1+1) \hat{j}+(2-1) \hat{k}=1 \hat{i}++0 \hat{j}+1 \hat{k}=\hat{i}+\hat{k}\)

⇒ \(|\vec{a}+\vec{b}|=\sqrt{1^2+1^2}=\sqrt{2}\)

Hence, the unit vector in the direction of \((\vec{a}+\vec{b})\) is \(\frac{(\vec{a}+\vec{b})}{|\vec{a}+\vec{b}|}=\frac{\hat{i}+\hat{k}}{\sqrt{2}}=\frac{1}{\sqrt{2}} \hat{i}+\frac{1}{\sqrt{2}} \hat{k}\)

Question 10. Find a vector in the direction of the vector \(5 \hat{\mathrm{i}}-\hat{\mathrm{j}}+2 \hat{\mathrm{k}}\) which has magnitude 8 units.
Solution:

Let \(\vec{a}=5 \hat{i}-\hat{j}+2 \hat{k}\)

∴ \(|\vec{a}|=\sqrt{5^2+(-1)^2+2^2}=\sqrt{25+1+4}=\sqrt{30}\)

⇒ \(\hat{a}=\frac{\vec{a}}{|\vec{a}|}=\frac{5 \hat{i}-\hat{j}+2 \hat{k}}{\sqrt{30}}\)

Hence, the vector in the direction of the vector \(5 \hat{i}-\hat{j}+2 \hat{k}\) which has magnitude of 8 units is given by,

8 \(\hat{\mathrm{a}}=8\left(\frac{5 \hat{\mathrm{i}}-\hat{\mathrm{j}}+2 \hat{\mathrm{k}}}{\sqrt{30}}\right)=\frac{40}{\sqrt{30}} \hat{\mathrm{i}}-\frac{8}{\sqrt{30}} \hat{\mathrm{j}}+\frac{16}{\sqrt{30}} \hat{\mathrm{k}}\)

Question 11. Show that the vectors \(2 \hat{i}-3 \hat{j}+4 \hat{k}\) and \(-4 \hat{i}+6 \hat{j}-8 \hat{k}\) are collinear.
Solution:

Let \(\vec{a}=2 \hat{i}-3 \hat{j}+4 \hat{k}\) and \(\vec{b}=-4 \hat{i}+6 \hat{j}-8 \hat{k}\)

It is observed that \(\vec{b}=-4 \hat{i}+6 \hat{j}-8 \hat{k}=-2(2 \hat{i}-3 \hat{j}+4 \hat{k})=-2 \vec{a}\)

∴ \(\overrightarrow{\mathrm{b}}=\lambda \overrightarrow{\mathrm{a}}\)

where, λ=-2,

Hence, the given vectors are collinear.

Question 12. Find the direction cosines of the vector \(\hat{i}+2 \hat{j}+3 \hat{k}\)
Solution:

Let \(\overrightarrow{\mathrm{a}}=\hat{\mathrm{i}}+2 \hat{\mathrm{j}}+3 \hat{\mathrm{k}}\)

∴ \(|\vec{a}|=\sqrt{1^2+2^2+3^2}=\sqrt{1+4+9}=\sqrt{14}\)

∴ \(\hat{a}=\frac{\vec{a}}{|\vec{a}|}\)

Hence, the direction cosines of a are \(\frac{1}{\sqrt{14}}, \frac{2}{\sqrt{14}}, \frac{3}{\sqrt{14}}\).

Question 13. Find the direction cosines of the vector joining the points A (1, 2, -3) and B(-l, -2, 1), directed from A to B.
Solution:

The given points are A (1,2, -3) and B (-1, -2, 1).

∴ \(\overrightarrow{\mathrm{AB}}=(-1-1) \hat{\mathrm{i}}+(-2-2) \hat{\mathrm{j}}+\{1-(-3)\} \hat{\mathrm{k}} \Rightarrow \overrightarrow{\mathrm{AB}}=-2 \hat{\mathrm{i}}-4 \hat{\mathrm{j}}+4 \hat{\mathrm{k}}\)

Direction ratios of \(\overrightarrow{\mathrm{AB}}\) are a = -2, b = -4, c = 4

Now direction cosines of \(\overrightarrow{\mathrm{AB}}\) are :

l \(=\frac{-2}{\sqrt{(-2)^2+(-4)^2+(4)^2}}=\frac{-2}{6}=-\frac{1}{3}, \mathrm{~m}=\frac{-4}{\sqrt{(-2)^2+(-4)^2+(4)^2}}=\frac{-4}{6}=-\frac{2}{3}\)

n = \(\frac{4}{\sqrt{(-2)^2+(-4)^2+(4)^2}}=\frac{4}{6}=\frac{2}{3}\)

Hence, the direction cosines of \(\overrightarrow{\mathrm{AB}}\) are \(-\frac{1}{3},-\frac{2}{3}, \frac{2}{3}\).

Question 14. Show that the vector \(\hat{\mathrm{i}}+\hat{\mathrm{j}}+\hat{\mathrm{k}}\) is equally inclined to the axes, OX, OY, and OZ.
Solution:

Let \(\overrightarrow{\mathrm{a}}=\hat{\mathrm{i}}+\hat{\mathrm{j}}+\hat{\mathrm{k}}\). Direction ratios of \(\overrightarrow{\mathrm{a}}\) are \(\vec{a}=\mathrm{b}=\mathrm{c}=1\)

Now, direction cosines are

l = \(\frac{1}{\sqrt{(1)^2+(1)^2+(1)^2}}=\frac{1}{\sqrt{3}}=\mathrm{m}=\mathrm{n}\)

Therefore, the direction cosines of \(\vec{a}\) are \(\frac{1}{\sqrt{3}}, \frac{1}{\sqrt{3}}, \frac{1}{\sqrt{3}}\).

Now, let \(\alpha, \beta, and \gamma\) be the angles formed by \(\vec{a}\) with the positive directions of x, y, and z axes.

Then, we have \(\cos \alpha=\frac{1}{\sqrt{3}}, \cos \beta=\frac{1}{\sqrt{3}}, \cos \gamma=\frac{1}{\sqrt{3}}, \alpha=\beta=\gamma=\cos ^{-1}\left(\frac{1}{\sqrt{3}}\right)\)

Hence, the given vector is equally inclined to axes OX, OY, and OZ.

Question 15. Find the position vector of a point R which divides the line joining two points P and Q whose position vectors are \(\hat{i}+2 \hat{j}-\hat{k}\) and \(-\hat{i}+\hat{j}+\hat{k}\) respectively, in the ratio 2:1

  1. Internally
  2. Externally

Solution:

Here, \(\overrightarrow{\mathrm{OP}}=\hat{\mathrm{i}}+2 \hat{\mathrm{j}}-\hat{\mathrm{k}}=\overrightarrow{\mathrm{a}}\) (let) and \(\overrightarrow{\mathrm{OQ}}=-\hat{\mathrm{i}}+\hat{\mathrm{j}}+\hat{\mathrm{k}}=\overrightarrow{\mathrm{b}}\) (let), also m=2, n=1 when R divides PQ internally in the ratio 2: 1, then

Vector Algebra Position Of Vector Internally

1. P . V. of \(R=\frac{m \vec{b}+n \vec{a}}{m+n}\)

= \(\frac{2(-\hat{i}+\hat{j}+\hat{k})+1(\hat{i}+2 \hat{j}-\hat{k})}{2+1}=\frac{(-2 \hat{i}+2 \hat{j}+2 \hat{k})+(\hat{i}+2 \hat{j}-\hat{k})}{3}\)

= \(\frac{-\hat{i}+4 \hat{j}+\hat{k}}{3}=-\frac{1}{3} \hat{i}+\frac{4}{3} \hat{j}+\frac{1}{3} \hat{k}\)

Vector Algebra Position Of Vector Externally

2. when R divides PQ externally in the ratio 2: 1 then,

P.V. of R = \(\frac{m \vec{b}-n \vec{a}}{m-n}\)

= \(\frac{2(-\hat{i}+\hat{j}+\hat{k})-1(\hat{i}+2 \hat{j}-\hat{k})}{2-1}\)

= \(-3 \hat{i}+3 \hat{k}\)

Question 16. Find the position vector of the midpoint of the vector joining the points P (2,3,4) and Q (4, 1, – 2).
Solution:

The position vector of P and Q are given by \(\vec{p}=2 \hat{i}+3 \hat{j}+4 \hat{k}\) and \(\vec{q}=4 \hat{i}+\hat{j}-2 \hat{k}\) respectively

∴ P.V. of midpoint of PQ = \(\frac{1}{2}(\vec{p}+\vec{q})\)

= \(\frac{(2 \hat{i}+3 \hat{j}+4 \hat{k})+(4 \hat{i}+\hat{j}-2 \hat{k})}{2}=\frac{6 \hat{i}+4 \hat{j}+2 \hat{k}}{2}=3 \hat{i}+2 \hat{j}+\hat{k}\)

Question 17. Show that the points A, B and C with position vectors, \(\vec{a}=3 \hat{i}-4 \hat{j}-4 \hat{k}, \vec{b}=2 \hat{i}-\hat{j}+\hat{k}\) and \(\vec{c}=\hat{i}-3 \hat{j}-5 \hat{k}\), respectively form the vertices of a right-angled triangle.
Solution:

Position vectors of points A, B, and C are respectively given as: \(\vec{a}=3 \hat{i}-4 \hat{j}-4 \hat{k}, \vec{b}=2 \hat{i}-\hat{j}+\hat{k}\) and \(\vec{c}=\vec{i}-3 \hat{j}-5 \hat{k}\)

∴ \(\overrightarrow{A B}=\vec{b}-\vec{a}=(2-3) \hat{i}+(-1+4) \hat{j}+(1+4) \hat{k}=-\hat{i}+3 \hat{j}+5 \hat{k}\)

⇒ \(\overrightarrow{B C}=\vec{c}-\vec{b}=(1-2) \hat{i}+(-3+1) \hat{j}+(-5-1) \hat{k}=-\hat{i}-2 \hat{j}-6 \hat{k}\)

⇒ \(\overrightarrow{C A}=\vec{a}-\vec{c}=(3-1) \hat{i}+(-4+3) \hat{j}+(-4+5) \hat{k}=2 \hat{i}-\hat{j}+\hat{k}\)

Now, \(\overrightarrow{\mathrm{AB}}+\overrightarrow{\mathrm{BC}}+\overrightarrow{\mathrm{CA}}=(-\hat{\mathrm{i}}+3 \hat{\mathrm{j}}+5 \hat{k})+(-\hat{\mathrm{i}}-2 \hat{\mathrm{j}}-6 \hat{\mathrm{k}})+(2 \hat{\mathrm{i}}-\hat{\mathrm{j}}+\hat{\mathrm{k}})=\overrightarrow{0}\)

∴ A, B, and C are vertices of the triangle

Now, \(|\overrightarrow{\mathrm{AB}}|^2=(-1)^2+3^2+5^2=1+9+25=35\)

⇒ \(|\overrightarrow{B C}|^2=(-1)^2+(-2)^2+(-6)^2=1+4+36=41\)

⇒ \(|\overrightarrow{\mathrm{CA}}|^2=2^2+(-1)^2+1^2=4+1+1=6\)

∴ \(|\overrightarrow{\mathrm{AB}}|^2+|\overrightarrow{\mathrm{CA}}|^2=|\overrightarrow{\mathrm{BC}}|^2=35+6=41\)

Hence, A, B, and C are vertices of a right-angled triangle.

Question 18. In triangle ABC which of the following is not true:

  1. \(\overrightarrow{\mathrm{AB}}+\overrightarrow{\mathrm{BC}}+\overrightarrow{\mathrm{CA}}=\overrightarrow{0}\)
  2. \(\overrightarrow{\mathrm{AB}}+\overrightarrow{\mathrm{BC}}-\overrightarrow{\mathrm{AC}}=\overrightarrow{0}\)
  3. \(\overrightarrow{\mathrm{AB}}+\overrightarrow{\mathrm{BC}}-\overrightarrow{\mathrm{CA}}=\overrightarrow{0}\)
  4. \(\overrightarrow{\mathrm{AB}}-\overrightarrow{\mathrm{CB}}+\overrightarrow{\mathrm{CA}}=\overrightarrow{0}\)

Solution:

Vector Algebra Triangle ABC

On applying the triangle law of addition in the given triangle, we have:

⇒ \(\overrightarrow{\mathrm{AB}}+\overrightarrow{\mathrm{BC}}=\overrightarrow{\mathrm{AC}}\)….(1)

⇒ \(\overrightarrow{\mathrm{AB}}+\overrightarrow{\mathrm{BC}}=-\overrightarrow{\mathrm{CA}} \Rightarrow \overrightarrow{\mathrm{AB}}+\overrightarrow{\mathrm{BC}}+\overrightarrow{\mathrm{CA}}=\overrightarrow{0}\)…..(2)

∴ The equation given in alternative A is true.

⇒ \(\overrightarrow{\mathrm{AB}}+\overrightarrow{\mathrm{BC}}=\overrightarrow{\mathrm{AC}} \Rightarrow \overrightarrow{\mathrm{AB}}+\overrightarrow{\mathrm{BC}}-\overrightarrow{\mathrm{AC}}=\overrightarrow{0}\)

∴ The equation given in alternative 2 is true. From equation (2), we have: \(\overrightarrow{\mathrm{AB}}\overrightarrow{\mathrm{CB}}+\overrightarrow{\mathrm{CA}}=\overrightarrow{0}\)

∴ The equation given in alternative 4 is true.

Now, consider the equation given in alternative C: \(\overrightarrow{\mathrm{AB}}+\overrightarrow{\mathrm{BC}}-\overrightarrow{\mathrm{CA}}=\overrightarrow{0} \Rightarrow \overrightarrow{\mathrm{AB}}+\overrightarrow{\mathrm{BC}}=\overrightarrow{\mathrm{CA}}[latex]
\)

From equation (1) and (3), we have: \(\overrightarrow{\mathrm{AC}}=\overrightarrow{\mathrm{CA}}\)

∴ \(\overrightarrow{\mathrm{AC}}=-\overrightarrow{\mathrm{AC}} \Rightarrow \overrightarrow{\mathrm{AC}}+\overrightarrow{\mathrm{AC}}=\overrightarrow{0} \Rightarrow 2 \overrightarrow{\mathrm{AC}}=\overrightarrow{0} \Rightarrow \overrightarrow{\mathrm{AC}}=\overrightarrow{0}\), which is not true.

Hence, the equation given in alternative C is incorrect.

The correct answer is (3).

Question 19. If \(\vec{a}\) and \(\vec{b}\) are two collinear vectors, then which of the following are incorrect:

  1. \(\vec{b}=\lambda \vec{a}\), for some scalar \(\lambda\)
  2. \(\overrightarrow{\mathrm{a}}= \pm \overrightarrow{\mathrm{b}}\)
  3. The respective components of \(\vec{a}\) and \(\vec{b}\) are proportional
  4. Both the vectors \(\vec{a}\) and \(\vec{b}\) have same direction, but different magnitudes

Solution:

If \(\vec{a}\) and \(\vec{b}\) are two collinear vectors, then they are parallel.

Therefore, we have: \(\vec{b}=\lambda \vec{a}\) (For some scalar \(\lambda\))

If \(\lambda= \pm 1\), then \(\vec{a}= \pm \vec{b}\).

If \(\vec{a}=a_1 \hat{i}+a_2 \hat{j}+a_3 \hat{k}\) and \(\vec{b}=b_1 \hat{i}+b_2 \hat{j}+b_3 \hat{k}\) and \(\vec{b}=\lambda \vec{a}\).

⇒ \(b_1 \hat{i}+b_2 \hat{j}+b_3 \hat{k}=\lambda\left(a_1 \hat{i}+a_2 \hat{j}+a_3 \hat{k}\right)\)

⇒ \(b_1 \hat{i}+b_2 \hat{j}+b_3 \hat{k}=\left(\lambda a_1\right) \hat{i}+\left(\lambda a_2\right) \hat{j}+\left(\lambda a_3\right) \hat{k}\)

⇒ \(\mathrm{b}_1=\lambda \mathrm{a}_1, \mathrm{~b}_2=\lambda \mathrm{a}_2, \mathrm{~b}_3=\lambda \mathrm{a}_3\)

⇒ \(\frac{b_1}{a_1}=\frac{b_2}{a_2}=\frac{b_3}{a_3}=\lambda\)

Thus, the respective scalar components of \(\vec{a}\) and \(\vec{b}\) are proportional.

However, vectors \(\vec{a}\) and \(\vec{b}\) can have different directions.

Hence, the statement given in 4 is incorrect.

The correct answer is 4.

Vector Algebra Exercise 10.3

Question 1. Find the angle between two vectors \(\vec{a}\) and \(\vec{b}\) with magnitude \(\sqrt{3}\) and 2, respectively having \(\vec{a} \cdot \vec{b}=\sqrt{6}\).
Solution:

It is given that, \(|\vec{a}|=\sqrt{3},|\vec{b}|=2\) and, \(\vec{a} \cdot \vec{b}=\sqrt{6}\)

Now, we know that \(\vec{a} \cdot \vec{b}=|\vec{a}||\vec{b}| \cos \theta\).

∴ \(\sqrt{6}=\sqrt{3} \times 2 \times \cos \theta \Rightarrow \cos \theta=\frac{\sqrt{6}}{\sqrt{3} \times 2} \Rightarrow \cos \theta=\frac{1}{\sqrt{2}} \Rightarrow \theta=\frac{\pi}{4}\)

Hence, the angle between the given vectors \(\vec{a}\) and \(\vec{b}\) is \(\frac{\pi}{4}\).

Question 2. Find the angle between the vectors \(\hat{i}-2 \hat{j}+3 \hat{k}\) and \(3 \hat{i}-2 \hat{j}+\hat{k}\).
Solution:

Let \(\overrightarrow{\mathrm{a}}=\hat{\mathrm{i}}-2 \hat{\mathrm{j}}+3 \hat{\mathrm{k}}\) and \(\overrightarrow{\mathrm{b}}=3 \hat{\mathrm{i}}-2 \hat{\mathrm{j}}+\hat{\mathrm{k}}\).

⇒ \(|\vec{a}|=\sqrt{1^2+(-2)^2+3^2}=\sqrt{1+4+9}=\sqrt{14}\)

⇒ \(|\vec{b}|=\sqrt{3^2+(-2)^2+1^2}=\sqrt{9+4+1}=\sqrt{14}\)

Now, \(\vec{a} \cdot \vec{b}=(\hat{i}-2 \hat{j}+3 \hat{k}) \cdot(3 \hat{i}-2 \hat{j}+\hat{k})=1 \cdot 3+(-2)(-2)+3 \cdot 1=3+4+3=10\)

Also, we know that \(\vec{a} \cdot \vec{b}=|\vec{a}| \cdot|\vec{b}| \cos \theta\)

∴ 10 = \(\sqrt{14} \sqrt{14} \cos \theta \Rightarrow \cos \theta=\frac{10}{14} \Rightarrow \theta=\cos ^{-1}\left(\frac{5}{7}\right)\)

Question 3. Find the projection of the vector \(\hat{i}-\hat{j}\) on the vector \(\hat{i}+\hat{j}\).
Solution;

Let \(\vec{a}=\hat{i}-\hat{j}\) and \(\vec{b}=\hat{i}+\hat{j}\)

Now, projection of vector \(\vec{a}\) on \(\vec{b}\) is given by, \(\frac{1}{|\vec{b}|}(\vec{a} \cdot \vec{b})=\frac{1}{\sqrt{1+1}}\{1 \cdot 1+(-1)(1)\}=\frac{1}{\sqrt{2}}(1-1)=0\)

Hence, the projection of vector \(\vec{a}\) on \(\vec{b}\) is 0 .

Question 4. Find the projection of the vector \(\hat{i}+3 \hat{j}+7 \hat{k}\) on the vector \(7 \hat{i}-\hat{j}+8 \hat{k}\).
Solution:

Let \(\vec{a}=\hat{i}+3 \hat{j}+7 \hat{k}\) and \(\hat{b}=7 \hat{i}-\hat{j}+8 \hat{k}\).

Now, projection of vector \(\vec{a}\) on \(\vec{b}\) is given by-

⇒ \(\frac{1}{|\vec{b}|}(\vec{a} \cdot \vec{b})=\frac{1}{\sqrt{7^2+(-1)^2+8^2}}\{1(7)+3(-1)+7(8)\}=\frac{7-3+56}{\sqrt{49+1+64}}=\frac{60}{\sqrt{114}}\)

Hence, the projection of vector \(\vec{a}\) on \(\vec{b}\) is \(\frac{60}{\sqrt{114}}\).

Question 5. Show that each of the given three vectors is a unit vector: \(\frac{1}{7}(2 \hat{i}+3 \hat{\mathrm{j}}+6 \hat{\mathrm{k}}) \cdot \frac{1}{7}(3 \hat{\mathrm{i}}-6 \hat{\mathrm{j}}+2 \hat{\mathrm{k}}), \frac{1}{7}(6 \hat{\mathrm{i}}+2 \hat{\mathrm{j}}-3 \hat{\mathrm{k}})\). Also, show that they are mutually perpendicular to each other.
Solution:

Let,

⇒ \(\overrightarrow{\mathrm{a}}=\frac{1}{7}(2 \hat{\mathrm{i}}+3 \hat{\mathrm{j}}+6 \hat{\mathrm{k}})=\frac{2}{7} \hat{\mathrm{i}}+\frac{3}{7} \hat{\mathrm{j}}+\frac{6}{7} \hat{\mathrm{k}}\)

∴ \(|\overrightarrow{\mathrm{a}}|=\sqrt{\left(\frac{2}{7}\right)^2+\left(\frac{3}{7}\right)^2+\left(\frac{6}{7}\right)^2}=\sqrt{\frac{4}{49}+\frac{9}{49}+\frac{36}{49}}=1\)

⇒ \(\overrightarrow{\mathrm{b}}=\frac{1}{7}(3 \hat{\mathrm{i}}-6 \hat{\mathrm{j}}+2 \hat{\mathrm{k}})=\frac{3}{7} \hat{\mathrm{i}}-\frac{6}{7} \hat{\mathrm{j}}+\frac{2}{7} \hat{\mathrm{k}}\)

∴ \(|\overrightarrow{\mathrm{b}}|=\sqrt{\left(\frac{3}{7}\right)^2+\left(-\frac{6}{7}\right)^2+\left(\frac{2}{7}\right)^2}=\sqrt{\frac{9}{49}+\frac{36}{49}+\frac{4}{49}}=1\)

⇒ \({\mathrm{c}}=\frac{1}{7}(6 \hat{\mathrm{i}}+2 \hat{\mathrm{j}}-3 \hat{\mathrm{k}})=\frac{6}{7} \hat{\mathrm{i}}+\frac{2}{7} \hat{\mathrm{j}}-\frac{3}{7} \hat{\mathrm{k}}\)

∴ \(|\overrightarrow{\mathrm{c}}|=\sqrt{\left(\frac{6}{7}\right)^2+\left(\frac{2}{7}\right)_{-}^2+\left(-\frac{3}{7}\right)^2}=\sqrt{\frac{36}{49}+\frac{4}{49}+\frac{9}{49}}=1\)

Thus, each of the given three vectors is a unit vector.

⇒ \(\overrightarrow{\mathrm{a}} \cdot \overrightarrow{\mathrm{b}}=\frac{2}{7} \times \frac{3}{7}+\frac{3}{7} \times\left(\frac{-6}{7}\right)+\frac{6}{7} \times \frac{2}{7}=\frac{6}{49}-\frac{18}{49}+\frac{12}{49}=0\)

⇒ \(\overrightarrow{\mathrm{b}} \cdot \overrightarrow{\mathrm{c}}=\frac{3}{7} \times \frac{6}{7}+\left(\frac{-6}{7}\right) \times \frac{2}{7}+\frac{2}{7} \times\left(\frac{-3}{7}\right)=\frac{18}{49}-\frac{12}{49}-\frac{6}{49}=0\)

⇒ \(\overrightarrow{\mathrm{c}} \cdot \overrightarrow{\mathrm{a}}=\frac{6}{7} \times \frac{2}{7}+\frac{2}{7} \times \frac{3}{7}+\left(\frac{-3}{7}\right) \times \frac{6}{7}=\frac{12}{49}+\frac{6}{49}-\frac{18}{49}=0\)

Hence, the given three vectors are mutually perpendicular to each other.

Question 6. Find \(|\vec{a}|\) and \(|\vec{b}|\), if \((\vec{a}+\vec{b}) \cdot(\vec{a}-\vec{b})=8\) and \(|\vec{a}|=8|\vec{b}|\).
Solution:

⇒ \((\vec{a}+\vec{b}) \cdot(\vec{a}-\vec{b})=8 \Rightarrow \vec{a} \cdot \vec{a}-\vec{a} \cdot \vec{b}+\vec{b} \cdot \vec{a}-\vec{b} \cdot \vec{b}=8\)

⇒ \(|\vec{a}|^2-|\vec{b}|^2=8\)

because \(\vec{a} \cdot \vec{a}=|\vec{a}|^2\)

⇒ \((8 \mid \vec{b})^2-|\vec{b}|^2=8\)

because \(|\vec{a}|=8|\vec{b}|\)

⇒ \(64|\vec{b}|^2-|\vec{b}|^2=8 \Rightarrow 63|\vec{b}|^2=8 \Rightarrow|\vec{b}|^2=\frac{8}{63}\)

⇒ \(|\vec{b}|=\sqrt{\frac{8}{63}}\) Magnitude of a vector is non-negative

⇒ \(|\vec{b}|=\frac{2 \sqrt{2}}{3 \sqrt{7}}\)

⇒ \(|\vec{a}|=8|\vec{b}|=\frac{8 \times 2 \sqrt{2}}{3 \sqrt{7}}=\frac{16 \sqrt{2}}{3 \sqrt{7}}\)

Question 7. Evaluate the product \((3 \vec{a}-5 \vec{b}) \cdot(2 \vec{a}+7 \vec{b})\).
Solution:

⇒ \((3 \vec{a}-5 \vec{b}) \cdot(2 \vec{a}+7 \vec{b})=6(\vec{a} \cdot \vec{a})+21(\vec{a} \cdot \vec{b})-10(\vec{b} \cdot \vec{a})-35(\vec{b} \cdot \vec{b})\)

= \(6(\vec{a} \cdot \vec{a})+21(\vec{a} \cdot \vec{b})-10(\vec{a} \cdot \vec{b})-35(\vec{b} \cdot \vec{b})\)

= \(6|\vec{a}|^2+11 \vec{a} \cdot \vec{b}-35|\vec{b}|^2\) (because \(\vec{a} \cdot \vec{a}=|\vec{a}|^2 \text { and } \vec{a} \cdot \vec{b}=\vec{b} \cdot \vec{a}\))

Question 8. Find the magnitude of two vectors \(\vec{a}\) and \(\vec{b}\), having the same magnitude and such that the angle between them is \(60^{\circ}\) and their scalar product is \(\frac{1}{2}\).
Solution:

Let \(\theta\) be the angle between the vectors \(\vec{a}\) and \(\vec{b}\).

It is given that \(|\vec{a}|=|\vec{b}|, \vec{a} \cdot \vec{b}=\frac{1}{2}\) and \(\theta=60^{\circ}\)…..(1)

We know that \(\vec{a} \cdot \vec{b}=|\vec{a}||\vec{b}| \cos \theta\).

∴ \(\frac{1}{2}=|\vec{a}||\vec{a}| \cos 60^{\circ}\) Using (1)

⇒ \(\frac{1}{2}=|\vec{a}|^2 \times \frac{1}{2} \Rightarrow|\vec{a}|^2=1 \Rightarrow|\vec{a}|=1 \Rightarrow|\vec{a}|=|\vec{b}|=1\)

Question 9. Find \(|\vec{x}|\), if for a unit vector \(\vec{a},(\vec{x}-\vec{a}) \cdot(\vec{x}+\vec{a})=12\).
Solution:

⇒ \((\vec{x}-\vec{a}) \cdot(\vec{x}+\vec{a})=12 \Rightarrow \vec{x} \cdot \vec{x}+\vec{x} \cdot \vec{a}-\vec{a} \cdot \vec{x}-\vec{a} \cdot \vec{a}=12\)

⇒ \(|\vec{x}|^2-|\vec{a}|^2=12\) (because \(\vec{a} \cdot \vec{a}=|\vec{a}|^2\))

and \(\vec{a} \cdot \vec{b}=\vec{b} \cdot \vec{a})\)

⇒ \(|\vec{x}|^2-1=12 \quad |\vec{a}|=1\) as \(\vec{a}\) is a unit vector

⇒ \(|\overrightarrow{\mathrm{x}}|^2=13\)

∴ \(|\mathrm{x}|=\sqrt{13}\)

Question 10. If \(\overrightarrow{\mathrm{a}}=2 \hat{\mathrm{i}}+2 \hat{\mathrm{j}}+3 \hat{\mathrm{k}}, \overrightarrow{\mathrm{b}}=-\hat{\mathrm{i}}+2 \hat{\mathrm{j}}+\hat{\mathrm{k}}\) and \(\overrightarrow{\mathrm{c}}=3 \hat{\mathrm{i}}+\hat{\mathrm{j}}\) are such that \(\overrightarrow{\mathrm{a}}+\lambda \overrightarrow{\mathrm{b}}\) is perpendicular to \(\overrightarrow{\mathrm{c}}\), then find the value of \(\lambda\).
Solution:

The given vectors are \(\vec{a}=2 \hat{i}+2 \hat{j}+3 \hat{k}, \vec{b}=-\hat{i}+2 \hat{j}+\hat{k}\) and [/latex]\vec{c}=3 \hat{i}+\hat{j}[/latex].

Now, \(\vec{a}+\lambda \vec{b}=(2 \hat{i}+2 \hat{j}+3 \hat{k})+\lambda(-\hat{i}+2 \hat{j}+\hat{k})=(2-\lambda) \hat{i}+(2+2 \lambda) \hat{j}+(3+\lambda) \hat{k}\)

If \((\vec{a}+\lambda \vec{b})\) is perpendicular to \(\vec{c}\), then \((\vec{a}+\lambda \vec{b}) \cdot \vec{c}=0\)

⇒ \([(2-\lambda) \hat{\mathrm{i}}+(2+2 \lambda) \hat{\mathrm{j}}+(3+\lambda) \hat{\mathrm{k}}] \cdot(3 \hat{\mathrm{i}}+\hat{\mathrm{j}})=0 \Rightarrow(2-\lambda) 3+(2+2 \lambda) 1+(3+\lambda) 0=0\)

⇒ 6 – \(3 \lambda+2+2 \lambda \Rightarrow 0 \Rightarrow-\lambda+8=0 \Rightarrow \lambda=8\)

Hence, the required value of \(\lambda\) is 8 .

Question 11. Show that \(|\vec{a}| \vec{b}+|\vec{b}| \vec{a}\) is perpendicular to \(|\vec{a}| \vec{b}-|\vec{b}| \vec{a}\), for any two nonzero vectors \(\vec{a}\) and \(\vec{b}\).
Solution:

⇒ \((|\vec{a}| \vec{b}+|\vec{b}| \vec{a}) \cdot(|\vec{a}| \vec{b}-|\vec{b}| \vec{a})\)

= \(|\vec{a}|^2(\vec{b} \cdot \vec{b})-|\vec{a}||\vec{b}|(\vec{b} \cdot \vec{a})+|\vec{b}||\vec{a}|(\vec{a} \cdot \vec{b})-|\vec{b}|^2(\vec{a} \cdot \vec{a})\)

= \(|\vec{a}|^2|\stackrel{\rightharpoonup}{b}|^2-|\vec{b}|^2|\vec{a}|^2=0\) (because \(\vec{a} \cdot \vec{a}=|\vec{a}|^2\) and \(\vec{a} \cdot \vec{b}=\vec{b} \cdot \vec{a}\)

Hence, \(|\vec{a}| \vec{b}+|\vec{b}| \vec{a}\) and \(|\vec{a}| |\vec{b}|-|\vec{b}| \vec{a}\) are perpendicular to each other for any two non-zero vectors \(\vec{a}\) and \(\vec{b}\).

Question 12. If \(\vec{a} \cdot \vec{a}=0\) and \(\vec{a} \cdot \vec{b}=0\), then what can be concluded about the vector \(\vec{b}\)?
Solution:

It is given that \(\vec{a} \cdot \vec{a}=0\) and \(\vec{a} \cdot \vec{b}=0\).

Now, \(\overrightarrow{\mathrm{a}} \cdot \overrightarrow{\mathrm{a}}=0 \Rightarrow|\overrightarrow{\mathrm{a}}|^2=0 \Rightarrow|\overrightarrow{\mathrm{a}}|=0\)

⇒ \(\overrightarrow{\mathrm{a}}\) is a zero vector.

Hence, vector \(\vec{b}\) satisfying \(\vec{a} \cdot \vec{b}=0\) can be any vector.

Question 13. If \(\vec{a}, \vec{b}, \vec{c}\) are unit vectors such that \(\vec{a}+\vec{b}+\vec{c}=\overrightarrow{0}\), find the value of \(\vec{a} \cdot \vec{b}+\vec{b} \cdot \vec{c}+\vec{c} \cdot \vec{a}\).
Solution;

Given \(\vec{a}, \vec{b}, \vec{c}\) are unit vectors, therefore

⇒ \(|\vec{a}|=1,|\vec{b}|=1 \text { and }|\vec{c}|=1 \text {. }\)…….(1)

Again given \(\overrightarrow{\mathrm{a}}+\overrightarrow{\mathrm{b}}+\overrightarrow{\mathrm{c}}=\overrightarrow{0}\)

⇒ \(|\vec{a}+\vec{b}+\vec{c}|=0 \quad \Rightarrow(\vec{a}+\vec{b}+\vec{c}) \cdot(\vec{a}+\vec{b}+\vec{c})=0\)

⇒ \(\vec{a} \cdot \vec{a}+\vec{b} \cdot \vec{b}+\vec{c} \cdot \vec{c}+2(\vec{a} \cdot \vec{b}+\vec{b} \cdot \vec{c}+\vec{c} \cdot \vec{a})=0\)

⇒ \(|\vec{a}|^2+|\vec{b}|^2+|\vec{c}|^2+2(\vec{a} \cdot \vec{b}+\vec{b} \cdot \vec{c}+\vec{c} \cdot \vec{a})=0\) (because \(\vec{a} \cdot \vec{a}=|\vec{a}|^2)\))

⇒ \(1+1+1+2(\vec{a} \cdot \vec{b}+\vec{b} \cdot \vec{c}+\vec{c} \cdot \vec{a})=0\) [Using equation (1)]

⇒ \(2(\vec{a} \cdot \vec{b}+\vec{b} \cdot \vec{c}+\vec{c} \vec{a})=-3 \Rightarrow \vec{a} \cdot \vec{b}+\vec{b} \cdot \vec{c}+\vec{c} \cdot \vec{a}=\frac{-3}{2}\)

Question 14. If either vector \(\vec{a}=\overrightarrow{0}\) or \(\vec{b}=\overrightarrow{0}\), then \(\vec{a} \cdot \vec{b}=0\). But the converse need not be true. Justify your answer with an example.
Solution:

Consider \(\vec{a}=2 \hat{i}+4 \hat{j}+3 \hat{k}\) and \(\vec{b}=3 \hat{i}+3 \hat{j}-6 \hat{k}\).

Then, \(\vec{a} \cdot \vec{b}=2 \cdot 3+4 \cdot 3+3(-6)=6+12-18=0\)

We now observe that: \(|\vec{a}|=\sqrt{2^2+4^2+3^2}=\sqrt{29}\)

⇒ \(\vec{a} \neq \overrightarrow{0}\)

⇒ \(|\overrightarrow{\mathrm{b}}|=\sqrt{3^2+3^2+(-6)^2}=\sqrt{54}\)

⇒ \(\overrightarrow{\mathrm{b}} \neq \overrightarrow{0}\)

Hence, the converse of the given statement need not be true.

Question 15. If the vertices A, B, and C of a triangle ABC are (1,2,3),(-1,0,0),(0,1,2), respectively, then find \(\angle \mathrm{ABC}\). \(\angle \mathrm{ABC}\) is the angle between the vectors \(\overrightarrow{\mathrm{BA}}\) and \(\overrightarrow{\mathrm{BC}}\).
Solution:

The vertices of \(\triangle \mathrm{ABC}\) are given as A(1,2,3), B}(-1,0,0), and C(0,1,2). Also, it is given that \(\angle \mathrm{ABC}\) is the angle between the vectors \(\overrightarrow{\mathrm{BA}}\) and \(\overrightarrow{\mathrm{BC}}\).

⇒ \(\overrightarrow{B A}=(1-(-1)) \hat{i}+(2-0) \hat{j}+(3-0) \hat{k}=2 \hat{i}+2 \hat{j}+3 \hat{k}\)

⇒ \(\overrightarrow{\mathrm{BC}}=(0-(-1)) \hat{\mathrm{i}}+(1-0) \hat{\mathrm{j}}+(2-0) \hat{k}=\hat{\mathrm{i}}+\hat{\mathrm{j}}+2 \hat{\mathrm{k}}\)

∴ \(\overrightarrow{\mathrm{BA}} \cdot \overrightarrow{\mathrm{BC}}=(2 \hat{\mathrm{i}}+2 \hat{\mathrm{j}}+3 \hat{\mathrm{k}}) \cdot(\hat{\mathrm{i}}+\hat{\mathrm{j}}+2 \hat{\mathrm{k}})=2 \times 1+2 \times 1+3 \times 2=2+2+6=10\)

⇒ \(|\overrightarrow{\mathrm{BA}}|=\sqrt{2^2+2^2+3^2}=\sqrt{4+4+9}=\sqrt{17}\)

⇒ \(|\overrightarrow{\mathrm{BC}}|=\sqrt{1+1+2^2}=\sqrt{6}\)

Now, it is known that: \(\overrightarrow{\mathrm{BA}} \cdot \overrightarrow{\mathrm{BC}}=|\overrightarrow{\mathrm{BA}}||\overrightarrow{\mathrm{BC}}| \cos (\angle \mathrm{ABC})\)

⇒ 10 = \(\sqrt{17} \times \sqrt{6} \cos (\angle \mathrm{ABC}) \Rightarrow \cos (\angle \mathrm{ABC})=\frac{10}{\sqrt{17} \times \sqrt{6}} \Rightarrow \angle \mathrm{ABC}=\cos ^{-1}\left(\frac{10}{\sqrt{102}}\right)\)

Question 16. Show that the points A(1,2,7), B(2,6,3) and C(3,10,-1) are collinear.
Solution:

The given points are A(1,2,7), B(2,6,3), and C(3,10,-1).

Position vectors of points A, B, and C are \(\vec{a}=\hat{i}+2 \hat{j}+7 \hat{k}, \vec{b}=2 \hat{i}+6 \hat{j}+3 \hat{k}\) and \(\vec{c}=3 \hat{i}+10 \hat{j}-\hat{k}\) respectively.

∴ \(\overrightarrow{\mathrm{AB}}=\overrightarrow{\mathrm{b}}-\overrightarrow{\mathrm{a}}=(2-1) \hat{i}+(6-2) \hat{j}+(3-7) \hat{k}=\hat{i}+4 \hat{j}-4 \hat{k}\)

∴ \(\overrightarrow{\mathrm{BC}}=\overrightarrow{\mathrm{c}}-\overrightarrow{\mathrm{b}}=(3-2) \hat{i}+(10-6) \hat{j}+(-1-3) \hat{k}=\hat{i}+4 \hat{j}-4 \hat{k}\)

Since \(\overrightarrow{\mathrm{AB}}=\overrightarrow{\mathrm{BC}} \Rightarrow \overrightarrow{\mathrm{AB}} \| \overrightarrow{\mathrm{BC}}\)

Here, point B is common in \(\overrightarrow{\mathrm{AB}}\) and \(\overrightarrow{\mathrm{BC}}\)

So, \(\overrightarrow{\mathrm{AB}}\) and \(\overrightarrow{\mathrm{BC}}\) are collinear.

Hence, A, B, and C are collinear

Question 17. Show that the vectors \(2 \hat{i}-\hat{j}+\hat{k}, \hat{i}-3 \hat{j}-5 \hat{k}\) and \(3 \hat{i}-4 \hat{j}-4 \hat{k}\) form the vertices of right angled triangle.
Solution:

Let vectors \(2 \hat{i}-\hat{j}+\hat{k}, \hat{i}-3 \hat{j}-5 \hat{k}\) and \(3 \hat{i}-4 \hat{j}-4 \hat{k}\) be position vectors of points A, B and C respectively.

i.e., \(\overline{O A}=2 \hat{i}-\hat{j}+\hat{k}, \overrightarrow{O B}=\hat{i}-3 \hat{j}-5 \hat{k}\) and \(\overline{O C}=3 \hat{i}-4 \hat{j}-4 \hat{k}\)

∴ \(\overrightarrow{A B}=(1-2) \hat{i}+(-3+1) \hat{j}+(-5-1) \hat{k}=-\hat{i}-2 \hat{j}-6 \hat{k}\)

∴ \(\overrightarrow{\mathrm{BC}}=(3-1) \hat{i}+(-4+3) \hat{j}+(-4+5) \hat{k}=2 \hat{i}-\hat{j}+\hat{k}\)

∴ \(\overrightarrow{\mathrm{AC}}=(3-2) \hat{\mathrm{i}}+(-4+1) \hat{\mathrm{j}}+(-4-1) \hat{k}=\hat{\mathrm{i}}-3 \hat{\mathrm{j}}-5 \hat{\mathrm{k}}\)

Now, \(\overrightarrow{A B}+\overrightarrow{B C}=(-\hat{i}-2 \hat{j}+6 \hat{k})+(2 \hat{i}-\hat{j}+\hat{k})=\hat{i}-3 \hat{j}-5 \hat{k}=\overrightarrow{A C}\)

A, B, and C are vertices of a triangle.

Now, \(|\overrightarrow{\mathrm{AB}}|=\sqrt{(-1)^2+(-2)^2+(-6)^2}=\sqrt{1+4+36}=\sqrt{41}\)

∴ \(|\overrightarrow{\mathrm{BC}}|=\sqrt{2^2+(-1)^2+1^2}=\sqrt{4+1+1}=\sqrt{6}\)

∴ \(|\overrightarrow{\mathrm{AC}}|=\sqrt{(1)^2+(-3)^2+(-5)^2}=\sqrt{1+9+25}=\sqrt{35}\)

∴ \(|\overrightarrow{\mathrm{BC}}|^2+|\overrightarrow{\mathrm{AC}}|^2=6+35=41=|\overrightarrow{\mathrm{AB}}|^2\)

Hence, A, B, and C are vertices of a right-angle triangle.

Question 18. If \(\vec{a}\) is a nonzero vector of magnitude ‘a’ and \(\lambda\) a nonzero scalar, then \(\lambda \vec{a}\) is unit vector if

  1. \(\lambda=1\)
  2. \(\lambda=-1\)
  3. \(a=|\lambda|\)
  4. \(a=\frac{1}{|\lambda|}\)

Solution: 4. \(a=\frac{1}{|\lambda|}\)

Given \(\lambda \vec{a}\) is a unit vector.

∴ \(|\lambda \vec{a}|=1 \quad \Rightarrow \quad|\lambda||\vec{a}|=1\)

⇒ \(|\vec{a}|=\frac{1}{|\lambda|} \quad[\lambda \neq 0]\)

⇒ \(a=\frac{1}{|\lambda|}\) (because \(|\vec{a}|\)=a)

Hence, vector \(\lambda \vec{a}\) is a unit vector if \(\mathrm{a}=\frac{1}{|\vec{\lambda}|}, \quad(\lambda \neq 0)\)

The correct answer is (4).

Vector Algebra Exercise 10.4

Question 1. Find \(|\overrightarrow{\mathrm{a}} \times \overrightarrow{\mathrm{b}}|\), if \(\overrightarrow{\mathrm{a}}=\hat{\mathrm{i}}-7 \hat{\mathrm{j}}+7 \hat{\mathrm{k}}\) and \(\overrightarrow{\mathrm{b}}=3 \hat{\mathrm{i}}-2 \hat{\mathrm{j}}+2 \hat{\mathrm{k}}\).
Solution:

We have, \(\vec{a}=\hat{i}-7 \hat{j}+7 \hat{k}\) and \(\vec{b}=3 \hat{i}-2 \hat{j}+2 \hat{k}\)

⇒ \(\overrightarrow{\mathrm{a}} \times \overrightarrow{\mathrm{b}}\)

= \(\left|\begin{array}{ccc}
\hat{\mathrm{i}} & \hat{\mathrm{j}} & \hat{\mathrm{k}} \\
1 & -7 & 7 \\
3 & -2 & 2
\end{array}\right|\)

= \(\hat{\mathrm{i}}(-14+14)-\hat{\mathrm{j}}(2-21)+\hat{\mathrm{k}}(-2+21)=0 \hat{\mathrm{i}}+19 \hat{\mathrm{j}}+19 \hat{\mathrm{k}}\)

∴ \(|\overrightarrow{\mathrm{a}} \times \overrightarrow{\mathrm{b}}|=\sqrt{(19)^2+(19)^2}=\sqrt{2 \times(19)^2}=19 \sqrt{2}\)

Question 2. Find a unit vector perpendicular to each of the vectors \(\vec{a}+\vec{b}\) and \(\vec{a}-\vec{b}\), where \(\vec{a}=3 \hat{i}+2 \hat{j}+2 \hat{k}\) and \(\vec{b}=\hat{i}+2 \hat{j}-2 \hat{k}\).
Solution:

We have, \(\vec{a}=3 \hat{i}+2 \hat{j}+2 \hat{k}\) and \(\vec{b}=\hat{i}+2 \hat{j}-2 \hat{k}\)

∴ \(\vec{a}+\vec{b}=4 \hat{i}+4 \hat{j}, \vec{a}-\vec{b}=2 \hat{i}+4 \hat{k}\)

∴ \((\vec{a}+\vec{b}) \times(\vec{a}-\vec{b})\)

= \(\left|\begin{array}{lll}
\hat{i} & \hat{j} & \hat{k} \\
4 & 4 & 0 \\
2 & 0 & 4
\end{array}\right|\)

= \(\hat{i}(16)-\hat{j}(16)+\hat{k}(-8)=16 \hat{i}-16 \hat{j}-8 \hat{k}\)

∴ \(|(\vec{a}+\vec{b}) \times(\vec{a}-\vec{b})|=\sqrt{16^2+(-16)^2+(-8)^2}=8 \sqrt{2^2+2^2+1}=8 \sqrt{9}=8 \times 3=24\)

Hence, the unit vector perpendicular to each of the vectors \(\vec{a}+\vec{b}\) and \(\vec{a}-\vec{b}\) is given by the relation,

± \(\frac{(\vec{a}+\vec{b}) \times(\vec{a}-\vec{b})}{|(\vec{a}+\vec{b}) \times(\vec{a}-\vec{b})|}= \pm \frac{(16 \hat{i}-16 \hat{j}-8 \hat{k})}{24}= \pm \frac{(2 \hat{i}-2 \hat{j}-\hat{k})}{3}\)

Required vector is \(\frac{2}{3} \hat{\mathrm{i}}-\frac{2}{3} \hat{\mathrm{j}}-\frac{1}{3} \hat{\mathrm{k}}\) or \(-\frac{2}{3} \hat{\mathrm{i}}+\frac{2}{3} \hat{\mathrm{j}}+\frac{1}{3} \hat{\mathrm{k}}\)

Question 3. If a unit vector a makes an angle \(\frac{\pi}{3}\) with \(\hat{i}, \frac{\pi}{4}\) with \(\hat{j}\) and an acute angle \(\theta\) with \(\hat{k}\), then find \(\theta\) and hence, the components of \(\vec{a}\).
Solution:

Let unit vector \(\vec{a}=a_1 \hat{i}+a_2 \hat{j}+a_3 \hat{k}\)

Since \(\vec{a}\) is a unit vector, \(|\vec{a}|=1\).

Also, it is given that \(\overrightarrow{\mathrm{a}}\) makes angle \(\frac{\pi}{3}\) with \(\hat{\mathrm{i}}, \frac{\pi}{4}\) with \(\hat{\mathrm{j}}\) and an acute angle \(\theta\) with \(\hat{k}\).

Then, we have: \(\cos \frac{\pi}{3}=\frac{a_1}{|\vec{a}|} \Rightarrow \frac{1}{2}=a_1 \quad[|\vec{a}|=1]\)

Also, \(\cos \frac{\pi}{4}=\frac{\mathrm{a}_2}{|\overrightarrow{\mathrm{a}}|} \Rightarrow \frac{1}{\sqrt{2}}=\mathrm{a}_2\) (\(|\vec{a}|=1\))

Also, \(\cos \theta=\frac{\mathrm{a}_3}{|\overrightarrow{\mathrm{a}}|} \Rightarrow \mathrm{a}_3=\cos \theta\)

Now, \(|\overrightarrow{\mathrm{a}}|=1 \Rightarrow \sqrt{\mathrm{a}_1^2+\mathrm{a}_2^2+\mathrm{a}_3^2}=1\)

⇒ \(\left(\frac{1}{2}\right)^2+\left(\frac{1}{\sqrt{2}}\right)^2+\cos ^2 \theta=1 \Rightarrow \frac{1}{4}+\frac{1}{2}+\cos ^2 \theta=1\)

∴ \(\frac{3}{4}+\cos ^2 \theta=1 \Rightarrow \cos ^2 \theta=1-\frac{3}{4}=\frac{1}{4} \Rightarrow \cos \theta=\frac{1}{2} \Rightarrow \theta=\frac{\pi}{3}\) (because \(\theta\) is acute angle)

∴ \(a_3=\cos \frac{\pi}{3}=\frac{1}{2}\)

Hence, \(\theta=\frac{\pi}{3}\) and the components of a are \(\frac{1}{2}, \frac{1}{\sqrt{2}}, \frac{1}{2}\).

Question 4. Show that \((\vec{a}-\vec{b}) \times(\vec{a}+\vec{b})=2(\vec{a} \times \vec{b})\)
Solution:

L.H.S. \((\vec{a}-\vec{b}) \times(\vec{a}+\vec{b})\)

= \((\vec{a}-\vec{b}) \times \vec{a}+(\vec{a}-\vec{b}) \times \vec{b}\) [By distributivity of vector product over addition]

= \(\vec{a} \times \vec{a}-\vec{b} \times \vec{a}+\vec{a} \times \vec{b}-\vec{b} \times \vec{b}\) [Again, by distributivity of vector product over addition]

= \(\overrightarrow{0}+\vec{a} \times \vec{b}+\vec{a} \times \vec{b}-\overrightarrow{0}\)

= \(2(\vec{a} \times \vec{b})\) R.H.S.

Question 5. Find \(\lambda\) and \(\mu\) if \((2 \hat{i}+6 \hat{j}+27 \hat{k}) \times(\hat{i}+\lambda \hat{j}+\mu \hat{k})=\overrightarrow{0}\).
Solution:

⇒ \((2 \hat{i}+6 \hat{j}+27 \hat{k}) \times(\hat{i}+\lambda \hat{j}+\mu \hat{k})=\overrightarrow{0}\)

⇒ \(\left|\begin{array}{ccc}
\hat{i} & \hat{j} & \hat{k} \\
2 & 6 & 27 \\
1 & \lambda & \mu
\end{array}\right|\)

= \(0 \hat{\mathrm{i}}+0 \hat{\mathrm{j}}+0 \hat{\mathrm{k}} \Rightarrow \hat{\mathrm{i}}(6 \mu-27 \lambda)-\hat{\mathrm{j}}(2 \mu-27)+\hat{\mathrm{k}}(2 \lambda-6)\)

=0 \(\hat{\mathrm{i}}+0 \hat{\mathrm{j}}+0 \hat{\mathrm{k}}\)

On comparing the corresponding components, we have :

6 \(\mu-27 \lambda=0\)….(1)

2 \(\mu-27=0\)….(2)

2 \(\lambda-6=0\)….(3)

Now, from equation (3), \(2 \lambda-6=0 \Rightarrow \lambda=3\)

from equation (2), \(2 \mu-27=0 \Rightarrow \mu=\frac{27}{2}\)

Here, values of \(\lambda\) and \(\mu\) satisfy to equation (1)

Hence, \(\lambda=3\) and \(\mu=\frac{27}{2}\)

Question 6. Given that \(\vec{a} \cdot \vec{b}=0\) and \(\vec{a} \times \vec{b}=\overrightarrow{0}\). What can you conclude about the vectors \(\vec{a}\) and \(\vec{b}\)?
Solution:

⇒ \(\vec{a}, \vec{b}=0\) Either \(|\vec{a}|=0\) or \(|\vec{b}|=0\) or \(\vec{a} \perp \vec{b}\)

⇒ \(\vec{a} \times \vec{b}=0\) Then, either \(|\vec{a}|=0\) or \(|\vec{b}|=0\) or \(\vec{a} \| \vec{b}\)

But, \(\vec{a}\) and \(\vec{b}\) cannot be perpendicular and parallel simultaneously.

Hence, \(|\vec{a}|=0\) or \(|\vec{b}|=0\). i.e. either \(\vec{a}=\overrightarrow{0}\) or \(\vec{b}=\overrightarrow{0}\)

Question 7. Let the vectors \(\vec{a}, \vec{b}, \vec{c}\) are given as \(a_1 \hat{i}+a_2 \hat{j}+a_3 \hat{k}, b_1 \hat{i}+b_2 \hat{j}+b_3 \hat{k}, c_1 \hat{i}+c_2 \hat{j}+c_3 \hat{k}\) respectively.

Then show that \(\vec{a} \times(\vec{b}+\vec{c})=\vec{a} \times \vec{b}+\vec{a} \times \vec{c}\).

Solution:

We have \(\vec{a}=a_1 \hat{i}+a_2 \hat{j}+a_3 \hat{k}, \vec{b}=b_1 \hat{i}+b_2 \hat{j}+b_3 \hat{k}, \vec{c}=c_1 \hat{i}+c_2 \hat{j}+c_3 \hat{k}\)

⇒ \((\vec{b}+\vec{c})=\left(b_1+c_1\right) \hat{i}+\left(b_2+c_2\right) \hat{j}+\left(b_3+c_3\right) \hat{k}\)

Now, \(\vec{a} \times(\vec{b}+\vec{c})\left|\begin{array}{ccc}\hat{i} & \hat{j} & \hat{k} \\ a_1 & a_2 & a_3 \\ b_1+c_1 & b_2+c_2 & b_3+c_3\end{array}\right|\)

= \(\hat{i}\left[a_2\left(b_3+c_3\right)-a_3\left(b_2+c_2\right)\right]-\hat{j}\left[a_1\left(b_3+c_3\right)-a_3\left(b_1+c_1\right)\right]+\hat{k}\left[a_1\left(b_2+c_2\right)-a_2\left(b_1+c_1\right)\right]\)

= \(\hat{i}\left[a_2 b_3+a_2 c_3-a_3 b_2-a_3 c_2\right]+\hat{j}\left[-a_1 b_3-a_1 c_3+a_3 b_1+a_3 c_1\right]\)+\(\hat{k}\left[a_1 b_2+a_1 c_2-a_2 b_1-a_2 c_1\right]\)…..(1)

⇒ \(\vec{a} \times \vec{b}=\left|\begin{array}{ccc}
\hat{i} & \hat{j} & \hat{k} \\
a_1 & a_2 & a_3 \\
b_1 & b_2 & b_3
\end{array}\right|\)

= \(\hat{i}\left[a_2 b_3-a_3 b_2\right]+\hat{j}\left[a_3 b_1-a_1 b_3\right]+\hat{k}\left[a_1 b_2-a_2 b_1\right]\)….(2)

⇒ \(\vec{a} \times \vec{c}=\left|\begin{array}{ccc}
\hat{i} & \hat{j} & \hat{k} \\
a_1 & a_2 & a_3 \\
c_1 & c_2 & c_3
\end{array}\right|\)

= \(\hat{i}\left[a_2 c_3-a_3 c_2\right]+\hat{j}\left[a_3 c_1-a_1 c_3\right]+\hat{k}\left[a_1 c_2-a_2 c_1\right]\)….(3)

On adding (2) and (3), we get :

(\(\vec{a} \times \vec{b})+(\vec{a} \times \vec{c})=\hat{i}\left[a_2 b_3+a_2 c_3-a_3 b_2-a_3 c_2\right]+\hat{j}\left[b_1 a_3+a_3 c_1-a_1 b_3-a_1 c_3\right]\)

+ \(\hat{k}\left[a_1 b_2+a_1 c_2-a_2 b_1-a_2 c_1\right]\)….(4)

Now, from (1) and (4), we have: \(\vec{a} \times(\vec{b}+\vec{c})=\vec{a} \times \vec{b}+\vec{a} \times \vec{c}\)

Question 8. If either \(\vec{a}=\overrightarrow{0}\) or \(\vec{b}=\overrightarrow{0}\), then \(\vec{a} \times \vec{b}=\overrightarrow{0}\). Is the converse true? Justify your answer with an example.
Solution:

Take any parallel non-zero vectors so that \(\vec{a} \times \vec{b}=\overrightarrow{0}\).

Let \(\vec{a}=2 \hat{i}+3 \hat{j}+4 \hat{k}, \vec{b}=4 \hat{i}+6 \hat{j}+8 \hat{k}\).

Then, \(\overrightarrow{\mathrm{a}} \times \overrightarrow{\mathrm{b}}\)

= \(\left|\begin{array}{lll}\hat{\mathrm{i}} & \hat{\mathrm{j}} & \hat{\mathrm{k}} \\ 2 & 3 & 4 \\ 4 & 6 & 8\end{array}\right|=\hat{\mathrm{i}}(24-24)-\hat{\mathrm{j}}(16-16)+\hat{\mathrm{k}}(12-12)=0 \hat{\mathrm{i}}+0 \hat{\mathrm{j}}+0 \hat{\mathrm{k}}\)

= \(\overrightarrow{0}\)

It can now be observed that : \(|\vec{a}|=\sqrt{2^2+3^2+4^2}=\sqrt{29}\)

⇒ \(\vec{a} \neq \overrightarrow{0}\)

⇒ \(|\vec{b}|=\sqrt{4^2+6^2+8^2}=\sqrt{116}\)

⇒ \(\vec{b} \neq \overrightarrow{0}\)

Hence, the converse of the given statement need not be true.

Question 9. Find the area of the triangle with vertices A(1,1,2), B(2,3,5) and C(1,5,5).
Solution:

The vertices of triangle ABC are given as A(1,1,2), B(2,3,5), and C(1,5,5).

Position vector of points A, B and C are \(\vec{a}=\hat{i}+\hat{j}+2 \hat{k}, \vec{b}=2 \hat{i}+3 \hat{j}+5 \hat{k}\) and \(\vec{c}=\hat{i}+5 \hat{j}+5 \hat{k}\) respectively.

The adjacent sides A, B and \(\overrightarrow{B C}\) of \(\triangle A B C\) are given as:

⇒ \(\overrightarrow{\mathrm{AB}}=\overrightarrow{\mathrm{b}}-\overrightarrow{\mathrm{a}} \Rightarrow \overrightarrow{\mathrm{AB}}=(2-1) \hat{\mathrm{i}}+(3-1) \hat{\mathrm{j}}+(5-2) \hat{\mathrm{k}}=\hat{\mathrm{i}}+2 \hat{\mathrm{j}}+3 \hat{\mathrm{k}}\)

and \(\overrightarrow{\mathrm{BC}}=\overrightarrow{\mathrm{c}}-\overrightarrow{\mathrm{b}} \Rightarrow \overrightarrow{\mathrm{BC}}=(1-2) \hat{\mathrm{i}}+(5-3) \hat{\mathrm{j}}+(5-5) \hat{k}=-\hat{\mathrm{i}}+2 \hat{\mathrm{j}}\)

∴ \(\overrightarrow{\mathrm{AB}} \times \overrightarrow{\mathrm{BC}}\)

= \(\left|\begin{array}{ccc}
\hat{i} & \hat{j} & \hat{k} \\
1 & 2 & 3 \\
-1 & 2 & 0
\end{array}\right|\)

= \(\hat{i}(-6)-\hat{j}(3)+\hat{k}(2+2)=-6 \hat{i}-3 \hat{j}+4 \hat{k}\)

∴ \(|\overrightarrow{\mathrm{AB}} \times \overrightarrow{\mathrm{BC}}|=\sqrt{(-6)^2+(-3)^2+4^2}=\sqrt{36+9+16}=\sqrt{61}\)

Area of \(\triangle \mathrm{ABC}\)=\(\frac{1}{2}|\overrightarrow{\mathrm{AB}} \times \overrightarrow{\mathrm{BC}}|=\frac{1}{2} \sqrt{61}\)

Area of \(\triangle \mathrm{ABC}=\frac{1}{2}|\overrightarrow{\mathrm{AB}} \times \overrightarrow{\mathrm{BC}}|=\frac{1}{2} \sqrt{61}\)

Hence, the area of \(\triangle \mathrm{ABC}\) is \(\frac{\sqrt{61}}{2}\) square units.

Question 10. Find the area of the parallelogram whose adjacent sides are determined by the vector \(\overrightarrow{\mathrm{a}}=\hat{\mathrm{i}}-\hat{\mathrm{j}}+3 \hat{\mathrm{k}}\) and \(\vec{b}=2 \hat{i}-7 \hat{j}+\hat{k}\).
Solution:

The area of the parallelogram whose adjacent sides are \(\vec{a}\) and \(\vec{b}\) is \(|\vec{a} \times \vec{b}|\).

Adjacent sides are given as: \(\vec{a}=\hat{i}-\hat{j}+3 \hat{k}\) and \(\vec{b}=2 \hat{i}-7 \hat{j}+\hat{k}\)

∴ \(\vec{a} \times \vec{b}\)

= \(\left|\begin{array}{ccc}
\hat{i} & \hat{j} & \hat{k} \\
1 & -1 & 3 \\
2 & -7 & 1
\end{array}\right|\)

= \(\hat{i}(-1+21)-\hat{j}(1-6)+\hat{k}(-7+2)=20 \hat{i}+5 \hat{j}-5 \hat{k}\)

⇒ \(|\overrightarrow{\mathrm{a}} \times \overrightarrow{\mathrm{b}}|=\sqrt{20^2+5^2+5^2}=\sqrt{400+25+25}=15 \sqrt{2}\)

Hence, the area of the given parallelogram is \(15 \sqrt{2}\) square units.

Choose The Correct Answer

Question 11. Let the vectors \(\vec{a}\) and \(\vec{b}\) be such that \(|\vec{a}|=3\) and \(|\vec{b}|=\frac{\sqrt{2}}{3}\), then \(\vec{a} \times \vec{b}\) is a unit vector, if the angle between \(\vec{a}\) and \(\vec{b}\) is

  1. \(\frac{\pi}{6}\)
  2. \(\frac{\pi}{4}\)
  3. \(\frac{\pi}{3}\)
  4. \(\frac{\pi}{2}\)

Solution: 2. \(\frac{\pi}{4}\)

It is given that \(|\vec{a}|=3\) and \(|\vec{b}|=\frac{\sqrt{2}}{3}\).

We know that \(\vec{a} \times \vec{b}=|\vec{a}||\vec{b}| \sin \theta \hat{n}\), where \(\hat{n}\) is a unit vector perpendicular to both \(\vec{a}\) and \(\vec{b}\) and \(\theta\) is the angle between \(\vec{a}\) and \(\vec{b}\).

Now, \(\vec{a} \times \vec{b}\) is a unit vector if \(|\vec{a} \times \vec{b}|=1\)

⇒ \(|\overrightarrow{\mathrm{a}} \times \overrightarrow{\mathrm{b}}|=1 \Rightarrow||\overrightarrow{\mathrm{a}}||\overrightarrow{\mathrm{b}}| \sin \theta|=1 \Rightarrow 3 \times \frac{\sqrt{2}}{3} \times \sin \theta=1 \Rightarrow \sin \theta=\frac{1}{\sqrt{2}} \Rightarrow \theta=\frac{\pi}{4}\)

Hence, \(\vec{a} \times \vec{b}\) is a unit vector if the angle between \(\vec{a}\) and \(\vec{b}\) is \(\frac{\pi}{4}\).

The correct answer is 2.

Question 12. Area of a rectangle having vertices A, B, C, and D with position vectors \(-\hat{i}+\frac{1}{2} \hat{j}+4 \hat{k}\) \(\hat{i}+\frac{1}{2} \hat{j}+4 \hat{k}, \hat{i}-\frac{1}{2} \hat{j}+4 \hat{k},-\hat{i}-\frac{1}{2} \hat{j}+4 \hat{k}\) respectively is-

  1. \(\frac{1}{2}\)
  2. 1
  3. 2
  4. 4

Solution: 3. 2

The position vectors of vertices A, B, C, and D of rectangle ABCD are given as:

⇒ \(\overrightarrow{O A}=-\hat{i}+\frac{1}{2} \hat{j}+4 \hat{k}, \overrightarrow{O B}=\hat{i}+\frac{1}{2} \hat{j}+4 \hat{k}, \overrightarrow{O C}=\hat{i}-\frac{1}{2} \hat{j}+4 \hat{k}, \overrightarrow{O D}=-\hat{i}-\frac{1}{2} \hat{j}+4 \hat{k}\)

The adjacent sides \(\overrightarrow{\mathrm{AB}}\) and \(\overrightarrow{\mathrm{BC}}\) of the given rectangle are given as:

⇒ \(\overrightarrow{\mathrm{AB}}=\overrightarrow{\mathrm{OB}}-\overrightarrow{\mathrm{OA}}=(1+1) \hat{\mathrm{i}}+\left(\frac{1}{2}-\frac{1}{2}\right) \hat{\mathrm{j}}+(4-4) \hat{\mathrm{k}}=2 \hat{\mathrm{i}}\)

⇒ \(\overrightarrow{\mathrm{BC}}=\overrightarrow{\mathrm{OC}}-\overrightarrow{\mathrm{OB}}=(1-1) \hat{\mathrm{i}}+\left(-\frac{1}{2}-\frac{1}{2}\right) \hat{\mathrm{j}}+(4-4) \hat{\mathrm{k}}=-\hat{\mathrm{j}}\)

∴ \(\overrightarrow{\mathrm{AB}} \times \overrightarrow{\mathrm{BC}}\)

= \(\left|\begin{array}{ccc}
\hat{\mathrm{i}} & \hat{\mathrm{j}} & \hat{\mathrm{k}} \\
2 & 0 & 0 \\
0 & -1 & 0
\end{array}\right|\)

= \(\hat{\mathrm{k}}(-2)=-2 \hat{\mathrm{k}} \Rightarrow|\overrightarrow{\mathrm{AB}} \times \overrightarrow{\mathrm{BC}}|=\sqrt{(-2)^2}=2\)

Now, it is known that the area of a rectangle whose adjacent sides are \(\vec{a}\) and \(\vec{b}\) is \(|\vec{a} \times \vec{b}|\)

Hence, the area of the given rectangle is \(|\overrightarrow{A B} \times \overrightarrow{B C}|=2\) square units.

The correct answer is 3.

Vector Algebra Miscellaneous Exercise

Question 1. Write down a unit vector in XY-plane, making an angle of 30n with the positive direction of the x-axis.
Solution:

If \(\overrightarrow{\mathrm{r}}\) is a unit vector in the \(\mathrm{XY}\)-plane, then \(\overrightarrow{\mathrm{r}}=\cos \theta \hat{\mathrm{i}}+\sin \theta \hat{\mathrm{j}}\).

Here, \(\theta\) is the angle made by the unit vector with the positive direction of the x-axis.

Therefore, for \(\theta=30^{\circ}\):

⇒ \(\overrightarrow{\mathrm{r}}=\cos 30^{\circ} \hat{\mathrm{i}}+\sin 30^{\circ} \hat{\mathrm{j}}=\frac{\sqrt{3}}{2} \hat{\mathrm{i}}+\frac{1}{2} \hat{\mathrm{j}}\)

Hence, the required unit vector is \(\frac{\sqrt{3}}{2} \hat{\mathrm{i}}+\frac{1}{2} \hat{\mathrm{j}}\)

Question 2. Find the scalar components and magnitude of the vector joining the points \(P\left(x_1, y_1, z_1\right)\) and \(\mathrm{Q}\left(\mathrm{x}_2, \mathrm{y}_2, \mathrm{z}_2\right)\).
Solution:

The vector joining the points \(\mathrm{P}\left(\mathrm{x}_1, \mathrm{y}_1, \mathrm{z}_1\right)\) and \(\mathrm{Q}\left(\mathrm{x}_2, \mathrm{y}_2, \mathrm{z}_2\right)\) can be obtained by,

⇒ \(\overrightarrow{P Q}\) = P.V. of Q-P.V. of P =\(\left(x_2-x_1\right) \hat{i}+\left(y_2-y_1\right) \hat{j}+\left(z_2-z_1\right) \hat{k}\)

⇒ \(|\overrightarrow{P Q}|=\sqrt{\left(x_2-x_1\right)^2+\left(y_2-y_1\right)^2+\left(z_2-z_1\right)^2}\)

Hence, the scalar components and the magnitude of the vector joining the given points are \(\left(x_2-x_1\right),\left(y_2-y_1\right),\left(z_2-z_1\right)\) and \(\sqrt{\left(x_2-x_1\right)^2+\left(y_2-y_1\right)^2+\left(z_2-z_1\right)^2}\) respectively

Question 3. A girl walks \(4 \mathrm{~km}\) towards west, then she walks \(3 \mathrm{~km}\) in a direction \(30^{\circ}\) east of north and stops. Determine the girl’s displacement from her initial point of departure.
Solution:

Let O and B be the initial and final positions of the girl respectively. Then, the girl’s position can be shown as:

OA= \(4 \mathrm{~km}, \mathrm{AB}=3 \mathrm{~km}, \overrightarrow{\mathrm{OA}}=-4 \hat{\mathrm{i}}\) and \(\overrightarrow{\mathrm{AB}}=\overrightarrow{\mathrm{AC}}+\overrightarrow{\mathrm{CB}} \)

⇒ \(\overrightarrow{\mathrm{AB}}=\left(|\overrightarrow{\mathrm{AB}}| \cos 60^{\circ}\right) \hat{\mathrm{i}}+\left(|\overrightarrow{\mathrm{AB}}| \sin 60^{\circ}\right) \hat{\mathrm{j}}\)

= \(3 \times \frac{1}{2} \hat{\mathrm{i}}+3 \times \frac{\sqrt{3}}{2} \hat{\mathrm{j}}=\frac{3}{2} \hat{\mathrm{i}}+\frac{3 \sqrt{3}}{2} \hat{\mathrm{j}}\)

Vector Algebra Triangle Law Of Vector

By the triangle law of vector addition, we have: \(\overrightarrow{\mathrm{OB}} =\overrightarrow{\mathrm{OA}}+\overrightarrow{\mathrm{AB}}=(-4 \hat{\mathrm{i}})+\left(\frac{3}{2} \hat{\mathrm{i}}+\frac{3 \sqrt{3}}{2} \hat{\mathrm{j}}\right)=\left(-4+\frac{3}{2}\right) \hat{\mathrm{i}}+\frac{3 \sqrt{3}}{2} \hat{\mathrm{j}}\)

= \(\left(\frac{-8+3}{2}\right) \hat{\mathrm{i}}+\frac{3 \sqrt{3}}{2} \hat{\mathrm{j}}=\left(\frac{-5}{2}\right) \hat{\mathrm{i}}+\frac{3 \sqrt{3}}{2} \hat{\mathrm{j}}\)

Hence, the girl’s displacement from her initial point of departure is \(\left(\frac{-5}{2}\right) \hat{\mathrm{i}}+\frac{3 \sqrt{3}}{2} \hat{\mathrm{j}}\)

Question 4. If \(\vec{a}=\vec{b}+\vec{c}\), then is it true that \(|\vec{a}|=|\vec{b}|+|\vec{c}|\)? Justify your answer.
Solution:

In \(\triangle \mathrm{ABC}\), let \(\overrightarrow{\mathrm{CB}}=\overrightarrow{\mathrm{a}}, \overrightarrow{\mathrm{CA}}=\overrightarrow{\mathrm{b}}\), and \(\overrightarrow{\mathrm{AB}}=\overrightarrow{\mathrm{c}}\) (as shown in the following figure).

Now, by the triangle law of vector addition, we have \(\vec{a}=\vec{b}+\vec{c}\).

It is clearly known that \(|\vec{a}|,|\vec{b}|\) and \(|\vec{c}|\) represent the sides of \(\triangle \mathrm{ABC}\).

Vector Algebra Triangle Law Of Vector Addition

Also, it is known that the sum of the lengths of any two sides of a triangle is greater than the third side.

∴ \(|\vec{a}|<|\vec{b}|+|\vec{c}|\)

Hence, it is not true that \(|\vec{a}|=|\vec{b}|+|\vec{c}|\).

Question 5. Find the value of x for which \(x(\hat{i}+\hat{j}+\hat{k})\) is a unit vector.
Solution:

If \(x(\hat{i}+\hat{j}+\hat{k})\) is a unit vector, then \(|x(\hat{i}+\hat{j}+\hat{k})|=1\)

⇒ \(\sqrt{\mathrm{x}^2+\mathrm{x}^2+\mathrm{x}^2}=1 \Rightarrow \sqrt{3 \mathrm{x}^2}=1 \Rightarrow \pm \sqrt{3} \mathrm{x}=1 \Rightarrow \mathrm{x}= \pm \frac{1}{\sqrt{3}}\)

Hence, the required value of x is \(\pm \frac{1}{\sqrt{3}}\).

Question 6. Find a vector of magnitude 5 units, and parallel to the resultant of the vectors \(\vec{a}=2 \hat{i}+3 \hat{j}-\hat{k}\) and \(\overrightarrow{\mathrm{b}}=\hat{\mathrm{i}}-2 \hat{\mathrm{j}}+\hat{\mathrm{k}}\).
Solution:

We have, \(\vec{a}=2 \hat{i}+3 \hat{j}-\hat{k}\) and \(\vec{b}=\hat{i}-2 \hat{j}+\hat{k}\)

Let \(\vec{c}\) be the resultant of \(\vec{a}\) and \(\vec{b}\).

Then, \(\vec{c}=\vec{a}+\vec{b}=(2+1) \hat{i}+(3-2) \hat{j}+(-1+1) \hat{k}=3 \hat{i}+\hat{j}\)

⇒ \(|\overrightarrow{\mathrm{c}}|=\sqrt{3^2+1^2}=\sqrt{9+1}=\sqrt{10}\)

⇒ \(\hat{\mathrm{c}}=\frac{\overrightarrow{\mathrm{c}}}{|\overrightarrow{\mathrm{c}}|}=\frac{(3 \hat{\mathrm{i}}+\hat{\mathrm{j}})}{\sqrt{10}}\)

Hence, the vector of magnitude 5 units and parallel to the resultant of vectors \(\vec{a}\) and \(\vec{b}\) is \(\pm 5\). \(\hat{\mathrm{c}}= \pm 5 \cdot \frac{1}{\sqrt{10}}(3 \hat{\mathrm{i}}+\hat{\mathrm{j}})= \pm \frac{3 \sqrt{10}}{2} \hat{\mathrm{i}} \pm \frac{\sqrt{10}}{2} \hat{\mathrm{j}}\).

Question 7. If \(\vec{a}=\hat{i}+\hat{j}+\hat{k}, \vec{b}=2 \hat{i}-\hat{j}+3 \hat{k}\) and \(\vec{c}=\hat{i}-2 \hat{j}+3 \hat{k}\), find a unit vector parallel to the vector \(2 \vec{a}-\vec{b}+3 \vec{c}\).
Solution:

We have, \(\vec{a}=\hat{i}+\hat{j}+\hat{k}, \vec{b}=2 \hat{i}-\hat{j}+3 \hat{k}\) and \(\vec{c}=\hat{i}-2 \hat{j}+3 \hat{k}\)

⇒ \(2 \vec{a}-\vec{b}+3 \vec{c}=2(\hat{i}+\hat{j}+\hat{k})-(2 \hat{i}-\hat{j}+3 \hat{k})+3(\hat{i}-2 \hat{j}+\hat{k})\)

= \(2 \hat{i}+2 \hat{j}+2 \hat{k}-2 \hat{i}+\hat{j}-3 \hat{k}+3 \hat{i}-6 \hat{j}+3 \hat{k}=3 \hat{i}-3 \hat{j}+2 \hat{k}\)

⇒ \(|2 \vec{a}-\vec{b}+3 \vec{c}|=\sqrt{3^2+(-3)^2+2^2}=\sqrt{9+9+4}=\sqrt{22}\)

Hence, the unit vector parallel to \(2 \vec{a}-\vec{b}+3 \vec{c}\) is.

± \(\frac{(2 \vec{a}-\vec{b}+3 \vec{c})}{2 \vec{a}-\vec{b}+3 \vec{c}}= \pm \frac{3 \hat{i}-3 \hat{j}+2 \hat{k}}{\sqrt{22}}= \pm \frac{3}{\sqrt{22}} \hat{i} \mp \frac{3}{\sqrt{22}} \hat{j} \pm \frac{2}{\sqrt{22}} \hat{k}\)

Question 8. Show that the points A(1,-2,-8), B(5,0,-2) and C(11,3,7) are collinear, and find the ratio in which B divides AC.
Solution:

The given points are A(1,-2,-8), B(5,0,-2), and C(11,3,7).

P.V. of point A is \(\vec{a}=\hat{i}-2 \hat{j}-8 \hat{k}\)

P.V. of point B is \(\vec{b}=5 \hat{i}-2 \hat{k}\)

P.V. of point C is \(\overrightarrow{\mathrm{c}}=11 \hat{\mathrm{i}}+3 \hat{\mathrm{j}}+7 \hat{\mathrm{k}}\)

⇒ \(\overrightarrow{\mathrm{AB}}=\overrightarrow{\mathrm{b}}-\overrightarrow{\mathrm{a}} =(5-1) \hat{\mathrm{i}}+(0+2) \hat{\mathrm{j}}+(-2+8) \hat{k}=4 \hat{\mathrm{i}}+2 \hat{\mathrm{j}}+6 \hat{\mathrm{k}}\)

⇒ \(\overrightarrow{\mathrm{BC}}=\overrightarrow{\mathrm{c}}-\overrightarrow{\mathrm{b}} =(11-5) \hat{\mathrm{i}}+(3-0) \hat{\mathrm{j}}+(7+2) \hat{k}=6 \hat{i}+3 \hat{\mathrm{j}}+9 \hat{\mathrm{k}}\)

= \(\frac{3}{2}(4 \hat{\mathrm{i}}+2 \hat{\mathrm{j}}+6 \hat{\mathrm{k}})=\frac{3}{2} \overrightarrow{\mathrm{AB}}\)

⇒\(\overrightarrow{\mathrm{BC}}=\frac{3}{2} \overrightarrow{\mathrm{AB}} \text { i.e. } \overrightarrow{\mathrm{BC}} \| \overrightarrow{\mathrm{AB}}\)

Here, B is common in \(\overrightarrow{\mathrm{BC}}\) and \(\overrightarrow{\mathrm{AB}}\)

So, \(\overrightarrow{\mathrm{BC}}\) and \(\overrightarrow{\mathrm{AB}}\) are collinear

Hence, A, B, and C are collinear

Now, let point B divide AC in the ratio \(\lambda: 1\). Then, we have:

⇒ \(\overrightarrow{\mathrm{b}}=\frac{\lambda \overrightarrow{\mathrm{c}}+\overrightarrow{\mathrm{a}}}{\lambda+1}\)

⇒ \(5 \hat{\mathrm{i}}-2 \hat{\mathrm{k}}=\frac{\lambda(11 \hat{\mathrm{i}}+3 \hat{\mathrm{j}}+7 \hat{\mathrm{k}})+(\hat{\mathrm{i}}-2 \hat{\mathrm{j}}-8 \hat{\mathrm{k}})}{\lambda+1}\)

⇒ \((\lambda+1)(5 \hat{\mathrm{i}}-2 \hat{\mathrm{k}})=(11 \lambda \hat{\mathrm{i}}+3 \lambda \hat{\mathrm{j}}+7 \lambda \hat{\mathrm{k}})+(\hat{\mathrm{i}}-2 \hat{\mathrm{j}}-8 \hat{\mathrm{k}})\)

⇒ \(5(\lambda+1) \hat{\mathrm{i}}-2(\lambda+1) \hat{\mathrm{k}}=(11 \lambda+1) \hat{\mathrm{i}}+(3 \lambda-2) \hat{\mathrm{j}}+(7 \lambda-8) \hat{\mathrm{k}}\)

On equating the corresponding components, we get: 5\((\lambda+1)=11 \lambda+1\)

⇒ 5\(\lambda+5=11 \lambda+1\)

⇒ 6\(\lambda=4 \Rightarrow \lambda=\frac{4}{6}=\frac{2}{3}\)

Hence, point B divides AC in the ratio 2: 3, internally.

Question 9. Find the position vector of a point R which divides the line joining two points P and Q whose position vectors are \((2 \vec{a}+\vec{b})\) and \((\vec{a}-3 \vec{b})\) externally in the ratio 1: 2. Also, show that P is the midpoint of the line segment RQ.
Solution:

It is given that \(\overrightarrow{O P}=2 \vec{a}+\vec{b}, \overrightarrow{O Q}=\vec{a}-3 \vec{b}\).

It is given that point R divides a line segment joining two points P and Q externally in the ratio 1: 2.

Then, on using the section formula, we get: \(\overrightarrow{\mathrm{OR}}=\frac{2(2 \overrightarrow{\mathrm{a}}+\overrightarrow{\mathrm{b}})-(\overrightarrow{\mathrm{a}}-3 \overrightarrow{\mathrm{b}})}{2-1}=\frac{4 \overrightarrow{\mathrm{a}}+2 \overrightarrow{\mathrm{b}}-\overrightarrow{\mathrm{a}}+3 \overrightarrow{\mathrm{b}}}{1}=3 \overrightarrow{\mathrm{a}}+5 \overrightarrow{\mathrm{b}}\)

Therefore, the position vector of point R is \(3 \vec{a}+5 \vec{b}\).

Position vector of the mid-point of RQ = \(\frac{\overrightarrow{\mathrm{OQ}}+\overrightarrow{\mathrm{OR}}}{2}=\frac{(\overrightarrow{\mathrm{a}}-3 \overrightarrow{\mathrm{b}})+(3 \overrightarrow{\mathrm{a}}+5 \overrightarrow{\mathrm{b}})}{-2}=2 \overrightarrow{\mathrm{a}}+\overrightarrow{\mathrm{b}}=\overrightarrow{\mathrm{OP}}\)

Hence, P is the mid-point of the line segment RQ.

Question 10. The two adjacent sides of a parallelogram are \(2 \hat{i}-4 \hat{j}+5 \hat{k}\) and \(\hat{i}-2 \hat{j}-3 \hat{k}\). Find the unit vector parallel to its diagonal. Also, find its area.
Solution:

Adjacent sides of a parallelogram are given as: \(\vec{a}=2 \hat{i}-4 \hat{j}+5 \hat{k}\) and \(\vec{b}=\hat{i}-2 \hat{j}-3 \hat{k}\).

Then, the diagonal of a parallelogram is given by \(\vec{a}+\vec{b}\).

= \((2+1) \hat{i}+(-4-2) \hat{j}+(5-3) \hat{k}=3 \hat{i}-6 \hat{j}+2 \hat{k}\)

Thus, the unit vector parallel to the diagonal is

± \(\frac{\vec{a}+\vec{b}}{|\vec{a}+\vec{b}|}= \pm \frac{3 \hat{i}-6 \hat{j}+2 \hat{k}}{\sqrt{3^2+(-6)^2+2^2}}= \pm \frac{3 \hat{i}-6 \hat{j}+2 \hat{k}}{\sqrt{9+36+4}}= \pm \frac{3 \hat{i}-6 \hat{j}+2 \hat{k}}{7}\)

= \(\pm\left(\frac{3}{7} \hat{i}-\frac{6}{7} \hat{j}+\frac{2}{7} \hat{k}\right)\).

Now, Area of parallelogram ABCD = \(|\vec{a} \times \vec{b}|\)

⇒ \(\vec{a} \times \vec{b}\)

= \(\left|\begin{array}{ccc}
\hat{i} & \hat{j} & \hat{k} \\
2 & -4 & 5 \\
1 & -2 & -3
\end{array}\right|\)

= \(\hat{i}(12+10)-\hat{j}(-6-5)+\hat{k}(-4+4)=22 \hat{i}+11 \hat{j}\)

⇒ \(\vec{a} \times \vec{b}=11(2 \hat{i}+\hat{j})\)

⇒ \(|\overrightarrow{\mathrm{a}} \times \overrightarrow{\mathrm{b}}|=11 \sqrt{2^2+1^2}=11 \sqrt{5}\)

Hence, the area of the parallelogram is \(11 \sqrt{5}\) square units.

Question 11. Show that the direction cosines of a vector equally inclined to the axes OX, OY, and OZ are \(\frac{1}{\sqrt{3}}, \frac{1}{\sqrt{3}}, \frac{1}{\sqrt{3}}\).
Solution:

Let a vector be equally inclined to axes OX, OY, and OZ at angle \(\alpha\).

Then, the direction cosines of the vector are cos\(\alpha\), cos\(\alpha\), and cos\(\alpha\).

Now, \(\cos ^2 \alpha+\cos ^2 \alpha+\cos ^2 \alpha=1 \Rightarrow 3 \cos ^2 \alpha=1 \Rightarrow \cos \alpha= \pm \frac{1}{\sqrt{3}}\)

Hence, the direction cosines of the vector which are equally inclined to the axes are \(\frac{1}{\sqrt{3}}, \frac{1}{\sqrt{3}}, \frac{1}{\sqrt{3}}\) or \(-\frac{1}{\sqrt{3}},-\frac{1}{\sqrt{3}},-\frac{1}{\sqrt{3}}\).

Question 12. Let \(\overrightarrow{\mathrm{a}}=\hat{\mathrm{i}}+4 \hat{\mathrm{j}}+2 \hat{\mathrm{k}}, \overrightarrow{\mathrm{b}}=3 \hat{\mathrm{i}}-2 \hat{\mathrm{j}}+7 \hat{\mathrm{k}}\) and \(\overrightarrow{\mathrm{c}}=2 \hat{\mathrm{i}}-\hat{\mathrm{j}}+4 \hat{\mathrm{k}}\). Find a vector \(\overrightarrow{\mathrm{d}}\) which is perpendicular to both \(\overrightarrow{\mathrm{a}}\) and \(\overrightarrow{\mathrm{b}}\), and \(\overrightarrow{\mathrm{c}} \cdot \overrightarrow{\mathrm{d}}=15\).
Solution:

Vector \(\vec{d}\) is perpendicular to both \(\vec{a}\) and \(\vec{b} \Rightarrow \vec{d}=\lambda(\vec{a} \times \vec{b})\)

Now, \(\overrightarrow{\mathrm{a}} \times \overrightarrow{\mathrm{b}}\)

= \(\left|\begin{array}{ccc}
\hat{\mathrm{i}} & \hat{\mathrm{j}} & \hat{\mathrm{k}} \\
1 & 4 & 2 \\
3 & -2 & 7
\end{array}\right|\)

= \(32 \hat{\mathrm{i}}-\hat{\mathrm{j}}-14 \hat{\mathrm{k}}\)

∴ \(\overrightarrow{\mathrm{d}}=\lambda(32 \hat{\mathrm{i}}-\hat{\mathrm{j}}-14 \hat{\mathrm{k}})\)

Now, \(\overrightarrow{\mathrm{c}} \cdot \overrightarrow{\mathrm{d}}=15 \Rightarrow(2 \hat{\mathrm{i}}-\hat{\mathrm{j}}+4 \hat{\mathrm{k}}) \cdot(32 \lambda \hat{\mathrm{i}}-\lambda \hat{\mathrm{j}}-14 \lambda \hat{\mathrm{k}})=15\)

⇒ 64 \(\lambda+\lambda-56 \lambda=15 \Rightarrow \lambda=\frac{5}{3} \Rightarrow \overrightarrow{\mathrm{d}}=\frac{5}{3}(32 \hat{\mathrm{i}}-\hat{\mathrm{j}}-14 \hat{\mathrm{k}})\)

Question 13. The scalar product of the vector \(\hat{i}+\hat{j}+\hat{k}\) with a unit vector along the sum of the vector \(2 \hat{i}+4 \hat{j}-5 \hat{k}\) and \(\lambda \hat{i}+2 \hat{j}+3 \hat{k}\) is equal to one. Find the value of \(\lambda\).
Solution:

Let, \(\vec{a}=\hat{i}+\hat{j}+\hat{k}, \vec{b}=2 \hat{i}+4 \hat{j}-5 \hat{k}, \vec{c}=\lambda \hat{i}+2 \hat{j}+3 \hat{k}\)

⇒ \(\vec{b}+\vec{c}=(2 \hat{i}+4 \hat{j}-5 \hat{k})+(\lambda \hat{i}+2 \hat{j}+3 \hat{k})=(2+\lambda) \hat{i}+6 \hat{j}-2 \hat{k}\)

Therefore, unit vector along \(\vec{b}+\vec{c}\) is given as:

± \(\frac{(2+\lambda) \hat{i}+6 \hat{j}-2 \hat{k}}{\sqrt{(2+\lambda)^2+6^2+(-2)^2}}= \pm \frac{(2+\lambda) \hat{i}+6 \hat{j}-2 \hat{k}}{\sqrt{4+4 \lambda+\lambda^2+36+4}}= \pm \frac{(2+\lambda) \hat{i}+6 \hat{j}-2 \hat{k}}{\sqrt{\lambda^2+4 \lambda+44}}\)

Scalar product of (\(\vec{a}\)) with this unit vector is 1 .

⇒ \((\hat{i}+\hat{j}+\hat{k}),\left\{ \pm \frac{(2+\lambda) \hat{i}+6 \hat{j}-2 \hat{k}}{\sqrt{\lambda^2+4 \lambda+44}}\right\}=1 \Rightarrow \frac{(2+\lambda)+6-2}{\sqrt{\lambda^2+4 \lambda+44}}= \pm 1\)

⇒ \(\sqrt{\lambda^2+4 \lambda+44}= \pm(\lambda+6) \Rightarrow \lambda^2+4 \lambda+44=(\lambda+6)^2\)

⇒ \(\lambda^2+4 \lambda+44=\lambda^2+12 \lambda+36 \Rightarrow 8 \lambda=8 \Rightarrow \lambda=1\)

Hence, the value of \(\lambda\) is 1 .

Question 14. If \(\vec{a}, \vec{b}, \vec{c}\) are mutually perpendicular vectors of equal magnitudes, show that the vector \(\vec{a}+\vec{b}+\vec{c}\) is equally inclined to \(\vec{a}, \vec{b}\) and \(\vec{c}\).
Solution:

Since \(\vec{a}, \vec{b}\) and \(\vec{c}\) are mutually perpendicular vectors, we have \(\vec{a} \cdot \vec{b}=\vec{b} \cdot \vec{c}=\vec{c} \cdot \vec{a}=0\)

It is given that: \(|\vec{a}|=|\vec{b}|=|\vec{c}|\)

Let vector \(\vec{a}+\vec{b}+\vec{c}\) be inclined to \(\vec{a}, \vec{b}\) and \(\vec{c}\) at angles \(\theta_1, \theta_2\) and \(\theta_3\) respectively.

Then, we have:

cos\(\theta_1=\frac{(\vec{a}+\vec{b}+\vec{c}) \cdot \vec{a}}{|\vec{a}+\vec{b}+\vec{c}||\vec{a}|}=\frac{\vec{a} \cdot \vec{a}+\vec{b} \cdot \vec{a}+\vec{c} \cdot \vec{a}}{|\vec{a}+\vec{b}+\vec{c}||\vec{a}|}\) (\(\vec{b} \cdot \vec{a}=\vec{c} \cdot \vec{a}=0\))

= \(\frac{|\vec{a}|^2}{|\vec{a}+\vec{b}+\vec{c}||\vec{a}|}=\frac{|\vec{a}|}{|\vec{a}+\vec{b}+\vec{c}|}\)

cos\(\theta_2=\frac{(\vec{a}+\vec{b}+\vec{c}) \cdot \vec{b}}{|\vec{a}+\vec{b}+\vec{c}||\vec{b}|}=\frac{\vec{a} \cdot \vec{b}+\vec{b} \cdot \vec{ba}+\vec{c} \cdot \vec{b}}{|\vec{a}+\vec{b}+\vec{c}||\vec{b}|}\) (\(\vec{b} \cdot \vec{a}=\vec{c} \cdot \vec{b}=0\))

= \(\frac{|\vec{b}|^2}{|\vec{a}+\vec{b}+\vec{c}||\vec{b}|}=\frac{|\vec{b}|}{|\vec{a}+\vec{b}+\vec{c}|}\)

cos\(\theta_3=\frac{(\vec{a}+\vec{b}+\vec{c}) \cdot \vec{c}}{|\vec{a}+\vec{b}+\vec{c}||\vec{c}|}=\frac{\vec{a} \cdot \vec{c}+\vec{b} \cdot \vec{c}+\vec{c} \cdot \vec{c}}{|\vec{a}+\vec{b}+\vec{c}||\vec{c}|}\)

= \(\frac{|\vec{c}|^2}{\mid \vec{a}+\vec{b}+\vec{c}=0]}=\frac{|\vec{c}|}{|\vec{a}+\vec{b}+\vec{c}|}\)

Now, as \(|\vec{a}|=|\vec{b}|=|\vec{c}|\)

∴ \(\cos \theta_1=\cos \theta_2=\cos \theta_3 \Rightarrow \theta_1=\theta_2=\theta_3\)

Hence, the vector \((\vec{a}+\vec{b}+\vec{c})\) is equally inclined to \(\vec{a}, \vec{b}\) and \(\vec{c}\).

Question 15. Prove that \((\vec{a}+\vec{b}) \cdot(\vec{a}+\vec{b})=|\vec{a}|^2+|\vec{b}|^2\), if and only if \(\vec{a}, \vec{b}\) are perpendicular, given \(\vec{a} \neq \overrightarrow{0}, \vec{b} \neq \overrightarrow{0}\).
Solution:

Let \((\vec{a}+\vec{b})(\vec{a}+\vec{b})=|\vec{a}|^2+|\vec{b}|^2\)

⇒ \(\vec{a} \cdot \vec{a}+\vec{a} \cdot \vec{b}+\vec{b} \cdot \vec{a}+\vec{b} \cdot \vec{b}=|\vec{a}|^2+|\vec{b}|^{-2}\) [Distributivity of scalar products over addition]

⇒ \(|\vec{a}|^2+2 \vec{a} \cdot \vec{b}+|\vec{b}|^2=|\vec{a}|^2+|\vec{b}|^2 \quad[\vec{a} \cdot \vec{b}=\vec{b} \cdot \vec{a}\) (Scalar product is commutative)

⇒ \(2 \vec{a} \cdot \vec{b}=0 \quad \Rightarrow \vec{a} \cdot \vec{b}=0\)

∴ \(\vec{a}\) and \(\vec{b}\) are perpendicular. \(\vec{a} \neq \overrightarrow{0}, \vec{b} \neq \overrightarrow{0}\) (Given)

Further, let \(\vec{a} \perp \vec{b} \Rightarrow \vec{a} \cdot \vec{b}=0\)

Now \((\vec{a}+\vec{b}) \cdot(\vec{a}+\vec{b})=\vec{a} \cdot \vec{a}+\vec{a} \cdot \vec{b}+\vec{b} \cdot \vec{a}+\vec{b} \cdot \vec{b}=|\vec{a}|^2+|\vec{b}|^2+2(\vec{a} \cdot \vec{b}) \cdot\)

= \(|\vec{a}|^2+|\vec{b}|^2\) (because \(\vec{a} \cdot \vec{b}=0\))

Hence, \((\vec{a}+\vec{b})(\vec{a}+\vec{b})=|\vec{a}|^2+|\vec{b}|^2\)

Choose The Correct Answer

Question 16. If \(\theta\) is the angle between two vectors \(\vec{a}\) and \(\vec{b}\), then \(\vec{a} \cdot \vec{b} \geq 0\) only when:

  1. \(0<\theta<\frac{\pi}{2}\)
  2. \(0 \leq \theta \leq \frac{\pi}{2}\)
  3. \(0<\theta<\pi\)
  4. \(0 \leq \theta \leq \pi\)

Solution: 2. \(0 \leq \theta \leq \frac{\pi}{2}\)

Let \(\theta\) be the angle between two vectors \(\vec{a}\) and \(\vec{b}\), if \(\vec{a} \cdot \vec{b} \geq 0\)

⇒ \(|\vec{a}||\vec{b}| \cos \theta \geq 0 \Rightarrow \cos \theta \geq 0\)

⇒ \([|\vec{a}| and |\vec{b}|\) are positive]

⇒ \(0 \leq \theta \leq \frac{\pi}{2}\)

Hence, \(\vec{a} \cdot \vec{b} \geq 0\) when \(0 \leq \theta \leq \frac{\pi}{2}\).

The correct answer is (B).

Question 17. Let \(\vec{a}\) and \(\vec{b}\) be two unit vectors and \(\theta\) is the angle between them. Then \(\vec{a}+\vec{b}\) is a unit vector if :

  1. \(\theta=\frac{\pi}{4}\)
  2. \(\theta=\frac{\pi}{3}\)
  3. \(\theta=\frac{\pi}{2}\)
  4. \(\theta=\frac{2 \pi}{3}\)

Solution: \(\theta=\frac{2 \pi}{3}\)

Let \(\vec{a}\) and \(\vec{b}\) be two unit vectors and \(\theta\) be the angle between them. Then, \(|\vec{a}|=|\vec{b}|=1\).

Now, \(\vec{a}+\vec{b}\) is a unit vector then \(|\vec{a}+\vec{b}|=1\)

⇒ \(|\vec{a}+\vec{b}|^2=1 \Rightarrow(\vec{a}+\vec{b}) \cdot(\vec{a}+\vec{b})=1\)

⇒ \(\vec{a} \cdot \vec{a}+\vec{a} \cdot \vec{b}+\vec{b} \cdot \vec{a}+\vec{b} \cdot \vec{b}=1\) (because \(\vec{a} \cdot \vec{a}=|\vec{a}|^2)\))

⇒ \(|\vec{a}|^2+2 \vec{a} \cdot \vec{b}+|\vec{b}|^2=1 \Rightarrow 1+2|\vec{a}||\vec{b}| \cos \theta+1=1 \Rightarrow \cos \theta=-\frac{1}{2} \Rightarrow \theta=\frac{2 \pi}{3}\)

Hence, \(\vec{a}+\vec{b}\) is a unit vector if \(\theta=\frac{2 \pi}{3}\).

The correct answer is (4).

Question 18. The value of \(\hat{i} \cdot(\hat{j} \times \hat{k})+\hat{j} \cdot(\hat{i} \times \hat{k})+\hat{k} \cdot(\hat{i} \times \hat{j})\) is

  1. 0
  2. -1
  3. 1
  4. 3

Solution: 3. 1

⇒ \(\hat{i} \cdot(\hat{j} \times \hat{k})+\hat{j} \cdot(\hat{i} \times \hat{k})+\hat{k} \cdot(\hat{i} \times \hat{j})=\hat{i} \cdot \hat{i}+\hat{j} \cdot(-\hat{j})+\hat{k} \cdot \hat{k}=1-1+1=1\)

The correct answer is (3).

Question 19. If \(\theta\) is the angle between any two vectors \(\vec{a}\) and \(\vec{b}\), then \(|\vec{a} \cdot \vec{b}|=|\vec{a} \times \vec{b}|\) when \(\theta\) is equal to

  1. 0
  2. \(\frac{\pi}{4}\)
  3. \(\frac{\pi}{2}\)
  4. \(\pi\)

Solution: 2. \(\frac{\pi}{4}\)

Let \(\theta\) be the angle between two vectors \(\vec{a}\) and \(\vec{b}\).

⇒ \(|\vec{a} \cdot \vec{b}|=|\vec{a} \times \vec{b}| \Rightarrow|\vec{a}||\vec{b}| \cos \theta=|\vec{a}||\vec{b}| \sin \theta\)

cos \(\theta=\sin \theta \Rightarrow \tan \theta=1 \Rightarrow \theta=\frac{\pi}{4}\)

Hence, \(|\vec{a} \cdot \vec{b}|=|\vec{a} \times \vec{b}|\) when \(\theta\) is equal to \(\frac{\pi}{4}\).

The correct answer is (2).

 

 

 

 

 

 

 

 

 

 

 

 

 

Differential Equations Class 12 Maths Important Questions Chapter 9

Differential Equation Exercise 9.1

Determine the Order And Degree (If Defined) Of Differential Equations

Question 1. \(\frac{d^4 y}{d x^4}+\sin \left(y^{\prime \prime \prime}\right)=0\)
Solution:

⇒ \(\frac{\mathrm{d}^4 \mathrm{y}}{\mathrm{dx}}+\sin \left(\mathrm{y}^{\prime \prime \prime}\right)=0 \Rightarrow \mathrm{y}^{\prime \prime \prime \prime}+\sin \left(\mathrm{y}^{\prime \prime \prime}\right)=0\)

The highest order derivative present in the differential equation is y””. Therefore, its order is four.

The given differential equation is not a polynomial equation in its derivatives. Hence, its degree is not defined.

Question 2. y’+5y = 0
Solution:

The given differential equation is: y’ + 5y = 0

The highest-order derivative present in the differential equation is y’. Therefore, its order is one.

It is a polynomial equation in y’. The highest power raised to y’ is 1. Hence, its degree is one.

Question 3. \(\left(\frac{\mathrm{ds}}{\mathrm{dt}}\right)^4+3 \mathrm{~s} \frac{\mathrm{d}^2 \mathrm{~s}}{\mathrm{dt}^2}=0\)
Solution:

The highest order derivative present in the given differential equation is \(\frac{\mathrm{d}^2 \mathrm{~s}}{\mathrm{dt}^2}=0\). Therefore, its order is two.

It is a polynomial equation in \(\frac{\mathrm{d}^2 \mathrm{~s}}{\mathrm{dt}^2}=0\) and \(\frac{ds}{dt}\). The power raised to \(\frac{\mathrm{d}^2 \mathrm{~s}}{\mathrm{dt}^2}=0\) is 1.

Hence, its degree is one.

Read and Learn More Class 12 Maths Chapter Wise with Solutions

Question 4. \(\left(\frac{d^2 y}{d x^2}\right)^2+\cos \left(\frac{d y}{d x}\right)=0\)
Solution:

The highest order derivative present in the given differential equation is \(\frac{d^2 y}{d x^2}\) order is 2.

The given differential equation is not a polynomial equation in its derivatives. Hence, its degree is not defined.

Question 5. \(\frac{d^2 y}{d x^2}=\cos 3 x+\sin 3 x\)
Solution:

⇒ \(\frac{d^2 y}{d x^2}=\cos 3 x+\sin 3 x \Rightarrow \frac{d^2 y}{d x^2}-\cos 3 x-\sin 3 x=0\)

The highest order derivative present in the differential equation is \(\frac{d^2 y}{d x^2}\). Therefore, its order is two.

It is a polynomial equation in \(\frac{d^2 y}{d x^2}\) and the power raised to \(\frac{d^2 y}{d x^2}\) is 1.

Hence, its degree is one.

CBSE Class 12 Maths Chapter 9 Differential Equations Important Question And Answers

Question 6. \(\left(y^{\prime \prime \prime}\right)^2+\left(y^{\prime \prime}\right)^3+\left(y^{\prime}\right)^4+y^3=0\)
Solution:

The highest order derivative present in the differential equation is \(\left(y^{\prime \prime \prime}\right)\).

Therefore, its order is three. The given differential equation is a polynomial equation \(iny^{\prime \prime \prime}, y^{\prime \prime} \text { and } y^{\prime} \text {. }\)

The highest power raised to \(y^{\prime \prime \prime}\) is 2. Hence, its degree is 2.

Question 7. \(y^{\prime \prime \prime}+2 y^{\prime \prime}+y^{\prime}=0\)
Solution:

The highest order derivative present in the differential equation is \(y^{\prime \prime \prime}\). Therefore, its order is three.It is a polynomial equation in \(y^{\prime \prime \prime}\), y” and y’. The highest power raised to y is 1. Hence, its degree is 1.

Question 8. y’ + y = ex
Solution:

y’ + y = ex ⇒ y’ + y = ex = 0

The highest-order derivative present in the differential equation is y’. Therefore, its order is one. The given differential equation is a polynomial equation and the highest power raised to y’ is one. Hence, its degree is one.

 

Quetsion 9. y”+(y’)² + 2y = 0
Solution:

The highest order derivative present in the differential equation is Therefore, its order is two.

The given differential equation is a polynomial equation in y” and y’ and the highest power raised to y” is one.

Hence, its degree is one. y”+ 2y’+ sin y = 0

Question 10. y” + 2y’ + sin y = 0
Solution:

The highest order derivative present in the differential equation is y”. Therefore, its order is two. This is a polynomial equation in y” and y’, and the highest power raised to y” is one.

Hence, its degree is one.

Question 11. The degree of the differential equation \(\left(\frac{d^2 y}{d x^2}\right)^3+\left(\frac{d y}{d x}\right)^2+\sin \left(\frac{d y}{d x}\right)+1=0\) is

  1. 3
  2. 2
  3. 1
  4. Not Defined

Solution:

The given differential equation is not a polynomial equation in its derivatives. Therefore, its degree is not defined.

Hence, the correct answer is D.

Question 12. The order of the differential equation \(2 x^2 \frac{d^2 y}{d x^2}-3 \frac{d y}{d x}+y=0\)

  1. 2
  2. 1
  3. 0
  4. Not Defined

Solution:

The highest order derivative present in the given differential equation is \(\frac{\mathrm{d}^2 \mathrm{y}}{\mathrm{dx}^2}\). Therefore, its order is two.

Hence, the correct answer is A.

Differential Equation Exercise 9.2

Verify That The Given Function (Explicit Or Implicit) Is A Solution Of The Corresponding Differential Equation:

Question 1. y = ex + 1 : y” – y’ = 0
Solution:

y = ex + 1

Differentiating both sides of this equation with respect to x, we get: \(\frac{d y}{d x}=\frac{d}{d x}\left(e^x+1\right) \Rightarrow y^{\prime}=e^x\)….(1)

Now, again differentiating equation (1) with respect to x, we get: \(\frac{d}{d x}\left(y^{\prime}\right)=\frac{d}{d x}\left(e^x\right) \Rightarrow y^{\prime \prime}=e^x\)

Substituting the values of y’ and y” in the given differential equation, we get the L.H.S. as y” – y’ = ex – ex = 0 = R.H.S.

Thus, the given function is the solution of the corresponding differential equation.

Question 2. y = x² + 2x + C : y’ – 2x – 2 = 0
Solution:

y = x² + 2x + C

Differentiating both sides of this equation with respect to x, we get:

y’ = \(\frac{d}{dx}\)(X² + 2X + C) ⇒ y’ = 2x+2 dx

Substituting the value of y’ in the given differential equation, we get:

L.H.S. = y’ – 2x – 2 =2x + 2-2x-2 = 0 = R.H.S.

Hence, the given function is the solution of the corresponding differential equation.

Question 3. y = cos x + C : y’ + sin x = 0
Solution:

y = cos x + C

Differentiating both sides of this equation with respect to x, we get:

y’ = \(\frac{d}{dx}\)(cos x + C) ⇒ y’ = -sin x

Substituting the value of y’ in the given differential equation, we get:

L.H.S. = y’ + sinx = – sinx + sinx = 0 = R.H.S.

Hence, the given function is the solution of the corresponding differential equation.

Question 4. \(y=\sqrt{1+x^2}: y^{\prime}=\frac{x y}{1+x^2}\)
Solution:

y = \(\sqrt{1+x^2}\)

Differentiating both sides of the equation with respect to x, we get:

⇒ \(y^{\prime}=\frac{d}{d x}\left(\sqrt{1+x^2}\right) \Rightarrow y^{\prime}=\frac{1}{2 \sqrt{1+x^2}} \cdot \frac{d}{d x}\left(1+x^2\right)\)

⇒ \(y^{\prime}=\frac{2 x}{2 \sqrt{1+x^2}}\)

⇒ \(y^{\prime}=\frac{x}{\sqrt{1+x^2}} \Rightarrow y^{\prime}=\frac{x}{1+x^2} \times \sqrt{1+x^2}\)

⇒ \(y^{\prime}=\frac{x}{1+x^2} \cdot y \Rightarrow y^{\prime}=\frac{x y}{1+x^2}\)

∴ L.H.S. = R.H.S.

Hence, the given function is the solution of the corresponding differential solution.

Question 5. y = Ax : xy’ = y (x ≠ 0)
Solution:

y = Ax

Differentiating both sides of the equation with respect to x, we get: \(y^{\prime}=\frac{d}{d x}(A x) \Rightarrow y^{\prime}=A\)

Substituting the value of y’ in the given differential equation, we get:

L.H.S. = xy’ = x-A = Ax = y = R.H.S.

Hence, the given function is the solution of the corresponding differential equation.

Question 6. \(y=x \sin x \quad: \quad x y^{\prime}=y+x \sqrt{x^2-y^2} \quad[x \neq 0 \text { and } x>y \text { or } x<-y]\)
Solution:

y = x sin x…..(1)

Differentiating both sides of this equation with respect to x, we get: \(y^{\prime}=\frac{d}{d x}(x \sin x)\)

⇒ \(y^{\prime}=\sin x \cdot \frac{d}{d x}(x)+x \cdot \frac{d}{d x}(\sin x) \Rightarrow y^{\prime}=\sin x+x \cos x\)

Substituting the value of y’ in the given differential equation, we get:

L.H.S. = x y’ = x(sin x + x cos x) = x sin x + x² cos x

= \(y+x^2 \cdot \sqrt{1-\sin ^2 x}=y+x^2 \sqrt{1-\left(\frac{y}{x}\right)^2}\) [Using Eq.(1)]

= \(y+x \sqrt{x^2-y^2}=\) R.H.S.

Hence, the given function is the solution of the corresponding differential equation

Question 7. \(x y=\log y+C: y^{\prime}=\frac{y^2}{1-x y} \quad(x y \neq 1)\)
Solution:

y – cos y = x….(1)

Differentiating both sides of the equation with respect to x, we get: \(\frac{d y}{d x}-\frac{d}{d x}(\cos y)=\frac{d}{d x}(x) \Rightarrow y^{\prime}+\sin y \cdot y^{\prime}\)=1

⇒ \(y^{\prime}(1+\sin y)=1 \Rightarrow y^{\prime}=\frac{1}{1+\sin y}\)….(2)

Substituting the value of y’ in the given differential equation, we get:

L.H.S. = \((y \sin y+\cos y+x) y^{\prime}=(y \sin y+\cos y+y-\cos y) \times \frac{1}{1+\sin y}\) [Using Eq.(1) and (2)]

= \(y(1+\sin y) \cdot \frac{1}{1+\sin y}=y=\text { R.H.S. }\)

Hence, the given function is the solution of the corresponding differential equation

Question 9. \(x+y=\tan ^{-1} y: y^2 y^{\prime}+y^2+1=0\)
Solution:

x+y = \(\tan ^{-1} y\)

Differentiating both sides of this equation with respect to x, we get:

⇒ \(\frac{d}{d x}(x+y)=\frac{d}{d x}\left(\tan ^{-1} y\right) \Rightarrow 1+y^{\prime}\)

= \(\left[\frac{1}{1+y^2}\right] y^{\prime} \Rightarrow y^{\prime}\left[\frac{1}{1+y^2}-1\right]=1\)

⇒ \(y^{\prime}\left[\frac{1-\left(1+y^2\right)}{1+y^2}\right]=1 \Rightarrow y^{\prime}\left[\frac{-y^2}{1+y^2}\right]\)=1

⇒ \(y^{\prime}=\frac{-\left(1+y^2\right)}{y^2}\)

Substituting the value of \(y^{\prime}\) in the given differential equation, \(y^2 y^1+y^2+1=0\) we get:

L.H.S. = \(y^2y^{\prime}+y^2+1=y^2\left[\frac{-\left(1+y^2\right)}{y^2}\right]+y^2+1=-1-y^2+y^2+1=0\)

Hence, the given function is the solution of the corresponding differential equation.

Question 10. y = \(\sqrt{a^2-x^2}, x \in(-a, a) \quad: x+y \frac{d y}{d x}=0(y \neq 0)\)
Solution:

y = \(\sqrt{a^2-x^2}\)….(1)

Differentiating both sides of this equation with respect to x, we get

⇒ \(\frac{d y}{d x}=\frac{d}{d x}\left(\sqrt{a^2-x^2}\right)\)

⇒ \(\frac{d y}{d x}=\frac{1}{2 \sqrt{a^2-x^2}} \cdot \frac{d}{d x}\left(a^2-x^2\right)=\frac{1}{2 \sqrt{a^2-x^2}}(-2 x)=\frac{-x}{\sqrt{a^2-x^2}}\)…(2)

Substituting the value of \(\frac{dy}{dx}\) in the given differential equation, we get

L.H.S. = \(x+y \frac{d y}{d x}=x+\sqrt{a^2-x^2} \times \frac{-x}{\sqrt{a^2-x^2}}\) (Using eq(1) and (2))

Hence, the given function is the solution of the corresponding differential equation.

Question 11. The number of arbitrary constants in the general solution of a differential equation of fourth order are:

  1. 0
  2. 2
  3. 3
  4. 4

Solution: 4. 4

We know that the number of constants in the general solution of a differential equation of order n is equal to its order.

Therefore, the number of constants in the general equation of fourth order differential equation is four.

Hence, the correct answer is (4).

Question 12. The number of arbitrary constants in the particular solution of a differential equation of third order are:

  1. 3
  2. 2
  3. 1
  4. 0

Solution: 4. 0

In a particular solution of a differential equation, there are no arbitrary constants.

Hence, the correct answer is (4).

Differential Equation Exercise 9.3

For Each Of The Differential Equations, Find The General Solution

Question 1. \(\frac{d y}{d x}=\frac{1-\cos x}{1+\cos x}\)
Solution:

The given differential equation is

⇒ \(\frac{d y}{d x}=\frac{1-\cos x}{1+\cos x} \Rightarrow \frac{d y}{d x}=\frac{2 \sin ^2 \frac{x}{2}}{2 \cos ^2 \frac{x}{2}}\)

= \(\tan ^2 \frac{x}{2} \quad \Rightarrow \frac{d y}{d x}=\left(\sec ^2 \frac{x}{2}-1\right)\)

Separating the variables, we get: \(\mathrm{dy}=\left(\sec ^2 \frac{\mathrm{x}}{2}-1\right) \mathrm{dx}\)

Now, integrating both sides of this equation, we get:

⇒ \(\int d y=\int\left(\sec ^2 \frac{x}{2}-1\right) d x=\int \sec ^2 \frac{x}{2} d x-\int d x \Rightarrow y=2 \tan \frac{x}{2}-x+C\)

This is the required general solution of the given differential equation.

Question 3. \(\frac{d y}{d x}+y=1 \quad(y \neq 1)\)
Solution:

The given differential equation is: \(\frac{d y}{d x}+y=1 \Rightarrow \frac{d y}{d x}=1-y\)

Separating the variables, we get: \(\frac{d y}{1-y}=d x\)

Now, integrating both sides of this equation, we get: \(\int \frac{d y}{1-y}=\int d x\)

⇒ \(-\log (1-y)=x+\log C \Rightarrow-\log C-\log (1-y)=x \Rightarrow \log C(1-y)=-x\)

⇒ \(C(1-y)=e^{-x} \Rightarrow 1-y=\frac{1}{C} e^{-x} \Rightarrow y=1-\frac{1}{C} e^{-x} \Rightarrow y=1+A e^{-x}\)

(where  A = \(-\frac{1}{C}\))

This is the required general solution of the given differential equation.

Question 4. sec²x tan y dx + sec²y tan x dy =0
Solution:

The given differential equation is: sec²x tan y dx + sec²y tan x dy = 0

⇒ sec²x tan y dx = -sec²y tan x dy

On separating the variables, we get:

⇒ \(\frac{\sec ^2 x}{\tan x} d x=-\frac{\sec ^2 y}{\tan y} d y\)

Integrating both sides of this equation, we get: \(\int \frac{\sec ^2 x}{\tan x} d x=-\int \frac{\sec ^2 y}{\tan y} d y\)

⇒ log |tan x| = -log |tan y| = log C (\(\int \frac{f^{\prime}(x)}{f(x)} d x=\log |f(x)|+C\))

⇒ log |tan x| + log |tan y| = log C

⇒ tan x tan y = C

This is the required general solution of the given differential equation.

Question 5. (ex + e-x)dy – (ex – e-x)dx = 0
Solution:

The given differential equation is: (ex + e-x)dy – (ex – e-x)dx = 0 ⇒ (ex+ e-x)dy = (ex – e-x)dx

Separating the variables, we get: \(d y=\left[\frac{e^x-e^{-x}}{e^x+e^{-x}}\right] d x\)

Integrating both sides of this equation, we get \(\int d y=\int\left[\frac{e^x-e^{-x}}{e^x+e^{-x}}\right] d x+C\)

⇒ y = \(\int\left[\frac{e^x-e^{-x}}{e^x+e^{-x}}\right] d x+C\)

⇒ y = \(\log \left(e^x+e^{-x}\right)+C\)

This is the required general solution of the given differential equation.

Question 6. \(\frac{d y}{d x}=\left(1+x^2\right)\left(1+y^2\right)\)
Solution:

The given differential equation is: \(\frac{d y}{d x}=\left(1+x^2\right)\left(1+y^2\right) \Rightarrow \frac{d y}{1+y^2}=\left(1+x^2\right) d x\)

Integrating both sides of this equation, we get:

⇒ \(\int \frac{d y}{1+y^2}=\int\left(1+x^2\right) d x \Rightarrow \tan ^{-1} y=\int d x+\int x^2 d x \Rightarrow \tan ^{-1} y=x+\frac{x^3}{3}+C\)

This is the required general solution of the given differential equation.

Question 7. y log y dx – x dy = 0
Solution:

The given differential equation is : y log y dx – x dy = 0 ⇒ y log y dx = x dy

Separating the variables, we get: \(\frac{d y}{y \log y}=\frac{d x}{x}\)

Integrating both sides, we get: \(\int \frac{d y}{y \log y}=\int \frac{d x}{x}\)….(1)

Let log y=t ⇒ \(\frac{1}{y} d y=d t\)

Substituting this value in equation (1), we get:

⇒ \(\int \frac{d t}{t}=\int \frac{d x}{x} \Rightarrow \log t=\log x+\log C \Rightarrow \log (\log y)=\log C x\)

⇒ \(\log y=C x \Rightarrow y=e^c\)

This is the required general solution of the given differential equation.

Question 8. \(x^5 \frac{d y}{d x}=-y^5\)
Solution:

The given differential equation is: \(x^5 \frac{d y}{d x}=-y^5\)

Separating the variables, we get: \(\frac{d y}{y^5}=-\frac{d x}{x^5} \Rightarrow \frac{d x}{x^5}+\frac{d y}{y^5}=0\)

Integrating both sides, we get: \(\int \frac{d x}{x^5}+\int \frac{d y}{y^5}=k\) where k is any constant

⇒ \(\int x^{-5} d x+\int y^{-5} d y=k \Rightarrow \frac{x^{-4}}{-4}+\frac{y^{-4}}{-4}=k \Rightarrow x^{-4}+y^{-4}=-4 k\)

⇒ \(x^{-4}+y^{-4}=C \quad(C=-4 k)\)

This is the required general solution of the given differential equation.

Question 9. \(\frac{d y}{d x}=\sin ^{-1} x\)
Solution:

The given differential equation \(\frac{d y}{d x}=\sin ^{-1} x \Rightarrow d y=\sin ^{-1} x d x\)

Integrating both sides, we get Integrating both sides, we get : \(\int d y=\int \sin ^{-1} x d x \Rightarrow y=\int\left(\sin ^{-1} x\right) \cdot 1 d x\)

⇒y = \(\sin ^{-1} x \cdot \int(1) d x-\int\left[\left(\frac{d}{d x}\left(\sin ^{-1} x\right) \cdot \int(1) d x\right)\right] d x\)

⇒ \(y=\sin ^{-1} x \cdot x-\int\left(\frac{1}{\sqrt{1-x^2}} \cdot x\right) d x\)

⇒ y = \(x \sin ^{-1} x+\int \frac{-x}{\sqrt{1-x^2}} d x\)….(1)

Let \(1-x^2=t \Rightarrow-2 x d x=d t\)

Substituting these values in equation (1), we get

y = \(x \sin ^{-1} x+\int \frac{1}{2 \sqrt{t}} d t \Rightarrow y=x \sin ^{-1} x+\frac{1}{2} \cdot \int(t)^{-\frac{1}{2}} d t\)

⇒ \(y=x \sin ^{-1} x+\frac{1}{2} \cdot \frac{t^{\frac{1}{2}}}{\frac{1}{2}}+C\)]

⇒ y = \(x \sin ^{-1} x+\sqrt{t}+C \Rightarrow y=x \sin ^{-1} x+\sqrt{1-x^2}+C\)

This is the required general solution of the given differential equation.

Question 10. ex tan y dx + (1-ex) sec²ydy = 0
Solution:

The given differential equation is: \(e^x \tan y d x+\left(1-e^x\right) \sec ^2 y d y=0 \Rightarrow\left(1-e^x\right) \sec ^2 y d y=-e^x \tan y d x\)

Separating the variables, we get \(\frac{\sec ^2 y}{\tan y} d y=\frac{-e^x}{1-e^x} d x\)

Integrating both sides, we get: \(\int \frac{\sec ^2 y}{\tan y} d y=\int \frac{-e^x}{1-e^x} d x\)

⇒ \(\log (\tan y)=\log \left(1-e^x\right)+\log C\) (because \(\int \frac{f^{\prime}(x)}{f(x)} d x=\log |f(x)|+C\))

⇒ \(\log (\tan y)=\log \left[C\left(1-e^x\right)\right]\)

⇒ \(\tan y=C\left(1-e^x\right)\)

This is the required general solution of the given differential equation.

For each of the differential equations, find a particular solution satisfying the given condition

Question 11. \(\left(x^3+x^2+x+1\right) \frac{d y}{d x}=2 x^2+x ; y=1\) when x=0
Solution:

The given differential equation is : \(\left(x^3+x^2+x+1\right) \frac{d y}{d x}=2 x^2+x \Rightarrow \frac{d y}{d x}=\frac{2 x^2+x}{\left(x^3+x^2+x+1\right)}\)

⇒ d y = \(\frac{2 x^2+x}{(x+1)\left(x^2+1\right)} d x\)

Integrating both sides, we get: \(\int d y=\int \frac{2 x^2+x}{(x+1)\left(x^2+1\right)} d x\)…..(1)

Let \(\frac{2 x^2+x}{(x+1)\left(x^2+1\right)}=\frac{A}{x+1}+\frac{B x+C}{x^2+1}\)……(2)

⇒ \(\frac{2 x^2+x}{(x+1)\left(x^2+1\right)}=\frac{A x^2+A+(B x+C)(x+1)}{(x+1)\left(x^2+1\right)}\)

⇒ \(2 x^2+x=A x^2+A+B x^2+B x+C x+C\)

⇒ \(2 x^2+x=(A+B) x^2+(B+C) x+(A+C)\)

Comparing the coefficients of x² and x, we get:

A + B = 2, B + C = 1,A + C = 0

Solving these equations, we get: A = 1/2, B = 3/2 and C = -1/2

Substituting the values of A, B, and C in equation (2), we get:

⇒ \(\frac{2 x^2+x}{(x+1)\left(x^2+1\right)}=\frac{1}{2} \cdot \frac{1}{(x+1)}+\frac{1}{2} \frac{(3 x-1)}{\left(x^2+1\right)}\)

Therefore, equation (1) becomes:

⇒ \(\int d y=\frac{1}{2} \int \frac{1}{x+1} d x+\frac{1}{2} \int \frac{3 x-1}{x^2+1} d x\)

⇒ \(y=\frac{1}{2} \log (x+1)+\frac{3}{2} \int \frac{x}{x^2+1} d x-\frac{1}{2} \int \frac{1}{x^2+1} d x\)

⇒ \(y=\frac{1}{2} \log (x+1)+\frac{3}{4} \cdot \int \frac{2 x}{x^2+1} d x-\frac{1}{2} \tan ^{-1} x+C\)

⇒ \(y=\frac{1}{2} \log (x+1)+\frac{3}{4} \log \left(x^2+1\right)-\frac{1}{2} \tan ^{-1} x+C \)

⇒ \(y=\frac{1}{4}\left[2 \log (x+1)+3 \log \left(x^2+1\right)\right]-\frac{1}{2} \tan ^{-1} x+C\)

⇒ \(y=\frac{1}{4}\left[\log (x+1)^2\left(x^2+1\right)^3\right]-\frac{1}{2} \tan ^{-1} x+C\)…(3)

Now, y=1 when x=0

⇒ \(1=\frac{1}{4} \log (1)-\frac{1}{2} \tan ^{-1} 0+C \Rightarrow 1=\frac{1}{4} \times 0-\frac{1}{2} \times 0+C \Rightarrow C=1\)

Substituting C=1 in equation (3), we get:

y = \(\frac{1}{4}\left[\log (x+1)^2\left(x^2+1\right)^3\right]-\frac{1}{2} \tan ^{-1} x+1\)

which is the required particular solution.

Question 12. \(x\left(x^2-1\right) \frac{d y}{d x}=1 ; y=0\) when x=2
Solution:

⇒ \(x\left(x^2-1\right) \frac{d y}{d x}=1\)

On separating the variables, we get : \(d y=\frac{d x}{x\left(x^2-1\right)} \Rightarrow \int d y=\int \frac{d x}{x^3\left(1-\frac{1}{x^2}\right)}\)

Put \(1-\frac{1}{x^2}=t \Rightarrow \frac{2}{x^3} d x=d t\),

⇒\(\int \mathrm{dy}=\frac{1}{2} \int \frac{\mathrm{dt}}{\mathrm{t}} \Rightarrow \mathrm{y}=\frac{1}{2} \log (\mathrm{t})+\log \mathrm{C}\)

⇒ \(\mathrm{y}=\frac{1}{2} \log \left|\frac{\mathrm{x}^2-1}{\mathrm{x}^2}\right|+\log \mathrm{C}\)…(1)

Now, y = 0 when x = 2

0 = \(\frac{1}{2} \log \left(\frac{3}{4}\right)+\log \mathrm{C}\) (From eq.(1))

(\(\log \mathrm{C}=-\frac{1}{2} \log \left(\frac{3}{4}\right)\))

log \(\mathrm{C}=\frac{1}{2} \log \left(\frac{4}{3}\right)\)

⇒ \(\mathrm{y}=\frac{1}{2} \log \left(\frac{\mathrm{x}^2-1}{\mathrm{x}^2}\right)+\frac{1}{2} \log \left(\frac{4}{3}\right)\)

⇒ \(\mathrm{y}=\frac{1}{2} \log \left[\frac{4\left(\mathrm{x}^2-1\right)}{3 \mathrm{x}^2}\right]\)

which is the required particular solution.

Question 13. \(\cos \left(\frac{d y}{d x}\right)=a(a \in R) ; y=1\) when x=0
Solution:

cos \(\left(\frac{d y}{d x}\right)=a \Rightarrow \frac{d y}{d x}=\cos ^{-1} a\)

On separating the variables, we get, dy = cos-1 a dx

Integrating both sides, we get: ∫dy = cos-1 a ∫dx

⇒ y = cos-1 a x + C ⇒ y = x cos-1 a + C….(1)

Now, y = 1 when x = 0 ⇒ 1 = 0 · cos-1 a + C ⇒ C = 1

Substituting C= 1 in equation (1), we get:

y = \(x \cos ^{-1} a+1 \Rightarrow \frac{y-1}{x}=\cos ^{-1} a \Rightarrow \cos \left(\frac{y-1}{x}\right)=a\)

which is the required particular solution.

Question 14. \(\frac{dy}{dx}\)= y tan x ; y = 1 when x = 0
Solution:

⇒ \(\frac{dy}{dx}\)= y tan x

Separating the variables, we get: \(\frac{dy}{dx}\) = tan x dx

Integrating both sides, we get: ∫\(\frac{dy}{dx}\) = ∫tan x dx

⇒ log y = log |sec x| + log C

⇒ log y = log(C sec x) ⇒ y C sec x……(1)

Now, y = 1 when x = 0 ⇒ 1= C x sec0 ⇒ 1 = C x 1 ⇒ C = 1

Substituting C = 1 in equation (1), we get y = sec x which is the required particular solution.

Question 15. Find the equation of a curve passing through the point (0, 0) and whose differential equation is y’ = ex sin x
Solution:

The differential equation of the curve is: y’ = ex sin x

⇒ \(\frac{dy}{dx}\) = ex sin x

Separating the variables, we get: dy = ex sin x dx

Integrating both sides, we get: J∫dy =∫ex sin x dx….(1)

Let I = \(\int{e^x} \sin x d x\)

⇒ \(I=\sin x \int e^x d x-\int\left(\frac{d}{d x}(\sin x) \cdot \int e^x d x\right) d x \Rightarrow I=\sin x \cdot e^x-\int \cos x \cdot e^x d x\)

⇒ \(I=\sin x \cdot e^x-\left[\cos x \cdot \int e^x d x-\int\left(\frac{d}{d x}(\cos x) \cdot \int e^x d x\right) d x\right]\)

⇒ \(I=\sin x e^x-\left[\cos x \cdot e^x-\int(-\sin x) \cdot e^x d x\right] \Rightarrow I=e^x \sin x-e^x \cos x-I\)

⇒ \(2 I=e^x(\sin x-\cos x) \Rightarrow I=\frac{e^x(\sin x-\cos x)}{2}\)

Substituting this value in equation (1), we get:

y = \(\frac{e^x(\sin x-\cos x)}{2}+C\)…(2)

Now, the curve passes through point (0,0).

∴ \(0=\frac{\mathrm{e}^0(\sin 0-\cos 0)}{2}+\mathrm{C} \Rightarrow 0=\frac{1(0-1)}{2}+\mathrm{C} \Rightarrow \mathrm{C}=\frac{1}{2}\)

Substituting C = \(\frac{1}{2}\) in equation (2), we get:

y = \(=\frac{\mathrm{e}^{\mathrm{x}}(\sin \mathrm{x}-\cos \mathrm{x})}{2}+\frac{1}{2}\)

⇒ \(2 \mathrm{y}=\mathrm{e}^{\mathrm{x}}(\sin \mathrm{x}-\cos \mathrm{x})+1 \Rightarrow 2 \mathrm{y}-1=\mathrm{e}^{\mathrm{x}}(\sin \mathrm{x}-\cos \mathrm{x})\)

Hence, the required equation of the curve is \(2 y-1=e^x(\sin x-\cos x)\)

Question 16. For the differential equation xy\(\frac{dy}{dx}\) = (x + 2)(y + 2). find the solution curve passing through the point (1,-1).
Solution:

The differential equation of the given curve is: xy \(\frac{dy}{dx}\)= (x + 2)(y + 2)

Separating the variables, we get: \(\left(\frac{y}{y+2}\right) d y=\left(\frac{x+2}{x}\right) d x \Rightarrow\left(1-\frac{2}{y+2}\right) d y=\left(1+\frac{2}{x}\right) d x\)

Integrating both sides, we get:

⇒ \(\int\left(1-\frac{2}{y+2}\right) d y=\int\left(1+\frac{2}{x}\right) d x \Rightarrow \int d y-2 \int \frac{1}{y+2} d y=\int d x+2 \int \frac{1}{x} d x\)

⇒ \( y-2 \log (y+2)=x+2 \log x+C \Rightarrow y-x-C=\log x^2+\log (y+2)^2\)

⇒ \(y-x-C=\log \left[x^2(y+2)^2\right]\)…(1)

Now, the curve passes through point (1,-1).

⇒ \(-1-1-\mathrm{C}=\log \left[(1)^2(-1+2)^2\right] \Rightarrow-2-\mathrm{C}=\log 1=0 \Rightarrow \mathrm{C}=-2\)

Substituting C=-2 in equation (1), we get : \(y-x+2=\log \left[x^2(y+2)^2\right]\)

This is the required solution of the given curve.

Question 17. Find the equation of a curve passing through the point (0, -2) given that at any point (x, y) on the curve, the product of the slope of its tangent and y-coordinate of the point is equal to the x-coordinate of the point.
Solution:

Let x and y be the x-coordinate and y-coordinate of the curve respectively.

We know that the slope of a tangent to the curve in the coordinate axis is given by the relation, \(\frac{dy}{dx}\)

According to the given information, we get:

The product of the slope of a tangent with y-coordinate = x-coordinate

y · \(\frac{dy}{dx}\) = x

Separating the variables, we get; y dy = x dx

Integrating both sides, we get: \(\int y d y=\int x d x \Rightarrow \frac{y^2}{2}=\frac{x^2}{2}+C \Rightarrow y^2-x^2=2 C\)….(1)

Now, the curve passes through the point (0, -2).

∴ (-2)² – 0² = 2C ⇒ 2C = 4

Substituting 2C = 4 in equation (1), we get: y² – x² = 4

This is the required equation of the curve.

Question 18. At any point (x, y) of a curve, the slope of the tangent is twice the slope of the line segment joining the point of contact to the point (-4, -3). Find the equation of the curve given that it passes through (-2, 1).
Solution:

It is given that (x, y) is the point of contact of the curve and its tangent.

The slope (m1) of the line segment joining (x, y) and (-4, -3) is \(\frac{y-(-3)}{x-(-4)}=\frac{y+3}{x+4}\)

We know that the slope of the tangent to the curve is given by the relation, \(\frac{dy}{dx}\)

∴ Slope (m2) of the tangent = \(\frac{dy}{dx}\)

According to the given information : \(\mathrm{m}_2=2 \mathrm{~m}_1 \Rightarrow \frac{\mathrm{dy}}{\mathrm{dx}}=\frac{2(\mathrm{y}+3)}{\mathrm{x}+4}\)

Separating the variables, we get: \(\frac{d y}{y+3}=\frac{2 d x}{x+4}\)

Integrating both sides, we get: \(\int \frac{d y}{y+3}=2 \int \frac{d x}{x+4} \Rightarrow \log (y+3)=2 \log (x+4)+\log C\)

⇒ log(y + 3) = log C (x + 4)² ⇒ y + 3 = C(x + 4)²….(1)

This is the general equation of the curve.

It is given that it passes through the point (-2, 1).

⇒ 1 + 3 = C(-2 + 4)² ⇒ 4 = 4C ⇒ C = 1

Substituting C = 1 in equation (1), we get; y + 3 = (x + 4)²

This is the required equation of the curve.

Question 19. The volume of a spherical balloon being inflated changes at a constant rate. If initially its radius is 3 units and after 3 seconds it is 6 units. Find the radius of the balloon after t seconds.
Solution:

Let the rate of change of the volume of the balloon be k (where k is a constant).

⇒ \(\frac{\mathrm{dv}}{\mathrm{dt}}=\mathrm{k} \Rightarrow \frac{\mathrm{d}}{\mathrm{dt}}\left(\frac{4}{3} \pi \mathrm{r}^3\right)=\mathrm{k}\)

(Volume of sphere = \(\frac{4}{3} \pi \mathrm{r}^3\))

⇒ \(\frac{4}{3} \pi \cdot 3 \mathrm{r}^2 \cdot \frac{\mathrm{dr}}{\mathrm{dt}}=\mathrm{k} \Rightarrow 4 \pi \mathrm{r}^2 \mathrm{dr}=\mathrm{kdt}\)

Integrating both sides, we get: \(4 \pi \int r^2 d r=k \int d t \Rightarrow 4 \pi \cdot \frac{r^3}{3}=k t+C \Rightarrow 4 \pi r^3=3(k t+C)\)….(1)

Now, at t=0, r=3

⇒ \(4 \pi \times 3^3=3(\mathrm{k} \times 0+\mathrm{C}) \Rightarrow 108 \pi=3 \mathrm{C} \Rightarrow \mathrm{C}=36 \pi\)

At t=3, r=6

⇒ \(4 \pi \times 6^3=3(\mathrm{k} \times 3+\mathrm{C}) \Rightarrow 864 \pi=3(3 \mathrm{k}+36 \pi) \Rightarrow 3 \mathrm{k}=288 \pi-36 \pi=252 \pi\)

⇒ k=84 \(\pi\)

Substituting the values of k and C in equation (1), we get:

⇒ \(4 \pi r^3=3[84 \pi t+36 \pi] \Rightarrow 4 \pi r^3=4 \pi[63 t+27] \Rightarrow r^3=63 t+27 \Rightarrow r=(63 t+27)^{\frac{1}{3}}\)

Thus, the radius of the balloon after t seconds is \((63 t+27)^{\frac{1}{3}}\).

Question 20. In a bank, the principal increases continuously at the rate of r% per year. Find the value of r if Rs 100 doubles itself in 10 years (loge2 = 0.6931).
Solution:

Let p, t, and r represent the principal, time, and rate of interest respectively.

It is given that the principal increases continuously at the rate of r% per year.

⇒ \(\frac{d p}{d t}=\left(\frac{r}{100}\right) p \Rightarrow \frac{d p}{p}=\left(\frac{r}{100}\right) d t\)

Integrating both sides, we get:

⇒ \(\int \frac{d p}{p}=\frac{r}{100} \int d t \Rightarrow \log p=\frac{r t}{100}+k \Rightarrow p=e^{\frac{rt}{100}+k}\)….(1)

It is given that when \(\mathrm{t}=0, \mathrm{p}=100. \Rightarrow 100=\mathrm{e}^{\mathrm{k}}\)…..(2)

Now, if t=10, then p=2 x 100=200.

Therefore, equation (1) becomes :

200 = \(e^{\frac{\mathrm{t}}{10}+\mathrm{k}} \Rightarrow 200=\mathrm{e}^{\frac{\mathrm{r}}{10}} \cdot \mathrm{e}^{\mathrm{k}} \Rightarrow 200=\mathrm{e}^{\frac{\mathrm{t}}{11}} \cdot 100\)(From (2)

⇒ \(\mathrm{e}^{\frac{r}{10}}=2 \Rightarrow \frac{r}{10}=\log _4 2 \Rightarrow \frac{r}{10}=0.6931 \Rightarrow r=6.931\)

Hence, the value of r is 6.93%.

Question 21. In a bank, the principal increases continuously at the rate of 5% per year. An amount of Rs 1000 is deposited with this bank, how much will it be worth after 10 years (e0.5 = 1.648)?
Solution:

Let p and t be the principal and time respectively.

It is given that the principal increases continuously at the rate of 5% per year.

⇒ \(\frac{d p}{d t}=\left(\frac{5}{100}\right) p \Rightarrow \frac{d p}{d t}=\frac{p}{20}\)

Separating the variables, we get: \(\frac{d p}{d t}=\frac{dt}{20}\)

Integrating both sides, we get: \(\int \frac{d p}{p}=\frac{1}{20} \int d t \Rightarrow \log p=\frac{t}{20}+C \Rightarrow p=e^{\frac{t}{20}+C}\)…….(1)

Now, when t = 0, p = 1000.

⇒ 1000 = ec

At t = 10, equation (1) becomes p=e1/2+ C ⇒ p = e0.5 x eC ⇒ p = 1.648 x 1000 (from (2))

⇒ p = 1648

Hence, after 10 years the amount will be worth Rs 1648.

Question 22. In a culture, the bacteria count is 1,00,000. The number is increased by 10% in 2 hours. In how many hours will the count reach 2,00.000, if the rate of growth of bacteria is proportional to the number present?
Solution:

Let y be the number of bacteria at any instant t.

It is given that the rate of growth of the bacteria is proportional to the number present.

∴ \(\frac{d y}{d t} \propto y \Rightarrow \frac{d y}{d t}=k y\) (where k is constant)

Separating the variables, we get: \(\frac{d y}{d t}\) = k dt

Integrating both sides, we get: \(\frac{d y}{d t}\) = k dt ⇒ log y = kt + C……(1)

Let y0 be the number of bacteria at t = 0. ⇒ log y0 = C

Substituting the value of C in equation (1), we get:

log y = \(k t+\log y_0 \Rightarrow \log y-\log y_0=k t\)

⇒ \(\log \left(\frac{y}{y_0}\right)=k t\)…..(2)

Also, it is given that the number of bacteria increases by 10 % in 2 hours.

⇒ \(y=\frac{110}{100} y_0 \Rightarrow \frac{y}{y_0}=\frac{11}{10}\)….(3)

Substituting this value from equation (3) in the equation

⇒ \(\mathrm{k} \cdot 2=\log \left(\frac{11}{10}\right) \Rightarrow \mathrm{k}=\frac{1}{2} \log \left(\frac{11}{10}\right)\)

Therefore, equation (2) becomes:

⇒ \(\frac{1}{2} \log \left(\frac{11}{10}\right) \cdot t=\log \left(\frac{y}{y_0}\right) \Rightarrow t=\frac{2 \log \left(\frac{y}{y_0}\right)}{\log \left(\frac{11}{10}\right)}\)…..(4)

Now, let the time when the number of bacteria increases from 100000 to 200000 be t.

⇒ \(\mathrm{y}=2 \mathrm{y}_0\) at \(\mathrm{t}=\mathrm{t}_1\)

From equation (4), we get \(t_1=\frac{2 \log \left(\frac{y}{y_0}\right)}{\log \left(\frac{11}{10}\right)}=\frac{2 \log 2}{\log \left(\frac{11}{10}\right)}\).

Hence, in \(\frac{2 \log 2}{\log \left(\frac{11}{10}\right)}\) hours the number of bacteria increases from 100000 to 200000.

Question 23. The general solution of the differential equation \(\frac{dy}{dt}\) = ex+y is

  1. \(e^x+e^{-y}=C\)
  2. \(e^x+e^y=C\)
  3. \(\mathrm{e}^{-x}+\mathrm{e}^y=\mathrm{C}\)
  4. \(\mathrm{e}^{-x}+\mathrm{e}^{-7}=\mathrm{C}\)

Solution: 1. \(e^x+e^{-y}=C\)

Given, \(\frac{d y}{d x}=e^{x+y}=e^x \cdot e^y\)

Separating the variables, we get : \(\frac{d y}{e^y}=e^x d x \Rightarrow e^{-y} d y=e^x d x\)

Integrating both sides, we get:

⇒ \(\int \mathrm{e}^{-y} \mathrm{dy}=\int \mathrm{e}^{\mathrm{x}} \mathrm{dx} \Rightarrow-\mathrm{e}^{-\mathrm{y}}=\mathrm{e}^{\mathrm{x}}+\mathrm{k} \Rightarrow \mathrm{e}^{\mathrm{x}}+\mathrm{e}^{-\mathrm{y}}=-\mathrm{k} \Rightarrow \mathrm{e}^x+\mathrm{e}^{-\mathrm{y}}=\mathrm{C} \quad(\mathrm{C}=-\mathrm{k})\)

Hence, the correct answer is 1.

Differential Equations Exercise 9.4

Show That The Given Differential Equation Is Homogeneous And Solve Each Of Them.

Question 1. (x² + xy)dy = (x² + y²)dx
Solution:

The given differential equation i.e., (x² + xy)dy = (x² + y²)dx can be written as:

⇒ \(\frac{d y}{d x}=\frac{x^2+y^2}{x^2+x y}\)….(1)

Let F(x, y) = \(\frac{x^2+y^2}{x^2+x y}\)

Now, \(F(\lambda x, \lambda y)=\frac{(\lambda x)^2+\left(\lambda y^2\right)}{(\lambda x)^2+(\lambda x)(\lambda y)}=\frac{\lambda^2\left(x^2+y^2\right)}{\lambda^2\left(x^2+x y\right)}=\lambda^0 \cdot F(x, y)\)

This shows that equation (1) is a homogeneous equation.

To solve it, we make the substitution as y = vx

Differentiating both sides with respect to x. we get: \(\frac{dy}{dx}\) = v + x \(\frac{dv}{dx}\)

Substituting the values of y and \(\frac{dv}{dx}\) in equation (1), we get:

v+x \(\frac{d v}{d x}=\frac{x^2+(v x)^2}{x^2+x(v x)} \Rightarrow v+x \frac{d v}{d x}=\frac{1+v^2}{1+v}\)

⇒ \(x \frac{d v}{d x}=\frac{1+v^2}{1+v}-v=\frac{\left(1+v^2\right)-v(1+v)}{1+v} \Rightarrow x \frac{d v}{d x}=\frac{1-v}{1+v}\)

⇒ \(\left(\frac{1+v}{1-v}\right) d v=\frac{d x}{x} \Rightarrow\left(\frac{2-1+v}{1-v}\right) d v=\frac{d x}{x} \Rightarrow\left(\frac{2}{1-v}-1\right) d v=\frac{d x}{x}\)

Integrating both sides, we get:-2 log (1-v)-v = log x-log k

⇒ v = -2 log (1-v)-log x+log k

⇒ v = \(\log \left[\frac{k}{x(1-v)^2}\right] \Rightarrow \frac{y}{x}=\log \left[\frac{k}{x\left(1-\frac{y}{x}\right)^2}\right] \Rightarrow \frac{y}{x}=\log \left[\frac{k x}{(x-y)^2}\right]\)

⇒ \(\frac{k x}{(x-y)^2}=e^{\frac{y}{x}} \Rightarrow(x-y)^2=k x e^{-\frac{x}{x}}\)

This is the required solution of the given differential equation.

Question 2. \(y^{\prime}=\frac{x+y}{x}\)
Solution:

The given differential equation is: \(y^{\prime}=\frac{x+y}{x} \Rightarrow \frac{d y}{d x}=\frac{x+y}{x}\)….(1)

Let F(x, y)= \(\frac{x+y}{x}\)

Now, \(\mathrm{F}(\lambda \mathrm{x}, \lambda \mathrm{y})=\frac{\lambda \mathrm{x}+\lambda \mathrm{y}}{\lambda \mathrm{x}}=\frac{\lambda(\mathrm{x}+\mathrm{y})}{\lambda \mathrm{x}}=\lambda^0 \mathrm{~F}(\mathrm{x}, \mathrm{y})\)

Thus, the given equation is a homogeneous equation.

To solve it, we make the substitution as y = vx

Substituting the values of y and \(\frac{dy}{dx}\) in equation (1), we get:

v+x \(\frac{d v}{d x}=\frac{x+v x}{x} \Rightarrow v+x \frac{d v}{d x}=1+v \Rightarrow x \frac{d v}{d x}=1 \Rightarrow \int d v=\int \frac{d x}{x}\)

Integrating both sides, we get:

v = \(\log x+C \Rightarrow \frac{y}{x}=\log x+C \Rightarrow y=x \log x+C x\)

This is the required solution of the given differential equation.

Question 3. (x – y) dy – (x + y)dx = 0
Solution:

The given differential equation is : (x – y) dy – (x + y)dx = 0

⇒ \(\frac{d y}{d x}=\frac{x+y}{x-y}\)…..(1)

Let F(x, y)= \(\frac{x+y}{x-y}\)

∴ \(\mathrm{F}(\lambda \mathrm{x}, \lambda \mathrm{y})=\frac{\lambda \mathrm{x}+\lambda \mathrm{y}}{\lambda \mathrm{x}-\lambda \mathrm{y}}=\frac{\lambda(\mathrm{x}+\mathrm{y})}{\lambda(\mathrm{x}-\mathrm{y})}=\lambda^0 \cdot \mathrm{F}(\mathrm{x}, \mathrm{y})\)

Thus, the given differential equation is a homogeneous equation.

To solve it, we make the substitution as : \(\mathrm{y}=\mathrm{vx}\)

⇒ \(\frac{d}{d x}(y)=\frac{d}{d x}(v x) \Rightarrow \frac{d y}{d x}=v+x \frac{d y}{d x}\)

Substituting the values of y and \(\frac{d y}{d x}\) in equation (1), we get:

v+x \(\frac{d v}{d x}=\frac{x+v x}{x-v x}=\frac{1+v}{1-v} \Rightarrow x \frac{d v}{d x}=\frac{1+v}{1-v}-v=\frac{1+v-v(1-v)}{1-v}\)

x \(\frac{d v}{d x}=\frac{1+v^2}{1-v} \Rightarrow \int \frac{1-v}{\left(1+v^2\right)} d v=\int \frac{d x}{x} \Rightarrow \int\left(\frac{1}{1+v^2}-\frac{v}{1+v^2}\right) d v\)

= \(\int \frac{d x}{x}\)

⇒ tan \(^{-1} v-\frac{1}{2} \log \left(1+v^2\right)=\log x+C \Rightarrow \tan ^{-1}\left(\frac{y}{x}\right)-\frac{1}{2} \log \left[1+\left(\frac{y}{x}\right)^2\right]=\log x+C\)

⇒ \(\tan ^{-1}\left(\frac{y}{x}\right)-\frac{1}{2} \log \left(\frac{x^2+y^2}{x^2}\right)=\log x+C\)

⇒ \(\tan ^{-1}\left(\frac{y}{x}\right)-\frac{1}{2}\left[\log \left(x^2+y^2\right)-\log x^2\right]=\log x+C\)

⇒ \(\tan ^{-1}\left(\frac{y}{x}\right)=\frac{1}{2} \log \left(x^2+y^2\right)+C\)

This is the required solution of the given differential equation.

Question 4. (x² – y²)dx + 2xy dy = 0
Solution:

The given differential equation is

⇒ \(\left(x^2-y^2\right) d x+2 x y d y=0 \Rightarrow \frac{d y}{d x}=\frac{-\left(x^2-y^2\right)}{2 x y}\)…..(1)

Let F(x, y) = \(\frac{-\left(x^2-y^2\right)}{2 x y}\)

∴ \(F(\lambda x, \lambda y)=-\left[\frac{(\lambda x)^2-(\lambda y)^2}{2(\lambda x)(\lambda y)}\right]=\frac{-\lambda^2\left(x^2-y^2\right)}{\lambda^2(2 x y)}=\lambda^0 \cdot F(x, y)\)

Therefore, the given differential equation is a homogeneous equation.

To solve it, we make the substitution as:

y = \(v x \Rightarrow \frac{d}{d x}(y)=\frac{d}{d x}(v x) \Rightarrow \frac{d y}{d x}=v+x \frac{d v}{d x}\)

Substituting the values of y and \(\frac{d y}{d x}\) in equation (1), we get:

v+x \(\frac{d v}{d x}=-\left[\frac{x^2-(v x)^2}{2 x \cdot(v x)}\right] \Rightarrow v+x \frac{d v}{d x}=\frac{v^2-1}{2 v}\)

⇒ \(x \frac{d v}{d x}=\frac{v^2-1}{2 v}-v=\frac{v^2-1-2 v^2}{2 v} \Rightarrow x \frac{d v}{d x}=-\frac{\left(1+v^2\right)}{2 v}\)

⇒ \(\frac{2 v}{1+v^2} d v=-\frac{d x}{x}\)

⇒ \(\int \frac{2 v}{1+v^2} d v=-\int \frac{d x}{x}\)

log \(\left(1+v^2\right)=-\log x+\log C=\log \frac{C}{x} \Rightarrow 1+v^2=\frac{C}{x} \Rightarrow\left[1+\frac{y^2}{x^2}\right]\)

= \(\frac{C}{x} \Rightarrow x^2+y^2=C x\)

This is the required solution of the given differential equation.

Question 5. \(x^2 \frac{d y}{d x}=x^2-2 y^2+x y\)
Solution:

The given differential equation is : \(x^2 \frac{d y}{d x}=x^2-2 y^2+x y\)

⇒ \(\frac{d y}{d x}=\frac{x^2-2 y^2+x y}{x^2}\)….(1)

Let F(x, y) = \(\frac{x^2-2 y^2+x y}{x^2}\)

∴ \(F(\lambda x, \lambda y)=\frac{(\lambda x)^2-2(\lambda y)^2+(\lambda x)(\lambda y)}{(\lambda x)^2}=\frac{\lambda^2\left(x^2-2 y^2+x y\right)}{\lambda^2\left(x^2\right)}=\lambda^0 \cdot F(x, y)\)

Therefore, the given differential equation is a homogeneous equation.

To solve it, we make the substitution as:

y = \(v x \Rightarrow \frac{d y}{d x}=v+x \frac{d v}{d x}\)

Substituting the values of y and \(\frac{d y}{d x}\) in equation (1), we get:

v+x \(\frac{d v}{d x}=\frac{x^2-2(v x)^2+x \cdot(v x)}{x^2}\)

⇒ \(v+x \frac{d v}{d x}=1-2 v^2+v \Rightarrow x \frac{d v}{d x}=1-2 v^2\)

⇒ \(\frac{d v}{1-2 v^2}=\frac{d x}{x} \Rightarrow \frac{1}{2} \int \frac{d v}{\frac{1}{2}-v^2}=\int \frac{d x}{x}\)

⇒ \(\frac{1}{2} \cdot \int\left[\frac{\mathrm{dv}}{\left(\frac{1}{\sqrt{2}}\right)^2-v^2}\right]=\int \frac{\mathrm{dx}}{\mathrm{x}}\)

⇒ \(\frac{1}{2} \cdot \frac{1}{2 \times \frac{1}{\sqrt{2}}} \log \left|\frac{\frac{1}{\sqrt{2}}+\mathrm{v}}{\frac{1}{\sqrt{2}}-\mathrm{v}}\right|=\log |\mathrm{x}|+\mathrm{C}\)

(because \(\int \frac{d x}{a^2-x^2}=\frac{1}{2 a} \log \left|\frac{a+x}{a-x}\right|+C\))

⇒ \(\frac{1}{2 \sqrt{2}} \log \left|\frac{\frac{1}{\sqrt{2}}+\frac{y}{x}}{\frac{1}{\sqrt{2}}-\frac{y}{x}}\right|=\log |x|+C \Rightarrow \frac{1}{2 \sqrt{2}} \log \left|\frac{x+\sqrt{2} y}{x-\sqrt{2} y}\right|=\log |x|+C \)

This is the required solution for the given differential equation.

Question 6. \(x d y-y d x=\sqrt{x^2+y^2} d x\)
Solution:

⇒ \(x d y-y d x=\sqrt{x^2+y^2} d x \Rightarrow x d y=\left[y+\sqrt{x^2+y^2}\right] d x\)

⇒ \(\frac{d y}{d x}=\frac{y+\sqrt{x^2+y^2}}{x}\)….(1)

Let F(x, y) = \(\frac{y+\sqrt{x^2+y^2}}{x}\)

∴ \(F(\lambda x, \lambda y)=\frac{\lambda x+\sqrt{(\lambda x)^2+(\lambda y)^2}}{\lambda x}=\frac{\lambda\left(y+\sqrt{x^2+y^2}\right)}{\lambda x}=\lambda^0 \cdot F(x, y)\)

Therefore, the given differential equation is a homogeneous equation.

To solve it, we make the substitution as y = vx

⇒ \(\frac{d}{d x}(y)=\frac{d}{d x}(v x) \Rightarrow \frac{d y}{d x}=v+x \frac{d v}{d x}\)

Substituting the values of y and \(\frac{d y}{d x}\) in equation (1), we get:

v+x \(\frac{d v}{d x}=\frac{v x+\sqrt{x^2+(v x)^2}}{x} \Rightarrow v+x \frac{d v}{d x}=v+\sqrt{1+v^2}\)

⇒ \(\frac{d v}{\sqrt{1+v^2}}=\frac{d x}{x} \Rightarrow \int \frac{d v}{\sqrt{1+v^2}}=\int \frac{d x}{x}\)

⇒ \(\log \left|v+\sqrt{1+v^2}\right|=\log |x|+\log C \Rightarrow \log \left|\frac{y}{x}+\sqrt{1+\frac{y^2}{x^2}}\right|=\log |C x|\)

⇒ \(\log \left|\frac{y+\sqrt{x^2+y^2}}{x}\right|=\log |C x| \Rightarrow y+\sqrt{x^2+y^2}=C x^2\)

This is the required solution of the given differential equation.

Question 7. \(\left\{x \cos \left(\frac{y}{x}\right)+y \sin \left(\frac{y}{x}\right)\right\} y d x=\left\{y \sin \left(\frac{y}{x}\right)-x \cos \left(\frac{y}{x}\right)\right\} x d y\)
Solution:

The given differential equation is:

⇒ \(\left\{x \cos \left(\frac{y}{x}\right)+y \sin \left(\frac{y}{x}\right)\right\} y d x=\left\{y \sin \left(\frac{y}{x}\right)-x \cos \left(\frac{y}{x}\right)\right\} x d y\)

⇒ \(\frac{d y}{d x}=\frac{\left\{x \cos \left(\frac{y}{x}\right)+y \sin \left(\frac{y}{x}\right)\right\} y}{\left\{y \sin \left(\frac{y}{x}\right)-x \cos \left(\frac{y}{x}\right)\right\} x}\)….(1)

Let F(x, y) = \(\frac{\left\{x \cos \left(\frac{y}{x}\right)+y \sin \left(\frac{y}{x}\right)\right\} y}{\left\{y \sin \left(\frac{y}{x}\right)-x \cos \left(\frac{y}{x}\right)\right\} x}\)

⇒ \(\mathrm{F}(\lambda \mathrm{x}, \lambda \mathrm{y})=\frac{\left\{\lambda x \cos \left(\frac{\lambda y}{\lambda x}\right)+\lambda y \sin \left(\frac{\lambda y}{\lambda x}\right)\right\} \lambda y}{\left\{\lambda y \sin \left(\frac{\lambda y}{\lambda x}\right)-\lambda x \cos \left(\frac{\lambda y}{\lambda x}\right)\right\} \lambda x}\)

= \(\frac{\lambda^2\left\{x \cos \left(\frac{y}{x}\right)+y \sin \left(\frac{y}{x}\right)\right\} y}{\lambda^2\left\{y \sin \left(\frac{y}{x}\right)-x \cos \left(\frac{y}{x}\right)\right\} x}=\lambda^0 \cdot F(x, y)\)

Therefore, the given differential equation is a homogeneous equation.

To solve it, we make the substitution as:

y = \(v x \Rightarrow \frac{d y}{d x}=v+x \times \frac{d v}{d x}\)

Substituting the values of y and \(\frac{dy}{dx}\) in equation {1), we get:

v+x \(\frac{d v}{d x}=\frac{(x \cos v+v x \sin v) \cdot v x}{(v x \sin v-x \cos v) \cdot x} \Rightarrow v+x \frac{d v}{d x}=\frac{v \cos v+v^2 \sin v}{v \sin v-\cos v}\)

⇒ \(x \frac{d v}{d x}=\frac{v \cos v+v^2 \sin v}{v \sin v-\cos v}-v\)

⇒ \(x \frac{d v}{d x}=\frac{v \cos v+v^2 \sin v-v^2 \sin v+v \cos v}{v \sin v-\cos v}\)

⇒ \(x \frac{d v}{d x}=\frac{2 v \cos v}{v \sin v-\cos v} \Rightarrow\left[\frac{v \sin v-\cos v}{v \cos v}\right] d v=\frac{2 d x}{x}\)

⇒ \(\left(\tan v-\frac{1}{v}\right) d v=\frac{2 d x}{x}\)

Integrating both sides, we get : \(\int\left(\tan v-\frac{1}{v}\right) d v=2 \int \frac{d x}{x}\)

log (sec v)-log v=2 log x+log C

⇒ \(\log \left(\frac{\sec v}{v}\right)=\log \left(C x^2\right) \Rightarrow\left(\frac{\sec v}{v}\right)=C^2 \Rightarrow \sec v=C^2 v\)

⇒ \(\sec \left(\frac{y}{x}\right)=C \cdot x^2 \cdot \frac{y}{x} \Rightarrow \sec \left(\frac{y}{x}\right)=C x y \Rightarrow \cos \left(\frac{y}{x}\right)=\frac{1}{C x y}=\frac{1}{C} \cdot \frac{1}{x y}\)

⇒ \(x y \cos \left(\frac{y}{x}\right)=k \quad\left(k=\frac{1}{C}\right)\)

This is the required solution of the given differential equation.

Question 8. \(x \frac{d y}{d x}-y+x \sin \left(\frac{y}{x}\right)=0\)
Solution:

x \(\frac{d y}{d x}-y+x \sin \left(\frac{y}{x}\right)=0 \Rightarrow x \frac{d y}{d x}=y-x \sin \left(\frac{y}{x}\right)\)

⇒ \(\frac{d y}{d x}=\frac{y-x \sin \left(\frac{y}{x}\right)}{x}\)…..(1)

Let F(x, y) = \(\frac{y-x \sin \left(\frac{y}{x}\right)}{x}\)

⇒ \(\mathrm{F}(\lambda \mathrm{x}, \lambda \mathrm{y})=\frac{\lambda \mathrm{y}-\lambda \mathrm{x} \sin \left(\frac{\lambda \mathrm{y}}{\lambda \mathrm{x}}\right)}{\lambda \mathrm{x}}=\frac{\lambda\left[\mathrm{y}-\mathrm{x} \sin \left(\frac{\mathrm{y}}{\mathrm{x}}\right)\right]}{\lambda \mathrm{x}}=\lambda^0 \cdot \mathrm{F}(\mathrm{x}, \mathrm{y})\)

Therefore, the given differential equation is a homogeneous equation.

To solve it, we make the substitution as:

y = \(v x \Rightarrow \frac{d}{d x}(y)=\frac{d}{d x}(v x) \Rightarrow \frac{d y}{d x}=v+x \frac{d v}{d x}\)

Substituting the values of x and \(\frac{dy}{d x}\) in equation (1), we get:

v+x \(\frac{d v}{d x}=\frac{v x-x \sin v}{x} \Rightarrow v+x \frac{d v}{d x}=v-\sin v\)

⇒ \(-\frac{d v}{\sin v}=\frac{d x}{x} \Rightarrow \mathrm{cosec} v d v=-\frac{d x}{x} \Rightarrow \int \mathrm{cosec} v d v=-\int \frac{d x}{x}\)

log \(|\mathrm{cosec} v-\cot v|=-\log x+\log C=\log \frac{C}{x}\)

⇒ cosec \(\left(\frac{y}{x}\right)-\cot \left(\frac{y}{x}\right)=\frac{C}{x}\)

⇒ \(\frac{1}{\sin \left(\frac{y}{x}\right)}-\frac{\cos \left(\frac{y}{x}\right)}{\sin \left(\frac{y}{x}\right)}=\frac{C}{x}\)

⇒ x \(\left[1-\cos \left(\frac{y}{x}\right)\right]=C \sin \left(\frac{y}{x}\right)\)

This is the required solution of the given differential equation.

Question 9. \(y d x+x \log \left(\frac{y}{x}\right) d y-2 x d y=0\)
Solution:

⇒ \(y d x+x \log \left(\frac{y}{x}\right) d y-2 x d y=0 \Rightarrow y d x=\left[2 x-x \log \left(\frac{y}{x}\right)\right] d y\)

⇒ \(\frac{\mathrm{dy}}{\mathrm{dx}}=\frac{\mathrm{y}}{2 \mathrm{x}-\mathrm{x} \log \left(\frac{\mathrm{y}}{\mathrm{x}}\right)}\)….(1)

Let F(x, y) = \(\frac{y}{2 x-x \log \left(\frac{y}{x}\right)}\)

∴ \(F(\lambda x, \lambda y)=\frac{\lambda y}{2(\lambda x)-(\lambda x) \log \left(\frac{\lambda y}{\lambda x}\right)}\)

= \(\frac{\lambda y}{\lambda\left[2 x-x \log \left(\frac{y}{x}\right)\right]}=\lambda^0 F \mathrm{~F}(x, y)\)

Therefore, the given differential equation is a homogeneous equation.

To solve it, we make the substitution as: y = \(v x \Rightarrow \frac{d y}{d x}=\frac{d}{d x}(v x) \Rightarrow \frac{d y}{d x}=v+x \frac{d v}{d x}\)

Substituting the values of y and \(\frac{d y}{d x}\) in equation (1), we get:

v+x \(\frac{d v}{d x}=\frac{v x}{2 x-x \log v} \Rightarrow v+x \frac{d v}{d x}=\frac{v}{2-\log v} \Rightarrow x \frac{d v}{d x}=\frac{v}{2-\log v}-v\)

⇒ \(x \frac{d v}{d x}=\frac{v-2 v+v \log v}{2-\log v} \Rightarrow x \frac{d v}{d x}=\frac{v \log v-v}{2-\log v} \Rightarrow \frac{2-\log v}{v(\log v-1)} d v=\frac{d x}{x}\)

⇒ \({\left[\frac{1+(1-\log v)}{v(\log v-1)}\right] d v=\frac{d x}{x} \Rightarrow\left[\frac{1}{v(\log v-1)}-\frac{1}{v}\right] d v=\frac{d x}{x} }\)

Integrating both sides, we get: \(\int \frac{1}{v(\log v-1)} d v-\int \frac{1}{v} d v=\int \frac{1}{x} d x\)

⇒ \(\int \frac{d v}{v(\log v-1)}-\log v=\log x+\log C\)…..(2)

Let log v-1=t

⇒ \(\frac{\mathrm{d}}{\mathrm{dv}}(\log \mathrm{v}-1)=\frac{\mathrm{dt}}{\mathrm{dv}} \Rightarrow \frac{1}{\mathrm{v}}=\frac{\mathrm{dt}}{\mathrm{dv}} \Rightarrow \frac{\mathrm{dv}}{\mathrm{v}}=\mathrm{dt}\)

Therefore, equation (2) becomes:

⇒ \(\int \frac{d t}{t}-\log v=\log x+\log C\)

⇒ \(\log t-\log v=\log (C x)=\log (\log v-1)-\log v=\log (C x)\)

⇒ \(\log \left[\log \left(\frac{y}{x}\right)-1\right]-\log \left(\frac{y}{x}\right)=\log (C x)\)

⇒ \(\log \left[\frac{\log \left(\frac{y}{x}\right)-1}{\frac{y}{x}}\right]=\log (C x) \Rightarrow \frac{x}{y}\left[\log \left(\frac{y}{x}\right)-1\right]=C x \Rightarrow \log \left(\frac{y}{x}\right)-1=C y\)

This is the required solution of the given differential equation.

Question 10. \(\left(1+e^{\frac{x}{y}}\right) d x+e^{\frac{x}{y}}\left(1-\frac{x}{y}\right) d y=0\)
Solution:

⇒ \(\left(1+e^{\frac{x}{y}}\right) d x+e^{\frac{x}{y}}\left(1-\frac{x}{y}\right) d y=0 \Rightarrow\left(1+e^{\frac{x}{y}}\right) d x=-e^{\frac{x}{y}}\left(1-\frac{x}{y}\right) d y\)

⇒ \(\frac{\mathrm{dx}}{\mathrm{dy}}=\frac{-\mathrm{e}^{\frac{x}{y}}\left(1-\frac{\mathrm{x}}{\mathrm{y}}\right)}{1+\mathrm{e}^{\frac{\mathrm{x}}{y}}}\)….(1)

Let F(x, y) = \(\frac{-e^{\frac{x}{y}}\left(1-\frac{x}{y}\right)}{1+e^{\frac{x}{y}}}\)

∴ \(F(\lambda x, \lambda y)=\frac{-e^{\frac{\lambda x}{\lambda y}}\left(1-\frac{\lambda x}{\lambda y}\right)}{1+e^{\frac{\lambda x}{\lambda y}}}\)

= \(\frac{-e^{\frac{x}{y}}\left(1-\frac{x}{y}\right)}{1+e^{\frac{x}{y}}}=\lambda^0 \cdot F(x, y)\)

Therefore, the given differential equation is a homogeneous equation.

To solve it, we make the substitution as: \(\mathrm{x}=\mathrm{vy} \Rightarrow \frac{\mathrm{d}}{\mathrm{dy}}(\mathrm{x})\)

= \(\frac{\mathrm{d}}{\mathrm{dy}}(\mathrm{vy}) \Rightarrow \frac{\mathrm{dx}}{\mathrm{dy}}=\mathrm{v}+\mathrm{y} \frac{\mathrm{dv}}{\mathrm{dy}}\)

Substituting the values of x and \(\frac{\mathrm{dx}}{\mathrm{dy}}\) in equation (1), we get:

v+y \(\frac{d v}{d y}=\frac{-e^v(1-v)}{1+e^v} \Rightarrow y \frac{d v}{d y}=\frac{-e^v+v^v}{1+e^v}-v\)

⇒ \(y \frac{d v}{d y}=\frac{-e^v+v^v-v-v e^v}{1+e^v} \Rightarrow y \frac{d v}{d y}=-\left[\frac{v+e^v}{1+e^v}\right]\)

⇒ \(\left[\frac{1+e^v}{v+e^v}\right] d v=-\frac{d y}{y}\)

Integrating both sides, we get:

⇒ \(\log \left(v+e^v\right)=-\log y+\log C=\log \left(\frac{C}{y}\right) \Rightarrow \log \left(v+e^v\right)=\log \left(\frac{C}{y}\right)\)

⇒ \(\frac{x}{y}+e^{\frac{x}{y}}=\frac{C}{y} \Rightarrow x+y e^y=C\)

This is the required solution of the given differential equation.

For Each Of The Differential Equations, Find The Particular Solution Satisfying The Given Condition:

Question 11. (x+y) d y+(x-y) d x=0 ; y=1 when x=1
Solution:

(x+y) d y+(x-y) d x=0 \Rightarrow(x+y) d y=-(x-y) d x

⇒ \(\frac{d y}{d x}=\frac{-(x-y)}{x+y}\)………(1)

The given differential equation is a homogeneous equation.

To solve it, we make the substitution as:

y = \(v x \Rightarrow \frac{d}{d x}(y)=\frac{d}{d x}(v x) \Rightarrow \frac{d y}{d x}=v+x \frac{d v}{d x}\)

Substituting the values of y and \(\frac{d y}{d x}\) in equation (1), we get:

⇒ \(v+x \frac{d v}{d x}=\frac{-(x-v x)}{x+v x} \Rightarrow v+x \frac{d v}{d x}=\frac{v-1}{v+1}\)

⇒ \(x \frac{d v}{d x}=\frac{v-1}{v+1}-v=\frac{v-1-v(v+1)}{v+1} \Rightarrow x \frac{d v}{d x}=\frac{v-1-v^2-v}{v+1}=\frac{-\left(1+v^2\right)}{v+1}\)

⇒ \(\frac{(v+1)}{1+v^2} d v=-\frac{d x}{x} \Rightarrow \int \frac{v+1}{1+v^2} d v=-\int \frac{d x}{x}\)

⇒ \(\int\left[\frac{v}{1+v^2}+\frac{1}{1+v^2}\right] d v=-\int \frac{d x}{x}\)

⇒ \(\frac{1}{2} \log \left(1+v^2\right)+\tan ^{-1} v=-\log x+k \Rightarrow \log \left(1+v^2\right)+2 \tan ^{-1} v=-2 \log x+2 k\)

⇒ \(\log \left(x^2+y^2\right)+2 \tan ^{-1} \frac{y}{x}=2 k\)…..(2)

Now, y=1 at x=1

⇒ \(\log 2+2 \tan ^{-1} 1=2 k \Rightarrow \log 2+2 \times \frac{\pi}{4}=2 k \Rightarrow \frac{\pi}{2}+\log 2=2 k\)

Substituting the value of 2k in equation (2), we get:

⇒ \(\log \left(x^2+y^2\right)+2 \tan ^{-1}\left(\frac{y}{x}\right)=\frac{\pi}{2}+\log 2\)

This is the required solution of the given differential equation.

Question 12. \(x^2 d y+\left(x y+y^2\right) d x=0\) ; y=1 when x=1
Solution:

⇒ \(x^2 d y+\left(x y+y^2\right) d x=0 \Rightarrow x^2 d y=-\left(x y+y^2\right) d x \)

⇒ \(\frac{d y}{d x}=\frac{-\left(x y+y^2\right)}{x^2}\)….(1)

The given differential equation is a homogeneous equation.

To solve it, we make the substitution as:

y = vx \(\frac{d}{d x}(y)=\frac{d}{d x}(v x) \Rightarrow \frac{d y}{d x}=v+x \frac{d v}{d x}\)

Substituting the values of y and \(\frac{d y}{d x}\) in equation (1), we get:

v+x \(\frac{d v}{d x}=\frac{-\left[x \cdot v x+(v x)^2\right]}{x^2}=-v-v^2\)

⇒ \(x \frac{d v}{d x}=-v^2-2 v=-v(v+2) \Rightarrow \frac{d v}{v(v+2)}=-\frac{d x}{x}\)

⇒ \(\int \frac{d v}{v(v+2)}=-\int \frac{d x}{x} \Rightarrow \int \frac{1}{2}\left[\frac{(v+2)-v}{v(v+2)}\right] d v\)

= \(-\int \frac{d x}{x} \Rightarrow \int \frac{1}{2}\left[\frac{1}{v}-\frac{1}{v+2}\right] d v=-\int \frac{d x}{x}\)

⇒ \(\frac{1}{2}[\log v-\log (v+2)]=-\log x+\log C \Rightarrow \frac{1}{2} \log \left(\frac{v}{v+2}\right)=\log \frac{C}{x}\)

⇒ \(\frac{v}{v+2}=\left(\frac{C}{x}\right)^2 \Rightarrow \frac{\frac{y}{x}}{\frac{y}{x}+2}=\left(\frac{C}{x}\right)^2\)

⇒ \(\frac{y}{y+2 x}=\frac{C^2}{x^2} \Rightarrow \frac{x^2 y}{y+2 x}=C^2\)….(2)

Put, y=1 and x=1 in eq.(2), we get \(\frac{1}{1+2}=C^2 \Rightarrow C^2=\frac{1}{3}\)

Substituting \(C^2=\frac{1}{3}\) in equation (2), we get: \(\frac{x^2 y}{y+2 x}=\frac{1}{3} \Rightarrow y+2 x=3 x^2 y\)

This is the required solution of the given differential equation

Question 13. \(\left[x \sin ^2\left(\frac{y}{x}\right)-y\right] d x+x d y=0 ; y=\frac{\pi}{4}\) when x=1
Solution:

⇒ \(\left[x \sin ^2\left(\frac{y}{x}\right)-y\right] d x+x d y=0 \Rightarrow \frac{d y}{d x}\)

= \(\frac{-\left[x \sin ^2\left(\frac{y}{x}\right)-y\right]}{x}\)…….(1)

The given differential equation is a homogeneous equation.

To solve this differential equation, we make the substitution as: \(\mathrm{y}=\mathrm{vx} \Rightarrow \frac{\mathrm{d}}{\mathrm{dx}}(\mathrm{y})=\frac{\mathrm{d}}{\mathrm{dx}}(\mathrm{vx}) \Rightarrow \frac{\mathrm{dy}}{\mathrm{dx}}=\mathrm{v}+\mathrm{x} \frac{\mathrm{dv}}{\mathrm{dx}}\)

Substituting the values of y and \(\frac{d y}{d x}\) in equation (1), we get:

v+x \(\frac{d v}{d x}=\frac{-\left[x \sin ^2 v-v x\right]}{x} \Rightarrow v+x \frac{d v}{d x}=-\left[\sin ^2 v-v\right]=v-\sin ^2 v\)

⇒ \(x \frac{d v}{d x}=-\sin ^2 v \Rightarrow \frac{d v}{\sin ^2 v}=-\frac{d x}{x} \)

⇒ \(\mathrm{cosec}^2 v d v=-\frac{d x}{x} \Rightarrow \int \mathrm{cosec}^2 v d v=-\int \frac{d x}{x}\)

⇒ \(\cot \left(\frac{y}{x}\right)=\log |x|-\log C \Rightarrow \cot \left(\frac{y}{x}\right)=\log \left|\frac{x}{C}\right|\)…..(2)

Now, \(y=\frac{\pi}{4}\) at x=1 \(\Rightarrow \cot \left(\frac{\pi}{4}\right)=\log \left|\frac{1}{C}\right| \Rightarrow 1=-\log C \Rightarrow C=e^{-t}\)

Substituting \(\mathrm{C}=\mathrm{e}^{-1}\) in equation (2), we get : \(\cot \left(\frac{\mathrm{y}}{\mathrm{x}}\right)=\log |\mathrm{ex}|\)

This is the required solution of the given differential equation.

Question 14. \(\frac{d y}{d x}-\frac{y}{x}+\mathrm{cosec}\left(\frac{y}{x}\right)=0 ; y=0\) when x=1
Solution:

⇒ \(\frac{d y}{d x}=\frac{y}{x}-\mathrm{cosec}\left(\frac{y}{x}\right)\)…..(1)

The given differential equation is a homogeneous equation.

To solve it, we make the substitution as:

y = \(v x \Rightarrow \frac{d}{d x}(y)=\frac{d}{d x}(v x) \Rightarrow \frac{d y}{d x}=v+x \frac{d v}{d x}\)

Substituting the values of y and \(\frac{d y}{d x}\) in equation (1), we get:

v+x \(\frac{d v}{d x}=v-\mathrm{cosec} v \Rightarrow-\frac{d v}{\mathrm{cosec} v}=\frac{d x}{x}\)

⇒ \(-\sin v d v=\frac{d x}{x} \Rightarrow-\int \sin v d v=\int \frac{d x}{x}\)

cos v = \(\log x+\log C=\log |C x| \Rightarrow \cos \left(\frac{y}{x}\right)=\log |C x|\)….(2)

This is the required solution of the given differential equation.

Now, y=0 at x=1

⇒ cos (0) = \(\log \mathrm{C} \Rightarrow 1=\log \mathrm{C} \Rightarrow \mathrm{C}=\mathrm{e}^{\prime}=\mathrm{e}\)

Substituting C=e in equation (2), we get: \(\cos \left(\frac{y}{x}\right)=\log |(e x)|\)

This is the required solution of the given differential equation.

Question 15. \(2 x y+y^2-2 x^2 \frac{d y}{d x}=0 ; y=2\) when x=1
Solution:

2xy + \(y^2-2 x^2 \frac{d y}{d x}=0 \Rightarrow 2 x^2 \frac{d y}{d x}=2 x y+y^2 \Rightarrow \frac{d y}{d x}=\frac{2 x y+y^2}{2 x^2}\)….(1)

The given differential equation is a homogeneous equation.

To solve it, we make the substitution as:

y = \(v x \Rightarrow \frac{d}{d x}(y)=\frac{d}{d x}(v x) \Rightarrow \frac{d y}{d x}=v+x \frac{d v}{d x}\)

Substituting the value of y and \(\frac{d y}{d x}\) in equation (1), we get:

v+x \(\frac{d v}{d x}=\frac{2 x(v x)+(v x)^2}{2 x^2} \Rightarrow v+x \frac{d v}{d x}=\frac{2 v+v^2}{2} \Rightarrow v+x \frac{d v}{d x}=v+\frac{v^2}{2}\)

⇒ \(\frac{2}{v^2} d v=\frac{d x}{x} \Rightarrow \int \frac{2}{v^2} d v=\int \frac{d x}{x}\)

⇒ \(2 \cdot \frac{v^{-2 d}}{-2+1}=\log |x|+C \Rightarrow-\frac{2}{v}=\log |x|+C\)

⇒ \(-\frac{2}{y}=\log |x|+C \Rightarrow-\frac{2 x}{y}=\log |x|+C\)…..(2)

Now, y = 2 at x = l.

⇒ -l = log(1) + C ⇒ C = -1

Substituting C = -1 in equation (2), we get:

– \(\frac{2 x}{y}=\log |x|-1 \Rightarrow \frac{2 x}{y}=1-\log |x| \Rightarrow y=\frac{2 x}{1-\log |x|},(x \neq 0, x \neq e)\)

This is the required solution of the given differential equation.

Question 16. A homogeneous differential equation of the form \(\frac{dx}{dy}\) = h(\(\frac{x}{y}\)) can be solved by making the substitution

  1. y = vx
  2. v = yx
  3. x = vy
  4. x = v

Solution:

For solving the homogeneous equation of the form \(\frac{dx}{dy}\) = h(\(\frac{x}{y}\)), we need to make the substitution as x = vy.

Hence, the correct answer is 3.

Question 17. Which of the following is a homogeneous differential equation?

  1. (4x + 6y + 5)dy – (3y + 2x + 4)dx = 0
  2. (xy)dx – (x³ + y³)dy = 0
  3. (x³ + 2y²)dx + 2xydy = 0
  4. y²dx + (x² – xy – y²)dy = 0

Solution:

Out of the given four options; option (4) is the only option in which all coefficients of dx and dy are of the same degree, therefore function F(x, y) is said to be the homogenous function of degree n, if

F(λx, λy) = λ” F(x, y) for any non-zero constant (X).

Consider the equation given in option D: y²dx + (x² – xy – y²)dy = 0

⇒ \(\frac{d y}{d x}=\frac{-y^2}{x^2-x y-y^2}=\frac{y^2}{y^2+x y-x^2}\)

Let F(x, y) = \(\frac{y^2}{y^2+x y-x^2} \Rightarrow F(\lambda x, \lambda y)=\frac{(\lambda y)^2}{(\lambda y)^2+(\lambda x)(\lambda y)-(\lambda x)^2}\)

= \(\frac{\lambda^2 y^2}{\lambda^2\left(y^2+x y-x^2\right)}=\lambda^0\left(\frac{y^2}{y^2+x y-x^2}\right)=\lambda^0 \cdot F(x, y)\)

Hence, the differential equation given in option 4 is a homogenous equation.

Differential Equation Exercise 9.5

For Each Of The Differential Equations, Find The General Solution

Question 1. \(\frac{d y}{d x}+2 y=\sin x\)
Solution:

The given differential equation is \(\frac{\mathrm{dy}}{\mathrm{dx}}+2 \mathrm{y}=\sin \mathrm{x}\)

This is a linear differential equation of the form \(\frac{d y}{d x}+P y=Q\) (where P=2 and Q= sin x)

Now, I.F. = \(\mathrm{e}^{\int \mathrm{Pdtx}}=\mathrm{e}^{\int 2 \mathrm{dx}}=\mathrm{e}^{2 \mathrm{x}}\)

The solution of the given differential equation is given by the relation,

y(I.F) = \(\int(Q \times \text { I.F. }) d x+C \Rightarrow y e^{2 x}=\int \sin x \cdot e^{2 x} d x+C\)…..(1)

Let \(I=\int_I \sin x \cdot e^{2 x} d x\)

⇒ \(I=\sin x \cdot \int e^{2 x} d x-\int\left(\frac{d}{d x}(\sin x) \cdot \int e^{2 x} d x\right) d x\)

⇒ \(I=\sin x \cdot \frac{e^{2 x}}{2}-\int\left(\cos x \cdot \frac{e^{2 x}}{2}\right) d x\)

⇒ I = \(\frac{e^{2 x} \sin x}{2}-\frac{1}{2}\left[\cos x \cdot \int e^{2 x}-\int\left(\frac{d}{d x}(\cos x) \cdot \int e^{2 x} d x\right) d x\right]\)

⇒ \(I=\frac{e^{2 x} \sin x}{2}-\frac{1}{2}\left[\cos x \cdot \frac{e^{2 x}}{2}-\int\left[(-\sin x) \cdot \frac{e^{2 x}}{2}\right] d x\right]\)

⇒ I = \(\frac{e^{2 x} \sin x}{2}-\frac{e^{2 x} \cos x}{4}-\frac{1}{4} \int\left(\sin x \cdot e^{2 x}\right) d x\)

⇒ \(I=\frac{e^{2 x}}{4}(2 \sin x-\cos x)-\frac{1}{4} I \Rightarrow \frac{5}{4} I=\frac{e^{2 x}}{4}(2 \sin x-\cos x)\)

⇒ \(I=\frac{e^{2 x}}{5}(2 \sin x-\cos x)\)

Therefore, equation (1) becomes: \(\mathrm{ye}^{2 \mathrm{x}}=\frac{\mathrm{e}^{2 \mathrm{x}}}{5}(2 \sin \mathrm{x}-\cos \mathrm{x})+\mathrm{C} \Rightarrow \mathrm{y}=\frac{1}{5}(2 \sin \mathrm{x}-\cos \mathrm{x})+\mathrm{Ce}^{-2 \mathrm{x}}\)

This is the required general solution of the given differential equation.

Question 2. \(\frac{d y}{d x}+3 y=e^{-2 x}\)
Solution:

The given differential equation is \(\frac{d y}{d x}+3 y=e^{-2 x}\).

This is a linear differential equation of the form \(\frac{d y}{d x}+P y=Q\) (where P=3 and Q = \(e^{-2 x}\))

Now, I.F. = \(\mathrm{e}^{\int \mathrm{Pdx}}=\mathrm{e}^{\int \frac{3 \mathrm{dx}}{\mathrm{s}}=\mathrm{e}^{3 \mathrm{x}}}\)

The solution of the given differential equation is given by the relation,

y(I.F.)= \(\int(Q \times \text { I.F. }) d x+C \Rightarrow y e^{3 x}=\int\left(e^{-2 x} \times e^{3 x}\right)+C \Rightarrow y e^{3 x}=\int e^x d x+C\)

⇒ \(y e^{3 x}=e^x+C \Rightarrow y=e^{-2 x}+C e^{-5 x}\)

This is the required general solution of the given differential equation.

Question 3. \(\frac{d y}{d x}+\frac{y}{x}=x^2\)
Solution:

The given differential equation is \(\frac{d y}{d x}+\frac{y}{x}=x^2\).

This is a linear differential equation of the form \(\frac{d y}{d x}+P y=Q\) (where \(P=\frac{1}{x}\) and \(Q=x^2\))

Now, I.F. = \(\mathrm{e}^{\int \text { Pdx }}=\mathrm{e}^{\int \frac{1}{x}-\mathrm{dx}}=\mathrm{e}^{\log \mathrm{x}}=\mathrm{x}\)

The solution of the given differential equation is given by,

y(I.F.) = \(\int(Q \times I . F .) d x+C \Rightarrow y(x)=\int\left(x^2 \cdot x\right) d x+C\)

⇒ \(x y=\int x^3 d x+C \Rightarrow x y=\frac{x^4}{4}+C\)

This is the required general solution of the given differential equation.

Question 4. \(\frac{d y}{d x}+(\sec x) y=\tan x\left(0 \leq x<\frac{\pi}{2}\right)\)
Solution:

The given differential equation is \(\frac{\mathrm{dy}}{\mathrm{dx}}+(\sec x) \mathrm{y}=\tan \mathrm{x}\).

This is a linear differential equation of the form \(\frac{d y}{d x}+P y=Q\) (where P=sec x and Q=tan x)

Now, I.F. = \(\mathrm{e}^{\int \mathrm{Prtx}}=\mathrm{e}^{\int \sec x d x}=\mathrm{e}^{\log (\sec x+\tan x)}=\sec \mathrm{x}+\tan \mathrm{x}\)

The general solution of the given differential equation is given by the relation,

y(I.F.) = \(\int(Q \times \text { I.F. }) d x+C\)

⇒ y(sec x+tan x) = \(\int \tan x(\sec x+\tan x) d x+C \Rightarrow y(\sec x+\tan x)=\int \sec x \tan x d x+\int \tan ^2 x d x+C\)

⇒ y(sec x+tan x) = \(\sec x+\int\left(\sec ^2 x-1\right) d x+C \Rightarrow y(\sec x+\tan x)=\sec x+\tan x-x+C\)

Question 5. \(\cos ^2 x \frac{d y}{d x}+y=\tan x\left(0 \leq x<\frac{\pi}{2}\right)\)
Solution:

It is given that \(\cos ^2 x \frac{d y}{d x}+y=\tan x \Rightarrow \frac{d y}{d x}+\sec ^2 x y=\sec ^2 x \tan x\)

This is a differential equation in the form of \(\frac{d y}{d x}+P y=Q\) (where, P= \(sec ^2 x\) and Q = \(\sec ^2 x \tan x\))

Now, I.F. = \(\mathrm{e}^{\int \mathrm{Pdx}}=\mathrm{e}^{\int \sec ^2 \mathrm{x} d \mathrm{x}}=\mathrm{e}^{\tan \mathrm{x}}\)

Thus, the solution of the given differential equation is given by the relation :

y(I.F.) = \(\int(Q \times I . F) d x+C\)

⇒ \(y \cdot e^{\mathrm{tan} x}=\int e^{\mathrm{tan} x} \sec ^2 x \tan x d x+C\)….(1)

Now, Let tan x = \(t \Rightarrow \sec ^2 x d x=d t\)

Thus, the equation (1) becomes

⇒ \(y \cdot e^{\tan x}=\int\left(e^t \cdot t\right) d t+C \Rightarrow y \cdot e^{\tan x}=t \cdot \int e^t d t-\int\left(\frac{d}{d t}(t) \cdot \int e^t d t\right) d t+C\)

⇒ \(y \cdot e^{\tan x}=t \cdot e^t-\int e^t d t+C \Rightarrow y e^{\tan x}=(t-1) e^t+C\)

⇒ \(y e^{\tan x}=(\tan x-1) e^{\tan x}+C \Rightarrow y=(\tan x-1)+C e^{-\tan x}\)

Therefore, the required general solution of the given differential equation \(y=(\tan x-1)+C e^{-\tan x}\)

Question 6. \(x \frac{d y}{d x}+2 y=x^2 \log x\)
Solution:

The given differential equation is : \(x \frac{d y}{d x}+2 y=x^2 \log x \Rightarrow \frac{d y}{d x}+\frac{2}{x} y=x \log x\)

This equation is in the form of a linear differential equation as: \(\frac{d y}{d x}\) +Py=Q (where P = \(\frac{2}{x}\) and Q = x log x)

Now, I.F. = \(\mathrm{e}^{\int \mathrm{Pdx}}=\mathrm{e}^{\int_{\mathrm{x}}^2 \mathrm{dx}}=\mathrm{e}^{2 \log x}=\mathrm{e}^{\log \mathrm{x}^2}=\mathrm{x}^2\)

The general solution of the given differential equation is given by the relation,

y(I.F.) = \(\int(Q \times I . F .) d x+C \Rightarrow y \cdot x^2=\int\left(x \log x \cdot x^2\right) d x+C\)

⇒ \(x^2 y=\int\left(x^3 \log x\right) d x+C \Rightarrow x^2 y=\log x \cdot \int x^3 d x-\int\left[\frac{d}{d x}(\log x) \cdot \int x^3 d x\right] d x+C\)

⇒ \(x^2 y=\log x \cdot \frac{x^4}{4}-\int\left(\frac{1}{x} \frac{x^4}{4}\right) d x+C \Rightarrow x^2 y=\frac{x^4 \log x}{4}-\frac{1}{4} \int x^3 d x+C\)

⇒ \(x^2 y=\frac{x^4 \log x}{4}-\frac{1}{4} \cdot \frac{x^4}{4}+C \Rightarrow x^2 y=\frac{1}{16} x^4(4 \log x-1)+C\)

⇒ \(y=\frac{1}{16} x^2(4 \log x-1)+C x^{-2}\)

Question 7. \(x \log x \frac{d y}{d x}+y=\frac{2}{x} \log x\)
Solution:

The given differential equation is: \(x \log x \frac{d y}{d x}+y=\frac{2}{x} \log x \Rightarrow \frac{d y}{d x}+\frac{y}{x \log x}=\frac{2}{x^2}\)

This equation is the form of a linear differential equation as: \(\frac{d y}{d x}+P y=Q \quad\left(\text { where } P=\frac{1}{x \log x} \text { and } Q=\frac{2}{x^2}\right)\)

Now, I.F. = \(\mathrm{e}^{\int \mathrm{Pdx}}=\mathrm{e}^{\int \frac{1}{x \log x} d x}=\mathrm{e}^{\log (\log x)}=\log \mathrm{x}\)

The general solution of the given differential equation is given by the relation,

y(I.F .) = \(\int(Q \times \text { I.F. }) d x+C \Rightarrow y \log x=\int\left(\frac{2}{x^2} \log x\right) d x+C\)

Now, \(\int\left(\frac{2}{x^2} \log x\right) d x=2 \int\left(\log x \cdot \frac{1}{x^2}\right) d x\)

= \(2\left[\log x \cdot \int \frac{1}{x^2} d x-\int\left\{\frac{d}{d x}(\log x) \cdot \int \frac{1}{x^2} d x\right\} d x\right]\)

= \(2\left[\log x\left(-\frac{1}{x}\right)-\int\left(\frac{1}{x} \cdot\left(-\frac{1}{x}\right)\right) d x\right]=2\left[-\frac{\log x}{x}+\int \frac{1}{x^2} d x\right]\)

= \(2\left[-\frac{\log x}{x}-\frac{1}{x}\right]=-\frac{2}{x}(1+\log x)\)

Substituting the value of \(\int\left(\frac{2}{x^2} \log x\right) d x\) in equation (1), we get : \(y \log x=-\frac{2}{x}(1+\log x)+C\)

This is the required general solution of the given differential equation.

Question 8. \(\left(1+x^2\right) d y+2 x y d x=\cot x d x(x \neq 0)\)
Solution:

(1+ \(x^2\)) dy+2 x y dx =cot x d x

⇒ \(\frac{d y}{d x}+\frac{2 x y}{1+x^2}=\frac{\cot x}{1+x^2}\)

This equation is a linear differential equation of the form:

⇒ \(\frac{d y}{d x}+P y=Q\)(where P = \(\frac{2 x}{1+x^2}\) and Q = \(\frac{\cot x}{1+x^2}\))

Now, I.F. = \(\mathrm{e}^{\int \mathrm{Pdx}}=\mathrm{e}^{\int \frac{2 \mathrm{x}}{1+\mathrm{x}^2} \mathrm{dx}}=\mathrm{e}^{\log \left(1+\mathrm{x}^2\right)}=1+\mathrm{x}^2\)

The general solution of the given differential equation is given by the relation,

y(I.F.) = \(\int(Q \times \text { I.F. }) d x+C \Rightarrow y\left(1+x^2\right)=\int\left[\frac{\cot x}{1+x^2} \times\left(1+x^2\right)\right] d x+C\)

⇒ y\(\left(1+x^2\right)=\int \cot x d x+C \Rightarrow y\left(1+x^2\right)=\log |\sin x|+C\)

Question 9. \(x \frac{d y}{d x}+y-x+x y \cot x=0(x \neq 0)\)
Solution:

⇒ \(x \frac{d y}{d x}+y-x+x y \cot x=0 \Rightarrow x \frac{d y}{d x}+y(1+x \cot x)=x \Rightarrow \frac{d y}{d x}+\left(\frac{1}{x}+\cot x\right) y=1\)

This equation is a linear differential equation of the form:

⇒ \(\frac{d y}{d x}+P y=Q\) (where P = \(\frac{1}{x}+\cot x\) and Q= 1)

Now, I.F. \(=\mathrm{e}^{\int P d x}=\mathrm{e}^{\int\left(\frac{1}{x}+\cot x\right) d x}=e^{\log x+\log (\sin x)}=e^{\log (x \sin x)}=x \sin x\)

The general solution of the given differential equation is given by the relation,

y(I.F.) = \(\int(Q \times \text { I.F. }) d x+C\)

⇒ \(y(x \sin x)=\int(1 \times x \sin x) d x+C \Rightarrow y(x \sin x)=\int(x \sin x) d x+C \)

⇒ \(y(x \sin x)=x \int \sin x d x-\int\left[\frac{d}{d x}(x) \cdot \int \sin x d x\right]+C \)

⇒ \(y(x \sin x)=x(-\cos x)-\int 1 \cdot(-\cos x) d x+C\)

⇒ \(y(x \sin x)=-x \cos x+\sin x+C \Rightarrow y=\frac{-x \cos x}{x \sin x}+\frac{\sin x}{x \sin x}+\frac{C}{x \sin x}\)

⇒ \(y=-\cot \cdot x+\frac{1}{x}+\frac{C}{x \sin x}\) (which is the required solution)

Question 10. \((x+y) \frac{d y}{d x}=1\)
Solution:

⇒ \((x+y) \frac{d y}{d x}=1 \Rightarrow \frac{d y}{d x}=\frac{1}{x+y} \Rightarrow \frac{d x}{d y}=x+y \Rightarrow \frac{d x}{d y}-x=y\)

This is a linear differential equation of the form: \(\frac{\mathrm{dx}}{\mathrm{dy}}+\mathrm{P}_1 \mathrm{x}=\mathrm{Q}_1\) (where \(\mathrm{P}_1=-1\) and \(\mathrm{Q}_1=\mathrm{y}\))

Now, I.F. = \(\mathrm{e}^{\int P_1 d y}=\mathrm{e}^{\int-\mathrm{dy}}=\mathrm{e}^{-\mathrm{y}}\)

The general solution of the given differential equation is given by the relation,

x(I.F.) = \(\int\left(Q_1 \times I . F .\right) d y+C\)

⇒ \(x^{-y}=\int\left(y \cdot e^{-y}\right) d y+C \Rightarrow x e^{-y}=y \cdot \int e^{-y} d y-\int\left[\frac{d}{d y}(y) \int e^{-y} d y\right] d y+C\)

⇒ \(x^{-y}=y\left(-e^{-y}\right)-\int\left(-e^{-y}\right) d y+C \Rightarrow \mathrm{xe}^{-y}=-y e^{-y}+\int e^{-y} d y+C\)

⇒ \(x^{-y}=-y e^{-y}-e^{-y}+C \Rightarrow x=-y-1+C e^z\)

⇒ \(x+y+1=C e^y\) (which is the required solution)

Question 11. \(y d x+\left(x-y^2\right) d y=0\)
Solution:

⇒ \(y d x+\left(x-y^2\right) d y=0 \Rightarrow y d x=\left(y^2-x\right) d y \Rightarrow \frac{d x}{d y}\)

= \(\frac{y^2-x}{y}=y-\frac{x}{y} \Rightarrow \frac{d x}{d y}+\frac{x}{y}=y\)

This is a linear differential equation of the form:

⇒ \(\frac{\mathrm{dx}}{\mathrm{dy}}+\mathrm{P}_1 \mathrm{x}=\mathrm{Q}_1\) (where \( \mathrm{P}_1=\frac{1}{\mathrm{y}}\) and \(\mathrm{Q}_1=\mathrm{y}\))

Now. I.F. = \(\mathrm{e}^{\int {p_1} d y}=\mathrm{e}^{\int \frac{1}{y} d y}=\mathrm{e}^{\log y}=y\)

The general solution of the given differential equation is given by the relation,

x( I.F.) = \(\int\left(Q_1 \times I . F .\right) d y+C \Rightarrow x y=\int(y \cdot y) d y+C \Rightarrow x y=\int y^2 d y+C\)

⇒ xy = \(\frac{y^3}{3}+C \Rightarrow x=\frac{y^2}{3}+\frac{C}{y}\)

Question 12. \(\left(x+3 y^2\right) \frac{d y}{d x}=y(y>0)\)
Solution:

⇒ \(\left(x+3 y^2\right) \frac{d y}{d x}=y \Rightarrow \frac{d y}{d x}=\frac{y}{x+3 y^2} \Rightarrow \frac{d x}{d y}\)

= \(\frac{x+3 y^2}{y}=\frac{x}{y}+3 y \Rightarrow \frac{d x}{d y}-\frac{x}{y}=3 y\)

This is a linear differential equation of the form:

⇒ \(\frac{\mathrm{dx}}{\mathrm{dy}}+\mathrm{P}_1 \mathrm{x}=\mathrm{Q}_1\) (where \(\mathrm{P}_1=-\frac{1}{\mathrm{y}}\) and \(\mathrm{Q}_{\mathbf{l}}=3 \mathrm{y}\))

Now, I.F. = \(\mathrm{e}^{\int P_1 d y}=\mathrm{e}^{-\int \frac{d y}{y}}=\mathrm{e}^{-\log y}=\mathrm{e}^{\log \left(\frac{1}{y}\right)}=\frac{1}{y}\)

The general solution of the given differential equation is given by the relation,

x(I .F.) = \(\int\left(Q_1 \times \text { I.F. }\right) d y+C\)

⇒ \(x \times \frac{1}{y}=\int\left(3 y \times \frac{1}{y}\right) d y+C \Rightarrow \frac{x}{y}=3 y+C \Rightarrow x=3 y^2+C y\)

For Each Of The Differential Equations, Find The Particular Solution Satisfying The Given Conditions:

Question 13. \(\frac{d y}{d x}+2 y \tan x=\sin x ; y=0\) when \(x=\frac{\pi}{3}\)
Solution:

The given differential equation is \(\frac{d y}{d x}+2 y \tan x=\sin x\)

This is a linear equation of the form: \(\frac{\mathrm{dy}}{\mathrm{dx}}+\mathrm{Py}=\mathrm{Q}\)(where \(\mathrm{P}=2 \tan \mathrm{x}\) and \(\mathrm{Q}=\sin \mathrm{x}\))

Now, I.F. = \(\mathrm{e}^{\int \mathrm{pdx}}=\mathrm{e}^{\int 2 \mathrm{sin} x d x}=\mathrm{e}^{2 \log |\sec x|}=\mathrm{e}^{\log \left(\sec ^2 \mathrm{x}\right)}=\sec ^2 \mathrm{x}\)

The general solution of the given differential equation is given by the relation,

y(I.F) = \(\int(Q \times \text { I.F. }) d x+C\)

⇒ \(y\left(\sec ^2 x\right)=\int\left(\sin x \cdot \sec ^2 x\right) d x+C \Rightarrow y \sec ^2 x=\int(\sec x \cdot \tan x) d x+C\)

⇒ \(y \sec ^2 x=\sec x+C\)

Put, y=0 and x \(=\frac{\pi}{3}\), in eq.(1)

Therefore, \(0 \times \sec ^2 \frac{\pi}{3}=\sec \frac{\pi}{3}+\mathrm{C} \Rightarrow 0=2+\mathrm{C} \Rightarrow \mathrm{C}=-2\)

Substituting C=-2 in equation (1), we get: \(y \sec ^2 x=\sec x-2 \Rightarrow y=\cos x-2 \cos ^2 x\)

Hence, the required solution of the given differential equation is \(y=\cos x-2 \cos ^2 x\).

Question 14. \(\left(1+x^2\right) \frac{d y}{d x}+2 x y=\frac{1}{1+x^2} ; y=0\) when x=1
Solution:

⇒ \(\left(1+x^2\right) \frac{d y}{d x}+2 x y=\frac{1}{1+x^2} \Rightarrow \frac{d y}{d x}+\frac{2 x y}{1+x^2}=\frac{1}{\left(1+x^2\right)^2}\)

This is a linear differential equation of the form: \(\frac{d y}{d x}+P y=Q\) (where P = \(\frac{2 x}{1+x^2}\) and Q = \(\frac{1}{\left(1+x^2\right)^2}\))

Now, I.F. \(=\mathrm{e}^{\int P \mathrm{Pdx}}=\mathrm{e}^{\int \frac{7 \mathrm{xdx}}{1+\mathrm{x}^2}}=\mathrm{e}^{\log \left(1+\mathrm{x}^2\right)}=1+\mathrm{x}^2\)

The general solution of the given differential equation is given by the relation,

y(I.F.) = \(\int(Q \times I . F .) d x+C\)

⇒ \(y\left(1+x^2\right)=\int\left[\frac{1}{\left(1+x^2\right)^2} \cdot\left(1+x^2\right)\right] d x+C\)

⇒ \(y\left(1+x^2\right)=\int \frac{1}{1+x^2} d x+C \Rightarrow y\left(1+x^2\right)=\tan ^{-1} x+C\)….(1)

Put y=0 and x=1 in eq.(1), we get

0 = \(\tan ^{-1} 1+C \Rightarrow C=-\frac{\pi}{4}\)

Substituting C = \(-\frac{\pi}{4}\) in equation (1), we get : \(y\left(1+x^2\right)=\tan ^{-1} x-\frac{\pi}{4}\)

This is the required general solution of the given differential equation.

Question 15. \(\frac{\mathrm{dy}}{\mathrm{dx}}-3 \mathrm{y} \cot \mathrm{x}=\sin 2 \mathrm{x} ; \mathrm{y}=2\) when \(\mathrm{x}=\frac{\pi}{2}\)
Solution:

The given differential equation is \(\frac{d y}{d x}-3 y \cot x=\sin 2 x\)

This is a linear differential equation of the form: \(\frac{d y}{d x}+P y=Q\) (where P=-3cot x and Q=sin 2 x)

Now, I.F. = \(\mathrm{e}^{\int P \mathrm{Pdx}}=\mathrm{e}^{-3 \int \cot x d x}=\mathrm{e}^{-3 \log |\sin x|}=\mathrm{e}^{\log \left|\frac{1}{\sin } \mathrm{x}\right|}=\frac{1}{\sin ^3 \mathrm{x}}\)

The general solution of the given differential equation is given by the relation,

y(I.F.) = \(\int(Q \times \text { I.F. }) d x+C\)

⇒ \(y \cdot \frac{1}{\sin ^3 x}=\int\left[\sin 2 x \cdot \frac{1}{\sin ^3 x}\right] d x+C \Rightarrow y \mathrm{cosec}^3 x=2 \int(\cot x \cos e c x) d x+C\)

⇒ \(y \mathrm{cosec}^3 x=-2 \mathrm{cosec} x+C \Rightarrow y=-\frac{2}{\mathrm{cosec}^2 x}+\frac{C}{\mathrm{cosec}^3 x}\)

⇒ y = \(-2 \sin ^2 x+C \sin ^3 x\)……(1)

Put y=2 and \(x=\frac{\pi}{2}\) in eq.(1) we get

2 = \(-2 \sin ^2 \frac{\pi}{2}+C \sin ^3 \frac{\pi}{2} \Rightarrow 2=-2+C \Rightarrow C=4\)

Substituting C=4 in equation (1), we get:

y = \(-2 \sin ^2 x+4 \sin ^3 x \Rightarrow y=4 \sin ^3 x-2 \sin ^2 x\)

Question 16. Find the equation of a curve passing through the origin given that the slope of the tangent to the curve at any point (x, y) is equal to the sum of the coordinates of the point.
Solution:

Lel F (x, y) is the curve passing through the origin.

At point (x, y), the scope of the curve will be \(\frac{dy}{dx}\)

According to the given information: \(\frac{d y}{d x}=x+y \Rightarrow \frac{d y}{d x}-y=x\)

This is a linear differential equation of the form: \(\frac{\mathrm{dy}}{\mathrm{dx}}+\mathrm{Py}=\mathrm{Q}\) (where P=-1 and Q = x)

Now, I.F. = \(\mathrm{e}^{\int 1 \mathrm{~d} d \mathrm{x}}=\mathrm{e}^{\int(-1) d x}=\mathrm{e}^{-\mathrm{x}}\)

The general solution of the given differential equation is given by the relation,

y(I.F.) = \(\int(Q \times \text { I.F. }) d x+C \Rightarrow \mathrm{ye}^{-x}=\int x e^{-x} d x+C\)……(1)

Now, \(\int x e^{-x} d x=x \int e^{-x} d x-\int\left[\frac{d}{d x}(x) \cdot \int e^{-x} d x\right] d x\)

= \(-x e^{-x}-\int-e^{-x} d x=-x e^{-x}+\left(-e^{-x}\right)=-e^{-x}(x+1)\)

Substituting in equation (1), we get:

⇒ \(\mathrm{ye}^{-x}=-\mathrm{e}^{-3}(\mathrm{x}+1)+\mathrm{C}\)

⇒ \(\mathrm{y}=-(\mathrm{x}+1)+\mathrm{Ce}^x \Rightarrow \mathrm{x}+\mathrm{y}+1=\mathrm{Ce}^{\mathrm{x}}\)….(2)

The curve passes through the origin. So, put x=0, y=0, and equation (2) becomes:

0+0+1= \(\mathrm{Ce}^0\) C=1

Substituting C =1 in equation (2), we get: x+y+1=\(e^x\)

Hence, the required equation of the curve passing through the origin is \(x+y+1=e^x\)

Question 17. Find the equation of a curve passing through the point (0, 2) given that the sum of the coordinates of any point on the curve exceeds the magnitude of ths slope of the tangent to the curve at that point by 5.
Solution:

Let F (x, y) be the curve, and let (x, y) be a point on the curve. The slope of the tangent to the curve at (x, y) is \(\frac{dy}{dx}\)

According to the given information: \(\frac{d y}{d x}+5=x+y \Rightarrow \frac{d y}{d x}-y=x-5\)

This is a linear differential equation of the form: \(\frac{d y}{d x}+P y=Q\) (where P=-1 and Q=x-5)

Now, I.F. = \(\mathrm{e}^{\int P d x}=\mathrm{e}^{\int(-1) d x}=\mathrm{e}^{-\mathrm{x}}\)

The general equation of the curve is given by the relation,

y(I.F.) = \(\int(Q \times I.F.) d x+C \Rightarrow y \cdot e^{-x}=\int(x-5) e^{-x} d x+C\)…..(1)

Now, \(\int(x-5) e^{-x} d x=(x-5) \int e^{-x} d x-\int\left[\frac{d}{d x}(x-5) \cdot \int e^{-x} d x\right] d x\)

= \((x-5)\left(-e^{-x}\right)-\int\left(-e^{-x}\right) d x=(5-x) e^{-x}+\left(-e^{-x}\right)=(4-x) e^{-x}\)

Therefore, equation (1) becomes: \(y^{-x x}=(4-x) e^{-x}+C\)

y = \(4-x+C e^x \Rightarrow x+y-4=C e^x\)…..(2)

The curve passes through the point (0,2)

Therefore, equation (2) becomes : \(0+2-4=\mathrm{Ce}^0 \Rightarrow-2=\mathrm{C} \Rightarrow \mathrm{C}=-2\)

Substituting C=-2 in equation (2), we get: \(\mathrm{x}+\mathrm{y}-4=-2 \mathrm{e}^{\mathrm{x}} \Rightarrow \mathrm{y}=4-\mathrm{x}-2 \mathrm{e}^{\mathrm{x}}\)

This is the required equation of the curve.

Choose The Correct Answer In The Following

Question 18. The integrating factor of the differential equation \(x \frac{d y}{d x}-y=2 x^2\) is

  1. \(\mathrm{e}^{x}\)
  2. \(\mathrm{e}^{-y}\)
  3. \(\frac{1}{\mathrm{x}}\)
  4. \(\mathrm{x}\)

Solution: 3. \(\frac{1}{\mathrm{x}}\)

The given differential equation is: \(x \frac{d y}{d x}-y=2 x^2 \Rightarrow \frac{d y}{d x}-\frac{y}{x}=2 x\)

This is a linear differential equation of the form: \(\frac{d y}{d x}+P y=Q\) (where \(P=-\frac{1}{x}\) and Q=2x)

The integrating factor (I.F.) is given by the relation, \(\mathrm{e}^{\int \mathrm{Pdx}}\)

∴ I.F. = \(\mathrm{e}^{\int-\frac{1}{x} d x}=\mathrm{e}^{-\log x}=\mathrm{e}^{\log \left[x^{-1}\right\}}=\mathrm{x}^{-1}=\frac{1}{\mathrm{x}}\)

Hence, the correct answer is 3.

Question 19. The integrating factor of the differential equation \(\left(1-y^2\right) \frac{d x}{d y}+y x=a y(-1<y<1)\) is

  1. \(\frac{1}{y^2-1}\)
  2. \(\frac{1}{\sqrt{y^2-1}}\)
  3. \(\frac{1}{1-y^2}\)
  4. \(\frac{1}{\sqrt{1-y^2}}\)

Solution: 4. \(\frac{1}{\sqrt{1-y^2}}\)

The given differential equation is: \(\left(1-y^2\right) \frac{d x}{d y}+y x=a y \Rightarrow \frac{d x}{d y}+\frac{y x}{1-y^2}=\frac{a y}{1-y^2}\)

This is a linear differential equation of the form: \(\frac{\mathrm{dx}}{\mathrm{dy}}+\mathrm{Px}=\mathrm{Q}\) (where \(\mathrm{P}=\frac{\mathrm{y}}{1-\mathrm{y}^2}\) and \(\mathrm{Q}=\frac{\mathrm{ay}}{1-\mathrm{y}^2}\))

The integrating factor (I.F.) is given by the relation,

∴ I.F. = \(\mathrm{e}^{\int \text { P.dy }}=\mathrm{e}^{\int \frac{y}{1-y^2} d y}=\mathrm{e}^{-\frac{1}{2} \log \left(1-y^2\right)}=\mathrm{e}^{\log \left[\frac{1}{\sqrt{1-y^2}}\right]}=\frac{1}{\sqrt{1-y^2}}\)

Hence, the correct answer is 4.

Differential Equations Miscellaneous Exercise

Question 1. For each of the differential equations given below, indicate its order and degree (if defined).

  1. \(\frac{d^2 y}{d x^2}+5 x\left(\frac{d y}{d x}\right)^2-6 y=\log x\)
  2. \(\left(\frac{d y}{d x}\right)^3-4\left(\frac{d y}{d x}\right)^2+7 y=\sin x\)
  3. \(\frac{\mathrm{d}^4 y}{\mathrm{dx}^4}-\sin \left(\frac{\mathrm{d}^3 \mathrm{y}}{\mathrm{dx}^3}\right)=0\)

Solution:

1. The differential equation is given as:

⇒ \(\frac{d^2 y}{d x^2}+5 x\left(\frac{d y}{d x}\right)^2-6 y=\log x \Rightarrow \frac{d^2 y}{d x^2}+5 x\left(\frac{d y}{d x}\right)^2-6 y-\log x=0\)

The highest order derivative present in the differential equation is \(\frac{\mathrm{d}^2 \mathrm{y}}{\mathrm{dx}^2}\).

Thus, its order is two. The highest power raised to \(\frac{\mathrm{d}^2 \mathrm{y}}{\mathrm{dx}^2}\) is one. Hence, its degree is one.

2. The differential equation is given as:

⇒ \(\left(\frac{d y}{d x}\right)^3-4\left(\frac{d y}{d x}\right)^2+7 y=\sin x \Rightarrow\left(\frac{d y}{d x}\right)^3-4\left(\frac{d y}{d x}\right)^2+7 y-\sin x=0\)

The highest order derivative present in the differential equation is \(\frac{d y}{d x}\).

Thus, its order is one. The highest power raised to \(\frac{\mathrm{dy}}{\mathrm{dx}}\) is three. Hence, its degree is three.

3. The differential equation is given as: \(\frac{d^4 y}{d x^4}-\sin \left(\frac{d^3 y}{d x^3}\right)=0\)

The highest order derivative p-=resent in the differential equation, \(\frac{d^4 y}{d x^4}\). Thus, its order is four.

However, the given differential equation is not a polynomial equation in its derivatives,

Hence, its degree is not defined.

Question 2. For each of the exercises given below, verify that the given function (implicit or explicit) is a solution of the corresponding differential equation.

  1. \(y=a e^x+b e^{-x}+x^2 \quad: \quad x \frac{d^2 y}{d x^2}+2 \frac{d y}{d x}-x y+x^2-2=0\)
  2. \(\mathrm{y}=\mathrm{e}^{\mathrm{x}}(\mathrm{a} \cos \mathrm{x}+\mathrm{b} \sin \mathrm{x}) \quad: \frac{\mathrm{d}^2 \mathrm{y}}{\mathrm{dx}^2}-2 \frac{\mathrm{dy}}{\mathrm{dx}}+2 \mathrm{y}=0\)
  3. \(\mathrm{y}=\mathrm{x} \sin 3 \mathrm{x} \quad: \frac{\mathrm{d}^2 \mathrm{y}}{\mathrm{dx}^2}+9 \mathrm{y}-6 \cos 3 \mathrm{x}=0\)
  4. \(x^2=2 y^2 \log y \quad: \left(x^2+y^2\right) \frac{d y}{d x}-x y=0\)

Solution:

1. \(\mathrm{y}=a \mathrm{e}^{\mathrm{x}}+b \mathrm{e}^{-\mathrm{x}}+\mathrm{x}^2\)

Differentiating both sides with respect to x, we get:

⇒ \(\frac{d y}{d x}=a \frac{d}{d x}\left(e^x\right)+b \frac{d}{d x}\left(e^{-x}\right)+\frac{d}{d x}\left(x^2\right) \Rightarrow \frac{d y}{d x}=a e^x-b e^{-x}+2 x\)

Again, differentiating both sides with respect to x, we get: \(\frac{d^2 y}{d x^2}=a e^x+b e^{-x}+2\)

Now, on substituting the values of \(\frac{\mathrm{dy}}{\mathrm{dx}}\) and \(\frac{\mathrm{d}^2 \mathrm{y}}{\mathrm{dx}^2}\) in the differential equation, we get:

L.H.S. = \(x \frac{d^2 y}{d x^2}+2 \frac{d y}{d x}-x y+x^2-2\)

= \(x\left(a e^x+b e^{-x}+2\right)+2\left(a e^x-b e^{-x}+2 x\right)-x\left(a e^x+b e^{-x}+x^2\right)+x^2-2\)

= \(\left(a x e^x+b x e^{-x}+2 x\right)+\left(2 a e^x-2 b e^{-x}+4 x\right)-\left(a x e^x+b x e^{-4}+x^3\right)+x^2-2\)

= \(2 \mathrm{ae}^{\mathrm{x}}-2 \mathrm{be}^{-\mathrm{x}}-\mathrm{x}^3+\mathrm{x}^2+6 \mathrm{x}-2 \neq 0\)

⇒ \({ L.H.S. } \neq \text { R.H.S. }\)

Hence, the given function is not a solution to the corresponding differential equation.

2. \(\mathrm{y}=\mathrm{e}^{\mathrm{x}}(\mathrm{a} \cos \mathrm{x}+\mathrm{b} \sin \mathrm{x})=a \mathrm{e}^{\mathrm{x}} \cos \mathrm{x}+b \mathrm{e}^x \sin \mathrm{x}\)

Differentiating both sides with respect to x, we get:

⇒ \(\frac{d y}{d x}=a \cdot \frac{d}{d x}\left(e^x \cos x\right)+b \cdot \frac{d}{d x}\left(e^x \sin x\right)\)

⇒ \(\frac{d y}{d x}=a\left(e^x \cos x-e^x \sin x\right)+b \cdot\left(e^x \sin x+e^x \cos x\right)\)

⇒ \(\frac{d y}{d x}=(a+b) e^x \cos x+(b-a) e^x \sin x\)

Again, differentiating both sides with respect to x, we get:

⇒ \(\frac{d^2 y}{d x^2}=(a+b) \cdot \frac{d}{d x}\left(e^x \cos x\right)+(b-a) \frac{d}{d x}\left(e^x \sin x\right)\)

⇒ \(\frac{d^2 y}{d x^2}=(a+b) \cdot\left[e^x \cos x-e^x \sin x\right]+(b-a)\left[e^x \sin x+e^x \cos x\right]\)

⇒ \(\frac{d^2 y}{d x^2}=e^x[(a+b)(\cos x-\sin x)+(b-a)(\sin x+\cos x]\)

⇒ \(\frac{d^2 y}{d x^2}=e^x[a \cos x-a \sin x+b \cos x-b \sin x+b \sin x+b \cos x-a \sin x-a \cos x]\)

⇒ \(\frac{d^2 y}{d x^2}=\left[2 e^x(b \cos x-a \sin x)\right]\)

Now, on substituting the values of \(\frac{\mathrm{d}^2 y}{\mathrm{dx}^2}\) and \(\frac{\mathrm{dy}}{\mathrm{dx}}\) in the L.H.S. of the given differential equation, we get:

⇒ \(\frac{d^2 y}{d x^2}+ 2 \frac{d y}{d x}+2 y\)

= \(2 e^x(b \cos x-a \sin x)-2 e^x[(a+b) \cos x+(b-a) \sin x]+2 e^x(a \cos x+b \sin x)\)

= \(e^x\left[\begin{array}{l}
(2 b \cos x-2 a \sin x)-(2 a \cos x+2 b \cos x) \\
-(2 b \sin x-2 a \sin x)+(2 a \cos x+2 b \sin x)
\end{array}\right]\)

= \(e^2[(2 b-2 a-2 b+2 a) \cos x]+e^x[(-2 a-2 b+2 a+2 b) \sin x]=0\)

Hence, the given function is a solution of the corresponding differential equation.

3. \(\mathrm{y}=\mathrm{x} \sin 3 \mathrm{x}\)

Differentiating both sides with respect to x, we get: \(\frac{d y}{d x}=\frac{d}{d x}(x \sin 3 x)=\sin 3 x+x \cdot \cos 3 x \cdot 3 \Rightarrow \frac{d y}{d x}=\sin 3 x+3 x \cos 3 x\)

Again, differentiating both sides with respect to x, we get:

⇒ \(\frac{d^2 y}{d x^2}=\frac{d}{d x}(\sin 3 x)+3 \frac{d}{d x}(x \cos 3 x)\)

⇒ \(\frac{d^2 y}{d x^2}=3 \cos 3 x+3[\cos 3 x+x(-\sin 3 x) \cdot 3] \Rightarrow \frac{d^2 y}{d x^2}=6 \cos 3 x-9 x \sin 3 x\)

Substituting the value of \(\frac{\mathrm{d}^2 y}{\mathrm{dx}^2}\) in the L.H.S. of the given differential equation, we get:

⇒ \(\left.\frac{d^2 y}{d x^2}+9 y-6 \cos 3 x=(6 \cos 3 x-9 x \sin 3 x)+9 x \sin 3 x-6 \cos 3 x\right)=0\)

Hence, the given function is a solution of the corresponding differential equation.

4. \(x^2=2 y^2 \log y\)

Differentiating both sides with respect to x, we get:

2x = \(2 \cdot \frac{d}{d x}\left[y^2 \log y\right]\)

⇒ \(x=\left[2 y \cdot \log y \cdot \frac{d y}{d x}+y^2 \cdot \frac{1}{y} \cdot \frac{d y}{d x}\right] \Rightarrow x=\frac{d y}{d x}(2 y \log y+y)\)

⇒ \(\frac{d y}{d x}=\frac{x}{y(1+2 \log y)}\)

Substituting the value of \(\frac{d y}{d x}\) in the L.H.S. of the given differential equation, we get:

L.H.S. = \(\left(x^2+y^2\right) \frac{d y}{d x}-x y=\left(2 y^2 \log y+y^2\right) \cdot \frac{x}{y(1+2 \log y)}-x y\)

= \(y^2(1+2 \log y) \cdot \frac{x}{y(1+2 \log y)}-x y=x y-x y=0\)

Hence, the given function is a solution of the corresponding differential equation.

Question 3. Prove that \(x^2-y^2=c\left(x^2+y^2\right)^2\) is the general solution of differential equation \(\left(x^3-3 x y^2\right) d x=\left(y^3-3 x^2 y\right) d y\), where c is a parameter.
Solution:

⇒ \(\left(x^3-3 x y^2\right) d x=\left(y^3-3 x^2 y\right) d y \Rightarrow \frac{d y}{d x}=\frac{x^3-3 x y^2}{y^3-3 x^2 y}\)

This is a homogeneous differential equation. To simplify it, we need to make the substitution as:

⇒ \(\frac{d}{d x}(y)=\frac{d}{d x}(v x) \Rightarrow \frac{d y}{d x}=v+x \frac{d v}{d x}\)

Substituting the values of y and \(\frac{d y}{d x}\) in equation (1), we get:

v+x \(\frac{d v}{d x}=\frac{x^3-3 x(v x)^2}{(v x)^3-3 x^2(v x)}\)

⇒ v+x \(\frac{d v}{d x}=\frac{1-3 v^2}{v^3-3 v} \Rightarrow x \frac{d v}{d x}=\frac{1-3 v^2}{v^3-3 v}-v\)

⇒ x \(\frac{d v}{d x}=\frac{1-3 v^2-v\left(v^3-3 v\right)}{v^3-3 v}\)

⇒ \(x \frac{d v}{d x}=\frac{1-v^4}{v^3-3 v} \Rightarrow\left(\frac{v^3-3 v}{1-v^4}\right) d v=\frac{d x}{x}\)

⇒ \(\int\left(\frac{v^3-3 v}{1-v^4}\right) d v=\int \frac{d x}{x}\)….(2)

⇒ \(\int\left(\frac{v^3-3 v}{1-v^4}\right) d v=\log x+\log C^{\prime}\)

Now, \(\int\left(\frac{v^3-3 v}{1-v^4}\right) d v=\int \frac{v^3 d v}{1-v^4}-3 \int \frac{v d v}{1-v^4}\)

⇒ \(\int\left(\frac{v^3-3 v}{1-v^4}\right) d v=I_1-3 I_2\), where \(I_1=\int \frac{v^3 d v}{1-v^4}\) and \(I_2=\int \frac{v d v}{1-v^4}\)….(3)

Let \(1-v^4=t\)

∴ \(\frac{d}{d v}\left(1-v^4\right)=\frac{d t}{d v} \Rightarrow-4 v^3=\frac{d t}{d v} \Rightarrow v^3 d v=-\frac{d t}{4}\)

Now, \(I_1=\int \frac{-d t}{4 t}=-\frac{1}{4} \log t=-\frac{1}{4} \log \left(1-v^4\right)\)

And, \(I_2=\int \frac{v d v}{1-v^4}=\int \frac{v d v}{1-\left(v^2\right)^2}\)

Let \(v^2=p\)

⇒ \(\frac{d}{d v}\left(v^2\right)=\frac{d p}{d v} \Rightarrow 2 v=\frac{d p}{d v} \Rightarrow v d v=\frac{d p}{2}\)

⇒ \(I_2=\frac{1}{2} \int \frac{d p}{1-p^2}=\frac{1}{2 \times 2} \log \left|\frac{1+p}{1-p}\right|=\frac{1}{4} \log \left|\frac{1+v^2}{1-v^2}\right|\)

Substituting the values of \(\mathrm{I}_1\) and \(\mathrm{I}_2\) in equation (3), we get:

⇒ \(\int\left(\frac{v^3-3 v}{1-v^4}\right) d v=-\frac{1}{4} \log \left(1-v^4\right)-\frac{3}{4} \log \left|\frac{1+v^2}{1-v^2}\right|\)

Therefore, equation (2) becomes:

⇒ \(-\frac{1}{4} \log \left(1-v^4\right)-\frac{3}{4} \log \left|\frac{1+v^2}{1-v^2}\right|=\log x+\log C^{\prime}\)

⇒ \(-\frac{1}{4} \log \left[\left(1-v^4\right)\left(\frac{1+v^2}{1-v^2}\right)^3\right]=\log C^{\prime} x\)

⇒ \(\frac{\left(1+v^2\right)^4}{\left(1-v^2\right)^2}=\left(C^{\prime} x\right)^{-1}\)

⇒ \(\frac{\left(1+\frac{y^2}{x^2}\right)^4}{\left(1-\frac{y^2}{x^2}\right)^2}=\frac{1}{\left(C^{\prime}\right)^4 x^4}\)

⇒ \(\frac{\left(x^2+y^2\right)^4}{x^4\left(x^2-y^2\right)^2}=\frac{1}{\left(C^{\prime}\right)^4 x^4}\)

⇒ \(\left(x^2-y^2\right)^2=\left(C^{\prime}\right)^4\left(x^2+y^2\right)^4\)

⇒ \(\left(x^2-y^2\right)=\left(C^{\prime}\right)^2\left(x^2+y^2\right)^2\)

⇒ \(x^2-y^2=c\left(x^2+y^2\right)^2\), where c = \(\left(C^{\prime}\right)^2\)

Hence, the given result is proved.

Question 4. Find the general solution of the differential equation \(\frac{d y}{d x}+\sqrt{\frac{1-y^2}{1-x^2}}=0\)
Solution:

⇒ \(\frac{d y}{d x}+\sqrt{\frac{1-y^2}{1-x^2}}=0 \Rightarrow \frac{d y}{d x}=-\sqrt{\frac{1-y^2}{1-x^2}}\)

⇒ \(\frac{d y}{\sqrt{1-y^2}}=\frac{-d x}{\sqrt{1-x^2}} \Rightarrow \int \frac{d y}{\sqrt{1-y^2}}=\int \frac{-d x}{\sqrt{1-x^2}}\)

⇒ \(\sin ^{-1} y=-\sin ^{-1} x+C \Rightarrow \sin ^{-1} x+\sin ^{-1} y=C\)

Question 5. Show that the general solution of the differential equation \(\frac{d y}{d x}+\frac{y^2+y+1}{x^2+x+1}=0\) is given by (x+y+1)=A(1-x-y-2 x y), where A is a parameter
Solution:

⇒ \(\frac{d y}{d x}+\frac{y^2+y+1}{x^2+x+1}=0 \Rightarrow \frac{d y}{d x}=-\frac{\left(y^2+y+1\right)}{x^2+x+1}\)

⇒ \(\frac{d y}{y^2+y+1}=\frac{-d x}{x^2+x+1} \Rightarrow \frac{d y}{y^2+y+1}+\frac{d x}{x^2+x+1}=0\)

⇒ \(\int \frac{d y}{y^2+y+1}+\int \frac{d x}{x^2+x+1}=C\)

⇒ \(\int \frac{d y}{\left(y+\frac{1}{2}\right)^2+\left(\frac{\sqrt{3}}{2}\right)^2}+\int \frac{d x}{\left(x+\frac{1}{2}\right)^2+\left(\frac{\sqrt{3}}{2}\right)^2}=C\)

⇒ \(\frac{2}{\sqrt{3}} \tan ^{-1}\left[\frac{y+\frac{1}{2}}{\frac{\sqrt{3}}{2}}\right]+\frac{2}{\sqrt{3}} \tan ^{-1}\left[\frac{x+\frac{1}{2}}{\frac{\sqrt{3}}{2}}\right]=C\)

⇒ \(\tan ^{-1}\left[\frac{2 y+1}{\sqrt{3}}\right]+\tan ^{-1}\left[\frac{2 x+1}{\sqrt{3}}\right]=\frac{\sqrt{3 C}}{2}\)

⇒ \(\tan ^{-1}\left[\frac{\frac{2 y+1}{\sqrt{3}}+\frac{2 x+1}{\sqrt{3}}}{1-\frac{(2 y+1)}{\sqrt{3}} \frac{(2 x+1)}{\sqrt{3}}}\right]\)

= \(\frac{\sqrt{3} \mathrm{C}}{2}\)

⇒ \(\tan ^{-1}\left[\frac{\frac{2 x+2 y+2}{\sqrt{3}}}{1-\left(\frac{4 x y+2 x+2 y+1}{3}\right)}\right]=\frac{\sqrt{3} C}{2}\)

⇒ \(\tan ^{-1}\left[\frac{2 \sqrt{3}(x+y+1)}{3-4 x y-2 x-2 y-1}\right]=\frac{\sqrt{3} C}{2}\)

⇒ \(\tan ^{-1}\left[\frac{\sqrt{3}(x+y+1)}{2(1-x-y-2 x y)}\right]=\frac{\sqrt{3} C}{2}\)

⇒ \(\frac{\sqrt{3}(x+y+1)}{2(1-x-y-2 x y)}=\tan \left(\frac{\sqrt{3} C}{2}\right)=B\) (where B = \(\tan \frac{\sqrt{3} C}{2}\))

⇒ x+y+1 = \(\frac{2 B}{\sqrt{3}}(1-x-y-2 x y)\)

⇒ \(\mathrm{x}+\mathrm{y}+\mathrm{I}=\mathrm{A}(1-\mathrm{x}-\mathrm{y}-2 \mathrm{xy})\) (where \(\mathrm{A}=\frac{2 \mathrm{~B}}{\sqrt{3}}\))

Hence, the given result is proved.

Question 6. Find the equation of the curve passing through the point \(\left(0, \frac{\pi}{4}\right)\) whose differential equation is, \(\sin x \cos y \mathrm{dx}+\cos \mathrm{x} \sin \mathrm{y} \mathrm{dy}=0\)
Solution:

The differential equation of the given curve is: sin x cos y d x+cos x sin y d y=0

⇒ \(\frac{\sin x \cos y d x+\cos x \sin y d y}{\cos x \cos y}=0 \Rightarrow \tan x d x+\tan y d y=0\)

Integrating both sides, we get \(\int \tan x d x+\int \tan y d y=\log C\)

⇒ log (sec x)+log (sec y)=log C

⇒ log (sec x sec y)=log C

⇒ sec x sec y=C….(1)

The curve passes through point \(\left(0, \frac{\pi}{4}\right)\)

∴ \(1 \times \sqrt{2}=\mathrm{C} \Rightarrow \mathrm{C}=\sqrt{2}\)

On substituting \(\mathrm{C}=\sqrt{2}\) in equation (1), we get:

⇒ \(\sec \mathrm{x} \cdot \sec \mathrm{y}=\sqrt{2}\)

⇒ \(\sec x \cdot \frac{1}{\cos y}=\sqrt{2} \Rightarrow \cos y=\frac{\sec x}{\sqrt{2}}\)

Hence, the required equation of the curve is \(\cos y=\frac{\sec x}{\sqrt{2}}\)

Question 7. Find the particular solution of the differential equation \(\left(1+e^{2 x}\right) d y+\left(1+y^2\right) e^x d x=0\), given that y=1 when x=0
Solution:

⇒ \(\left(1+e^{2 x}\right) d y+\left(1+y^2\right) e^x d x=0\)

⇒ \(\frac{d y}{1+y^2}+\frac{e^x d x}{1+e^{2 x}}=0 \Rightarrow \int \frac{d y}{1+y^2}+\int \frac{e^x d x}{1+e^{2 x}}=C\)

⇒ \(\tan ^{-1} y+\int \frac{e^x d x}{1+e^{2 x}}=C\)…..(1)

Let \(\mathrm{e}^{\mathrm{x}}=\mathrm{t} \Rightarrow \mathrm{e}^{2 \mathrm{x}}=\mathrm{t}^2\)

⇒ \(\frac{\mathrm{d}}{\mathrm{dx}}\left(\mathrm{e}^{\mathrm{x}}\right)=\frac{\mathrm{dt}}{\mathrm{dx}}\)

⇒ \(\mathrm{e}^{\mathrm{x}}=\frac{\mathrm{dt}}{\mathrm{dx}} \Rightarrow \mathrm{e}^{\mathrm{x}} \mathrm{dx}=\mathrm{dt}\)

Substituting these values in equation (1), we get: \(\tan ^{-1} \mathrm{y}+\int \frac{\mathrm{dt}}{1+\mathrm{t}^2}=\mathrm{C}\)

⇒ \(\tan ^{-1} \mathrm{y}+\tan ^{-1} \mathrm{t}=\mathrm{C} \Rightarrow \tan ^{-1} \mathrm{y}+\tan ^{-1}\left(\mathrm{e}^{\mathrm{x}}\right)=\mathrm{C}\)….(2)

Now, y=1 at x=0.

Therefore, equation (2) becomes: \(\tan ^{-1} 1+\tan ^{-1} 1=\mathrm{C} \Rightarrow \frac{\pi}{4}+\frac{\pi}{4}=\mathrm{C} \Rightarrow \mathrm{C}=\frac{\pi}{2}\)

Substituting \(C=\frac{\pi}{2}\) in equation (2), we get: \(\tan ^{-1} y+\tan ^{-1}\left(e^x\right)=\frac{\pi}{2}\)

This is the required particular solution of the given differential equation.

Question 8. Solve the differential equation \(y e^{\frac{x}{y}} d x=\left(x e^{\frac{x}{y}}+y^2\right) d y(y \neq 0)\)
Solution:

⇒ \(y e^{\frac{x}{y}} d x=\left(x e^{\frac{x}{y}}+y^2\right) d y \Rightarrow y e^{\frac{x}{y}} \frac{d x}{d y}=x e^{\frac{x}{y}}+y^2\)

⇒ \(e^{\frac{x}{y}}\left[y \cdot \frac{d x}{d y}-x\right]=y^2\)…..(1)

Let \(\mathrm{e}^{\frac{\mathrm{x}}{y}}=\mathrm{z}\)

Differentiating it with respect to y, we get:

⇒ \(\frac{d}{d y}\left(e^{\frac{x}{y}}\right)=\frac{d z}{d y} \Rightarrow e^{\frac{x}{y}} \cdot \frac{d}{d y}\left(\frac{x}{y}\right)=\frac{d z}{d y}\)

⇒ \(e^{\frac{x}{y}} \cdot \frac{\left.y \cdot \frac{d x}{d y}-x\right]}{y^2}=\frac{d z}{d y}\)…..(2)

From equation (1) and equation (2), we get : \(\frac{\mathrm{dz}}{\mathrm{dy}}=1 \Rightarrow \mathrm{dz}=\mathrm{dy}\)

Integrating both sides, we get : \(\int d z=\int d y \Rightarrow z=y+C \Rightarrow e^{\frac{x}{y}}=y+C\)

Question 9. Find a particular solution of the differential equation (x-y)(d x+d y)=d x-d y, given that y=-1, when x=0(Hint : put x-y=t)
Solution:

(x-y)(d x+d y)=d x-d y

⇒ (x-y+1) d y=(1-x+y) d x

⇒ \(\frac{d y}{d x}=\frac{1-x+y}{x-y+1} \quad \Rightarrow \frac{d y}{d x}=\frac{1-(x-y)}{1+(x-y)}\)

Let x-y=t

⇒ \(\frac{d}{d x}(x-y)=\frac{d t}{d x} \Rightarrow 1-\frac{d y}{d x}=\frac{d t}{d x}\)….(1)

⇒ \(1-\frac{d t}{d x}=\frac{d y}{d x}\)

Substituting the values of x-y and \(\frac{d y}{d x}\) in equation (1), we get :

⇒ \(1-\frac{d t}{d x}=\frac{1-t}{1+t}\)

⇒ \(\frac{d t}{d x}=1-\left(\frac{1-t}{1+t}\right) \Rightarrow \frac{d t}{d x}=\frac{(1+t)-(1-t)}{1+t} \Rightarrow \frac{d t}{d x}=\frac{2 t}{1+t}\)

⇒ \(\left(\frac{1+t}{t}\right) d t=2 d x \Rightarrow\left(1+\frac{1}{t}\right) d t=2 d x\)

Integrating both sides, we get: \(\int\left(1+\frac{1}{t}\right) d t=2 \int 1 \cdot d x\)

t + \(\log |t|=2 x+C \Rightarrow(x-y)+\log |x-y|=2 x+C\)

⇒ log |x-y|=x+y+C…..(2)

Now, y=-1 at x=0

Therefore, equation (2) becomes: log 1=0-1+C

⇒ C=1

Substituting C=1 in equation (2) we get: log |x-y|=x+y+1

This is the required particular solution of the given differential equation.

Question 10. Solve the differential equation \(\left[\frac{e^{-2 \sqrt{x}}}{\sqrt{x}}-\frac{y}{\sqrt{x}}\right] \frac{d x}{d y}=1(x \neq 0)\)
Solution:

⇒ \(\left[\frac{e^{-2 \sqrt{x}}}{\sqrt{x}}-\frac{y}{\sqrt{x}}\right] \frac{d x}{d y}=1 \Rightarrow \frac{d y}{d x}\)

= \(\frac{e^{-2 \sqrt{x}}}{\sqrt{x}}-\frac{y}{\sqrt{x}} \Rightarrow \frac{d y}{d x}+\frac{y}{\sqrt{x}}=\frac{e^{-2 \sqrt{x}}}{\sqrt{x}}\)

This equation is a linear differential equation of the form \(\frac{d y}{d x}+P y=Q\), where P = \(\frac{1}{\sqrt{x}}\) and Q = \(\frac{e^{-2 \sqrt{x}}}{\sqrt{x}}\)

Now, I.F. = \(\mathrm{e}^{\int \mathrm{Pdx}}=\mathrm{e}^{\int \frac{1}{\sqrt{x}} \mathrm{dx}}=\mathrm{e}^{2 \sqrt{x}}\)

The general solution of the given differential equation is given by,

y(I.F.) = \(\int(\text { Q } \times \text { I.F. }) \mathrm{dx}+\mathrm{C}\)

⇒ \(\mathrm{ye}^{2 \sqrt{x}}=\int\left(\frac{\mathrm{e}^{-2 \sqrt{x}}}{\sqrt{\mathrm{x}}} \times \mathrm{e}^{2 \sqrt{\mathrm{x}}}\right) \mathrm{dx}+\mathrm{C}\)

⇒ \(\mathrm{ye}^{2 \sqrt{\mathrm{x}}}=\int \frac{1}{\sqrt{\mathrm{x}}} \mathrm{dx}+\mathrm{C} \Rightarrow \mathrm{ye}^{2 \sqrt{x}}=2 \sqrt{\mathrm{x}}+\mathrm{C}\)

Question 11. Find a particular solution of the differential equation \(\frac{d y}{d x}+y \cot x=4 x \mathrm{cosec} x(x \neq 0)\), given that y=0 where \(x=\frac{\pi}{2}\).
Solution:

The given differential equation is : \(\frac{d y}{d x}+y \cot x=4 x \mathrm{cosec} x\)

This equation is a linear differential equation of the form \(\frac{d y}{d x}+P y=0\), where P = cot x and Q = 4x cosec x

Now, I.F, \(=e^{\int P d x}=e^{\int \cot x d x}=e^{\log \sin x x}=\sin x\)

The general solution of the given differential equation is given by, \(y(\mathrm{I} . \mathrm{F} .)=\int(\mathrm{Q} \times \mathrm{I} . \mathrm{F} .) \mathrm{dx}+\mathrm{C}\)

⇒ \(\mathrm{y} \sin \mathrm{x}=\int(4 \mathrm{x} \mathrm{cosec} \mathrm{x} \cdot \sin \mathrm{x}) \mathrm{dx}+\mathrm{C}\)

⇒ \(\mathrm{y} \sin \mathrm{x}=4 \int \mathrm{xdx}+\mathrm{C}\)

⇒ \(\mathrm{y} \sin \mathrm{x}=4 \cdot \frac{\mathrm{x}^2}{2}+\mathrm{C} \Rightarrow \mathrm{y} \sin \mathrm{x}=2 \mathrm{x}^2+\mathrm{C}\)…..(1)

Now, y=0 at x = \(\frac{\pi}{2}\)

Therefore, equation (1) becomes : 0 = \(2 \times \frac{\pi^2}{4}+\mathrm{C} \Rightarrow \mathrm{C}=-\frac{\pi^2}{2}\)

Substituting C = \(-\frac{\pi^2}{2}\) in equation (1), we get: \(y \sin x=2 x^2-\frac{\pi^2}{2}\)

This is the required particular solution of the given differential equation.

Question 12. Find a particular solution of the differential equation \((x+1) \frac{d y}{d x}=2 e^{-y}-1\), given that y=0 when x=0
Solution:

(x+1) \(\frac{d y}{d x}=2 e^{-y}-1 \Rightarrow \frac{d y}{2 e^{-y}-1}=\frac{d x}{x+1} \Rightarrow \frac{e^y d y}{2-e^y}=\frac{d x}{x+1}\)

Integrating both sides, we get : \(\int \frac{e^y d y}{2-e^y}=\int \frac{d x}{x+1}\)

⇒ \(\int \frac{\mathrm{e}^y \mathrm{dy}}{2-\mathrm{e}^y}=\log |\mathrm{x}+1|+\log \mathrm{C}\)…..(1)

Let \(2-e^{y}=\mathrm{t} \Rightarrow e^y d y=-d t\)

Substituting this value in equation (1), we get : \(-\int \frac{\mathrm{dt}}{\mathrm{t}}=\log |\mathrm{x}+1|+\log \mathrm{C}\)

⇒ \(-\log |\mathrm{t}|=\log |\mathrm{C}(\mathrm{x}+1)| \Rightarrow-\log \left|2-\mathrm{e}^{\mathrm{y}}\right|\)

= \(\log |\mathrm{C}(\mathrm{x}+1)| \Rightarrow \frac{1}{2-\mathrm{e}^{\mathrm{y}}}=\mathrm{C}(\mathrm{x}+1)\)

⇒ \(2-\mathrm{e}^{\mathrm{y}}=\frac{1}{\mathrm{C}(\mathrm{x}+1)}\)……(2)

Now, at x=0 and y=0, equation (2) becomes : 2-1 = \(\frac{1}{C}\)

C=1

Substituting C=1 in equation (2), we get : \(2-e^y=\frac{1}{x+1}\)

⇒ \(e^y=2-\frac{1}{x+1} \Rightarrow e^y=\frac{2 x+2-1}{x+1} \Rightarrow e^y=\frac{2 x+1}{x+1}\)

⇒ \(y=\log \left|\frac{2 x+1}{x+1}\right|,(x \neq-1)\)

This is the required particular solution of the given differential equation.

Choose The Correct Answer

Question 13. The general solution of the differential equation \(\frac{y d x-x d y}{y}=0\) is

  1. xy=C
  2. \(x=C y^2\)
  3. y=Cx
  4. \(\mathrm{y}=\mathrm{Cx}^2\)

Solution: 3. y=Cx

The given differential equation is: \(\frac{y \mathrm{dx}-\mathrm{x} d \mathrm{y}}{\mathrm{y}}=0\)

⇒ \(\frac{1}{\mathrm{x}} \mathrm{dx}-\frac{1}{\mathrm{y}} \mathrm{dy}=0\)

Integrating both sides, we get : \(\int \frac{1}{x} d x-\int \frac{1}{y} d y=0 \Rightarrow \log |x|-\log |y|=\log k \Rightarrow \log \left|\frac{x}{y}\right|=\log k\)

⇒ \(\frac{x}{y}=k \Rightarrow y=\frac{1}{k} x \Rightarrow y=C x\) where c = \(\frac{1}{k}\)

Hence, the correct answer is (3).

Question 14. The general solution of a differential equation of the type \(\frac{d x}{d y}+P_1 x=Q_1\) is

  1. \(y \mathrm{e}^{\int P_1 d y}=\int\left(Q_1 e^{\int P_1 d y}\right) d y+C\)
  2. \(y \cdot e^{\int P_1 d x}=\int\left(Q_1 e^{\int P_1{d x}}\right) d y+C\)
  3. \(x e^{\int P_1 d y}=\int\left(Q_1 e^{\int P_1 d y}\right) d y+C\)
  4. \(x e^{\int P_1 d x}=\int\left(Q_1 e^{\int P_1 d x}\right) d y+C\)

Solution: 3. \(x e^{\int P_1 d y}=\int\left(Q_1 e^{\int P_1 d y}\right) d y+C\)

The integrating factor of the given differential equation \(\frac{d x}{d y}+P_1 x=Q_1\) is \(e^{\int P_1 d y}\).

The general solution of the differential equation is given by,

x(I .F.) = \(\int\left(Q_1 \times \text { I.F. }\right) d y+C \Rightarrow x \cdot e^{\int P_1 d y}=\int\left(Q_1 e^{\int P_1 d y}\right) d y+C\)

Hence, the correct answer is (3).

Question 15. The general solution of the differential equation \(e^x d y+\left(y e^x+2 x\right) d x=0\) is

  1. \(x e^y+x^2=C\)
  2. \(x e^y+y^2=C\)
  3. \(y \mathrm{e}^{\mathrm{x}}+\mathrm{x}^2=\mathrm{C}\)
  4. \(\mathrm{ye}^y+\mathrm{x}^2=\mathrm{C}\)

Solution: 3. \(y \mathrm{e}^{\mathrm{x}}+\mathrm{x}^2=\mathrm{C}\)

The given differential equation is: \(e^x d y+\left(y e^x+2 x\right) d x=0\)

⇒ \(e^x \frac{d y}{d x}+y e^x+2 x=0 \quad \Rightarrow \quad \frac{d y}{d x}+y=-2 x e^{-x}\)

This is a linear differential equation of the form

⇒ \(\frac{\mathrm{dy}}{\mathrm{dx}}+\mathrm{Py}=\mathrm{Q}\) where P =1 and Q=-2 \(\mathrm{x} \mathrm{e}^{-\mathrm{t}}\)

Now, I.F. = \(\mathrm{e}^{\int P d x}=\mathrm{e}^{\int d x}=\mathrm{e}^\pi\)

The general solution of the given differential equation is given by, \(y(\text { I.F. })=\int(\mathrm{Q} \times \text { I.F. }) \mathrm{dx}+C\)

⇒ \(\mathrm{ye}^{\mathrm{x}}=\int\left(-2 \mathrm{xe}^{-\mathrm{x}} \cdot \mathrm{e}^{\mathrm{x}}\right) \mathrm{dx}+\mathrm{C}\)

⇒ \(\mathrm{ye}^{\mathrm{x}}=-\int 2 \mathrm{xdx}+\mathrm{C}\)

⇒ \(\mathrm{ye}^{\mathrm{x}}=-\mathrm{x}^2+\mathrm{C}\)

⇒ \(\mathrm{ye}^{\mathrm{x}}+\mathrm{x}^2=\mathrm{C}\)

Hence, the correct answer is 3.

 

 

 

 

Integrals Class 12 Maths Important Questions Chapter 7

Integrals Exercise 7.1

Find An antiderivative (Or Integral) Of the following function by the method of inspection.

Question 1. sin2x
Solution:

The anti-derivate of sin2x is a function of x whose derivative is sin2x. It is known that,

∵ \(\frac{d}{d x}(\cos 2 x)=-2 \sin 2 x \Rightarrow-\frac{1}{2} \frac{d}{d x}(\cos 2 x)=\sin 2 x \Rightarrow \frac{d}{d x}\left(-\frac{1}{2} \cos 2 x\right)=\sin 2 x\)

Therefore, the anti-derivates of sin2x is –\(\frac{1}{2}\) cos2x

Question 2. cos 3x
Solution:

The anti-derviative of cos 3x is a function of x whose derivative is cos 3x.

∵ \(\frac{d}{d x}(\sin 3 x)=3 \cos 3 x \Rightarrow \frac{1}{3} \frac{d}{d x}(\sin 3 x)=\cos 3 x \Rightarrow \frac{d}{d x}\left(\frac{1}{3} \sin 3 x\right)=\cos 3 x\)

Therefore, the anti-derivates of sos 3x is –\(\frac{1}{3}\) sin 3x.

Question 3. e2x
Solution:

The anti-derviative of cos 3x is a function of x whose derivative is cos 3x.

∵ \(\frac{\mathrm{d}}{\mathrm{dx}}\left(\mathrm{e}^{2 \mathrm{x}}\right)=2 \mathrm{e}^{2 \mathrm{x}} \Rightarrow \frac{1}{2} \frac{\mathrm{d}}{\mathrm{dx}}\left(\mathrm{e}^{2 \mathrm{x}}\right)=\mathrm{e}^{2 \mathrm{x}} \Rightarrow \frac{\mathrm{d}}{\mathrm{dx}}\left(\frac{1}{2} \mathrm{e}^{2 \mathrm{x}}\right)=\mathrm{e}^{2 \mathrm{x}}\)

Therefore, the anti-derivates of e2x is –\(\frac{1}{2}\) e2x.

Question 4. (ax + b)²
Solution:

The anti-derviative of (ax+b)² is a function of x whose derivative is (ax+b)².

It is known that,

∵ \(\frac{d}{d x}(a x+b)^3=3 a(a x+b)^2 \Rightarrow \frac{1}{3 a} \frac{d}{d x}(a x+b)^3\)

= \((a x+b)^2 \Rightarrow \frac{d}{d x}\left(\frac{1}{3 a}(a x+b)^3\right)=(a x+b)^2\)

Therefore, the anti-derivates of (ax+b) is –\(\frac{1}{3a}\) (ax+b).

Question 5. sin 2x – 4e3x
Solution:

The anti-derivative of (sin 2x – 4e3x) is a function of x whose derivative is (sin 2x – 4e3x).

∵ \(\frac{d}{d x}(\cos 2 x)=-2 \sin 2 x \Rightarrow-\frac{1}{2} \frac{d}{d x}(\cos 2 x)=\sin 2 x \Rightarrow \frac{d}{d x}\left(-\frac{1}{2} \cos 2 x\right)=\sin 2 x\)

Similarly, \(\frac{d}{d x}\left(\frac{4}{3} e^{3 x}\right)=4 e^{3 x}\)

∵ \(\frac{d}{d x}\left(-\frac{1}{2} \cos 2 x\right)-\frac{d}{d x}\left(\frac{4}{3} e^{3 x}\right)=\sin 2 x-4 e^{3 x} \Rightarrow \frac{d}{d x}\left(-\frac{1}{2} \cos 2 x-\frac{4}{3} e^{3 x}\right)=\sin 2 x-4 e^{3 x}\)

Therefore, the antiderivative of \(\left(\sin 2 x-4 e^{3 x}\right)\) is \(\left(-\frac{1}{2} \cos 2 x-\frac{4}{3} e^{3 x}\right)\)

Find The Following Integrals

Question 6. \(\int\left(4 e^{3 x}+1\right) d x\)
Solution:

Let \(\mathrm{I}=\int\left(4 \mathrm{e}^{3 \mathrm{x}}+1\right) \mathrm{dx}\)

= \(4 \int e^{3 x} d x+\int 1 d x=4\left(\frac{e^{3 x}}{3}\right)+x+C=\frac{4}{3} e^{3 x}+x+C\)

Question 7. \(\int x^2\left(1-\frac{1}{x^2}\right) d x\)
Solution:

⇒ \(\int x^2\left(1-\frac{1}{x^2}\right) d x=\int\left(x^2-1\right) d x=\int x^2 d x-\int 1 d x=\frac{x^3}{3}-x+C\)

Read and Learn More Class 12 Maths Chapter Wise with Solutions

Question 8. \(\int\left(a x^2+b x+c\right) d x\)
Solution:

Let \(I=\int\left(a x^2+b x+c\right) d x=a \int x^2 d x+b \int x d x+c \int 1 . d x\)

= \(a\left(\frac{x^3}{3}\right)+b\left(\frac{x^2}{2}\right)+c x+C=\frac{a x^3}{3}+\frac{b x^2}{2}+c x+C\)

Question 9. \(\int\left(2 x^2+e^x\right) d x\)
Solution:

Let \(\mathrm{I}=\int\left(2 \mathrm{x}^2+\mathrm{e}^{\mathrm{x}}\right) \mathrm{dx}=2 \int \mathrm{x}^2 \mathrm{dx}+\int \mathrm{e}^{\mathrm{x}} \mathrm{dx}=2\left(\frac{\mathrm{x}^3}{3}\right)+\mathrm{e}^{\mathrm{x}}+\mathrm{C}=\frac{2}{3} \mathrm{x}^3+\mathrm{e}^{\mathrm{x}}+\mathrm{C}\)

Question 10. \(\int\left(\sqrt{\mathrm{x}}-\frac{1}{\sqrt{\mathrm{x}}}\right)^2 \mathrm{dx}\)
Solution:

Let \(I=\int\left(\sqrt{x}-\frac{1}{\sqrt{x}}\right)^2 d x=\int\left(x+\frac{1}{x}-2\right) d x\)

= \(\int x d x+\int \frac{1}{x} d x-2 \int 1, d x=\frac{x^2}{2}+\log |x|-2 x+C\)

Question 11. \(\int \frac{x^3+5 x^2-4}{x^2} d x\)
Solution:

Let \(I=\int \frac{x^3+5 x^2-4}{x^2} d x=\int\left(x+5-4 x^{-2}\right) d x=\int x d x+5 \int 1 . d x-4 \int x^{-2} d x\)

= \(\frac{x^2}{2}+5 x-4\left(\frac{x^{-1}}{-1}\right)+C=\frac{x^2}{2}+5 x+\frac{4}{x}+C\)

Question 12. \(\int \frac{x^3+3 x+4}{\sqrt{x}} d x\)
Solution:

Let \(\mathrm{I}=\int \frac{\mathrm{x}^3+3 \mathrm{x}+4}{\sqrt{\mathrm{x}}} \mathrm{dx}=\int\left(\mathrm{x}^{\frac{5}{2}}+3 \mathrm{x}^{\frac{1}{2}}+4 \mathrm{x}^{\frac{-1}{2}}\right) \mathrm{dx}\)

(because \(\int \mathrm{x}^{\mathrm{n}} \mathrm{dx}=\frac{\mathrm{x}^{\mathrm{n}+1}}{\mathrm{n}+1}+\mathrm{C}\))

= \(\frac{x^{\left(\frac{7}{2}\right)}}{\frac{7}{2}}+\frac{3\left(x^{\frac{3}{2}}\right)}{\frac{3}{2}}+\frac{4\left(x^{\frac{1}{2}}\right)}{\frac{1}{2}}+C=\frac{2}{7} x^{\frac{7}{2}}+2 x^{\frac{3}{2}}+8 x^{\frac{1}{2}}+C=\frac{2}{7} x^{\frac{7}{2}}+2 x^{\frac{3}{2}}+8 \sqrt{x}+C\)

Question 13. \(\int \frac{x^3-x^2+x-1}{x-1} d x\)
Solution:

Let \(\mathrm{I}=\int \frac{\mathrm{x}^3-\mathrm{x}^2+\mathrm{x}-1}{\mathrm{x}-1} \mathrm{dx}\)

On dividing, we obtain

I = \(\int \frac{x^2(x-1)+1(x-1)}{x-1} d x=\int \frac{(x-1)\left(x^2+1\right)}{(x-1)} d x=\int x^2 d x+\int 1 \cdot d x=\frac{x^3}{3}+x+C\)

Question 14. \(\int(1-x) \sqrt{x} d x\)
Solution:

Let \(I=\int(1-x) \sqrt{x} d x=\int\left(\sqrt{x}-x^{\frac{3}{2}}\right) d x=\int x^{\frac{1}{2}} d x-\int x^{\frac{3}{2}} d x\)

=\(\frac{x^{\frac{3}{2}}}{3 / 2}-\frac{x^{\frac{5}{2}}}{5 / 2}+C=\frac{2}{3} x^{3 / 2}-\frac{2}{5} x^{5 / 2}+C\)

Question 15. \(\int \sqrt{\mathrm{x}}\left(3 \mathrm{x}^2+2 \mathrm{x}+3\right) \mathrm{dx}\)
Solution:

Let \(\mathrm{I}=\int \sqrt{\mathrm{x}}\left(3 \mathrm{x}^2+2 \mathrm{x}+3\right) \mathrm{dx}=\int\left(3 \mathrm{x}^{\frac{5}{2}}+2 \mathrm{x}^{\frac{3}{2}}+3 \mathrm{x}^{\frac{1}{2}}\right) \mathrm{dx}\)

= \(3 \int \mathrm{x}^{-\frac{5}{2}} \mathrm{dx}+2 \int \mathrm{x}^{\frac{3}{2}} \mathrm{dx}+3 \int \mathrm{x}^{\frac{1}{2}} d x\)

= \(3\left(\frac{x^{\frac{7}{2}}}{\frac{7}{2}}\right)+2\left(\frac{x^{\frac{5}{2}}}{\frac{5}{2}}\right)+3\left(\frac{x^{\frac{3}{2}}}{\frac{3}{2}}\right)+C=\frac{6}{7} x^{\frac{7}{2}}+\frac{4}{5} x^{\frac{5}{2}}+2 x^{\frac{3}{2}}+C\)

Question 16. \(\int\left(2 x-3 \cos x+e^x\right) d x\)
Solution:

Let \(I=\int\left(2 x-3 \cos x+e^x\right) d x=2 \int x d x-3 \int \cos x d x+\int e^x d x\)

= \(\frac{2 \mathrm{x}^2}{2}-3(\sin \mathrm{x})+\mathrm{e}^{\mathrm{x}}+\mathrm{C}=\mathrm{x}^2-3 \sin \mathrm{x}+\mathrm{e}^{\mathrm{x}}+\mathrm{C}\)

Question 17. \(\int\left(2 x^2-3 \sin x+5 \sqrt{x}\right) d x\)
Solution:

Let \(I=\int\left(2 x^2-3 \sin x+5 \sqrt{x}\right) d x=2 \int x^2 d x-3 \int \sin x d x+5 \int x^{\frac{1}{2}} d x\)

= \(\frac{2 x^3}{3}-3(-\cos x)+5\left(\frac{x^{\frac{3}{2}}}{\frac{3}{2}}\right)+C=\frac{2}{3} x^3+3 \cos x+\frac{10}{3} x^{\frac{3}{2}}+C\)

Question 18. \(\int \sec x(\sec x+\tan x) d x\)
Solution:

Let \(I=\int \sec x(\sec x+\tan x) d x=\int\left(\sec ^2 x+\sec x \tan x\right) d x\)

= \(\tan \mathrm{x}+\sec \mathrm{x}+\mathrm{C}\)

Question 19. \(\int \frac{\sec ^2 x}{\mathrm{cosec}^2 x} d x\)
Solution:

Let \(I=\int \frac{\sec ^2 x}{\mathrm{cosec}^2 x} d x=\int \frac{\frac{1}{\cos ^2 x}}{\frac{1}{\sin ^2 x}} d x=\int \frac{\sin ^2 x}{\cos ^2 x} d x\)

= \(\int \tan ^2 x d x=\int\left(\sec ^2 x-1\right) d x=\int \sec ^2 x d x-\int 1 d x=\tan x-x+C\)

Question 20. \(\int \frac{2-3 \sin x}{\cos ^2 x} d x\)
Solution:

Let I = \(\int \frac{2-3 \sin x}{\cos ^2 x} d x=\int\left(\frac{2}{\cos ^2 x}-\frac{3 \sin x}{\cos ^2 x}\right) d x\)

= \(\int 2 \sec ^2 x d x-3 \int \tan x \sec x d x=2 \tan x-3 \sec x+C\)

Choose The Correct Answer In The Following

Question 21. The anti derivative of \(\left(\sqrt{\mathrm{x}}+\frac{1}{\sqrt{\mathrm{x}}}\right)\) equals?

  1. \(\frac{1}{3} \mathrm{x}^{\frac{1}{3}}+2 \mathrm{x}^{\frac{1}{2}}+C\)
  2. \(\frac{2}{3} \mathrm{x}^{\frac{2}{3}}+\frac{1}{2} \mathrm{x}^2+\mathrm{C}\)
  3. \(\frac{2}{3} \mathrm{x}^{\frac{3}{2}}+2 \mathrm{x}^{\frac{1}{2}}+\mathrm{C}\)
  4. \(\frac{3}{2} \mathrm{x}^{\frac{3}{2}}+\frac{1}{2} \mathrm{x}^{\frac{1}{2}}+\mathrm{C}\)

Solution: 3. \(\frac{2}{3} \mathrm{x}^{\frac{3}{2}}+2 \mathrm{x}^{\frac{1}{2}}+\mathrm{C}\)

Let \(\mathrm{I}=\int\left(\sqrt{\mathrm{x}}+\frac{1}{\sqrt{\mathrm{x}}}\right) \mathrm{dx}=\int \mathrm{x}^{\frac{1}{2}} \mathrm{dx}+\int \mathrm{x}^{-\frac{1}{2}} \mathrm{dx}=\frac{\mathrm{x}^{\frac{3}{2}}}{3 / 2}+\frac{\mathrm{x}^{\frac{1}{2}}}{1 / 2}+\mathrm{C}=\frac{2}{3} \mathrm{x}^{\frac{3}{2}}+2 \mathrm{x}^{\frac{1}{2}}+\mathrm{C}\)

Hence, the correct Answer is 3.

Question 22. If \(\frac{\mathrm{d}}{\mathrm{dx}}(f(\mathrm{x}))=4 \mathrm{x}^3-\frac{3}{\mathrm{x}^4}\) such that f(2)=0, then f(x) is?

  1. \(x^4+\frac{1}{x^3}-\frac{129}{8}\)
  2. \(x^3+\frac{1}{x^4}+\frac{129}{8}\)
  3. \(\mathrm{x}^4+\frac{1}{\mathrm{x}^3}+\frac{129}{8}\)
  4. \(\mathrm{x}^3+\frac{1}{\mathrm{x}^4}-\frac{129}{8}\)

Solution: 1. \(x^4+\frac{1}{x^3}-\frac{129}{8}\)

It is given that, \(\frac{\mathrm{d}}{\mathrm{dx}}(f(\mathrm{x}))=4 \mathrm{x}^3-\frac{3}{\mathrm{x}^4}\)

∴ Integrating both sides with respect to x

∴ \(f(\mathrm{x})=\int\left(4 \mathrm{x}^3-\frac{3}{\mathrm{x}^4}\right) \mathrm{dx} \Rightarrow f(\mathrm{x})=4 \int \mathrm{x}^3 \mathrm{dx}-3 \int\left(\mathrm{x}^{-4}\right) \mathrm{dx}\)

⇒ \(f(\mathrm{x})=4\left(\frac{\mathrm{x}^4}{4}\right)-3\left(\frac{\mathrm{x}^{-3}}{-3}\right)+\mathrm{C}\)

∴ \(f(\mathrm{x})=\mathrm{x}^4+\frac{1}{\mathrm{x}^3}+\mathrm{C}\)

Also, f(2)=0

∴ \(f(2)=(2)^4+\frac{1}{(2)^3}+\mathrm{C}=0 \Rightarrow 16+\frac{1}{8}+\mathrm{C}=0 \Rightarrow \mathrm{C}=-\left(16+\frac{1}{8}\right) \Rightarrow \mathrm{C}=-\frac{129}{8}\)

Put the value of C in equation (1)

f(x) = \(x^4+\frac{1}{x^3}-\frac{129}{8} \text {. }\)

Hence, the correct answer is (1).

CBSE Class 12 Maths Chapter 7 Integrals Important Question And Answers

Integrals Exercise 7.2

IntegrateThe Functions

Question 1. \(\int \frac{2 \mathrm{x}}{1+\mathrm{x}^2} \mathrm{dx}\)
Solution:

Let \(\mathrm{I}=\int \frac{2 \mathrm{x}}{1+\mathrm{x}^2} \mathrm{dx}\)

Put \(1+\mathrm{x}^2=\mathrm{t} \Rightarrow 2 \mathrm{xdx}=\mathrm{dt}\)

⇒ \(\mathrm{I}=\int_{\mathrm{t}}^1 \frac{1}{\mathrm{dt}}=\log |\mathrm{t}|+\mathrm{C}=\log \left|1+\mathrm{x}^2\right|+\mathrm{C}=\log \left(1+\mathrm{x}^2\right)+\mathrm{C}\)

Question 2. \(\int \frac{(\log |\mathrm{x}|)^2}{\mathrm{x}} \mathrm{dx}\)
Solution:

Let \(\mathrm{I}=\int \frac{(\log |\mathrm{x}|)^2}{\mathrm{x}} \mathrm{dx}\)

Put \(\log |x|=t \Rightarrow \frac{1}{x} d x=d t \Rightarrow I=\int t^2 d t=\frac{t^3}{3}+C=\frac{(\log |x|)^3}{3}+C\)

Question 3. \(\int \frac{1}{x+x \log x} d x\)
Solution:

Let \(I=\int \frac{1}{x+x \log x} d x=\int \frac{1}{x(1+\log x)} d x\)

Put \(1+\log \mathrm{x}=\mathrm{t} \Rightarrow \frac{1}{\mathrm{x}} \mathrm{dx}=\mathrm{dt}\)

Question 4. \(\int \sin x \cdot \sin (\cos x) d x\)
Solution:

Let \(\mathrm{I}=\int \sin \mathrm{x}, \sin (\cos \mathrm{x}) \mathrm{dx}\)

Put \(\cos \mathrm{x}=\mathrm{t} \Rightarrow-\sin \mathrm{x} \mathrm{dx}=\mathrm{dt}\)

⇒ \(\mathrm{I}=-\int \sin \mathrm{tdt}=-[-\cos \mathrm{t}]+\mathrm{C}=\cos (\cos \mathrm{x})+\mathrm{C}\)

Question 5.\(\int \sin (a x+b) \cos (a x+b) d x\)
Solution:

Let \(\mathrm{I}=\int \sin (a \mathrm{ax}+\mathrm{b}) \cos (a \mathrm{ax}+\mathrm{b}) \mathrm{dx}=\frac{1}{2} \int \sin 2(a \mathrm{a}+\mathrm{b}) \mathrm{dx}\)

Put \(2(a x+b)=t \Rightarrow d x=\frac{d t}{2 a}\)

⇒ I = \(\frac{1}{2} \int \frac{\sin t d t}{2 a}=\frac{1}{4 a}[-\cos t]+C=\frac{-1}{4 a} \cos 2(a x+b)+C\)

Question 6. \(\int \sqrt{a x+b} d x\)
Solution:

Let \(I=\int \sqrt{a x+b} d x\)

Put \(a x+b=t^2 \Rightarrow a d x=2 t d t\)

∴ I = \(\int t \cdot \frac{2 t}{a} d t=\frac{2}{a} \int t^2 d t=\frac{2}{a} \cdot \frac{t^3}{3}+C=\frac{2}{3 a}(a x+b)^{3 / 2}+C\)

Question 7. \(\int x \sqrt{x+2} d x\)
Solution:

Let \(\mathrm{I}=\int \mathrm{x} \sqrt{\mathrm{x}+2} \mathrm{dx}\)

Put \(x+2=t^2 \Rightarrow d x=2 t d t\)

∴ \(\mathrm{I}=\int\left(\mathrm{t}^2-2\right) \cdot \mathrm{t} \cdot 2 \mathrm{tdt}=\int\left(2 \mathrm{t}^4-4 \mathrm{t}^2\right) \mathrm{dt}\)

= \(\frac{2}{5} \mathrm{t}^5-\frac{4}{3} \mathrm{t}^3+\mathrm{C}=\frac{2}{5}(\mathrm{x}+2)^{5 / 2}-\frac{4}{3}(\mathrm{x}+2)^{3 / 2}+\mathrm{C}\)

Question 8. \(\int x \sqrt{1+2 x^2} d x\)
Solution:

Let \(\mathrm{I}=\int \mathrm{x} \sqrt{1+2 \mathrm{x}^2} \mathrm{dx}\)

Put \(1+2 x^2=t^2 \Rightarrow 4 x d x=2 t d t \Rightarrow x d x=\frac{t}{2} d t\)

∴ I = \(\int t \cdot \frac{t}{2} d t=\frac{1}{2} \int t^2 d t=\frac{1}{6} t^3+C=\frac{1}{6}\left(1+2 x^2\right)^{3 / 2}+C\)

Question 9. \(\int(4 x+2) \sqrt{x^2+x+1} d x\)
Solution:

Let \(\mathrm{I}=\int(4 \mathrm{x}+2) \sqrt{\mathrm{x}^2+\mathrm{x}+1} \mathrm{dx}\)

Put \(x^2+x+1=t^2 \Rightarrow(2 x+1) d x=2 t d t\)

⇒ \(\mathrm{I}=2 \int \mathrm{t} \cdot 2 \mathrm{tdt}=4 \int \mathrm{t}^2 \mathrm{dt}=\frac{4}{3} \mathrm{t}^3+\mathrm{C}=\frac{4}{3}\left(\mathrm{x}^2+\mathrm{x}+1\right)^{3 / 2}+\mathrm{C}\)

Question 10. \(\int \frac{1}{x-\sqrt{x}} d x\)
Solution:

Let \(I=\int \frac{1}{x-\sqrt{x}} d x=\int \frac{1}{\sqrt{x}(\sqrt{x}-1)} d x\)

Put \((\sqrt{\mathrm{x}}-1)=\mathrm{t} \Rightarrow \frac{1}{2 \sqrt{\mathrm{x}}} \mathrm{dx}=\mathrm{dt} \Rightarrow \frac{1}{\sqrt{\mathrm{x}}} \mathrm{dx}=2 \mathrm{dt}\)

∴ I = \(\int \frac{1}{t} \cdot 2 d t=2 \log |t|+C=2 \log |\sqrt{x}-1|+C\)

Question 11. \(\int \frac{\mathrm{x}}{\sqrt{\mathrm{x}+4}} \cdot \mathrm{dx}, \mathrm{x}>0\)
Solution:

Let \(\mathrm{I}=\int \frac{\mathrm{x}}{\sqrt{\mathrm{x}+4}} \cdot \mathrm{dx}, \mathrm{x}>0\)

Put \(x+4=t^2 \Rightarrow x=t^2-4 \Rightarrow d x=2 t d t\)

∴ I = \(\int \frac{\left(t^2-4\right)}{t} \cdot 2 t d t=2 \int\left(t^2-4\right) d t=2\left[\frac{t^3}{3}-4 t\right]+C\)

= \(\frac{2}{3} t\left[t^2-12\right]+C=\frac{2}{3} \sqrt{x+4}(x-8)+C\)

Question 12. \(\int\left(x^3-1\right)^{\frac{1}{3}} x^5 d x\)
Solution:

Let \(I=\int\left(x^3-1\right)^{\frac{1}{3}} x^5 d x\)

I = \(\int\left(x^3-1\right)^{1 / 3} \cdot x^3 \cdot x^2 d x\)

Put \(x^3-1=t^3 \Rightarrow x^3=t^3+1 \Rightarrow x^2 d x=t^2 d t\)

∴ I = \(\int t \cdot\left(t^3+1\right) \cdot t^2 d t=\int\left(t^6+t^3\right) d t=\frac{t^7}{7}+\frac{t^4}{4}+C=\frac{1}{7}\left(x^3-1\right)^{7 / 3}+\frac{1}{4}\left(x^3-1\right)^{4 / 3}+C\)

Question 13. \(\int \frac{x^2}{\left(2+3 x^3\right)^3} d x\)
Solution:

Let \(I=\int \frac{x^2}{\left(2+3 x^3\right)^3} d x\)

Put \(2+3 x^3=t \Rightarrow 9 x^2 d x=d t\)

∴ \(\mathrm{I}=\int \frac{1}{\mathrm{t}^3}, \frac{\mathrm{dt}}{9}=\frac{1}{9} \int \frac{1}{\mathrm{t}^3} \mathrm{dt}=\frac{1}{9}\left[\frac{\mathrm{t}^{-2}}{-2}\right]+\mathrm{C}=-\frac{1}{18}\left[\frac{1}{\mathrm{t}^2}\right]+\mathrm{C}=-\frac{1}{18\left(2+3 \mathrm{x}^3\right)^2}+\mathrm{C}\)

Question 14. \(\int \frac{1}{x(\log x)^m} d x, x>0, m \neq 1\)
Solution:

Let \(\mathrm{I}=\int \frac{1}{\mathrm{x}(\log \mathrm{x})^{\mathrm{m}}} \mathrm{dx}\)

Put \(\log \mathrm{x}=\mathrm{t} \Rightarrow \frac{1}{\mathrm{x}} \mathrm{dx}=\mathrm{dt}\)

⇒ \(\mathrm{I}=\int \frac{\mathrm{dt}}{(\mathrm{t})^{\mathrm{m}}}=\int \mathrm{t}^{-\mathrm{m}} \mathrm{dt}=\left(\frac{\mathrm{t}^{1-\mathrm{m}}}{1-\mathrm{m}}\right)+\mathrm{C}=\frac{(\log \mathrm{x})^{1-\mathrm{m}}}{(1-\mathrm{m})}+\mathrm{C}\)

Question 15. \(\int \frac{x}{9-4 x^2} d x\)
Solution:

Let \(I=\int \frac{x}{9-4 x^2} d x\)

Put \(9-4 \mathrm{x}^2=\mathrm{t} \Rightarrow-8 \mathrm{xdx}=\mathrm{dt}\)

∴ I = \(\frac{-1}{8} \int_t^1 \frac{1}{t}=\frac{-1}{8} \log |t|+C=\frac{-1}{8} \log \left|9-4 x^2\right|+C\)

Question 16. \(\int \mathrm{e}^{2 \mathrm{x}+3} \mathrm{dx}\)
Solution:

Let \(I=\int e^{2 x+3} d x\)

Put \(2 \mathrm{x}+3=\mathrm{t} \Rightarrow \mathrm{dx}=\frac{\mathrm{dt}}{2}\)

∴ \(\mathrm{I}=\frac{1}{2} \int \mathrm{e}^{\mathrm{t}} \mathrm{dt}=\frac{1}{2}\left(\mathrm{e}^{\mathrm{t}}\right)+\mathrm{C}=\frac{1}{2} \mathrm{e}^{(2 \mathrm{x}+3)}+\mathrm{C}\)

Question 17. \(\int \frac{\mathrm{x}}{\mathrm{e}^{\mathrm{x}^2}} \mathrm{dx}\)
Solution:

Let \(\mathrm{I}=\int \frac{\mathrm{x}}{\mathrm{e}^{\mathrm{x}^2}} \mathrm{dx}\)

Put \(\mathrm{x}^2=\mathrm{t} \Rightarrow 2 \mathrm{xdx}=\mathrm{dt}\)

∴ \(\mathrm{I}=\frac{1}{2} \int \frac{1}{\mathrm{e}^{\mathrm{t}}} \mathrm{dt}=\frac{1}{2} \int \mathrm{e}^{-t} \mathrm{dt}=\frac{1}{2}\left(\frac{\mathrm{e}^{-1}}{-1}\right)+\mathrm{C}=-\frac{1}{2} \mathrm{e}^{-\mathrm{x}^2}+\mathrm{C}=\frac{-1}{2 \mathrm{e}^{\mathrm{x}^2}}+\mathrm{C}\)

Question 18. \(\int \frac{e^{\sin -1 x}}{1+x^2} d x\)
Solution:

Let \(\mathrm{I}=\int \frac{\mathrm{e}^{\mathrm{lin}^{-1} \mathrm{x}}}{1+\mathrm{x}^2} \mathrm{dx}\)

Put \(\tan ^{-1} \mathrm{x}=\mathrm{t} \Rightarrow \frac{1}{1+\mathrm{x}^2} \mathrm{dx}=\mathrm{dt}\)

∴ \(\mathrm{I}=\int \mathrm{e}^t \mathrm{dt}=\mathrm{e}^{\mathrm{t}}+\mathrm{C}=\mathrm{e}^{\tan ^{-1} \mathrm{x}}+\mathrm{C}
\)

Question 19. \(\int \frac{\mathrm{e}^{2 \mathrm{x}}-1}{\mathrm{e}^{2 \mathrm{x}}+1} \mathrm{dx}\)
Solution:

Let \(\mathrm{I}=\int \frac{\mathrm{e}^{2 \mathrm{x}}-1}{\mathrm{e}^{2 \mathrm{x}}+1} \mathrm{dx} \Rightarrow \mathrm{I}=\int \frac{\mathrm{e}^{\mathrm{x}}-\mathrm{e}^{-\mathrm{x}}}{\mathrm{e}^{\mathrm{x}}+\mathrm{e}^{-\mathrm{x}}} \mathrm{dx}\)

Dividing numerator and denominator by \(\mathrm{e}^{\mathrm{x}}\)

Put \(\mathrm{e}^{\mathrm{x}}+\mathrm{e}^{-\mathrm{x}}=\mathrm{t} \Rightarrow\left(\mathrm{e}^{\mathrm{x}}-\mathrm{e}^{-x}\right) \mathrm{dx}=\mathrm{dt}\)

∴ \(I=\int \frac{d t}{t}=\log |t|+C=\log \left|e^x+e^{-x}\right|+C\)

Question 20. \(\int \frac{e^{2 x}-e^{-2 x}}{e^{2 x}+e^{-2 x}} d x\)
Solution:

Let \(\mathrm{I}=\int \frac{\mathrm{e}^{2 \mathrm{x}}-\mathrm{e}^{-2 \mathrm{x}}}{\mathrm{e}^{2 \mathrm{x}}+\mathrm{e}^{-2 \mathrm{x}}} \mathrm{dx}\)

Put \(\mathrm{e}^{2 \mathrm{x}}+\mathrm{e}^{-2 \mathrm{x}}=\mathrm{t} \Rightarrow\left(2 \mathrm{e}^{2 \mathrm{x}}-2 \mathrm{e}^{-2 \mathrm{x}}\right) \mathrm{dx}=\mathrm{dt} \Rightarrow\left(\mathrm{e}^{2 \mathrm{x}}-\mathrm{e}^{-2 \mathrm{x}}\right) \mathrm{dx}=\frac{\mathrm{dt}}{2}\)

∴ \(\mathrm{I}=\frac{1}{2} \int_{\mathrm{t}}^1 \mathrm{dt}=\frac{1}{2} \log |\mathrm{t}|+\mathrm{C}=\frac{1}{2} \log \left|\mathrm{e}^{2 \mathrm{x}}+\mathrm{e}^{-2 \mathrm{x}}\right|+\mathrm{C}\)

Question 21. \(\int \tan ^2(2 x-3) d x\)
Solution:

Let \(\mathrm{I}=\int \tan ^2(2 \mathrm{x}-3) \mathrm{dx}\)

Put \(2 \mathrm{x}-3=\mathrm{t} \Rightarrow \mathrm{dx}=\frac{\mathrm{dt}}{2}\)

∴ I = \(\frac{1}{2} \int \tan ^2 t \cdot d t=\frac{1}{2} \int\left(\sec ^2 t-1\right) d t\)

= \(\frac{1}{2}[\tan t-t]+C_1=\frac{1}{2}[\tan (2 x-3)-(2 x-3)]+C_1\)

= \(\frac{1}{2} \tan (2 x-3)-x+\frac{3}{2}+C_1=\frac{1}{2} \tan (2 x-3)-x+C \text {; where } C_1=C_1+\frac{3}{2}\)

Question 22. \(\int \sec ^2(7-4 x) d x\)
Solution:

Let \(\mathrm{I}=\int \sec ^2(7-4 \mathrm{x}) \mathrm{dx}\)

Put \(7-4 \mathrm{x}=\mathrm{t} \Rightarrow-4 \mathrm{dx}=\mathrm{dt}\)

∴ \(\mathrm{I}=-\frac{1}{4} \int \sec ^2 \mathrm{tdt}=\frac{-1}{4}(\tan \mathrm{t})+\mathrm{C}=\frac{-1}{4} \tan (7-4 \mathrm{x})+\mathrm{C}\)

Question 23. \(\int \frac{\sin ^{-1} x}{\sqrt{1-x^2}} d x\)
Solution:

Let \(\mathrm{I}=\int \frac{\sin ^{-1} \mathrm{x}}{\sqrt{1-\mathrm{x}^2}} \mathrm{dx}\)

Put \(\sin ^{-1} \mathrm{x}=\mathrm{t} \Rightarrow \frac{1}{\sqrt{1-\mathrm{x}^2}} \mathrm{dx}=\mathrm{dt}\)

∴ \(\mathrm{I}=\int \mathrm{tdt}=\frac{\mathrm{t}^2}{2}+\mathrm{C}=\frac{\left(\sin ^{-1} \mathrm{x}\right)^2}{2}+\mathrm{C}\)

Question 24. \(\int \frac{2 \cos x-3 \sin x}{6 \cos x+4 \sin x} d x\)
Solution:

Let \(\mathrm{I}=\int \frac{2 \cos \mathrm{x}-3 \sin \mathrm{x}}{6 \cos \mathrm{x}+4 \sin \mathrm{x}} \mathrm{dx}=\frac{1}{2} \int \frac{2 \cos \mathrm{x}-3 \sin \mathrm{x}}{3 \cos \mathrm{x}+2 \sin \mathrm{x}} \mathrm{dx}\)

Put \(3 \cos x+2 \sin x=t \Rightarrow(-3 \sin x+2 \cos x) d x=d t\)

⇒ \((2 \cos x-3 \sin x) d x=d t\)

⇒ I = \(\frac{1}{2} \int_t^1 \frac{d t}{t}=\frac{1}{2} \log |t|+C=\frac{1}{2} \log |2 \sin x+3 \cos x|+C\)

Question 25. \(\int \frac{1}{\cos ^2 x(1-\tan x)^2} d x\)
Solution:

Let \(I=\int \frac{1}{\cos ^2 x(1-\tan x)^2} d x=\int \frac{\sec ^2 x}{(1-\tan x)^2} d x\)

Put \((1-\tan x)=t \Rightarrow \sec ^2 x d x=-d t\)

∴ I = \(\int \frac{-d t}{t^2}=-\int t^{-2} d t=\frac{1}{t}+C=\frac{1}{(1-\tan x)}+C\)

Question 26. \(\int \frac{\cos \sqrt{x}}{\sqrt{x}} d x\)
Solution:

Let \(I=\int \frac{\cos \sqrt{x}}{\sqrt{x}} d x\)

Put \(\sqrt{\mathrm{x}}=\mathrm{t} \Rightarrow \frac{1}{2 \sqrt{\mathrm{x}}} \mathrm{dx}=\mathrm{dt}\)

∴ I = \(2 \int \cos t d t=2 \sin \mathrm{t}+C=2 \sin \sqrt{x}+C\)

Question 27. \(\int \sqrt{\sin 2 x} \cos 2 x d x\)
Solution:

Let \(\mathrm{I}=\int \sqrt{\sin 2 \mathrm{x}} \cos 2 \mathrm{x} d \mathrm{x}\)

Put \(\sin 2 x=t^2 \Rightarrow 2 \cos 2 x d x=2 t d t \Rightarrow \cos 2 x d x=t d t\)

∴ \(\mathrm{I}=\int \mathrm{t} \cdot \mathrm{tdt}=\int \mathrm{t}^2 \mathrm{dt}=\frac{\mathrm{t}^3}{3}+\mathrm{C}=\frac{1}{3}(\sin 2 \mathrm{x})^{3 / 2}+\mathrm{C}\)

Question 28. \(\int \frac{\cos x}{\sqrt{1+\sin x}} d x\)
Solution:

Let \(I=\int \frac{\cos x}{\sqrt{1+\sin x}} d x\)

Put \(1+\sin x=t^2 \Rightarrow \cos x d x=2 t d t\)

∴ \(\mathrm{I}=\int \frac{1}{\mathrm{t}} \cdot 2 \mathrm{tdt}=2 \int 1 \cdot \mathrm{dt}=2 \mathrm{t}+\mathrm{C}=2 \sqrt{1+\sin \mathrm{x}}+\mathrm{C}\)

Question 29. \(\int \cot x \log \sin x d x\)
Solution:

Let \(\mathrm{I}=\int \cot \mathrm{x} \log \sin \mathrm{x} \mathrm{dx}\)

Put \(\log \sin x=t \Rightarrow \frac{1}{\sin x} \cdot \cos x d x=d t \Rightarrow \cot x d x=d t\)

∴ \(\mathrm{I}=\int \mathrm{tdt}=\frac{\mathrm{t}^2}{2}+\mathrm{C}=\frac{1}{2}(\log \sin \mathrm{x})^2+\mathrm{C}\)

Question 30. \(\int \frac{\sin x}{1+\cos x} d x\)
Solution:

Let \(I=\int \frac{\sin x}{1+\cos x} d x\)

Put \(1+\cos x=t \Rightarrow-\sin x d x=d t\)

∴ I =\(\int-\frac{d t}{t}=-\log |t|+C=-\log |1+\cos x|+C\)

Question 31. \(\int \frac{\sin x}{(1+\cos x)^2} d x\)
Solution:

Let \(I=\int \frac{\sin x}{(1+\cos x)^2} d x\)

Put \(1+\cos \mathrm{x}=\mathrm{t} \Rightarrow-\sin \mathrm{x} d \mathrm{x}=\mathrm{dt}\)

∴ I = \(\int-\frac{d t}{t^2}=-\int t^{-2} d t=\frac{1}{t}+C=\frac{1}{1+\cos x}+C\)

Question 32. \(\int \frac{1}{1+\cot x} d x\)
Solution:

Let \(I=\int \frac{1}{1+\cot x} d x=\int \frac{1}{1+\left(\frac{\cos x}{\sin x}\right)} d x=\int \frac{\sin x}{\sin x+\cos x} d x=\frac{1}{2} \int \frac{2 \sin x}{\sin x+\cos x} d x\)

= \(\frac{1}{2} \int \frac{(\sin x+\cos x)+(\sin x-\cos x)}{(\sin x+\cos x)} d x=\frac{1}{2} \int 1 d x-\frac{1}{2} \int \frac{\cos x-\sin x}{\sin x+\cos x} d x\)

= \(\frac{x}{2}-\frac{1}{2} \log |\sin x+\cos x|+C\) (because \(\int \frac{f^{\prime}(x)}{f(x)} d x=\log |f(x)|+C\))

Question 33. \(\int \frac{1}{1-\tan x} d x\)
Solution:

Let I = \(\int \frac{1}{1-\tan x} d x=\int \frac{1}{1-\left(\frac{\sin x}{\cos x}\right)} d x=\int \frac{\cos x}{\cos x-\sin x} d x=\frac{1}{2} \int \frac{2 \cos x}{\cos x-\sin x} d x\)

= \(\frac{1}{2} \int \frac{(\cos x-\sin x)+(\cos x+\sin x)}{(\cos x-\sin x)} d x=\frac{1}{2} \int 1 d x-\frac{1}{2} \int \frac{(-\sin x-\cos x)}{(\cos x-\sin x)} d x\)

= \(\frac{x}{2}-\frac{1}{2} \log |\cos x-\sin x|+C\) (because \(\int \frac{f^{\prime}(x)}{f(x)} d x=\log |f(x)|+C\))

Question 34. \(\int \frac{\sqrt{\tan x}}{\sin x \cos x} d x\)
Solution:

Let \(I=\int \frac{\sqrt{\tan x}}{\sin x \cos x} d x=\int \frac{\sqrt{\tan x}}{\frac{\sin x}{\cos x} \cdot \cos ^2 x} d x=\int \frac{\sqrt{\tan x}}{\tan x \cdot \cos ^2 x} d x\)

⇒\(\mathrm{I}=\int \frac{\sec ^2 x d x}{\sqrt{\tan x}}\)

Let \(\tan x=t^2 \Rightarrow \sec ^2 x d x=2 t d t\)

∴ I = \(\int \frac{1}{t} \cdot 2 t d t=2 \int 1 d t=2 t+C=2 \sqrt{\tan x}+C\)

Question 35. \(\int \frac{(1+\log x)^2}{x} d x\)
Solution:

Let \(\mathrm{I}=\int \frac{(1+\log \mathrm{x})^2}{\mathrm{x}} \mathrm{dx}\)

Put \(1+\log x=t \Rightarrow \frac{1}{x} d x=d t\)

∴ I = \(\int t^2 d t=\frac{t^3}{3}+C=\frac{(1+\log x)^3}{3}+C\)

Question 36. \(\int \frac{(x+1)(x+\log x)^2}{x} d x\)
Solution:

Let \(I=\int \frac{(x+1)(x+\log x)^2}{x} d x\)

Put \((x+\log x)=t \Rightarrow\left(1+\frac{1}{x}\right) d x=d t \Rightarrow\left(\frac{x+1}{x}\right) d x=d t\)

∴ \(\mathrm{I}=\int \mathrm{t}^2 \mathrm{dt}=\frac{\mathrm{t}^3}{3}+\mathrm{C}=\frac{1}{3}(\mathrm{x}+\log \mathrm{x})^3+\mathrm{C}\)

Question 37. \(\int \frac{x^3 \sin \left(\tan ^{-1} x^4\right)}{1+x^8} d x\)
Solution:

Let \(I=\int \frac{x^3 \sin \left(\tan ^{-1} x^4\right)}{1+x^8} d x\)

Put \(\tan ^{-1} x^4=t \Rightarrow \frac{4 x^3}{\left(1+x^8\right)} d x=d t \Rightarrow \frac{x^3}{\left(1+x^8\right)} d x=\frac{d t}{4}\)

∴ \(I=\frac{1}{4} \int \sin t \cdot d t=\frac{1}{4}(-\cos t)+C=-\frac{1}{4} \cos \left(\tan ^{-1} x^4\right)+C\)

Choose The Correct Answer

Question 38. \(\int \frac{10 x^9+10^x \log _e 10}{x^{10}+10^x} d x\) equals:

  1. \(10^x-\mathrm{x}^{10}+\mathrm{C}\)
  2. \(10^5+\mathrm{x}^{10}+\mathrm{C}\)
  3. \(\left(10^5-x^{10}\right)^{-1}+C\)
  4. \(\log \left(10^x+x^{10}\right)+C\)

Solution: 4. \(\log \left(10^x+x^{10}\right)+C\)

Let \(\mathrm{I}=\int \frac{10 \mathrm{x}^9+10^{\mathrm{x}} \log _{\mathrm{c}} 10}{\mathrm{x}^{10}+10^{\mathrm{x}}} \mathrm{dx}\)

Put \(x^{10}+10^x=t \Rightarrow\left(10 x^9+10^x \log , 10\right) d x=d t\)

∴ \(I=\int \frac{d t}{t}=\log |t|+C=\log \left|10^x+x^{10}\right|+C\)

Hence, the correct answer is (4)

Question 39. \(\int \frac{d x}{\sin ^2 x \cos ^2 x}\) equals?

  1. \(\tan x+\cot x+C\)
  2. \(\tan x-\cot x+C\)
  3. \(\tan \mathrm{x} \cdot \cot \mathrm{x}+\mathrm{C}\)
  4. \(\tan x-\cot 2 x+C\)

Solution: 2. \(\tan x-\cot x+C\)

Let I = \(\int \frac{d x}{\sin ^2 x \cos ^2 x}=\int \frac{1}{\sin ^2 x \cos ^2 x} d x\)

= \(\int \frac{\sin ^2 x+\cos ^2 x}{\sin ^2 x \cos ^2 x} d x\)

= \(\int \frac{\sin ^2 x}{\sin ^2 x \cos ^2 x} d x+\int \frac{\cos ^2 x}{\sin ^2 x \cos ^2 x} d x\)

= \(\int \sec ^2 x d x+\int \mathrm{cosec}^2 x d x\)

= \(\tan x-\cot x+C\) (because \(\sin ^2 x+\cos ^2=1\))

Hence, the correct answer is (2).

Integrals Exercise 7.3

Find The Integrals Of The Function

Question 1. \(\int \sin ^2(2 x+5) d x\)
Solution:

Let I = \(\int \sin ^2(2 x+5) d x=\frac{1}{2} \int 2 \sin ^2(2 x+5) d x\)

I = \(\frac{1}{2} \int\{1-\cos (4 x+10)\} d x=\frac{1}{2} \int 1 d x-\frac{1}{2} \int \cos (4 x+10) d x\)

= \(\frac{1}{2} x-\frac{1}{2}\left(\frac{\sin (4 x+10)}{4}\right)+C=\frac{1}{2} x-\frac{1}{8} \sin (4 x+10)+C\)

Question 2. \(\int \sin 3 x \cos 4 x d x\)
Solution:

Let \(\mathrm{I}=\int \sin 3 \mathrm{x} \cos 4 \mathrm{xdx}=\frac{1}{2} \int 2 \sin 3 \mathrm{x} \cos 4 \mathrm{xdx}=\frac{1}{2} \int\{\sin 7 \mathrm{x}+\sin (-\mathrm{x})\} \mathrm{dx}\)

= \(\frac{1}{2} \int\{\sin 7 x-\sin x\} d x=\frac{1}{2} \int \sin 7 x d x-\frac{1}{2} \int \sin x d x=\frac{1}{2}\left(\frac{-\cos 7 x}{7}\right)-\frac{1}{2}(-\cos x)+C\)

= \(\frac{-\cos 7 x}{14}+\frac{\cos x}{2}+C\)

Question 3. \(\int \cos 2 x \cos 4 x \cos 6 x d x\)
Solution:

Let \(I=\int \cos 2 x \cos 4 x \cos 6 x d x=\frac{1}{2} \int \cos 2 x(2 \cos 4 x \cos 6 x) d x\)

= \(\frac{1}{2} \int \cos 2 x[\cos (4 x+6 x)+\cos (4 x-6 x)] d x\)

= \(\frac{1}{2} \int\{\cos 2 x \cos 10 x+\cos 2 x \cos (-2 x)\} d x\) (because \(\cos (-\theta)=\cos \theta\))

= \(\frac{1}{2} \int\left\{\cos 2 x \cos 10 x+\cos ^2 2 x\right\} d x=\frac{1}{4} \int\left\{2 \cos 2 x \cos 10 x+2 \cos ^2 2 x\right\} d x\)

= \(\frac{1}{4} \int(\cos 12 x+\cos 8 x+1+\cos 4 x) d x=\frac{1}{4}\left[\frac{\sin 12 x}{12}+\frac{\sin 8 x}{8}+x+\frac{\sin 4 x}{4}\right]+C\)

Question 4. \(\int \sin ^3(2 x+1) \mathrm{d} x\)
Solution:

Let \(\mathrm{I}=\int \sin ^3(2 \mathrm{x}+1) \mathrm{dx}=\int\left(1-\cos ^2(2 \mathrm{x}+1)\right\} \sin (2 \mathrm{x}+1) \mathrm{dx}\)

Put \(\cos (2 \mathrm{x}+1)=\mathrm{t} \Rightarrow-2 \sin (2 \mathrm{x}+1) \mathrm{dx}=\mathrm{dt} \Rightarrow \sin (2 \mathrm{x}+1) \mathrm{dx}=-\frac{\mathrm{dt}}{2}\)

∴ I = \(\frac{-1}{2} \int\left(1-t^2\right) d t=\frac{-1}{2}\left\{t-\frac{t^3}{3}\right\}+C=\frac{-1}{2}\left\{\cos (2 x+1)-\frac{\cos ^3(2 x+1)}{3}\right\}+C\)

= \(\frac{-\cos (2 x+1)}{2}+\frac{\cos ^3(2 x+1)}{6}+C\)

Question 5. \(\int \sin ^3 x \cos ^3 x d x\)
Solution:

Let \(I=\int \sin ^3 x \cos ^3 x \cdot d x=\int \cos ^3 x \cdot \sin ^2 x \cdot \sin x \cdot d x=\int \cos ^3 x\left(1-\cos ^2 x\right) \sin x \cdot d x\)

Put \(\cos \mathrm{x}=\mathrm{t} \Rightarrow-\sin \mathrm{x} \cdot \mathrm{dx}=\mathrm{dt}\)

∴ I = \(-\int t^3\left(1-t^2\right) d t=-\int\left(t^3-t^5\right) d t=-\left\{\frac{t^4}{4}-\frac{t^6}{6}\right\}+C\)

= \(-\left\{\frac{\cos ^4 x}{4}-\frac{\cos ^6 x}{6}\right\}+C=\frac{\cos ^6 x}{6}-\frac{\cos ^4 x}{4}+C\)

Question 6. \(\int \sin x \sin 2 x \sin 3 x d x\)
Solution:

Let I = \(\int \sin x \sin 2 x \sin 3 x d x=\frac{1}{2} \int \sin x\{2 \sin 2 x \sin 3 x\} d x\)

∴ I = \(\frac{1}{2} \int[\sin x \cdot\{\cos (2 x-3 x)-\cos (2 x+3 x)\}] d x\)

= \(\frac{1}{2} \int(\sin x \cos (-x)-\sin x \cos 5 x) d x=\frac{1}{2} \int(\sin x \cos x-\sin x \cos 5 x) d x\)

= \(\frac{1}{4} \int(2 \sin x \cos x-2 \sin x \cos 5 x) d x=\frac{1}{4} \int \sin 2 x d x-\frac{1}{4} \int\{\sin 6 x+\sin (-4 x)\} d x\)

= \(\frac{1}{4} \int \sin 2 x d x-\frac{1}{4} \int\{\sin 6 x-\sin 4 x\} d x\)

= \(\frac{-\cos 2 x}{8}-\frac{1}{4}\left[\frac{-\cos 6 x}{6}+\frac{\cos 4 x}{4}\right]+C=-\frac{\cos 2 x}{8}-\frac{1}{8}\left[-\frac{\cos 6 x}{3}+\frac{\cos 4 x}{2}\right]+C\)

= \(\frac{1}{8}\left[\frac{\cos 6 x}{3}-\frac{\cos 4 x}{2}-\cos 2 x\right]+C\)

Question 7. \(\int \sin 4 x \sin 8 x d x\)
Solution:

Let \(I=\int \sin 4 x \sin 8 x d x=\frac{1}{2} \int 2 \sin 4 x \sin 8 x d x\)

∴ I = \(\frac{1}{2} \int\{\cos (4 x-8 x)-\cos (4 x+8 x)\} d x=\frac{1}{2} \int(\cos (-4 x)-\cos 12 x) d x\)

= \(\frac{1}{2} \int(\cos 4 x-\cos 12 x) d x=\frac{1}{2}\left[\frac{\sin 4 x}{4}-\frac{\sin 12 x}{12}\right]+C\)

Question 8. \(\int \frac{1-\cos x}{1+\cos x} d x\)
Solution:

Let \(I=\int \frac{1-\cos x}{1+\cos x} d x=\int \frac{2 \sin ^2 x / 2}{2 \cos ^2 x / 2} d x=\int \tan ^2 \frac{x}{2} d x=\int\left(\sec ^2 \frac{x}{2}-1\right) d x\)

= \(\left[\frac{\tan \frac{x}{2}}{\frac{1}{2}}-x\right]+C=2 \tan \frac{x}{2}-x+C\)

Question 9. \(\int \frac{\cos x}{1+\cos x} d x\)
Solution:

Let \(I=\int \frac{\cos x}{1+\cos x} d x=\int \frac{\cos ^2 \frac{x}{2}-\sin ^2 \frac{x}{2}}{2 \cos ^2 \frac{x}{2}} d x\)

∴ I = \(\frac{1}{2} \int\left(1-\tan ^2 \frac{x}{2}\right) d x=\frac{1}{2} \int\left(1-\sec ^2 \frac{x}{2}+1\right) d x=\frac{1}{2} \int\left(2-\sec ^2 \frac{x}{2}\right) d x\)

= \(\frac{1}{2}\left[2 x-\frac{\tan \frac{x}{2}}{1 / 2}\right]+C=x-\tan \frac{x}{2}+C\)

Question 10. \(\int \sin ^4 x d x\)
Solution:

Let \(I=\int \sin ^4 x d x=\int \sin ^2 x \sin ^2 x d x=\int\left(\frac{1-\cos 2 x}{2}\right)\left(\frac{1-\cos 2 x}{2}\right) d x\)

= \(\int \frac{1}{4}(1-\cos 2 x)^2 d x=\frac{1}{4} \int\left[1+\cos ^2 2 x-2 \cos 2 x\right] d x\)

= \(\frac{1}{4} \int\left[1+\left(\frac{1+\cos 4 x}{2}\right)-2 \cos 2 x\right] d x\)

= \(\frac{1}{4} \int\left[1+\frac{1}{2}+\frac{1}{2} \cos 4 x-2 \cos 2 x\right] d x=\frac{1}{4} \int\left[\frac{3}{2}+\frac{1}{2} \cos 4 x-2 \cos 2 x\right] d x\)

= \(\frac{3 x}{8}+\frac{1}{32} \sin 4 x-\frac{1}{4} \sin 2 x+C\)

Question 11. \(\int \cos ^4 2 x d x\)
Solution:

Let \(I=\int \cos ^4 2 x d x=\int\left(\cos ^2 2 x\right)^2 d x=\int\left(\frac{1+\cos 4 x}{2}\right)^2 d x\)

= \(\frac{1}{4} \int\left[1+\cos ^2 4 x+2 \cos 4 x\right] d x\)

= \(\frac{1}{4} \int\left[1+\left(\frac{1+\cos 8 x}{2}\right)+2 \cos 4 x\right] d x=\frac{1}{4} \int\left[1+\frac{1}{2}+\frac{\cos 8 x}{2}+2 \cos 4 x\right] d x\)

= \(\frac{1}{4} \int\left[\frac{3}{2}+\frac{\cos 8 x}{2}+2 \cos 4 x\right] d x=\int\left(\frac{3}{8}+\frac{\cos 8 x}{8}+\frac{\cos 4 x}{2}\right) d x\)

= \(\frac{3}{8} x+\frac{\sin 8 x}{64}+\frac{\sin 4 x}{8}+C\)

Question 12. \(\int \frac{\sin ^2 x}{1+\cos x} d x\)
Solution:

Let \(I=\int \frac{\sin ^2 x}{1+\cos x} d x=\int \frac{\left(1-\cos ^2 x\right)}{(1+\cos x)} d x=\int(1-\cos x) d x=x-\sin x+C\)

Question 13. \(\int \frac{\cos 2 x-\cos 2 \alpha}{\cos x-\cos \alpha} d x\)
Solution:

Let, I=\(\int \frac{\cos 2 x-\cos 2 \alpha}{\cos x-\cos \alpha} d x=\int \frac{\left(2 \cos ^2 x-1\right)-\left(2 \cos ^2 \alpha-1\right)}{(\cos x-\cos \alpha)} d x=\int \frac{2\left(\cos ^2 x-\cos ^2 \alpha\right)}{(\cos x-\cos \alpha)} d x\)

= \(2 \int(\cos x+\cos \alpha) d x=2 \sin x+2 x \cos \alpha+C=2[\sin x+x \cos \alpha]+C\)

Question 14. \(\int \frac{\cos x-\sin x}{1+\sin 2 x} d x\)
Solution:

Let \(I=\int \frac{\cos x-\sin x}{1+\sin 2 x} d x=\int \frac{\cos x-\sin x}{(\sin x+\cos x)^2} d x\) (because \(1+\sin 2 x=(\sin x+\cos x)^2\))

Put \(\sin \mathrm{x}+\cos \mathrm{x}=\mathrm{t} \Rightarrow(\cos \mathrm{x}-\sin \mathrm{x}) \mathrm{dx}=\mathrm{dt}\)

∴ I = \(\int \frac{1}{t^2} d t=-\frac{1}{t}+C=-\frac{1}{(\sin x+\cos x)}+C\)

Question 15. \(\int \tan ^3 2 x \sec 2 x d x\)
Solution:

Let \(I=\int \tan ^3 2 x \sec 2 x d x=\int \tan ^2 2 x \cdot \sec 2 x \tan 2 x d x\)

= \(\int\left(\sec ^2 2 x-1\right) \sec 2 x \tan 2 x d x\)

Put sec 2x=t

⇒ sec 2x tan 2x dx = \(\frac{d t}{2}\)

∴ I = \(\int\left(t^2-1\right) \cdot \frac{d t}{2}=\frac{1}{2} \int\left(t^2-1\right) d t=\frac{1}{2}\left[\frac{t^3}{3}-t\right]+C\)

= \(\frac{1}{6}(\sec 2 x)^3-\frac{1}{2}(\sec 2 x)+C=\frac{1}{6} \sec ^3 2 x-\frac{1}{2} \sec 2 x+C\)

Question 16. \(\int \tan ^4 x d x\)
Solution:

Let \(I=\int \tan ^4 x d x=\int \tan ^2 x \cdot \tan ^2 x d x=\int\left(\sec ^2 x-1\right) \tan ^2 x d x\)

= \(\int\left(\tan ^2 x \sec ^2 x-\tan ^2 x\right) d x=\int \tan ^2 x \sec ^2 x d x-\int \sec ^2 x d x+\int 1 d x\)

= \(\frac{\tan ^3 x}{3}-\tan x+x+C\) (because \(\int\{f(x)\}^{\prime \prime} \cdot f^{\prime}(x) d x=\frac{\{f(x)\}^{n+1}}{n+1}+C\))

Question 17. \(\int \frac{\sin ^3 x+\cos ^3 x}{\sin ^2 x \cos ^2 x} d x\)
Solution:

Let \(I=\int \frac{\sin ^3 x+\cos ^3 x}{\sin ^2 x \cos ^2 x} d x=\int\left(\frac{\sin ^3 x}{\sin ^2 x \cos ^2 x}+\frac{\cos ^3 x}{\sin ^2 x \cos ^2 x}\right) d x\)

= \(\int\left(\frac{\sin x}{\cos ^2 x}+\frac{\cos x}{\sin ^2 x}\right) d x=\int(\tan x \sec x+\cot x \mathrm{cosec} x) d x=\sec x-\mathrm{cosec} x+C\)

Question 18. \(\int \frac{\cos 2 x+2 \sin ^2 x}{\cos ^2 x} d x\)
Solution:

Let \(I=\int \frac{\cos 2 x+2 \sin ^2 x}{\cos ^2 x} d x=\int \frac{\left(1-2 \sin ^2 x\right)+2 \sin ^2 x}{\cos ^2 x} d x\)

= \(\int \frac{1}{\cos ^2 x} d x=\int \sec ^2 x d x=\tan x+C\)

Question 19. \(\int \frac{1}{\sin x \cos ^3 x} d x\)
Solution:

Let \(I=\int \frac{1}{\sin x \cos ^3 x} d x=\int \frac{\sin ^2 x+\cos ^2 x}{\sin x \cos ^3 x} d x=\int\left(\frac{\sin x}{\cos ^3 x}+\frac{1}{\sin x \cos x}\right) d x\)

= \(\int\left(\tan x \sec ^2 x+\frac{\sec ^2 x}{\tan x}\right) d x\)

∴ I = \(\int \tan x \sec ^2 x d x+\int \frac{\sec ^2 x}{\tan x} d x\)

Put tan x = \(t \Rightarrow \sec ^2 \mathrm{x} d \mathrm{x}=\mathrm{dt}\)

⇒ \(\mathrm{I}=\int \mathrm{tdt}+\int \frac{1}{\mathrm{t}} \mathrm{dt}=\frac{\mathrm{t}^2}{2}+\log |\mathrm{t}|+\mathrm{C}=\frac{1}{2} \tan ^2 \mathrm{x}+\log |\tan \mathrm{x}|+\mathrm{C}\)

Question 20. \(\int \frac{\cos 2 x}{(\cos x+\sin x)^2} d x\)
Solution:

Let \(I=\int \frac{\cos 2 x}{(\cos x+\sin x)^2} d x=\int \frac{\left(\cos ^2 x-\sin ^2 x\right)}{(\cos x+\sin x)^2} d x=\int \frac{(\cos x+\sin x)(\cos x-\sin x)}{(\cos x+\sin x)^2} d x\)

= \(\int \frac{(\cos x-\sin x)}{(\cos x+\sin x)} d x=\log |\sin x+\cos x|+C\) (because \(\int \frac{f^{\prime}(x)}{f(x)} d x=\log |f(x)|+C\))

Question 21. \(\int \sin ^{-1}(\cos x) d x\)
Solution:

Let \(I=\int \sin ^{-1}(\cos x) d x=\int \sin ^{-1}\left[\sin \left(\frac{\pi}{2}-x\right)\right] d x=\int\left(\frac{\pi}{2}-x\right) d x=\frac{\pi}{2} x-\frac{x^2}{2}+C\)

Question 22. \(\int \frac{1}{\cos (x-a) \cos (x-b)} d x\)
Solution:

Let \(I=\int \frac{1}{\cos (x-a) \cos (x-b)} d x=\frac{1}{\sin (a-b)} \int\left[\frac{\sin (a-b)}{\cos (x-a) \cos (x-b)}\right] d x\)

= \(\frac{1}{\sin (a-b)} \int\left[\frac{\sin [(x-b)-(x-a)]}{\cos (x-a) \cos (x-b)}\right] d x\)

= \(\frac{1}{\sin (a-b)} \int\left[\frac{[\sin (x-b) \cos (x-a)-\cos (x-b) \sin (x-a)]}{\cos (x-a) \cos (x-b)}\right] d x\)

= \(\frac{1}{\sin (a-b)} \int[\tan (x-b)-\tan (x-a)] d x\)

= \(\frac{1}{\sin (a-b)}[-\log |\cos (x-b)|+\log |\cos (x-a)|]\)

= \(\frac{1}{\sin (a-b)}\left[\log \left|\frac{\cos (x-a)}{\cos (x-b)}\right|\right]+C\)

Choose The Correct Answer

Question 23. \(\int \frac{\sin ^2 x-\cos ^2 x}{\sin ^2 x \cos ^2 x} d x\) is equal to ?

  1. \(\tan x+\cot x\)+C
  2. \(\tan x+\mathrm{cosec}x\)+C
  3. \(-\tan x+\cot x\)+C
  4. \(\tan x+\sec x+\)C

Solution: 1. \(\tan x+\cot x\)+C

Let \(I=\int \frac{\sin ^2 x-\cos ^2 x}{\sin ^2 x \cos ^2 x} d x=\int\left(\frac{\sin ^2 x}{\sin ^2 x \cos ^2 x}-\frac{\cos ^2 x}{\sin ^2 x \cos ^2 x}\right) d x\)

= \(\int\left(\sec ^2 \mathrm{x}-\mathrm{cosec}^2 \mathrm{x}\right) \mathrm{dx}=\tan \mathrm{x}+\cot \mathrm{x}+\mathrm{C}\)

Hence, the correct answer is (1).

Question 24. \(\int \frac{e^x(1+x)}{\cos ^2\left(e^x x\right)} d x\) equals ?

  1. \(-\cot \left(e x^x\right)+C\)
  2. \(\tan \left(\mathrm{xe}^x\right)+C\)
  3. \(\tan \left(e^x\right)+C\)
  4. \(\cot \left(e^x\right)+C\)

Solution: 2. \(\tan \left(\mathrm{xe}^x\right)+C\)

Let I = \(\int \frac{e^x(1+x)}{\cos ^2\left(e^x x\right)} d x\)

Put \(e^x \cdot x=t \Rightarrow\left(e^x \cdot x+e^x \cdot 1\right) d x=d t\)

⇒ \(e^x(x+1) d x=d t\)

∴ \(I=\int \frac{d t}{\cos ^2 t}=\int \sec ^2 t d t\)

= \(\tan t+C=\tan \left(e^x \cdot x\right)+C=\tan \left(x \cdot e^x\right)+C\)

Hence, the correct answer is (2).

Integrals Exercise 7.4

Integrate The Function

Question 1. \(\int \frac{3 x^2}{x^6+1} d x\)
Solution:

Let \(\mathrm{I}=\int \frac{3 \mathrm{x}^2}{\mathrm{x}^6+1} \mathrm{dx}\)

Put \(\mathrm{x}^3=\mathrm{t} \Rightarrow 3 \mathrm{x}^2 \mathrm{dx}=\mathrm{dt}\)

∴ I = \(\int \frac{3 x^2}{\left(x^3\right)^2+1} d x=\int \frac{d t}{t^2+1}=\tan ^{-1} t+C=\tan ^{-1}\left(x^3\right)+C\)

Question 2. \(\int \frac{1}{\sqrt{1+4 \mathrm{x}^2}} \mathrm{dx}\)
Solution:

Let \(\mathrm{I}=\int \frac{1}{\sqrt{1+4 \mathrm{x}^2}} \mathrm{dx}\)

Put \(2 \mathrm{x}=\mathrm{t} \Rightarrow 2 \mathrm{dx}=\mathrm{dt}\)

∴ \(\mathrm{I}=\int \frac{1}{\sqrt{1+(2 \mathrm{x})^2}} \mathrm{dx}=\frac{1}{2} \int \frac{\mathrm{dt}}{\sqrt{1+\mathrm{t}^2}}=\frac{1}{2}\left[\log \left|\mathrm{t}+\sqrt{\mathrm{t}^2+1}\right|\right]+\mathrm{C}=\frac{1}{2} \log \left|2 \mathrm{x}+\sqrt{4 \mathrm{x}^2+1}\right|+\mathrm{C}\)

Question 3. \(\int \frac{1}{\sqrt{(2-x)^2+1}} d x\)
Solution:

Let \(I=\int \frac{1}{\sqrt{(2-x)^2+1}} d x\)

Put \(2-\mathrm{x}=\mathrm{t} \Rightarrow-\mathrm{dx}=\mathrm{dt}\)

∴ I = \(-\int \frac{1}{\sqrt{t^2+1}} d t=-\log \left|t+\sqrt{t^2+1}\right|+C=-\log \left|(2-x)+\sqrt{(2-x)^2+1}\right|+C\)

= \(\log \left|\frac{1}{(2-x)+\sqrt{x^2-4 x+5}}\right|+C\)

Question 4. \(\int \frac{1}{\sqrt{9-25 x^2}} d x\)
Solution:

Let \(\mathrm{I}=\int \frac{1}{\sqrt{9-25 \mathrm{x}^2}} \mathrm{dx}\)

Put \(5 \mathrm{x}=\mathrm{t} \Rightarrow 5 \mathrm{dx}=\mathrm{dt}\)

∴ \(I=\frac{1}{5} \int \frac{1}{\sqrt{9-t^2}} d t=\frac{1}{5} \int \frac{1}{\sqrt{3^2-t^2}} d t=\frac{1}{5} \sin ^{-1}\left(\frac{t}{3}\right)+C=\frac{1}{5} \sin ^{-1}\left(\frac{5 x}{3}\right)+C\)

Question 5. \(\int \frac{3 x}{1+2 x^4} d x\)
Solution:

Let \(\mathrm{I}=\int \frac{3 \mathrm{x}}{1+2 \mathrm{x}^4} \mathrm{dx}\)

Put \(\sqrt{2} \mathrm{x}^2=\mathrm{t} \Rightarrow 2 \sqrt{2} \mathrm{xdx}=\mathrm{dt}\)

∴ \(\mathrm{I}=\frac{3}{2 \sqrt{2}} \int \frac{\mathrm{dt}}{1+\mathrm{t}^2}=\frac{3}{2 \sqrt{2}}\left(\tan ^{-1} \mathrm{t}\right)+\mathrm{C}=\frac{3}{2 \sqrt{2}} \tan ^{-1}\left(\sqrt{2} \mathrm{x}^2\right)+\mathrm{C}\)

Question 6. \(\int \frac{x^2}{1-x^6} d x\)
Solution:

Let \(\mathrm{I}=\int \frac{\mathrm{x}^2}{1-\mathrm{x}^6} \mathrm{dx} \Rightarrow \mathrm{I}=\int \frac{\mathrm{x}^2}{1-\left(\mathrm{x}^3\right)^2} \mathrm{dx}\)

Put \(\mathrm{x}^3=\mathrm{t} \Rightarrow 3 \mathrm{x}^2 \mathrm{dx}=\mathrm{dt}\)

= \(\frac{1}{3} \int \frac{\mathrm{dt}}{1-\mathrm{t}^2}=\frac{1}{3}\left[\frac{1}{2} \log \left|\frac{1+\mathrm{t}}{1-\mathrm{t}}\right|\right]+\mathrm{C}=\frac{1}{6} \log \left|\frac{1+\mathrm{x}^3}{1-\mathrm{x}^3}\right|+\mathrm{C}\)

Question 7. \(\int \frac{\mathrm{x}-1}{\sqrt{\mathrm{x}^2-1}} \mathrm{dx}\)
Solution:

Let \(I=\int \frac{x-1}{\sqrt{x^2-1}} d x \Rightarrow \int \frac{x}{\sqrt{x^2-1}} d x-\int \frac{1}{\sqrt{x^2-1}} d x\)

= \(\int \frac{x}{\sqrt{x^2-1}} d x-\log \left|x+\sqrt{x^2-1}\right|\)

Put \(x^2-1=t^2 \Rightarrow 2 x d x=2 t d t\)

∴ I = \(\int \frac{t d t}{t}-\log \left|x+\sqrt{x^2-1}\right|=\int 1 d t-\log \left|x+\sqrt{x^2-1}\right|\)

= \(t-\log \left|x+\sqrt{x^2-1}\right|=\sqrt{x^2-1}-\log \left|x+\sqrt{x^2-1}\right|+C\)

Question 8. \(\int \frac{x^2}{\sqrt{x^6+a^6}} d x\)
Solution:

Let \(I=\int \frac{x^2}{\sqrt{x^6+a^6}} d x \Rightarrow \int \frac{x^2}{\sqrt{\left(x^3\right)^2+\left(a^3\right)^2}} d x\)

Put \(\mathrm{x}^3=\mathrm{t} \Rightarrow 3 \mathrm{x}^2 \mathrm{dx}=\mathrm{dt}\)

= \(\frac{1}{3} \int \frac{\mathrm{dt}}{\sqrt{\mathrm{t}^2+\left(\mathrm{a}^3\right)^2}}=\frac{1}{3} \log \left|\mathrm{t}+\sqrt{\mathrm{t}^2+\mathrm{a}^6}\right|+\mathrm{C}=\frac{1}{3} \log \left|\mathrm{x}^3+\sqrt{\mathrm{x}^6+\mathrm{a}^6}\right|+\mathrm{C}\)

Question 9. \(\int \frac{\sec ^2 x}{\sqrt{\tan ^2 x+4}} d x\)
Solution:

Let \(\mathrm{I}=\int \frac{\sec ^2 \mathrm{x}}{\sqrt{\tan ^2 \mathrm{x}+4}} \mathrm{dx}\)

Put \(\tan \mathrm{x}=\mathrm{t} \Rightarrow \sec ^2 \mathrm{x} d \mathrm{x}=\mathrm{dt}\)

∴ \(I=\int \frac{d t}{\sqrt{t^2+2^2}}=\log \left|t+\sqrt{t^2+4}\right|+C=\log \left|\tan x+\sqrt{\tan ^2 x+4}\right|+C\)

Question 10. \(\int \frac{1}{\sqrt{\mathrm{x}^2+2 \mathrm{x}+2}} \mathrm{dx}\)
Solution:

Let \(I=\int \frac{1}{\sqrt{x^2+2 x+2}} d x=\int \frac{1}{\sqrt{(x+1)^2+1}} d x\)

Put \(\mathrm{x}+1=\mathrm{t} \Rightarrow \mathrm{dx}=\mathrm{dt}\)

∴ I = \(\int \frac{1}{\sqrt{t^2+1}} d t=\log \left|t+\sqrt{t^2+1}\right|+C\)

= \(\log \left|(x+1)+\sqrt{(x+1)^2+1}\right|+C\)

= \(\log \left|(x+1)+\sqrt{x^2+2 x+2}\right|+C\)

Question 11. \(\int \frac{1}{\left(9 x^2+6 x+5\right)} d x\)
Solution:

Let \(\mathrm{I}=\int \frac{1}{\left(9 \mathrm{x}^2+6 \mathrm{x}+5\right)} \mathrm{dx}=\int \frac{1}{(3 \mathrm{x}+1)^2+4} \mathrm{dx}\)

Put \((3 \mathrm{x}+1)=\mathrm{t} \Rightarrow 3 \mathrm{dx}=\mathrm{dt}\)

∴ \(\mathrm{I}=\frac{1}{3} \int \frac{1}{\mathrm{t}^2+2^2} \mathrm{dt}=\frac{1}{3}\left[\frac{1}{2} \tan ^{-1}\left(\frac{\mathrm{t}}{2}\right)\right]+\mathrm{C}=\frac{1}{6} \tan ^{-1}\left(\frac{3 \mathrm{x}+1}{2}\right)+\mathrm{C}\)

Question 12. \(\int \frac{1}{\sqrt{7-6 x-x^2}} d x\)
Solution:

Let \(I=\int \frac{1}{\sqrt{7-6 x-x^2}} d x=\int \frac{1}{\sqrt{16-(x+3)^2}} d x\)

Put \(\mathrm{x}+3=\mathrm{t} \Rightarrow \mathrm{dx}=\mathrm{dt}\)

∴ I = \(\int \frac{1}{\sqrt{(4)^2-(t)^2}} d t=\sin ^{-1}\left(\frac{t}{4}\right)+C=\sin ^{-1}\left(\frac{x+3}{4}\right)+C\)

Question 13. \(\int \frac{1}{\sqrt{(x-1)(x-2)}} d x\)
Solution:

Let \(I=\int \frac{1}{\sqrt{(x-1)(x-2)}} d x=\int \frac{1}{\sqrt{x^2-3 x+2}} d x=\int \frac{1}{\sqrt{\left(x-\frac{3}{2}\right)^2-\frac{1}{4}}} d x\)

Put \(x-\frac{3}{2}=t \Rightarrow d x=d t\)

∴ I = \(\int \frac{1}{\sqrt{t^2-\left(\frac{1}{2}\right)^2}} d t=\log \left|t+\sqrt{t^2-\left(\frac{1}{2}\right)^2}\right|+C=\log \left|\left(x-\frac{3}{2}\right)+\sqrt{x^2-3 x+2}\right|+C\)

Question 14. \(\int \frac{1}{\sqrt{8+3 \mathrm{x}-\mathrm{x}^2}} \mathrm{dx}\)
Solution:

Let \(\mathrm{I}=\int \frac{1}{\sqrt{8+3 \mathrm{x}-\mathrm{x}^2}} \mathrm{dx} \Rightarrow \mathrm{I}=\int \frac{1}{\sqrt{\frac{41}{4}-\left(\mathrm{x}-\frac{3}{2}\right)^2}} \mathrm{dx}\)

Put \(\mathrm{x}-\frac{3}{2}=\mathrm{t} \Rightarrow \mathrm{dx}=\mathrm{dt}\)

∴ I = \(\int \frac{1}{\left(\frac{\sqrt{41}}{2}\right)^2-t^2} d t=\sin ^{-1}\left(\frac{t}{\frac{\sqrt{41}}{2}}\right)+C=\sin ^{-1}\left(\frac{x-3 / 2}{\frac{\sqrt{41}}{2}}\right)+C=\sin ^{-1}\left(\frac{2 x-3}{\sqrt{41}}\right)+C\)

Question 15. \(\int \frac{1}{\sqrt{(x-a)(x-b)}} d x\)
Solution:

Let \(I=\int \frac{1}{\sqrt{(x-a)(x-b)}} d x=\int \frac{1}{\sqrt{x^2-(a+b) x+a b}} d x=\int \frac{1}{\sqrt{\left\{x-\left(\frac{a+b}{2}\right)\right\}^2-\left(\frac{a-b}{2}\right)^2}} d x\)

Put \(\mathrm{x}-\left(\frac{\mathrm{a}+\mathrm{b}}{2}\right)=\mathrm{t} \Rightarrow \mathrm{dx}=\mathrm{dt}\)

⇒ \(\int \frac{1}{\sqrt{\left\{x-\left(\frac{a+b}{2}\right)\right\}^2-\left(\frac{a-b}{2}\right)^2}} d x=\int \frac{1}{\sqrt{t^2-\left(\frac{a-b}{2}\right)^2}} d t\)

= \(\log \left|t+\sqrt{t^2-\left(\frac{a-b}{2}\right)^2}\right|+C=\log \left|\left(x-\frac{a+b}{2}\right)+\sqrt{\left\{x-\left(\frac{a+b}{2}\right)\right\}^2-\left(\frac{a-b}{2}\right)^2}\right|+C\)

= \(\log \left|\left\{x-\left(\frac{a+b}{2}\right)\right\}+\sqrt{(x-a)(x-b)}\right|+C\)

Question 16. \(\int \frac{4 \mathrm{x}+1}{\sqrt{2 \mathrm{x}^2+\mathrm{x}-3}} \mathrm{dx}\)
Solution:

Let \(\mathrm{I}=\int \frac{4 \mathrm{x}+1}{\sqrt{2 \mathrm{x}^2+\mathrm{x}-3}} \mathrm{dx}\)

Put \(2 \mathrm{x}^2+\mathrm{x}-3=\mathrm{t}^2 \Rightarrow(4 \mathrm{x}+1) \mathrm{dx}=2 \mathrm{t} d \mathrm{t}\)

∴ \(\mathrm{I}=\int \frac{2 \mathrm{t}}{\mathrm{t}} \mathrm{dt}=2 \int 1 \mathrm{dt}=2 \mathrm{t}+\mathrm{C}=2 \sqrt{2 \mathrm{x}^2+\mathrm{x}-3}+\mathrm{C}\)

Question 17. \(\int \frac{x+2}{\sqrt{x^2-1}} d x\)
Solution:

Let \(\mathrm{I}=\int \frac{\mathrm{x}+2}{\sqrt{\mathrm{x}^2-1}} \mathrm{dx}\)

Let x+2=A \(\frac{d}{d x}\left(x^2-1\right)+B\)

⇒ \(\mathrm{x}+2=\mathrm{A}(2 \mathrm{x})+\mathrm{B}\)

Equating the coefficients of x and constant term on both sides, we get 2A=1 \(A=\frac{1}{2}, B=2\)

From (1), we obtain, \((x+2)=\frac{1}{2}(2 x)+2\)

∴ I = \(\int \frac{\frac{1}{2}(2 x)+2}{\sqrt{x^2-1}} d x=\frac{1}{2} \int \frac{2 x}{\sqrt{x^2-1}} d x+\int \frac{2}{\sqrt{x^2-1}} d x\)

= \(\frac{1}{2} \int \frac{2 x}{\sqrt{x^2-1}} d x+2 \int \frac{1}{\sqrt{x^2-1}} d x=\frac{1}{2} \int \frac{2 x}{\sqrt{x^2-1}} d x+2 \log \left|x+\sqrt{x^2-1}\right|\)

Put \(x^2-1=t^2 \Rightarrow 2 x d x=2 t d t\)

∴ \(\mathrm{I}=\frac{1}{2} \int \frac{2 t}{t} \mathrm{dt}+2 \log \left|\mathrm{x}+\sqrt{\mathrm{x}^2-1}\right|=\int 1 \mathrm{dt}+2 \log \left|\mathrm{x}+\sqrt{\mathrm{x}^2-1}\right|\)

= \(\mathrm{t}+2 \log \left|\mathrm{x}+\sqrt{\mathrm{x}^2-1}\right|+\mathrm{C}=\sqrt{\mathrm{x}^2-1}+2 \log \left|\mathrm{x}+\sqrt{\mathrm{x}^2-1}\right|+\mathrm{C}\)

Question 18. \(\int \frac{5 x-2}{1+2 x+3 x^2} d x\)
Solution:

Let \(I=\int \frac{5 x-2}{1+2 x+3 x^2} d x\)

Let \(5 \mathrm{x}-2=\mathrm{A} \frac{\mathrm{d}}{\mathrm{dx}}\left(1+2 \mathrm{x}+3 \mathrm{x}^2\right)+\mathrm{B} \Rightarrow 5 \mathrm{x}-2=\mathrm{A}(2+6 \mathrm{x})+\mathrm{B}\)

Equating the coefficients of x and the constant term on both sides, we get

6\(\mathrm{~A}=5 \Rightarrow \mathrm{A}=\frac{5}{6}, 2 \mathrm{~A}+\mathrm{B}=-2 \Rightarrow \mathrm{B}=-2-2 \mathrm{~A}=-\frac{11}{3} \Rightarrow 5 \mathrm{x}-2=\frac{5}{6}(2+6 \mathrm{x})-\frac{11}{3}\)

∴ \(\mathrm{I}=\int \frac{\frac{5}{6}(2+6 \mathrm{x})-\frac{11}{3}}{1+2 \mathrm{x}+3 \mathrm{x}^2} \mathrm{dx}=\frac{5}{6} \int \frac{2+6 \mathrm{x}}{1+2 \mathrm{x}+3 \mathrm{x}^2} \mathrm{dx}-\frac{11}{3} \int \frac{1}{1+2 \mathrm{x}+3 \mathrm{x}^2} \mathrm{dx}\)

Let \(I_1=\int \frac{2+6 x}{1+2 x+3 x^2} d x\) and \(I_2=\int \frac{1}{1+2 x+3 x^2} d x\)

∴ \(\mathrm{I}=\frac{5}{6} \mathrm{I}_1-\frac{11}{3} \mathrm{I}_2\)……(1)

Now, \(I_1=\int \frac{2+6 x}{1+2 x+3 x^2} d x=\log \left|1+2 x+3 x^2\right|+C_1\)….(2)

(because \(\int \frac{f^{\prime}(x)}{f(x)} d x=\log |f(x)|+C\))

⇒ \(I_2=\int \frac{1}{1+2 x+3 x^2} d x=\int \frac{1}{3\left[x^2+\frac{2 x}{3}+\frac{1}{3}\right]} d x=\frac{1}{3} \int \frac{1}{\left[\left(x+\frac{1}{3}\right)^2+\frac{2}{9}\right]} d x\)

= \(\frac{1}{3} \int \frac{1}{\left(x+\frac{1}{3}\right)^2+\left(\frac{\sqrt{2}}{3}\right)} d x=\frac{1}{3} \cdot \frac{1}{(\sqrt{2} / 3)} \tan ^{-1}\left\{\frac{(x+1 / 3)}{\sqrt{2} / 3}\right\}+C_2\)

⇒ \(I_2=\frac{1}{\sqrt{2}} \tan ^{-1}\left(\frac{3 x+1}{\sqrt{2}}\right)+C_2\)

Using eq. (2) and (3) in eq. (1)

∴ \(\mathrm{I}=\frac{5}{6} \log \left|1+2 \mathrm{x}+3 \mathrm{x}^2\right|+\frac{5}{6} \mathrm{C}_1-\frac{11}{3}\left[\frac{1}{\sqrt{2}} \tan ^{-1}\left(\frac{3 \mathrm{x}+1}{\sqrt{2}}\right)\right]-\frac{11}{3} \mathrm{C}_2\)

= \(\frac{5}{6} \log \left|1+2 \mathrm{x}+3 \mathrm{x}^2\right|-\frac{11}{3 \sqrt{2}} \tan ^{-1}\left(\frac{3 \mathrm{x}+1}{\sqrt{2}}\right)+\mathrm{C}\)

where \(\mathrm{C}=\frac{5}{6} \mathrm{C}_1-\frac{11}{3} \mathrm{C}_2\)

Question 19. \(\int \frac{6 x+7}{\sqrt{(x-5)(x-4)}} d x\)
Solution:

Let \(I=\int \frac{6 x+7}{\sqrt{(x-5)(x-4)}} d x=\int \frac{6 x+7}{\sqrt{x^2-9 x+20}} d x\)

Let \(6 x+7=A \frac{d}{d x}\left(x^2-9 x+20\right)+B \Rightarrow 6 x+7=A(2 x-9)+B\)

Equating the coefficients of x and the constant term, we get

⇒ \(2 \mathrm{~A}=6 \Rightarrow \mathrm{A}=3\)

⇒ \(-9 \mathrm{~A}+\mathrm{B}=7 \Rightarrow \mathrm{B}=7+9 \mathrm{~A}=34\)

⇒ \(6 \mathrm{x}+7=3(2 \mathrm{x}-9)+34\)

∴ \(\mathrm{I}=\int \frac{3(2 \mathrm{x}-9)+34}{\sqrt{\mathrm{x}^2-9 x+20}} d x=3 \int \frac{2 \mathrm{x}-9}{\sqrt{\mathrm{x}^2-9 \mathrm{x}+20}} \mathrm{dx}+34 \int \frac{1}{\sqrt{\mathrm{x}^2-9 \mathrm{x}+20}} \mathrm{dx}\)

Let \(I_1=\int \frac{2 x-9}{\sqrt{x^2-9 x+20}} d x\) and \(I_2=\int \frac{1}{\sqrt{x^2-9 x+20}} d x\)

∴ \(\mathrm{I}=3 \mathrm{I}_1+34 \mathrm{I}_2\)….(1)

Now, \(I_1=\int \frac{2 x-9}{\sqrt{x^2-9 x+20}} d x\)

Put \(\mathrm{x}^2-9 \mathrm{x}+20=\mathrm{t}^2 \Rightarrow(2 \mathrm{x}-9) \mathrm{dx}=2 \mathrm{t} d \mathrm{t}\)

∴ \(I_1=\int \frac{2 t}{t} d t=2 \int 1 d t=2 t+C_1=2 \sqrt{x^2-9 x+20}+C_1\)….(2)

and \(I_2=\int \frac{1}{\sqrt{x^2-9 x+20}} d x=\int \frac{1}{\sqrt{\left(x-\frac{9}{2}\right)^2-\frac{1}{4}}} d x\)

⇒ \(I_2=\log \left|\left(x-\frac{9}{2}\right)+\sqrt{x^2-9 x+20}\right|+C_2\)

Using equations (2) and (3) in (1), we get

I = \(3\left[2 \sqrt{x^2-9 x+20}\right]+3 C_1+34 \log \left[\left(x-\frac{9}{2}\right)+\sqrt{x^2-9 x+20}\right]+34 C_2\)

∴ I = \(6 \sqrt{x^2-9 x+20}+34 \log \left[\left(x-\frac{9}{2}\right)+\sqrt{x^2-9 x+20}\right]+ \) where \(C=3 C_1+34 C_2\)

Question 20. \(\int \frac{\mathrm{x}+2}{\sqrt{4 \mathrm{x}-\mathrm{x}^2}} \mathrm{dx}\)
Solution:

Let \(I=\int \frac{x+2}{\sqrt{4 x-x^2}} d x\)

Let \(\mathrm{x}+2=\mathrm{A} \frac{\mathrm{d}}{\mathrm{dx}}\left(4 \mathrm{x}-\mathrm{x}^2\right)+\mathrm{B} \Rightarrow \mathrm{x}+2=\mathrm{A}(4-2 \mathrm{x})+\mathrm{B}\)

Equating the coefficients of x and constant term on both sides, we get \(-2 \mathrm{~A}=1 \Rightarrow \mathrm{A}=\frac{-1}{2} \Rightarrow 4 \mathrm{~A}+\mathrm{B}=2 \Rightarrow \mathrm{B}=2-4 \mathrm{~A}=4 \Rightarrow(\mathrm{x}+2)=-\frac{1}{2}(4-2 \mathrm{x})+4\)

∴ \(\mathrm{I}=\int \frac{-\frac{1}{2}(4-2 \mathrm{x})+4}{\sqrt{4 \mathrm{x-x^{2 }}}} \mathrm{dx}=-\frac{1}{2} \int \frac{4-2 \mathrm{x}}{\sqrt{4 \mathrm{x}-\mathrm{x}^2}} \mathrm{dx}+4 \int \frac{1}{\sqrt{4 \mathrm{x}-\mathrm{x}^2}} d x\)

Let \(I_1=\int \frac{4-2 x}{\sqrt{4 x-x^2}} d x\) and \(I_2=\int \frac{1}{\sqrt{4 x-x^2}} d x\)

∴ I = \(-\frac{1}{2} I_1+4 I_2\)….(1)

Now, \(I_1=\int \frac{4-2 x}{\sqrt{4 x-x^2}} d x\)

Put \(4 \mathrm{x}-\mathrm{x}^2=\mathrm{t}^2 \Rightarrow(4-2 \mathrm{x}) \mathrm{dx}=2 \mathrm{tdt}\)

∴ \(I_1=\int \frac{2 t}{t} d t=2 \int 1 d t=2 t+C_1=2 \sqrt{4 x-x^2}+C_1\)….(2)

Again, \(I_2=\int \frac{1}{\sqrt{4 x-x^2}} d x=\int \frac{1}{\sqrt{4-(x-2)^2}} d x\)

= \(\int \frac{1}{\sqrt{(2)^2-(x-2)^2}} d x=\sin ^{-1}\left(\frac{x-2}{2}\right)+C_2\)

Using equations (2) and (3) in (1), we get

I = \(-\frac{1}{2}\left(2 \sqrt{4 x-x^2}\right)-\frac{1}{2} C_1+4 \sin ^{-1}\left(\frac{x-2}{2}\right)+4 C_2\)

= \(-\sqrt{4 x-x^2}+4 \sin ^{-1}\left(\frac{x-2}{2}\right)+C\) where C=4 \(C_2-\frac{1}{2} C_1\)

Question 21. \(\int \frac{\mathrm{x}+2}{\sqrt{\mathrm{x}^2+2 \mathrm{x}+3}} \mathrm{dx}\)
Solution:

Let \(I=\int \frac{x+2}{\sqrt{x^2+2 x+3}} d x=\frac{1}{2} \int \frac{2(x+2)}{\sqrt{x^2+2 x+3}} d x=\frac{1}{2} \int \frac{(2 x+2)+2}{\sqrt{x^2+2 x+3}} d x\)

= \(\frac{1}{2} \int \frac{2 \mathrm{x}+2}{\sqrt{\mathrm{x}^2+2 \mathrm{x}+3}} \mathrm{dx}+\frac{1}{2} \int \frac{2}{\sqrt{\mathrm{x}^2+2 \mathrm{x}+3}} \mathrm{dx}=\frac{1}{2} \int \frac{2 \mathrm{x}+2}{\sqrt{\mathrm{x}^2+2 \mathrm{x}+3}} \mathrm{dx}+\int \frac{1}{\sqrt{\mathrm{x}^2+2 \mathrm{x}+3}} \mathrm{dx}\)

Let \(I_1=\int \frac{2 x+2}{\sqrt{x^2+2 x+3}} d x\) and \(I_2=\int \frac{1}{\sqrt{x^2+2 x+3}} d x\)

∴ I=\(\frac{1}{2} I_1+I_2\)…..(1)

Now, \(\mathrm{I}_1=\int \frac{2 \mathrm{x}+2}{\sqrt{\mathrm{x}^2+2 \mathrm{x}+3}} \mathrm{dx}\)

Put \(\mathrm{x}^2+2 \mathrm{x}+3=\mathrm{t}^2 \Rightarrow(2 \mathrm{x}+2) \mathrm{dx}=2 \mathrm{tdt}\)

∴ \(I_1=\int \frac{2 t}{t} d t=2 \int 1 d t=2 t+C_1 \Rightarrow I_1=2 \sqrt{x^2+2 x+3}+C_1\)…..(2)

Again, \(\mathrm{J}_2=\int \frac{1}{\sqrt{\mathrm{x}^2+2 \mathrm{x}+3}} \mathrm{dx}=\int \frac{1}{\sqrt{(\mathrm{x}+1)^2+(\sqrt{2})^2}} \mathrm{dx}\)

∴ \(I_2=\log \left|(x+1)+\sqrt{x^2+2 x+3}\right|+C_2\)…..(3)

Using equations (2) and (3) in (1), we get

I = \(\frac{1}{2}\left[2 \sqrt{x^2+2 x+3}\right]+\frac{1}{2} C_1+\log \left|(x+1)+\sqrt{x^2+2 x+3}\right|+C_2\)

= \(\sqrt{x^2+2 x+3}+\log \left|(x+1)+\sqrt{x^2+2 x+3}\right|+C\) where C = \(\frac{1}{2} C_1+C_2\)

Question 22. \(\int \frac{x+3}{x^2-2 x-5} d x\)
Solution:

Let \(I=\int \frac{x+3}{x^2-2 x-5} d x\)

Let \(\mathrm{x}+3=\mathrm{A} \frac{\mathrm{d}}{\mathrm{dx}}\left(\mathrm{x}^2-2 \mathrm{x}-5\right)+\mathrm{B} \Rightarrow \mathrm{x}+3=\mathrm{A}(2 \mathrm{x}-2)+\mathrm{B}\)

Equating the coefficients of x and the constant term on both sides, we obtain

2\(\mathrm{~A}=1 \Rightarrow \mathrm{A}=\frac{1}{2} \Rightarrow-2 \mathrm{~A}+\mathrm{B}=3 \Rightarrow \mathrm{B}=3+2 \mathrm{~A}=4\)

therefore \((\mathrm{x}+3)=\frac{1}{2}(2 \mathrm{x}-2)+4\)

∴ \(\mathrm{I}=\int \frac{\frac{1}{2}(2 \mathrm{x}-2)+4}{\mathrm{x}^2-2 \mathrm{x}-5} \mathrm{dx}=\frac{1}{2} \int \frac{(2 \mathrm{x}-2)}{\mathrm{x}^2-2 \mathrm{x}-5} \mathrm{dx}+4 \int \frac{1}{\mathrm{x}^2-2 \mathrm{x}-5} \mathrm{dx}\)

Let \(I_1=\int \frac{2 x-2}{x^2-2 x-5} d x\) and \(I_2=\int \frac{1}{x^2-2 x-5} d x\)

∴ \(\mathrm{I}=\frac{1}{2} \mathrm{I}_1+4 \mathrm{I}_2\)…..(1)

Now, \(I_1=\int \frac{2 x-2}{x^2-2 x-5} d x\)

Put \(x^2-2 x-5=t \Rightarrow(2 x-2) d x=d t=\int \frac{d t}{t}=\log |t|+C_1\)

∴ \(I_1=\log \left|x^2-2 x-5\right|+C_1\)….(2)

Again, \(I_2=\int \frac{1}{x^2-2 x-5} d x=\int \frac{1}{\left(x^2-2 x+1\right)-6} d x=\int \frac{1}{(x-1)^2-(\sqrt{6})^2} d x\)

= \(\frac{1}{2 \sqrt{6}} \log \left(\frac{x-1-\sqrt{6}}{x-1+\sqrt{6}}\right)+C_2\)….(3)

Using equations (2) and (3) in (1), we get:

I = \(\frac{1}{2} \log \left|x^2-2 x-5\right|+\frac{1}{2} C_1+\frac{4}{2 \sqrt{6}} \log \left|\frac{x-1-\sqrt{6}}{x-1+\sqrt{6}}\right|+4 C_2\)

= \(\frac{1}{2} \log \left|x^2-2 x-5\right|+\frac{2}{\sqrt{6}} \log \left|\frac{x-1-\sqrt{6}}{x-1+\sqrt{6}}\right|+C \text { where } C=\frac{1}{2} C_1+4 C_2\)

Question 23. \(\int \frac{5 x+3}{\sqrt{x^2+4 x+10}} d x\)
Solution:

Let \(I=\int \frac{5 x+3}{\sqrt{x^2+4 x+10}} d x\)

Let \(5 \mathrm{x}+3=\mathrm{A} \frac{\mathrm{d}}{\mathrm{dx}}\left(\mathrm{x}^2+4 \mathrm{x}+10\right)+\mathrm{B} \Rightarrow 5 \mathrm{x}+3=\mathrm{A}(2 \mathrm{x}+4)+\mathrm{B}\)

Equating the coefficients of x and the constant term, we get

⇒ \(2 \mathrm{~A}=5 \Rightarrow \mathrm{A}=\frac{5}{2}, 4 \mathrm{~A}+\mathrm{B}=3\)

⇒ \(\mathrm{B}=3-4\left(\frac{5}{2}\right)=-7 \Rightarrow 5 \mathrm{x}+3=\frac{5}{2}(2 \mathrm{x}+4)-7\)

∴ \(\mathrm{I}=\int \frac{\frac{5}{2}(2 \mathrm{x}+4)-7}{\sqrt{\mathrm{x}^2+4 \mathrm{x}+10}} \mathrm{dx}=\frac{5}{2} \int \frac{2 \mathrm{x}+4}{\sqrt{\mathrm{x}^2+4 \mathrm{x}+10}} \mathrm{dx}-7 \int \frac{1}{\sqrt{\mathrm{x}^2+4 \mathrm{x}+10}} \mathrm{dx}\)

Let, \(I_1=\int \frac{2 x+4}{\sqrt{x^2+4 x+10}} d x\) and \(I_2=\int \frac{1}{\sqrt{x^2+4 x+10}} d x\)

∴ \(\mathrm{I}=\frac{5}{2} \mathrm{I}_1-7 \mathrm{I}_2\)

Now, \(\mathrm{I}_1=\int \frac{2 \mathrm{x}+4}{\sqrt{\mathrm{x}^2+4 \mathrm{x}+10}} \mathrm{dx}\)

Put \(x^2+4 x+10=t^2 \Rightarrow(2 x+4) d x=2 t d t\)

∴ \(I_1=\int \frac{2 t}{t} d t=2 \int 1 d t=2 t+C_1=2 \sqrt{x^2+4 x+10}+C_1\)

Again, \(I_2=\int \frac{1}{\sqrt{x^2+4 x+10}} d x=\int \frac{1}{\sqrt{\left(x^2+4 x+4\right)+6}} d x=\int \frac{1}{\sqrt{(x+2)^2+(\sqrt{6})^2}} d x\)

= \(\log \left|(\mathrm{x}+2)+\sqrt{\mathrm{x}^2+4 \mathrm{x}+10}\right|+\mathrm{C}_2\)

Using equations (2) and (3) in (1), we get

I = \(\frac{5}{2}\left[2 \sqrt{x^2+4 x+10}\right]+\frac{5}{2} C_1-7 \log \left|(x+2)+\sqrt{x^2+4 x+10}\right|-7 C_2\),

= \(5 \sqrt{x^2+4 x+10}-7 \log \left|(x+2)+\sqrt{x^2+4 x+10}\right|+C ; \text { where } C=\frac{5}{2} C_1-7 C_2\)

Choose The Correct Answer In The Following

Question 24. \(\int \frac{\mathrm{dx}}{\mathrm{x}^2+2 \mathrm{x}+2}\) equals ?

  1. \(x \tan ^{-1}(x+1)+C\)
  2. \(\tan ^{-1}(x+1)+C\)
  3. \((x+1) \tan ^{-1} x+C\)
  4. \(\tan ^{-1} x+C\)

Solution: 2. \(\tan ^{-1}(x+1)+C\)

Let \(\tan ^{-1}(x+1)+C\)

I = \(\int \frac{\mathrm{dx}}{\mathrm{x}^2+2 \mathrm{x}+2}\)

= \(\int \frac{\mathrm{dx}}{\left(\mathrm{x}^2+2 \mathrm{x}+1\right)+1}=\int \frac{1}{(\mathrm{x}+1)^2+(1)^2} \mathrm{dx}\)

= \(\left[\tan ^{-1}(\mathrm{x}+1)\right]+C\)

⇒ \(\tan ^{-1}(x+1)+C\)

Hence, the correct answer is (2).

Question 25. \(\int \frac{\mathrm{dx}}{\sqrt{9 \mathrm{x}-4 \mathrm{x}^2}}\) { equals?

  1. \(\frac{1}{9} \sin ^{-1}\left(\frac{9 x-8}{8}\right)+C\)
  2. \(\frac{1}{2} \sin ^{-1}\left(\frac{8 x-9}{9}\right)+C\)
  3. \(\frac{1}{3} \sin ^{-1}\left(\frac{9 x-8}{8}\right)+C\)
  4. \(\frac{1}{2} \sin ^{-1}\left(\frac{9 x-8}{9}\right)+C\)

Solution: 2. \(\frac{1}{2} \sin ^{-1}\left(\frac{8 x-9}{9}\right)+C\)

Let I = \(\int \frac{d x}{\sqrt{9 x-4 x^2}}=\int \frac{1}{\sqrt{-4\left(x^2-\frac{9}{4} x\right)}} d x=\int \frac{1}{\sqrt{-4\left(x^2-\frac{9}{4} x+\frac{81}{64}-\frac{81}{64}\right)}} d x\)

= \(\int \frac{1}{\left.\sqrt{-4\left[\left(x-\frac{9}{8}\right)^2-\left(\frac{9}{8}\right)^2\right.}\right]} d x=\frac{1}{2} \int \frac{1}{\sqrt{\left(\frac{9}{8}\right)^2-\left(x-\frac{9}{8}\right)^2}} d x\)

= \(\frac{1}{2}\left[\sin ^{-1}\left(\frac{x-\frac{9}{8}}{\frac{9}{8}}\right)\right]+C=\frac{1}{2} \sin ^{-1}\left(\frac{8 x-9}{9}\right)+C\)

Hence, the correct answer is (2).

Integrals Exercise 7.5

Integrate The Rational Functions

Question 1. \(\int \frac{x}{(x+1)(x+2)} d x\)
Solution:

Let \(I=\int \frac{x}{(x+1)(x+2)} d x\)

Let \(\frac{x}{(x+1)(x+2)}=\frac{A}{(x+1)}+\frac{B}{(x+2)}\)

x=A(x+2)+B(x+1)…..(1)

In equation (1)

Put x=-1 ⇒ A=-1

x=-2 ⇒ -B=-2 ⇒ B=2

∴ \(\frac{x}{(x+1)(x+2)}=\frac{-1}{(x+1)}+\frac{2}{(x+2)}\)

∴ I = \(\int \frac{-1}{(x+1)} d x+\int \frac{2}{(x+2)} d x\)

= \(-\log |x+1|+2 \log |x+2|+C=\log (x+2)^2-\log |x+1|+C=\log \left|\frac{(x+2)^2}{(x+1)}\right|+C\)

Question 2. \(\int \frac{1}{x^2-9} d x\)
Solution:

Let \(I=\int \frac{1}{x^2-9} d x=\int \frac{1}{(x+3)(x-3)} d x\)

Let \(\frac{1}{(x+3)(x-3)}=\frac{A}{(x+3)}+\frac{B}{(x-3)} \Rightarrow 1=A(x-3)+B(x+3)\)…..(1)

From equation (1)

Put \(x=-3 \Rightarrow-6 A=1 \Rightarrow A=\frac{-1}{6}\) and \(x=3 \Rightarrow 6 B=1 \Rightarrow B=\frac{1}{6}\)

∴ \(\frac{1}{(x+3)(x-3)}=\frac{-1}{6(x+3)}+\frac{1}{6(x-3)}\)

∴ I = \(\int\left(\frac{-1}{6(x+3)}+\frac{1}{6(x-3)}\right) d x\)

= \(-\frac{1}{6} \log |x+3|+\frac{1}{6} \log |x-3|+C=\frac{1}{6} \log \left|\frac{(x-3)}{(x+3)}\right|+C\)

Question 3. \(\int \frac{3 x-1}{(x-1)(x-2)(x-3)} d x\)
Solution:

Let \(I=\int \frac{3 x-1}{(x-1)(x-2)(x-3)} d x\)

Let \(\frac{3 x-1}{(x-1)(x-2)(x-3)}=\frac{A}{(x-1)}+\frac{B}{(x-2)}+\frac{C}{(x-3)}\)

3\(\mathrm{x}-1=\mathrm{A}(\mathrm{x}-2)(\mathrm{x}-3)+\mathrm{B}(\mathrm{x}-1)(\mathrm{x}-3)+\mathrm{C}(\mathrm{x}-1)(\mathrm{x}-2)\)….(1)

Put x=1,2, and 3 in equation (1), we get

A=1, \(\mathrm{~B}=-5\), and \(\mathrm{C}=4\) respectively

Now, \(\int \frac{3 x-1}{(x-1)(x-2)(x-3)} d x=\int\left(\frac{1}{(x-1)}-\frac{5}{(x-2)}+\frac{4}{(x-3)}\right) d x\)

= \(\log |\mathrm{x}-1|-5 \log |\mathrm{x}-2|+4 \log |\mathrm{x}-3|+\mathrm{C}\)

Question 4. \(\int \frac{x}{(x-1)(x-2)(x-3)} d x\)
Solution:

Let \(\mathrm{I}=\int \frac{\mathrm{x}}{(\mathrm{x}-1)(\mathrm{x}-2)(\mathrm{x}-3)} \mathrm{dx}\)

Let \(\frac{x}{(x-1)(x-2)(x-3)}=\frac{A}{(x-1)}+\frac{B}{(x-2)}+\frac{C}{(x-3)}\)

x= \(\mathrm{A}(\mathrm{x}-2)(\mathrm{x}-3)+\mathrm{B}(\mathrm{x}-1)(\mathrm{x}-3)+\mathrm{C}(\mathrm{x}-1)(\mathrm{x}-2)\)……(1)

Put x=1,2 and 3 in equation (1), and we get \(\mathrm{A}=\frac{1}{2}, \mathrm{~B}=-2\), and \(\mathrm{C}=\frac{3}{2}\) respectively

∴ \(\frac{x}{(x-1)(x-2)(x-3)}=\frac{1}{2(x-1)}-\frac{2}{(x-2)}+\frac{3}{2(x-3)}\)

∴ \(I=\int\left\{\frac{1}{2(x-1)}-\frac{2}{(x-2)}+\frac{3}{2(x-3)}\right\} d x=\frac{1}{2} \log |x-1|-2 \log |x-2|+\frac{3}{2} \log |x-3|+C\)

Question 5. \(\int \frac{2 x}{x^2+3 x+2} d x\)
Solution:

Let \(I=\int \frac{2 x}{x^2+3 x+2} d x, \frac{2 x}{x^2+3 x+2}=\frac{2 x}{(x+1)(x+2)}=\frac{A}{(x+1)}+\frac{B}{(x+2)}\)

⇒ \(2 \mathrm{x}=\mathrm{A}(\mathrm{x}+2)+\mathrm{B}(\mathrm{x}+1)\)….(1)

Put x=-1 and -2 equation (1), we get A=-2 and B=4 respectively

⇒ \(\frac{2 x}{(x+1)(x+2)}=\frac{-2}{(x+1)}+\frac{4}{(x+2)}\)

⇒ I = \(\int\left\{\frac{4}{(x+2)}-\frac{2}{(x+1)}\right\} d x=4 \log |x+2|-2 \log |x+1|+C\)

Question 6. \(\int \frac{1-x^2}{x(1-2 x)} d x\)
Solution:

Let \(I=\int \frac{1-x^2}{x(1-2 x)} d x\)

Given integrand is an improper rational function. So, on dividing we get \(\frac{1-x^2}{x(1-2 x)}=\frac{1}{2}+\frac{1}{2}\left(\frac{2-x}{x(1-2 x)}\right)\)….(1)

Let \(\frac{2-x}{x(1-2 x)}=\frac{A}{x}+\frac{B}{(1-2 x)} \Rightarrow 2-x=A(1-2 x)+B x\)

Put x=0 and \(\frac{1}{2}\) in equation (1), we get A=2 and B=3 respectively

∴ \(\frac{2-x}{x(1-2 x)}=\frac{2}{x}+\frac{3}{1-2 x}\)

∴ I = \(\int\left\{\frac{1}{2}+\frac{1}{2}\left(\frac{2}{x}+\frac{3}{1-2 x}\right)\right\} d x\)

= \(\frac{x}{2}+\log |x|+\frac{3}{2(-2)} \log |1-2 x|+C=\frac{x}{2}+\log |x|-\frac{3}{4} \log |1-2 x|+C\)

Question 7. \(\int \frac{x}{\left(x^2+1\right)(x-1)} d x\)
Solution:

Let \(I=\int \frac{x}{\left(x^2+1\right)(x-1)} d x\)

Put \(\frac{x}{\left(x^2+1\right)(x-1)}=\frac{A x+B}{\left(x^2+1\right)}+\frac{C}{(x-1)}, x=(A x+B)(x-1)+C\left(x^2+1\right)\)….(1)

In eq. (1), Put x=1 \(\Rightarrow C=\frac{1}{2}\)

Equating the coefficients of x² and the constant term, we get

A+C = \(0 \Rightarrow A=-C=-\frac{1}{2},-B+C=0 \Rightarrow B=C=\frac{1}{2}\)

∴ \(\frac{x}{\left(x^2+1\right)(x-1)}=\frac{\left(-\frac{1}{2} x+\frac{1}{2}\right)}{x^2+1}+\frac{\frac{1}{2}}{(x-1)}\)

∴ I = \(-\frac{1}{2} \int \frac{x}{x^2+1} d x+\frac{1}{2} \int \frac{1}{x^2+1} d x+\frac{1}{2} \int \frac{1}{x-1}\)

= \(-\frac{1}{4} \int \frac{2 x}{x^2+1} d x+\frac{1}{2} \tan ^{-1} x+\frac{1}{2} \log |x-1|+C\)

= \(-\frac{1}{4} \log \left|\mathrm{x}^2+1\right|+\frac{1}{2} \tan ^{-1} \mathrm{x}+\frac{1}{2} \log |\mathrm{x}-1|+\mathrm{C}\)

= \(\frac{1}{2} \log |\mathrm{x}-1|-\frac{1}{4} \log \left|\mathrm{x}^2+1\right|+\frac{1}{2} \tan ^{-1} \mathrm{x}+\mathrm{C}\)

Question 8. \(\int \frac{x}{(x-1)^2(x+2)} d x\)
Solution:

Let \(I=\int \frac{x}{(x-1)^2(x+2)} d x\)

Let \(\frac{x}{(x-1)^2(x+2)}=\frac{A}{(x-1)}+\frac{B}{(x-1)^2}+\frac{C}{(x+2)}\)

x = \(=\mathrm{A}(\mathrm{x}-1)(\mathrm{x}+2)+\mathrm{B}(\mathrm{x}+2)+\mathrm{C}(\mathrm{x}-1)^2\)

In equation (1)

Put x = \(1 \Rightarrow \mathrm{B}=\frac{1}{3}, \mathrm{x}=-2 \Rightarrow \mathrm{C}=-\frac{2}{9}\)

On equating the coefficients of \(\mathrm{x}^2, \mathrm{~A}+\mathrm{C}=0 \Rightarrow \mathrm{A}=-\mathrm{C}=\frac{2}{9}\)

∴ \(\frac{x}{(x-1)^2(x+2)}=\frac{2}{9(x-1)}+\frac{1}{3(x-1)^2}-\frac{2}{9(x+2)}\)

∴ I = \(\frac{2}{9} \int \frac{1}{(x-1)} d x+\frac{1}{3} \int \frac{1}{(x-1)^2} d x-\frac{2}{9} \int \frac{1}{(x+2)} d x\)

= \(\frac{2}{9} \log |x-1|+\frac{1}{3}\left(\frac{-1}{x-1}\right)-\frac{2}{9} \log |x+2|+C=\frac{2}{9} \log \left|\frac{x-1}{x+2}\right|-\frac{1}{3(x-1)}+C\)

Question 9. \(\int \frac{3 x+5}{x^3-x^2-x+1} d x\)
Solution:

Let \(I=\int \frac{3 x+5}{x^3-x^2-x+1} d x\)

Let \(\frac{3 x+5}{(x-1)^2(x+1)}=\frac{A}{(x-1)}+\frac{B}{(x-1)^2}+\frac{C}{(x+1)}\)

3 \(\mathrm{x}+5=\mathrm{A}(\mathrm{x}-1)(\mathrm{x}+1)+\mathrm{B}(\mathrm{x}+1)+\mathrm{C}(\mathrm{x}-1)^2\)…..(1)

Put x=1 and x=-1 in equation (1), we get B=4 and C = \(\frac{1}{2}\)

Equating the coefficients of \(x^2\) we get A+C=0 ⇒ A=-C=\(-\frac{1}{2}\)

∴ \(\frac{3 x+5}{(x-1)^2(x+1)}=\frac{-1}{2(x-1)}+\frac{4}{(x-1)^2}+\frac{1}{2(x+1)}\)

∴ I = \(-\frac{1}{2} \int \frac{1}{x-1} d x+4 \int \frac{1}{(x-1)^2} d x+\frac{1}{2} \int \frac{1}{(x+1)} d x\)

= \(-\frac{1}{2} \log |x-1|+4\left(\frac{-1}{x-1}\right)+\frac{1}{2} \log |x+1|+C\)

= \(\frac{1}{2} \log \left|\frac{x+1}{x-1}\right|-\frac{4}{(x-1)}+C\)

Question 10. \(\int \frac{2 \mathrm{x}-3}{\left(\mathrm{x}^2-1\right)(2 \mathrm{x}+3)} \mathrm{dx}\)
Solution:

Let I = \(\int \frac{2 x-3}{\left(x^2-1\right)(2 x+3)} d x=\int \frac{2 x-3}{(x+1)(x-1)(2 x+3)} \cdot d x\)

Let \(\frac{2 x-3}{(x+1)(x-1)(2 x+3)}=\frac{A}{(x+1)}+\frac{B}{(x-1)}+\frac{C}{(2 x+3)}\)

⇒ \((2 \mathrm{x}-3)=\mathrm{A}(\mathrm{x}-1)(2 \mathrm{x}+3)+\mathrm{B}(\mathrm{x}+1)(2 \mathrm{x}+3)+\mathrm{C}(\mathrm{x}+1)(\mathrm{x}-1)\)….(1)

Put x=-1,1 and \(-\frac{3}{2}\) in eq. (1), we get

∴ \(\mathrm{A}=\frac{5}{2}, \mathrm{~B}=-\frac{1}{10}, \mathrm{C}=-\frac{24}{5}\) respectively

∴ \(\frac{2 x-3}{(x+1)(x-1)(2 x+3)}=\frac{5}{2(x+1)}-\frac{1}{10(x-1)}-\frac{24}{5(2 x+3)} \)

∴ \(\mathrm{I}=\frac{5}{2} \int \frac{1}{(\mathrm{x}+1)} \mathrm{dx}-\frac{1}{10} \int \frac{1}{\mathrm{x}-1} \mathrm{dx}-\frac{24}{5} \int \frac{1}{(2 \mathrm{x}+3)} \mathrm{dx}\)

= \(\frac{5}{2} \log |\mathrm{x}+1|-\frac{1}{10} \log |\mathrm{x}-1|-\frac{24}{5 \times 2} \log |2 \mathrm{x}+3|+\mathrm{C}\)

= \(\frac{5}{2} \log |x+1|-\frac{1}{10} \log |x-1|-\frac{12}{5} \log |2 x+3|+C\)

Question 11. \(\int \frac{5 x}{(x+1)\left(x^2-4\right)} d x\)
Solution:

Let \(I=\int \frac{5 x}{(x+1)\left(x^2-4\right)} d x=\int \frac{5 x}{(x+1)(x+2)(x-2)} d x\)

Let \(\frac{5 x}{(x+1)(x+2)(x-2)}=\frac{A}{(x+1)}+\frac{B}{(x+2)}+\frac{C}{(x-2)}\)

5\(\mathrm{x}=\mathrm{A}(\mathrm{x}+2)(\mathrm{x}-2)+\mathrm{B}(\mathrm{x}+1)(\mathrm{x}-2)+\mathrm{C}(\mathrm{x}+1)(\mathrm{x}+2)\)…..(1)

Put x=-1,-2, and 2 in equation (1), we get

∴ \(\mathrm{A}=\frac{5}{3}, \mathrm{~B}=-\frac{5}{2}\) and \(\mathrm{C}=\frac{5}{6}\) respectively

∴ \(\frac{5 x}{(x+1)(x+2)(x-2)}=\frac{5}{3(x+1)}-\frac{5}{2(x+2)}+\frac{5}{6(x-2)}\)

I = \(\frac{5}{3} \int \frac{1}{(x+1)} d x-\frac{5}{2} \int \frac{1}{(x+2)} d x+\frac{5}{6} \int \frac{1}{(x-2)} d x\)

= \(\frac{5}{3} \log |x+1|-\frac{5}{2} \log |x+2|+\frac{5}{6} \log |x-2|+C\)

Question 12. \(\int \frac{x^3+x+1}{x^2-1} d x\)
Solution:

Let \(\mathrm{I}=\int \frac{\mathrm{x}^3+\mathrm{x}+1}{\mathrm{x}^2-1} \mathrm{dx}\)

Given integrand is an improper rational function.

So, on dividing \(\left(x^3+x+1\right)\) by \(x^2-1\), we get \(\frac{x^3+x+1}{x^2-1}=x+\frac{2 x+1}{x^2-1}\)

Let \(\frac{2 x+1}{x^2-1}=\frac{A}{(x+1)}+\frac{B}{(x-1)} \Rightarrow 2 x+1=A(x-1)+B(x+1)\)

Put x=-1 and 1 in equation (1), we get \(A=\frac{1}{2}\) and \(B=\frac{3}{2}\)

∴ \(\frac{x^3+x+1}{x^2-1}=x+\frac{1}{2(x+1)}+\frac{3}{2(x-1)}\)

∴ I = \(\int x d x+\frac{1}{2} \int \frac{1}{(x+1)} d x+\frac{3}{2} \int \frac{1}{(x-1)} d x=\frac{x^2}{2}+\frac{1}{2} \log |x+1|+\frac{3}{2} \log |x-1|+C\)

Question 13. \(\int \frac{2}{(1-\mathrm{x})\left(1+\mathrm{x}^2\right)} \mathrm{dx}\)
Solution:

Let \(I=\int \frac{2}{(1-x)\left(1+x^2\right)} d x=\int \frac{2}{(1-x)\left(1+x^2\right)} d x\)

Let \(\frac{2}{(1-x)\left(1+x^2\right)}=\frac{A}{(1-x)}+\frac{B x+C}{\left(1+x^2\right)}\)

2 = \(A\left(1+x^2\right)+(B x+C)(1-x)\)….(1)

In equation (1), Put x=1 ⇒ A=1

Now, on equating the coefficient of \(x^2\) and constant term, we get

∴ \(\mathrm{A}-\mathrm{B}=0 \Rightarrow \mathrm{B}=\mathrm{A}=1\)

∴ \(\mathrm{~A}+\mathrm{C}=2 \Rightarrow \mathrm{C}=2-\mathrm{A}=1\)

∴ \(\frac{2}{(1-\mathrm{x})\left(1+\mathrm{x}^2\right)}=\frac{1}{1-\mathrm{x}}+\frac{\mathrm{x}+1}{1+\mathrm{x}^2}\)

⇒ \(\mathrm{I}=\int \frac{1}{1-\mathrm{x}} \mathrm{dx}+\int \frac{\mathrm{x}}{1+\mathrm{x}^2} \mathrm{dx}+\int \frac{1}{1+\mathrm{x}^2} \mathrm{dx}\)

= \(-\int \frac{1}{\mathrm{x}-1} \mathrm{dx}+\frac{1}{2} \int \frac{2 \mathrm{x}}{1+\mathrm{x}^2} \mathrm{dx}+\int \frac{1}{1+\mathrm{x}^2} \mathrm{dx}\)

= \(-\log |\mathrm{x}-1|+\frac{1}{2} \log \left|1+\mathrm{x}^2\right|+\tan ^{-1} \mathrm{x}+\mathrm{C}\)

Question 14. \(\int \frac{3 x-1}{(x+2)^2} d x\)
Solution:

Let \(I=\int \frac{3 x-1}{(x+2)^2} d x=\int \frac{3 x-1}{(x+2)^2} d x\)

Let \(\frac{3 x-1}{(x+2)^2}=\frac{A}{(x+2)}+\frac{B}{(x+2)^2} \Rightarrow 3 x-1=A(x+2)+B\)

In eq. (1), Put x=-2 ⇒ B=-7

On equating the coefficients of x, we get A=3

∴ \(\frac{3 x-1}{(x+2)^2}=\frac{3}{(x+2)}-\frac{7}{(x+2)^2}\)

⇒ I = \(3 \int \frac{1}{(x+2)} d x-7 \int \frac{1}{(x+2)^2} d x=3 \log |x+2|-7\left(\frac{-1}{(x+2)}\right)+C=3 \log |x+2|+\frac{7}{(x+2)}+C\)

Question 15. \(\int \frac{1}{\mathrm{x}^4-1} \mathrm{dx}\)
Solution:

Let \(I=\int \frac{1}{x^4-1} d x \Rightarrow \frac{1}{\left(x^4-1\right)}=\frac{1}{\left(x^2-1\right)\left(x^2+1\right)}=\frac{1}{(x+1)(x-1)\left(1+x^2\right)}\)

Let \(\frac{1}{(x+1)(x-1)\left(x^2+1\right)}=\frac{A}{(x+1)}+\frac{B}{(x-1)}+\frac{C x+D}{\left(x^2+1\right)}\)

I = \(A(x-1)\left(x^2+1\right)+B(x+1)\left(x^2+1\right)+(C x+D)\left(x^2-1\right)\)….(1)

In equation (1)

Put x=-1 and 1 we get \(A=-\frac{1}{4}\) and \(B=\frac{1}{4}\)

On equating the coefficients of x³ and the constant term, we get

A+B+C =0 ⇒ C=-(A+B)=0

-A+B-D=1 ⇒ D=-A+B-1= \(-\frac{1}{2}\)

∴ \(\frac{1}{x^4-1}=\frac{-1}{4(x+1)}+\frac{1}{4(x-1)}-\frac{1}{2\left(x^2+1\right)}\)

∴ I = \(-\frac{1}{4} \int \frac{1}{x+1} d x+\frac{1}{4} \int \frac{1}{x-1} d x-\frac{1}{2} \int \frac{1}{x^2+1} d x\)

= \(-\frac{1}{4} \log |x+1|+\frac{1}{4} \log |x-1|-\frac{1}{2} \tan ^{-1} x+C=\frac{1}{4} \log \left|\frac{x-1}{x+1}\right|-\frac{1}{2} \tan ^{-1} x+C\)

Question 16. \(\int \frac{1}{x\left(x^n+1\right)} d x\)
Solution:

Let \(\mathrm{I}=\int \frac{1}{\mathrm{x}\left(\mathrm{x}^{\mathrm{n}}+1\right)} \mathrm{dx}\)

⇒ \(\frac{1}{\mathrm{x}\left(\mathrm{x}^n+1\right)}=\frac{\mathrm{x}^{n-1}}{\mathrm{x}^n\left(\mathrm{x}^n+1\right)}\) (because Multiplying numerator and denominator by \(\mathrm{x}^{n-1}\))

Put \(\mathrm{x}^{\mathrm{t}}=\mathrm{t} \Rightarrow \mathrm{nx}^{\mathrm{s}-1} \mathrm{dx}=\mathrm{dt}\)

∴ I = \(\int \frac{x^{n-1}}{x^n\left(x^n+1\right)} d x=\frac{1}{n} \int \frac{1}{t(t+1)} d t\)

⇒ \(\frac{1}{n} \int\left\{\frac{1}{t}-\frac{1}{(t+1)}\right\} d x=\frac{1}{n}[\log |t|-\log |t+1|]+C\)

= \(\frac{1}{n}\left[\log \left|x^n\right|-\log \left|x^n+1\right|\right]+C=\frac{1}{n} \log \left|\frac{x^n}{x^n+1}\right|+C\)

Alternate :

I = \(\int \frac{1}{x\left(x^n+1\right)} d x=\int \frac{1}{x^{n+1}\left(1+x^{-n}\right)} d x\)

Put \(1+x^{-n}=t \Rightarrow-n x^{-a-1} d x=d t \Rightarrow \frac{1}{x^{n+1}} d x=-\frac{d t}{n}\)

∴ I = \(-\frac{1}{n} \int_t^1 \frac{d t}{t}=-\frac{1}{n} \log |t|+C=-\frac{1}{n} \log \left|1+x^{-n}\right|+C=\frac{1}{n} \log \left|\frac{x^n}{x^n+1}\right|+C\)

Question 17. \(\int \frac{\cos x}{(1-\sin x)(2-\sin x)} d x\)
Solution:

Let I = \(\int \frac{\cos x}{(1-\sin x)(2-\sin x)} d x\)

Put sin x=t ⇒ cos x dx = dt

∴ I = \(\int \frac{d t}{(1-t)(2-t)}\)

Let \(\frac{1}{(1-t)(2-t)}=\frac{A}{(1-t)}+\frac{B}{(2-t)}\)

I = \(\mathrm{A}(2-\mathrm{t})+\mathrm{B}(1-\mathrm{t})\)….(1)

Put t=1 and t=2 in equation (1), we get A=1 and B=-1 respectively

∴ \(\frac{1}{(1-t)(2-t)}=\frac{1}{(1-t)}-\frac{1}{(2-t)}\)

⇒ \(I=\int\left\{\frac{1}{1-t}-\frac{1}{(2-t)}\right\} d x=-\log |1-t|+\log |2-t|+C\)

= \(\log \left|\frac{2-t}{1-t}\right|+C=\log \left|\frac{2-\sin x}{1-\sin x}\right|+C\)

Question 18. \(\int \frac{\left(x^2+1\right)\left(x^2+2\right)}{\left(x^2+3\right)\left(x^2+4\right)} d x\)
Solution:

Let \(I=\int \frac{\left(x^2-1\right)\left(x^2+2\right)}{\left(x^2+3\right)\left(x^2+4\right)} d x, x^2=y\), then \(\frac{(y+1)(y+2)}{(y+3)(y+4)}=1+\frac{(-4 y-10)}{(y+3)(y+4)}=1+\frac{A}{y+3}+\frac{B}{y+4}\)

y+1)(y+2)=(y+3)(y+4)+A(y+4)+B(y+3) …..(1)

In equation (1)

Put y=-3 and y=-4, we get A=2 and B=-6 respectively

∴ \(\frac{(y+1)(y+2)}{(y+3)(y+4)}=1+\frac{2}{(y+3)}-\frac{6}{(y+4)}\)

∴ \(\frac{\left(x^2+1\right)\left(x^2+2\right)}{\left(x^2+3\right)\left(x^2+4\right)}\)

= \(1+\frac{2}{x^2+3}-\frac{6}{x^2+4}\) (because \(x^2=y\))

I = \(\int 1 d x+2 \int \frac{1}{x^2+3} d x-6 \int \frac{1}{x^2+4} d x\)

= \(x+2 \cdot \frac{1}{\sqrt{3}} \tan ^{-1} \frac{x}{\sqrt{3}}-6 \cdot \frac{1}{2} \tan ^{-1} \frac{x}{2}+C=x+\frac{2}{\sqrt{3}} \tan ^{-1} \frac{x}{\sqrt{3}}-3 \tan ^{-1} \frac{x}{2}+C\)

Question 19. \(\int \frac{2 \mathrm{x}}{\left(\mathrm{x}^2+1\right)\left(\mathrm{x}^2+3\right)} \mathrm{dx}\)
Solution:

Let \(I=\int \frac{2 x}{\left(x^2+1\right)\left(x^2+3\right)} d x\)

Put \(\mathrm{x}^2=\mathrm{t} \Rightarrow 2 \mathrm{xdx}=\mathrm{dt}\)

∴ I = \(\int \frac{d t}{(t+1)(t+3)}=\int \frac{1}{(t+1)(t+3)} d x\)

Let \(\frac{A}{t+1}+\frac{B}{t+3}\)

I = \((t+3) A+(t+1) B, 1=(A+B) t+(3 t+B)\)

⇒ A = \(\frac{1}{2} \text { and } B=-\frac{1}{2}\)

∴ I = \(\int \frac{1}{2}\left[\frac{1}{(t+1)}-\frac{1}{(t+3)}\right] d t=\frac{1}{2} \int \frac{1}{t+1} d t-\frac{1}{2} \int \frac{1}{t+3} d t=\frac{1}{2} \log |t+1|-\frac{1}{2} \log |t+3|+C\)

= \(\frac{1}{2} \log \left|\frac{t+1}{t+3}\right|+C=\frac{1}{2} \log \left|\frac{x^2+1}{x^2+3}\right|+C\)

Question 20. \(\int \frac{1}{\mathrm{x}\left(\mathrm{x}^4-1\right)} \mathrm{dx}\)
Solution:

Let \(I=\int \frac{1}{x\left(x^4-1\right)} d x=\int \frac{x^3}{x^4\left(x^4-1\right)} d x \) (Multiply Nr. and Dr. by \(x^3\))

Put \(\mathrm{x}^4=\mathrm{t} \Rightarrow 4 \mathrm{x}^3 \mathrm{dx}=\mathrm{dt}\)

∴\(\int \frac{1}{x\left(x^4-1\right)} d x =\frac{1}{4} \int \frac{d t}{t(t-1)}=\frac{1}{4} \int\left[\frac{1}{t-1}-\frac{1}{t}\right] d t=\frac{1}{4} \int \frac{1}{t-1} d t-\frac{1}{4} \int \frac{1}{t} d t\)

= \(\frac{1}{4} \log |t-1|-\frac{1}{4} \log |t|+C=\frac{1}{4} \log \left|\frac{t-1}{t}\right|+C=\frac{1}{4} \log \left|\frac{x^4-1}{x^4}\right|+C\)

Question 21. \(\int \frac{1}{\left(e^{\mathrm{x}}-1\right)} \mathrm{dx}\)
Solution:

Let \(\mathrm{I}=\int \frac{1}{\left(\mathrm{e}^{\mathrm{x}}-1\right)} \mathrm{dx}\)

Let \(\mathrm{e}^{\mathrm{x}}=\mathrm{t} \Rightarrow \mathrm{e}^{\mathrm{x}} \mathrm{dx}=\mathrm{dt} \Rightarrow \mathrm{dx}=\frac{\mathrm{dt}}{\mathrm{t}}\)

∴ I = \(\int \frac{1}{t-1} \times \frac{d t}{t}=\int \frac{1}{t(t-1)} d t=\int\left[\frac{1}{t-1}-\frac{1}{t}\right] d t=\log |t-1|-\log |t|+C\)

= \(\log \left|\frac{t-1}{t}\right|+C=\log \left|\frac{e^x-1}{e^x}\right|+C\)

Choose The Correct Answer

Question 22. \(\int \frac{x d x}{(x-1)(x-2)}\) equals ?

  1. \(\log \left|\frac{(x-1)^2}{x-2}\right|+C\)
  2. \(\log \left|\frac{(x-2)^2}{x-1}\right|+C\)
  3. \(\log \left|\left(\frac{(x-1)}{x-2}\right)^2\right|+C\)
  4. \(\log |(x-1)(x-2)|+C\)

Solution: 2. \(\log \left|\frac{(x-2)^2}{x-1}\right|+C\)

Let \(I=\int \frac{x d x}{(x-1)(x-2)}\)

x = \(\mathrm{A}(\mathrm{x}-2)+\mathrm{B}(\mathrm{x}-1)\)

Put x=1 and 2 in (1), we get A=-1 and B=2 respectively,

∴ \(\frac{x}{(x-1)(x-2)}=-\frac{1}{(x-1)}+\frac{2}{(x-2)}\)

I = \(\int\left\{\frac{-1}{(x-1)}+\frac{2}{(x-2)}\right\} d x=-\log |x-1|+2 \log |x-2|+C=\log \left|\frac{(x-2)^2}{(x-1)}\right|+C\)

Hence, the correct answer is (2).

Question 23. \(\int \frac{\mathrm{dx}}{\mathrm{x}\left(\mathrm{x}^2+1\right)}\) equals?

  1. \(\log |\mathrm{x}|-\frac{1}{2} \log \left(\mathrm{x}^2+1\right)+\mathrm{C}\)
  2. \(\log |\mathrm{x}|+\frac{1}{2} \log \left(\mathrm{x}^2+1\right)+\mathrm{C}\)
  3. –\(\log |x|+\frac{1}{2} \log \left(x^2+1\right)+C\)
  4. \(\frac{1}{2} \log |\mathrm{x}|+\log \left(\mathrm{x}^2+1\right)+\mathrm{C}\)

Solution: 1. \(\log |\mathrm{x}|-\frac{1}{2} \log \left(\mathrm{x}^2+1\right)+\mathrm{C}\)

Let \(\mathrm{I}=\int \frac{\mathrm{dx}}{\mathrm{x}\left(\mathrm{x}^2+1\right)} \mathrm{dx}\)

Let \(\frac{1}{x\left(x^2+1\right)}=\frac{A}{x}+\frac{B x+C}{x^2+1}\)

I = \(\mathrm{A}\left(\mathrm{x}^2+1\right)+(\mathrm{Bx}+\mathrm{C}) \mathrm{x}\)

In equation (1), Put x=0 ⇒ A=1

On equating the coefficients of \(x^2, x\), we get A+B=0 ⇒ B=-A=-1, A=1 and C=0

∴ \(\frac{1}{x\left(x^2+1\right)}=\frac{1}{x}+\left(\frac{-x}{x^2+1}\right)\)

I = \(\int\left\{\frac{1}{x}-\frac{x}{x^2+1}\right\} d x=\log |x|-\frac{1}{2} \log \left|x^2+1\right|+C\)

Hence, the correct answer is (1).

Integrals Exercise 7.6

Integrate The Function

Question 1. \(\int x \sin x d x\)
Solution:

Let \(I=\int x \sin x d x\)

Taking x as the first function and sin x as the second function and integrating by parts, we obtain

I = \(x \int \sin x d x-\int\left\{\left(\frac{d}{d x}(x)\right) \int \sin x d x\right\} d x\)

= \(x(-\cos x)+\int 1 \cdot(\cos x) d x=-x \cos x+\sin x+C\)

Question 2. Let \(\int x \sin 3 x d x\)
Solution:

I = \(\int x \sin 3 x d x\)

Taking x as the first function and sin 3x as the second function and integrating by parts, we obtain

I = \(x \int \sin 3 x d x-\int\left\{\left(\frac{d}{d x}(x)\right) \int \sin 3 x d x\right\} d x=x\left(\frac{-\cos 3 x}{3}\right)-\int 1 \cdot\left(\frac{-\cos 3 x}{3}\right) d x\)

= \(\frac{-x \cos 3 x}{3}+\frac{1}{3} \int \cos 3 x d x=\frac{-x \cos 3 x}{3}+\frac{1}{9} \sin 3 x+C\)

Question 3. \(\int x^2 e^x d x\)
Solution:

Let \(\mathrm{I}=\int \mathrm{x}^2 \mathrm{e}^{\mathrm{x}} \mathrm{dx}\)

Taking \(x^2\) as first function and \(\mathrm{e}^{\mathrm{x}}\) as second function and integrating by parts, we obtain

I = \(x^2 \int e^x d x-\int\left\{\left(\frac{d}{d x}\left(x^2\right)\right) \int e^x d x\right\} d x=x^2 e^x-\int 2 x \cdot e^x d x=x^2 e^x-2 \int x \cdot e^x d x\)

Again integrating by parts, we obtain

I = \(x^2 e^x-2\left[x \int e^x d x-\int\left\{\left(\frac{d}{d x}(x)\right) \cdot \int e^x d x\right] d x\right]=x^2 e^x-2\left[x e^x-\int e^x d x\right]\)

= \(x^2 e^x-2\left[x e^x-e^x\right]=x^2 e^x-2 x e^x+2 e^x+C=e^x\left(x^2-2 x+2\right)+C\)

Question 4. \(\int x \log x d x\)
Solution:

Let \(\mathrm{I}=\int \mathrm{x} \log \mathrm{x} \mathrm{x}\)

Taking \(\log \mathrm{x}\) as first function and x as second function and integrating by parts, we obtain

I = \(\log x \int x d x-\int\left\{\left(\frac{d}{d x}(\log x)\right) \int x d x\right\} d x=\log x \cdot \frac{x^2}{2}-\int \frac{1}{x} \cdot \frac{x^2}{2} d x\)

= \(\frac{x^2 \log x}{2}-\int \frac{x}{2} d x=\frac{x^2 \log x}{2}-\frac{x^2}{4}+C\)

Question 5. \(\int x \log 2 x d x\)
Solution:

Let \(\mathrm{I}=\int \mathrm{x} \log 2 \mathrm{x} d \mathrm{x}\)

Taking log 2x as the first function and x as the second function and integrating by parts, we obtain

I = \(\log 2 x \int x d x-\int\left\{\left(\frac{d}{d x}(\log 2 x)\right) \int x d x\right\} d x=\log 2 x \cdot \frac{x^2}{2}-\int\left(\frac{2}{2 x} \cdot \frac{x^2}{2}\right) d x\)

= \(\frac{x^2 \log 2 x}{2}-\int \frac{x}{2} d x=\frac{x^2 \log 2 x}{2}-\frac{x^2}{4}+C\)

Question 6. \(\int x^2 \log x d x\)
Solution:

Let \(\mathrm{I}=\int \mathrm{x}^2 \log \mathrm{x} d \mathrm{x}\)

Taking log x as the first function and \(x^2\) as second function and integrating by parts, we obtain

I = \(\log x \int x^2 d x-\int\left\{\left(\frac{d}{d x}(\log x)\right) \int x^2 d x\right\} d x=\log x\left(\frac{x^3}{3}\right)-\int \frac{1}{x} \cdot \frac{x^3}{3} d x\)

= \(\frac{x^3 \log x}{3}-\int \frac{x^2}{3} d x=\frac{x^3 \log x}{3}-\frac{x^3}{9}+C\)

Question 7. \(\int x \sin ^{-1} x d x\)
Solution:

Let \(I=\int x \sin ^{-1} x d x\)

put \(\mathrm{x}=\sin \mathrm{t}, \mathrm{dx}=\cos \mathrm{dt}\)

= \(\int \sin t \sin ^{-1}(\sin t) \cos t d t=\int t \sin t \cos t d t\)

= \(\frac{1}{2} \int t(2 \sin t \cos t) d t=\frac{1}{2} \int t \sin 2 t d t\)

Taking t as the first function and sin 2t as the second function and integrating by parts, we obtain

= \(\frac{1}{2}\left[t \int \sin 2 t d t-\int\left(\frac{d(t)}{d t} \int \sin 2 t d t\right) d t\right]\)

= \(\frac{1}{2}\left[-t \frac{\cos 2 t}{2}+\int \frac{\cos 2 t}{2} d t\right]\)

= \(\frac{1}{2}\left[-\frac{t}{2} \cos 2 t+\frac{1}{4} \sin 2 t\right]+C=\frac{-t}{4} \cos 2 t+\frac{1}{8} \sin 2 t+C\)

= \(\frac{-t}{4}\left[1-2 \sin ^2 t\right]+\frac{1}{8} 2 \sin t \cos t+C=\frac{-t}{4}\left(1-2 \sin ^2 t\right)+\frac{1}{4} \sin t \sqrt{1-\sin ^2 t}+C\)

= \(\frac{-\sin ^{-1} x}{4}\left(1-2 x^2\right)+\frac{x}{4} \sqrt{1-x^2}+C=\frac{1}{4}\left(2 x^2-1\right) \sin ^{-1} x+\frac{x}{4} \sqrt{1-x^2}+C\)

Question 8. \(\int x \tan ^{-1} x d x\)
Solution:

I = \(\int x \tan ^{-1} x d x\)

Taking \(\tan ^{-1} \mathrm{x}\) as first function and x as second function and integrating by parts, we obtain

I = \(\tan ^{-1} x \int x d x-\int\left\{\left(\frac{d}{d x}\left(\tan ^{-1} x\right)\right) \int x d x\right\} d x=\tan ^{-1} x\left(\frac{x^2}{2}\right)-\int \frac{1}{1+x^2} \cdot \frac{x^2}{2} d x\)

= \(\frac{x^2 \tan ^{-1} x}{2}-\frac{1}{2} \int \frac{x^2}{1+x^2} d x=\frac{x^2 \tan ^{-1} x}{2}-\frac{1}{2} \int\left(\frac{x^2+1}{1+x^2}-\frac{1}{1+x^2}\right) d x\)

= \(\frac{x^2 \tan ^{-1} x}{2}-\frac{1}{2} \int\left(1-\frac{1}{1+x^2}\right) d x=\frac{x^2 \tan ^{-1} x}{2}-\frac{1}{2}\left(x-\tan ^{-1} x\right)+C\)

= \(\frac{x^2}{2} \tan ^{-1} x-\frac{x}{2}+\frac{1}{2} \tan ^{-1} x+C\)

Question 9. \(\int x \cos ^{-1} x d x\)
Solution:

Let \(I=\int x \cos ^{-1} x d x\)

put \(\mathrm{x}=\cos \mathrm{t}, \mathrm{dx}=-\sin \mathrm{dt}\)

I = \(-\int \cos t \cos ^{-1}(\cos t) \sin t d t=-\int t \sin t \cos t d t\)

= \(-\frac{1}{2} \int t(2 \sin t \cos t) d t=-\frac{1}{2} \int t(\sin 2 t) d t\)

Taking t as the first function and sin2 t as the second function and integrating by parts, we obtain

= \(-\frac{1}{2}\left[t \int \sin 2 t d t-\int\left(\frac{d(t)}{d t} \int \sin 2 t d t\right) d t=-\frac{1}{2}\left[-t \frac{\cos 2 t}{2}+\int \frac{\cos 2 t}{2} d t\right]\right. \)

= \(-\frac{1}{2}\left[-t \frac{\cos 2 t}{2}+\frac{\sin 2 t}{4}\right]+C=\frac{t}{4} \cos 2 t-\frac{\sin 2 t}{8}+C\)

= \(\frac{t}{4}\left(2 \cos ^2 t-1\right)-\frac{1}{8} 2 \sin t \cos t+C=\frac{t}{4}\left(2 \cos ^2 t-1\right)-\frac{1}{4} \cos t \sqrt{1-\cos ^2} t+C\)

= \(\frac{\cos ^{-1} x}{4}\left(2 x^2-1\right)-\frac{1}{4} x \sqrt{1-x^2}+C=\frac{\left(2 x^2-1\right)}{4} \cos ^{-1} x-\frac{x}{4} \sqrt{1-x^2}+C\)

Question 10. \(\int\left(\sin ^{-1} x\right)^2 \mathrm{dx}\)
Solution:

Let \(I=\int\left(\sin ^{-1} x\right)^2 \cdot 1 d x\)

Taking \(\left(\sin ^{-1} x\right)^2\) as the first function and 1 as the second function and integrating by parts, we obtain

I = \(\left(\sin ^{-1} x\right)^2 \int 1 d x-\int\left\{\frac{d}{d x}\left(\sin ^{-1} x\right)^2 \cdot \int 1 \cdot d x\right\} d x=x\left(\sin ^{-1} x\right)^2-2 \int \frac{x \sin ^{-1} x}{\sqrt{1-x^2}} d x\)

= \(x\left(\sin ^{-1} x\right)^2-2 \int t \cdot \sin t d t\)

∴ \(\left(\begin{array}{l}
\text { Put } \sin ^{-1} x=t \Rightarrow x=\sin t \\
\frac{1}{\sqrt{1-x^2}} d x=d t
\end{array}\right)\)

Again integrating by parts, we obtain

I = \(x\left(\sin ^{-1} x\right)^2-2\left[-t \cos t-\int(-\cos t) d t\right]=x\left(\sin ^{-1} x\right)^2+2 t \cos t-2 \sin t+C\)

I = \(x\left(\sin ^{-1} x\right)^2+2 t \sqrt{1-\sin ^2 t}-2 \sin t+C=x\left(\sin ^{-1} x\right)^2+2 \sin ^{-1} x \sqrt{1-x^2}-2 x+C\)

Question 11. \(\int \frac{x \cos ^{-1} x}{\sqrt{1-x^2}} d x\)
Solution:

Let \(\mathrm{I}=\int \frac{\mathrm{x} \cos ^{-1} \mathrm{x}}{\sqrt{1-\mathrm{x}^2}} \mathrm{dx}\)

Put \(\cos ^{-1} \mathrm{x}=\mathrm{t} \Rightarrow \mathrm{x}=\cos \mathrm{t}\)

⇒ \(\frac{1}{\sqrt{1-x^2}} d x=-d t\)

⇒ I = \(\int t \cos t d t\) (Using interagration by parts)

= \(-\left[t \sin t-\int \sin t d t\right]=-[t \sin t+\cos t]+C\)

⇒I = \(-\left[t \sqrt{1-\cos ^2 t}+\cos t\right]+C=-\left[\sqrt{1-x^2} \cos ^{-1} x+x\right]+C\)

Question 12. \(\int x \sec ^2 x d x\)
Solution:

Let \(\mathrm{I}=\int \mathrm{x} \sec ^2 \mathrm{xdx}\)

Taking x as first function and \(\sec ^2 \mathrm{x}\) as second function and integrating by parts, we obtain

I = \(x \int \sec ^2 x d x-\int\left\{\left\{\frac{d}{d x}(x)\right\} \int \sec ^2 x d x\right\} d x\)

= \(x \tan x-\int 1 \cdot \tan x d x=x \tan x+\log |\cos x|+C\)

Question 13. \(\int \tan ^{-1} x d x\)
Solution:

Let \(I=\int 1 \cdot \tan ^{-1} x d x\)

Taking \(\tan ^{-1} x\) as the first function and 1 as the second function and integrating by parts, we obtain

I = \(\tan ^{-1} x \int 1 d x-\int\left\{\left(\frac{d}{d x}\left(\tan ^{-1} x\right)\right) \int 1 \cdot d x\right\} d x\)

= \(\tan ^{-1} x \cdot x-\int \frac{1}{1+x^2} \cdot x d x=x \tan ^{-1} x-\frac{1}{2} \int \frac{2 x}{1+x^2} d x\)

Let \(1+\mathrm{x}^2=\mathrm{t} \Rightarrow 2 \mathrm{xdx}=\mathrm{dt}\)

⇒ I = \(x \tan ^{-1} x-\frac{1}{2} \int \frac{1}{t} d t=x \tan ^{-1} x-\frac{1}{2} \log |t|+C=x \tan ^{-1} x-\frac{1}{2} \log \left|1+x^2\right|+C\)

Question 14. \(\int x(\log x)^2 d x\)
Solution:

Let \(\mathrm{I}=\int \mathrm{x}(\log \mathrm{x})^2 \mathrm{dx}\)

Taking \((\log x)^2\) as first function and x as second function and integrating by parts, we obtain

I = \((\log x)^2 \int x d x-\int\left\{\left(\frac{d}{d x}(\log x)^2\right) \int x d x\right\} d x\)

= \(\frac{x^2}{2}(\log x)^2-\int\left[2 \log x \cdot \frac{1}{x} \cdot \frac{x^2}{2}\right] d x=\frac{x^2}{2}(\log x)^2-\int x \log x d x\)

Again integrating by parts, we obtain

I = \(\frac{x^2}{2}(\log x)^2-\left[\log x \int x d x-\int\left\{\left(\frac{d}{d x}(\log x)\right) \int x d x\right\} d x\right]\)

= \(\frac{x^2}{2}(\log x)^2-\left[\frac{x^2}{2} \log x-\int \frac{1}{x} \cdot \frac{x^2}{2} d x\right]=\frac{x^2}{2}(\log x)^2-\frac{x^2}{2} \log x+\frac{1}{2} \int x d x\)

= \(\frac{x^2}{2}(\log x)^2-\frac{x^2}{2} \log x+\frac{x^2}{4}+C\)

Question 15. \(\int\left(x^2+1\right) \log x d x\)
Solution:

Let \(I=\int\left(x^2+1\right) \log x d x=\int x^2 \log x d x+\int \log x d x\)

Let \(\mathrm{I}=\mathrm{I}_1+\mathrm{I}_2\)…..(1)

Where, \(I_1=\int x^2 \log x d x\) and \(I_2=\int \log x d x\)

⇒ \(I_1=\int x^2 \log x d x\)

Taking logx as first function and \(\mathrm{x}^2\) as second function and integrating by parts, we obtain

⇒ \(I_1=\log x \cdot \int x^2 d x-\int\left\{\left(\frac{d}{d x}(\log x)\right) \int x^2 d x\right\} d x=\log x \cdot \frac{x^3}{3}-\int \frac{1}{x} \cdot \frac{x^3}{3} d x\)

= \(\frac{x^3}{3} \log x-\frac{1}{3}\left(\int x^2 d x\right)=\frac{x^3}{3} \log x-\frac{x^3}{9}+C_1\)

∴ \(I_2=\int \log x d x\)

Taking log x as the first function and 1 as the second function and integrating by parts, we obtain

⇒ \(I_2 =\log x \int 1 \cdot d x-\int\left\{\left(\frac{d}{d x}(\log x)\right) \int 1 \cdot d x\right\} d x\)

= \(\log x \cdot x-\int \frac{1}{x} \cdot x d x=x \log x-\int 1 d x\)

= \(x \log x-x+C_2\)

Using equations (2) and (3) in (1), we obtain

I = \(\frac{x^3}{3} \log x-\frac{x^3}{9}+C_1+x \log x-x+C_2=\frac{x^3}{3} \log x-\frac{x^3}{9}+x \log x-x+\left(C_1+C_2\right)\)

= \(\left(\frac{x^3}{3}+x\right) \log x-\frac{x^3}{9}-x+C\) (because \(C_1+C_2=C\))

Question 16. \(\int e^x(\sin x+\cos x) d x\)
Solution:

Let \(I=\int e^x(\sin x+\cos x) d x\)

Let \(\mathrm{f}(\mathrm{x})=\sin \mathrm{x}\)

⇒ \(\mathrm{f}(\mathrm{x})=\cos \mathrm{x}\)

⇒ \(\mathrm{I}=\int \mathrm{e}^{\mathrm{x}}\left\{\mathrm{f}(\mathrm{x})+\mathrm{f}^{\prime}(\mathrm{x})\right\} \mathrm{dx}\)

It is known that, \(\int \mathrm{e}^{\mathrm{x}}\left\{\mathrm{f}(\mathrm{x})+\mathrm{f}^{\prime}(\mathrm{x})\right\} \mathrm{dx}=\mathrm{e}^{\mathrm{x}} \mathrm{f}(\mathrm{x})+\mathrm{C}\)

∴ \(\mathrm{I}=\mathrm{e}^{\mathrm{x}} \sin \mathrm{x}+\mathrm{C}\)

Question 17. \(\int \frac{\mathrm{xe}^{\mathrm{x}}}{(1+\mathrm{x})^2} \mathrm{dx}\)
Solution:

Let \(\mathrm{I}=\int \frac{\mathrm{xe}}{(1+\mathrm{x})^2} \mathrm{dx}=\int \mathrm{e}^x\left\{\frac{\mathrm{x}}{(1+\mathrm{x})^2}\right\} \mathrm{dx}=\int \mathrm{e}^{\mathrm{x}}\left\{\frac{1+\mathrm{x}-1}{(1+\mathrm{x})^2}\right\} \mathrm{dx}=\int \mathrm{e}^{\mathrm{x}}\left\{\frac{1}{1+\mathrm{x}}-\frac{1}{(1+\mathrm{x})^2}\right\} \mathrm{dx}\)

Let \(f(x)=\frac{1}{1+x} \Rightarrow f^{\prime}(x)=\frac{-1}{(1+x)^2} \Rightarrow \int \frac{x e^x}{(1+x)^2} d x=\int e^x\left\{f(x)+f^{\prime}(x)\right\} d x\)

It is known that, \(\int \mathrm{e}^{\mathrm{x}}\left\{\mathrm{f}(\mathrm{x})+\mathrm{f}^{\prime}(\mathrm{x})\right\} \mathrm{dx}=\mathrm{e}^{\mathrm{x}} \mathrm{f}(\mathrm{x})+\mathrm{C}\)

∴ \(\int \frac{x e^x}{(1+x)^2} d x=\frac{e^x}{1+x}+C\)

Question 18. \(\int e^x\left(\frac{1+\sin x}{1+\cos x}\right) d x\)
Solution:

⇒ \(e^x\left(\frac{1+\sin x}{1+\cos x}\right)=e^x\left(\frac{\sin ^2 \frac{x}{2}+\cos ^2 \frac{x}{2}+2 \sin \frac{x}{2} \cos \frac{x}{2}}{2 \cos ^2 \frac{x}{2}}\right)\)

= \(\frac{e^x\left(\sin \frac{x}{2}+\cos \frac{x}{2}\right)^2}{2 \cos ^2 \frac{x}{2}}\)

= \(\frac{1}{2} e^x \cdot\left(\frac{\sin \frac{x}{2}+\cos \frac{x}{2}}{\cos \frac{x}{2}}\right)^2=\frac{1}{2} e^x\left[\tan \frac{x}{2}+1\right]^2=\frac{1}{2} e^x\left[1+\tan \frac{x}{2}\right]^2\)

= \(\frac{1}{2} e^x\left[1+\tan ^2 \frac{x}{2}+2 \tan \frac{x}{2}\right]=\frac{1}{2} e^x\left[\sec ^2 \frac{x}{2}+2 \tan \frac{x}{2}\right]\)

⇒ \(\int \frac{e^x(1+\sin x) d x}{(1+\cos x)}=\int e^x\left[\tan \frac{x}{2}+\frac{1}{2} \sec ^2 d x\right] d x \ldots .(1)\)

Let \(f(x)=\tan \frac{x}{2} \Rightarrow f^{\prime}(x)=\frac{1}{2} \sec ^2 \frac{x}{2}\)

It is known that, \(\int \mathrm{e}^{\mathrm{x}}\left\{\mathrm{f}(\mathrm{x})+\mathrm{f}^{\prime}(\mathrm{x})\right\} \mathrm{dx}=\mathrm{e}^{\mathrm{x}} \mathrm{f}(\mathrm{x})+\mathrm{C}\)

From equation (1), we obtain, \(\int \frac{e^x(1+\sin x)}{(1+\cos x)} d x=e^x \tan \frac{x}{2}+C\)

Question 19. \(\int e^x\left(\frac{1}{x}-\frac{1}{x^2}\right) d x\)
Solution:

Let \(I=\int e^x\left[\frac{1}{x}-\frac{1}{x^2}\right] d x\)

Also, let \(f(x)=\frac{1}{x} \Rightarrow f^{\prime}(x)=\frac{-1}{x^2}\)

It is known that, \(\int \mathrm{e}^{\mathrm{x}}\left\{\mathrm{f}(\mathrm{x})+\mathrm{f}^{\prime}(\mathrm{x})\right\} \mathrm{dx}=\mathrm{e}^{\mathrm{x}} \mathrm{f}(\mathrm{x})+\mathrm{C}\)

∴ \(\mathrm{I}=\frac{\mathrm{e}^{\mathrm{x}}}{\mathrm{x}}+\mathrm{C}\)

Question 21. \(\int e^{2 x} \sin x d x\)
Solution:

Let \(\mathrm{I}=\int \mathrm{e}^{2 \mathrm{x}} \sin \mathrm{x} d \mathrm{x}\)

Integrating by parts, we obtain. \(I=\sin x \int e^{2 x} d x-\int\left\{\left(\frac{d}{d x}(\sin x)\right) \int e^{2 x} d x\right\} d x\)

⇒ \(\mathrm{I}=\sin \mathrm{x} \cdot \frac{\mathrm{e}^{2 \mathrm{x}}}{2}-\int \cos \mathrm{x} \cdot \frac{\mathrm{e}^{2 \mathrm{x}}}{2} \mathrm{dx} \Rightarrow \mathrm{I}\)

= \(\frac{\mathrm{e}^{2 \mathrm{x}} \sin \mathrm{x}}{2}-\frac{1}{2} \int \mathrm{e}^{2}\)

Again integrating by parts, we obtain \(I=\frac{e^{2 x} \sin x}{2}-\frac{1}{2}\left[\cos x \int e^{2 x} d x-\int\left\{\left(\frac{d}{d x}(\cos x)\right) \int e^{2 x} d x\right\} d x\right]\)

⇒ \(\mathrm{I}=\frac{\mathrm{e}^{2 \mathrm{x}} \sin \mathrm{x}}{2}-\frac{1}{2}\left[\cos \mathrm{x} \cdot \frac{\mathrm{e}^{2 \mathrm{x}}}{2}-\int(-\sin \mathrm{x}) \frac{\mathrm{e}^{2 \mathrm{x}}}{2} \mathrm{dx}\right]\)

⇒ \(\mathrm{I}=\frac{\mathrm{e}^{2 \mathrm{x}} \cdot \sin \mathrm{x}}{2}-\frac{1}{2}\left[\frac{\mathrm{e}^{2 \mathrm{x}} \cos \mathrm{x}}{2}+\frac{1}{2} \int \mathrm{e}^{2 \mathrm{x}} \sin \mathrm{x} d \mathrm{x}\right]\)

⇒ \(\mathrm{I}=\frac{\mathrm{e}^{2 \mathrm{x}} \sin \mathrm{x}}{2}-\frac{\mathrm{e}^{2 \mathrm{x}} \cos \mathrm{x}}{4}-\frac{1}{4} \mathrm{I}\)

⇒ \(\mathrm{I}+\frac{1}{4} \mathrm{I}\)

= \(\frac{\mathrm{e}^{2 \mathrm{x}} \cdot \sin \mathrm{x}}{2}-\frac{\mathrm{e}^{2 \mathrm{x}} \cos \mathrm{x}}{4} \Rightarrow \frac{5}{4} \mathrm{I}=\frac{\mathrm{e}^{2 \mathrm{x}} \sin \mathrm{x}}{2}-\frac{\mathrm{e}^{2 \mathrm{x}} \cos \mathrm{x}}{4}\)

⇒ I = \(\frac{4}{5}\left[\frac{e^{2 x} \sin x}{2}-\frac{e^{2 x} \cos x}{4}\right]+C \Rightarrow I=\frac{e^{2 x}}{5}[2 \sin x-\cos x]+C\)

Question 22. \(\int \sin ^{-1}\left(\frac{2 x}{1+x^2}\right) d x\)
Solution:

Let \(\mathrm{x}=\tan \theta \Rightarrow d x=\sec ^2 \theta d \theta\)

∴ \(\sin ^{-1}\left(\frac{2 x}{1+x^2}\right)=\sin ^{-1}\left(\frac{2 \tan \theta}{1+\tan ^2 \theta}\right)=\sin ^{-1}(\sin 2 \theta)=2 \theta\)

⇒ \(\mathrm{I}=\int \sin ^{-1}\left(\frac{2 \mathrm{x}}{1+\mathrm{x}^2}\right) \mathrm{dx}\)

= \(\int 2 \theta \cdot \sec ^2 \theta \mathrm{d} \theta=2 \int \theta \cdot \sec ^2 \theta \mathrm{d} \theta\)

Integrating by parts, we obtain

I = \(2\left[\theta \cdot \int \sec ^2 \theta d \theta-\int\left\{\left(\frac{d}{d \theta}(\theta)\right) \int \sec ^2 \theta d \theta\right\} d \theta\right]\)

= \(2\left[\theta \cdot \tan \theta-\int \tan \theta d \theta\right]\)

= \(2[\theta \tan \theta+\log |\cos \theta|+C]=2\left[x \tan ^{-1} x+\log \left|\frac{1}{\sqrt{1+x^2}}\right|+C\right]\)

= \(2 x \tan ^{-1} x+2 \log \left(1+x^2\right)^{-\frac{1}{2}}+C\)

= \(2 x \tan ^{-1} x+2\left[-\frac{1}{2} \log \left(1+x^2\right)\right]+C=2 x \tan ^{-1} x-\log \left(1+x^2\right)+C\)

Choose The Correct Answer

Question 23. \(\int x^2 e^{x^7} d x\) equals

  1. \(\frac{1}{3} \mathrm{e}^{\mathrm{x}^3}+\mathrm{C}\)
  2. \(\frac{1}{3} \mathrm{e}^{\mathrm{x}^2}+\mathrm{C}\)
  3. \(\frac{1}{2} \mathrm{e}^{\mathrm{x}^1}+\mathrm{C}\)
  4. \(\frac{1}{2} e^{x^2}+C\)

Solution: 1. \(\frac{1}{3} \mathrm{e}^{\mathrm{x}^3}+\mathrm{C}\)

Let \(\mathrm{I}=\int \mathrm{x}^2 \mathrm{e}^{\mathrm{x}^3} \mathrm{dx}\)

Put \(x^3=t \Rightarrow 3 x^2 d x=d t \Rightarrow I=\frac{1}{3} \int e^t d t=\frac{1}{3}\left(e^t\right)+C=\frac{1}{3} e^{x^3}+C\)

Hence, the correct answer is (1).

Question 24. \(\int e^x \sec x(1+\tan x) d x\)

  1. \(\mathrm{e}^{\mathrm{x}} \cos \mathrm{x}+\mathrm{C}\)
  2. \(e^x \sec x+C\)
  3. \(e^x \sin x+C\)
  4. \(\mathrm{e}^{\mathrm{x}} \tan \mathrm{x}+\mathrm{C}\)

Solution: 2. \(e^x \sec x+C\)

Let \(I=\int e^x \sec x(1+\tan x) d x\)

= \(\int \mathrm{e}^x(\sec x+\sec x \tan x) d x\)

Also, if sec x = f(x) ⇒ \(\sec \mathrm{x} \tan \mathrm{x}=\mathrm{f}^{\prime}(\mathrm{x})\)

It is known that, \(\int \mathrm{e}^x\left\{f(x)+f^{\prime}(x)\right\} d x=e^x f(x)+C\)

I = \(e^x \sec x+C\)

Hence, the correct answer is (2).

Integrals Exercise 7.7

Integrate The Functions

Question 1. \(\int \sqrt{4-x^2} d x\)
Solution:

Let \(\mathrm{I}=\int \sqrt{4-\mathrm{x}^2} \mathrm{dx}=\int \sqrt{(2)^2-(\mathrm{x})^2} \mathrm{dx}\)

It is known that, \(\int \sqrt{a^2-x^2} d x=\frac{x}{2} \sqrt{a^2-x^2}+\frac{a^2}{2} \sin ^{-1}\left(\frac{x}{a}\right)+C\)

∴ I = \(\frac{x}{2} \sqrt{4-x^2}+\frac{4}{2} \sin ^{-1}\left(\frac{x}{2}\right)+C=\frac{x}{2} \sqrt{4-x^2}+2 \sin ^{-1}\left(\frac{x}{2}\right)+C
\)

Question 2. \(\int \sqrt{1-4 \mathrm{x}^2} \mathrm{dx}\)
Solution:

Let \(\mathrm{I}=\int \sqrt{1-4 \mathrm{x}^2} \mathrm{dx}=\int \sqrt{(1)^2-(2 \mathrm{x})^2} \mathrm{dx}\), Let \(2 \mathrm{x}=\mathrm{t} \Rightarrow 2 \mathrm{dx}=\mathrm{dt}\)

I = \(\frac{1}{2} \int \sqrt{(1)^2-(t)^2} d t\)

It is known that, (because \(\int \sqrt{a^2-x^2} d x=\frac{x}{2} \sqrt{a^2-x^2}+\frac{a^2}{2} \sin ^{-1}\left(\frac{x}{a}\right)+C\))

⇒ I = \(\frac{1}{2}\left[\frac{\mathrm{t}}{2} \sqrt{1-\mathrm{t}^2}+\frac{1}{2} \sin ^{-1} \mathrm{t}\right]+\mathrm{C}=\frac{\mathrm{t}}{4} \sqrt{1-\mathrm{t}^2}+\frac{1}{4} \sin ^{-1} \mathrm{t}+\mathrm{C}\)

= \(\frac{2 \mathrm{x}}{4} \sqrt{1-4 \mathrm{x}^2}+\frac{1}{4} \sin ^{-1}(2 \mathrm{x})+\mathrm{C}=\frac{\mathrm{x}}{2} \sqrt{1-4 \mathrm{x}^2}+\frac{1}{4} \sin ^{-1}(2 \mathrm{x})+\mathrm{C}\)

Question 3. \(\int \sqrt{x^2+4 x+6} d x\)
Solution:

Let \(I=\int \sqrt{x^2+4 x+6} d x=\int \sqrt{x^2+4 x+4+2} d x\)

= \(\int \sqrt{\left(x^2+4 x+4\right)+2} d x=\int \sqrt{(x+2)^2+(\sqrt{2})^2} d x\)

It is known that, (because \(\int \sqrt{\mathrm{x}^2+\mathrm{a}^2} \mathrm{dx}=\frac{\mathrm{x}}{2} \sqrt{\mathrm{x}^2+\mathrm{a}^2}+\frac{\mathrm{a}^2}{2} \log \left|\mathrm{x}+\sqrt{\mathrm{x}^2+\mathrm{a}^2}\right|+\mathrm{C}\))

∴ I = \(\frac{(x+2)}{2} \sqrt{x^2+4 x+6}+\frac{2}{2} \log \left|(x+2)+\sqrt{x^2+4 x+6}\right|+C\)

= \(\frac{(x+2)}{2} \sqrt{x^2+4 x+6}+\log \left|(x+2)+\sqrt{x^2+4 x+6}\right|+C\)

Question 4. \(\int \sqrt{x^2+4 x+1} d x\)
Solution:

Let \(I=\int \sqrt{x^2+4 x+1} d x=\int \sqrt{\left(x^2+4 x+4\right)-3} d x=\int \sqrt{(x+2)^2-(\sqrt{3})^2} d x\)

It is known that, (because \(\int \sqrt{x^2-a^2} d x=\frac{x}{2} \sqrt{x^2-a^2}-\frac{a^2}{2} \log \left|x+\sqrt{x^2-a^2}\right|+C\))

∴ I = \(\frac{(x+2)}{2} \sqrt{x^2+4 x+1}-\frac{3}{2} \log \left|(x+2)+\sqrt{x^2+4 x+1}\right|+C\)

Question 5. \(\int \sqrt{1-4 x-x^2} d x\)
Solution:

Let \(\mathrm{I}=\int \sqrt{1-4 \mathrm{x}-\mathrm{x}^2} d \mathrm{x}=\int \sqrt{1-\left(\mathrm{x}^2+4 \mathrm{x}+4-4\right)} \mathrm{dx}\)

= \(\int \sqrt{1+4-(\mathrm{x}+2)^2} \mathrm{dx}=\int \sqrt{(\sqrt{5})^2-(\mathrm{x}+2)^2} \mathrm{dx}\)

It is known that, \(\int \sqrt{a^2-x^2} d x=\frac{x}{2} \sqrt{a^2-x^2}+\frac{a^2}{2} \sin ^{-1}\left(\frac{x}{a}\right)+C\)

∴ I = \(\frac{(x+2)}{2} \sqrt{1-4 x-x^2}+\frac{5}{2} \sin ^{-1}\left(\frac{x+2}{\sqrt{5}}\right)+C\)

Question 6. \(\int \sqrt{x^2+4 x-5} d x\)
Solution:

Let \(I=\int \sqrt{x^2+4 x-5} d x=\int \sqrt{\left(x^2+4 x+4-9\right)} d x=\int \sqrt{(x+2)_{-}^2-(3)^2} d x\)

It is known that, \(\int \sqrt{x^2-a^2} d x=\frac{x}{2} \sqrt{x^2-a^2}-\frac{a^2}{2} \log \left|x+\sqrt{x^2-a^2}\right|+C\)

∴ I = \(\frac{(x+2)}{2} \sqrt{x^2+4 x-5}-\frac{9}{2} \log \left|(x+2)+\sqrt{x^2+4 x-5}\right|+C\)

Question 7. \(\int \sqrt{1+3 \mathrm{x}-\mathrm{x}^2} \mathrm{dx}\)
Solution:

Let \(I=\int \sqrt{1+3 x-x^2} d x=\int \sqrt{1-\left(x^2-3 x+\frac{9}{4}-\frac{9}{4}\right)} d x\)

= \(\int \sqrt{\left(1+\frac{9}{4}\right)-\left(x-\frac{3}{2}\right)^2} d x=\int \sqrt{\left(\frac{\sqrt{13}}{2}\right)^2-\left(x-\frac{3}{2}\right)^2} d x\)

It is known that, \(\left\{\int \sqrt{a^2-x^2} d x=\frac{x}{2} \sqrt{a^2-x^2}+\frac{a^2}{2} \sin ^{-1}\left(\frac{x}{a}\right)+C\right\}\)

∴ I = \(\frac{\left(x-\frac{3}{2}\right)}{2} \sqrt{1+3 x-x^2}+\frac{13}{4 \times 2} \sin ^{-1}\left(\frac{x-\frac{3}{2}}{\frac{\sqrt{13}}{2}}\right)+C\)

= \(\frac{2 x-3}{4} \sqrt{1+3 x-x^2}+\frac{13}{8} \sin ^{-1}\left(\frac{2 x-3}{\sqrt{13}}\right)+C\)

Question 8. \(\int \sqrt{\mathrm{x}^2+3 \mathrm{x}} d \mathrm{x}\)
Solution:

Let \(I=\int \sqrt{x^2+3 x} d x=\int \sqrt{x^2+3 x+\frac{9}{4}-\frac{9}{4}} d x=\int \sqrt{\left(x+\frac{3}{2}\right)^2-\left(\frac{3}{2}\right)^2} d x\)

It is known that, (because \(\int \sqrt{\mathrm{x}^2-\mathrm{a}^2} \mathrm{dx}=\frac{\mathrm{x}}{2} \sqrt{\mathrm{x}^2-\mathrm{a}^2}-\frac{\mathrm{a}^2}{2} \log \left|\mathrm{x}+\sqrt{\mathrm{x}^2-\mathrm{a}^2}\right|+\mathrm{C}\}\)

∴I = \(\frac{\left(x+\frac{3}{2}\right)}{2} \sqrt{x^2+3 x}-\frac{9}{2} \log \left|\left(x+\frac{3}{2}\right)+\sqrt{x^2+3 x}\right|+C\)

= \(\frac{(2 x+3)}{4} \sqrt{x^2+3 x}-\frac{9}{8} \log \left|\left(x+\frac{3}{2}\right)+\sqrt{x^2+3 x}\right|+C\)

Question 9. \(\int \sqrt{1+\frac{x^2}{9}} d x\)
Solution:

Let \(\mathrm{I}=\int \sqrt{1+\frac{\mathrm{x}^2}{9}} \mathrm{dx}=\frac{1}{3} \int \sqrt{9+\mathrm{x}^2} d x=\frac{1}{3} \int \sqrt{(3)^2+\mathrm{x}^2} d x\)

It is known that, \(\int \sqrt{x^2+a^2} d x=\frac{x}{2} \sqrt{x^2+a^2}+\frac{a^2}{2} \log \left|x+\sqrt{x^2+a^2}\right|+C\)

∴ I = \(\frac{1}{3}\left[\frac{x}{2} \sqrt{x^2+9}+\frac{9}{2} \log \left|x+\sqrt{x^2+9}\right|\right]+C=\frac{x}{6} \sqrt{x^2+9}+\frac{3}{2} \log \left|x+\sqrt{x^2+9}\right|+C\)

Choose The Correct Answer

Question 10. \(\int \sqrt{1+\mathrm{x}^2} \mathrm{dx}\) is equal to?

  1. \(\frac{\mathrm{x}}{2} \sqrt{1+\mathrm{x}^2}+\frac{1}{2} \log \left|\mathrm{x}+\sqrt{1+\mathrm{x}^2}\right|+\mathrm{C}\)
  2. \(\frac{2}{3}\left(1+x^2\right)^{\frac{3}{2}}+C\)
  3. \(\frac{2}{3} x\left(1+x^2\right)^{\frac{3}{2}}+C\)
  4. \(\frac{x^2}{2} \sqrt{1+x^2}+\frac{1}{2} x^2 \log \left|x+\sqrt{1+x^2}\right|+C\)

Solution:

It is known that, \(\int \sqrt{a^2+x^2} d x=\frac{x}{2} \sqrt{a^2+x^2}+\frac{a^2}{2} \log \left|x+\sqrt{x^2+a^2}\right|+C\)

∴ \(\int \sqrt{1+\mathrm{x}^2} \mathrm{dx}=\frac{\mathrm{x}}{2} \sqrt{1+\mathrm{x}^2}+\frac{1}{2} \log \left|\mathrm{x}+\sqrt{1+\mathrm{x}^2}\right|+\mathrm{C}\).

Hence, the correct answer is (1).

Question 11. \(\int \sqrt{x^2-8 x+7} d x\) is equal to ?

  1. \(\frac{1}{2}(x-4) \sqrt{x^2-8 x+7}+9 \log \left|x-4+\sqrt{x^2-8 x+7}\right|+C\)
  2. \(\frac{1}{2}(x+4) \sqrt{x^2-8 x+7}+9 \log \left|x+4+\sqrt{x^2-8 x+7}\right|+C\)
  3. \(\frac{1}{2}(x-4) \sqrt{x^2-8 x+7}-3 \sqrt{2} \log \left|x-4+\sqrt{x^2-8 x+7}\right|+C\)
  4. \(\frac{1}{2}(x-4) \sqrt{x^2-8 x+7}-\frac{9}{2} \log \left|x-4+\sqrt{x^2-8 x+7}\right|+C\)

Solution:

Let \(\mathrm{I}=\int \sqrt{\mathrm{x}^2-8 \mathrm{x}+7} \mathrm{dx}=\int \sqrt{\left(\mathrm{x}^2-8 \mathrm{x}+16\right)-9} \mathrm{dx}=\int \sqrt{(\mathrm{x}-4)^2-(3)^2} \mathrm{dx}\)

It is known that, \(\int \sqrt{\mathrm{x}^2-\mathrm{a}^2} \mathrm{dx}=\frac{\mathrm{x}}{2} \sqrt{\mathrm{x}^2-\mathrm{a}^2}-\frac{\mathrm{a}^2}{2} \log \left|\mathrm{x}+\sqrt{\mathrm{x}^2-\mathrm{a}^2}\right|+\mathrm{C}\)

∴ \(I=\frac{(x-4)}{2} \sqrt{x^2-8 x+7}-\frac{9}{2} \log \left|(x-4)+\sqrt{x^2-8 x+7}\right|+C\).

Hence, the correct answer is (4).

Integrals Exercise 7.8

Evaluate The Definite Integrals

Question 1. \(\int_{-1}^1(x+1) d x\)
Solution:

Let \(\int_{-1}^1(x+1) d x\)=\(\left(\frac{\mathrm{x}^2}{2}+\mathrm{x}\right)_{-1}^1=\left(\frac{1}{2}+1\right)-\left(\frac{1}{2}-1\right)=\frac{1}{2}+1-\frac{1}{2}+\mathrm{I}=2\)

Question 2. \(\int_2^3 \frac{1}{\mathrm{x}} \mathrm{dx}\)
Solution:

Let \(I=\int_2^3 \frac{1}{x} d x=[\log |x|]_2^3=\log |3|-\log |2|=\log \frac{3}{2}\)

Question 3. \(\int_1^2\left(4 x^3-5 x^2+6 x+9\right) d x\)
Solution:

Let \(\mathrm{I}=\int_1^2\left(4 \mathrm{x}^3-5 \mathrm{x}^2+6 \mathrm{x}+9\right) \mathrm{dx}\)

⇒ \(\int_1^2\left(4 x^3-5 x^2+6 x+9\right) d x=\left[4\left(\frac{x^4}{4}\right)-5\left(\frac{x^3}{3}\right)+6\left(\frac{x^2}{2}\right)+9(x)\right]_1^2\)

I = \(\left\{2^4-\frac{5 \cdot(2)^3}{3}+3(2)^2+9(2)\right\}-\left\{(1)^4-\frac{5(1)^3}{3}+3(1)^2+9(1)\right\}\)

= \(\left(16-\frac{40}{3}+12+18\right)-\left(1-\frac{5}{3}+3+9\right)=16-\frac{40}{3}+12+18-1+\frac{5}{3}-3-9\)

= \(33-\frac{35}{3}=\frac{99-35}{3}=\frac{64}{3}\)

Question 4. \(\int_0^{\frac{\pi}{4}} \sin 2 x d x\)
Solution:

Let \(I=\int_0^{\frac{\pi}{4}} \sin 2 x d x=\left(\frac{-\cos 2 x}{2}\right)_0^{\frac{\pi}{4}}=-\frac{1}{2}\left[\cos 2\left(\frac{\pi}{4}\right)-\cos 0\right]=-\frac{1}{2}\left[\cos \left(\frac{\pi}{2}\right)-\cos 0\right]\)

= \(-\frac{1}{2}[0-1]=\frac{1}{2}\)

Question 5. \(\int_0^{\frac{\pi}{2}} \cos 2 x d x\)
Solution:

Let \(I=\int_0^{\frac{\pi}{2}} \cos 2 x d x=\left(\frac{\sin 2 x}{2}\right)_0^{\frac{\pi}{2}}=\frac{1}{2}\left[\sin 2\left(\frac{\pi}{2}\right)-\sin 0\right]=\frac{1}{2}[\sin \pi-\sin 0]=\frac{1}{2}[0-0]=0\)

Question 6. \(\int_4^5 e^x d x\)
Solution:

Let \(\mathrm{I}=\int_4^5 \mathrm{e}^{\mathrm{x}} \mathrm{dx}=\left(\mathrm{e}^{\mathrm{x}}\right)_4^5=\mathrm{e}^5-\mathrm{e}^4=\mathrm{e}^4(\mathrm{e}-1)\)

Question 7. \(\int_0^{\frac{\pi}{4}} \tan x d x\)
Solution:

Let I = \(\int_0^{\frac{\pi}{4}} \tan x d x=[-\log |\cos x|]_0^{\frac{\pi}{4}}=-\log \left|\cos \frac{\pi}{4}\right|+\log |\cos 0|\)

= \(-\log \left|\frac{1}{\sqrt{2}}\right|+\log |1|\) (because log (1)=0)

= \(-\log (2)^{-\frac{1}{2}}=\frac{1}{2} \log 2\)

Question 8. \(\int_{\frac{\pi}{6}}^{\frac{\pi}{4}} \mathrm{cosec} x d x\)
Solution:

Let \(I =\int_{\frac{\pi}{6}}^{\frac{\pi}{4}} \mathrm{cosec} x d x=[\log |\mathrm{cosec} x-\cot x|]_{\frac{\pi}{6}}^{\frac{\pi}{4}}\)

= \(\log \left|\mathrm{cosec} \frac{\pi}{4}-\cot \frac{\pi}{4}\right|-\log \left|\mathrm{cosec} \frac{\pi}{6}-\cot \frac{\pi}{6}\right|\)

= \(\log |\sqrt{2}-1|-\log |2-\sqrt{3}|=\log \left(\frac{\sqrt{2}-1}{2-\sqrt{3}}\right)\)

Question 9. \(\int_0^1 \frac{\mathrm{dx}}{\sqrt{1-\mathrm{x}^2}}\)
Solution:

Let \(\mathrm{I}=\int_0^1 \frac{\mathrm{dx}}{\sqrt{1-\mathrm{x}^2}}=\left[\sin ^{-1} \mathrm{x}\right]_0^{\mathrm{t}}=\sin ^{-1}(1)-\sin ^{-1}(0)=\frac{\pi}{2}-0=\frac{\pi}{2}\)

Question 10. \(\int_0^1 \frac{\mathrm{dx}}{1+\mathrm{x}^2}\)
Solution:

Let \(\mathrm{I}=\int_0^1 \frac{\mathrm{dx}}{1+\mathrm{x}^2}=\left[\tan ^{-1} \mathrm{x}\right]_0^1\)

= \(\tan ^{-1}(1)-\tan ^{-1}(0)=\frac{\pi}{4}\)

Question 11. \(\int_2^3 \frac{\mathrm{dx}}{\mathrm{x}^2-1}\)
Solution:

Let \(I=\int_2^3 \frac{d x}{x^2-1}=\left[\frac{1}{2} \log \left|\frac{x-1}{x+1}\right|\right]_2^3\)

= \(\frac{1}{2}\left[\log \left|\frac{3-1}{3+1}\right|-\log \left|\frac{2-1}{2+1}\right|\right]=\frac{1}{2}\left[\log \left|\frac{2}{4}\right|-\log \left|\frac{1}{3}\right|\right]\)

= \(\frac{1}{2}\left[\log \frac{1}{2}-\log \frac{1}{3}\right]=\frac{1}{2}\left[\log \frac{3}{2}\right]\)

Question 12. \(\int_0^{\frac{\pi}{2}} \cos ^2 x d x\)
Solution:

Let \(I=\int_0^{\frac{\pi}{2}} \cos ^2 x d x\)

I = \(\int_0^{\frac{\pi}{2}}\left(\frac{1+\cos 2 x}{2}\right) d x=\frac{1}{2}\left(x+\frac{\sin 2 x}{2}\right)_0^{\frac{\pi}{2}}\)

= \(\frac{1}{2}\left[\left(\frac{\pi}{2}+\frac{\sin \pi}{2}\right)-\left(0+\frac{\sin 0}{2}\right)\right]=\frac{1}{2}\left[\frac{\pi}{2}+0-0-0\right]=\frac{\pi}{4}\)

Question 13. \(\int_2^3 \frac{x d x}{x^2+1}\)
Solution:

Let \(I=\int_2^3 \frac{\mathrm{x}}{\mathrm{x}^2+1} \mathrm{dx}=\frac{1}{2} \int_2^3 \frac{2 \mathrm{x}}{\mathrm{x}^2+1} \mathrm{dx}=\frac{1}{2}\left[\log \left(1+\mathrm{x}^2\right)\right]_2^3\)

(because \(\mathrm{x}^2+1=\mathrm{t}, 2 \mathrm{x} d \mathrm{x}=\mathrm{dt}\))

= \(\frac{1}{2}\left[\log \left(1+(3)^2\right)-\log \left(1+(2)^2\right)\right]=\frac{1}{2}[\log (10)-\log (5)]=\frac{1}{2} \log \left(\frac{10}{5}\right)=\frac{1}{2} \log 2\)

Question 14. \(\int_0^1 \frac{2 x+3}{5 x^2+1} \mathrm{dx}\)
Solution:

Let \(\mathrm{I}=\int_0^1 \frac{2 \mathrm{x}+3}{5 \mathrm{x}^2+1} \mathrm{dx}=\frac{1}{5} \int_0^1 \frac{5(2 \mathrm{x}+3)}{5 \mathrm{x}^2+1} \mathrm{dx}=\frac{1}{5} \int_0^1 \frac{(10 \mathrm{x}+15)}{5 \mathrm{x}^2+1} \mathrm{dx}\)

= \(\frac{1}{5} \int_0^1 \frac{10 \mathrm{x}}{5 \mathrm{x}^2+1} \mathrm{dx}+3 \int_0^1 \frac{1}{5 \mathrm{x}^2+1} \mathrm{dx}\)

= \(\frac{1}{5} \int_0^1 \frac{10 \mathrm{x}}{5 \mathrm{x}^2+1} \mathrm{dx}+3 \int_0^1 \frac{1}{5\left(\mathrm{x}^2+\left(\frac{1}{\sqrt{5}}\right)^2\right)} \mathrm{dx}\)

= \(\frac{1}{5}\left[\log \left(5 \mathrm{x}^2+1\right)\right]_0^1+\frac{3}{5} \cdot \frac{1}{\frac{1}{\sqrt{5}}}\left[\tan ^{-1}\left(\frac{\mathrm{x}}{\frac{1}{\sqrt{5}}}\right)\right]_0^1\)

= \(\frac{1}{5}\left[\log \left(5 \mathrm{x}^2+1\right)\right]_0^1+\frac{3}{\sqrt{5}}\left[\tan ^{-1}(\sqrt{5} \mathrm{x})\right]_1^1\)

= \(\left\{\frac{1}{5} \log (5+1)-\frac{1}{5} \log (1)\right\}+\left\{\frac{3}{\sqrt{5}} \tan ^{-1}(\sqrt{5})-\frac{3}{\sqrt{5}} \tan ^{-1}(0)\right\}\)

= \(\frac{1}{5} \log 6+\frac{3}{\sqrt{5}} \tan ^{-1} \sqrt{5}\) (because log (1)=0 and \(\tan ^{-1}(0)=0\))

Question 15. \(\int_0^1 x e^{x^2} d x\)
Solution:

Let \(\mathrm{I}=\int_0^1 x \mathrm{e}^{x^2} \mathrm{dx}\)

Put \(\mathrm{x}^2=\mathrm{t} \Rightarrow 2 \mathrm{xdx}=\mathrm{dt}\)

As \(x \rightarrow 0, t \rightarrow 0\) and as \(x \rightarrow 1, t \rightarrow 1\)

∴ \(\mathrm{I}=\frac{1}{2} \int_0^1 \mathrm{e}^{\mathrm{t}} \mathrm{dt}=\frac{1}{2}\left(\mathrm{e}^{\mathrm{t}}\right)_0^1=\frac{1}{2} \mathrm{e}-\frac{1}{2} \mathrm{e}^0=\frac{1}{2}(\mathrm{e}-1)\)

Question 16. \(\int_1^2 \frac{5 x^2}{x^2+4 x+3} d x\)
Solution:

Let \(\mathrm{I}=\int_1^2 \frac{5 \mathrm{x}^2}{\mathrm{x}^2+4 \mathrm{x}+3} \mathrm{dx}\)

Dividing \(5 x^2\) by \(x^2+4 x+3\), we obtain

I = \(\int_1^2\left\{5-\frac{20 x+15}{x^2+4 x+3}\right\} d x=\int_1^2 5 d x-\int_1^2 \frac{20 x+15}{x^2+4 x+3} d x=[5 x]_1^2-\int_1^2 \frac{20 x+15}{x^2+4 x+3} d x\)

⇒ \(\mathrm{I}=5-\mathrm{I}_1\), where \(\mathrm{I}_{\mathrm{t}}=\int_1^2 \frac{20 \mathrm{x}+15}{\mathrm{x}^2+4 \mathrm{x}+3} \mathrm{dx}\)….(1)

Consider \(I_1=\int_1^2 \frac{20 x+15}{x^2+4 x+3} d x\)

Let \(20 \mathrm{x}+15=\mathrm{A} \frac{\mathrm{d}}{\mathrm{dx}}\left(\mathrm{x}^2+4 \mathrm{x}+3\right)+\mathrm{B}=2 \mathrm{Ax}+(4 \mathrm{~A}+\mathrm{B})\)

Equating the coefficients of x and the constant term, we obtain

A = 10 and B=-25

⇒ \(I_1=10 \int_1^2 \frac{2 x+4}{x^2+4 x+3} d x-25 \int_1^2 \frac{d x}{x^2+4 x+3}\)

Let \(\mathrm{x}^2+4 \mathrm{x}+3=\mathrm{t}\)

⇒ \((2 \mathrm{x}+4) \mathrm{dx}=\mathrm{dt}\)

⇒ \(\mathrm{I}_1=10 \int_8^{15} \frac{\mathrm{dt}}{\mathrm{t}}-25 \int_1^2 \frac{\mathrm{dx}}{(\mathrm{x}+2)^2-1^2}=10[\log \mathrm{t}]_{\mathrm{s}}^{15}-25\left[\frac{1}{2} \log \left(\frac{\mathrm{x}+2-1}{\mathrm{x}+2+1}\right)\right]_1^2\)

= \([10 \log 15-10 \log 8]-25\left[\frac{1}{2} \log \frac{3}{5}-\frac{1}{2} \log \frac{2}{4}\right]\)

= \([10 \log (5 \times 3)-10 \log (4 \times 2)]-\frac{25}{2}[\log 3-\log 5-\log 2+\log 4] \)

= \([10 \log 5+10 \log 3-10 \log 4-10 \log 2]-\frac{25}{2}[\log 3-\log 5-\log 2+\log 4]\)

= \(\left[10+\frac{25}{2}\right] \log 5+\left[-10-\frac{25}{2}\right] \log 4+\left[10-\frac{25}{2}\right] \log 3+\left[-10+\frac{25}{2}\right] \log 2\)

= \(\frac{45}{2} \log 5-\frac{45}{2} \log 4-\frac{5}{2} \log 3+\frac{5}{2} \log 2=\frac{45}{2} \log \frac{5}{4}-\frac{5}{2} \log \frac{3}{2}\)

Substituting the value of \(I_1\) in (1), we obtain

I = \(5-\left[\frac{45}{2} \log \frac{5}{4}-\frac{5}{2} \log \frac{3}{2}\right]=5-\frac{5}{2}\left[9 \log \frac{5}{4}-\log \frac{3}{2}\right]\)

Question 17. \(\int_0^{\frac{\pi}{4}}\left(2 \sec ^2 x+x^3+2\right) d x\)
Solution:

Let \(I=\int_0^{\frac{\pi}{4}}\left(2 \sec ^2 x+x^3+2\right) d x=\left(2 \tan x+\frac{x^4}{4}+2 x\right)_0^{\frac{\pi}{4}}\)

= \(\left\{\left(2 \tan \frac{\pi}{4}+\frac{1}{4}\left(\frac{\pi}{4}\right)^4+2\left(\frac{\pi}{4}\right)\right)-(2 \tan 0+0+0)\right\}=2 \tan \frac{\pi}{4}+\frac{\pi^4}{4^5}+\frac{\pi}{2}=2+\frac{\pi}{2}+\frac{\pi^4}{1024}\)

Question 18. \(\int_0^\pi\left(\sin ^2 \frac{x}{2}-\cos ^2 \frac{x}{2}\right) d x\)
Solution:

Let \(I=\int_0^\pi\left(\sin ^2 \frac{x}{2}-\cos ^2 \frac{x}{2}\right) d x=-\int_0^\pi\left(\cos ^2 \frac{x}{2}-\sin ^2 \frac{x}{2}\right) d x\)

= \(-\int_0^\pi \cos x d x=(-\sin x)_0^\pi=-[\sin \pi-\sin 0]=0\)

Question 19. \(\int_0^2 \frac{6 x+3}{x^2+4} d x\)
Solution:

Let \(I=\int_0^2 \frac{6 x+3}{x^2+4} d x=3 \int_0^2 \frac{2 x+1}{x^2+4} d x=3 \int_0^2 \frac{2 x}{x^2+4} d x+3 \int_0^2 \frac{1}{x^2+2^2} d x\)

(because \(\int \frac{d x}{a^2+x^2}=\frac{1}{a} \tan ^{-1} \frac{x}{a}\))

= \(3\left[\log \left(x^2+4\right)\right]_0^2+\frac{3}{2}\left(\tan ^{-1} \frac{x}{2}\right)_0^2\)

= \(\left\{3 \log \left(2^2+4\right)+\frac{3}{2} \tan ^{-1}\left(\frac{2}{2}\right)\right\}-\left\{3 \log (0+4)+\frac{3}{2} \tan ^{-1}\left(\frac{0}{2}\right)\right\}\)

= \(3 \log 8+\frac{3}{2} \tan ^{-1} 1-3 \log 4-\frac{3}{2} \tan ^{-1} 0=3 \log 8+\frac{3}{2}\left(\frac{\pi}{4}\right)-3 \log 4-0\)

= \(3 \log \left(\frac{8}{4}\right)+\frac{3 \pi}{8}=3 \log 2+\frac{3 \pi}{8}\)

Question 20. \(\int_0^1\left(x e^x+\sin \frac{\pi x}{4}\right) d x\)
Solution:

Let \(\mathrm{I}=\int_0^1\left(x e^x+\sin \frac{\pi x}{4}\right) \mathrm{dx}=\left(x \mathrm{e}^{\mathrm{x}}\right)_0^1-\int_0^1 \mathrm{e}^{\mathrm{x}} \mathrm{dx}-\left[\frac{4}{\pi} \cos \frac{\pi \mathrm{x}}{4}\right]_0^1\)

= \(\left[x e^x-e^x-\frac{4}{\pi} \cos \frac{\pi x}{4}\right]_0^1=\left(1 \cdot e^1-e^1-\frac{4}{\pi} \cos \frac{\pi}{4}\right)-\left(0 \cdot e^0-e^0-\frac{4}{\pi} \cos 0\right)\)

= \(e-e-\frac{4}{\pi}\left(\frac{1}{\sqrt{2}}\right)+1+\frac{4}{\pi}=1+\frac{4}{\pi}-\frac{2 \sqrt{2}}{\pi}\)

Choose The Correct Answer

Question 21. \(\int_1^{\sqrt{3}} \frac{\mathrm{dx}}{1+\mathrm{x}^2}\) equals

  1. \(\frac{\pi}{3}\)
  2. \(\frac{2 \pi}{3}\)
  3. \(\frac{\pi}{6}\)
  4. \(\frac{\pi}{12}\)

Solution: 4. \(\frac{\pi}{12}\)

I = \(\int_1^{\sqrt{3}} \frac{\mathrm{dx}}{1+\mathrm{x}^2}=\left(\tan ^{-1} \mathrm{x}\right)_1^{\sqrt{3}}=\tan ^{-1} \sqrt{3}-\tan ^{-1} 1=\frac{\pi}{3}-\frac{\pi}{4}=\frac{\pi}{12}\).

Hence, the correct answer is (4).

Question 22. \(\int_0^{\frac{2}{3}} \frac{\mathrm{dx}}{4+9 \mathrm{x}^2}\) equals?

  1. \(\frac{\pi}{6}\)
  2. \(\frac{\pi}{12}\)
  3. \(\frac{\pi}{24}\)
  4. \(\frac{\pi}{4}\)

Solution: 3. \(\frac{\pi}{24}\)

I = \(\int_0^{\frac{2}{3}} \frac{d x}{4+9 x^2}=\int_0^{\frac{2}{3}} \frac{d x}{(2)^2+(3 x)^2}=\frac{1}{3}\left[\frac{1}{2} \tan ^{-1} \frac{3 x}{2}\right]_0^{\frac{2}{3}}=\frac{1}{6}\left[\tan ^{-1}\left(\frac{3 x}{2}\right)\right]_0^{\frac{2}{3}}\)

= \(\frac{1}{6} \tan ^{-1}\left(\frac{3}{2} \cdot \frac{2}{3}\right)-\frac{1}{6} \tan ^{-1} 0=\frac{1}{6} \tan ^{-1} 1-0=\frac{1}{6} \times \frac{\pi}{4}=\frac{\pi}{24}\).

Hence, the correct answer is (3).

Integrals Exercise 7.9

Evaluate The Integrals

Question 1. \(\int_0^1 \frac{x}{x^2+1} d x\)
Solution:

Let \(I=\int_0^1 \frac{x}{x^2+1} d x\)

Put \(\mathrm{x}^2+1=\mathrm{t} \Rightarrow 2 \mathrm{xdx}=\mathrm{dt}\)

When x=0, t=1 and when x=1, t=2

⇒ \(\int_0^1 \frac{\mathrm{x}}{\mathrm{x}^2+1} \mathrm{dx}=\frac{1}{2} \int_1^2 \frac{\mathrm{dt}}{\mathrm{t}}=\frac{1}{2}[\log |\mathrm{t}|]_1^2=\frac{1}{2}[\log 2-\log 1]=\frac{1}{2} \log 2\)

Question 2. \(\int_0^{\frac{\pi}{2}} \sqrt{\sin \phi} \cos ^5 \phi \mathrm{d} \phi\)
Solution:

Let \(I=\int_0^{\frac{\pi}{2}} \sqrt{\sin \phi} \cos ^5 \phi \mathrm{d} \phi=\int_0^{\frac{\pi}{2}} \sqrt{\sin \phi} \cos ^4 \phi \cos \phi \mathrm{d} \phi\)

= \(\int_0^{\pi / 2} \sqrt{\sin \phi}\left(1-\sin ^2 \phi\right)^2 \cdot \cos \phi d \phi\)

(because \(\cos ^2 x=1-\sin ^2 x\))

Put \(\sin \phi=\mathrm{t} \Rightarrow \cos \phi \mathrm{d} \phi=\mathrm{dt}\)

When \(\phi=0, t=0\) and when \(\phi=\frac{\pi}{2}, \mathrm{t}=1\)

∴ I = \(\int_0^1 \sqrt{t}\left(1-t^2\right)^2 d t=\int_0^1 t^{\frac{1}{2}}\left(1+t^4-2 t^2\right) d t=\int_0^1\left[t^{\frac{1}{2}}+t^{\frac{9}{2}}-2 t^{\frac{5}{2}}\right] d t\)

= \(\left[\frac{t^{\frac{3}{2}}}{\frac{3}{2}}+\frac{t^{\frac{11}{2}}}{\frac{11}{2}}-\frac{2 t^{\frac{7}{2}}}{\frac{7}{2}}\right]_0^1=\frac{2}{3}+\frac{2}{11}-\frac{4}{7}=\frac{154+42-132}{231}=\frac{64}{231}\)

Question 3. \(\int_0^1 \sin ^{-1}\left(\frac{2 x}{1+x^2}\right) d x\)
Solution:

Let \(I=\int_0^1 \sin ^{-1}\left(\frac{2 x}{1+x^2}\right) d x\)

Put \(x=\tan \theta \Rightarrow d x=\sec ^2 \theta d \theta\)

When \(\mathrm{x}=0, \theta=0\) and when \(\mathrm{x}=1, \theta=\frac{\pi}{4}\)

∴ I = \(\int_0^{\frac{\pi}{4}} \sin ^{-1}\left(\frac{2 \tan \theta}{1+\tan ^2 \theta}\right) \sec ^2 \theta d \theta \Rightarrow I=\int_0^{\frac{\pi}{4}} \sin ^{-1}(\sin 2 \theta) \sec ^2 \theta d \theta\)

I = \(\int_0^{\frac{\pi}{4}} 2 \theta \cdot \sec ^2 \theta d \theta=2 \int_0^{\frac{\pi}{4}} \theta \cdot \sec ^2 \theta d \theta\)

Taking \(\theta\) as first function and \(\sec ^2 \theta\) as second function and integrating by parts, we obtain

I = \(2\left[\theta \int \sec ^2 \theta d \theta-\int\left\{\left(\frac{d}{d \theta}(\theta)\right) \int \sec ^2 \theta d \theta\right\} d \theta\right]_0^{\frac{\pi}{4}}=2\left[\theta \tan \theta-\int \tan \theta d \theta\right]_0^{\pi / 4}\)

= \(2[\theta \tan \theta+\log |\cos \theta|]_0^{\frac{\pi}{4}}=2\left[\frac{\pi}{4} \tan \frac{\pi}{4}+\log \left|\cos \frac{\pi}{4}\right|-\log |\cos 0|\right]\)

= \(2\left[\frac{\pi}{4}+\log \left(\frac{1}{\sqrt{2}}\right)-\log 1\right]=2\left[\frac{\pi}{4}-\frac{1}{2} \log 2\right]=\frac{\pi}{2}-\log 2\)

Question 4. \(\int_0^2 x \sqrt{x+2} d x\).
Solution:

Let \(\mathrm{I}=\int_0^2 \mathrm{x} \sqrt{\mathrm{x}+2} \mathrm{dx}\)

Put \(x+2=t^2 \Rightarrow d x=2 t d t\), when, \(x=0, t=\sqrt{2}\) and when \(x^2=2, t=2\)

∴ \(\int_1^2 x \sqrt{x+2} d x=\int_{\sqrt{2}}^2\left(t^2-2\right) \sqrt{t^2} 2 t d t=2 \int_{\sqrt{2}}^2\left(t^2-2\right) t^2 d t=2 \int_{\sqrt{2}}^2\left(t^4-2 t^2\right) d t\)

= \(2\left[\frac{t^5}{5}-\frac{2 t^3}{3}\right]_{\sqrt{2}}^2=2\left[\frac{32}{5}-\frac{16}{3}-\frac{4 \sqrt{2}}{5}+\frac{4 \sqrt{2}}{3}\right]=2\left[\frac{96-80-12 \sqrt{2}+20 \sqrt{2}}{15}\right]\)

= \(2\left[\frac{16+8 \sqrt{2}}{15}\right]=\frac{16(2+\sqrt{2})}{15}=\frac{16 \sqrt{2}(\sqrt{2}+1)}{15}\)

Question 5. \(\int_0^{\frac{\pi}{2}} \frac{\sin x}{1+\cos ^2 x} d x\)
Solution:

Let \(\mathrm{I}=\int_0^{\frac{\pi}{2}} \frac{\sin \mathrm{x}}{1+\cos ^2 \mathrm{x}} \mathrm{dx}\)

Put cos x=t \Rightarrow-sin x d x=d t

When x=0, t=1 and when \(x=\frac{\pi}{2}, t=0\)

⇒ \(\int_0^{\frac{\pi}{2}} \frac{\sin \mathrm{x}}{1+\cos ^2 \mathrm{x}} \mathrm{dx}\)

= \(-\int_1^0 \frac{\mathrm{dt}}{1+\mathrm{t}^2}=-\left[\tan ^{-1} \mathrm{t}\right]_1^0=-\left[\tan ^{-1} 0-\tan ^{-1} 1\right]=-\left[-\frac{\pi}{4}\right]=\frac{\pi}{4}\)

Question 6. \(\int_0^2 \frac{d x}{x+4-x^2}\)
Solution:

Let \(\mathrm{I}=\int_0^2 \frac{\mathrm{dx}}{\mathrm{x}+4-\mathrm{x}^2}=\int_0^2 \frac{\mathrm{dx}}{-\left(\mathrm{x}^2-\mathrm{x}-4\right)}\)

= \(\int_0^2 \frac{d x}{-\left(x^2-x+\frac{1}{4}-\frac{1}{4}-4\right)}=\int_0^2 \frac{d x}{-\left[\left(x-\frac{1}{2}\right)^2-\frac{17}{4}\right]}\)

= \(\int_0^2 \frac{d x}{\left(\frac{\sqrt{17}}{2}\right)^2-\left(x-\frac{1}{2}\right)^2}\)

Question 7. \(\int_{-1}^1 \frac{d x}{x^2+2 x+5}\)
Solution:

Let \(I=\int_{-1}^1 \frac{d x}{x^2+2 x+5}=\int_{-1}^1 \frac{d x}{\left(x^2+2 x+1\right)+4}=\int_{-1}^1 \frac{d x}{(x+1)^2+(2)^2}\)

Put \(\mathrm{x}+1=\mathrm{t} \Rightarrow \mathrm{dx}=\mathrm{dt}\)

When x=-1, t=0 and when x=1, t=2

∴ \(\int_{-1}^1 \frac{\mathrm{dx}}{(\mathrm{x}+1)^2+(2)^2}=\int_0^2 \frac{\mathrm{dt}}{\mathrm{t}^2+2^2}\)

= \(\left[\frac{1}{2} \tan ^{-1} \frac{\mathrm{t}}{2}\right]_0^2=\frac{1}{2} \tan ^{-1} 1-\frac{1}{2} \tan ^{-1} 0=\frac{1}{2}\left(\frac{\pi}{4}\right)=\frac{\pi}{8}\)

Question 8. \(\int_1^2\left(\frac{1}{x}-\frac{1}{2 x^2}\right) e^{2 x} d x\)
Solution:

Let \(I=\int_1^2\left(\frac{1}{x}-\frac{1}{2 x^2}\right) e^{2 x} d x\)

Put \(2 \mathrm{x}=\mathrm{t} \Rightarrow 2 \mathrm{dx}=\mathrm{d} t\)

When x=1, t=2 and when x=2, t=4

∴ \(\int_1^2\left(\frac{1}{x}-\frac{1}{2 x^2}\right) e^{2 x} d x=\frac{1}{2} \int_2^4\left(\frac{2}{t}-\frac{2}{t^2}\right) e^t d t\)

= \(\int_2^4\left(\frac{1}{t}-\frac{1}{t^2}\right) e^t d t\) (therefore \(\int e^t\left[f(t)+f^{\prime}(t)\right] d t=e^1 f(t)+C\))

= \(\left[\frac{e^t}{t}\right]_2^4=\frac{e^4}{4}-\frac{e^2}{2}=\frac{e^2\left(e^2-2\right)}{4}\)

Choose The Correct Answer

Question 9. The value of the integral \(\int_{\frac{1}{3}}^1 \frac{\left(x-x^3\right)^{\frac{1}{3}}}{x^4} d x\) is ?

  1. 6
  2. 0
  3. 3
  4. 4

Solution:

Let \(I=\int_{\frac{1}{3}}^1 \frac{\left(x-x^3\right)^{\frac{1}{3}}}{x^4} d x \Rightarrow I=\int_{\frac{1}{3}}^1 \frac{x\left(\frac{1}{x^2}-1\right)^{\frac{1}{3}}}{x^4} d x \Rightarrow I=\int_{\frac{1}{3}}^1 \frac{\left(\frac{1}{x^2}-1\right)^{\frac{1}{3}}}{x^3} d x\)

Put \(\frac{1}{x^2}-1=t^3,-\frac{2}{x^3} \cdot d x=3 t^2 d t, \quad \frac{d x}{x^3}=\frac{-3}{2} t^2 d t\)

When x=1 then t=0 and when \(x=\frac{1}{3}\) then t=2

= \(-\frac{3}{2} \int_2^0\left(\mathrm{t}^3\right)^{1 / 3} \cdot \mathrm{t}^2 \mathrm{dt}=\frac{3}{2} \int_0^2 \mathrm{t}^3 \mathrm{dt}=\frac{3}{2}\left[\frac{\mathrm{t}^4}{4}\right]_0^2=\frac{3}{2} \times \frac{1}{4}\left[2^4-0\right]=\frac{3}{8} \times 16=6\)

Hence, the correct answer is (1).

Question 10. If \(f(x)=\int_0^x t \sin t d t\), then \(f^{\prime}(x)\) is

  1. \(\cos \mathrm{x}+\mathrm{x} \sin \mathrm{x}\)
  2. \(x \sin \mathrm{x}\)
  3. \(x \cos x\)
  4. \(\sin x+x \cos x\)

Solution: 2. \(x \sin \mathrm{x}\)

f(x) = \(\int_0^x t \sin t d t\)

Differentiation on both sides \(f^4(x)=(t \sin t)_0^x\)

∴ \(f^4(x)=x \sin x\)

Hence, the correct answer is (2).

Integrals Exercise 7.10

By using the properties of definite integrals.

Question 1. \(\int_0^{\frac{\pi}{2}} \cos ^2 x d x\)
Solution:

I = \(\int_0^{\frac{\pi}{2}} \cos ^2 x d x\)….(1)

⇒ I = \(\int_0^{\frac{\pi}{2}} \cos ^2\left(\frac{\pi}{2}-x\right) d x\) (because \(\int_0^a f(x) d x=\int_0^a f(a-x) d x\))

⇒ I = \(\int_0^{\frac{\pi}{2}} \sin ^2 x d x\)….(2)

Adding (1) and (2), we obtain

2I = \(\int_0^{\frac{\pi}{2}}\left(\sin ^2 x+\cos ^2 x\right) d x \Rightarrow 2 I=\int_0^{\frac{\pi}{2}} 1 \cdot d x\)

2I = \([x]_0^{\frac{\pi}{2}} \Rightarrow 2 I=\frac{\pi}{2} \Rightarrow I=\frac{\pi}{4}\)

Question 2. \(\int_0^{\frac{\pi}{2}} \frac{\sqrt{\sin x}}{\sqrt{\sin x}+\sqrt{\cos x}} d x\)
Solution:

Let \(I=\int_0^{\frac{\pi}{2}} \frac{\sqrt{\sin x}}{\sqrt{\sin x}+\sqrt{\cos x}} d x\)….(1)

I = \(\int_0^{\frac{\pi}{2}} \frac{\sqrt{\sin \left(\frac{\pi}{2}-x\right)}}{\sqrt{\sin \left(\frac{\pi}{2}-x\right)+\sqrt{\cos \left(\frac{\pi}{2}-x\right)}}} d x\)

⇒ I = \(\int_0^{\frac{\pi}{2}} \frac{\sqrt{\cos x}}{\sqrt{\cos x}+\sqrt{\sin x}} d x\)….(2)

Adding (1) and (2), we obtain

2I = \(int_0^{\frac{\pi}{2}} \frac{\sqrt{\sin x}+\sqrt{\cos x}}{\sqrt{\sin x}+\sqrt{\cos x}} d x\)

2I = \(\int_0^{\frac{\pi}{2}} 1 \cdot d x \Rightarrow 2 I=[x]_0^{\frac{\pi}{2}} \Rightarrow 2 I=\frac{\pi}{2} \Rightarrow I=\frac{\pi}{4}\)

Question 3. \(\int_0^{\frac{\pi}{2}} \frac{\sin ^{\frac{3}{2}} x d x}{\sin ^{\frac{3}{2}} x+\cos ^{\frac{3}{2}} x}\)
Solution:

Let \(I=\int_0^{\frac{\pi}{2}} \frac{\sin ^{\frac{3}{2}} x}{\sin ^{\frac{3}{2}} x+\cos ^{\frac{3}{2}} x} d x\)….(1)

⇒ I = \(\int_0^{\frac{\pi}{2}} \frac{\sin ^{\frac{3}{2}}\left(\frac{\pi}{2}-x\right)}{\sin ^{\frac{3}{2}}\left(\frac{\pi}{2}-x\right)+\cos ^{\frac{3}{2}}\left(\frac{\pi}{2}-x\right)} d x\)

(because \(\int_0^a f(x) d x=\int_0^a f(a-x) d x\))

⇒ I = \(\int_0^{\frac{\pi}{2}} \frac{\cos ^{\frac{3}{2}} \mathrm{x}}{\sin ^{\frac{3}{2}} \mathrm{x}+\cos ^{\frac{3}{2}} \mathrm{x}} \mathrm{dx}\)….(2)

Adding (1) and (2), we obtain

2I = \(int_0^{\frac{\pi}{2}} \frac{\sin ^{\frac{3}{2}} x+\cos ^{\frac{3}{2}} x}{\sin ^{\frac{3}{2}} x+\cos ^{\frac{3}{2}} x} d x \Rightarrow 2 I=\int_0^{\frac{\pi}{2}} 1 \cdot d x=[x]_0^{\frac{\pi}{2}} \Rightarrow 2 I=\frac{\pi}{2} \Rightarrow I=\frac{\pi}{4}\)

Question 4. \(\int_0^{\frac{\pi}{2}} \frac{\cos ^5 x d x}{\sin ^5 x+\cos ^5 x}\)
Solution:

Let \(I=\int_0^{\frac{\pi}{2}} \frac{\cos ^5 x}{\sin ^5 x+\cos ^5 x} d x\)

⇒ I = \(\int_0^{\frac{\pi}{2}} \frac{\cos ^5\left(\frac{\pi}{2}-x\right)}{\sin ^5\left(\frac{\pi}{2}-x\right)+\cos ^5\left(\frac{\pi}{2}-x\right)} d x\)

(because \(\int_0^a f(x) d x=\int_0^a f(a-x) d x\))

⇒ I = \(\int_0^{\frac{\pi}{2}} \frac{\sin ^5 x}{\sin ^5 x+\cos ^5 x} d x\)

Adding (1) and (2), we obtain

2I = \(\int_0^\pi 2 \frac{\sin ^5 x+\cos ^5 x}{\sin ^5 x+\cos ^5 x} d x \Rightarrow 2 I=\int_0^{\frac{\pi}{2}} 1 \cdot d x \Rightarrow 2 I=[x]_0^{\frac{\pi}{2}} \Rightarrow 2 I=\frac{\pi}{2} \Rightarrow I=\frac{\pi}{4}\)

Question 5. \(\int_{-5}^5|x+2| d x\)
Solution:

Let \(I=\int_{-5}^5|x+2| d x\)

I = \(\int_{-5}^{-2}|x+2| d x+\int_{-2}^5|x+2| d x\)

∴ I = \(\int_{-5}^{-2}-(x+2) d x+\int_{-2}^5(x+2) d x\) (because \(\int_a^b f(x) d x=\int_a^c f(x) d x+\int_c^b f(x) d x\))

I = \(-\left[\frac{x^2}{2}+2 x\right]_{-5}^{-2}+\left[\frac{x^2}{2}+2 x\right]_{-2}^5\)

= \(-\left[\frac{(-2)^2}{2}+2(-2)-\frac{(-5)^2}{2}-2(-5)\right]+\left[\frac{(5)^2}{2}+2(5)-\frac{(-2)^2}{2}-2(-2)\right]\)

= \(-\left[2-4-\frac{25}{2}+10\right]+\left[\frac{25}{2}+10-2+4\right]=-2+4+\frac{25}{2}-10+\frac{25}{2}+10-2+4\)

= 29

Question 6. \(\int_2^8|x-5| d x\)
Solution:

Let \(\mathrm{I}=\int_2^8|\mathrm{x}-5| \mathrm{dx}\)

I = \(\int_2^5|x-5| d x+\int_5^8|x-5| d x\)

I = \(\int_2^5-(x-5) d x+\int_5^8(x-5) d x\) (because \(\int_a^b f(x) d x=\int_a^c f(x) d x+\int_c^b f(x) d x\))

= \(-\left[\frac{x^2}{2}-5 x\right]_2^5+\left[\frac{x^2}{2}-5 x\right]_5^8\)

= \(-\left[\frac{25}{2}-25-2+10\right]+\left[32-40-\frac{25}{2}+25\right]=9\)

Question 7. \(\int_0^1 x(1-x)^n d x\)
Solution:

Let \(\mathrm{I}=\int_0^1 \mathrm{x}(1-\mathrm{x})^n \mathrm{dx}\)

∴ I = \(\int_0^1(1-x)(1-(1-x))^n d x\) (because \(\int_0^a f(x) d x=\int_0^a f(a-x) d x\))

= \(\int_0^1(1-x)(x)^n d x=\int_0^1\left(x^n-x^{n+1}\right) d x\)

= \(\left[\frac{x^{n+1}}{n+1}-\frac{x^{n+2}}{n+2}\right]_0^1=\left[\frac{1}{n+1}-\frac{1}{n+2}\right]\)

= \(\frac{(n+2)-(n+1)}{(n+1)(n+2)}=\frac{1}{(n+1)(n+2)}\)

Question 8. \(\int_0^{\frac{\pi}{4}} \log (1+\tan x) d x\)
Solution:

Let \(\mathrm{I}=\int_0^{\frac{\pi}{4}} \log (1+\tan \mathrm{x}) \mathrm{dx}\)

∴ I = \(\int_0^{\frac{\pi}{4}} \log \left[1+\tan \left(\frac{\pi}{4}-x\right)\right] d x \)

(because \(\int_0^\pi f(x) d x=\int_0^a f(a-x) d x\))

⇒ I = \(\int_0^{\frac{\pi}{4}} \log \left\{1+\frac{\tan \frac{\pi}{4}-\tan x}{1+\tan \frac{\pi}{4} \tan x}\right\} d x \Rightarrow I=\int_0^{\frac{\pi}{4}} \log \left\{1+\frac{1-\tan x}{1+\tan x}\right\} d x\)

⇒ I = \(\int_0^{\frac{\pi}{4}}\left\{\log \frac{2}{(1+\tan x)}\right\} d x \Rightarrow I=\int_0^{\frac{\pi}{4}} \log 2 d x-\int_0^{\frac{\pi}{4}} \log (1+\tan x) d x\)

⇒ I = \(\int_0^{\frac{\pi}{4}} \log 2 d x-I\) [From (1)]

⇒ 2I = \([x \log 2]_0^{\frac{\pi}{4}} \Rightarrow 2 I=\frac{\pi}{4} \log 2 \Rightarrow I=\frac{\pi}{8} \log 2\)

Question 9. \(\int_0^{-2} x \sqrt{2-x} d x\)
Solution:

Let \(\mathrm{I}=\int_0^2 \mathrm{x} \sqrt{2-\mathrm{x}} \mathrm{dx}\)

I = \(\int_0^2(2-x) \sqrt{x} d x\) (because \(\int_0^{11} f(x) d x=\int_0^0 f(a-x) d x\))

= \(\int_0^2\left\{2 x^{\frac{1}{2}}-x^{\frac{3}{2}}\right\} d x=\left[2\left(\frac{x^{\frac{3}{2}}}{\frac{3}{2}}\right)-\frac{x^{\frac{5}{2}}}{\frac{5}{2}}\right]_0^2\)

= \(\left[\frac{4}{3} x^{\frac{3}{2}}-\frac{2}{5} x^{\frac{5}{2}}\right]_0^2=\frac{4}{3}(2)^{\frac{3}{2}}-\frac{2}{5}(2)^{\frac{5}{2}}\)

= \(\frac{4 \times 2 \sqrt{2}}{3}-\frac{2}{5} \times 4 \sqrt{2}=\frac{8 \sqrt{2}}{3}-\frac{8 \sqrt{2}}{5}=\frac{40 \sqrt{2}-24 \sqrt{2}}{15}=\frac{16 \sqrt{2}}{15}\)

Question 10. \(\int_0^{\frac{\pi}{2}}(2 \log \sin x-\log \sin 2 x) d x\)
Solution:

Let \(I=\int_0^{\frac{\pi}{2}}(2 \log \sin x-\log \sin 2 x) d x\)

I = \(\int_0^{\frac{\pi}{2}} \log \left(\frac{\sin ^2 x}{\sin 2 x}\right) d x=\int_0^{\frac{\pi}{2}} \log \left(\frac{\sin ^2 x}{2 \sin x \cos x}\right) d x\)

= \(\int_0^{\frac{\pi}{2}} \log \left(\frac{\tan x}{2}\right) d x=\int_0^{\frac{\pi}{2}} \log \tan x d x-\int_0^{\frac{\pi}{2}} \log 2 d x\)

I = \(I^{\prime}-\log 2(x)^{\frac{\pi}{2}} \Rightarrow I=I^{\prime}-\frac{\pi}{2} \log 2\)….(1)

Now, \(I^{\prime}=\int_0^{\frac{\pi}{2}} \log \tan x d x\)….(2)

= \(\int_0^{\frac{\pi}{2}} \log \tan \left(\frac{\pi}{2}-x\right) d x\) (because \(\int_0^2 f(x) d x=\int_0^\pi f(a-x) d x\))

= \(\int_0^{\frac{\pi}{2}} \log \cot x d x\)….(3)

Adding equation (2) and (3)

⇒ \(2 I^{\prime}=\int_0^{\frac{\pi}{2}}(\log \tan x+\log \cot x) d x=\int_0^{\frac{\pi}{2}}(\log \tan x \cot x) d x\)

= \(\int_0^{\frac{\pi}{2}} \log 1 d x\)

2I’ =0

I’=0 put in equation (1)

I = \(0-\frac{\pi}{2} \log 2\)

I = \(\frac{\pi}{2} \log \frac{1}{2}\)

Question 11. \(\int_{\frac{-\pi}{2}}^{\frac{\pi}{2}} \sin ^2 x d x\)
Solution:

Let \(I=\int_{\frac{-\pi}{2}}^{\frac{\pi}{2}} \sin ^2 x d x \Rightarrow 2 \int_0^{\frac{\pi}{2}} \sin ^2 x d x\)….(1)

(\(\int_{-a}^2 f(x) d x=2 \int_0^n f(x) d x\), when f(x) is even function

⇒ I = \(2 \int_0^{\frac{\pi}{2}} \sin ^2\left(\frac{\pi}{2}-x\right) d x\) (because \(\int_0^a f(x) d x=\int_0^a f(a-x) d x\))

⇒ I = \(2 \int_0^{\frac{\pi}{2}} \cos ^2 x d x\)….(2)

Adding equation (1) and (2),

2I = \(2 \int_0^{\frac{\pi}{2}}\left(\sin ^2 x+\cos ^2 x\right) d x=2 \int_0^{\frac{\pi}{2}} 1 d x=2(x)_0^{\frac{\pi}{2}}\)

= \(2 \cdot \frac{\pi}{2}=\pi \Rightarrow 2 I=\pi \Rightarrow I=\frac{\pi}{2} \Rightarrow I=\frac{\pi}{2}\)

Question 12. \(\int_0^\pi \frac{x}{1+\sin x} d x\)
Solution:

Let \(I=\int_0^\pi \frac{x}{1+\sin x} d x\)….(1)

⇒ I = \(\int_0^\pi \frac{(\pi-x)}{1+\sin (\pi-x)} d x\) (because \(\int_0^\pi f(x) d x=\int_0^a f(a-x) d x\))

⇒ I = \(\int_0^\pi \frac{(\pi-x)}{1+\sin x} d x\)….(2)

Adding eq. (1) and (2), we obtain

2I =\(\int_0^\pi \frac{\pi}{1+\sin x} d x \Rightarrow 2 I=\pi \int_0^\pi \frac{(1-\sin x)}{(1+\sin x)(1-\sin x)} d x \Rightarrow 2 I=\pi \int_0^\pi \frac{1-\sin x}{\cos ^2 x} d x\)

⇒ 2I = \(\pi \int_0^\pi\left\{\sec ^2 x-\tan x \sec x\right\} d x\)

⇒ 2I = \(\pi[\tan x-\sec x]_0^\pi=\pi[\tan \pi-\tan 0]-\pi[\sec \pi-\sec 0]=\pi[0-0]-\pi[-1-1]\)

⇒ 2I = \(2 \pi \Rightarrow I=\pi\)

Question 13. \(\int_{-\frac{\pi}{2}}^{\frac{\pi}{2}} \sin ^7 x d x\)
Solution:

Let \(\mathrm{I}=\int_{-\frac{\pi}{2}}^{\frac{\pi}{2}} \sin ^7 \mathrm{xdx}\)

As \(\sin ^7(-x)=(\sin (-x))^7=(-\sin x)^7=-\sin ^7 x\), therefore, \(\sin ^7 x\) is an odd function.

It is known that, if f(x) is an odd function, then \(\int_{-\alpha}^a f(x) d x=0\)

∴ \(I=\int_{-\frac{\pi}{2}}^{\frac{\pi}{2}} \sin ^7 x d x=0\)

Question 14. \(\int_0^{2 \pi} \cos ^5 x d x\)
Solution:

Let \(I=\int_0^{2 \pi} \cos ^5 x d x\)

f(x) = \(\cos ^5 x \text {, then } f(2 \pi-x)=\cos ^5(2 \pi-x)=\cos ^5 x=f(x)\)

(because \(\int_0^{2 a} f(x) d x=2 \int_0^a f(x) d x \text {, if } f(2 a-x)=f(x)\))

∴ I = \(2 \int_0^\pi \cos ^5 x d x\)

I = \(2 \int_0^\pi \cos ^5(\pi-x) d x=-2 \int_0^\pi \cos ^3 x d x\) (because \(\int_0^a f(x) d x=\int_0^a f(a-x) d x\))

I = \(-2 \int_0^\pi \cos ^5 x d x=-I \Rightarrow 2 I=0 \Rightarrow I=0\) [From eq. (1)]

Question 15. \(\int_0^{\frac{\pi}{2}} \frac{\sin x-\cos x}{1+\sin x \cos x} d x\)
Solution:

Let \(I=\int_0^{\frac{\pi}{2}} \frac{\sin x-\cos x}{1+\sin x \cos x} d x\)….(1)

⇒ I = \(\int_0^{\frac{\pi}{2}} \frac{\sin \left(\frac{\pi}{2}-x\right)-\cos \left(\frac{\pi}{2}-x\right)}{1+\sin \left(\frac{\pi}{2}-x\right) \cos \left(\frac{\pi}{2}-x\right)} d x\)

⇒ I = \(\int_0^{\frac{\pi}{2}} \frac{\cos x-\sin x}{1+\sin x \cos x} d x\)…..(2)

Adding (1) and (2), we obtain, \(2 I=\int_0^{\frac{\pi}{2}} \frac{0}{1+\sin x \cos x} d x \Rightarrow I=0\)

Question 16. \(\int_0^\pi \log (1+\cos \mathrm{x}) \mathrm{dx}\)
Solution:

Let \(\mathrm{I}=\int_0^\pi \log (1+\cos \mathrm{x}) \mathrm{dx}\)….(1)

⇒ \(\mathrm{I}=\int_0^\pi \log (1+\cos (\pi-\mathrm{x})) \mathrm{dx}\) (because \(\int_0^a f(x) d x=\int_0^a f(a-x) d x\))

⇒ \(\mathrm{I}=\int_0^\pi \log (1-\cos \mathrm{x}) \mathrm{dx}\)…..(2)

So, \(1+\sin x \cos x\)

Adding (1) and (2), we obtain

2I = \(\int_0^\pi\{\log (1+\cos x)+\log (1-\cos x)\} d x \Rightarrow 2 I=\int_0^\pi \log \left(1-\cos ^2 x\right) d x\)

⇒ 2I = \(\int_0^\pi \log \sin ^2 x d x \Rightarrow 2 I=2 \int_0^\pi \log \sin x d x\)

⇒ I = \(\int_0^\pi \log \sin x d x\) (because \(sin (\pi-x)=\sin x\))…(3)

⇒ I = \(2 \int_0^{\frac{\pi}{2}} \log \sin x d x\)….(4)

∴ I = \(2 \int_0^{\frac{\pi}{2}} \log \sin \left(\frac{\pi}{2}-x\right) d x=2 \int_0^{\frac{\pi}{2}} \log \cos x d x\)…(5)

Adding (4) and (5), we obtain

2I = \(2 \int_0^{\frac{\pi}{2}}(\log \sin \mathrm{x}+\log \cos \mathrm{x}) \mathrm{dx} \Rightarrow \mathrm{I}=\int_0^{\frac{\pi}{2}}(\log \sin \mathrm{x}+\log \cos \mathrm{x}+\log 2-\log 2) \mathrm{dx}\)

⇒ \(\mathrm{I}=\int_0^{\frac{\pi}{2}}(\log 2 \sin \mathrm{x} \cos \mathrm{x}-\log 2) \mathrm{dx} \Rightarrow \mathrm{I}=\int_0^{\frac{\pi}{2}} \log \sin 2 \mathrm{xdx}-\int_0^{\frac{\pi}{2}} \log 2 \mathrm{dx}\)

Let \(2 \mathrm{x}=\mathrm{t} \Rightarrow 2 \mathrm{dx}=\mathrm{dt}\)

When \(\mathrm{x}=0, \mathrm{t}=0\) and when \(\mathrm{x}=\frac{\pi}{2}, \mathrm{t}=\pi\)

∴ I = \(\frac{1}{2} \int_0^\pi \log \sin t d t-\frac{\pi}{2} \log 2 \Rightarrow I=\frac{1}{2} \int_0^\pi \log \sin x d x-\frac{\pi}{2} \log 2\)

⇒ I = \(\frac{1}{2} I-\frac{\pi}{2} \log 2 \Rightarrow \frac{I}{2}=-\frac{\pi}{2} \log 2 \Rightarrow I=-\pi \log 2\)

(because \(int_a^b f(x) d x=\int_a^b f(t) d t\)) [From eq.(3)]

Question 17. \(\int_0^2 \frac{\sqrt{\mathrm{x}}}{\sqrt{\mathrm{x}}+\sqrt{\mathrm{a}-\mathrm{x}}} \mathrm{dx}\)
Solution:

Let \(I=\int_0^a \frac{\sqrt{x}}{\sqrt{x}+\sqrt{a-x}} d x\)…..(1)

I = \(\int_0^a \frac{\sqrt{a-x}}{\sqrt{a-x}+\sqrt{x}} d x\)….(2) (because \(\int_0^a f(x) d x=\int_0^a f(a-x) d x\))

Adding eq. (1) and (2), we obtain

2I = \(\int_0^a \frac{\sqrt{x}+\sqrt{a-x}}{\sqrt{x}+\sqrt{a-x}} d x \Rightarrow 2 I=\int_0^\pi 1 d x \Rightarrow 2 I=[x]_0^\pi \Rightarrow 2 I=a \Rightarrow I=\frac{a}{2}\)

Question 18. \(\int_0^4|x-1| d x\)
Solution:

Let \(\mathrm{I}=\int_0^4|\mathrm{x}-1| \mathrm{dx}\)

I = \(\int_0^1|x-1| d x+\int_1^4|x-1| d x\)

= \(\int_0^1-(x-1) d x+\int_1^4(x-1) d x\) (because \(\int_a^b f(x) d x=\int_a^0 f(x) d x+\int_c^b f(x) d x\))

I = \(-\left[\frac{x^2}{2}-x\right]_0^1+\left[\frac{x^2}{2}-x\right]_1^4\)

= \(-\left[\left(\frac{1}{2}-1\right)-0\right]+\left[\left(\frac{16}{2}-4\right)-\left(\frac{1}{2}-1\right)\right]=\frac{1}{2}+8-4+\frac{1}{2}=5\)

Question 19. Show that \(\int_0^a f(x) g(x) d x=2 \int_0^a f(x) d x\), if f and g are defined as f(x)=f(a-x) and \(\mathrm{g}(\mathrm{x})+\mathrm{g}(\mathrm{a}-\mathrm{x})=4\)
Solution:

Let I = \(\int_0^a f(x) g(x) d x\)….(1)

⇒ \(\mathrm{I}=\int_0^{\mathrm{a}} \mathrm{f}(\mathrm{a}-\mathrm{x}) \mathrm{g}(\mathrm{a}-\mathrm{x}) \mathrm{dx}\)

⇒ \(\mathrm{I}=\int_0^{\mathrm{a}} \mathrm{f}(\mathrm{x}) \mathrm{g}(\mathrm{a}-\mathrm{x}) \mathrm{dx}\)

⇒ \(\left(\int_0^a f(x) d x=\int_0^a f(a-x) d x\right)\)….(2)

Adding eq. (1) and (2), we obtain

2I = \(\int_0^a\{f(x) g(x)+f(x) g(a-x)\} d x\)

⇒ 2I = \(\int_0^a f(x)\{g(x)+g(a-x)\} d x \Rightarrow 2 I=\int_0^a f(x) \times 4 d x\) (Given g(x)+g(a-x)=4)

⇒ \(2 I=4 \int_0^a f(x) d x\)

⇒ \(I=2 \int_0^a f(x) d x\)

Hence proved

Choose The Correct Answer

Question 20. The value of \(\int_{\frac{\pi}{2}}^{\frac{\pi}{2}}\left(x^3+x \cos x+\tan ^5 x+1\right) d x\) is?

  1. 0
  2. 2
  3. \(\pi\)
  4. 1

Solution: 3. \(\pi\)

Let \(I=\int_{-\frac{\pi}{2}}^{\frac{\pi}{2}}\left(x^3+x \cos x+\tan ^5 x+1\right) d x\)

⇒ I = \(\int_{-\frac{\pi}{2}}^{\frac{\pi}{2}} x^2 d x+\int_{-\frac{\pi}{2}}^{\frac{\pi}{2}} x \cos x+\int_{-\frac{\pi}{2}}^{\frac{\pi}{2}} \tan ^5 x d x+\int_{-\frac{\pi}{2}}^{\frac{\pi}{2}} 1 . d x\)

It is known that if f(x) is an even function, then \(\int_{-a}^a f(x) d x=2 \int_0^a f(x) d x\) and if f(x) is an odd function, then \(\int_{-2}^a f(x) d x=0\)

⇒ \(\mathrm{I}=0+0+0+\int_{\frac{-\pi}{2}}^{\frac{\pi}{2}} 1 \cdot \mathrm{dx}\) (because \(x^3, x \cos x\), and \(\tan ^5 x\) are odd functions)

⇒ \(\mathrm{I}=[\mathrm{x}]_{\frac{-\pi}{2}}^{\frac{\pi}{2}}\)

⇒ \(\mathrm{I}=\left[\frac{\pi}{2}+\frac{\pi}{2}\right]=\frac{2 \pi}{2}=\pi\)

Hence, the correct answer is (3).

Question 21. The value of \(\int_0^{\frac{\pi}{2}} \log \left(\frac{4+3 \sin x}{4+3 \cos x}\right) d x\) is?

  1. 2
  2. \(\frac{3}{4}\)
  3. 0
  4. -2

Solution: 3. 0

Let \(I=\int_0^{\frac{\pi}{2}} \log \left(\frac{4+3 \sin x}{4+3 \cos x}\right) d x\)

⇒ I = \(\int_0^{\frac{\pi}{2}} \log \left[\frac{4+3 \sin \left(\frac{\pi}{2}-x\right)}{4+3 \cos \left(\frac{\pi}{2}-x\right)}\right] d x\) (because \(\int_0^a f(x) d x=\int_0^a f(a-x) d x\))

⇒ \(\mathrm{I}=\int_0^{\frac{\pi}{2}} \log \left(\frac{4+3 \cos \mathrm{x}}{4+3 \sin \mathrm{x}}\right) \mathrm{dx}\)

Adding (1) and (2), we obtain

2I = \(\int_0^{\frac{\pi}{2}}\left\{\log \left(\frac{4+3 \sin x}{4+3 \cos x}\right)+\log \left(\frac{4+3 \cos x}{4+3 \sin x}\right)\right\} d x\)

2I = \(\int_0^{\frac{\pi}{2}}\left\{\log \left(\frac{4+3 \sin x}{4+3 \cos x} \times \frac{4+3 \cos x}{4+3 \sin x}\right)\right\} d x\)

⇒ 2I = \(\int_0^{\frac{\pi}{2}} \log 1 d x\)

⇒ 2I = \(\int_0^{\frac{\pi}{2}} 0 d x \Rightarrow I=0\)

Hence, the correct answer is(3).

Miscellaneous Exercise Integrals

Integrated The Functions

Question 1. \(\int \frac{1}{x-x^3} d x\)
Solution:

Let \(I=\int \frac{1}{x-x^3} d x=\int \frac{1}{x\left(1-x^2\right)} d x=\int \frac{1}{x(1-x)(1+x)} d x\)

⇒ \(\frac{1}{x(1-x)(1+x)}=\frac{A}{x}+\frac{B}{(1-x)}+\frac{C}{(1+x)}\)

I = \(A\left(1-x^2\right)+B x(1+x)+C x(1-x)\)

I = \(A-A x^2+B x+B x^2+C x-C x^2\)…(1) (using partial fraction)

Equating the coefficients of \(\mathrm{x}^2, \mathrm{x}\), and constant term, we obtain

– \(\mathrm{A}+\mathrm{B}-\mathrm{C}=0, \mathrm{~B}+\mathrm{C}=0, \mathrm{~A}=1\)

On solving these equations, we obtain \(\mathrm{A}=1, \mathrm{~B}=\frac{1}{2}\), and \(\mathrm{C}=-\frac{1}{2}\)

From equation (1), we obtain

⇒ \(\frac{1}{x(1-x)(1+x)}=\frac{1}{x}+\frac{1}{2(1-x)}-\frac{1}{2(1+x)}\)

⇒ \(\int \frac{1}{x(1-x)(1+x)} d x=\int \frac{1}{x} d x+\frac{1}{2} \int \frac{1}{1-x} d x-\frac{1}{2} \int \frac{1}{1+x} d x\)

= \(\log |x|-\frac{1}{2} \log |(1-x)|-\frac{1}{2} \log |(1+x)|=\log |x|-\log \left|(1-x)^{\frac{1}{2}}\right|-\log \left|(1+x)^{\frac{1}{2}}\right|\)

= \(\log \left|\frac{x}{(1-x)^{\frac{1}{2}}(1+x)^{\frac{1}{2}}}\right|+C=\log \left|\left(\frac{x^2}{1-x^2}\right)^{\frac{1}{2}}\right|+C=\frac{1}{2} \log \left|\frac{x^2}{1-x^2}\right|+C\)

Question 2. \(\int \frac{1}{\sqrt{(x+a)}+\sqrt{(x+b)}} d x\)
Solution:

Let \(I=\int \frac{1}{\sqrt{(x+a)}+\sqrt{(x+b)}} d x\)

Now, \(\int \frac{1}{\sqrt{x+a}+\sqrt{x+b}} d x=\int \frac{1}{\sqrt{x+a}+\sqrt{x+b}} \times \frac{\sqrt{x+a}-\sqrt{x+b}}{\sqrt{x+a}-\sqrt{x+b}} d x\)

⇒ \(\int \frac{1}{\sqrt{x+a}+\sqrt{x+b}} d x=\frac{1}{a-b} \int(\sqrt{x+a}-\sqrt{x+b}) d x \)

⇒ I = \(\frac{1}{(a-b)}\left[\frac{(x+a)^{\frac{3}{2}}}{\frac{3}{2}}-\frac{(x+b)^{\frac{3}{2}}}{\frac{3}{2}}\right]+C=\frac{2}{3(a-b)}\left[(x+a)^{\frac{3}{2}}-(x+b)^{\frac{3}{2}}\right]+C\)

Question 3. \(\int \frac{1}{x \sqrt{a x-x^2}} d x\)
Solution:

Let \(I=\int \frac{1}{x \sqrt{a x-x^2}} d x\)

Put \(x=\frac{a}{t} \Rightarrow d x=-\frac{a}{t^2} d t\)

⇒ \(\int \frac{1}{x \sqrt{a x-x^2}} d x=\int \frac{1}{\frac{a}{t} \sqrt{a \cdot \frac{a}{t}-\left(\frac{a}{t}\right)^2}}\left(-\frac{a}{t^2} d t\right)=-\int \frac{1}{a t} \cdot \frac{1}{\sqrt{\frac{1}{t}-\frac{1}{t^2}}} d t=-\frac{1}{a} \int \frac{1}{\sqrt{\frac{t^2}{t}-\frac{t^2}{t^2}}} d t\)

= \(-\frac{1}{a} \int \frac{1}{\sqrt{t-1}} d t=-\frac{1}{a}[2 \sqrt{t-1}]+C=-\frac{1}{a}\left[2 \sqrt{\frac{a}{x}-1}\right]+C\)

= \(-\frac{2}{a}\left(\frac{\sqrt{a-x}}{\sqrt{x}}\right)+C=-\frac{2}{a}\left(\sqrt{\frac{a-x}{x}}\right)+C\)

Question 4. \(\int \frac{1}{x^2\left(x^4+1\right)^{\frac{3}{4}}} d x\)
Solution:

Let \(I=\int \frac{1}{x^2\left(x^4+1\right)^{\frac{3}{4}}} d x\)

I = \(\int \frac{d x}{x^2\left(x^4\right)^{3 / 4}\left(1+\frac{1}{x^4}\right)^{3 / 4}}=\int \frac{d x}{x^5\left(1+\frac{1}{x^4}\right)^{3 / 4}}\)

I = \(\int\left(1+x^{-4}\right)^{-3 / 4} x^{-5} d x\)

Put \(\left(1+x^{-4}\right)=t \Rightarrow-4 x^{-5} d x=d t\) or \(x^{-5} d x=\frac{-d t}{4}\)

= \(\int \mathrm{t}^{-3 / 4}\left(\frac{-\mathrm{dt}}{4}\right)=-\frac{1}{4} \int \mathrm{t}^{-3 / 4} \mathrm{dt}=-\frac{1}{4} \) \(\frac{\mathrm{t}^{-\frac{3}{4}+1}}{-\frac{3}{4}+1}+\mathrm{C}=\frac{1}{4}\left[\frac{\mathrm{t}^{1 / 4}}{\frac{1}{4}}\right]+\mathrm{C}=-\left(1+\frac{1}{\mathrm{x}^4}\right)^{\frac{1}{4}}+\mathrm{C}\)

Question 5. \(\int \frac{1}{x^{\frac{1}{2}}+x^{\frac{1}{3}}} d x\)
Solution:

Let \(\mathrm{I}=\int \frac{1}{\mathrm{x}^{\frac{1}{2}}+\mathrm{x}^{\frac{1}{3}}} \mathrm{dx}\)

Put \(x=t^6 \Rightarrow d x=6 t^5 d t\)

∴ \(\int \frac{1}{x^{\frac{1}{2}}+x^{\frac{1}{3}}} d x=\int \frac{1}{t^{6 / 2}+t^{6 / 3}} \cdot\left(6 t^5\right) d t=\int \frac{6 t^3}{t^2(1+t)} d t=6 \int \frac{t^3}{(1+t)} d t\)

On dividing, we obtain \(\int \frac{1}{x^{\frac{1}{2}}+x^{\frac{1}{3}}} d x=6 \int\left[\left(t^2-t+1\right)-\frac{1}{1+t}\right] d t\)

= \(=6\left[\left(\frac{t^3}{3}\right)-\left(\frac{t^2}{2}\right)+t-\log |1+t|\right]+C\)

= \(2 x^{\frac{1}{2}}-3 x^{\frac{1}{3}}+6 x^{\frac{1}{6}}-6 \log \left(1+x^{\frac{1}{6}}\right)+C=2 \sqrt{x}-3 x^{\frac{1}{3}}+6 x^{\frac{1}{6}}-6 \log \left(1+x^{\frac{1}{6}}\right)+C\)

Question 6. \(\int \frac{5 x}{(x+1)\left(x^2+9\right)} d x\)
Solution:

Let \(\mathrm{I}=\int \frac{5 \mathrm{x}}{(\mathrm{x}+1)\left(\mathrm{x}^2+9\right)} \mathrm{dx}\)

⇒ \(\frac{5 \mathrm{x}}{(\mathrm{x}+1)\left(\mathrm{x}^2+9\right)}=\frac{\mathrm{A}}{(\mathrm{x}+1)}+\frac{\mathrm{Bx}+\mathrm{C}}{\left(\mathrm{x}^2+9\right)} \quad \ldots(1)\) [using partial fraction]

⇒ \(5 \mathrm{x}=\mathrm{A}\left(\mathrm{x}^2+9\right)+(\mathrm{Bx}+\mathrm{C})(\mathrm{x}+1) \Rightarrow 5 \mathrm{x}=\mathrm{Ax}^2+9 \mathrm{~A}+\mathrm{Bx}^2+\mathrm{Bx}+\mathrm{Cx}+\mathrm{C}\)

Equating the coefficients of \(\mathrm{x}^2, \mathrm{x}\), and constant term, we obtain

⇒ \(\mathrm{A}+\mathrm{B}=0, \mathrm{~B}+\mathrm{C}=5,9 \mathrm{~A}+\mathrm{C}=0\)

On solving these equations, we obtain \(\mathrm{A}=-\frac{1}{2}, \mathrm{~B}=\frac{1}{2}\), and \(\mathrm{C}=\frac{9}{2}\)

From equation (1), we obtain

⇒ \(\frac{5 x}{(x+1)\left(x^2+9\right)}=\frac{-1}{2(x+1)}+\frac{\frac{x}{2}+\frac{9}{2}}{\left(x^2+9\right)}\)

⇒ \(\int \frac{5 x}{(x+1)\left(x^2+9\right)} d x=\int\left\{\frac{-1}{2(x+1)}+\frac{(x+9)}{2\left(x^2+9\right)}\right\} d x\)

= \(-\frac{1}{2} \log |x+1|+\frac{1}{2} \int \frac{x}{x^2+9} d x+\frac{9}{2} \int \frac{1}{x^2+9} d x\)

= \(-\frac{1}{2} \log |x+1|+\frac{1}{4} \int \frac{2 x}{x^2+9} d x+\frac{9}{2} \int \frac{1}{x^2+9} d x\)

= \(-\frac{1}{2} \log |x+1|+\frac{1}{4} \log \left|x^2+9\right|+\frac{9}{2} \cdot\left(\frac{1}{3} \tan ^{-1} \frac{x}{3}\right)+C\)

(because \(\int \frac{d x}{a^2+x^2}=\frac{1}{a} \tan ^{-1}\left(\frac{x}{a}\right)+C\))

= \(-\frac{1}{2} \log |x+1|+\frac{1}{4} \log \left(x^2+9\right)+\frac{3}{2} \tan ^{-1}\left(\frac{x}{3}\right)+C\)

Question 7. \(\int \frac{\sin x}{\sin (x-a)} d x\)
Solution:

Let \(\mathrm{I}=\int \frac{\sin \mathrm{x}}{\sin (\mathrm{x}-\mathrm{a})} \mathrm{dx}\)

Put x-a=t ⇒ d x=d t

⇒ \(\int \frac{\sin x}{\sin (x-a)} d x=\int \frac{\sin (t+a)}{\sin t} d t\)

= \(\int \frac{\sin t \cos a+\cos t \sin a}{\sin t} d t=\int(\cos a+\cot t \sin a) d t\)

= \(t \cos a+\sin a \log |\sin t|+C_1=(x-a) \cos a+\sin a \log |\sin (x-a)|+C_1\)

= \(x \cos a+\sin a \log |\sin (x-a)|-a \cos a+C_1\)

= \(\sin a \log |\sin (x-a)|+x \cos a+C\) (because-\(a \cos a+C_1=C_{1}\))

Question 8. \(\int \frac{e^{5 \log x}-e^{4 \log x}}{e^{3 \log x}-e^{2 \log x}} d x\)
Solution:

Let \(I=\int \frac{e^{\log x}-e^{4 \log x}}{e^{3 \log x}-e^{2 \log x}} d x=\int \frac{e^{\log x^3}-e^{\log x^4}}{e^{\log x^2}-e^{\log x^2}} d x=\int \frac{x^5-x^4}{x^3-x^2} d x=\int \frac{x^4(x-1)}{x^2(x-1)} d x=\int x^2 d x=\frac{x^3}{3}+C\)

Question 9. \(\int \frac{\cos x}{\sqrt{4-\sin ^2 x}} d x\)
Solution:

Let \(\mathrm{I}=\int \frac{\cos \mathrm{x}}{\sqrt{4-\sin ^2 \mathrm{x}}} \mathrm{dx}\)

Put sin x = \(t \Rightarrow \cos x d x=d t\)

⇒ I = \(\int \frac{\cos x}{\sqrt{4-\sin ^2 x}} d x=\int \frac{d t}{\sqrt{(2)^2-(t)^2}}=\sin ^{-1}\left(\frac{t}{2}\right)+C=\sin ^{-1}\left(\frac{\sin x}{2}\right)+C\)

Question 10. \(\int \frac{\sin ^8 x-\cos ^8 x}{1-2 \sin ^2 x \cos ^2 x} d x\)
Solution:

Let \(I=\int \frac{\sin ^8 x-\cos ^8 x}{1-2 \sin ^2 x \cos ^2 x} d x\)

= \(\int \frac{\left(\sin ^4 x+\cos ^4 x\right)\left(\sin ^4 x-\cos ^4 x\right)}{\sin ^2 x+\cos ^2 x-\sin ^2 x \cos ^2 x-\sin ^2 x \cos ^2 x} d x\)

= \(\int \frac{\left(\sin ^4 x+\cos ^4 x\right)\left(\sin ^2 x-\cos ^2 x\right)}{\sin ^2 x\left(1-\cos ^2 x\right)+\cos ^2 x\left(1-\sin ^2 x\right)} d x\)

= \(-\int \frac{\left(\sin ^4 x+\cos ^4 x\right)\left(\cos ^2 x-\sin ^2 x\right)}{\left(\sin ^4 x+\cos ^4 x\right)} d x\)

= \(-\int \cos 2 x d x=-\frac{\sin 2 x}{2}+C\)

Question 11. \(\int \frac{1}{\cos (x+a) \cos (x+b)} d x\)
Solution:

Let \(I=\int \frac{1}{\cos (x+a) \cos (x+b)} d x\)

Multiplying and dividing by sin (a-b), we obtain, \(\int \frac{1}{\cos (x+a) \cos (x+b)} d x=\frac{1}{\sin (a-b)} \int\left[\frac{\sin (a-b)}{\cos (x+a) \cos (x+b)}\right] d x\)

= \(\frac{1}{\sin (a-b)} \int\left[\frac{\sin [(x+a)-(x+b)]}{\cos (x+a) \cos (x+b)}\right] d x\)

= \(\frac{1}{\sin (a-b)} \int\left[\frac{\sin (x+a) \cos (x+b)-\cos (x+a) \cdot \sin (x+b)}{\cos (x+a) \cos (x+b)}\right] d x\)

= \(\frac{1}{\sin (a-b)} \int\left[\frac{\sin (x+a)}{\cos (x+a)}-\frac{\sin (x+b)}{\cos (x+b)}\right] d x=\frac{1}{\sin (a-b)} \int[\tan (x+a)-\tan (x+b)] d x\)

= \(\frac{1}{\sin (a-b)}[-\log |\cos (x+a)|+\log |\cos (x+b)|]+C=\frac{1}{\sin (a-b)} \log \left|\frac{\cos (x+b)}{\cos (x+a)}\right|+C\)

Question 12. \(\int \frac{\mathrm{x}^3}{\sqrt{1-\mathrm{x}^8}} \mathrm{dx}\)
Solution:

Let \(\mathrm{I}=\int \frac{\mathrm{x}^3}{\sqrt{1-\mathrm{x}^8}} \mathrm{dx}\)

Put \(x^4=t \Rightarrow 4 x^3 d x=d t \Rightarrow I=\int \frac{x^3}{\sqrt{1-x^8}} d x=\frac{1}{4} \int \frac{d t}{\sqrt{1-t^2}}=\frac{1}{4} \sin ^{-1} t+C=\frac{1}{4} \sin ^{-1}\left(x^4\right)+C\)

Question 13. \(\int \frac{e^x}{\left(1+e^x\right)\left(2+e^x\right)} d x\)
Solution:

Let \(I=\int \frac{e^x}{\left(1+e^x\right)\left(2+e^x\right)} d x\)

Put \(\mathrm{e}^{\mathrm{x}}=\mathrm{t} \Rightarrow \mathrm{e}^{\mathrm{x}} \mathrm{dx}=\mathrm{dt}\)

⇒ \(\int \frac{e^x}{\left(1+e^x\right)\left(2+e^x\right)} d x=\int \frac{d t}{(t+1)(t+2)}=\frac{1}{(t+1)(t+2)}=\frac{A}{t+1}+\frac{B}{t+2}=1=A(t+2)+B(t+2)\)

put t=-2, B=-1, put t=-1, A=1

I = \(\int\left[\frac{1}{(t+1)}-\frac{1}{(t+2)}\right] d t=\log |t+1|-\log |t+2|+C=\log \left|\frac{t+1}{t+2}\right|+C=\log \left|\frac{1+e^x}{2+e^x}\right|+C\)

Question 14. \(\int \frac{1}{\left(\mathrm{x}^2+1\right)\left(\mathrm{x}^2+4\right)} \mathrm{dx}\)
Solution:

Here, \(\frac{1}{\left(x^2+1\right)\left(x^2+4\right)}=\frac{1}{(y+1)(y+4)}\) (put \(x^2=y\))

⇒ \(\frac{1}{(y+1)(y+4)}=\frac{A}{y+1}+\frac{B}{y+4}\)

⇒ \(\frac{1}{(y+1)(y+4)}=\frac{A(y+4)+B(y+1)}{(y+1)(y+4)} \Rightarrow 1=A(y+4)+B(y+1)\) [By using partial fraction]

Put y=-1, then A = \(\frac{1}{3}\) and put y=-4, then \(B=-\frac{1}{3}\)

⇒ \(\frac{1}{(y+1)(y+4)}=\frac{1}{3(y+1)}-\frac{1}{3(y+4)}\)

⇒ \(\frac{1}{\left(x^2+1\right)\left(x^2+4\right)}=\frac{1}{3\left(x^2+1\right)}-\frac{1}{3\left(x^2+4\right)}\)

⇒ \(\int \frac{1}{\left(x^2+1\right)\left(x^2+4\right)} d x=\frac{1}{3} \int \frac{1}{x^2+1} d x-\frac{1}{3} \int \frac{1}{x^2+4} d x=\frac{1}{3} \tan ^{-1} x-\frac{1}{3} \cdot \frac{1}{2} \tan ^{-1} \frac{x}{2}+C\)

= \(\frac{1}{3} \tan ^{-1} x-\frac{1}{6} \tan ^{-1} \frac{x}{2}+C\)

Question 15. \(\int \cos ^3 x e^{\log \sin x} d x\)
Solution:

Let \(I=\int \cos ^3 x e^{\log \sin x} d x \Rightarrow \int \cos ^3 x e^{\log \sin x} d x=\int \cos ^3 x \sin x \hat{d x}\)

Put \(\cos x=t \Rightarrow-\sin x d x=d t \Rightarrow I=-\int t^3 \cdot d t=-\frac{t^4}{4}+C=-\frac{\cos ^4 x}{4}+C\)

Question 16. \(\int \mathrm{e}^{3 \log x}\left(\mathrm{x}^4+1\right)^{-1} \mathrm{dx}\)
Solution:

Let \(\mathrm{I}=\int \mathrm{e}^{3 \log x}\left(\mathrm{x}^4+1\right)^{-1} \mathrm{dx}\)

⇒ \(\int e^{3 \log x}\left(x^4+1\right)^{-1} d x=\int e^{\log x^3}\left(x^4+1\right)^{-1} d x=\int \frac{x^3}{\left(x^4+1\right)} d x\)

Put \(x^4+1=t \Rightarrow 4 x^3 d x=d t \Rightarrow I=\frac{1}{4} \int \frac{d t}{t}=\frac{1}{4} \log |t|+C=\frac{1}{4} \log \left|x^4+1\right|+C\)

Question 17. \(\int f^{\prime}(a x+b)[f(a x+b)]^n d x\)
Solution:

Let \(I=\int f^{\prime}(a x+b)[f(a x+b)]^n d x\)

Put \(f(a x+b)=t \Rightarrow a f^{\prime}(a x+b) d x=d t\)

⇒ \(\int f^{\prime}(a x+b)[f(a x+b)]^n d x=\frac{1}{a} \int t^n d t=\frac{1}{a}\left[\frac{t^{n+1}}{n+1}\right]+C=\frac{1}{a(n+1)}(f(a x+b))^{n+1}+C\)

Question 18. \(\int \frac{1}{\sqrt{\sin ^3 x \sin (x+\alpha)}} d x\)
Solution:

Let \(I=\int \frac{1}{\sqrt{\sin ^3 x \sin (x+\alpha)}} d x=\int \frac{1}{\sqrt{\sin ^3 x(\sin x \cos \alpha+\cos x \sin \alpha)}} d x\)

= \(\int \frac{1}{\sqrt{\sin ^4 x \cos \alpha+\sin ^3 x \cos x \sin \alpha}} d x=\int \frac{1}{\sin ^2 x \sqrt{\cos \alpha+\cot x \sin \alpha}} d x\)

= \(\int \frac{\mathrm{cosec}^2 x}{\sqrt{\cos \alpha+\cot x \sin \alpha}} d x\)

Put \(\cos \alpha+\cot x \cdot \sin \alpha=\mathrm{t} \Rightarrow-\mathrm{cosec}^2 \mathrm{x} \sin \alpha \mathrm{d} x=\mathrm{dt}\)

⇒ \(\mathrm{I}=\frac{-1}{\sin \alpha} \int \frac{\mathrm{dt}}{\sqrt{\mathrm{t}}}=\frac{-1}{\sin \alpha}[2 \sqrt{\mathrm{t}}]+\mathrm{C}=\frac{-1}{\sin \alpha}[2 \sqrt{\cos \alpha+\cot \mathrm{x} \sin \alpha}]+\mathrm{C}\)

= \(\frac{-2}{\sin \alpha} \sqrt{\cos \alpha+\frac{\cos \mathrm{x} \sin \alpha}{\sin x}}+\mathrm{C}=\frac{-2}{\sin \alpha} \sqrt{\frac{\sin \mathrm{x} \cos \alpha+\cos \mathrm{x} \sin \alpha}{\sin x}}+\mathrm{C}=\frac{-2}{\sin \alpha} \sqrt{\frac{\sin (\mathrm{x}+\alpha)}{\sin x}}+\mathrm{C}\)

Question 19. \(\int \sqrt{\frac{1-\sqrt{x}}{1+\sqrt{x}}} d x\)
Solution:

Let \(\mathrm{I}=\int \sqrt{\frac{1-\sqrt{\mathrm{x}}}{1+\sqrt{\mathrm{x}}}} d \mathrm{x}\)

put \(x=\cos ^2 \theta \Rightarrow d x=-2 \sin \theta \cos \theta d \theta\)

⇒ I = \(\int \sqrt{\frac{1-\cos \theta}{1+\cos \theta}}(-2 \sin \theta \cos \theta) d \theta=-\int \sqrt{\frac{2 \sin ^2 \frac{\theta}{2}}{2 \cos ^2 \frac{\theta}{2}}} \sin 2 \theta d \theta=-\int \tan \frac{\theta}{2} \cdot 2 \sin \theta \cos \theta d \theta\)

= \(-2 \int \frac{\sin \frac{\theta}{2}}{\cos \frac{\theta}{2}}\left(2 \sin \frac{\theta}{2} \cos \frac{\theta}{2}\right) \cos \theta d \theta=-4 \int \sin ^2 \frac{\theta}{2} \cos \theta d \theta=-4 \int \sin ^2 \frac{\theta}{2} \cdot\left(2 \cos ^2 \frac{\theta}{2}-1\right) d \theta\)

= \(-4 \int\left(2 \sin ^2 \frac{\theta}{2} \cos ^2 \frac{\theta}{2}-\sin ^2 \frac{\theta}{2}\right) d \theta=-8 \int \sin ^2 \frac{\theta}{2} \cdot \cos ^2 \frac{\theta}{2} d \theta+4 \int \sin ^2 \frac{\theta}{2} d \theta\)

= \(-2 \int \sin ^2 \theta d \theta+4 \int \sin ^2 \frac{\theta}{2} d \theta=-2 \int\left(\frac{1-\cos 2 \theta}{2}\right) d \theta+4 \int \frac{1-\cos \theta}{2} d \theta\)

= \(-2\left[\frac{\theta}{2}-\frac{\sin 2 \theta}{4}\right]+4\left[\frac{\theta}{2}-\frac{\sin \theta}{2}\right]+C=-\theta+\frac{\sin 2 \theta}{2}+2 \theta-2 \sin \theta+C\)

= \(\theta+\frac{\sin 2 \theta}{2}-2 \sin \theta+C=\theta+\frac{2 \sin \theta \cos \theta}{2}-2 \sin \theta+C\)

= \(\theta+\sqrt{1-\cos ^2 \theta} \cdot \cos \theta-2 \sqrt{1-\cos ^2 \theta}+C=\cos ^{-1} \sqrt{x}+\sqrt{1-x} \cdot \sqrt{x}-2 \sqrt{1-x}+C\)

= \(-2 \sqrt{1-x}+\cos ^{-1} \sqrt{x}+\sqrt{x(1-x)}+C=-2 \sqrt{1-x}+\cos ^{-1} \sqrt{x}+\sqrt{x-x^2}+C\)

Question 20. \(\int\left(\frac{2+\sin 2 x}{1+\cos 2 x}\right) e^x d x\)
Solution:

Let \(I=\int\left(\frac{2+\sin 2 x}{1+\cos 2 x}\right) e^x d x=\int\left(\frac{2+2 \sin x \cos x}{2 \cos ^2 x}\right) e^x d x=\int\left(\frac{1+\sin x \cos x}{\cos ^2 x}\right) e^x d x\)

= \(\int\left(\sec ^2 x+\tan x\right) e^x d x=\int\left(\tan x+\sec ^2 x\right) e^x d x\)

Let f(x) = \(\tan x \Rightarrow f^{\prime}(x)=\sec ^2 x\)

I = \(\int\left[f(x)+f^{\prime}(x)\right] e^x d x=e^x \cdot f(x)+C=e^x \tan x+C\)

Question 21. \(\int \frac{x^2+x+1}{(x+1)^2(x+2)} d x\)
Solution:

Let \(I=\int \frac{x^2+x+1}{(x+1)^2(x+2)} d x\)

⇒ \(\frac{\mathrm{x}^2+\mathrm{x}+1}{(\mathrm{x}+1)^2(\mathrm{x}+2)}=\frac{\mathrm{A}}{(\mathrm{x}+1)}+\frac{\mathrm{B}}{(\mathrm{x}+1)^2}+\frac{\mathrm{C}}{(\mathrm{x}+2)} \ldots(1)\)

⇒ \(\mathrm{x}^2+\mathrm{x}+1=\mathrm{A}(\mathrm{x}+1)(\mathrm{x}+2)+\mathrm{B}(\mathrm{x}+2)+\mathrm{C}\left(\mathrm{x}^2+2 \mathrm{x}+1\right)\)

⇒ \(\mathrm{x}^2+\mathrm{x}+1=\mathrm{A}\left(\mathrm{x}^2+3 \mathrm{x}+2\right)+\mathrm{B}(\mathrm{x}+2)+\mathrm{C}\left(\mathrm{x}^2+2 \mathrm{x}+1\right)\)

⇒ \(\mathrm{x}^2+\mathrm{x}+1=(\mathrm{A}+\mathrm{C}) \mathrm{x}^2+(3 \mathrm{~A}+\mathrm{B}+2 \mathrm{C}) \mathrm{x}+(2 \mathrm{~A}+2 \mathrm{~B}+\mathrm{C})\) (Using partial fraction)

Equating the coefficients of \(\mathrm{x}^2, \mathrm{x}\), and constant term, we obtain \(\mathrm{A}+\mathrm{C}=1,3 \mathrm{~A}+\mathrm{B}+2 \mathrm{C}=1,2 \mathrm{~A}+2 \mathrm{~B}+\mathrm{C}=1\)

On solving these equations, we obtain A=-2, B=1, and C=3

From equation (1), we obtain \(\frac{x^2+x+1}{(x+1)^2(x+2)}=\frac{-2}{(x+1)}+\frac{1}{(x+1)^2}+\frac{3}{(x+2)}\)

⇒ \(\int \frac{x^2+x+1}{(x+1)^2(x+2)} d x=-2 \int \frac{1}{x+1} d x+\int \frac{1}{(x+1)^2} d x+3 \int \frac{1}{(x+2)} d x\)

= \(-2 \log |x+1|-\frac{1}{(x+1)}+3 \log |x+2|+C\)

Question 22. \(\int \tan ^{-1} \sqrt{\frac{1-\mathrm{x}}{1+\mathrm{x}}} \mathrm{dx}\)
Solution:

Let \(I=\tan ^{-1} \sqrt{\frac{1-x}{1+x}} d x\)

Put \(x=\cos \theta \Rightarrow d x=-\sin \theta d \theta\)

I = \(\int \tan ^{-1} \sqrt{\frac{1-\cos \theta}{1+\cos \theta}}(-\sin \theta d \theta)=-\int \tan ^{-1} \sqrt{\frac{2 \sin ^2 \frac{\theta}{2}}{2 \cos ^2 \frac{\theta}{2}}} \sin \theta d \theta=-\int \tan ^{-1} \tan \frac{\theta}{2} \cdot \sin \theta d \theta\)

= \(-\frac{1}{2} \int \theta \cdot \sin \theta d \theta=-\frac{1}{2}\left[\theta \cdot(-\cos \theta)-\int 1 \cdot(-\cos \theta) d \theta\right]+C=-\frac{1}{2}[-\theta \cos \theta+\sin \theta]+C\)

= \(+\frac{1}{2} \theta \cos \theta-\frac{1}{2} \sin \theta+C=\frac{1}{2} \cos ^{-1} x \cdot x-\frac{1}{2} \sqrt{1-x^2}+C=\frac{x}{2} \cos ^{-1} x-\frac{1}{2} \sqrt{1-x^2}+C\)

= \(\frac{1}{2}\left(x \cos ^{-1} x-\sqrt{1-x^2}\right)+C\)

Question 23. \(\int \frac{\sqrt{x^2+1} \log \left(x^2+1\right)-2 \log x}{x^4} d x\)
Solution:

Let \(\mathrm{I}=\int \frac{\sqrt{\mathrm{x}^2+1} \log \left(\mathrm{x}^2+1\right)-2 \log \mathrm{x}}{\mathrm{x}^4} \mathrm{dx}\)

⇒ \(\int \frac{\sqrt{x^2+1} \log \left(x^2+1\right)-2 \log x}{x^4} d x=\int \frac{\sqrt{x^2+1}}{x^4}\left[\log \left(x^2+1\right)-\log x^2\right] d x\)

= \(\int \frac{\sqrt{x^2+1}}{x^4}\left[\log \left(\frac{x^2+1}{x^2}\right)\right]=\int \frac{1}{x^3} \sqrt{\frac{x^2+1}{x^2}} \log \left(1+\frac{1}{x^2}\right) d x=\int \frac{1}{x^3} \sqrt{1+\frac{1}{x^2}} \log \left(1+\frac{1}{x^2}\right) d x\)

Put \(1+\frac{1}{x^2}=t \Rightarrow \frac{-2}{x^3} d x=d t \Rightarrow I=-\frac{1}{2} \int \sqrt{t} \log t d t=-\frac{1}{2} \int t^{\frac{1}{2}}+\log t d t\)

= \(-\frac{1}{2}[\log \mathrm{t} \cdot \frac{\mathrm{t}^{\frac{3}{2}}}{\frac{3}{2}}-\int \frac{1}{\mathrm{t}} \cdot \frac{\mathrm{t}^{\frac{3}{2}}}{\frac{3}{2}} \mathrm{dt}\) (Integrating by parts, we obtain)

= \(-\frac{1}{2}\left[\frac{2}{3} \mathrm{t}^{\frac{3}{2}} \log \mathrm{t}-\frac{2}{3} \int \mathrm{t}^{\frac{1}{2}} \mathrm{dt}\right]=-\frac{1}{2}\left[\frac{2}{3} \mathrm{t}^{\frac{3}{2}} \log \mathrm{t}-\frac{4}{9} \mathrm{t}^{\frac{3}{2}}\right]+\mathrm{C}=-\frac{1}{3} \mathrm{t}^{\frac{3}{2}} \log \mathrm{t}+\frac{2}{9} \mathrm{t}^{\frac{3}{2}}+\mathrm{C}\)

= \(-\frac{1}{3} \mathrm{t}^{\frac{3}{2}}\left[\log \mathrm{t}-\frac{2}{3}\right]+\mathrm{C}=-\frac{1}{3}\left(1+\frac{1}{\mathrm{x}^2}\right)^{\frac{3}{2}}\left[\log \left(1+\frac{1}{\mathrm{x}^2}\right)-\frac{2}{3}\right]+\mathrm{C}\)

Question 24. \(\int_{\frac{\pi}{2}}^\pi e^x\left(\frac{1-\sin x}{1-\cos x}\right) d x\)
Solution:

Let \(I=\int_{\frac{\pi}{2}}^\pi e^x\left(\frac{1-\sin x}{1-\cos x}\right) d x \Rightarrow I=\int_{\frac{\pi}{2}}^\pi e^x\left(\frac{1-2 \sin \frac{x}{2} \cos \frac{x}{2}}{2 \sin ^2 \frac{x}{2}}\right) d x\)

I = \(\int_{\frac{\pi}{2}}^\pi e^x\left(\frac{\mathrm{cosec}^2 \frac{x}{2}}{2}-\cot \frac{x}{2}\right) d x\)

Let f(x) = \(-\cot \frac{x}{2} \Rightarrow f^{\prime}(x)=-\left(-\frac{1}{2} \mathrm{cosec}^2 \frac{x}{2}\right)=\frac{1}{2} \mathrm{cosec}^2 \frac{x}{2}\)

I = \(\int_{\frac{\pi}{2}}^\pi e^x\left[f(x)+f^{\prime}(x)\right] d x=\left[e^x \cdot f(x) d x\right]_{\frac{\pi}{2}}^x=-\left[e^x \cdot \cot \frac{x}{2}\right]_{\frac{\pi}{2}}^\pi \)

= \(-\left[e^\pi \times \cot \frac{\pi}{2}-e^{\frac{\pi}{2}} \times \cot \frac{\pi}{4}\right]=-\left[e^x \times 0-e^{\frac{\pi}{2}} \times 1\right]=e^{\frac{\pi}{2}}\)

Question 25. \(\int_0^{\frac{\pi}{4}} \frac{\sin x \cos x}{\cos ^4 x+\sin ^4 x} d x\)
Solution:

Let \(I=\int_0^{\frac{\pi}{4}} \frac{\sin x \cos x}{\cos ^4 x+\sin ^4 x} d x \Rightarrow I=\int_0^{\frac{\pi}{4}} \frac{\frac{\sin x \cos x}{\cos ^4 x}}{\frac{\left(\cos ^4 x+\sin ^4 x\right)}{\cos ^4 x}} d x\) (Nr. and Dr. divided by \(\cos ^4 x\))

⇒ \(\mathrm{I}=\int_0^{\frac{\pi}{4}} \frac{\tan x \sec ^2 \mathrm{x}}{1+\tan ^4 \mathrm{x}} \mathrm{dx}\)

Put \(\tan ^2 x=t \Rightarrow 2 \tan x \sec ^2 x d x=d t\)

When x=0, t=0 and when \(x=\frac{\pi}{4}, t=1\)

⇒ \(\mathrm{I}=\frac{1}{2} \int_0^1 \frac{\mathrm{dt}}{1+\mathrm{t}^2}=\frac{1}{2}\left[\tan ^{-1} \mathrm{t}\right]_0^1=\frac{1}{2}\left[\tan ^{-1} 1-\tan ^{-1} 0\right]=\frac{1}{2}\left[\frac{\pi}{4}\right]=\frac{\pi}{8}\)

Question 26. \(\int_0^{\frac{\pi}{2}} \frac{\cos ^2 x}{\cos ^2 x+4 \sin ^2 x} d x\)
Solution:

Let \(I=\int_0^{\frac{\pi}{2}} \frac{\cos ^2 x}{\cos ^2 x+4 \sin ^2 x} d x\) (Nr. and Dr. divided by \(\cos ^2 x\))

I = \(\int_0^{\frac{\pi}{2}} \frac{d x}{1+4 \tan ^2 x} \Rightarrow I=\int_0^{\frac{\pi}{2}} \frac{\sec ^2 x d x}{\sec ^2 x\left(1+4 \tan ^2 x\right)}\)(multiplying and divide by \(\sec ^2 x\))

I = \(\int_0^{\frac{\pi}{2}} \frac{\sec ^2 x d x}{\left(1+\tan ^2 x\right)\left(1+4 \tan ^2 x\right)}\)

Put \(\tan x=t \Rightarrow \sec ^2 x d x=d t\)

when x=0, then t=0, when \(x=\frac{\pi}{2}\), then t=\(\infty\)

I \(=\int_0^{\infty} \frac{d t}{\left(1+t^2\right)\left(1+4 t^2\right)}\) (put } \(t^2=y\)

= \(\int_0^{\infty} \frac{d t}{(1+y)(1+4 y)} \Rightarrow \frac{1}{(1+y)(1+4 y)}=\frac{A}{1+y}+\frac{B}{1+4 y}\)

= \(\frac{1}{(1+y)(1+4 y)}=\frac{A(1+4 y)+B(1+y)}{(1+y)(1+4 y)}\)

I = \(A(1+4 y)+B(1+y)\) (By using partial fraction)

Put y=-1, then A = \(-\frac{1}{3}\)

Put \(\mathrm{y}=-\frac{1}{4}\), then \(\mathrm{B}=\frac{4}{3}\)

⇒ \(\mathrm{I}=-\frac{1}{3} \int_0^{\infty} \frac{\mathrm{dt}}{1+\mathrm{t}^2}+\frac{4}{3} \int_0^{\infty} \frac{\mathrm{dt}}{1+4 \mathrm{t}^2}=-\frac{1}{3}\left[\tan ^{-1} \mathrm{t}\right]_0^{\infty}+\frac{4}{3} \int_0^{\infty} \frac{\mathrm{dt}}{1+(2 \mathrm{t})^2}\)

= \(-\frac{1}{3}\left[\tan ^{-1} \infty-\tan ^{-1} 0\right]+\frac{4}{3} \times \frac{1}{2}\left[\tan ^{-1} 2 \mathrm{t}\right]_0^{\infty}=-\frac{1}{3}\left[\frac{\pi}{2}-0\right]+\frac{2}{3}\left[\tan ^{-1} \infty-\tan ^{-1} 0\right]\)

= \(-\frac{\pi}{6}+\frac{2}{3} \cdot \frac{\pi}{2}=-\frac{\pi}{6}+\frac{\pi}{3}=\frac{\pi}{6}\)

Question 27. \(\int_{\frac{\pi}{6}}^{\frac{\pi}{3}} \frac{\sin x+\cos x}{\sqrt{\sin 2 x}} d x\)
Solution:

Let \(I=\int_{\frac{\pi}{6}}^{\frac{\pi}{3}} \frac{\sin x+\cos x}{\sqrt{\sin 2 x}} d x=\int_{\frac{\pi}{6}}^{\frac{\pi}{3}} \frac{\sin x+\cos x}{\sqrt{1+(\sin 2 x-1)}} d x=\int_{\frac{\pi}{6}}^{\frac{\pi}{3}} \frac{\sin x+\cos x}{\sqrt{1-(1-\sin 2 x)}} d x\)

I = \(\int_{\frac{\pi}{6}}^{\frac{\pi}{3}} \frac{(\sin x+\cos x) d x}{\sqrt{1-(\sin x-\cos x)^2}}\)

Put \((\sin \mathrm{x}-\cos \mathrm{x})=\mathrm{t} \Rightarrow(\sin \mathrm{x}+\cos \mathrm{x}) \mathrm{dx}=\mathrm{dt}\)

When \(x=\frac{\pi}{6}, t=\left(\frac{1-\sqrt{3}}{2}\right)\) and when \(x=\frac{\pi}{3}, t=\left(\frac{\sqrt{3}-1}{2}\right)\)

I = \(\int_{\frac{1-\sqrt{3}}{2}}^{\frac{\sqrt{3}-1}{2}} \frac{d t}{\sqrt{1-t^2}} \Rightarrow I=\int_{-\left(\frac{\sqrt{3}-1}{2}\right)}^{\frac{\sqrt{3}-1}{2}} \frac{d t}{\sqrt{1-t^2}}\)

As \(\frac{1}{\sqrt{1-(-t)^2}}=\frac{1}{\sqrt{1-t^2}}\), therefore, \(\frac{1}{\sqrt{1-t^2}}\) is an even function.

It is known that if f(x) is an even function, then \(\int_{-a}^a f(x) d x=2 \int_0^a f(x) d x\)

⇒ \(\mathrm{I}=2 \int_0^{\frac{\sqrt{3}-1}{2}} \frac{\mathrm{dt}}{\sqrt{1-\mathrm{t}^2}}=\left[2 \sin ^{-1} \mathrm{t}\right]_0^{\frac{\sqrt{3}-1}{2}}=2 \sin ^{-1}\left(\frac{\sqrt{3}-1}{2}\right)\)

Question 28. \(\int_0^1 \frac{d x}{\sqrt{1+x}-\sqrt{x}}\)
Solution:

Let \(I=\int_0^1 \frac{d x}{\sqrt{1+x}-\sqrt{x}}\)

I = \(\int_0^1 \frac{d x}{\sqrt{1+x}-\sqrt{x}} \times \frac{(\sqrt{1+x}+\sqrt{x})}{(\sqrt{1+x}+\sqrt{x})} d x=\int_0^1 \frac{\sqrt{1+x}+\sqrt{x}}{1+x-x} d x=\int_0^1 \sqrt{1+x} d x+\int_0^1 \sqrt{x} d x\)

= \(\left[\frac{2}{3}(1+x)^{\frac{3}{2}}\right]_0^1+\left[\frac{2}{3}(x)^{\frac{3}{2}}\right]_0^1=\frac{2}{3}\left[(2)^{\frac{3}{2}}-1\right]+\frac{2}{3}[1]=\frac{2}{3}(2)^{\frac{3}{2}}=\frac{2 \cdot 2 \sqrt{2}}{3}=\frac{4 \sqrt{2}}{3}\)

Question 29. \(\int_0^{\frac{\pi}{4}} \frac{\sin x+\cos x}{9+16 \sin 2 x} d x\)
Solution:

Let \(I=\int_0^{\frac{\pi}{4}} \frac{\sin x+\cos x}{9+16 \sin 2 x} d x\)

Put \(\sin x-\cos x=t \Rightarrow(\cos x+\sin x) d x=d t\)

When \(\mathrm{x}=0, \mathrm{t}=-1\) and when \(\mathrm{x}=\frac{\pi}{4}, \mathrm{t}=0\)

⇒ \((\sin x-\cos x)^2=t^2 \Rightarrow \sin ^2 x+\cos ^2 x-2 \sin x \cos x=t^2\)

⇒ \(1-\sin 2 x=t^2 \Rightarrow \sin 2 x=1-t^2\)

∴ \(I=\int_{-1}^0 \frac{d t}{9+16\left(1-t^2\right)}=\int_{-1}^0 \frac{d t}{9+16-16 t^2}=\int_{-1}^0 \frac{d t}{25-16 t^2}=\int_{-1}^0 \frac{d t}{(5)^2-(4 t)^2}\)

= \(\frac{1}{4}\left[\frac{1}{2(5)} \log \left|\frac{5+4 t}{5-4 t}\right|\right]_{-1}^0=\frac{1}{40}\left[\log (1)-\log \left|\frac{1}{9}\right|\right]=\frac{1}{40} \log 9\)

Question 30. \(\int_0^{\frac{\pi}{2}} \sin 2 x \tan ^{-1}(\sin x) d x\)
Solution:

Let \(I=\int_0^{\frac{\pi}{2}} \sin 2 x \tan ^{-1}(\sin x) d x=\int_0^{\frac{\pi}{2}} 2 \sin x \cos x \tan ^{-1}(\sin x) d x\)

Put \(\sin \mathrm{x}=\mathrm{t} \Rightarrow \cos \mathrm{x} d \mathrm{x}=\mathrm{dt}\)

When x=0, t=0 and when \(x=\frac{\pi}{2}, t=1\)

⇒ \(\mathrm{I}=2 \int_0^{\mathrm{t}} \mathrm{t} \tan ^{-1}(\mathrm{t}) \mathrm{dt}\)

Consider \(\int \mathrm{t} \cdot \tan ^{-1} \mathrm{t} d \mathrm{t}=\tan ^{-1} \mathrm{t} \cdot \int \mathrm{t} \mathrm{dt}-\int\left\{\frac{\mathrm{d}}{\mathrm{dt}}\left(\tan ^{-1} \mathrm{t}\right) \int \mathrm{t} d \mathrm{t}\right\} \mathrm{dt}\)

= \(\tan ^{-1} \mathrm{t} \cdot \frac{\mathrm{t}^2}{2}-\int \frac{1}{1+\mathrm{t}^2} \cdot \frac{\mathrm{t}^2}{2} \mathrm{dt}=\frac{\mathrm{t}^2 \tan ^{-1} \mathrm{t}}{2}-\frac{1}{2} \int \frac{\mathrm{t}^2+1-1}{1+\mathrm{t}^2} \mathrm{dt}=\frac{\mathrm{t}^2 \tan ^{-1} \mathrm{t}}{2}-\frac{1}{2} \int 1 \mathrm{dt}+\frac{1}{2} \int \frac{1}{1+\mathrm{t}^2} \mathrm{dt}\)

= \(\frac{t^2 \tan ^{-1} t}{2}-\frac{1}{2} \cdot t+\frac{1}{2} \tan ^{-1} t\)

⇒ \(\int_0^1 \mathrm{t} \cdot \tan ^{-1} \mathrm{tdt}=\left[\frac{\mathrm{t}^2 \cdot \tan ^{-1} \mathrm{t}}{2}-\frac{\mathrm{t}}{2}+\frac{1}{2} \tan ^{-1} \mathrm{t}\right]_0^1=\frac{1}{2}\left[\frac{\pi}{4}-1+\frac{\pi}{4}\right]\)

= \(\frac{1}{2}\left[\frac{\pi}{2}-1\right]=\frac{\pi}{4}-\frac{1}{2}\)

From equation (1) and (2). We obtain: \(\mathrm{I}=2\left[\frac{\pi}{4}-\frac{1}{2}\right]=\frac{\pi}{2}-1\)

Question 31. \(\int_1^4[|x-1|+|x-2|+|x-3|] d x\)
Solution:

Let \(\mathrm{I}=\int_1^4[|\mathrm{x}-1|+|\mathrm{x}-2|+|\mathrm{x}-3|] \mathrm{dx}\)

= \(\int_1^2[(x-1)-(x-2)-(x-3)] d x+\int_2^3[(x-1)+(x-2)-(x-3)] d x\)

+ \(\int_3^4[(x-1)+(x-2)+(x-3)] d x\)

= \(\int_1^2(-x+4) d x+\int_2^3(x) d x+\int_3^4(3 x-6) d x\)

= \(\left[-\frac{x^2}{2}+4 x\right]_1^2+\left[\frac{x^2}{2}\right]_2^3+\left[\frac{3 x^2}{2}-6 x\right]_3^4\)

= \(\left[\left(-\frac{4}{2}+8\right)-\left(-\frac{1}{2}+4\right)\right]+\frac{1}{2}\left[3^2-2^2\right]+\left[\left(\frac{3 \times 16}{2}-6 \times 4\right)-\left(3 \times \frac{9}{2}-6 \times 3\right)\right]\)

= \(\left[6-\frac{7}{2}\right]+\frac{1}{2} \times 5+\left[(24-24)-\left(\frac{27}{2}-18\right)\right]=\frac{5}{2}+\frac{5}{2}+\frac{9}{2}=\frac{19}{2}\)

Question 32. Prove that: \(\int_1^3 \frac{\mathrm{dx}}{\mathrm{x}^2(\mathrm{x}+1)}=\frac{2}{3}+\log \frac{2}{3}\)
Solution:

Let \(I=\int_1^3 \frac{d x}{x^2(x+1)}\)

Let \(\frac{1}{x^2(x+1)}=\frac{A}{x}+\frac{B}{x^2}+\frac{C}{x+1}\)…..(1) (using partial fraction)

I = \(\mathrm{Ax}(\mathrm{x}+1)+\mathrm{B}(\mathrm{x}+1)+\mathrm{C}\left(\mathrm{x}^2\right) \Rightarrow 1=\mathrm{Ax}^2+\mathrm{Ax}+\mathrm{Bx}+\mathrm{B}+\mathrm{Cx}^2\)

Equating the coefficients of \(x^2, x\), and constant term, we obtain \(\mathrm{A}+\mathrm{C}=0, \mathrm{~A}+\mathrm{B}=0, \mathrm{~B}=1\)

On solving these equations, we obtain \(\mathrm{A}=-1, \mathrm{C}=1\), and \(\mathrm{B}=1\)

∴ \(\frac{1}{x^2(x+1)}=\frac{-1}{x}+\frac{1}{x^2}+\frac{1}{(x+1)}\) (from (1))

⇒ \(I=\int_1^3\left\{-\frac{1}{x}+\frac{1}{x^2}+\frac{1}{(x+1)}\right\} d x=\left[-\log x-\frac{1}{x}+\log (x+1)\right]_1^3=\left[\log \left(\frac{x+1}{x}\right)-\frac{1}{x}\right]_1^3\)

= \(\log \left(\frac{4}{3}\right)-\frac{1}{3}-\log \left(\frac{2}{1}\right)+1=\log 4-\log 3-\log 2+\frac{2}{3}=\log 2-\log 3+\frac{2}{3}=\log \left(\frac{2}{3}\right)+\frac{2}{3}\)

Hence, the given result is proved.

Question 33. Prove that: \(\int_0^1 x e^x d x=1\)
Solution:

Let \(\mathrm{I}=\int_0^1 \mathrm{xe}^{\mathrm{x}} \mathrm{dx}\)

Integrating by parts, we obtain

I = \(\left[x \int e^x d x\right]_0^1-\int_0^1\left\{\left(\frac{d}{d x}(x)\right) \int e^x d x\right\} d x=\left[x e^x\right]_0^1-\int_0^1 e^x d x=\left[x e^x\right]_0^1-\left[e^x\right]_0^1\)

= \(\mathrm{e}-\mathrm{e}+1=1\)

Hence, the given result is proved.

Question 34. Prove that: \(\int_{-1}^1 x^{17} \cos ^4 x d x=0\)
Solution:

Let \(\mathrm{I}=\int_{-1}^1 \mathrm{x}^{17} \cos ^4 \mathrm{x} d \mathrm{x}\)

Also, let \(f(x)=x^{17} \cos ^4 x \Rightarrow f(-x)=(-x)^{17} \cos ^4(-x)=-x^{17} \cos ^4 x=-f(x)\)

Therefore, f(x) is an odd function.

It is known that if f(x) is an odd function, then \(\int_{-a}^a f(x) d x=0\)

∴ \(\mathrm{I}=\int_{-1}^1 \mathrm{x}^{17} \cos ^4 \mathrm{x} \mathrm{dx}=0\)

Hence, the given result is proved.

Question 35. Prove that: \(\int_0^{\frac{\pi}{2}} \sin ^3 x d x=\frac{2}{3}\)
Solution:

Let \(\mathrm{I}=\int_0^{\frac{\pi}{2}} \sin ^3 \mathrm{xdx}\)

⇒ \(\mathrm{I}=\int_0^{\frac{\pi}{2}} \sin ^2 x \cdot \sin x d x=\int_0^{\frac{\pi}{2}}\left(1-\cos ^2 x\right) \sin x d x=\int_0^{\frac{\pi}{2}} \sin x d x-\int_0^{\frac{\pi}{2}} \cos ^2 x \cdot \sin x d x\)

= \([-\cos x]_0^{\frac{\pi}{2}}+\left[\frac{\cos ^3 x}{3}\right]_0^{\frac{\pi}{2}}=1+\frac{1}{3}[-1]=1-\frac{1}{3}=\frac{2}{3}\)

Hence, the given result is proved.

Question 36. Prove that: \(\int_0^{\frac{\pi}{4}} 2 \tan ^3 x d x=1-\log 2\)
Solution:

Let \(I=\int_0^{\frac{\pi}{4}} 2 \tan ^3 x d x\)

I = \(2 \int_0^{\frac{\pi}{4}} \tan ^2 x \tan x d x=2 \int_0^{\frac{\pi}{4}}\left(\sec ^2 x-1\right) \tan x d x=2 \int_0^{\frac{\pi}{4}} \sec ^2 x \tan x d x-2 \int_0^{\frac{\pi}{4}} \tan x d x\)

= \(2\left[\frac{\tan ^2 x}{2}\right]_0^{\frac{\pi}{4}}+2[\log \cos x]_0^{\frac{\pi}{4}} \quad\left[\tan ^2 \frac{\pi}{4}-\tan ^2 0\right]+2\left[\log \cos \frac{\pi}{4}-\log \cos 0\right]\)

= \(1+2\left[\log \frac{1}{\sqrt{2}}-\log 1\right]=1-2 \log \sqrt{2}-2 \log 1=1-\frac{2}{2} \log 2-0\)

= \(1-\log 2=1-\log 2\)

Hence, the given result is proved.

Question 37. Prove that: \(\int_0^1 \sin ^{-1} x d x=\frac{\pi}{2}-1\)
Solution:

Let \(\mathrm{I}=\int_0^1 \sin ^{-1} \mathrm{x} d x \Rightarrow \int_0^1 \sin ^{-1} \mathrm{x} \cdot 1 \mathrm{dx}\)

Integrating by parts, we obtain \(1=\left[\sin ^{-1} x \cdot x\right]_0^1-\int_0^1 \frac{1}{\sqrt{1-x^2}} \cdot x d x=\left[x \sin ^{-1} x\right]_0^1+\frac{1}{2} \int_0^1 \frac{(-2 x)}{\sqrt{1-x^2}} d x\)

Put \(1-x^2=t \Rightarrow-2 x d x=d t\); When x=0, t=1 and when x=1, t=0

I = \(\left[x \sin ^{-1} x\right]_0^1+\frac{1}{2} \int_1^0 \frac{d t}{\sqrt{t}}=\left[x \sin ^{-1} x\right]_0^1+\frac{1}{2}[2 \sqrt{t}]_1^0=\sin ^{-1}(1)+[-\sqrt{1}]=\frac{\pi}{2}-1\)

Hence, the given result is proved.

Choose The Correct Answers

Question 38. \(\int \frac{\mathrm{dx}}{\mathrm{e}^{\mathrm{x}}+\mathrm{e}^{-\mathrm{x}}}\) is equal to ?

  1. \(\tan ^{-1}\left(e^x\right)+C\)
  2. \(\tan ^{-1}\left(e^{-x}\right)+C\)
  3. \(\log \left(e^x-e^{-x}\right)+C\)
  4. \(\log \left(e^x+e^{-x}\right)+C\)

Solution: 1. \(\tan ^{-1}\left(e^x\right)+C\)

Let \(I=\int \frac{d x}{e^x+e^{-x}} d x=\int \frac{e^x}{e^{2 x}+1} d x\)

Put \(\mathrm{e}^{\mathrm{x}}=\mathrm{t} \Rightarrow \mathrm{e}^{\mathrm{x}} d \mathrm{x}=\mathrm{dt}\)

I = \(\int \frac{d t}{1+t^2}=\tan ^{-1} t+C=\tan ^{-1}\left(e^x\right)+C\)

Hence, the correct Answer is 1.

Question 39. \(\int \frac{\cos 2 x}{(\sin x+\cos x)^2} d x\) is equal to ?

  1. \(\frac{-1}{\sin x+\cos x}+C\)
  2. \(\log |\sin x+\cos x|+C\)
  3. \(\log |\sin x-\cos x|+C\)
  4. \(\frac{1}{(\sin x+\cos x)^2}\)

Solution:

Let I = \(\frac{\cos 2 x}{(\cos x+\sin x)^2}\)

I = \(\int \frac{\cos ^2 x-\sin ^2 x}{(\cos x+\sin x)^2} d x=\int \frac{(\cos x+\sin x)(\cos x-\sin x)}{(\cos x+\sin x)^2} d x=\int \frac{\cos x-\sin x}{\cos x+\sin x} d x\)

Put \(\cos \mathrm{x}+\sin \mathrm{x}=\mathrm{t} \Rightarrow(\cos \mathrm{x}-\sin \mathrm{x}) \mathrm{dx}=\mathrm{dt}\)

⇒ \(\mathrm{I}=\int \frac{\mathrm{dt}}{\mathrm{t}}=\log |\mathrm{t}|+\mathrm{C}=\log |\cos \mathrm{x}+\sin \mathrm{x}|+\mathrm{C}\)

Hence, the correct answer is (2).

Question 40. If f(a+b-x)=f(x), then \(\int_a^b x f(x) d x\) is equal to ?

  1. \(\frac{a+b}{2} \int_a^b f(b-x) d x\)
  2. \(\frac{a+b}{2} \int_a^b f(b+x) d x\)
  3. \(\frac{b-a}{2} \int_a^b f(x) d x\)
  4. \(\frac{a+b}{2} \int_a^b f(x) d x\)

Solution: Let \(I=\int_a^b x f(x) d x\)

I = \(\int_a^b(a+b-x) f(a+b-x) d x \quad\left(\int_a^b f(x) d x=\int_a^b f(a+b-x) d x\right)\)

⇒ \(I=\int_a^b(a+b-x) f(x) d x \Rightarrow I=(a+b) \int_a^b f(x) d x-I\)

⇒ \(I+I=(a+b) \int_a^b f(x) d x \Rightarrow 2 I=(a+b) \int_a^b f(x) d x\)

⇒ \(\mathrm{I}=\left(\frac{\mathrm{a}+\mathrm{b}}{2}\right) \int_{\mathrm{a}}^{\mathrm{b}} \mathrm{f}(\mathrm{x}) \mathrm{dx}\)

Hence, the correct answer is (4).

 

 

 

Application of Integrals Class 12 Maths Important Questions Chapter 8

Applications Of Integrals Exercise 8.1

Question 1. Find the area of the region bounded by the ellipse \(\frac{x^2}{16}+\frac{y^2}{9}=1\)
Solution:

The given curve is an ellipse with center at (0, 0) and symmetrical about both the X-axis and Y-axis (the power of x and y both are even)

Area bounded by the ellipse = 4 x (Area of the shaded region in the first, quadrant only)

(∵ By symmetry)

= \(4 \times \int_a^b y d x=4 \int_0^4 y d x=4 \int_0^4 \frac{3}{4} \sqrt{16-x^2} d x\)

(because \(\frac{x^2}{16}+\frac{y^2}{9}=1\) therefore \(y=\frac{3}{4} \sqrt{16-x^2}\))

= \(3 \int_0^4 \sqrt{4^2-x^2} d x=3\left[\frac{x}{2} \sqrt{4^2-x^2}+\frac{4^2}{2} \sin ^{-1}\left(\frac{x}{4}\right)\right]_0^4\)

Integrals Area Of The Region Bounded By Ellipse

(because \(\int \sqrt{a^2-x^2} d x=\frac{x}{2} \sqrt{a^2-x^2}+\frac{a^2}{2} \sin ^{-1}\left(\frac{x}{a}\right)+C\))

= \(3\left[2 \sqrt{16-16}+8 \sin ^{-1}(1)-0-8 \sin ^{-1}(0)\right]\)

= \(3\left[0+8 \sin ^{-1}(1)-0\right]=3 \times 8 \times\left(\frac{\pi}{2}\right)=12 \pi \) (sq. units.)

Therefore, the area bounded by the ellipse is \(12 \pi\) sq. units.

Question 2. Find the area of the region bounded by the ellipse \(\frac{x^2}{4}+\frac{y^2}{9}=1\).
Solution:

The given curve is an ellipse with center at (0, 0) and symmetrical about both the X-axis and Y-axis. The area bounded by the ellipse

= 4x (Area of the shaded region in the first quadrant only) (7 By symmetry)

= \(4 \times \int_a^b y d x\)

= \(4 \int_0^2 y d x=4 \int_0^2 \frac{3}{2} \sqrt{4-x^2} d x\)

Integrals Area Of Shaded Region In The First Quadrant

(because \(\frac{x^2}{4}+\frac{y^2}{9}=1\), therefore \(y=\frac{3}{2} \sqrt{4-x^2}\))

= \(6 \int_0^2 \sqrt{2^2-x^2} d x=6\left[\frac{x}{2} \sqrt{4-x^2}+\frac{2^2}{2} \sin ^{-1}\left(\frac{x}{2}\right)\right]_0^2\)

(because \(\int \sqrt{a^2-x^2} d x=\frac{x}{2} \sqrt{a^2-x^2}+\frac{a^2}{2} \sin ^{-1}\left(\frac{x}{a}\right)+C\))

= \(6\left\{0+2 \sin ^{-1}(1)-0\right\}=6 \times 2 \times\left(\frac{\pi}{2}\right)=6 \pi\) (sq. units)

Read and Learn More Class 12 Maths Chapter Wise with Solutions

Choose The Correct Answer

Question 3. The area lying in the first quadrant and bounded by the circle x²+y² =4 and the lines x = 0 and x = 2 is.

  1. \(\pi\)
  2. \(\frac{\pi}{2}\)
  3. \(\frac{\pi}{3}\)
  4. \(\frac{\pi}{4}\)

Solution: 1. \(\pi\)

The area bounded by the circle and the lines x = 0 and x = 2, in the first quadrant is represented by a shaded region.

Integrals Area Bounded By The Circles

Required area = \(\int_0^2 y d x=\int_0^2 \sqrt{4-x^2} d x\)

= \(\left[\frac{x}{2} \sqrt{4-x^2}+\frac{4}{2} \sin ^{-1}\left(\frac{x}{2}\right)\right]_0^2\)

= \(0+2 \sin ^{-1}(1)-0=2 \times \frac{\pi}{2}=\pi \text { sq units. }\)

Thus the correct option is (1)

Question 4. Area of the region bounded by the curve y² = dx. Y-axis and the line y = 3 is.

  1. 2
  2. 9/4
  3. 9/3
  4. 9/2

Solution: 2. 9/4

The area bounded by the curve, y² = 4x, Y-axis, and y = 3 is represented in the figure by a shaded region.

Integrals Area Bounded By The Curve

Required area = \(\int_0^3 x d y\)

= \(\int_0^3 \frac{y^2}{4} d y=\frac{1}{4}\left[\frac{y^3}{3}\right]_0^3=\frac{1}{12}\left(3^3-0\right)=\frac{1}{12}(27)=\frac{9}{4}\) (sq units.)

Applications Of Integrals Miscellaneous Exercise

Question 1. Find the area under given curves and given lines.

  1. y = x²; x = 1, x = 2 and X-axis
  2. y = x4; x = 1, x = 5 and X-axis

Solution:

1. The given curve y = x² represents an upward parabola with vertex (0, 0) and axis along the Y-axis.

∴ Required area (Shown in shaded region)

= Area under the curve y = x² and between x = l1,x = 2

= \(\int_1^2 y d x=\int_1^2 x^2 d x=\left[\frac{x^3}{3}\right]_1^2=\frac{1}{3}\left[2^3-1^3\right]=\frac{8}{3}-\frac{1}{3}=\frac{7}{3}\) (sq. units.)

Integrals Area Under The Curve

2. Given y = x4

Here, x is even so the curve is symmetrical about Y -the axis and passes through the origin (0, 0)

∴ Required area = Area under the curve y = x4 and between x = 1 , x = 5

Integrals Area Under Given Curves

CBSE Class 12 Maths Chapter 8 Application Of Integrals Important Question And Answers

Question 2. Sketch the graph of y=|x+3| and evaluate \(\int_{-6}^0|x+3| d x\).
Solution:

y=|x+3|= \(\begin{cases}(x+3) & \text { for } x \geq-3 \\ -(x+3) & \text { for } x<-3\end{cases}\)

Now, \(\int_{-6}^0|x+3| d x\)

Integrals Area For The Graph

Required area = Area (ΔABC)+ Area(ΔOAD)

= \(\int_{-6}^{-3}(-\mathrm{x}-3) \mathrm{dx}+\int_{-3}^0(\mathrm{x}+3) \mathrm{dx}=\left[\frac{-\mathrm{x}^2}{2}-3 \mathrm{x}\right]_{-6}^{-3}+\left[\frac{\mathrm{x}^2}{2}+3 \mathrm{x}\right]_{-7}^0\)

= \(\left[\left(\frac{-(-3)^2}{2}-3 \times(-3)\right)-\left(\frac{-(-6)^2}{2}-3 \times(-6)\right)\right]+\left[0-\left(\frac{(-3)^2}{2}+3 \times(-3)\right)\right]\)

= \(\left[\left(\frac{-9}{2}+9\right)-(-18+18)\right]+\left[\frac{9}{2}\right]=\frac{9}{2}+\frac{9}{2}=9\) (Sq. units)

Question 3. Find the area bounded by the curve y = sin x between x = 0 and x = 2π
Solution:

The area of the region bounded by y = sin x, x = 0, and x = 2π is shown

Integrals Area Bounded By The Curve (2)

Required area = Area OABO + Area BCDB

= \(\int_0^\pi|\sin x| \mathrm{dx}+\int_\pi^{2 \pi}|\sin \mathrm{x}| \mathrm{dx}\)

= \(\int_0^\pi \sin \mathrm{x} x+\int_\pi^{2 \pi}(-\sin \mathrm{x}) \mathrm{dx}\)

(because \(\sin \mathrm{x} \geq 0\) for \(\mathrm{x} \in[0, \pi]\) and \(\sin \mathrm{x} \leq 0\) for \(\mathrm{x} \in[\pi, 2 \pi]\)

= \([-\cos \mathrm{x}]_0^\pi+[\cos \mathrm{x}]_\pi^{2 \pi}=-\cos \pi+\cos 0+\cos 2 \pi-\cos \pi=-(-1)+1+1-(-1)=4\) sq units.

Choose The Correct Answer

Question 4. The area bounded by the curve y = x³, the X-axis, and coordinates x = -2 and x = 1 is

  1. -9
  2. -15/4
  3. 15/4
  4. 17/4

Solution: 4. 17/4

Given curve is y = x³

On putting x = – x and y = — y we get y = x³

Therefore, the curve is symmetrical in the opposite quadrant and passes through (0, 0).

Integrals Curve Is Symmetrical Opposite Quadrant

∴ Required area

= \(\int_{-2}^1\left|\mathrm{x}^3\right| \mathrm{dx}\)

= \(\int_{-2}^0\left(-\mathrm{x}^3\right) \mathrm{dx}+\int_0^1 \mathrm{x}^3 \mathrm{dx}\) (because \(\left|\mathrm{x}^3\right|\))

= \(\int_{-2}^0\left(-x^3\right) \mathrm{d} x+\int_0^1 x^3 \mathrm{~d} x\)

(because \(\left|x^3\right|=\left\{\begin{array}{c}
x^3, \text { if } x \geq 0 \\
-x^3, \text { if } x<0
\end{array}\right\}\))

= \(-\left[\frac{x^4}{4}\right]_{-2}^0+\left[\frac{x^4}{4}\right]_0^1=-\left[0-\frac{(-2)^4}{4}\right]+\left[\frac{1}{4}-0\right]\)

= \(-(-4)+\left(\frac{1}{4}\right)=\frac{17}{4}\) sq. units.

So, the correct option is (4).

Question 5. The area bounded by the curve y = x|x|, X-axis, and the coordinates x = -1 and x = 1 is given by

  1. 0
  2. 1/3
  3. 2/3
  4. 4/3

Solution: 3. 2/3

Given y \(=x|x|=\left\{\begin{array}{l}
x^2, x \geq 0 \\
-x^2, x<0
\end{array}\right.\)

Required area

= 2 (Area under the curve y = x² between x = 0, x = 1)

Integrals Area Bounded By The Coordinates

The curve is symmetrical in the opposite; quadrant

= \(2 \int_0^1 x|x|=2 \int_0^1 x^2 d x\)

= \(2\left[\frac{x^3}{3}\right]_0^1=\frac{2}{3}\left(1^3-0^3\right)\)

= \(\frac{2}{3}\) sq. units

So, the correct option is (3).

Application of Derivatives Class 12 Maths Important Questions Chapter 6

Application Of Derivatives Exercise – 6.1

Question 1. Find the rate of change of the area of a circle with respect to its radius r when

r = 3 cm

r = 4 cm

Solution:

The area of a circle (A) with radius (r) is given by, A =\(\pi r^2\)

Now, the rate of change of the area with respect to its radius is given by,
\(\frac{d \Lambda}{d r}=\frac{d}{d r}\left(\pi r^2\right)=2 \pi r\)

1. When r = 3 cm,\(\frac{\mathrm{dA}}{\mathrm{dr}}=2 \pi(3)\) = 6 \(\pi\)

Hence, the area of the circle is changing at the rate of 6\(\pi {cm}^2 /cm\) when its radius is 3 cm.

2. When r = 4cm,\(\frac{d A}{d r}=2 \pi(4)=8 \pi\)

Hence, the area of the circle is changing at the rate of 8\(\pi \mathrm{cm}^2 / \mathrm{cm}\) when its radius is 4 cm.

Question 2. The volume of a cube is increasing at the rate of 8 \({cm}^2\)s. How fast is the surface area increasing when the length of an edge is 12 cm?

Solution:

Let x be the length of the side, V be the volume and S be the surface area of the cube.

Then, V=\(x^3\) and S=\(6 x^2\)

It is given that \(\frac{\mathrm{dV}}{\mathrm{dt}}=8 \mathrm{~cm}^3 / \mathrm{s}\)

Then, by using the chain rule, we have:

8=\(\frac{d V}{d t}=\frac{d}{d t}\left(x^3\right)-3 x^2, \frac{d x}{d t} \Rightarrow \frac{d x}{d t}=\frac{8}{3 x^2}\)

Now; \(\frac{d S}{d t}=\frac{d}{d t}\left(6 x^2\right)=\frac{d}{d x}\left(6 x^2\right) \cdot \frac{d x}{d t}\) [By chain rule]

=12 x \(\cdot \frac{d x}{d t}=12 x \cdot\left(\frac{8}{3 x^2}\right)=\frac{32}{x}\) [From (1)]

Thus, when x=12\( \mathrm{~cm}, \frac{\mathrm{dS}}{\mathrm{dt}}=\frac{32}{12} \mathrm{~cm}^2 / \mathrm{s}=\frac{8}{3} \mathrm{~cm}^2 / \mathrm{s}\)

Hence, if the length of the edge of the cube is 12 cm, then the surface area is increasing at the rate of \(\frac{8}{3} \mathrm{~cm}^2 / \mathrm{s}\).

Read and Learn More Class 12 Maths Chapter Wise with Solutions

Question 3. The radius of a circle is increasing uniformly at the rate of 3 cm/s. Find the rate at which the area of the circle is increasing when the radius is 10 cm.

Solution:

The area of a circle (A) with radius (r) is given by, \(\mathrm{A}=\pi r^2\)

Now, the rate of change of area (A) with respect to time (t) is given by,
\(\frac{d \mathrm{~A}}{\mathrm{dt}}=\frac{\mathrm{d}}{\mathrm{dt}}\left(\pi r^2\right)=2 \pi r \frac{\mathrm{dr}}{\mathrm{dt}}\)

It is given that, \(\frac{\mathrm{dr}}{\mathrm{dt}}=3 \mathrm{~cm} / \mathrm{s}\)

⇒ \(\frac{d A}{d t}=2 \pi r(3)=6 \pi r\)

Thus, when \(\mathrm{r}=10 \mathrm{~cm}, \frac{\mathrm{dA}}{\mathrm{dt}}=6 \pi(10)=60 \pi \mathrm{cm}^2 / \mathrm{s}\)

Hence, the rate at which the area of the circle is increasing when the radius is 10 cm, is 60n \({~cm}^2/s\).

Question 4. An edge of a variable cube is increasing at the rate of 3 cm/s. How fast is the volume of the cube increasing when the edge is 10 cm long?

Solution:

Let x be the length of a side and V be the volume of the cube. Then, V = \(x^2\)

⇒ \(\frac{\mathrm{dV}}{\mathrm{dt}}=3 \mathrm{x}^2 \cdot \frac{\mathrm{dx}}{\mathrm{dt}}\)

It is given that, \(\frac{\mathrm{dx}}{\mathrm{dt}}=3 \mathrm{~cm} / \mathrm{s}\)

⇒ \(\frac{d V}{d t}=3 x^2(3)=9 x^2\)

Therefore, when x=10 \(\mathrm{~cm}, \frac{d V}{d t}=9(10)^2=900 \mathrm{~cm}^3 / \mathrm{s}\)

Hence, the volume of the cube is increasing at the rate of 900 \({~cm}^3\) when the edge is 10 cm long.

Question 5. A stone is dropped into a quiet lake and waves move in circles at the speed of 5 cm/s. At the instant when the radius of the circular wave is 8 cm, how fast is the enclosed area increasing?

Solution:

The area of a circle (A) with radius (r) is given by A = \(\pi r^2\).

Therefore, the rate of change of area (A) with respect to time (t) is given by,

⇒ \(\frac{\mathrm{d} A}{\mathrm{dt}}=\frac{\mathrm{d}}{\mathrm{dt}}\left(\pi \mathrm{r}^2\right)=\frac{\mathrm{d}}{\mathrm{dr}}\left(\pi \mathrm{r}^2\right) \frac{\mathrm{dr}}{\mathrm{dt}}\)

=2 \(\pi \mathrm{r} \frac{\mathrm{dr}}{\mathrm{dt}}\) [By chain rule]

It is given that \(\frac{\mathrm{dr}}{\mathrm{dt}}=5 \mathrm{~cm} / \mathrm{s}\)

Thus, when \(\mathrm{r}=8 \mathrm{~cm}, \frac{\mathrm{dA}}{\mathrm{dt}}=2 \pi(8)(5)=80 \pi\)

Hence, when the radius of the circular wave is 8 cm, the enclosed area is increasing at the rate of 80n \({~cm}^2\).

CBSE Class 12 Maths Chapter 6 Application Of Derivatives Important Question And Answers

Question 6. The radius of a circle is increasing at the rate of 0.7 cm/s. What is the rate of increase of its circumference?

Solution:

The circumference of a circle (C) with radius ( r ) is given by \(\mathrm{C}-2 \pi \mathrm{r}\).

Therefore, the rate of change of circumference (C) with respect to time (t) is given by,

⇒ \(\frac{\mathrm{dC}}{\mathrm{dt}} =\frac{\mathrm{dC}}{\mathrm{dr}} \cdot \frac{\mathrm{dr}}{\mathrm{dt}}\)

⇒ \(=\frac{\mathrm{d}}{\mathrm{dr}}(2 \pi r) \frac{\mathrm{dr}}{\mathrm{dt}}=2 \pi \cdot \frac{\mathrm{dr}}{\mathrm{dt}}
\) [By chain rule]

It is given that \(\frac{\mathrm{dr}}{\mathrm{dt}}=0.7 \mathrm{~cm} / \mathrm{s}\)
Hence, the rate of increase of the circumference is \(2 \pi(0.7)=1.4 \pi \mathrm{cm} / \mathrm{s}\).

Question 7. The length x of a rectangle is decreasing at the rate of 5 cm/minute and the width y is increasing at the rate of 4 cm/minute. When x = 8 cm and y = 6 cm., find the rates of change of (a) the perimeter, and (b) the area of the rectangle..

Solution:

Since the length (x) is decreasing at the rate of 5 cm/minute and the width (y) is increasing at the rate of 4 cm/minute, we have:

⇒ \(\frac{\mathrm{dx}}{\mathrm{dt}}=-5 \mathrm{~cm} / \mathrm{min} \) and \(\frac{\mathrm{dy}}{\mathrm{dt}}=4 \mathrm{~cm} / \mathrm{min}
\)

1. The perimeter (P) of a rectangle is given by, P=2(x+y)

⇒ \(\frac{\mathrm{dP}}{\mathrm{dt}}=2\left(\frac{\mathrm{dx}}{\mathrm{dt}}+\frac{\mathrm{dy}}{\mathrm{dt}}\right)=2(-5+4)=-2 \mathrm{~cm} / \mathrm{min}\)

Hence, the perimeter is decreasing at the rate of \(2 \mathrm{~cm} / \mathrm{min}\).

2. The area (A) of a rectangle is given by, A=\(x \times y\)

⇒ \(\frac{d A}{d t}=\frac{d x}{d t} \cdot y+x \cdot \frac{d y}{d t}=-5 y+4 x\)

When x=8 cm and y=6 cm, \(\frac{\mathrm{dA}}{\mathrm{dt}}=(-5 \times 6+4 \times 8) \mathrm{cm}^2 / \mathrm{min}=2 \mathrm{~cm}^2 / \mathrm{min}\)

Hence, the area of the rectangle is increasing at the rate of 2 \({~cm}^2\)min.

Question 8. A balloon, which always remains spherical on inflation, is being inflated by pumping in 900 cubic centimeters of gas per second. Find the rate at which the radius of the balloon increases when the radius is 15 cm.

Solution:

The volume of a sphere (V) with radius (r) is given by, V =\(\frac{4}{3} \pi \mathrm{r}^3\)

The rate of change of volume (V) with respect to time (t) is given by,

⇒ \(\frac{d V}{d t} =\frac{d V}{d r} \cdot \frac{d r}{d t}\) [Chain rule]

=\(\frac{d}{d r}\left(\frac{4}{3} \pi r^3\right) \cdot \frac{d r}{d t}=4 \pi r^2 \cdot \frac{d r}{d t}\)

It is given that \(\frac{\mathrm{dV}}{\mathrm{dt}}=900 \mathrm{~cm}^3 / \mathrm{s}\)

900=4 \(\pi \mathrm{r}^2 \cdot \frac{\mathrm{dr}}{\mathrm{dt}} \Rightarrow \frac{\mathrm{dr}}{\mathrm{dt}}=\frac{900}{4 \pi \mathrm{r}^2}=\frac{225}{\pi \mathrm{r}^2}\)

Therefore, when radius -15 \(\mathrm{~cm}, \frac{\mathrm{dr}}{\mathrm{dt}}=\frac{225}{\pi(15)^2}=\frac{1}{\pi}\)

Hence, the rate at which the radius of the balloon increases, when the radius is 15 cm, is \(\frac{1}{\pi} \mathrm{cm} / \mathrm{s}\).

Question 9. A balloon, which always remains spherical has a variable radius. Find the rate at which its volume is increasing with the radius when the latter is 10 cm.

Solution:

The volume of a sphere (V) with radius (r) is given by \(\mathrm{V}=\frac{4}{3} \pi \mathrm{r}^3\).

The rate of change of volume (V) with respect to its radius (r) is given by,

⇒ \(\frac{d V}{d r}=\frac{d}{d r}\left(\frac{4}{3} \pi r^3\right)=\frac{4}{3} \pi\left(3 r^2\right)=4 \pi r^2\)

Therefore when radius =10 \(\mathrm{~cm}, \frac{d V}{d r}=4 \pi(10)^2=400 \pi\)

Hence, the volume of the balloon is increasing at the rate of 400 \(\pi \mathrm{cm}^7 / \mathrm{cm}\).

Question 10. A ladder 5 m long is leaning against a wall. The bottom of the ladder is pulled along the ground, away from the wall, at the rate of 2 cm/s. How fast is its height on the wall decreasing when the foot of the ladder is 4 m away from the wall?

Solution:

Let y m be the height of the wall at which the ladder touches. Also, let the foot of the ladder be x m away from the wall.

Then, by Pythagoras theorem, we have: \(x^2+y^2\)=25 [Length of the ladder =5m]

y=\(\sqrt{25-x^2}\)

Then, the rate of change of height (y) with respect to time (t) is given by \(\frac{d y}{d t}=\frac{-x}{\sqrt{25-x^2}} \cdot \frac{d x}{d t}\)

It is given that \(\frac{d x}{d t}=0.02 \mathrm{~m} / \mathrm{s}  \frac{d y}{d t}=\frac{-x}{\sqrt{25-x^2}} \times \frac{2}{100} \mathrm{~m} / \mathrm{s}\)

Now when x=4 \(\mathrm{~m}, we have: \frac{d y}{d t}=\frac{-4 \times 2}{100 \times 3} \mathrm{~m} / \mathrm{s}\)

=-\(\frac{8}{300} \mathrm{~m} / \mathrm{s}=-\frac{8}{3} \mathrm{~cm} / \mathrm{s}\)

Hence, the height of the ladder on the wall is decreasing at the rate of \(\frac{8}{3} \mathrm{~cm} / \mathrm{s}\),

Question 11. A particle moves along the curve \(6 y=x^3+2\). Find the points on the curve at which the y-coordinate is changing 8 times as fast as the x-coordinate.

Solution:

The equation of the curve is given as:

6 y=\(x^3+2\)

The rate of change of the position of the particle with respect to time (t) is given by

6 \(\frac{d y}{d t}=3 x^2 \frac{d x}{d t}+0 \Rightarrow 2 \frac{d y}{d t}=x^2 \frac{d x}{d t}\)

When the y-coordinate of the particle changes 8 times as fast as the x-coordinate i.e., \(\left(\frac{\mathrm{dy}}{\mathrm{dt}}=8 \frac{\mathrm{dx}}{\mathrm{dt}}\right)\), we have:

2\(\left(8 \frac{d x}{d t}\right)=x^2 \frac{d x}{d t} \Rightarrow 16 \frac{d x}{d t}=x^2 \frac{d x}{d t}\)

⇒ \(\left(x^2-16\right) \frac{d x}{d t}=0 \Rightarrow x^2=16 \Rightarrow x= \pm 4\)

When x=4, y=\(\frac{4^3+2}{6}=\frac{66}{6}=11\)

When x=-4, y=\(\frac{(-4)^3+2}{6}=-\frac{62}{6}=-\frac{31}{3}\)

Hence, the points on the curve are (4,11) and \(\left(-4, \frac{-31}{3}\right)\)

Question 12. The radius of an air bubble is increasing at the rate of \(\frac{1}{2} \mathrm{~cm} / \mathrm{s}\). At what rate is the volume of the bubble increasing when the radius is 1 cm ?

Solution:

The air bubble is in the shape of a sphere.

Now, the volume of an air bubble (V) with radius ( r ) is given by, V=\(\frac{4}{3} \pi r^3\)

The rate of change of volume (V) with respect to time (t) is given by,

⇒ \(\frac{\mathrm{dV}}{\mathrm{dt}} =\frac{4}{3} \pi \frac{\mathrm{d}}{\mathrm{dr}}\left(\mathrm{r}^3\right) \cdot \frac{\mathrm{dr}}{\mathrm{dt}}\)

=\(\frac{4}{3} \pi\left(3 \mathrm{r}^2\right) \frac{\mathrm{dr}}{\mathrm{dt}}-4 \pi \mathrm{r}^2 \frac{\mathrm{dr}}{\mathrm{dt}}\) [By chain rule]

It is given that \(\frac{\mathrm{dr}}{\mathrm{dt}}=\frac{1}{2} \mathrm{~cm} / \mathrm{s}\)

Therefore, when \(\mathrm{r}=1 \mathrm{~cm}, \frac{\mathrm{dV}}{\mathrm{dt}}=4 \pi(1)^2\left(\frac{1}{2}\right)=2 \pi \mathrm{cm}^3 / \mathrm{s}\)

Hence, the rate at which the volume of the bubble increases is 2 \(\pi \mathrm{cm}^3 / \mathrm{s}\).

Question 13. A balloon, which always remains spherical, has a variable diameter \(\frac{3}{2}(2 x+1)\). Find the rate of change of its volume with respect to x.

Solution:

The volume of a sphere (V) with radius (r) is given by, V = \(\frac{4}{3} \pi r^3\)

It is given that:

Diameter =\(\frac{3}{2}(2 x+1) \Rightarrow r=\frac{3}{4}(2 x+1)\)

V=\(\frac{4}{3} \pi\left(\frac{3}{4}\right)^3(2 x+1)^3=\frac{9}{16} \pi(2 x+1)^3\)

Hence, the rate of change of volume with respect to x is:

⇒ \(\frac{\mathrm{dV}}{\mathrm{dx}}=\frac{9}{16} \pi \frac{\mathrm{d}}{\mathrm{dx}}(2 \mathrm{x}+1)^3\)

= \(\frac{9}{16} \pi \times 3(2 \mathrm{x}+1)^2 \cdot \times 2=\frac{27}{8} \pi(2 \mathrm{x}+1)^2\)

Question 14. Sand is poured from a pipe at the rate of 12 cm. The falling sand forms a cone on the ground in such a way that the height of the cone is always one-sixth of the radius of the base. How fast is the height of the sand cone increasing when the height is 4 cm?

Solution:

The volume of a cone (V) with radius (r) and height (h) is given by, \(\mathrm{V}=\frac{1}{3} \pi r^2 \mathrm{~h}\)

It is given that, h=\(\frac{1}{6} r \Rightarrow r=6 h\)

V=\(\frac{1}{3} \pi(6 h)^7 h=12 \pi h^3\)

The rate of change of volume with respect to time (t) is given by,

⇒ \(\frac{d V}{d t}=12 \pi \frac{d}{d h}\left(h^2\right) \cdot \frac{d h}{d t}\)

=12 \(\pi\left(3 h^2\right) \frac{d h}{d t}=36 \pi h^2 \frac{d h}{d t}\) [By Chain rule]

It is also given that \(\frac{d V}{d t}=12 \mathrm{~cm}^3 / \mathrm{s}\)

Therefore, when h=4 cm, we have:

12=36 \(\pi(4)^2 \frac{\mathrm{dh}}{\mathrm{dt}}\)

⇒ \(\frac{\mathrm{dh}}{\mathrm{dt}}=\frac{12}{36 \pi(16)}=\frac{1}{48 \pi}\)

Hence, when the height of the sand cone is 4 cm, its height is increasing at the rate of \(\frac{1}{48 \pi} \mathrm{cm} / \mathrm{s}\).

Question 15. The total cost C (x) in Rupees associated with the production of x units of an item is given by C(x)=\(0.007 x^3-0.003 x^2+15\) x+4000. Find the marginal cost when 17 units are produced.

Solution:

Marginal cost is the rate of change of total cost with respect to output.

Marginal cost (M C)=\(\frac{d C}{d x}=0.007\left(3 x^2\right)\)-0.003(2 x)+15=0.021 x^2-0.006 x+15

When x=17, MC =0.021\(\left(17^3\right)\)-0,006(17)+15

= 0.021(289) – 0.006(17) + 15 – 6.069 – 0.102 + 15 – 20.967

Hence, when 17 units are produced, the marginal cost is Rs, 20.967.

Question 16. The total revenue in Rupees received from the sale of x units of a product is given by R(x) = \(13 x^1+26 x+15\). Find the marginal revenue when x = 7.

Solution:

Marginal revenue is the rate of change of total revenue with respect to the number of units sold.

Marginal Revenue(M R)=\(\frac{d R}{d x}\)=13(2 x)+26=26 x+26

When x = 7, MR = 26(7) + 26 = 182 + 26 = 208.

Hence, the required marginal revenue is Rs 208.

Question 17. The rate of change of the area of a circle with respect to its radius r at r = 6 cm is?

  1. 10 \(\pi\)
  2. 12 \(\pi\)
  3. 8 \(\pi\)
  4. 11 \(\pi\)

Solution: 2. 12 \(\pi\)

The area of a circle (A) with radius (r) is given by, A=\(\pi \mathrm{r}^2\)

Therefore, the rate of change of the area with respect to its radius r is \(\frac{\mathrm{dA}}{\mathrm{dr}}=\frac{\mathrm{d}}{\mathrm{dr}}\left(\pi \mathrm{r}^2\right)=2 \pi r\).

When \(\mathrm{r}=6 \mathrm{~cm}, \frac{\mathrm{dA}}{\mathrm{dr}}=2 \pi \times 6=12 \pi \mathrm{cm}^2 / \mathrm{cm}\)

Hence, the required rate of change of the area of a circle is 12 \(\pi \mathrm{cm}^2 cm\)

The correct answer is 2.

Question 18. The total revenue in Rupees received from the sale of x units of a product is given by R(x) = \(3 x^2+36 x+5\). The marginal revenue, when x = 15, is?

  1. 116
  2. 96
  3. 90
  4. 126

Solution: 4. 126

Marginal revenue is the rate of change of total revenue with respect to the number of units sold.

Marginal Revenue (MR) = \(\frac{\mathrm{dR}}{\mathrm{dx}}=3(2 \mathrm{x})+36=6 x +36\)

When x = 15, MR = 6(15) + 36 = 90 + 36 = 126

Hence, the required marginal revenue is Rs 126.

The correct answer is 4.

Applications Of Derivatives Exercise 6.2

Question 1. Show that the function given by f(x)=3 x+17 is strictly increasing on R.

Solution:

Let \(x_2, x_2 \in\) R, such that \(x_1<x_1 \Rightarrow 3 x_1<3 x_1 \Rightarrow 3 x_1+17<3 x_2+17 \Rightarrow f\left(x_1\right)<f\left(x_2\right)\)

Hence, f is strictly increasing on R.

Alternate method:

Given \(f^{\prime}(x)=3 x+17\). On diff. w.r.t. x, we get \(f^{\prime}(x)=3>0\), in every interval of R.

Thus, the function is strictly increasing on R.

Question 2. Show that the function given by $f(x)=e^{2 n}$ is strictly increasing on R.

Solution:

Let \(x_1, x_2 \in\) R, such that \(x_1<x_1\)

⇒ \(2 x_1<2 x_1 \Rightarrow e^{2 x_1}<e^{2 x_1}\)

f\(\left(x_1\right)<f\left(x_2\right)\)

Hence, f is strictly increasing on R.

Question 3. Show that the function given by f(x)=\(\sin x\) is

  1. strictly increasing in \(\left(0, \frac{\pi}{2}\right)\)
  2. strictly decreasing in \(\left(\frac{\pi}{2}, \pi\right)\)
  3. neither increasing nor decreasing in (0, \(\pi\))

Solution:

The given function is f(x)-\(\sin x\).

⇒ \(f^{\prime}(x)=\cos \mathrm{x}\)

1. Since for each x \(\in\left(0, \frac{\pi}{2}\right), \cos x>0\) we have \(f^{\prime}(x)>0\),

Hence, f is strictly increasing in \(\left(0, \frac{\pi}{2}\right)\).

2. Since for each x \(\in\left(\frac{\pi}{2}, \pi\right), \cos x<0\), we have \(f^{\prime \prime}(x)<0\).

Hence, f is strictly decreasing in \(\left(\frac{\pi}{2}, \pi\right)\)

3. From the results obtained in (1) and (2) it is clear that f is strictly increasing in \(\left(0, \frac{\pi}{2}\right)[\) and strictly decreasing in \(\left(\frac{\pi}{2}, \pi\right)\)

So, f is neither increasing nor decreasing in (0, \(\pi\)).

Question 4. Find the intervals in which the function f given by f(x)=2 \(x^2-3\) x is

  1. strictly increasing
  2. strictly decreasing

Solution:

The given function is f(x)=2 \(x^2-3 x, f^{\prime}(x)\)=4 x-3

Putting f(x)=0 \(\Rightarrow 4 x-3=0 \Rightarrow x=\frac{3}{4}\)

Now, the point \(\frac{3}{4}\) divides the real line into two disjoint intervals i.e. \(\left(-\infty, \frac{3}{4}\right) and \left(\frac{3}{4}, \infty\right)\)

Applications Of Derivatives Two Disjoint Intervals

In interval \(\left(-\infty, \frac{3}{4}\right), \mathrm{f}^{\prime}(\mathrm{x})<0\)

Hence, the given function (f) is strictly decreasing in the interval \(\left(-\infty, \frac{3}{4}\right)\).

In interval \(\left(\frac{3}{4}, \infty\right), \mathrm{f}^{\prime}(\mathrm{x})>0\)

Hence, the given function (f) is strictly increasing in the interval \(\left(\frac{3}{4}, \infty\right)\).

Question 5. Find the intervals in which the function f given by f(x)=2 \(x^3-3 x^2-36 x+7\) is

  1. Strictly increasing
  2. Strictly decreasing

Solution:

The given function is f(x)=\(2 x^3-3 x^2-36 x+7\).

⇒ \(f(x)=6 x^2-6 x-36=6\left(x^2-x-6\right)=6(x+2)(x-3)\)

⇒ \(\mathrm{f}(\mathrm{x})=0 \Rightarrow \mathrm{x}=-2,3\)

The points x=-2 and x =3 divide the real line into three disjoint intervals i.e, \((-\infty,-2),(-2,3) and (3, \infty)\)

Applications Of Derivatives Three Disjoint Intervals

In intervals \((-\infty,-2) and (3, \infty), f^{\prime}(x)\) is positive while in interval \((-2,3), f^{\prime}(x)\) is negative.

Hence, the given function f(x) is strictiy increasing in intervals \((-\infty,-2) \cup(3, \infty)\), while function f(x) is strictly decreasing in interval (-2,3).

Question 6. Find the intervals in which the following functions are strictly increasing or decreasing:

  1. \(x^2+2 x-5\)
  2. 10-6 \(\mathrm{x}-2 \mathrm{x}^2\)
  3. \(-2 x^3-9 x^2-12 x+1\)
  4. \(6-9 x-x^2\)

Solution 1. \(x^2+2 x-5\)

⇒ \((x+1)^3(x-3)^2\)

1. We have, \(f(x)=x^7+2 x-5\)

⇒ \(f^{\prime}(x)=2 x+2\)

Now, \(\mathrm{f}^{\prime}(\mathrm{x})=0 \Rightarrow \mathrm{x}=-1\)

Point x=-1 divides the real line into two disjoint intervals i.e., \((-\infty,-1) and (-1, \infty)\)

Applications Of Derivatives The Real Line Into Two Disjoint Intervals

In interval \((-\infty,-1), f^{\prime}(x)<0\).

f is strictly decreasing in the interval (-\infty,-1)

Thus, f is strictly decreasing for x<-1.

In interval (-1, \(\infty), f^{\prime}(x)>0\)

f is strictly increasing in interval \((-1, \infty)\)

Thus, f is strictly increasing for x>-1

2. We have, f(x)=\(10-6 x-2 x^3\)

\(f^{\prime}(x)=-6-4 x\)

Now, \(\mathrm{r}^{\prime}(\mathrm{x})=0 \Rightarrow \mathrm{x}=-\frac{3}{2}\)

The point x=-\(\frac{3}{2}\) divides the real line into two disjoint intervals i.e., \(\left(-\infty,-\frac{3}{2}\right) and \left(-\frac{3}{2}, \infty\right)\)

Applications Of Derivatives The Point Divides The Real Line Into Two Disjoint Intervals

In interval \(\left(-\infty,-\frac{3}{2}\right)\) i.e., when \(x<-\frac{3}{2}, f^{\prime}(x)>0\).

f is strictly increasing for \(\mathrm{x}<-\frac{3}{2}\).

In interval \(\left(-\frac{3}{2}, \infty\right)\) i.e., when \(x>-\frac{3}{2}, f^{\prime}(x)<0\).

f is strictly decreasing for \(x>-\frac{3}{2}\)

3. We have, f(x)=\(-2 x^3-9 x^2-12 x+1\)

f(x)=\(-6 x^2-18 x-12=-6\left(x^2+3 x+2\right)=-6(x+1)(x+2)\)

Now; \(f^{\prime}(x)=0 \Rightarrow x=-1\) and x=-2

Points x=-1 and x=-2 divide the real line into three disjoint intervals
i.e., \((-\infty,-2),(-2,-1)\) and (-1, \(\infty)\)

Applications Of Derivatives The Real Line Into Three Disjoint Intervals

In intervals \((-\infty,-2) and (-1, \infty)\) i.e., when x<-2 and \(x>-1, f^{\prime}(x)<0\)

f is strictly decreasing for x<-2 \(\cup x>-1.\)

Now, in interval (-2,-1) i.e., when \(-2<x<-1, f^{\prime}(x)>0.\)

f is strietly increasing for \(-2<\mathrm{x}<-1\).

4. We have, f(x)=\(6-9 x-x^2\)

⇒ \(f^{\prime}(x)\)=-9-2 x

Now, \(f^{\prime}(x)\)=0 gives x=-\(\frac{9}{2}\)

The point x =\(-\frac{9}{2}\) divides the real line into two disjoint intervals ie. \(\left(-\infty,-\frac{9}{2}\right)\) and \(\left(-\frac{9}{2}, \infty\right)\)

Applications Of Derivatives The Point X Divides The Real Line Into Two Disjoint Intervals

In interval \(\left(-\infty,-\frac{9}{2}\right)\) i.e., for \(x<-\frac{9}{2}, f^{\prime}(x)>0\).

f is strictly increasing for \(\mathrm{x}<-\frac{9}{2}\)

In interval \(\left(-\frac{9}{2}, \infty\right)\) i.e., for \(x>-\frac{9}{2}, f^{\prime}(x)<0\).

f is strictly decreasing for \(\mathrm{x}>-\frac{9}{2}\).

5. We have, f(x)=\((x+1)^3(x-3)^5\)

⇒ \(f^{\prime}(x) =3(x+1)^2(x-3)^3+3(x-3)^2(x+1)^3=3(x+1)^2(x-3)^2[x-3+x+1]\)

=\(3(x+1)^2(x-3)^2(2 x-2)=6(x+1)^4(x-3)^2(x-1)\)

Now, \(f^{\prime}(x)=0 \Rightarrow x=-i, 3, i\)

The points x=-1, x 1, and x=3 divide the real line into four disjoint intervals i.e., \((-\infty,-1),(-1,1),(1,3)\) and \((3, \infty\)).

Applications Of Derivatives Four Disjoint Intervals

In intervals \((-\infty,-1)\) and \((-1,1), \mathrm{f}^{\prime}(\mathrm{x})<0\).

f is strictly decreasing in intervals \((-\infty,-1) \cup(-1,1)\).

In intervals (1,3) and (3, \(\infty), f^{\prime}(x)>0\).

f is strictly increasing in intervals (1,3) \(\cup(3, \infty)\)

Question 7. Show that y=\(\log (1+x)-\frac{2 x}{2+x}\), x>-1, is an increasing function of x throughout its domain.

Solution:

We have, y=\(\log (1+x)-\frac{2 x}{2+x}\)

⇒ \(\frac{d y}{d x}=\frac{1}{1+x}-\frac{(2+x)(2)-2 x(1)}{(2+x)^2}=\frac{1}{1+x}-\frac{4}{(2+x)^2}=\frac{x^2}{(1+x)(2+x)^2}\)

When x \(\in(-1, \infty), then \frac{x^2}{(2+x)^2}>0\) and (1+x)>0

⇒ \(\frac{d y}{d x}>0\) when x>-1.

y is an increasing function of x throughout its domain, i.e. (x>-1)

Question 8. Find the values of x for which y=\([x(x-2)]^2\) is an increasing function.

Solution:

We have, y=\([x(x-2)]^2-\left[x^2-2 x\right]^1\)

⇒ \(\frac{d y}{d x}=2\left(x^2-2 x\right)(2 x-2)=4 x(x-2)(x-1)\)

⇒ \(\frac{d y}{d x}=0 \Rightarrow\) x=0, x=2, x=1

The points x =0, x =1, and x=2 divide the real line into four disjoint intervals i.e., \((-\infty, 0), [0,1][1,2]\) and \([2, \infty)\)

Applications Of Derivatives The Real Line Into Four Disjoint Intervals

In intervals \((-\infty, 0]\) and \([1,2], \frac{\mathrm{dy}}{\mathrm{dx}} \leq 0\)

y is decreasing in intervals \((-\infty, 0] \cup[1,2]\).

However, in intervals [0,1] and [2, x), \(\frac{d y}{d x} \geq 0\)

y is increasing in intervals [0,1] \(\cup[2, \infty)\).

Question 9. Prove that y=\(\frac{4 \sin \theta}{(2+\cos \theta)}-\theta\) is an increasing function of \(\theta\) in \(\left[0, \frac{\pi}{2}\right]\).

Solution:

We have, y=\(\frac{4 \sin \theta}{(2+\cos \theta)}-\theta\)

⇒ \(\frac{d y}{d \theta} =\frac{(2+\cos \theta)(4 \cos \theta)-4 \sin \theta(-\sin \theta)}{(2+\cos \theta)^2}-1\)

=\(\frac{8 \cos \theta+4 \cos ^2 \theta+4 \sin ^2 \theta}{(2+\cos \theta)^2}-1=\frac{8 \cos \theta+4}{(2+\cos \theta)^2}-1\)

⇒ \(\frac{d y}{d \theta}=\frac{8 \cos \theta+4-\left(4+\cos ^2 \theta+4 \cos \theta\right)}{(2+\cos \theta)^2}-\frac{4 \cos \theta-\cos ^2 \theta}{(2+\cos \theta)^2}\)

=\(\frac{\cos \theta(4-\cos \theta)}{(2+\cos \theta)^2}\)

In interval \(\left[0, \frac{\pi}{2}\right]\), we have \(\cos \theta \geq 0\). Also, \(4>\cos \theta \Rightarrow 4-\cos \theta>0\)

⇒ \(\cos \theta(4-\cos \theta)>0\) and also \((2+\cos \theta)^2>0\)

⇒ \(\frac{\cos \theta(4-\cos \theta)}{(2+\cos \theta)^2} \geq 0 \Rightarrow \frac{d y}{d x} \geq 0\)

Therefore, y is increasing in the interval \(\left[0, \frac{\pi}{2}\right]\).

Question 10. Prove that the logarithmic function is strictly increasing on (0, \(\infty)\).

Solution:

Let f(x)=\(\log x\).

⇒ \(\mathrm{f}^{\prime}(\mathrm{x})=\frac{1}{\mathrm{x}}\)

It is clear that for x>0, \(f^{\prime}(x)=\frac{1}{x}>0\)

Hence, f(x)= log x is strictly increasing in the interval \((0, \infty)\).

Question 11. Prove that the function f given by f(x)=\(x^2-x+1\) is neither strictly increasing nor strictly decreasing on (-1,1).

Solution:

The given function is f(x)=\(x^2-x+1\).

⇒ \(f^{\prime}(x)=2 x-1\)

Now, \(f^{\prime}(x)=\theta \Rightarrow x=\frac{1}{2}\)

The point \(\frac{1}{2}\) divides the interval (-1,1) into two disjoint intervals i.e., \(\left(-1, \frac{1}{2}\right) and \left(\frac{1}{2}, 1\right)\)

Applications Of Derivatives The Point Divides The Interval Into Two Disjoint Intervals

Now, in interval \(\left(-1, \frac{1}{2}\right), \mathrm{f}^{\prime}(\mathrm{x})=2 \mathrm{x}-1<0\)

Therefore, f is strictly decreasing in the interval \(\left(-1, \frac{1}{2}\right)\).

However, in interval \(\left(\frac{1}{2}, 1\right), f^{\prime}(x)=2 x-1>0\)

Therefore, f is strictly increasing in the interval \(\left(\frac{1}{2}, 1\right)\).

Hence, f is neither strictly increasing nor strictly decreasing in interval (-1,1).

Question 12. Which of the following functions are strictly decreasing on \(\left(0, \frac{\pi}{2}\right)\)?

  1. \(\cos x\)
  2. \(\cos 2 x\)
  3. \(\cos 3 x\)
  4. \(\tan x\)

Solution:

1. Let \(f_2(x)=\cos x\)

⇒ \(f_1^{\prime}(x)=-\sin x\)

In interval \(\left(0, \frac{\pi}{2}\right), f_1^{\prime}(x)=-\sin x<0\)

⇒ \(\mathrm{f}_1(\mathrm{x})\) is strictly decreasing in interval \(\left(0, \frac{\pi}{2}\right)\).

2. Let \(f_2(x)=\cos 2 x\)

⇒ \(f_2(x)=-2 \sin 2 x\)

Now, \(0<x<\frac{\pi}{2} \Rightarrow 0<2 x<\pi \Rightarrow \sin 2 x>0 \Rightarrow-2 \sin 2 x<0\)

⇒ \(\mathrm{f}_2^{\prime}(\mathrm{x})=-2 \sin 2 \mathrm{x}<0 on \left(0, \frac{\pi}{2}\right)\)

⇒ \(\mathrm{f}_2(\mathrm{x})\) is strictly decreasing in interval \(\left(0, \frac{\pi}{2}\right)\).

3. Let \(f_1(x)=\cos 3 x\).

⇒ \(f_5^{\prime}(x)=-3 \sin 3 x\)

Now, \(f^{\prime}(x)=0\)

⇒ \(\sin 3 x=0 \Rightarrow 3 x=\pi, as x \in\left(0, \frac{\pi}{2}\right) \Rightarrow x=\frac{\pi}{3}\)

The point \(\mathrm{x}=\frac{\pi}{3}\) divides the interval \(\left(0, \frac{\pi}{2}\right)\) into two disjoint intervals i.e., \(\left(0, \frac{\pi}{3}\right)\) and \(\left(\frac{\pi}{3}, \frac{\pi}{2}\right)\)

Now in interval \(\left(0, \frac{\pi}{3}\right), f_3^{\prime}(x)=-3 \sin 3 x<0\) \(\left[\right. as \left.0<x<\frac{\pi}{3} \Rightarrow 0<3 x<\pi\right]\)

⇒ \(\mathrm{f}(\mathrm{x})\) is strictly decreasing in interval \(\left(0, \frac{\pi}{3}\right)\)

However, in interval \(\left(\frac{\pi}{3}, \frac{\pi}{2}\right), \mathrm{f}^{\prime}(\mathrm{x})\)

=\(-3 \sin 3 \mathrm{x}>0 \left[\right.as\left.\frac{\pi}{3}<\mathrm{x}<\frac{\pi}{2} \Rightarrow \pi<3 \mathrm{x}<\frac{3 \pi}{2}\right]\)

⇒ \(\mathrm{f}_3(\mathrm{x})\) is strictly increasing in interval \(\left(\frac{\pi}{3}, \frac{\pi}{2}\right)\).

Hence, \(f_1\) is neither strictly increasing nor strictly decreasing in the interval \(\left(0, \frac{\pi}{2}\right)\).

4. Let \(f_4(x)=\tan x\)

⇒ \(f_4^{\prime}(x)=\sec ^2 x\)

In interval \(\left(0, \frac{\pi}{2}\right), \mathrm{f}_4^{\prime}(\mathrm{x})=\sec ^2 \mathrm{x}>0\)

⇒ \(\mathrm{f}_4\) is strictly increasing in interval \(\left(0, \frac{\pi}{2}\right)\)

Therefore, functions \(\cos x\) and \(\cos 2 x\) are strictly decreasing in \(\left(0, \frac{\pi}{2}\right)\)

Hence, the correct answers are 1 and 2.

Question 13. On which of the following intervals is the function f given by f(x)=\(x^{100}+\sin x-1\) strictly decreasing?

Solution:

  1. (0,1)
  2. \(\left(\frac{\pi}{2}, \pi\right)\)
  3. \(\left(0, \frac{\pi}{2}\right)\)
  4. None of these

Solution: 4. None of these

We have, \(f(x)=x^{i n g}+\sin x-1\)

⇒ \(f^{\prime}(x)=100 x^{00}+\cos x\)

In interval (0,1),cos x>0 and 100 x^n>0

⇒ \(f^{\prime}(x)>0\)

Thus, function f is strictly increasing in interval (0,1).

In interval \(\left(\frac{\pi}{2}, \pi\right), \cos x<0 and 100 x^{1 / 4}>0. Also, 100 x^*>\cos x\)

⇒ \(f^{\prime}(x)>0 in \left(\frac{\pi}{2}, \pi\right)\).

Thus, the function is strictly increasing in the interval \(\left(\frac{\pi}{2}, \pi\right)\)

In interval \(\left(0, \frac{\pi}{2}\right), \cos x>0\) and \(100 x^{\infty}>0 100 x^n+\cos x>0\)

⇒ \(\mathrm{f}^{\prime}(\mathrm{x})>0 on \left(0, \frac{\pi}{2}\right)\)

f is strictly increasing in interval \(\left(0, \frac{\pi}{2}\right)\)

Hence function f is strictly decreasing in none of the intervals.

The correct answer is 4.

Question 14. Find the least value of a such that the function f given by f(x)=\(x^2+a x+1\) is strictly increasing on [1,2].

Solution:

We have, f(x)=\(x^2+a x+1\)

⇒ \(\mathrm{f}^{\prime}(\mathrm{x})=2 \mathrm{x}+\mathrm{a}\)

Now, \(1<\mathrm{x}<2 \Rightarrow 2+\mathrm{a}<2 \mathrm{x}+\mathrm{a}<4+\mathrm{a} \Rightarrow 2+\mathrm{a}<\mathrm{f}^{\prime}(\mathrm{x})<4+\mathrm{a}\)

Now, function f will be strictly increasing in [1,2], if \(f^{\prime}(x)>0 in [1,2]\).

⇒ \(\Rightarrow 2+a \geq 0 \Rightarrow a \geq-2\)

The least Value of a for f to be strictly increasing in [1,2] is a=-2,

Question 15. Let I be any interval disjoint from [-1,1]. Prove that the function f given by \(f(x)=x+\frac{1}{x}\) is strictly increasing on I.

Solution:

We have, \(f(x)=x+\frac{1}{x} f^{\prime}(x)=1-\frac{1}{x^2} \Rightarrow f^{\prime}(x)=\frac{x^2-1}{x^2}\)

Now, \(\mathrm{f}^{\prime}(\mathrm{x})-0 \Rightarrow \mathrm{x}= \pm 1\)

The points x =1 and x=-1 divide the real line in three disjoint intervals i.e., \((-\infty,-1),(-1,1)\) and \((1, \infty)\).

f is strictly increasing on \((-\infty,-1) \cup(1, \infty)\)

Hence, function f is strictly increasing in the interval I disjoint from [-1,1].

Hence, the given result is proved.

Question 16. Prove that the function f given by f(x)=\(\log \sin x\) is strictly increasing on \(\left(0, \frac{\pi}{2}\right)\) and strictly decreasing on \(\left(\frac{\pi}{2}, \pi\right)\).

Solution:

We have, f(x)=\(\log \sin x\)

⇒ \(f^{\prime}(x)=\frac{1}{\sin x}(\cos x)=\cot x\)

In interval \(\left(0, \frac{\pi}{2}\right), \mathrm{f}^{\prime}(\mathrm{x})=\cot \mathrm{x}>0 \mathrm{f}\) is strictly increasing on \(\left(0, \frac{\pi}{2}\right)\).

In interval \(\left(\frac{\pi}{2}, \pi\right), f^{\prime}(x)=\cot x<0\), f is strictly decreasing on \(\left(\frac{\pi}{2}, \pi\right)\).

Question 17. Prove that the function f given by f(x)=\(\log \cos x\) is strictly decreasing on \(\left(0, \frac{\pi}{2}\right)\) and strictly increasing on \(\left(\frac{\pi}{2}, \pi\right)\).

Solution:

We have, f(x)=\(\log \cos x\) \(f^{\prime}(x)=\frac{1}{\cos x}(-\sin x)=-\tan x\)

In interval \(\left(0, \frac{\pi}{2}\right), \tan x>0 \Rightarrow-\tan x<0\)

⇒ \(\mathrm{f}^{\prime}(\mathrm{x})<0 on \left(0, \frac{\pi}{2}\right) \mathrm{f}\) is strictly decreasing on \(\left(0, \frac{\pi}{2}\right)\).

In interval \(\left(\frac{\pi}{2}, \pi\right), \tan x<0 \Rightarrow-\tan x>0\)

⇒ \(f^{\prime}(x)>0 on \left(\frac{\pi}{2}, \pi\right)\) f is strictly increasing on \(\left(\frac{\pi}{2}, \pi\right)\)

Question 18. Prove that the function given by \(f(x)=x^3-3 x^2+3 x-100\) is increasing in R.

Solution:

We have, f(x)=\(x^3-3 x^2+3 x-100\)

⇒ \(f^{\prime}(x)=3 x^2-6 x+3-3\left(x^2-2 x+1\right)-3(x-1)^2\)

For any x \(\in R,(x-1)^2 \geq 0\).

3\((x-1)^2 \geq 0 \Rightarrow f^{\prime}(x) \geq 0 \forall x \in R\)

Hence, the given function f(x) is increasing in \(\mathbf{R}\).

Question 19. The interval in which \(y=x^2 e^{-3}\) is increasing is

  1. \((-\infty, \infty)\)
  2. (-2,0)
  3. \((2, \infty)\)
  4. (0,2)

Solution: 4. (0,2)

We have, y=\(x^2 e^{-7}\)

⇒ \(\frac{d y}{d x} =2 x e^{-x}-x^2 e^{-5}=x e^{-x}(2-x)\)

=-\(x e^{-x}(x-2)=0 \Rightarrow x=0,2 (\mathrm{e}^{-3}\) is always positive)

For increasing function, we have :

⇒ \(-x e^{-x}(x-2) \geq 0\)

⇒ \(x^{-4}(x-2) \leq 0\)

Hence, the required interval is (0,2).

Application Of Derivatives Exercise – 6.3

Question 1: Find the maximum and minimum values, if any, of the following functions given by:

  1. f(x)=\((2 x-1)^2+3\)
  2. f(x)=\(9 x^4+12 x+2\)
  3. f(x)=\(-(x-1)^2+10\)
  4. g(x)=\(x^{\prime}+1\)

Solution:

1. The given function is \(f(x)=(2 x-1)^2+3\).

It can be observed that \((2 x-1)^2 \geq 0\) for every x \(\in\) R.

Therefore, \(f(x)=(2 x-1)^2+3 \geq 3 \)for every x \(\in\) R.

Minimum value of f(x)=3

Minimum value of f(x) = 3

and function f(x) does not have a maximum value.

2. The given function is f(x)=\(9 x^2+12 x+2=(3 x+2)^2-2\).

It can be observed that \((3 \mathrm{x}+2)^2 \geq 0\) for every \(\mathrm{x} \in \mathbf{R}\).

Therefore, f(x)=\((3 x+2)^3-2 \geq-2\) for every \(x \in R\).

Minimum value of f(x)=-2

and function f does not have a maximum value.

3. The given function is f(x)=-\((x-1)^3+10\).

It can be observed that \((x-1)^2 \geq 0\) for every x \(\in\) R.

–\((x-1)^2 \leq 0\) for every x \(\in R\)

Therefore, f(x)=\(-(x-1)^2+10 \leq 10\) for every x \(\in \mathbf{R}\).

Maximum value of f(x)=10

and function f(x) does not have a minimum value.

4. The given function is \(g(x)=x^{\prime}+1\). and

At x \(\rightarrow \infty \quad g(x) \rightarrow \infty\)

At \(\mathrm{x} \rightarrow-\infty \quad \mathrm{g}(\mathrm{x}) \rightarrow-\infty\)

Hence, functioning (x neither has a maximum value nor a minimum value.

Question 2. Find the maximum and minimum values, if any, of the following functions given by 0

  1. f(x)=|x+2|-1
  2. g(x)=-|x+1|+3
  3. \(\mathrm{h}(\mathrm{x})=\sin (2 \mathrm{x})+5\)
  4. f(x)=\(|\sin 4 x+3|\)
  5. \(h(x)=x+1, x \in(-1,1)\)
  6. f(x)=|x+2|-1

Solution:

1. We know that |x+2| \(\geq 0\) for every x \(\in\) R.

Therefore, f(x)=|x+2|-1 \(\geq-1\) for every x \(\in\) R.

Minimum value of f(x)=-1

and function f(x) does not have a maximum value.

2. \(g(x)=x^3-3 x\)

⇒ \(g^{\prime}(x)=3 x^2-3\) and \(g^7(x)-6 x\)

Now for maxima or minima \(g^{\prime}(x)=0\)

⇒ \(3 x^2-3=0 \Rightarrow x- \pm 1 \)when x-1

when x=\(-1 g^{\prime \prime}(1)=6>0 \)

⇒  \(g'(-1)=-6<0\)

By the second derivative test, x=1 is a point of local minima and local minimum value of \(\mathrm{g}(\mathrm{x})\) at x=1 is \(g(1)=1^3-3=1-3=-2\).

However, x=-1 is a point of local maxima, and the local maximum value of g(x) at x=-1 is

⇒  \(g(-1)-(-1)^3-3(-1)–1+3=2\)

3. \(\mathrm{h}(\mathrm{x})-\sin \mathrm{x}+\cos \mathrm{x}, 0<\mathrm{x}<\frac{\pi}{2}\)

⇒  \(h^{\prime}(x)=\cos x-\sin x\)

for maxima or minima, \(h^{\prime}(x)=0 \Rightarrow \sin x=\cos x \Rightarrow \tan x=1\)

x=\(\frac{\pi}{4} \in\left(0, \frac{\pi}{2}\right)\)

⇒  \(h^{\prime \prime}(x)=-\sin x-\cos x=-(\sin x+\cos x)\)

⇒  \(h^{\prime
\prime}\left(\frac{\pi}{4}\right)=-\left(\frac{1}{\sqrt{2}}+\frac{1}{\sqrt{2}}\right)=-\frac{2}{\sqrt{2}}=-\sqrt{2}<0\)

Therefore, by the second derivative test, x=\(\frac{\pi}{4}\) is a point of local maxima and the local maximum value of h(x) at x=\(\frac{\pi}{4} is h\left(\frac{\pi}{4}\right)\)

=\(\sin \frac{\pi}{4}+\cos \frac{\pi}{4}=\frac{1}{\sqrt{2}}+\frac{1}{\sqrt{2}}=\sqrt{2}\)

4. \(f(x)=\sin x-\cos x, 0<x<2 \pi\)

⇒  \(f^{\prime}(x)=\cos x+\sin x\)

for maxima or minima

⇒  \(f^{\prime}(x)=0 \Rightarrow \cos x=-\sin x=\tan x=-1 \Rightarrow x=\frac{3 \pi}{4}, \frac{7 \pi}{4} \in(0,2 \pi)\)

⇒  \(f^{\prime \prime}(x)=-\sin x+\cos x\)

⇒  \(f^{\prime \prime}\left(\frac{3 \pi}{4}\right)=-\sin \frac{3 \pi}{4}+\cos \frac{3 \pi}{4}=-\frac{1}{\sqrt{2}}-  \frac{1}{\sqrt{2}}=-\sqrt{2}<0\)

⇒ \(f^{\prime \prime}\left(\frac{7 \pi}{4}\right)=-\sin \frac{7 \pi}{4}+\cos \frac{7 \pi}{4}=\frac{1}{\sqrt{2}}+\frac{1}{\sqrt{2}}=\sqrt{2}>0\)

Therefore, by second derivative test, x=\(\frac{3 \pi}{4}\) is a point of local maxima and the local maximum value of f(x) at \(x=\frac{3 \pi}{4} is f\left(\frac{3 \pi}{4}\right)\)

=\(\sin \frac{3 \pi}{4}-\cos \frac{3 \pi}{4}=\frac{1}{\sqrt{2}}+\frac{1}{\sqrt{2}}=\sqrt{2}\).

However, x=\(\frac{7 \pi}{4}\) is a point of local minima and the local minimum value of f(x) at x=\(\frac{7 \pi}{4} is f\left(\frac{7 \pi}{4}\right)\)

= \(\sin \frac{7 x}{4}-\cos \frac{7 \pi}{4}=-\frac{1}{\sqrt{2}}-\frac{1}{\sqrt{2}}=-\sqrt{2}\)

5. f(x)=\(x^3-6 x^2+9 x+15\) and \(f^{\prime}(x)=3 x^2-12 x+9\)

for maxima or minima

⇒  \(f^{\prime}(x)=0 \Rightarrow 3\left(x^2-4 x+3\right)=0 \Rightarrow 3(x-1)(x-3)-0 \Rightarrow x=1,3\)

Now,\(f^{\prime}(x)=6 x-12=6(x-2)\)

when x=1, \(f^{\prime}(1)=6(1-2)=-6<0\)

when x=3, \(f^{\prime}(3)=6(3-2)\)=6>0

Therefore, by the second derivative test, x =1 is a point of local maxima and the local maximum value of f(x) at x=1 is f(1)=1-6+9+15=19.

However, x=3 is a point of local minima, and the local minimum value of f(x) at x=3 is f(3)=27-54+27+15=15

6. \(g(x)=\frac{x}{2}+\frac{2}{x}, x>0\) and \(g^{\prime}(x)=\frac{1}{2}-\frac{2}{x^2}\)

for maxima or minimal

⇒  \(g^{\prime}(x)=0 \text { gives } \frac{2}{x^2}=\frac{1}{2} \Rightarrow x^2=4 \Rightarrow x= \pm 2\)

Since x>0, we take x=2.

Now, \(g^{\prime \prime}(x)=\frac{4}{x^3} \Rightarrow g^{\prime \prime}(2)=\cdot \frac{4}{2^{\prime}}=\frac{1}{2}>0\)

Therefore, by the second derivative test, x=2 is a point of local minima and the local minimum value of g(x) at x-2 is \(g(2)-\frac{2}{2}+\frac{2}{2}=1+1=2\)

7. g(x)=\(\frac{1}{x^2+2}\) and \(g^{\prime}(x)=\frac{-(2 x)}{\left(x^2+2\right)^2}\)

for maxima or minima, \(g^{\prime}(x)=0 \Rightarrow \frac{-2 x}{\left(x^2+2\right)^2}=0 \Rightarrow\) x=0

Now, for values close to x=0 and to the left of 0, \(g^{\prime}(x)>0\).

Also, for values close to x=0 and to the right of 0, \(\mathrm{~g}^{\prime}(\mathrm{x})<0\).

Therefore, by the first derivative test, x=0 is a point of local maximum and the local maximum value of g(0) is \(\frac{1}{0+2}=\frac{1}{2}\)

8. \(f(\mathrm{x})=\mathrm{x} \sqrt{1-\mathrm{x}}, 0<\mathrm{x}<1\)

⇒  \(f^{\prime}(x)=\sqrt{1-x}+x \cdot \frac{1}{2 \sqrt{1-x}}(-1)=\sqrt{1-x}-\frac{x}{2 \sqrt{1-x}}=\frac{2(1-x)-x}{2 \sqrt{1-x}}=\frac{2-3 x}{2 \sqrt{1-x}}\)

for maxima or minima

⇒  \(f^{\prime}(\mathrm{x})=0 \Rightarrow \frac{2-3 \mathrm{x}}{2 \sqrt{1-x}}=0 \Rightarrow 2-3 \mathrm{x}=0 \Rightarrow \mathrm{x}\)

=\(\frac{2}{3} \Rightarrow \mathrm{f}^{\prime}(\mathrm{x})=\frac{2-3 \mathrm{x}}{2 \sqrt{1-\mathrm{x}}}\)

⇒  \(f^{\prime \prime}(x)=\frac{1}{2}\left[\frac{\sqrt{1-x}(-3)-(2-3 x)\left(\frac{-1}{2 \sqrt{1-x}}\right)}{1-x}\right]\)

=\(\frac{\sqrt{1-x}(-3)+(2-3 x)\left(\frac{1}{2 \sqrt{1-x}}\right)}{2(1-x)} \)

=\(\frac{-6(1-x)+(2-3 x)}{4(1-x)^{\frac{2}{2}}}-\frac{3 x-4}{4(1-x)^{\frac{2}{2}}}\)

⇒  \(\mathrm{f} *\left(\frac{2}{3}\right)=\frac{3\left(\frac{2}{3}\right)^{-4}}{4\left(1-\frac{2}{3}\right)^{\frac{2}{3}}}\)

=\(\frac{2-4}{4\left(\frac{1}{3}\right)^3}=\frac{-1}{2\left(\frac{1}{3}\right)^{\frac{1}{3}}}<0 \)

Therefore, by the second derivative test, x=\(\frac{2}{3}\) is a point of local maxima and the local maximum value of f(x) at x=\(\frac{2}{3}\) is \( f\left(\frac{2}{3}\right)\)

=\(\frac{2}{3} \sqrt{1-\frac{2}{3}}=\frac{2}{3} \sqrt{\frac{1}{3}}=\frac{2}{3 \sqrt{3}}=\frac{2 \sqrt{3}}{9}\)

Question 4. Prove that the following functions do not have maxima or minima:

  1. \(f(x)=e^x\)
  2. \(g(x)=\log x\)
  3. \(h(x)=x^2+x^2+x+1\)

Solution:

1. We have, f(x)=\(e^”\)

⇒  \(f^{\prime}(x)=e^x\)

Now, if \(\mathrm{f}^{\prime}(\mathrm{x})\)=0, then \(\mathrm{e}^{\prime}\)=0, But, the exponential function can never assume 0 for any value of x.

Therefore, there does not exist x \(\in\) R such that \(f^{\prime}(x)\)=0

Hence, function f does not have maxima or minima.

2. We have, \(\mathrm{g}(\mathrm{x})=\log \mathrm{x}\)

⇒  \(g^{\prime}(\mathrm{x})=\frac{1}{\mathrm{x}}\)

Since log x is defined for a positive number x, \(\mathrm{g}^{\prime}(\mathrm{x})>0\) for any x.

Therefore, there does not exist x \(\in R\) such that \(g^{\prime}(x)\)=0.

Hence, function g does not have maxima or minima.

3. We have, \(\mathrm{h}(\mathrm{x})=\mathrm{x}^2+\mathrm{x}^2+\mathrm{x}+1\)

⇒  \(h^{\prime}(\mathrm{x})-3 \mathrm{x}^2+2 \mathrm{x}+1\)

Now, \(h^{\prime}(x)=0 \Rightarrow 3 x^2+2 x+1=0 \Rightarrow x=\frac{-2 \pm 2 \sqrt{2} i}{6}=\frac{-1 \pm \sqrt{2} i}{3} \notin R\)

Therefore, there does not exist x \(\in\) R such that \(h^{\prime}(x)\)=0.

Hence, function b does not have maxima or minima.

Question 5. Find the absolute maximum value and the absolute minimum value of the following functions in the given intervals:

  1. f(x)=\(x^3, x \in[-2,2]\)
  2. f(x)=\(\sin x+\cos x, x \in[0, \pi]\)
  3. \(f(\mathrm{x})=4 \mathrm{x}-\frac{1}{2} \mathrm{x}^2, \mathrm{x} \in\left[-2, \frac{9}{2}\right]\)
  4. f(x)=\((x-1)^2+3, x \in[-3,1]\)

Solution:

1. The given function is f(x)=x.

⇒  \(\mathrm{f}(\mathrm{x})=3 \mathrm{x}^2\)

Now, for maxima or minima

Put \(f^{\prime}(x)=0 \Rightarrow x=0\)

Then, we evaluate the value of f(x) at critical point x=0 and at endpoints of the interval [-2,2].

At x=0, f(0)=0

At x=-2, f(-2)=\((-2)^3\)=-8

At x=2, f(2)=\((2)^3\)=8

Hence, we can conclude that the absolute maximum value of f(x) on [-2,2] is 8 occurring at x = 2.

Also, the absolute minimum value of f(x) on [-2,2] is -8 occurring at x=-2.

2. The given function is f(x)=\(\sin x+\cos x\).

⇒  \(f^{\prime}(x)=\cos x-\sin x\)

Now, for maxima or minima \(f^{\prime}(x)=0 \Rightarrow \sin x=\cos x \Rightarrow \tan x=1 \Rightarrow x=\frac{\pi}{4}\)

Then, we evaluate the value of f(x) at critical point x=\(\frac{\pi}{4}\) and at the end points of the interval \([0, \pi]\).

⇒  \(f\left(\frac{\pi}{4}\right)=\sin \frac{\pi}{4}+\cos \frac{\pi}{4}=\frac{1}{\sqrt{2}}+\frac{1}{\sqrt{2}}=\frac{2}{\sqrt{2}}=\sqrt{2}\)

f(0)=\(\sin 0+\cos\) 0=0+1=1

f\((\pi)=\sin \pi+\cos \pi\)=0-1=-1

Hence, we can conclude that the absolute maximum value of f(x) on \([0, \pi]\) is \(\sqrt{2}\) occurring at x=\(\frac{\pi}{4}\) and the absolute minimum value of f(x) on \(\{0, \pi\}\) is -1 occurring at x=\(\pi\).

The given function is f(x)=\(4 x-\frac{1}{2} x^2\) and \(f^{\prime}(x)=4-\frac{1}{2}(2 x)=4-x\)

Now, for maxima or minima \(\mathrm{f}^{\prime}(\mathrm{x})=0 \Rightarrow \mathrm{x}-4\)

Then, we evaluate the value of f(x) at critical point x=4 and at the endpoints of the interval \(\left[-2, \frac{9}{2}\right]\).

⇒  \(f(4)=16-\frac{1}{2}(16)\)=16-8=8

⇒  \(f(-2)=-8-\frac{1}{2}(4)\)=-8-2=-10

⇒  \(f\left(\frac{9}{2}\right)=4\left(\frac{9}{2}\right)-\frac{1}{2}\left(\frac{9}{2}\right)^2=18-\frac{81}{8}=18-10.125=7.875\)

Hence, we can conclude that the absolute maximum value of f(x) on \(\left[-2, \frac{9}{2}\right]\) is 8 occurring at x=4 and the absolute minimum value of f(x) on \(\left[-2, \frac{9}{2}\right]\) is -10 occurring at x=-2.

4. The given function is f(x)=\((x-1)^2+3\) and \(f^{\prime}(x)=2(x-1)\)

Now, for maxima or minima

⇒  \(f^{\prime}(x)=0 \Rightarrow 2(x-1)=0 \Rightarrow x=1\)

Then, we evaluate the value of f(x) at critical point x=1 and at the endpoints of the interval [-3,1].

⇒ \(f(1)=(1-1)^2+3=0+3=3 \)

f(-3)=(-3-1)^2+3=16+3=19

Hence, we can conclude that the absolute maximum value of f(x) on [-3,1] is 19 occurring at x=-3, and the minimum value of f(x) on [-3,1] is 3 occurring at x=1.

Question 6. Find the maximum profit that a company can make, if the profit function is given by p(x)=41-72 x-18\( x^2\)

Solution:

The profit function is, p(x)=\(41-72 x-18 x^2\)

⇒  \(\Rightarrow p^{\prime \prime}(x)\)=-72-36 x and \(p^{\prime \prime}(x)\)=-36

for maxima or minima

⇒  \(p^{\prime}(x)\)=0

⇒ \(-72-36 x=0 \Rightarrow x=\frac{-72}{36} \Rightarrow x=-2 \)

⇒  \(p^{\prime \prime}(-2)=-36<0\)

Then, by the second derivative test, x=-2 is the point of local maxima of p(x).

So, the local maximum value is \(p(-2)=41-72(-2)-18(-2)^2=41+144-72=113\)

Therefore, the maximum profit that the company can make is 113 units.

Question 7. Find both the maximum value and minimum value of 3 \(x^{\prime}-8 x^{\prime}+12 x^{\prime}-48 x+25\) on the interval [0.3].

Solution:

Let f(x)=\(3 x^4-8 x^3+12 x^2-48 x+25\) on [0.3]

⇒  \(f^{\prime}(x)\)=12 x^3-24 x^2+24 x-48

for maxima or minima \(\mathrm{f}^{\prime}(\mathrm{x})\)=0

12 \(\mathrm{x}^2-24 \mathrm{x}^2+24 \mathrm{x}-48=0 \)

⇒  \(\mathrm{x}^3-2 \mathrm{x}^2+2 \mathrm{x}-4=0 \Rightarrow(\mathrm{x}-2)\left(\mathrm{x}^2+2\right)=0\)

x =2 \(\left(\mathrm{x}^2+2 \neq 0\right)\)

x=2 is the turning point.

Now we evaluate the value of f(x) at x=2 and at the endpoints of the interval [0,3]

f(2)=3(16)-8(8)+12(4)-48(2)+25=-39

f(0)=25

f(3)=3(81)-8(27)+12(9)-48(3)+25-16

Therefore. the absolute minimum value is -39 at x=2 and the absolute maximum value is 25 at x=0.

Question 8. At what points in the interval [0.2 \(\pi\)]? does the function sin 2 x attain its maximum value?

Solution:

Let f(x)=sin 2 x.

⇒  \(f^{\prime}(x)=2 \cos 2 x\)

Now, for maxima or minima

⇒  \(\mathrm{f}^{\prime}(\mathrm{x})=0 \Rightarrow \cos 2 \mathrm{x}\)=0

2 \(\mathrm{x}=\frac{\pi}{2} \cdot \frac{3 \pi}{2}, \frac{5 \pi}{2} \cdot \frac{7 \pi}{2}\)

⇒  \(\mathrm{x}=\frac{\pi}{4}, \frac{3 \pi}{4}, \frac{5 \pi}{4}, \frac{7 \pi}{4}\)

Then. we evaluate the values of f(x) at critical points x=\(\frac{\pi}{4}, \frac{3 \pi}{4}, \frac{5 \pi}{4}, \frac{7 \pi}{4}\), and at the end points of the interval [0.2\( \pi\)].

⇒  \(f\left(\frac{\pi}{4}\right)=\sin \frac{\pi}{2}\)=1,

⇒  \(f\left(\frac{3 \pi}{4}\right)=\sin \frac{3 \pi}{2}\)=-1,

⇒  \(f\left(\frac{5 \pi}{4}\right)=\sin \frac{5 \pi}{2}\)=1,

⇒  \(f\left(\frac{7 \pi}{4}\right)=\sin \frac{7 \pi}{2}\)=-1,

⇒  \(f(0)=\sin 0=0, f(2 \pi)=\sin 2 \pi\)=0

Hence, we can conclude that the absolute maximum value of f(x) on [0,2 \(\pi\)] is 1 occurring at

x=\(\frac{\pi}{4}\) and x=\(\frac{5 \pi}{4}\)

Question 9. What is the maximum value of the function sin x+cos x?

Solution:

Let f(x)=\(\sin x+\cos x\).

⇒  \(f^{\prime}(x)=\cos x-\sin x\)

for maxima or minima

⇒  \(f^{\prime}(x)=0 \Rightarrow \sin x=\cos x \Rightarrow \tan x=1 \Rightarrow x=\frac{\pi}{4}, \frac{5 \pi}{4} \ldots, n \)

⇒  \(f^{\prime \prime}(x)=-\sin x-\cos x=-(\sin x+\cos x)\)

Now, \(\mathrm{f}^{\prime}(\mathrm{x})\) will be negative when \((\sin \mathrm{x}+\cos \mathrm{x})\) is positive i.e., when sin x and cos x are both positive.

Also, we know that sin x and cos x both are positive in the first quadrant. Then, \(\mathrm{P}^{\prime}(x)\) will be negative when x \(\in\left(0, \frac{\pi}{2}\right)\)

Thus, we consider x =\(\frac{\pi}{4}\)

⇒  \(f^{\prime \prime}\left(\frac{\pi}{4}\right)=-\left(\sin \frac{\pi}{4}+\cos \frac{\pi}{4}\right)\)

=-\(\left(\frac{2}{\sqrt{2}}\right)=-\sqrt{2}<0\)

By second derivative test, \(\mathrm{f}(\mathrm{x})\) will be the maximum at x =\frac{\pi}{4} and the maximum value of \(\mathrm{f}(\mathrm{x})\) is

f\(\left(\frac{\pi}{4}\right)=\sin \frac{\pi}{4}+\cos \frac{\pi}{4}\)

=\(\frac{1}{\sqrt{2}}+\frac{1}{\sqrt{2}}=\frac{2}{\sqrt{2}}=\sqrt{2}\)

Question 10. Find the maximum value of \(2 x^3-24 x+107\) in the interval [1,3]. Find the maximum value of the same function in [-3,-1].

Solution:

Let \(f(x)-2 x^3-24 x+107\).

⇒ \(f^{\prime}(x)=6 x^2-24=6\left(x^2-4\right)\)

Now, for maxima or minima

⇒  \(f^{\prime}(x)=0 \Rightarrow 6\left(x^2-4\right)=0 \Rightarrow x^2=4 \Rightarrow x= \pm 2\)

We first consider the interval [1,3].

Then, we evaluate the value of f(x) at the critical point x-2 \(\in\)[1,3] and at the endpoints of the interval [1,3].

f(1)=2(1)-24(1)+107=2-24+107=85

f(2)=2(8)-24(2)+107=16-48+107=75

f(3)=2(27)-24(3)+107=54-72+107=89

Hence the absolute maximum value of f(x) in the interval [1,3] is 89 occurring at x=3.

Next, we consider the interval [-3,-1]

Now, we evaluate the value of f(x) at the critical point x = -2 \(\in\) [-3, -1] and at the endpoints of the interval [-3, -1 ].

f(-1) = 2(-1)- 24 (-1)+ 107 = -2 + 24 + 107= 129

f(~2) = 2(-8)- 24 (-2)+ 107 = -16 + 48 + 107 = 139

f(-3) = 2 (-27)- 24(-3) + 1 07 = -54 + 72 + 1 07 = 125

Hence, the absolute maximum value of (x) in the interval [-3, -1] is 139 occurring at x= -2.

Question 11. It is given that at x=1, the function \(x^4-62 x^2+a x+9\) attains its maximum value, on the interval [0,2].

Solution:

Find the value of a.

Let \(f(x)=x^4-62 x^2+a x+9\).

⇒  \(f^{\prime}(x)=4 x^2-124 x+a\)

It is given that function f attains its maximum value on the interval [0,2] at x=1.

⇒  \(\mathrm{f}^{\prime}(1)=0\)

4-124+a=0 \(\Rightarrow\) a=120. Hence, the value of a is 120

Question 12. Find the maximum and minimum values of x+sin 2 x on [0,2 \(\pi\)].

Solution:

Let f(x)=x+\(\sin 2 x and f^{\prime}(x)=1+2 \cos 2 x\)

Now, for maxima or minima

⇒  \(f(\mathrm{x})=0 \Rightarrow \cos 2 \mathrm{x}=-\frac{1}{2}=-\cos \frac{\pi}{3}=\cos \left(\pi-\frac{\pi}{3}\right)=\cos \frac{2 \pi}{3}\)

2\( \mathrm{x}=2 \pi \pm \frac{2 \pi}{3}, \mathrm{n} \in \mathrm{Z} \)

⇒ \(\mathrm{x}=\mathrm{n} \pi \pm \frac{\pi}{3}, \mathrm{n} \in \mathrm{Z} \Rightarrow \mathrm{x}=\frac{\pi}{3}, \frac{2 \pi}{3}, \frac{4 \pi}{3}, \frac{5 \pi}{3} \in[0,2 \pi]\)

Then, we evaluate the value of f(x) at critical points x=\(\frac{\pi}{3}, \frac{2 \pi}{3}, \frac{4 \pi}{3}\), \(\frac{5 \pi}{3}\) and at the endpoints of the interval [0,2 \(\pi\)].

⇒ \(f\left(\frac{\pi}{3}\right)=\frac{\pi}{3}+\sin \frac{2 \pi}{3}=\frac{\pi}{3}+\frac{\sqrt{3}}{2}\) ;

f\(\left(\frac{2 \pi}{3}\right)=\frac{2 \pi}{3}+\sin \frac{4 \pi}{3}=\frac{2 \pi}{3}-\frac{\sqrt{3}}{2}\)

⇒ \(f\left(\frac{4 \pi}{3}\right)=\frac{4 \pi}{3}+\sin \frac{8 \pi}{3}=\frac{4 \pi}{3}+\frac{\sqrt{3}}{2}\) ;

⇒ \(f\left(\frac{5 \pi}{3}\right)=\frac{5 \pi}{3}+\sin \frac{10 \pi}{3}=\frac{5 \pi}{3}-\frac{\sqrt{3}}{2}\)

f(0)=0+sin 0=0

⇒ \(f(2 \pi)=2 \pi+\sin 4 \pi=2 \pi+0=2 \pi+0=2 \pi\)

Hence, we can conclude that the absolute maximum value of f(x) in the interval [0,2 \(\pi\)] is 2 \(\pi\) occurring at x=2 \(\pi\) and the absolute minimum value of f(x) in the interval \([0,2 \pi]\) is \(\theta\) occurring at x=0.

Question 13. Find two numbers whose sum is 24 and whose product is as large as possible.

Solution:

Let the two numbers be x and y.

According to the question, x+y=24

y=24-x

And let z is the product of x and y.

⇒ \(\mathrm{z}=x \mathrm{y}\)

⇒ \(\mathrm{z}=\mathrm{x}(24-\mathrm{x})\)

⇒ \(\mathrm{z}=24 \mathrm{x}-\mathrm{x}^2 \Rightarrow \frac{\mathrm{dz}}{\mathrm{dx}}=24-2 \mathrm{x}\) and \(\frac{\mathrm{d}^2 \mathrm{z}}{\mathrm{dx}^2}=-2\) [From equation (1)]

Now to find turning point, \(\frac{\mathrm{dz}}{\mathrm{dx}}\)=0

24-2 x=0 \(\Rightarrow\) x=12

At x=12, \(\frac{d^2 z}{d^2}\)=-2<0

x =12 is a point of local maxima and z is maximum at x=12.

From equation (1), y=24-12=12

Therefore, the two required numbers are 12 and 12.

Question 14. Find two positive numbers x and y such that x+y=60 and \(x y^3\) is maximum.

Solution:

The two numbers are x and y such that x+y=60.

y=60-x

Let f(x)=\(x y^3\).

⇒ \(\mathrm{f}(\mathrm{x})=\mathrm{x}(60-\mathrm{x})^2\)

⇒ \(\mathrm{f}^{\prime}(\mathrm{x})=(60-\mathrm{x})^3-3 \mathrm{x}(60-\mathrm{x})^2-(60-\mathrm{x})^2[60-\mathrm{x}-3 \mathrm{x}]=(60-\mathrm{x})^2(60-4 \mathrm{x})\)

And \(f^{\prime \prime}(x)=-2(60-x)(60-4 x)-4(60-x)^2=-2(60-x)[60-4 x+2(60-x)]\)

=-2(60-x)(180-6 x)

=-12(60-x)(30-x)

for maxima or minima

Now \(\mathrm{f}^{\prime}(\mathrm{x})=0=\mathrm{x}=60 or \mathrm{x}=15\)

When x=\(60 f^{\prime}(x)\)=0

When x=15. \(f^{\prime \prime}(x)=-12(60-15)(30-15)=-12 \times 45 \times 15<0=-8100<0\)

By second order derivative test, x=15 is a point of local maxima of f(x).

Thus, function \(x y^3\) is maximum when x=15 and y=60-15=45.

Hence, the required numbers are 15 and 45.

Question 15. Find two positive numbers x and y such that their sum is 35 and the product \(x^2 y^3\) is a maximum.

Solution:

Let one number be x. Then, the other number is y =(35-x).

Let P(x)=\(x^2 y^2\), Then, we have

P(x)=\(x^2(35-x)^5\)

⇒ \(P^{\prime}(x) =2 x(35-x)^3-5 x^2(35-x)^4\)

=\(x(35-x)^4[2(35-x)-5 x]=x(35-x)^4(70-7 x)-7 x(35-x)^4(10-x)\)

And, \(\mathrm{P}^{\prime \prime}(\mathrm{x})\)

=7\((35-\mathrm{x})^4(10-\mathrm{x})+7 \mathrm{x}\left[-(35-\mathrm{x})^4-4(35-\mathrm{x})^3(10-\mathrm{x})\right]\)

=\(7(35-\mathrm{x})^4(10-\mathrm{x})-7 \mathrm{x}(35-\mathrm{x})^4-28 \mathrm{x}(35-\mathrm{x})^3(10-\mathrm{x})\)

⇒ \(-7(35-\mathrm{x})^3[(35-\mathrm{x})(10-\mathrm{x})-\mathrm{x}(35-\mathrm{x})-4 \mathrm{x}(10-\mathrm{x})]\)

=\(7(35-\mathrm{x})^3[(350-45 \mathrm{x}+\mathrm{x}^2-35 \mathrm{x}+\mathrm{x}^2-40 \mathrm{x}+4 \mathrm{x}^2]\).

=7\((35-\mathrm{x})^3\left(6 \mathrm{x}^2-120 \mathrm{x}+350\right)\)

for maxima or minima

Now, \(\mathrm{P}^{\prime}(\mathrm{x})=0 \Rightarrow \mathrm{x}-0,35,10\)

When x=35 or x=0. This will make the product \(x^3 y^3 \)equal to 0 .

x=0 and y =35 cannot be the possible values of x.

When x=10, we have

⇒ \(\mathrm{P}^{-}(\mathrm{x})=7(35-10)^3(6 \times 100-120 \times 10+350)=7(25)^3(-250)<0\)

By second order derivative test, P(x) will be the maximum when x=10 and y=35-10=25. Hence, the required numbers are 10 and 25.

Question 17. A square piece of tin side 18 cm is to be made into a box without a top, by cutting a square from each corner and folding up the flaps to form the box. What should be the side of the square to be cut off so that the volume of the box is the maximum possible?

Solution:

Let the side of the square to be cut off be x cm. Then, the length and the breadth of the box will be (1 8- 2x) cm each and the height of the box is x cm. Therefore, the volume V(x) of the box is given by

V(x) =x\((18-2 x)^2\)

⇒ \(V^{\prime}(x) =(18-2 x)^2-4 x(18-2 x)=(18-2 x)[18-2 x-4 x]\)

=(18-2 x)(18-6 x)=12(9-x)(3-x)

And \(V^{\prime \prime}(x) =12[-(9-x)-(3-x)]=-12(9-x+3-x)\)

Applications Of Derivatives A Square Piece Of Tin Is Made Into A Box

=-12(12-2 x)=-24(6-x)

for maxima or minima

⇒ \(\mathrm{V}^{\prime}(\mathrm{x})=0 \Rightarrow \mathrm{x}=9 or x=3\)

If x=9, then the length and the breadth will become 0.

x \(\neq 9 \Rightarrow\) x=3

Now, \(\mathrm{V}^*(3)=-24(6-3)=-72<0\)

By second order derivative test, x == 3 is the point of maxima of V,

Hence, if we remove a square of side 3 cm from each corner of the square tin and make a box.

Question 18. A rectangular sheet of tin 45 cm by 24 cm is to be made into a box without a top, by cutting off a square from each corner and folding up the flaps. What should be the side of the square to be cut off so that the volume of the box is the maximum possible?

Solution:

Let the side of the square to be cut off be x cm. Then, the height of the box is x, the length is 45- 2x, and the breadth is 24- 2x.

Therefore, the volume V(x) of the box is given by

⇒ \(V(x)=x(45-2 x)(24-2 x)=x\left(1080-90 x-48 x+4 x^2\right)=4 x^3-138 x^2+1080 x\)

⇒ \(V^{\prime \prime}(x)=12 x^2-276 x+1080-12\left(x^3-23 x+90\right)=12(x-18)(x-5)\)

⇒ \(V^{\prime \prime}(x)=24 x-276=12(2 x-23)\)

for maxima or minima

V’ (x) = 0 ⇒ x = 18 and x- 5

Applications Of Derivatives A Rectangle Sheet Of Tin Is Made Into A Box Without Top

It is not possible to cut off a square of side 18 cm from each corner of the rectangular sheet.

Thus, x cannot be equal to 18.

x \(\neq\) 18

x = 5

Now, V” (5) = 12 (10- 23) = 1 2 (- 1 3) = – 156 < 0

By second order derivative test, x = 5 is the point of maxima.

Question 19. Show that of all the rectangles inscribed in a given fixed circle, the square has the maximum area.

Solution:

Let a rectangle of length l and breadth b be inscribed in the given circle of radius a.

Then, the diagonal passes through the center and is of length 2 cm.

Now. by applying the Pythagoras theorem, we have:

⇒ \((2 a)^2=\ell^2+b^2\)

⇒ \(b^2=4 a^2-c^2 \Rightarrow b=\sqrt{4 a^2-\ell^2}\)

Applications Of Derivatives The Rectangle Inscribed In A Given Fixed Circle

Area of the rectangle, \(\Lambda=\ell \sqrt{4 \mathrm{a}^2-\ell^2} \Rightarrow A^2=\ell^2\left(4 \mathrm{a}^2-\ell^2\right)=\mathrm{B}(\text { let })\)

If A is the maximum B is also maximum

⇒ \(\frac{d B}{d \ell}=8 \mathrm{a}^2 \ell-4 \ell^3 \Rightarrow \frac{d^2 B}{d \ell^2}=8 a^2-12 \ell^2\)

for maxima or minima \(\frac{\mathrm{dB}}{\mathrm{d} \ell}=0 \Rightarrow 8 \mathrm{a}^2 \ell-4 \ell^3=0 \Rightarrow \ell=0\)

or \(\ell=\sqrt{2} \mathrm{a}\)

b=\(\sqrt{4 a^2-2 a^2}=\sqrt{2 a^2}=\sqrt{2 a}\)

Now, when \(\ell=\sqrt{2} \mathrm{a}\)

⇒ \(\frac{d^2 B}{d \ell^2}=8 a^2-12\left(2 a^2\right)=-16 a^2<0\)

By the second order derivative test, when \(\ell=\sqrt{2}\) a, then the area of the rectangle is the maximum.

Since \(\ell=b=\sqrt{2} \mathrm{a}\), the rectangle is a square.

Hence, it has been proved that of all the rectangles inscribed in the given fixed circle, the square has the maximum area.

Question 20. Show that the right circular cylinder of a given surface and maximum volume is such that its height is equal to the diameter of the base.

Solution:

Let r and h be the radius and height of the cylinder respectively.

Then, the surface area (S’) of the cylinder is given by,

⇒ \(\mathrm{S}=2 \pi \mathrm{r}^2+2 \pi \mathrm{h} \Rightarrow \mathrm{h}=\frac{\mathrm{S}-2 \pi \mathrm{r}^2}{2 \pi \mathrm{r}} \Rightarrow \mathrm{h}=\frac{\mathrm{S}}{2 \pi \mathrm{r}}-\mathrm{r}\) →  Equation 1

Let V be the volume of the cylinder, Then,

Applications Of Derivatives The Right Circular Cylinder Of A Given Surface

⇒ \(\mathrm{V}=\pi \mathrm{r}^2 \mathrm{~h}=\pi \mathrm{r}^2\left[\frac{\mathrm{S}}{2 \pi}-\mathrm{r}\right]=\frac{\mathrm{Sr}}{2}-\pi \mathrm{r}^3\)

Then, \(\frac{\mathrm{dV}}{\mathrm{dr}}=\frac{\mathrm{S}}{2}-3 \pi \mathrm{r}^2, \frac{\mathrm{d}^2 \mathrm{~V}}{\mathrm{dt}}=-6 \pi \mathrm{rr}\)

for maxima or minima

⇒ \(\frac{d V}{d r}=0 \Rightarrow \frac{S}{2}=3 \pi r^2 \)

⇒ \(r^2=\frac{S}{6 \pi}\)

When \(r^2=\frac{S}{6 \pi}, then \frac{d^2 V}{d r^2}=-6 \pi\left(\sqrt{\frac{S}{6 \pi}}\right)<0\)

By second order derivative test, the volume is the maximum when \(r^2=\frac{S}{6 \pi}\)

Now, when \(r^2=\frac{S}{6 \pi}, or S=6 \pi r^2\) then h=\(\frac{6 \pi r^2}{2 \pi r}-r=3 r-r=2 r\)

Hence, the volume is maximum when the height is twice the radius ie., when the height is equal to the diameter.

Question 21. Of all the closed cylindrical cans (right circular), of a given volume of 100 cubic centimeters. find the dimensions of the can which has the minimum surface area.

Solution:

Let r and h be the radius and height of the cylinder respectively, then the volume (V) of the cylinder is given by,

V =\(\pi r^2 \mathrm{~h}=100\) (given)

h =\(\frac{100}{\pi r^2}\)

The surface area (S) of the cylinder is given by, S =\(2 \pi \mathrm{r}^2+2 \pi \mathrm{h}=2 \pi \mathrm{r}^2+\frac{200}{\mathrm{r}}\)

⇒ \(\frac{\mathrm{dS}}{\mathrm{dr}}=4 \pi \mathrm{r}-\frac{200}{\mathrm{r}^2}, \frac{\mathrm{d}^2 \mathrm{~S}}{\mathrm{dr}^2}=4 \pi+\frac{400}{\mathrm{r}^3}\)

for maxima or minima

⇒ \(\frac{d \mathrm{~S}}{\mathrm{dr}} =0 \Rightarrow 4 \pi r=\frac{200}{\mathrm{r}^2}\)

⇒ \(\mathrm{r}^3=\frac{200}{4 \pi}=\frac{50}{\pi}\)

⇒ \(r^2=\frac{50}{\pi}\)

r=\(\left(\frac{50}{\pi}\right)^{\frac{1}{3}}\)

Now, it is observed that when \(\mathrm{r}=\left(\frac{50}{\pi}\right)^{\frac{1}{3}} \cdot \frac{\mathrm{d}^2 \mathrm{~S}}{\mathrm{dt}^2}>0\)

By second order derivative test, the surface area is the minimum when the radius of the cylinder is

⇒ \(\left(\frac{50}{\pi}\right)^{\frac{1}{3}} \mathrm{~cm}\)

Now, \(\frac{d^2 S}{d r^2}=\frac{400}{r^{\prime}}+4 \pi=8 \pi+4 \pi=12 \pi>0\)

When r=\(\left(\frac{50}{\pi}\right)^{\frac{1}{2}}, \mathrm{~h}=\frac{100}{\pi\left(\frac{50}{\pi}\right)^{\frac{1}{3}}}=\frac{2 \times 50}{(50)^2\left(\frac{1}{\pi}\right)^{\frac{2}{3}} \pi}=2\left(\frac{50}{\pi}\right)^{\frac{1}{3}} \mathrm{~cm}\).

Hence, the required dimensions of the can which has the minimum surface area is given by radius =\(\left(\frac{50}{\pi}\right)^{\frac{1}{3}} \mathrm{~cm}\)

and height =\(2\left(\frac{50}{\pi}\right)^{\frac{3}{3}} \mathrm{~cm}\).

Question 22. A wire of length 28 m is to be cut into two pieces. One of the pieces is to be made into a square and the other into a circle. What should be the length of the two pieces so that the combined area of the square and the circle is minimal?

Solution:

Let a piece of length l be cut from the given wire to make a square.

Then, the other piece of wire to be made into a circle is of length (28- i) m

Now, side of square =\(\frac{t}{4}\).

Let r be the radius of the circle. Then, 2 \(\pi r=28-\ell \Rightarrow r=\frac{1}{2 \pi}(28-\theta)\)

The combined areas of the square and the circle (A) are given by, A= (side of the square) \(^2+\pi \mathrm{se}^{\prime}\)

=\(\frac{\ell^2}{16}+\pi\left[\frac{1}{2 \pi}(28-\ell)\right]^2=\frac{\ell^2}{16}+\frac{1}{4 \pi}(28-)^2 \)

⇒ \(\frac{\mathrm{dA}}{\mathrm{d} \ell}=\frac{2 \ell}{16}+\frac{2}{4 \pi}(28-\ell)(-1)=\frac{\ell}{8}-\frac{1}{2 \pi}(28-\ell)\)

⇒ \(\frac{\mathrm{dA}}{\mathrm{d} \ell}=\frac{\ell}{8}-\frac{1}{2 \pi}(28-\ell) \Rightarrow \frac{\mathrm{d}^2 \mathrm{~A}}{\mathrm{~d} \ell^2}=\frac{1}{8}+\frac{1}{2 \pi}\)

for maxima or minima, \(\frac{\mathrm{dA}}{\mathrm{d} \ell}=0 \Rightarrow \frac{\ell}{8}-\frac{1}{2 \pi}(28-\ell)\)=0

⇒ \(\frac{\pi i-4(28-\theta)}{8 \pi}=0 \Rightarrow(\pi+4(-112=0 \Rightarrow t=\frac{112}{\pi+4}\).

Thus, when \(\ell=\frac{112}{\pi+4}, \frac{d^2 \mathrm{~A}}{\mathrm{~d} \ell^2}>0\)

By second order derivative test, the area (A) is the minimum when \(\ell=\frac{112}{\pi+4}\).

Hence, the combined area is minimum when the length of the wire in making the square is \(\frac{112}{\pi+4}\) m while the length of the wire in making the circle is \(28-\frac{112}{\pi+4}=\frac{28 \pi}{\pi+4} \mathrm{~m}\)

Question 23. Prove that the volume of the largest cone that can be inscribed in a sphere of radius R is \(\frac{8}{27}\) of the volume of the sphere.

Solution:

Let r and h be the radius and height of the cone respectively inscribed in a sphere of radius R.

Let V be the volume of the cone.

Then, V=\(\frac{1}{3} \pi r^2 h\)

The height of the cone is given by,

Applications Of Derivatives The Volume Of Largest Cone That Can Be Inscribed In A Sphere

⇒ \(\mathrm{h}-\mathrm{R}+\mathrm{AB}=\mathrm{R}+\sqrt{\mathrm{R}^2-\mathrm{r}^2}\)[ABC is a right triangle]

⇒ \(\mathrm{r}^2=2 \mathrm{hR}-\mathrm{h}^2\)

⇒ \(\mathrm{V}=\frac{1}{3} \pi \mathrm{h}\left(2 \mathrm{hR}-\mathrm{h}^2\right) \Rightarrow \mathrm{V}=\frac{1}{3} \pi\left(2 \mathrm{~h}^2 \mathrm{R}-\mathrm{h}^3\right)\)

⇒ \(\frac{\mathrm{dV}}{\mathrm{dh}}=\frac{1}{3} \pi\left(4 \mathrm{hR}-3 \mathrm{~h}^2\right) and \frac{\mathrm{d}^2 \mathrm{~V}}{\mathrm{dh}^2}=\frac{1}{3} \pi(4 \mathrm{R}-6 \mathrm{~h})\)

for maxima or minima

⇒ \(\frac{\mathrm{dV}}{\mathrm{dh}}=0 \Rightarrow 4 \mathrm{hR}-3 \mathrm{~h}^2=0 \Rightarrow \mathrm{h}=0 or \mathrm{h}=\frac{4 \mathrm{R}}{3}\)

rejecting h =0 , h=\(\frac{4 \mathrm{R}}{3}\)

When h=\(\frac{4 \mathrm{R}}{3}, then \frac{\mathrm{d}^2 \mathrm{~V}}{\mathrm{dh}^2}=\frac{1}{3} \pi(4 \mathrm{R}-8 \mathrm{R})=-\frac{4 \pi \mathrm{R}}{3}<0\)

from (1) \(r^2=2 h R-h^2\)

when h=\(\frac{4 \mathrm{R}}{3}, \mathrm{r}^2=\frac{8 \mathrm{R}^2}{9}\)

By second order derivative test, the volume of the cone is the maximum when \(r^2=\frac{8}{9} R^2\).

Therefore, the volume of a cone

⇒ \(\mathrm{V}=\frac{1}{3} \pi\left(\frac{8}{9} \mathrm{R}^2\right)\left(\frac{4}{3} \mathrm{R}\right)\)

=\(\frac{8}{27}\left(\frac{4}{3} \pi \mathrm{R}^3\right)=\frac{8}{27} \times\) (Volume of the sphere)

Hence, the volume of the largest cone that can be inscribed in the sphere is \(\frac{8}{27}\) the volume of the sphere.

Question 24. The point on the curve \(x^2=2\) y which is nearest to the point (0,5) is?

  1. \((2 \sqrt{2}, 4)\)
  2. \((2 \sqrt{2}, 0)\)
  3. (0,0)
  4. (2,2)

Solution: 1. \((2 \sqrt{2}, 4)\)

Given curve is \(x^2=2 y\)

Let the point (x, y) on the curve nearest to the point (0,5).

Now distance (D) between (x, y) and (0,5) is

D=\(\sqrt{x^2+(y-5)^2} \Rightarrow D=\sqrt{2 y+(y-5)^2}\)

⇒ \(D^2=2 y+y^2+25-10 y \Rightarrow D^2=y^2-8 y+25\)

Let \(\mathrm{D}^{\mathrm{t}}=\mathrm{z}\)

Then \(z=y^3-8 y+25\)

⇒ \(x^{\prime}=2 y-8\) and \(z^{\prime \prime}=2\)

for maxima or minima

Now, \(z^{\prime}=0 \Rightarrow y=4\)

At \(y=4, x^n=2>0\)

i.e. y=4 is the point of minima.

At y=4, x= \(\pm 2 \sqrt{2}\)

Hence, the required point is ( \(\pm 2 \sqrt{2}, 4\)).

The correct answer is 1.

Question 25. For all real values of x, the minimum value of \(\frac{1-x+x^2}{1+x+x^3}\) is?

  1. 0
  2. 1
  3. 3
  4. \(\frac{1}{3}\)

Solution: 4.

Let f(x)=\(\frac{1-x+x^2}{1+x+x^2}\)

⇒ \(f^{\prime}(x)=\frac{\left(1+x+x^2\right)(-1+2 x)-\left(1-x+x^2\right)(1+2 x)}{\left(1+x+x^2\right)^2}\)

=\(\frac{-1+2 x-x+2 x^2-x^2+2 x^3-1-2 x+x+2 x^2-x^2-2 x^3}{\left(1+x+x^2\right)^2}\)

=\(\frac{2 x^2-2}{\left(1+x+x^2\right)^2}=\frac{2\left(x^2-1\right)}{\left(1+x+x^2\right)^2}\)

for maxima or minima

f(x)=0 \(\Rightarrow x^2=1 \Rightarrow x= \pm 1\)

Now,f(x)=\(\frac{2\left[\left(1+x+x^2\right)^2(2 x)-\left(x^2-1\right)(2)\left(1+x+x^2\right)(1+2 x)\right]}{\left(1+x+x^2\right)^4}\)

=\(\frac{4\left(1+x+x^2\right)\left[\left(1+x+x^2\right) x-\left(x^2-1\right)(1+2 x)\right]}{\left(1+x+x^2\right)^4}\)

=\(\frac{4\left[x+x^2+x^3-x^3-2 x^3+1+2 x\right]}{\left(1+x+x^2\right)}=\frac{4\left[1+3 x-x^3\right]}{\left(1+x+x^2\right)}\)

And, \(f(1)=\frac{4[1+3-1]}{(1+1+1)^3}=\frac{4(3)}{(3 y}=\frac{4}{9}>0\)

Also, \(f^*(-1)=\frac{4[1-3+1]}{(1-1+1)^2}=4(-1)=-4<0\)

By second order derivative test, f is minimum at x=1 and the minimum value is given by \(f(1)=\frac{1-1+1}{1+1+1}=\frac{1}{3}\),

The correct answer is D.

Question 26. The maximum value of \([x(x-1)+1], 0 \leq x \leq 1\) is ?

  1. \(\left(\frac{1}{3}\right)^{\frac{1}{1}}\)
  2. \(\frac{1}{2}\)
  3. 1
  4. 0

Solution: 3. 1

Let f(x)=\([x(x-1)+1]^{\frac{1}{2}}\)

⇒ \((\mathrm{x})=\frac{2 \mathrm{x}-1}{3[\mathrm{x}(\mathrm{x}-1)+1]^{\frac{2}{3}}}\)

for maxima or minima

f(x)=0 \(\Rightarrow x=\frac{1}{2}\)

Then, we evaluate the value of f at critical point x=\(\frac{1}{2}\) and at the endpoints of the interval [0,1] {i.e., at x=0 and x=1}.

f(0)=\([0(0-1)+1]^{\frac{1}{j}}=1\)

f(1)=\([1(1-1)+1]^{\frac{1}{5}}=1\)

⇒ \(f\left(\frac{1}{2}\right)=\left[\frac{1}{2}\left(-\frac{1}{2}\right)+1\right]^{\frac{1}{3}}=\left(\frac{3}{4}\right)^{\frac{1}{3}}\)

Hence, we can conclude that the maximum value of f in the interval [0,1] is 1.

The correct answer is 3.

Application Of Derivatives Miscellaneous Exercise

Question 1. Show that the function given by f(x)=\(\frac{\log x}{x}\) has maximum at x=e.

Solution:

The given function is f(x)=\(\frac{\log x}{x}\),

⇒ \(f^{\prime}(x)=\frac{x\left(\frac{1}{x}\right)-\log x}{x^2}=\frac{1-\log x}{x^2}\)

for maxima or minima \(f^{\prime}(x)=0 \Rightarrow \frac{1-\log x}{x^2}=0 \Rightarrow \log x=1 \Rightarrow \log x=\log e \Rightarrow x=c\)

Now, \(f”(x)=\frac{x^2\left(-\frac{1}{x}\right)-(1-\log x)(2 x)}{x^4}=\frac{-x-2 x(1-\log x)}{x^4}\)

= \(\frac{-3+2 \log x}{x^3} at x=e, f^{\prime}(e)=\frac{-3+2 \log e}{e^3}=\frac{-3+2}{e^3}=\frac{-1}{e^3}<0\)

Therefore, by second order derivative test, f(x) is maximum at x=e.

Question 2. The two equal sides of an isosceles triangle with fixed base b are decreasing at the rate of 3 cm per second. How fast is the area decreasing when the two equal sides are equal to the base ?

Solution:

Let \(\triangle\) A B C be isosceles where BC is the base of fixed length b.

Let the length of the two equal sides of \triangle \(\mathrm{ABC}\) be a.

Draw AD \(\perp\) BC

Applications Of Derivatives Two Equal Sides Of An Isosceles Triangle With Fixed Base

Now, in \(\triangle\) A D C, by applying the Pythagoras theorem,

we have: A D=\(\sqrt{a^2-\frac{b^2}{4}}\)

Area of triangle A B C is A=\(\frac{1}{2} B C \cdot A D=\frac{b}{2} \sqrt{a^2-\frac{b^3}{4}}\)

The rate of change of the area with respect to time (t) is given by,

⇒ \(\frac{d A}{d t}=\frac{1}{2} b \cdot \frac{2 a}{2 \sqrt{a^2-\frac{b^2}{4}}} \frac{d a}{d t}=\frac{a b}{\sqrt{4 a^2-b^2}} \frac{d a}{d t}\)

It is given that the two equal sides of the triangle are decreasing at the rate of 3 cm per second.

⇒ \(\frac{\mathrm{da}}{\mathrm{dt}}=-3 \mathrm{~cm} / \mathrm{s}\)

⇒ \(\frac{\mathrm{dA}}{\mathrm{dt}}=\frac{-3 \mathrm{ab}}{\sqrt{4 \mathrm{a}^2-\mathrm{b}^2}}\)

Then, when a=b, we have: \(\frac{d A}{d t}=\frac{-3 b^2}{\sqrt{4 b^2-b^2}}=\frac{-3 b^2}{\sqrt{3 b^2}}=-\sqrt{3} \mathrm{~cm}^2 / \mathrm{s}\)

Hence, if the two equal sides are equal to the base, then the area of the triangle is decreasing at the rate of \(\sqrt{3} \mathrm{~b} \mathrm{~cm}^2 / \mathrm{s}\).

Question 3. Find the intervals in which the function f is given by \(f(\mathrm{x})=\frac{4 \sin \mathrm{x}-2 \mathrm{x}-\mathrm{x} \cos \mathrm{x}}{2+\cos \mathrm{x}}\) is

  1. increasing
  2. decreasing

Solution:

f(x) =\(\frac{4 \sin x-2 x-x \cos x}{2+\cos x}\)

⇒ \(f^{\prime}(x)=\frac{(2+\cos x)(4 \cos x-2-\cos x+x \sin x)-(4 \sin x-2 x-x \cos x)(-\sin x)}{(2+\cos x)^2}\)

=\(\frac{(2+\cos x)(3 \cos x-2+x \sin x)+\sin x(4 \sin x-2 x-x \cos x)}{(2+\cos x)^2}\)

=\(\frac{6 \cos x-4+2 x \sin x+3 \cos ^2 x-2 \cos x+x \sin x \cos x+4 \sin ^2 x-2 x \sin x-x \sin x \cos x}{(2+\cos x)^2}\)

=\(\frac{4 \cos x-4+3 \cos ^2 x+4 \sin ^2 x}{(2+\cos x)^2}=\frac{4 \cos x-4+3 \cos ^2 x+4-4 \cos ^2 x}{\left(2+\cos ^2 x\right)^2-}\)

=\(\frac{4 \cos x-\cos ^2 x}{(2+\cos x)^2}=\frac{\cos x(4-\cos x)}{(2+\cos x)^2}\)

Now, x=\(\frac{\pi}{2}\) and x=\(\frac{3 \pi}{2}\) divides [0,2 \(\pi\)] into three intervals i.e. \(\left[0, \frac{\pi}{2}\right],\left[\frac{\pi}{2}, \frac{3 \pi}{2}\right]\) and \(\left[\frac{3 \pi}{2}, 2 \pi\right]\).

In intervals \(\left[0, \frac{\pi}{2}\right]\) and \(\left[\frac{3 \pi}{2}, 2 \pi\right], \mathrm{f}^{\prime}(\mathrm{x})>0\)

Thus, f(x) is increasing for \(\left[0, \frac{\pi}{2}\right]\) and \(\left[\frac{3 \pi}{2}, 2 \pi\right]\)

In the interval \(\left[\frac{\pi}{2}, \frac{3 \pi}{2}\right], \mathrm{f}^{\prime}(\mathrm{x}) \leq 0\).

Thus, \(\mathrm{f}(\mathrm{x})\) is decreasing for \(\left[\frac{\pi}{2}, \frac{3 \pi}{2}\right]\).

Question 4. Find the intervals in which the function f given by \(f(\mathrm{x})=\mathrm{x}^3+\frac{1}{\mathrm{x}^3}, \mathrm{x} \neq\) 0 is

  1. increasing
  2. decreasing

Solution:

⇒ \(f(\mathrm{x})=\mathrm{x}^3+\frac{1}{\mathrm{x}^3}\)

⇒ \(f^{\prime}(\mathrm{x})=3 \mathrm{x}^2-\frac{3}{\mathrm{x}^4}=\frac{3 \mathrm{x}^6-3}{\mathrm{x}^4}\)

Then, \(f^{\prime}(x)=0 \Rightarrow 3 x^6-3=0 \Rightarrow x^6=1 \Rightarrow x= \pm 1\)

Now, the points x=1 and x=-1 divide the real line into three intervals i.e., \((-\infty,-1],[-1,1]\) and \([1, \infty)\).

In intervals \((-\infty,-1] and [1, \infty) f^{\prime}(x) \geq 0\)

⇒ f(x) is increasing in \((-\infty,-1] \cup[1, \infty)\)

In interval [-1,1] \(\mathrm{f}^{\prime}(\mathrm{x})<0\).

f(x) is decreasing in [-1,1]

Question 5. Find the maximum area of an isosceles triangle inscribed in the ellipse \(\frac{x^2}{a^2}+\frac{y^2}{b^2}\)=1 with its vertex at one end of the major axis.

Solution:

The given ellipse is \(\frac{x^2}{a^2}+\frac{y^2}{b^2}\)=1, Its paranctric form is x=a \(\cos \theta, y=b \sin \theta\)

Let the major axis be along the x-axis.

Applications Of Derivatives The Maximum Arear Of An Isosceles Triangle

Let ABC be the triangle inscribed in the ellipse where vertex C is at (a, 0)

Since the ellipse is symmetrical with respect to the x-axis and y-axis, we can assume the coordinates of A to be \((-\mathrm{a} \cos \theta, \mathrm{b} \sin \theta)\) and the coordinates of B to be \((-\mathrm{a} \cos \theta,-\mathrm{b} \sin \theta )\)

⇒ \(\mathrm{AB}=2 \mathrm{~b} \sin \theta\) and \(\mathrm{DC}=(\mathrm{a}+\mathrm{a} \cos \theta)\)

the area of triangle ABC is given by,

A=\(\frac{1}{2} A B \cdot D C \Rightarrow A=\frac{1}{2}(2 b \sin \theta) \cdot(a+a \cos \theta)\)

A=a b \(\sin \theta(1+\cos \theta)\)

⇒ \(\mathrm{AB}=2 \mathrm{bsin} \theta\) and \(\mathrm{DC}=(\mathrm{a}+\mathrm{a} \cos \theta)\)

the area of triangle A B C is given by,

A=\(\frac{1}{2} A B \cdot D C \Rightarrow A=\frac{1}{2}(2 b \sin \theta) \cdot(a+a \cos \theta)\)

A=a b \(\sin \theta(1+\cos \theta)\)

⇒ \(\frac{d A}{d \theta}=a b[\sin \theta(-\sin \theta)+(1+\cos \theta) \cos \theta] \Rightarrow \frac{d A}{d \theta}\)

=a b\(\left[\cos \theta+\cos ^2 \theta-\sin ^2 \theta\right]\)

and \(\frac{d^2 A}{d \theta^2}=a b[-\sin \theta-2 \sin \theta \cos \theta-2 \sin \theta \cos \theta] \Rightarrow \frac{d^2 A}{d \theta^2}=-a b[\sin \theta+4 \sin \theta \cos \theta]\)

for maxima or minima \(\frac{\mathrm{dA}}{\mathrm{d} \theta}\)=0

a b\(\left[\cos \theta+\cos ^2 \theta-\sin ^2 \theta\right]=0 \Rightarrow \cos \theta+\cos ^2 \theta-1+\cos ^2 \theta=0 \)

⇒ \(2 \cos ^2 \theta+\cos \theta-1=0 \Rightarrow \cos \theta=-1 or \cos \theta=\frac{1}{2}\)

⇒ ⇒ \(\theta=\pi or \theta=\frac{\pi}{3}\)

rejecting \(\theta=\pi\)

\(\theta=\frac{\pi}{3}\)

Now at \(\theta=\frac{\pi}{3}, \frac{d^2 A}{d \theta^2}=-a b\left[\sin \frac{\pi}{3}+4 \sin \frac{\pi}{3} \cos \frac{\pi}{3}\right]\)

= \(-a b\left[\frac{\sqrt{3}}{2}+4 \cdot \frac{\sqrt{3}}{2} \cdot \frac{1}{2}\right]<0\)

By the second derivative test, the Area (A) of an isosceles triangle is maximum when \(\theta=\frac{\pi}{3}\)

The maximum area of the triangle is given by,

⇒ \(\left.A=a b \sin \theta(1+\cos \theta)=a b \sin \frac{\pi}{3}\left(1+\cos \frac{\pi}{3}\right)-a b \cdot \frac{\sqrt{3}}{2} / 1+\frac{1}{2}\right)\)

=\(\frac{3 \sqrt{3}}{4} a b \cdot s q \cdot\) units

Question 6. A tank with a rectangular base and rectangular sides, open at the top is to be constructed so that its deÿtdis lava volume is 8 ark If the buildup of the tank costs Rs 70 per square meter for the base and Rs 45 per square meter for the sides, What is the cost of least expensive tank?

Solution:

Let l, b, and h represent the length, breadth, and height of the tank respectively.

Then, we have height (h) = 2 m

Volume of the tank. = 8m³ ; Volume of the tank =L x b x h

Now, the area of the base = lb = 4

Area of the 4 walls (A) = 2h(l + b)

A =4\(\left(\ell+\frac{4}{\ell}\right) \Rightarrow \frac{\mathrm{dA}}{\mathrm{d} \ell}=4\left(1-\frac{4}{\ell^2}\right)\)

for maxima or minima

⇒ \(\frac{\mathrm{dA}}{\mathrm{d} \ell}=0 \Rightarrow 1-\frac{4}{\ell^2}=0 \Rightarrow \ell^2=4 \Rightarrow \ell= \pm 2\)

However, the length cannot be negative. Therefore, we have \(\ell\)=2

b=\(\frac{4}{\ell}=\frac{4}{2}\)=2

Now, \(\frac{\mathrm{d}^2 \mathrm{~A}}{\mathrm{~d} \ell^2}=\frac{32}{\ell^3}\)

When \(\ell=2, \frac{\mathrm{d}^2 \mathrm{~A}}{\mathrm{~d} \ell^2}=\frac{32}{8}=4>0\).

Thus, by second order derivative test, the area is the minimum when \(\ell\)=2.

We have t=\(\mathrm{b}=\mathrm{h}\)=2.

Cost of building the base = Rs 70 \(\times(\ell b)\)= Rs 70(4)= Rs 280

Cost of building the walls = Rs 2 h\((\ell+\mathrm{b}) \times 45\)= Rs 90(2)(2+2)= Rs 720

Required total cost =Rs(280+720)= Rs 1000

Hence, the total cost of the tank will be Rs 1000.

Question 7. The sum of the perimeter of a circle and square is k, where k is some constant. Prove that the sum of their areas is least when the side of the square is double the radius of the circle.

Solution:

Let ‘r’ be the radius of the circle and ‘a’ be the side of the square.

Then, we have :

2 \(\pi r+4 a=k\) (where k is constant)

a=\(\frac{\mathrm{k}-2 \pi \mathrm{r}}{4}\)

The sum of the areas of the circle and the square (A) is given by,

A =\(\pi \mathrm{r}^2+\mathrm{a}^2=\pi\)

⇒ \(\mathrm{r}^2+\frac{(\mathrm{k}-2 \pi \mathrm{r})^2}{16}\)

⇒ \(\frac{\mathrm{dA}}{\mathrm{dr}}=2 \pi \mathrm{r}+\frac{2(\mathrm{k}-2 \pi \mathrm{r})(-2 \pi)}{16}=2 \pi \mathrm{r}-\frac{\pi(\mathrm{k}-2 \pi \mathrm{r})}{4}\)

for maxima or minima

⇒ \(\frac{\mathrm{dA}}{\mathrm{dr}}\) =0

2 \(\pi \mathrm{r} =\frac{\pi(\mathrm{k}-2 \pi \mathrm{r})}{4}\)

8 \(\mathrm{r}=\mathrm{k}-2 \pi \mathrm{r} \Rightarrow(8+2 \pi) \mathrm{r}=\mathrm{k} \Rightarrow \mathrm{r}=\frac{\mathrm{k}}{8+2 \pi}=\frac{\mathrm{k}}{2(4+\pi)}\)

Now, \(\frac{\mathrm{d}^2 \mathrm{~A}}{\mathrm{dr}^2}=2 \pi+\frac{\pi^2}{2}\)

When r=\(\frac{k}{2(4+\pi)}, \frac{d^2 A}{d r^2}>0\).

The sum of the areas is least when r =\(\frac{\mathrm{k}}{2(4+\pi)}\).

When r=\(\frac{k}{2(4+\pi)}, a=\frac{k-2 \pi\left[\frac{k}{2(4+\pi)}\right]}{4}=\frac{k-\frac{\pi k}{(4+\pi)}}{4}=\frac{4 k+\pi k-\pi k}{4(4+\pi)}=\frac{k}{4+\pi}=2 r\)

Hence, it has been proved that the sum of their areas is least when the side of the square is Double the radius of the circle.

Question 8. A window is in the form of a rectangle surmounted by a semicircular opening. The total perimeter of the window is 10 m. Find the dimensions of the window to admit maximum light through the whole opening.

Solution:

Let 2x and y be the length and breadth of the rectangular window respectively.

The radius of the semicircular opening = x

It is given that the perimeter of the window is 10 m.

⇒ \(2 x+2 y+\pi x=10 \Rightarrow y=5-x-\frac{\pi x}{2}\)

Now, the Area of the window is given by.

Applications Of Derivatives A Window Is In The Form Of Rectangle Surmounted By A Semicircular Opening

A =2 \(\mathrm{xy}+\frac{\pi \mathrm{x}^2}{2} \Rightarrow \mathrm{A}=2 \mathrm{x}\left(5-\mathrm{x}-\frac{\pi \mathrm{x}}{2}\right)+\frac{\pi \mathrm{x}^2}{2}\)

⇒ \(\mathrm{A}=10 \mathrm{x}-2 \mathrm{x}^2-\pi \mathrm{x}^2+\frac{\pi \mathrm{x}^2}{2} \Rightarrow \mathrm{A}=10 \mathrm{x}-2 \mathrm{x}^2-\frac{\pi \mathrm{x}^2}{2}\)

⇒ \(\frac{\mathrm{dA}}{\mathrm{dx}}=10-4 \mathrm{x}-\pi \mathrm{x} \frac{\mathrm{d}^2 \mathrm{~A}} {\mathrm{dx}^2}=-4-\pi\)

for maxima or minima

⇒ \(\frac{\mathrm{dA}}{\mathrm{dx}}=0 \Rightarrow 10-4 \mathrm{x}-\pi \mathrm{x}=0 \Rightarrow \mathrm{x}=\frac{10}{\pi+4}\)

Thus, when x=\(\frac{10}{\pi+4}, then \frac{d^2 A}{d x^2}<0\).

Therefore, by second-order derivative test, the area is the maximum when length 2 x=\(\frac{20}{\pi+4} m \)

Now,y=5-\(\frac{10}{\pi+4}-\frac{\pi}{2}\left(\frac{10}{\pi+4}\right) \Rightarrow y=\frac{10}{\pi+4}\)

Hence, the required dimensions of the window to admit maximum light is given by length =\(\frac{20}{\pi+4}\) m and breadth =\(\frac{10}{\pi+4}\) m.

Question 9. A point on the hypotenuse of a triangle is at distance ‘ a ‘ and ‘ b ‘ from the sides of the triangle.
Show that the minimum length of the hypotenuse is \(\left(a^{\frac{2}{3}}+b^{\frac{2}{3}}\right)^{\frac{2}{2}}\)

Solution:

Let \(\triangle\) A B C be right-angled at B. Let A B=x and B C=y.

Let P be a point on the hypotenuse of the triangle such that P is at a distance of a and b from the sides AB and BC respectively.

Let \(\angle \mathrm{C}=\theta\).

We have, A C=\(\sqrt{x^2+y^2}\)

Now, P C=b \({cosec} \theta\) and, \(A P=a \sec \theta\)

AC=A P+P C

Applications Of Derivatives A Point On The Hypotenuse Of A Triangle Is At A Distance

AC=b {cosec} \(\theta+a \sec \theta\)

⇒ \(\frac{d(A-C)}{d \theta}=-b {cosec} \theta \cot \theta+a \sec \theta \tan \theta\)

⇒ \(\frac{d(A-C)}{d \theta}=-b {cosec} \theta \cot \theta+a \sec \theta \tan \theta\)

and \(\frac{d^2(A-C)}{d \theta^2}=b\left({cosec} \theta \cot ^2 \theta+{cosec}^3 \theta\right)+a\left(\sec ^3 \theta+\sec \theta \tan ^2 \theta\right)\)

for maxima or minima

⇒ \(\frac{\mathrm{d}(\mathrm{AC})}{\mathrm{d} \theta}=0 \Rightarrow \mathrm{asec} \theta \tan \theta=b {cosec} \theta \cot \theta\)

⇒ \(\frac{\mathrm{a}}{\cos \theta} \cdot \frac{\sin \theta}{\cos \theta}=\frac{\mathrm{b}}{\sin \theta} \frac{\cos \theta}{\sin \theta}\)

⇒ \({asin}^3 \theta=b \cos ^3 \theta \Rightarrow(\mathrm{a})^{\frac{1}{3}} \sin \theta=(b)^{\frac{1}{3}} \cos \theta \Rightarrow \tan \theta=\left(\frac{b}{a}\right)^{\frac{1}{3}}\)

⇒ \(\sin \theta =\frac{(b)^{\frac{1}{3}}}{\sqrt{a^{\frac{2}{3}}+b^{\frac{2}{3}}}} \text { and } \cos \theta=\frac{(\mathrm{a})^{\frac{1}{3}}}{\sqrt{a^{\frac{2}{3}}+b^{\frac{2}{3}}}}\)

Since, \(0<\theta<\frac{\pi}{2}\), so trigonometric ratios are positive.

Also, a>0 and b>0 \(\frac{d^2(A C)}{d \theta^2}\) is positive.

It can be clearly shown that \(\frac{\mathrm{d}^2(\mathrm{AC})}{\mathrm{d} \theta^2}>0\) when\(\tan \theta=\left(\frac{\mathrm{b}}{\mathrm{a}}\right)^{\frac{1}{3}}\)

Therefore, by second order derivative test, the length of the hypotenuse is minimum when

⇒ \(\tan \theta=\left(\frac{b}{a}\right)^{\frac{1}{3}}\) .

Now, when \(\tan \theta=\left(\frac{b}{a}\right)^{\frac{1}{3}}\),

we have : A C=\(\frac{b \sqrt{a^{\frac{2}{3}}+b^{\frac{2}{3}}}}{b^{\frac{1}{3}}}+\frac{a \sqrt{a^{\frac{2}{3}}+b^{\frac{2}{3}}}}{a^{\frac{1}{3}}}\)

=\(\sqrt{a^{\frac{2}{3}}+b^{\frac{2}{3}}\left(b^{\frac{2}{3}}+a^{\frac{2}{3}}\right)=\left(a^{\frac{2}{3}}+b^{\frac{2}{3}}\right)^{\frac{1}{2}}} [Using (1) and (2)]\)

Hence, the minimum length of the hypotenuse is \(\left(a^{\frac{2}{3}}+b^{\frac{2}{3}}\right)^{\frac{3}{2}}.\)

Question 10. Find the points at which the function f given by \(f(x)=(x-2)^4(x+1)^3\) has

  1. local maxima
  2. local minima
  3. point of inflection

Solution:

The given function is f(x)=\((x-2)^4(x+1)^3\).

⇒ \(f^{\prime}(x) =4(x-2)^3(x+1)^3+3(x+1)^2(x-2)^4\)

=\((x-2)^5(x+1)^2[4(x+1)+3(x-2)]=(x-2)^3(x+1)^2(7 x-2)\)

For maxima or minima

⇒ \(\mathrm{f}(\mathrm{x})=0 \Rightarrow(\mathrm{x}-2)^3(\mathrm{x}+1)^2(7 \mathrm{x}-2)\)=0

⇒ \(f^{\prime}(x)=0 \Rightarrow x=-1 and x=\frac{2}{7} or x=2\)

Now, for values of x close to \(\frac{2}{7}\) and to the left of \(\frac{2}{7}, f^{\prime}(x)>0\).

Also, for values of x close to \(\frac{2}{7}\) and to the right of \(\frac{2}{7}, \mathrm{f}^{\prime}(\mathrm{x})<0\).

Thus, x=\(\frac{2}{7}\) is the point of local maxima.

Now, for values of x close to 2 and to the left of 2, \(f^{\prime}(x)<0\). Also, for values of x close to 2 and to the right of 2, \(f^{\prime}(\mathrm{x})>0\)

Thus, x=2 is the point of local minima.

Now, as the value of x varies through -1, \(f^{\prime}(x)\) does not change its sign.

Thus, x=-1 is the point of inflection.

Question 11. Find the absolute maximum and minimum values of the function f given by

Solution:

f(x)=\(\cos ^2 x+\sin x, x \in[0, \pi]\)

f(x)=\(\cos ^2 x+\sin x\)

⇒ \(f^{\prime}(x)=2 \cos x(-\sin x)+\cos x=-2 \sin x \cos x+\cos x\)

Now, \(\mathrm{f}^{\prime}(\mathrm{x})\)=0

2 \(\sin x \cos x=\cos x\)

⇒ \(\cos x(2 \sin x-1)=0 \Rightarrow \sin x=\frac{1}{2}\)

or \(\cos x=0 \Rightarrow x=\frac{\pi}{6}\) or \(\frac{\pi}{2}\) as x \(\in[0, \pi]\)

Now, evaluating the value of f at critical points x=\(\frac{\pi}{2}\) and x=\(\frac{\pi}{6}\) and the endpoints of the interval [0, \(\pi\)] (i.e., at x=0 and \(x=\pi\) ), we have:

⇒ \(f\left(\frac{\pi}{6}\right)=\cos ^2 \frac{\pi}{6}+\sin \frac{\pi}{6}=\left(\frac{\sqrt{3}}{2}\right)^2+\frac{1}{2}=\frac{5}{4}\)

⇒ \(\mathrm{f}(0)=\cos ^2 0+\sin 0=1+0=1\)

⇒ \(\mathrm{f}(\pi)=\cos ^2 \pi+\sin \pi-(-1)^3+0=1\)

⇒ \(\mathrm{f}\left(\frac{\pi}{2}\right)=\cos ^2 \frac{\pi}{2}+\sin \frac{\pi}{2}=0+1=1\)

Hence, the absolute maximum value of f is \(\frac{5}{4} \)occurring at x =\(\frac{\pi}{6}\) and the absolute minimum value of f is 1 occurring at x=0, \(\frac{\pi}{2}\)/ and \(\pi\).

Question 12. Show that the altitude of the right circular cone of maximum volume that can be inscribed in a sphere or radius r is \(\frac{4 r}{3}\).

Solution:

A sphere of fixed radius (r) is given.

Let R and h be the radius and the height of the cone respectively,

Applications Of Derivatives The Altitude Of The Right Circular Cone Of Maximum Volume

The volume \((\mathrm{V})\) of the cone is given by,V =\(\frac{1}{3} \pi \mathrm{R}^2 \mathrm{~h}\)

In \(\triangle\) B C D, B C=\(\sqrt{r^2-R^2}\)

⇒ \(\mathrm{h}=\mathrm{r}+\sqrt{\mathrm{r}^2-\mathrm{R}^2} \Rightarrow \mathrm{R}^2=2 h r-\mathrm{h}^2\)

⇒ \(\mathrm{V}=\frac{1}{3} \pi h\left(2 h r-h^2\right)=\frac{1}{3} \pi\left(2 \mathrm{~h}^2 \mathrm{r}-\mathrm{h}^3\right)\)

⇒ \(\frac{\mathrm{dV}}{\mathrm{dh}}=\frac{1}{3} \pi\left(4 \mathrm{hr}-3 \mathrm{~h}^2\right) and \frac{\mathrm{d}^2 \mathrm{~V}}{\mathrm{dh}^2}=\frac{1}{3} \pi(4 \mathrm{r}-6 \mathrm{~h})\)

for maxima or minima

⇒ \(\frac{\mathrm{dV}}{\mathrm{dh}}=0 \Rightarrow 4 \mathrm{hr}-3 \mathrm{~h}^2=0 \Rightarrow \mathrm{h}=0 or \mathrm{h}=\frac{4 \mathrm{r}}{3}\)

rejecting h 0 \(\mathrm{h}=\frac{4 \mathrm{r}}{3}\)

when h =\(\frac{4 \mathrm{r}}{3}, \frac{\mathrm{d}^2 \mathrm{~V}}{\mathrm{dh}^2}=\frac{1}{3} \pi(4 \mathrm{r}-8 \mathrm{r})=-\frac{4 \mathrm{r} \pi}{3}<0\)

V is maximum when \(h=\frac{4 \mathrm{r}}{3}\)

By second order derivative test, the volume of the cone is maximum when the altitude of the cone is \(\frac{4 \mathrm{r}}{3}\)

Question 13. Let f be a function defined on [a, b] such that \(f^{\prime}(x)>0\), for all x \(\in\)(a, b), then prove that f is an increasing function on (a, b).

Solution:

Since, \(f^{\prime}(x)>0\) on (a, b)

Then, f is a differentiable function on (a, b)

Also, every differentiable function is continuous,

Therefore, f is continuous on [a, b]

Let \(x_1, x_2 \in(a, b)\) such that, \(x_2>x_1\) then by LMV theorem, there exists c \(\in(a, b)\) s.t.

⇒ \(f^{\prime}(c)=\frac{f\left(x_2\right)-f\left(x_1\right)}{x_2-x_1}\)

⇒ \(f\left(x_2\right)-f\left(x_1\right)=\left(x_2-x_1\right) f^{\prime}(c)\)

⇒ \(f\left(x_2\right)-f\left(x_1\right)>0 \text { as } x_2>x_1 \text { and } f^{\prime}(x)>0\)

⇒ \(f\left(x_1\right)>f\left(x_1\right)\)

for \(\mathrm{x}_1<\mathrm{x}_2 \Rightarrow \mathrm{f}\left(\mathrm{x}_1\right)<\mathrm{f}\left(\mathrm{x}_2\right)\).

Therefore, f is an increasing function.

Question 14. A cylindrical tank of radius 10 m is being filled with wheat at the rate of 314 cubic meters per hour. Then the depth of the wheat is increasing at the rate of?

  1. 1 \(\mathrm{~m} / \mathrm{h}\)
  2. 0.1 \(\mathrm{~m} / \mathrm{h}\)
  3. 1.1 \(\mathrm{~m} / \mathrm{h}\)
  4. 0.5 \(\mathrm{~m} / \mathrm{h}\)

Solution: 1. 1\(\mathrm{~m} / \mathrm{h}\)

Let r be the radius of the cylinder

Then, volume \((\mathrm{V})\) of the cylinder is given by =\(\pi r^t \mathrm{~h}\),

V =\(\pi(\text { radius })^2 \times \text { height }\)

⇒ \(\left.=\pi(10)^2 \mathrm{~h} \quad \quad \text { (radius }=10 \mathrm{~m}\right)\)

=100 \(\pi \mathrm{h}\)

Differentiating with respect to time t, we have :

⇒ \(\frac{\mathrm{dV}}{\mathrm{dt}}=100 \pi \frac{\mathrm{dh}}{\mathrm{dt}}\)

The tank is being filled with wheat at the rate of 314 cubic meters per hour.

⇒ \(\frac{\mathrm{dV}}{\mathrm{dt}}=314 \mathrm{~m}^3 / \mathrm{h}\)

Thus, we have :

314=100 \(\pi \frac{\mathrm{dh}}{\mathrm{dt}} \Rightarrow \frac{\mathrm{dh}}{\mathrm{dt}}=\frac{314}{100(3.14)}=\frac{314}{314}=1\)

Hence, the depth of wheat is increasing at the rate of 1 \(\mathrm{~m} / \mathrm{h}\).

The correct answer is 1.

 

Continuity and Differentiability Class 12 Maths Important Questions Chapter 5

Continuity And Differentiability Exercise 5.1

Question 1. Prove that the function f(x) = 5x-3 is continuous at x = 0, at x = -3 and x = 5.

Solution:

The given function is f(x) = 5x-3

At x= 0, f(0) =5×0-3=-3

⇒ \(\lim _{x \rightarrow 0} f(x)=\lim _{x \rightarrow 0}(5 x-3)=5 \times\) 0-3=-3

⇒ \(\lim _{x \rightarrow 0}\) f(x)=f(0)

Therefore, f is continuous at x =0

At x=-3, f(-3)=5 \(\times(-3)\)-3=-18

⇒ \(\lim _{x \rightarrow-3} f(x)=\lim _{x \rightarrow-3}(5 x-3)=5 \times(-3)-3=-18 \Rightarrow \lim _{x \rightarrow-3} f(x)^{-}=f(-3)\)

Therefore, f is continuous at x=-3

At x=5, f(x)=f(5)=5 \(\times\) 5-3=25-3=22

⇒ \(\lim _{x \rightarrow 5} f(x)=\lim _{x \rightarrow 5}(5 x-3)=5 \times 5-3=22 \Rightarrow \lim _{x \rightarrow 5} f(x)=f(5)\)

Therefore, is continuous at x = 5

Question 2. Examine the continuity of the function \(\mathrm{f}(\mathrm{x})=2 \mathrm{x}^2-1\) at x=3.

Solution:

The given function is \(\mathrm{f}(\mathrm{x})=2 \mathrm{x}^2-1\)

At x=3, f(x)=f(3)=2 \(\times 3^2-1=17\)

⇒ \(\lim _{x \rightarrow 3} f(x)=\lim _{x \rightarrow 3}\left(2 x^2-1\right)=2 \times 3^2-1=17 \lim _{x \rightarrow 3} f(x)=f(3)\)

Thus, f is continuous at x =3

Question 3. Examine the following functions for continuity.

f(x)=x-5
f(x)=\(\frac{1}{x-5}, x \neq 5\)
f(x)=\(\frac{x^2-25}{x+5}, x \neq-5\)
f(x)=|x-5|

Solution :

1. The given function is f(x) = x- 5

It is evident that f is defined at every real number k and its value at k is k – 5

It is also observed that, \(\lim _{x \rightarrow k} f(x)=\lim _{x \rightarrow k}(x-5)=k-5=f(k)\)

⇒ \(\lim _{x \rightarrow k} f(x)=f(k)\)

Hence, f is continuous at every real number and therefore, it is a continuous function

Read and Learn More Class 12 Maths Chapter Wise with Solutions

2. The given function is f(x)=\(\frac{1}{x-5}, x \neq 5\)

For any real number k \(\neq 5\), we obtain \(\lim _{x \rightarrow k} f(x)=\lim _{x \rightarrow k}\left(\frac{1}{x-5}\right)=\frac{1}{k-5}\)

Also, f(k)=\(\frac{1}{k-5}( At k \neq 5) \Rightarrow \lim _{x \rightarrow k} f(x)=f(k)\)

Hence,f is continuous at every point in the domain of f and therefore, it is a continuous function.

3. The given function is f(x)=\(\frac{x^2-25}{x+5}\), x \(\neq-5\)

For any real number c \(\neq-5\), we obtain

⇒ \(\lim _{x \rightarrow c} f(x)=\lim _{x \rightarrow c} \frac{x^2-25}{x+5}=\lim _{x \rightarrow c} \frac{(x+5)(x-5)}{x+5}=\lim _{x \rightarrow c}(x-5)=(c-5)\)

Also, f(c)=\(\frac{(c+5)(c-5)}{c+5}=(c-5)(as c \neq-5)\)

\(\lim _{x \rightarrow c} f(x)=f(c)\)

Hence, f is continuous at every point in the domain of f and therefore, it is a continuous function.

4. The given function is f(x)=|x-5|=\(\begin{cases}{l}5-x, \text { if } x<5 \\ x-5, \text { if } x \geq 5\end{cases}\).

This function f is defined at all points of the real line.

Let c be a point on a real line. Then, c < 5 or c =5 or c > 5

Case 1: c < 5

Then, f (c)=5 – c

⇒ \(\lim _{x \rightarrow c} f(x)=\lim _{x \rightarrow c}(5-x)=5-c \Rightarrow \lim _{x \rightarrow c} f(x)=f(c)\)

Therefore, f is continuous at all real numbers less than 5.

Case 2: c=5

Then, f(c)=f(5)=(5-5)=0

⇒ \(\lim _{x \rightarrow 5} f(x)=\lim _{x \rightarrow 5}(5-x)=(5-5)=0 \Rightarrow \lim _{x \rightarrow 5^{+}} f(x)=\lim _{x \rightarrow 5}\)(x-5)=0

⇒ \(\lim _{x \rightarrow c^{-}} f(x)=\lim _{x \rightarrow c^{+}} f(x)=f(c)\)

Therefore, f is continuous at x = 5

Case 3: c > 5

Then, f(c)=f(5)=c-5

⇒ \(\lim _{x \rightarrow c} f(x)=\lim _{x \rightarrow c}(x-5)=c-5 \Rightarrow \lim _{x \rightarrow c} f(x)=f(c)\)

Therefore, f is continuous at all real numbers greater than 5.

Hence, f is continuous at every real number and therefore, it is a continuous function.

CBSE Class 12 Maths Chapter 5 Continuity And Differentiability Important Question And Answers

Question 4. Prove that the function f {x} =\(\mathrm{x}^{\mathrm{n}}\) is continuous at x= n, where n is a positive integer.

Solution:

The given function is \(\mathrm{f}(\mathrm{x})=\mathrm{x}^{\mathrm{n}}\)

It is evident that f is defined at all positive integers, n, and its value at n is n.

Then, \(\lim _{x \rightarrow \pi} f(x)=\lim _{x \rightarrow n}\left(x^n\right)=n^n \Rightarrow \lim _{x \rightarrow \pi}\)

f(x)=f(n)

Therefore, f is continuous at n, where n is a positive integer.

Question 5. Is the function f defined by f(x)=\(\begin{cases}{l}x,\text { if } x \leq 1 \\ 5, \text { if } x>1\end{cases}\).continuous at x=0 ? At x=1? At x=2?

Solution:

The given function f(x)=\(\begin{cases}{l}x, \text { if } x \leq 1 \\ 5, \text { if } x>1\end{cases}\).

⇒ \(\mathrm{f}(\mathrm{x})\) may be discontinuous at doubtful point x=1 and except this point \(\mathrm{f}(\mathrm{x})\) be continuous.

Now, at x =1,

The left hand {limit} of f at x=1 is, \(\lim _{x \rightarrow 1^{-}} f(x)=\lim _{x \rightarrow 1^{-}}\) x=1

The right hand limit of f at x=1 is, \(\lim _{x \rightarrow 1^{+}} f(x)=\lim _{x \rightarrow 1^{+}}(5)\)=5

⇒ \(\lim _{x \rightarrow 1^{-}} f(x) \neq \lim _{x \rightarrow 1^{+}} f(x)\)

Hence, f is not continuous at x = 1

Question 6. Find all points of discontinuity of f, where f is defined by f(x)=\(\begin{cases}{l}2 x+3, \text { if } x \leq 2 \\ 2 x-3, \text { if } x>2\end{cases}\).

Solution:

The given function f(x)=\(\begin{cases}{l}2 x+3, \text { if } x \leq 2 \\ 2 x-3, \text { if } x>2\end{cases}\). may be discontinuous at doubtful point x=2 and except this point \(\mathrm{f}(\mathrm{x}) \)be continuous.

At x=2

The left hand limit of f at x=2 is, \(\lim _{x \rightarrow 2^{-}} f(x)=\lim _{x \rightarrow 2^{-}}(2 x+3)=2 \times\) 2+3=7

The right hand limit of f at x=2 is \(\lim _{x \rightarrow 2^{+}} f(x)=\lim _{x \rightarrow 2^{+}}(2 x-3)=2 \times\) 2-3=1

⇒ \(\lim _{x \rightarrow 2^{-}} f(x) \neq \lim _{x \rightarrow 2^{+}} f(x)\)

Therefore, f is not continuous at x=2. Hence, x=2 is the only point of discontinuity of f.

Question 7. Find all points of discontinuity of f, where f is defined by f(x)=\(\begin{cases}{l}|x|+3, \text { if } x \leq-3 \\ -2 x, \text { if }-3<x<3 \\ 6 x+2, \text { if } x \geq 3\end{cases}\).

Solution:

The given function f is f(x)=\(\begin{cases}{l}|x|+3=-x+3, \text { if } x \leq-3 \\ -2 x, \text { if }-3<x<3 \\ 6 x+2, \text { if } x \geq 3\end{cases}\).

Given fn. f(x) may be discontinuous at doubtful points x=-3 and x=3 and except these two points f(x) be continuous.

at x=-3, f(-3)=-(-3)+3=6

L.H.L =\(\lim _{x \rightarrow-3^{-}} f(x)=\lim _{x \rightarrow-3^{-}}(-x+3)=-(-3)+3=6\)

R.H.L =\(\lim _{x \rightarrow-3^{+}} f(x)=\lim _{x \rightarrow-3^{+}}(-2 x)=-2 \times(-3)\)=6

⇒ \(\lim _{x \rightarrow-3} f(x)=f(-3)\)

Therefore, f is continuous at x = -3

Now at x=3, the left hand limit of f at x=3 is, \(\lim _{x \rightarrow 3^{-}} f(x)=\lim _{x \rightarrow 3^{-}}(-2 x)=-2 \times 3\)=-6

The right hand limit of f at x=3 is, \(\lim _{x \rightarrow 3^{+}} f(x)=\lim _{x \rightarrow 3^{+}}(6 x+2)=(6 \times 3)+2=20\)

It is observed that the left and right-hand limits of f at x=3 do not coincide.

Therefore, f is not continuous at x=3. Hence, x=3 is the only point of discontinuity of f.

Question 8. Find all points of discontinuity of f, where f is defined by f(x)= \(\begin{cases}|x| & \text { if } x \neq 0 \\ 0, & \text { if } x=0\end{cases}\)

Solution:

The given function f is f(x)=\(\begin{cases}{l}|x| \text { if } x \neq 0 \\ 0, \text { if }x=0\end{cases}\).

It is known that, x<0 \(\Rightarrow|x|\)=-x and x>0 \(\Rightarrow|x|\)=x

Therefore, the given function can be rewritten as f(x)=\(\left\{\begin{array}{l}
\frac{|x|}{x}=\frac{-x}{x}=-1 \text { if } x<0 \\
0, \text { if } x=0 \\
\frac{|x|}{x}=\frac{x}{x}=1, \text { if } x>0
\end{array}\right.\).

Given f. f(x) may be discontinuous at doubtful point x=0 and except these point f(x) be continuous.

Now at x=0, then the left hand limit of f at x=0 is, \(\lim _{x \rightarrow 0^{-}} f(x)=\lim _{x \rightarrow 0^{-}}(-1)\)=-1

The right hand limit of f at x=0 is, \(\lim _{x \rightarrow 0^{+}} f(x)=\lim _{x \rightarrow 0^{+}}(1)\)=1

It is observed that the left and right-hand limit of f at x=0 do not coincide.

Therefore, f is not continuous at x=0

Hence, x=0 is the only point of discontinuity of f.

Question 9. Find all points of discontinuity of f, where f is defined by f(x)= \(\begin{cases}\frac{x}{|x|} & \text { if } x<0 \\ -1, & \text { if } x \geq 0\end{cases}\)

Solution:

The given function f is f(x)= \(\begin{cases}\frac{x}{|x|}, & \text { if } x<0 \\ -1, & \text { if } x \geq 0\end{cases}\) It is known that, x<0 \(\Rightarrow|x|\)=-x

⇒ \(\mathrm{f}(\mathrm{x})\)=-1 for all x \(\in \mathrm{R}\)

Let c be any real number. Then, \(\lim _{x \rightarrow c} f(x)=\lim _{x \rightarrow c}(-1)\)=-1

Also, f(c)=\(-1=\lim _{x \rightarrow c} f(x)\)

Therefore, the given function is a continuous function.

Hence, the given function has no point of discontinuity.

Question 10. Find all points of discontinuity of f, where f is defined by f(x)=\(\begin{cases}{l}x+1, \text { if } x \geq 1 \\ x^2+1, \text { if } x<1\end{cases}\).

Solution:

The given function f is f(x)=\(\begin{cases}{l}x+1, \text { if } x \geq 1 \\ x^2+1, \text { if } x<1\end{cases}\).

Function \(\mathrm{f}(\mathrm{x})\) may be discontinuous at doubtful point x=1 and except these point f(x) be continuous.

Now at x=1, f(1)=1+1=2

The left hand limit of f at x=1 is, \(\lim _{x \rightarrow 1^{-}} f(x)=\lim _{x \rightarrow 1^{-}}\left(x^2+1\right)=1^2+1=2\)

The right hand limit of f at x=1 is, \(\lim _{x \rightarrow 1^{+}} f(x)=\lim _{x \rightarrow 1^{+}}(x+1)\)=1+1=2

⇒ \(\lim _{x \rightarrow 1} \mathrm{f}(\mathrm{x})=\mathrm{f}(1)\)

Therefore, f is continuous at x=1. Hence, the given function f has no point of discontinuity.

Question 11. Find all points of discontinuity of f, where f is defined by f(x)=\(\begin{cases}{l}x^3-3, \text { if } x \leq 2 \\ x^2+1, \text { if } x>2\end{cases}\).

Solution:

The given function f is f(x)=\(\left\{\begin{array}{l}x^3-3, \text { if } x \leq 2 \\ x^2+1, \text { if } x>2\end{array}\right.\)

Function \(\mathrm{f}(\mathrm{x})\) may be discontinuous at doubtful point x = 2 and except these point \(\mathrm{f}(\mathrm{x})\) be continuous.

Now at x=2, f(2)=\(2^3-3=5\)

L.H.L =\(\lim _{x \rightarrow 2^{-}} f(x)=\lim _{x \rightarrow 2^{-}}\left(x^3-3\right)=2^3-3=5\)

R.H.L =\(\lim _{x \rightarrow 2^{+}} f(x)=\lim _{x \rightarrow 2^2}\left(x^2+1\right)=2^2+1=5\)

L.H.L = R.H.L =5

⇒ \(\lim _{x \rightarrow 2} f(x)=f(2)\)

Therefore, f is continuous at x = 2. Thus, the given function f is continuous at every point on the real line, Hence, f has no point of discontinuity.

Question 12. Find all points of discontinuity of f, where f is defined by f(x)= \(\begin{cases}x^{10}-1, & \text { if } x \leq 1 \\ x^2, & \text { if } x>1\end{cases}\)

Solution:

The given function is f(x)= \(\begin{cases}x^{10}-1, & \text { if } x \leq 1 \\ x^2, & \text { if } x>1\end{cases}\)

The function \(\mathrm{f}(\mathrm{x})\) may be discontinuous at doubtful point x=1 and except this point f(x) be continuous.

Now, at x=1, then the left hand limit of f at x=1 is, \(\lim _{x \rightarrow 1^{-}} f(x)=\lim _{x \rightarrow 1^{-}}\left(x^{10}-1\right)=1^{10}-1=1-1=0\)

The right hand limit of f at x=1 is, \(\lim _{x \rightarrow 1^{+}} f(x)=\lim _{x \rightarrow 1^{+}}\left(x^2\right)=1^2=1\)

It is observed that the left and right-hand limit of f at x=1 do not coincide.

Therefore,f is not continuous at x=1

Thus, from the above observation, it can be concluded that x=1 is the only point of discontinuity of f.

Question 13. Is the function defined by f(x)=\(\begin{cases}{l}x+5, \text { if } x \leq 1 \\ x-5, \text { if } x>1\end{cases}\).. a continuous function?

Solution:

The given function is f(x)=\(\begin{cases}{l}x+5, \text { if } x \leq 1 \\ x-5, \text { if } x>1\end{cases}\).

Function \(\mathrm{f}(\mathrm{x})\) may be discontinuous at doubtful point x=1 and except these point \(\mathrm{f}(\mathrm{x})\) be continuous.

Now at x=1, f(1)=1+5=6

The left hand limit of f at x=1 is, \(\lim _{x \rightarrow 1^{-}} f(x)=\lim _{x \rightarrow 1^{-}}(x+5)=1+5=6\)

The right hand limit of f at x=1 is, \(\lim _{x \rightarrow 1^{+}} f(x)=\lim _{x \rightarrow 1^{+}}(x-5)\)=1-5=-4

It is observed that the left and right-hand limits of f at x 1 do not coincide.

Therefore, f is not continuous at x- 1

Thus, from the above observation, it can be concluded that x- 1 is the only point of discontinuity of f.

Question 14. Discuss the continuity of the function f, where f is defined by f(x)= \(\begin{cases}3, & \text { if } 0 \leq x \leq 1 \\ 4, & \text { if } 1<x<3 \\ 5, & \text { if } 3 \leq x \leq 10\end{cases}\)

Solution:

The given function is \(f(\mathrm{x})=\{\begin{array}{l}3, \text { if } 0 \leq \mathrm{x} \leq 1 \\ 4, \text { if } 1<x<3 \\ 5, \text { if } 3 \leq \mathrm{x} \leq 10\end{array}\).

Function \(\mathrm{f}(\mathrm{x})\) may be discontinuous at doubtful points x=1 and x=3 and except these two points \(\mathrm{f}(\mathrm{x})\) be continuous.

Now at x=1, {f}(1)=3

The left hand limit of f at x=1 is, \(\lim _{x \rightarrow 1^{-}} f(x)=\lim _{x \rightarrow 1^{-}}(3)\)=3

The right hand limit of f at x=1 is, \(\lim _{x \rightarrow 1^{+}} f(x)=\lim _{x \rightarrow 1^{+}}(4)=4\)

It is observed that the left and right-hand limits of f at x=1 do not coincide.

Therefore, f is not continuous at x=1 and at x=3, f(3)=5

The left hand limit of f at x=3 is, \(\lim _{x \rightarrow 3^{-}} f(x)=\lim _{x \rightarrow 3^{-}}(4)=4\)

The right hand limit of f at x=3 is, \(\lim _{x \rightarrow 3^{+}} f(x)=\lim _{x \rightarrow 3^{+}}(5)=5\)

It is observed that the left and right-hand limits of f at x=3 do not coincide.

Therefore. f is not continuous at x=3. Hence, f is not continuous at x=1 and x =3

Question 15. Discuss the continuity of the function f, where f is defined by f(x)= \(\begin{cases}2 x, & \text { if } x<0 \\ 0, & \text { if } 0 \leq x \leq 1 \\ 4 x, & \text { if } x>1\end{cases}\)

Solution:

The given function is f(x)=\(\{\begin{array}{l}2 x, \text { if } x<0 \\ 0, \text { if } 0 \leq x \leq 1 \\ 4 x, \text { if } x>1\end{array}\).

Function \(\mathrm{f}(\mathrm{x})\) may be discontinuous at doubtful points x=0 and x = 1 and except these two points \mathrm{f}(\mathrm{x}) be continuous.

Now, at x=0, f(0)=0

The left hand limit of f at x=0 is, \(\lim _{x \rightarrow 0^{-}} f(x)=\lim _{x \rightarrow 0^{-}}(2 x)=2 \times 0=0\)

The right hand limit of f at x=0 is, \(\lim _{x \rightarrow 0^{+}} f(x)=\lim _{x \rightarrow 0^{+}}(0)=0\)

⇒ \(\lim _{x \rightarrow 0^{-}} f(x)=\lim _{x \rightarrow 0^{+}} f(x) \Rightarrow \text { limit exist }\)

⇒ \(\lim _{x \rightarrow 0} f(x)=f(0)\)

Therefore, f is continuous at x=0 and at x=1, f(1)=0

The left hand limit of f at x=1 is, \(\lim _{x \rightarrow 1^{-}} f(x)=\lim _{x \rightarrow 1^{-}}(0)=0\)

The right hand limit of f at x=1 is, \(\lim _{x \rightarrow 1^{+}} f(x)=\lim _{x \rightarrow 1^{+}}(4 x)=4 \times\) 1=4

It is observed that the left and right-hand limits of fat x = 1 do not coincide.

Therefore, f is not continuous at x = 1. Hence, f is not continuous only at x. = 1

Question 16. Discuss the continuity of the function f, where f is defined by f(x)= \(\begin{cases}-2, & \text { if } x \leq-1 \\ 2 x, & \text { if }-1<x \leq 1 \\ 2, & \text { if } x>1\end{cases}\)

Solution:

The given function f is f(x)= \(\begin{cases}-2, & \text { if } x \leq-1 \\ 2 x, & \text { if }-1<x \leq 1 \\ 2, & \text { if } x>1\end{cases}\)

Function \(\mathrm{f}(\mathrm{x})\) may be discontinuous at doubtful points x=-1 and x=1 and except these two points \(\mathrm{f}(\mathrm{x})\) be continuous.

Now, at x=-1, f(-1)=-2

The right hand limit of f at x=-1 is, \(\lim _{x \rightarrow-1^{+}} f(x)=\lim _{x \rightarrow-1^{+}}(2 x)=2 x(-1)=-2\)

⇒ \(\lim _{x \rightarrow-1^{-}} f(x)=\lim _{x \rightarrow-1^{+}} f(x) \Rightarrow \text { limit exist } \)

⇒ \(\lim _{x \rightarrow-1^{-}} f(x)=f(-1)\)

Therefore, f is continuous at x =-1 and at x=1, \(\mathrm{f}(1)\)=2 \times 1=2

The left hand limit of f at x=1 is, \(\lim _{x \rightarrow 1^{-}} f(x)=\lim _{x \rightarrow 1^{-}}(2 x)=2 \times 1\)=2

The right hand limit of f at x=1 is, \(\lim _{x \rightarrow 1^{+}} f(x)=\lim _{x \rightarrow 1^{+}}\) 2=2

⇒ \(\lim _{x \rightarrow 1^{-}} f(x)=\lim _{x \rightarrow 1^{+}} f(x) \Rightarrow \text { limit exist }\)

⇒ \(\lim _{x \rightarrow 1} f(x)=f(1)\)

Thus, from the above observations, it can be concluded that is continuous at all points of the real line.

Question 17. Find the relationship between a and b so that the function f defined by f(x)= \(\begin{cases} an x+1, & \text { if } x \leq 3 \\ b x+3, & \text { if } x>3\end{cases}\) is continuous at x=3.

Solution: 

The given function f is f(x)=\(\{\begin{array}{ll}a x+1, & \text { if } x \leq 3 \\ b x+3, & \text { if } x>3\end{array}\).

If f is continuous at x=3, then

⇒ \(\lim _{x \rightarrow 3^{+}} f(x)=\lim _{x \rightarrow 3^{+}} f(x)=f(3)\)

Also, \(\lim _{x \rightarrow 3^{-}} f(x)=\lim _{x \rightarrow 3^{-}}(a x+1)=3 a+1\)

⇒ \(\lim _{x \rightarrow 3^{+}} f(x)=\lim _{x \rightarrow 3^{+}}(b x+3)=3 b+3 \Rightarrow f(3)\)=3 a+1

Therefore, from (1), we obtain.

3 a+1=3 b+3=3 a+1 \(\Rightarrow 3 a+1=3 b+3 \Rightarrow 3 a=3 b+2 \Rightarrow a-b=\frac{2}{3}\)

Therefore, the required relationship is given by a-b=\(\frac{2}{3}\)

Question 18. For what value of \(\lambda\) is the function defined by f(x)=\(\begin{cases} {cc}\lambda(x^2-2 x), & \text { if } x \leq 0 \\ 4 x+1, & \text { if } x>0\end{cases} \). continuous at x=0? What about continuity at x=1?

Solution:

The given function f is f(x)=\(\begin{cases}{cc}\lambda(x^2-2 x), & \text { if } x \leq 0 \\ 4 x+1, & \text { if } x>0\end{cases}\).

If f is continuous at x=0, then \(\lim _{x \rightarrow 0^{-}} f(x)=\lim _{x \rightarrow 0^{+}} f(x)=f(0)\)

\(\lim _{x \rightarrow 0^{-}} \lambda\left(x^2-2 x\right)=\lim _{x \rightarrow 0^{+}}(4 x+1)=\lambda\left(0^2-2 \times 0\right)\)

\(\lambda\left(0^2-2 \times 0\right)=4 \times 0+1=0 \Rightarrow 0=1=0\), which is not possible

Therefore, there is no value of \(\lambda\) for which f is continuous at x=0 At x=1, f(1)=4 x+1=4 \(\times \)1+1=5

⇒ \(\lim _{x \rightarrow 1}(4 x+1)=4 \times 1+1=5 \Rightarrow \lim _{x \rightarrow 1} f(x)=f(1)\)

Therefore, for any value of \lambda, f is continuous at x=1

Question 19. Show that the function defined by g(x)=x-[x] is discontinuous at all integral points. Here [x] denotes the greatest integer less than or equal to x.

Solution:

The given function is \(\mathrm{g}(\mathrm{x})=\mathrm{x}-[\mathrm{x}]\). It is evident that g is defined at all integral points.

Let n be an integer. Then, g(n)=n-[n]=n-n=0

The left hand limit of g at x=n is, \(\lim _{x \rightarrow n^{-}} g(x)=\lim _{x \rightarrow n^{-}}(x-[x])=\lim _{x \rightarrow n^{-}}(x)-\lim _{x \rightarrow n^{-}}[x]=n-(n-1)=1\)

The right hand limit of g at x=n is, \(\lim _{x \rightarrow n^{+}} g(x)=\lim _{x \rightarrow n^{+}}(x-[x])=\lim _{x \rightarrow n^{+}}(x)-\lim _{x \rightarrow n^{+}}[x]\)=n-n=0

It is observed that the left and right-hand limits of g at x=n do not coincide.

Therefore, g is not continuous at x=n

Hence,g is discontinuous at all integral points.

Question 20. Is the function defined by \(\mathrm{f}(\mathrm{x})=\mathrm{x}^2-\sin \mathrm{x}+5\) continuous at x=\(\pi\) ?

Solution:

The given function is f(x)=\(x^2-\sin x+5\)

At x=\(\pi, f(\pi)=\pi^2-\sin \pi+5=\pi^2-0+5=\pi^2+5\) and \(\lim _{x \rightarrow \pi} f(x)=\lim _{x \rightarrow \pi}\left(x^2-\sin x+5\right)\)

⇒ \(\lim _{x \rightarrow \pi} f(x)=\pi^2-\sin \pi+5 \Rightarrow \lim _{x \rightarrow \pi} f(x)=\pi^2+5\)

⇒ \(\lim _{x \rightarrow \pi} f(x)=f(\pi)=\pi^2+5\)

Question 21. Discuss the continuity of the following functions.

  1. f(x) = sin x + cos x
    f(x) = sin x- cos x
    f(x) = sin x x cos x

Solution:

By the property of continuity, we know that addition, subtraction, and multiplication of two continuous functions are always continuous.

Here, h(x) = sin x and g(x) = cos x are two continuous functions in R.

Therefore, sin x +cosx, sinx-cosx, and sin x.cosx are also continuous in R.

Question 22. Discuss the continuity of the cosine, cosecant, secant, and cotangent functions.

Solution:

It is known that if g and h are two continuous functions, then

⇒ \(\frac{\mathrm{h}(\mathrm{x})}{\mathrm{g}(\mathrm{x})}, \mathrm{g}(\mathrm{x}) \neq 0\) is continuous

⇒ \(\frac{1}{\mathrm{~g}(\mathrm{x})}, \mathrm{g}(\mathrm{x}) \neq 0\) is continuous

⇒ \(\frac{1}{h(x)}, h(x) \neq 0\) is continuous

It has to be proved first that g(x)=sin x and h(x)=cos x are continuous functions.

Let g(x)=sin x

It is evident that gx=sin x is defined for every real number.

Let c be a real number. Put x = c + h

If x \(\rightarrow\) c, then h \(\rightarrow\) 0

gc = sin c

⇒ \(\lim _{x \rightarrow c} g(x)=\lim _{x \rightarrow c} \sin x=\lim _{h \rightarrow 0} \sin (c+h)\)

= \(\lim _{h \rightarrow 0}[\sin c \cos h+\cos c \sin h]=\lim _{h \rightarrow 0}(\sin c \cos h)+\lim _{h \rightarrow 0}(\cos c \sin h) \)

= \(\sin \mathrm{c} \cos 0+\cos \mathrm{c} \sin 0=\sin \mathrm{c}+0=\sin \mathrm{c}\)

⇒ \(\lim _{x \rightarrow c} g(x)=g(c)\)

Therefore, g is a continuous function. Let \(\mathrm{h}(\mathrm{x})= cos x \)It is evident that h}(x)=cos x is defined for every real number.

Let c be a real number. Put x=c+h

If x \(\rightarrow\) c, then h \(\rightarrow\) 0, h{c}=cos c

⇒ \(\lim _{x \rightarrow c} h(x)=\lim _{x \rightarrow c} \cos x=\lim _{h \rightarrow 0} \cos (c+h)\)

=\(\lim _{h \rightarrow 0}[\cos c \cos h-\sin c \sin h]=\lim _{h \rightarrow 0} \cos c \cos h-\lim _{h \rightarrow 0} \sin c \sin h \)

=\(\cos c \cos 0-\sin c \sin 0=\cos c \times 1-\sin c \times 0=\cos c\)

⇒ \(\lim _{x \rightarrow c} h(x)=h(c)\)

Therefore, h(x)=cos x is a continuous function.

It can be concluded that,\({cosec} x=\frac{1}{\sin x}, \sin x \neq 0\) is continuous

⇒ \({cosec} x, x \neq n \pi(n \in Z)\) is continuous

Therefore, cosecant is continuous except at x=n p, n \(\in\) Z

sec x=\(\frac{1}{\cos x}, \cos x \neq 0\) is continuous

⇒ \(\sec x, x \neq(2 n+1) \frac{\pi}{2}(n \in Z)\) is continuous

Therefore, secant is continuous except at x=\((2 n+1) \frac{\pi}{2}(n \in Z)\)

⇒ \(\cot x=\frac{\cos x}{\sin x}, \sin x \neq 0\) is continuous

⇒ \(\cot x, x \neq n \pi(n \in Z)\) is continuous

Therefore, cotangent is continuous except at x=n \(\pi, n \in Z\)

Question 23. Find the points of discontinuity of f, where f(x)= \(\begin{cases}\frac{\sin x}{x}, & \text { if } x<0 \\ x+1, & \text { if } x \geq 0\end{cases}\)

Solution:

The given function f(x)= \(\begin{cases}\frac{\sin x}{x}, & \text { if } x<0 \\ x+1, & \text { if } x \geq 0\end{cases}\)

f(x) may be discontinuous at doubtful point x=0 and except this point $f(x)$ be continuous.

Now, at x=0, f(0)=0+1=1

The left hand limit of f at x=0 is, \(\lim _{x \rightarrow 0^{\circ}} f(x)=\lim _{x \rightarrow 0} \frac{\sin x}{x}=1\)

The right hand limit of f at x=0 is, \(\lim _{x \rightarrow 0} f(x)=\lim _{x \rightarrow 0^{+}}(x+1)=1\)

⇒ \(\lim _{x \rightarrow 0^{-}} f(x)=\lim _{x \rightarrow 0^{+}} f(x)=f(0)\)

Therefore, f is continuous at x =0

Thus, f has no point of discontinuity,

Question 24. Determine if f defined by f(x)=\(\begin{cases}{cl}x^2 \sin \frac{1}{x}, & \text { if } x \neq 0 \\ 0, & \text { if } x=0\end{cases}\). is a continuous function?

Solution:

The given function f is f(x)=\(\begin{cases}{cc}x^2 \sin \frac{1}{x}, & \text { if } x \neq 0 \\ 0 . & \text { if } x=0\end{cases}\).

f(x) may be discontinuous at doubtful point x=0 and except this point f(x) be continuous,

Now, at x =0, f(0)=0

⇒ \(\lim _{x \rightarrow 0} f(x)=\lim _{s \rightarrow 0} x^2 \sin \frac{1}{x}=0 \times(\text { Finite value from } 0 \text { to 1) }\)

⇒ \(\lim _{x \rightarrow 0} f(x)\)=0

Here, \(\lim _{x \rightarrow \infty} f(x)\)=f(0)

Therefore, f is continuous at x=0

Question 25. Examine the continuity of f, where f is defined by \(f(x)=\begin{cases}{cl}\sin x-\cos x & \text {, if } x \neq 0 \\ -1 & \text {, if } x=0\end{cases}\).

Solution:

The given function f(x)=\(\begin{cases}{cl}\sin x-\cos x & \text {, if } x \neq 0 \\ -1 & \text {, if } x=0\end{cases}\).

f(x) may be discontinuous at doubtful point x=0 and except this point f(x) be continuous. Now, at x=0, f(0)=-1

L.H.L =\(\lim _{x \rightarrow 0^{\circ}} f(x)=\lim _{x \rightarrow 0}(\sin x-\cos x)=\sin \theta-\cos 0=0-1=-1\)

R.H.L =\(\lim _{x \rightarrow x^2} f(x)=\lim _{x \rightarrow x^0}(\sin x-\cos x)=\sin 0-\cos 0=0-1=-1\)

⇒ \(\lim _{x \rightarrow 0^{-}} f(x)=\lim _{x \rightarrow+0^{+}} f(x)=f(0)\)

Therefore, f is continuous at x=0

Thus, f is a continuous function.

Question 26. Find the values of k so that the function f is continuous at the indicated point.

Solution:

f(x)=\(\begin{cases}{cc}\frac{k \cos x}{\pi-2 x} & \text {, if } x \neq \frac{\pi}{2} \\ 3 & \text {,if } x=\frac{\pi}{2}\end{cases}\). at x=\(\frac{\pi}{2}\)

The given function f(x)=\(\{\begin{cases}{cl}\frac{k \cos x}{\pi-2 x} & \text {, if } x \neq \frac{\pi}{2} \\ 3 & \text {, if } x=\frac{\pi}{2}\end{cases}\). is continuous at x=\(\frac{\pi}{2}\),

⇒ \(\lim _{x \rightarrow} f(x)=f\left(\frac{\pi}{2}\right)\)=3

Now, \(\lim _{\rightarrow \rightarrow-1} f(x)=\lim _{x \rightarrow \frac{\pi}{2}} \frac{k \cos x}{\pi-2 x}\)

Pet x=\(\frac{\pi}{2}+h, then, x \rightarrow \frac{\pi}{2} \Rightarrow h \rightarrow 0\)

⇒ \(\lim _{x \rightarrow+} f(x)=\lim _{x \rightarrow \frac{\pi}{2}} \frac{k \cos x}{\pi-2 x}=\lim _{x \rightarrow 4} \frac{k \cos \left(\frac{\pi}{2}+h\right)}{\pi-2\left(\frac{\pi}{2}+h\right)}\)

=k \(\lim _{b \rightarrow 0} \frac{-\sin h}{-2 h}=\frac{k}{2} \lim _{x \rightarrow \infty} \frac{\sin h}{h}=\frac{k}{2}, 1=\frac{k}{2}\)

⇒ \(\lim _{\rightarrow-\frac{1}{2}} f(x)=f\left(\frac{\pi}{2}\right) (From eq. (1))\)

⇒ \(\frac{k}{2}\)=3

k=6

Question 27. Find the values of k so that the function f is continuous at the indicated point:

Solution:

f(x)=\(\begin{cases}{l}
k x^2 & \text {, if } x \leq 2 \\
3 & \text {,if } x>2
\end{cases} \text { at } x=2t\).

The given function is f(x)=\(\begin{cases}{cl}k x^3 & \text { if } x \leq 2 \\ 3 & \text {, if } x>2\end{cases}\).

Is continuously at x=2,

⇒ \(\lim _{x \rightarrow x^2} f(x)=\lim _{x \rightarrow 0} f(x)=f(2) \Rightarrow \lim _{x \rightarrow 7}\left(k x^2\right)=\lim _{x \rightarrow 1}(3)=4 k\)

k \(\times 2^2=3=4 k \Rightarrow 4 k=3=4 k \Rightarrow 4 k=3 \Rightarrow k=\frac{3}{4}\)

Therefore, the required value of k is \(\frac{3}{4}\)

Question 28. Find the values of k so that the function f is continuous at the indicated point:

Solution:

f(x)=\(\begin{cases}{ll}
k x+1, & \text { if } x \leq \pi \\
\cos x, & \text { if } x>\pi
\end{cases} \text { at } x=\pi\).

The given function f(x)= \(\begin{cases}k x+1, & \text { if } x \leq \pi \\ \cos x, & \text { if } x>\pi\end{cases}\)

Is continuous at \(\mathrm{x}=\pi\)

⇒ \(\lim _{x \rightarrow 1^{-}} f(x)=\lim _{x \rightarrow m^{+}} f(x)=f(\pi) \Rightarrow \lim _{x \rightarrow z^{+}}(k x+1)=\lim _{x \rightarrow \pm^{-}} \cos x=k \pi+1 \)

k \(\pi+1=\cos \pi=k \pi+1 \Rightarrow k \pi+1=-1=k \pi+1 \Rightarrow\)

k=-\(\frac{2}{\pi}\)

Therefore, the required value of k is \(-\frac{2}{\pi}\)

Question 29. Find the values of k so that the function f is continuous at the indicated point:

Solution:

f(x)=\(\{\begin{array}{ll}
k x+1, & \text { if } x \leq 5 \\
3 x-5, & \text { if } x>5
\end{array}\) at x = 5

The given function f(x)= \(\begin{cases}k x+1, & \text { if } x \leq 5 \\ 3 x-5, & \text { if } x>5\end{cases}\)

Is continuous at x=5,

⇒ \(\lim _{x \rightarrow 5^5} f(x)=\lim _{x \rightarrow 5^5} f(x)=f(5) \Rightarrow \lim _{x \rightarrow 5^{(}}(k x+1)=\lim _{x \rightarrow 5^{(}}(3 x-5)=5 k+1\)

5 k+1=15-5=5 k+1 \(\Rightarrow 5 k+1=10 \Rightarrow 5 k=9 \Rightarrow k=\frac{9}{5}\)

Therefore, the required value of k is \(\frac{9}{5}\)

Question 30. Find the values of a and b such that the function defined by f(x)=\(\begin{cases}{cl}5, & \text { if } x \leq 2 \\ an x+b, & \text { if } 2<x<10 \\ 21, & \text { if } x \geq 10\end{cases}\) is a continuous function.

Solution:

The given function f(x)=\(\begin{cases}{cl}5, & \text { if } x \leq 2 \\ a x+b, & \text { if } 2<x<10 \text { is continuous function. } \\ 21, & \text { if } x \geq 10\end{cases}.\)

Since f is continuous at x=2, we obtain \(\lim _{x \rightarrow 2} f(x)=\lim _{x \rightarrow z^2} f(x)=f(2)\)

⇒ \(\lim _{x \rightarrow 5}(5)=\lim _{x \rightarrow 2}(a x+b)=5 \Rightarrow 5=2 a+b=5 \Rightarrow 2 a+b=5\)  → Equation 1

Since f is continuous at x=10, we obtain

⇒ \(\lim _{x \rightarrow 10^{-}} f(x)=\lim _{x \rightarrow 10^{+}} f(x)=f(10) \Rightarrow \lim _{x \rightarrow 10^{-}}(a x+b)=\lim _{x \rightarrow 10^0}(21)=21\) →  Equation 2

10 a+b=21=21 \(\Rightarrow\) 10 a+b=21

By subtracting equation (1) from equation (2), we obtain

8 \(\mathrm{a}=16 \Rightarrow \mathrm{a}=2\)

By putting a-2 in equation ( 1 ), we obtain

2 \(\approx 2+b=5 \Rightarrow 4+b=5 \Rightarrow\) b=1

Therefore, the values of a and b for which f is a continuous function are 2 and 1 respectively.

Question 31. Show that the function defined by f(x)=p\(\cos \left(x^2\right)\) is a continuous function.

Solution:

Given f(x)=\(\cos x^2\)

Let g(x)=\(\mathrm{x}^2 and h(x)=cos x\)

⇒ \(\mathrm{f}(\mathrm{x})=\cos (\mathrm{g}(\mathrm{x}))=\mathrm{h}(\mathrm{g}(\mathrm{x})) \Rightarrow f(\mathrm{x})={hog}(\mathrm{x})\)

g(x) is a polynomial function, which is continuous in its domain R so,g(x) is continuous.

Again h(x)=cos x is a trigonometric function that is continuous in {R}.

So, it is also continuous.

Since g(x) and h(x) is continuous.

i.e. h o g(x) is also continuous.

Hence, f(x) is a continuous function.

Question 32. Show that the function defined by f(x)=\(\mid \cos x\) is a continuous function.

Solution:

Given f(x)=|cos x|

Let g(x)= cos x and h(x)=|x|

⇒ \(\mathrm{f}(\mathrm{x})=|\mathrm{g}(\mathrm{x})|=\mathrm{h}(\mathrm{g}(\mathrm{x})) \Rightarrow \mathrm{f}(\mathrm{x})={hog}(\mathrm{x})\)

g(x) is a trigonometric function, which is continuous in its domain R.

So, g(x) is continuous

h}(x)=|x| is a modulus function, which is continuous in R.

So, it is also continuous.

Since g(x) and h(x) is continuous.

i.e. hog (x) is also continuous.

Hence, \(\mathrm{f}(\mathrm{x})=|\cos \mathrm{x}|\) is continuous function.

Question 33. Examine that sin x \(\mid\) is a continuous function.

Solution:

Let f(x)=sin |x|

This function f is defined for every real number and f can be written as the composition of two functions as,

Given f(x)=sin |x|

Let g(x)-|x| and h(x)=sin x

f(x)=sin (g(x))=h(g(x)) \(\Rightarrow\) f(x)=h o g(x)

g(x) is the module’s function, which is continuous in its domain R.

So, g(x) is continuous.

Again h (x) = sin x is a trigonometric function, which is continuous in R.

So, it is also continuous.

Since, g (x) and h (x) is continuous,

i.e. hog (x) is also continuous.

Hence, f (x)- sin jx] is a continuous function.

Question 34. Find all the points of discontinuity of f defined by f(x)=|x|-|x+1|

Solution:

Given that f(x)=|x|-|x+1|

f(x)=\(\begin{cases}{cc}
1 & \text { if } x<-1 \\
-2 x-1 & \text { if }-1 \leq x<0 \\
-1 & \text { if } x \geq 0
\end{cases}\)

Here, f(x) may be discontinuous at doubtful points x-1, x=0 and except these two points f(x) be continuous,

Now at x=-1

L.H.L =\(\lim _{x \rightarrow-1} f(x)=\lim _{x \rightarrow-1}(1)\)=1

R.H.L. =\(\lim _{x \rightarrow-1} f(x)=\lim _{x \rightarrow-1}(-2 x-1)\)=1

L.H.S. = R.H.S. \(\Rightarrow\) limit exist and {f}(-1)=1

Here \(\lim f(x)=f(-1)\)

So, f(x) is continuous at x=-1

Again at x=0

L.H.L. =\(\lim _{x \rightarrow 0^{+}} f(x)=\lim _{x \rightarrow 0^{+}}(-2 x-1)=-1 \)

R.H.L.=\(\lim _{x \rightarrow 0^{+}} f(x)=\lim _{x \rightarrow 0^{+}}(-1)=-1 \)

L.H.L = R.H.L \(\Rightarrow \text { limit exist and }\) f(0)=-1

Here \(\lim _{x \rightarrow 0} f(x)\)=f(0)

So, f(x) is continuous at x=0

Hence, there is no point of discontinuity for f(x).

Continuity And Differentiability Exercise 5.2

Question 1. \(\sin \left(x^2+5\right)\)

Solution:

⇒ \(\frac{d}{d x}\left[\sin \left(x^2+5\right)\right] =\cos \left(x^2+5\right) \cdot \frac{d}{d x}\left(x^2+5\right)\)

= \(\cos \left(x^4+5\right) \cdot\left[\frac{d}{d x}\left(x^2\right)+\frac{d}{d x}(5)\right]=\cos \left(x^2+5\right) \cdot[2 x+0]=2 x \cos \left(x^2+5\right)\)

Question 2. \(\cos (\sin x) \)

Solution:

⇒ \(\frac{d}{d x}[\cos (\sin x)]=-\sin (\sin x) \cdot \frac{d}{d x}(\sin x)\)

=-\(\sin (\sin x) \cdot \cos x=-\cos x \sin (\sin x)\)

Question 3. sin (a x+b)

Solution:

⇒ \(\frac{d}{d x}[\sin (a x+b)]=\cos (a x+b) \cdot \frac{d}{d x}(a x+b)\)

=\(\cos (a x+b) \cdot\left[\frac{d}{d x}(a x)+\frac{d}{d x}(b)\right]=\cos (a x+b) \cdot(a+0)=a \cos (a x+b)\)

Question 4. \(\sec (\tan (\sqrt{x}))\)

Solution:

⇒ \({\frac{d x}{d x}[\sec (\tan \sqrt{x})]} =\sec (\tan \sqrt{x}) \cdot \tan (\tan \sqrt{x}) \cdot \frac{d}{d x}(\tan \sqrt{x})\)

= \(\sec (\tan \sqrt{x}) \cdot \tan (\tan \sqrt{x}) \cdot \sec ^2(\sqrt{x}) \cdot \frac{d}{d x}(\sqrt{x}) \)

= \(\sec (\tan \sqrt{x}) \cdot \tan (\tan \sqrt{x}) \cdot \sec ^2(\sqrt{x}) \cdot \frac{1}{2 \sqrt{x}}\)

= \(\frac{\sec (\tan \sqrt{x}) \cdot \tan (\tan \sqrt{x}) \sec ^2(\sqrt{x})}{2 \sqrt{x}}\)

Question 5. \(\frac{\sin (a x+b)}{\cos (c x+d)}\)

Solution:

Let f(x)=\(\frac{\sin (a x+b)}{\cos (c x+d)}=\frac{u(x)}{v(x)}\)

⇒ \(f^{\prime}(x)=\frac{v(x)\left[u^{\prime}(x)\right]-u(x)\left[v^{\prime}(x)\right]}{[v(x)]^2}\)

⇒ \(\mathrm{f}(\mathrm{x})=\frac{\cos (c x+d) \cdot a \cos (a x+b)+\sin (a x+b) \cdot c \sin (c x+d)}{\cos ^2(c x+d)}\)

⇒ \(f^{\prime}(x)=a \cos (a x+b) \cdot \sec (c x+d)+c \sin (a x+b) \cdot \tan (c x+d) \cdot \sec (c x+d)\)

Question 6. \(\cos x^3 \cdot \sin ^2\left(x^5\right)\)

Solution:

The given function is \(\cos x^3-\sin ^2\left(x^3\right)\)

⇒ \(\frac{d}{d x}\left[\cos x^3 \cdot \sin ^2\left(x^5\right)\right]\)

= \(\sin ^2\left(x^5\right) \times \frac{d}{d x}\left(\cos x^3\right)+\cos x^3 \times \frac{d}{d x}\left[\sin ^2\left(x^5\right)\right]\)

= \(\sin ^2\left(x^5\right) \times\left(-\sin x^3\right) \times \frac{d}{d x}\left(x^3\right)+\cos x^5 \times 2 \sin \left(x^5\right) \cdot \frac{d}{d x}\left[\sin x^4\right]\)

=-\(\sin x^3 \sin ^2\left(x^5\right) \times 3 x^2+2 \sin x^5 \cos x^3 \cdot \cos x^5 \times \frac{d}{d x}\left(x^5\right)\)

=-\(3 x^2 \sin x^3 \cdot \sin ^2\left(x^5\right)+2 \sin x^5 \cos x^5 \cos x^3 \times 5 x^4 \)

= \(10 x^4 \sin x^4 \cos x^4 \cos x^3-3 x^7 \sin x^5 \sin ^2\left(x^5\right)\)

Question 7. 2 \(\sqrt{\cot \left(x^2\right)}\)

Solution:

⇒ \(\frac{d}{d x}\left[2 \sqrt{\cot \left(x^2\right)}\right]=2 \cdot \frac{1}{2 \sqrt{\cot \left(x^2\right)}} \times \frac{d}{d x}\left[\cot \left(x^2\right)\right]\)

=\(\sqrt{\frac{\sin \left(x^2\right)}{\cos \left(x^2\right)}} \times-{cosec}^2\left(x^2\right) \times \frac{d}{d x}\left(x^2\right)\)

=-\(\sqrt{\frac{\sin \left(x^2\right)}{\cos \left(x^2\right)}} \times \frac{1}{\sin ^2\left(x^2\right)} \times(2 x)\)

= \(\frac{-2 x}{\sqrt{\cos x^2 \sqrt{\sin x^2} \sin x^2}}=\frac{-2 \sqrt{2} x}{\sqrt{2 \sin x^2 \cos x^2}\left(\sin x^2\right)}\)

= \(\frac{-2 \sqrt{2} x}{\sin x^2 \sqrt{\sin 2 x^2}}\)

Question 8. \(\cos (\sqrt{x}\)

Solution:

⇒ \(\frac{d}{d x}[\cos (\sqrt{x})]=-\sin (\sqrt{x}) \cdot \frac{d}{d x}(\sqrt{x})=\frac{-\sin \sqrt{x}}{2 \sqrt{x}}\)

Question 9. Prove that the function f is given by f(x)=|x-1|, x \(\in \mathbf{R}\) is not differentiable at x=1.

Solution:

The given function is f(x)=\(|\mathrm{x}-1|, \mathrm{x} \in \mathbf{R}\)

It is known that a function f is differentiable at a point x=c in its domain if both \(\lim _{\mathrm{h} \rightarrow 0}\)

⇒ \(\frac{\mathrm{f}(\mathrm{c}-\mathrm{h})-\mathrm{f}(\mathrm{c})}{-\mathrm{h}} and \lim _{\mathrm{h} \rightarrow 0}\)

⇒ \(\frac{\mathrm{f}(\mathrm{c}+\mathrm{h})-\mathrm{f}(\mathrm{c})}{\mathrm{h}}\)are finite and equal.

To check the differentiability of the given function at x=1, consider the left-hand derivative of f at x=1

L.H.D. =\(\lim _{h \rightarrow 0} \frac{f(1-h)-f(1)}{-h}=\lim _{h \rightarrow 0} \frac{|1-h-1|-|1-1|}{-h}=\lim _{h \rightarrow 0}\left(\frac{h-0}{-h}\right)=-1\)

Consider the right-hand derivative of f at x=1

R.H.D. =\(\lim _{h \rightarrow 0} \frac{f(1+h)-f(1)}{h}=\lim _{h \rightarrow 0} \frac{|1+h-1|-|1-1|}{h}=\lim _{h \rightarrow 0}\left(\frac{h-0}{h}\right)=1\)

Since the left and right-hand derivatives of f at x=1 are not equal, f is not differentiable at x-1

Question 10. Prove that the greatest integer function defined by \(\mathrm{f}(\mathrm{x})=[\mathrm{x}], 0<\mathrm{x}<3\), is not differentiable at x-1 and x-2.

Solution:

The given function f is f(x)=[x], 0<x<3

It is known that a function f is differentiable at a point x=c in its domain if both \(\lim _{\mathrm{h} \rightarrow 0} \frac{f(c-h)-f(c)}{-h} and \lim _{x \rightarrow \infty} \frac{f(c+h)-f(c)}{h}\) are finite and equal.

To check the differentiability of the given function at x=1, consider the left-hand derivative of f at x=1,

L.H.D. =\(\lim _{h \rightarrow 0} \frac{f(1-h)-f(1)}{-h}=\lim _{h \rightarrow 0} \frac{[1-h]-[1]}{-h}=\lim _{h \rightarrow 0} \frac{0-1}{-h}=\infty\)

Consider the right-hand derivative of f at x=1

R.H.D. =\(\lim _{h \rightarrow h} \frac{f(1+h)-f(h)}{h}=\lim _{h \rightarrow 0} \frac{[1+h]-[t]}{h}=\lim _{h \rightarrow 1} \frac{1-r}{h}=\lim _{h \rightarrow 0}\) 0=0

Since the left and right-hand derivatives of f at x=1 are not equal, f is not differentiable at x=1

To check the differentiability of the given function at x=2, consider the left-hand derivative of f at x-2

L.H.D. =\(\lim _{h \rightarrow h} \frac{f(2-h)-f(2)}{-h}=\lim _{h \rightarrow 0} \frac{[2-h]-[2]}{-h}=\lim _{h \rightarrow 0} \frac{1-2}{-h}=\lim _{h \rightarrow 0} \frac{-1}{-h}=\infty\)

Consider the right-hand derivative of f at x-1.

R.H.D \(-\lim _{h \rightarrow 0} \frac{f(2+h)-f(2)}{h}=\lim _{h \rightarrow h} \frac{[2+h]-[2]}{h}=\lim _{h \rightarrow 0} \frac{2-2}{h}=\lim _{h \rightarrow 0}\) 0=0

Since the left and right-hand derivatives of f at x=2 are not equal, f is not differentiable at x=2

Continuity And Differentiability Exercise 5.3

Question 1.1. 2 x+3 y = sin x

Solution:

⇒ \(\frac{d}{d x}(2 x+3 y)=\frac{d}{d x}(\sin x)\)

⇒ \(\frac{d}{d x}(2 x)+\frac{d}{d x}(3 y)=\cos x\)

2+3 \(\frac{d y}{d x}=\cos x \)

3 \(\frac{d y}{d x}=\cos x-2 \)

⇒ \(\frac{d y}{d x}=\frac{\cos x-2}{3}\)

Question 2. 2 x+3 y=sin y

Solution:

⇒ \(\frac{d}{d x}(2 x)+\frac{d}{d x}(3 y)=\frac{d}{d x}(\sin y)\)

⇒ \(2+3 \frac{d y}{d x}=\cos y \frac{d y}{d x}\) [By using chain rule] }

⇒ \(2=(\cos y-3) \frac{d y}{d x} \Rightarrow \frac{d y}{d x}=\frac{2}{\cos y-3}\)

Question 3. a x+b \(y^2=\cos y\)

Solution:

⇒ \(\frac{d}{d x}(a x)+\frac{d}{d x}\left(b y^2\right)=\frac{d}{d x}(\cos y)\)

a+b \(\frac{d}{d x}\left(y^2\right)=\frac{d}{d x}(\cos y)\)

Using the chain rule, we obtain

⇒ \(\frac{d}{d x}\left(y^2\right)=2 y \frac{d y}{d x} and \frac{d}{d x}(\cos y)=-\sin y \frac{d y}{d x}\)

From (1) and (2), we obtain

a+b \(\times 2 y \frac{d y}{d x}=-\sin y \frac{d y}{d x}\)

⇒ \((2 b y+\sin y) \frac{d y}{d x}=-a\)

⇒ \(\frac{d y}{d x}=\frac{-a}{2 b y+\sin y}\)

Question 4. \(x y+y^2=\tan x+y\)

Solution :

⇒ \(\frac{d}{d x}\left(x y+y^2\right)=\frac{d}{d x}(\tan x+y) \Rightarrow \frac{d}{d x}(x y)+\frac{d}{d x}\left(y^2\right)\)

= \(\frac{d}{d x}(\tan x)+\frac{d y}{d x}\)

⇒ \(\left[y \cdot \frac{d}{d x}(x)+x \cdot \frac{d y}{d x}\right]+2 y \frac{d y}{d x}\)

= \(\sec ^2 x+\frac{d y}{d x}\) [Using product rule and chain rule]

y \(\cdot 1+x \cdot \frac{d y}{d x}+2 y \frac{d y}{d x}=\sec ^2 x+\frac{d y}{d x} \Rightarrow(x+2 y-1) \frac{d y}{d x}=\sec ^2 x-y\)

⇒ \(\frac{d y}{d x}=\frac{\sec ^2 x-y}{(x+2 y-1)}\)

Question 5. \( x^2+x y+y^2\)=100

Solution:

⇒ \(\frac{d}{d x}\left(x^2+x y+y^2\right)=\frac{d}{d x}(100)\)

⇒ \(\frac{d}{d x}\left(x^2\right)+\frac{d}{d x}(x y)+\frac{d}{d x}\left(y^2\right)\)=0 [Derivative of constant function is 0 ]

⇒ \(2 x+\left[y \cdot \frac{d}{d x}(x)+x \cdot \frac{d y}{d x}\right]+2 y \frac{d y}{d x}\)=0 [Using product rule and chain rule]

⇒ \(2 x+y \cdot 1+x \cdot \frac{d y}{d x}+2 y \frac{d y}{d x}\)=0

⇒ \(2 x+y+(x+2 y) \frac{d y}{d x}\)=0

⇒ \(\frac{d y}{d x}=-\frac{2 x+y}{x+2 y}\)

Question 6. \(x^3+x^2 y+x y^2+y^3=81\)

Solution:

⇒ \(\frac{d}{d x}\left(x^3+x^2 y+x y^2+y^3\right)=\frac{d}{d x}(81)\)

⇒ \(\frac{d}{d x}\left(x^3\right)+\frac{d}{d x}\left(x^2 y\right)+\frac{d}{d x}\left(x y^2\right)+\frac{d}{d x}\left(y^3\right)\)=0

⇒ \(3 x^2+\left[y \frac{d}{d x}\left(x^2\right)+x^2 \frac{d y}{d x}\right]+\left[y^2 \frac{d}{d x}(x)+x \frac{d}{d x}\left(y^2\right)\right]+3 y^2 \frac{d y}{d x}\)=0

⇒ \(3 x^2+\left[y \cdot 2 x+x^2 \frac{d y}{d x}\right]+\left[y^2 \cdot 1+x \cdot 2 y \cdot \frac{d y}{d x}\right]+3 y^2 \frac{d y}{d x}\)=0

⇒ \(\left(x^2+2 x y+3 y^2\right) \frac{d y}{d x}+\left(3 x^2+2 x y+y^2\right)\)=0

⇒ \(\frac{d y}{d x}=\frac{-\left(3 x^2+2 x y+y^2\right)}{\left(x^2+2 x y+3 y^2\right)}\)

Question 7. \(\sin ^2 y+\cos x y=k\)

Solution:

⇒ \(\frac{d}{d x}\left(\sin ^2 y+\cos x y\right)=\frac{d}{d x}(k)\)

⇒ \(\frac{d}{d x}\left(\sin ^2 y\right)+\frac{d}{d x}(\cos x y)=0\)

Using the chain rule, we obtain

⇒ \(\frac{d}{d x}\left(\sin ^2 y\right) =2 \sin y \frac{d}{d x}(\sin y)=2 \sin y \cos y \frac{d y}{d x}\)

⇒ \(\frac{d}{d x}(\cos x y)=-\sin x y \frac{d}{d x}(x y)=-\sin x y\left[y \frac{d}{d x}(x)+x \frac{d y}{d x}\right]\)

=-\(\sin x y\left[y \cdot 1+x \frac{d y}{d x}\right]=-y \sin x y-x \sin x y \frac{d y}{d x}\)

From (1), (2), and (3), we obtain

2 \(\sin y \cos y \frac{d y}{d x}-y \sin x y-x \sin x y \frac{d y}{d x}=0 \)

⇒ \((2 \sin y \cos y-x \sin x y) \frac{d y}{d x}=y \sin x y\)

⇒ \((\sin 2 y-x \sin x y) \frac{d y}{d x}=y \sin x y\)

⇒ \(\frac{d y}{d x}=\frac{y \sin x y}{\sin 2 y-x \sin x y}\)

Question 8. \(\sin ^2 x+\cos ^2 y=1\)

Solution:

⇒ \(\frac{d}{d x}\left(\sin ^2 x+\cos ^2 y\right)=\frac{d}{d x}(1)\)

⇒ \(\frac{d}{d x}\left(\sin ^2 x\right)+\frac{d}{d x}\left(\cos ^2 y\right)=0 \Rightarrow 2 \sin x \cdot \frac{d}{d x}(\sin x)+2 \cos y \cdot \frac{d}{d x}(\cos y)=0\)

2 \(\sin x \cos x+2 \cos y(-\sin y) \cdot \frac{d y}{d x}\)=0

⇒ \(\sin 2 x-\sin 2 y \frac{d y}{d x}\)=0

⇒ \(\frac{d y}{d x}=\frac{\sin 2 x}{\sin 2 y}\)

Question 9. y=\(\sin ^{-1}\left(\frac{2 x}{1+x^2}\right)\)

Solution:

y=\(\sin ^{-1}\left(\frac{2 x}{1+x^2}\right)\)

Put x=\(\tan \theta\)

⇒ \(\theta=\tan ^{-t}\) x

⇒ \(y=\sin ^{-1}\left(\frac{2 x}{1+x^2}\right)=\sin ^{-1}\left(\frac{2 \tan \theta}{1+\tan ^2 \theta}\right)\)

y=\(\sin ^{-1}(\sin 2 \theta)=2 \theta\)

y=2 \(\tan ^{-1} x\)

∴ \(\frac{d y}{d x}=\frac{2}{1+x^2}\)

Question 10. y=\(\tan ^{-1}\left(\frac{3 x-x^3}{1-3 x^2}\right),-\frac{1}{\sqrt{3}}<x<\frac{1}{\sqrt{3}} \)

Solution:

y=\(\tan ^{-1}\left(\frac{3 x-x^3}{1-3 x^2}\right)\)

Put x=\(\tan \theta \Rightarrow \theta=\tan ^{-1}x\)

y=\(\tan ^{-1}\left(\frac{3 \tan \theta-\tan ^3 \theta}{1-3 \tan ^2 \theta}\right)=\tan ^{-1}(\tan 3 \theta)\)

y=3 \(\theta=3 \tan ^{-1} x\)

⇒ \(\frac{d y}{d x}=\frac{3}{1+x^2}\)

Question 11. y=\(\cos ^{-1}\left(\frac{1-x^2}{1+x^2}\right), 0<x<1\)

Solution:

y=\(\cos ^{-1}\left(\frac{1-x^2}{1+x^2}\right)\)

Put x=\(\tan \theta \Rightarrow \theta=\tan ^{-1}\) x Equation 1

y=\(\cos ^{-1}\left(\frac{1-\tan ^2 \theta}{1+\tan ^2 \theta}\right) \left[ \cos 2 \theta=\frac{1-\tan ^2 \theta}{1+\tan ^2 \theta}\right]\)

y=\(\cos ^{-1}(\cos 2 \theta)=2 \theta \quad \Rightarrow y=2 \tan ^{-1}\) x From Equation 1

⇒ \(\frac{d y}{d x}=2 \cdot \frac{1}{1+x^2}\)

Question 12. y=\(\sin ^{-1}\left(\frac{1-x^2}{1+x^2}\right)\), 0<x<1

Solution:

y=\(\sin ^{-1}\left(\frac{1-x^2}{1+x^2}\right)\)

Put x=\(\tan \theta \Rightarrow \theta=\tan ^{-1} x\)

y=\(\sin ^{-1}\left(\frac{1-\tan ^2 \theta}{1+\tan ^2 \theta}\right) \Rightarrow y=\sin ^{-1}(\cos 2 \theta)\)

y=\(\sin ^{-1}\left\{\sin \left(\frac{\pi}{2}-2 \theta\right)\right\}\)

y=\(\frac{\pi}{2}-20=\frac{\pi}{2}-2 \tan ^{-1} x\)

⇒ \(\frac{d y}{d x}=-2 \cdot\left(\frac{1}{1+x^2}\right)=\frac{-2}{1+x^2}\)

Question 13. y=\(\cos ^{-1}\left(\frac{2 x}{1+x^2}\right),-1<x<1 \)

Solution:

y=\(\cos ^{-1}\left(\frac{2 x}{1+x^2}\right)\)

Put x=\(\tan \theta \Rightarrow \theta=\tan ^{-1} x\) Equation 1

y=\(\cos ^{-1}\left(\frac{2 \tan \theta}{1+\tan ^2 \theta}\right)\) \(\left( \sin 2 \theta=\frac{2 \tan \theta}{1+\tan ^2 \theta}\right)\)

y=\(\cos ^{-1}(\sin 2 \theta)\)

y=\(\cos ^{-1}\left\{\cos \left(\frac{\pi}{2}-2 \theta\right)\right\} \Rightarrow y=\frac{\pi}{2}-2 \theta\)

⇒ \( y=\frac{\pi}{2}-2 \tan ^{-1} x\) [From eq. (1)]

⇒ \(\frac{d y}{d x}=\frac{-2}{1+x^2}\)

Question 14. y=\(\sin ^{-1}\left(2 x \sqrt{1-x^2}\right),-\frac{1}{\sqrt{2}}<x<\frac{1}{\sqrt{2}}\)

Solution:

y=\(\sin ^{-1}\left(2 x \sqrt{1-x^2}\right)\)

Put x=\(\sin \theta \Rightarrow \theta=\sin ^{-1}\) x Equation 1

y=\(\sin ^{-1}\left(2 \sin \theta \sqrt{1-\sin ^2 \theta}\right)=\sin ^{-1}(2 \sin \theta \cos \theta)\)

y=\(\sin ^{-1}(\sin 2 \theta)=2 \theta \Rightarrow y=2 \sin ^{-1} x\) From Equation 1

⇒ \(\frac{d y}{d x}=\frac{2}{\sqrt{1-x^2}}\)

Question 15. y=\(\sec ^{-1}\left(\frac{1}{2 x^2-1}\right), 0<x<\frac{1}{\sqrt{2}}\)

Solution:

y=\(\sec ^{-1}\left(\frac{1}{2 x^2-1}\right)\)

Put x=\(\cos \theta \Rightarrow \theta=\cos ^{-1}\) x

y=\(\sec ^{-1}\left(\frac{1}{2 \cos ^2 \theta-1}\right)=\sec ^{-1}\left(\frac{1}{\cos 2 \theta}\right)=\sec ^{-1}(\sec 2 \theta)=2 \theta \)

y=2 \(\cos ^{-1}\) x (From eq. (1))

∴ \(\frac{d y}{d x}=\frac{-2}{\sqrt{1-x^2}}\)

Continuity And Differentiability Exercise 5.4

Question 1. \(\frac{e^x}{\sin x}\)

Solution:

Let y=\(\frac{e^4}{\sin x}\)

By using the quotient rule, we obtain

⇒ \(\frac{d y}{d x} =\frac{\sin x \frac{d}{d x}\left(e^x\right)-e^x \frac{d}{d x}(\sin x)}{\sin ^2 x}\)

=\(\frac{e^x(\sin x-\cos x)}{\sin ^2 x}, x \neq n \pi, n \in Z\) \(\left[\frac{d}{d x}\left(\frac{u}{v}\right)=\frac{v \frac{d}{d x}(u)-u \frac{d}{d x}(v)}{v^2}\right]\)

Question 2. \(e^{sin^{-1} x}\)

Solution:

Let y=\(e^{sin x^{-1}}\)

By using the chain rule, we obtain

⇒ \(\frac{d y}{d x}=\frac{d}{d x}\left(e^{n=-1}\right)\)

= \(\frac{d y}{d x}=e^{n+1}+\frac{d}{d x}\left(\sin ^{-1} x\right)\)

= \(\frac{e^{-n^{-1}} x}{\sqrt{1-x^2}}\)

⇒ \(\frac{d y}{d x}=\frac{e^{m n^{-1} x}}{\sqrt{1-x^2}}, x \in(-1,1)\)

Question 3. \(e^{x^2}\)

Solution:

Let y=\(e^{x^2}\) (By using the chain rule, we obtain)

⇒ \(\frac{d y}{d x}=\frac{d}{d x}\left(e^{x^{\prime}}\right)=e^{x^3} \cdot 3 x^2=3 x^2 e^{x^{\prime}}\)

Question 4. \(\sin \left(\tan ^{-1} e^{-x}\right)\)

Solution:

Let y=\(\sin \left(\tan ^{-1} e^{-\pi}\right)\) (By using chain rule, we obtain)

⇒ \(\frac{d y}{d x}=\frac{d}{d x}\left[\sin \left(\tan ^{-1} e^{-x}\right)\right]\)

=\(\cos \left(\tan ^{-1} e^{-x}\right) \cdot \frac{d}{d x}\left(\tan ^{-t} e^{-1}\right)\)

=\(\cos \left(\tan ^{-1} e^{-x}\right) \frac{1}{1+\left(e^{-1}\right)^2} \cdot \frac{d}{d x}\left(e^{-x}\right)\)

=\(\frac{\cos \left(\tan ^{-1} e^{-1}\right)}{1+e^{-2 x}} \cdot e^{-x} \cdot \frac{d}{d x}(-x)\)

=\(\frac{e^{-3} \cos \left(\tan ^{-1} e^{-1}\right)}{1+e^{-t x}} \times(-1)=\frac{-e^{-1} \cos \left(\tan ^{-1} e^{-t}\right)}{1+e^{-5 x}}\)

Question 5. \(\sin \left(\tan ^{-1} e^{-x}\right)\)

Solution:

Let y=\(\sin \left(\tan ^{-1} e^{-x}\right)\)(By using chain rule, we obtain)

⇒ \(\frac{d y}{d x} =\frac{d}{d x}\left[\sin \left(\tan ^{-1} e^{-1}\right)\right]\)

= \(\cos \left(\tan ^{-1} e^{-x}\right) \cdot \frac{d}{d x}\left(\tan ^{-1} e^{-1}\right)\)

= \(\cos \left(\tan ^{-1} e^{-1}\right) \frac{1}{1+\left(e^{-x}\right)} \cdot \frac{d}{d x}\left(e^{-x}\right)\)

= \(\frac{\cos \left(\tan ^{-1} e^{-1}\right)}{1+e^{-2 x}} \cdot e^{-x} \cdot \frac{d}{d x}(-x)\)

= \(\frac{e^{-1} \cos \left(\tan ^{-1} e^{-1}\right)}{1+e^{-3 x}} \times(-1)=\frac{-e^{-1} \cos \left(\tan ^{-1} e^{-x}\right)}{1+e^{-3 x}}\)

Question 6. \(e^x+e^{x^2}+\ldots .+e^{x^2}\)

Solution:

⇒ \(\frac{d}{d x}\left(e^x+e^{x^2}+\ldots .+e^{x^1}\right)\)

= \(\frac{d}{d x}\left(e^x\right)+\frac{d}{d x}\left(e^{x^2}\right)+\frac{d}{d x}\left(e^{x^3}\right)+\frac{d}{d x}\left(e^{x^4}\right)+\frac{d}{d x}\left(e^{x^2}\right)\)

=\(e^x+\left[e^{x^2} \times \frac{d}{d x}\left(x^2\right)\right]+\left[e^{x^1} \cdot \frac{d}{d x}\left(x^3\right)\right]+\left[e^{x^4} \cdot \frac{d}{d x}\left(x^4\right)\right]+\left[e^{x^3} \cdot \frac{d}{d x}\left(x^5\right)\right]\)

= \(e^x+\left(e^{x^1} \times 2 x\right)+\left(e^{x^3} \times 3 x^2\right)+\left(e^{x^4} \times 4 x^3\right)+\left(e^{x^3} \times 5 x^4\right)\)

=\(e^x+2 e^{x^1}+3 x^2 e^{x^1}+4 x^3 e^{x^1}+5 x^4 e^{x^5}\)

Question 7. \(\sqrt{e^{\sqrt{x}}}, x>0\)

Solution:

Let y=\(\sqrt{e^{\sqrt{x}}}\)

Then, \(y^2=e^{\sqrt{x}}\) (By differentiating this relationship with respect to x, we obtain)

2 y \(\frac{d y}{d x}=e^{\sqrt{x}} \frac{d}{d x}(\sqrt{x})\) [By applying chain rule]

2 y \(\frac{d y}{d x}=e^{\sqrt{x}} \frac{1}{2} \cdot \frac{1}{\sqrt{x}}\)

⇒ \(\frac{d y}{d x}\)

= \(\frac{e^{\sqrt{x}}}{4 y \sqrt{x}}\)

⇒ \(\frac{d y}{d x}=\frac{e^{\sqrt{x}}}{4 \sqrt{e^{\sqrt{x}}} \sqrt{x}}\)

⇒ \(\frac{d y}{d x}=\frac{e^{\sqrt{x}}}{4 \sqrt{x e^{\sqrt{x}}}}\), x>0

Question 8. \(\log (\log x)\), x>1

Solution:

Let y =\(\log (\log \mathrm{x})\) (By using chain rule, we obtain)

⇒ \(\frac{d y}{d x}=\frac{d}{d x}[\log (\log x)]=\frac{1}{\log x} \cdot \frac{d}{d x}(\log x)\)

=\(\frac{1}{\log x} \cdot \frac{1}{x}=\frac{1}{x \log x}, x>1\)

Question 9. \(\frac{\cos x}{\log x}\), x>0

Solution:

Let y=\(\frac{\cos x}{\log x}\)

By using the quotient rule, we obtain

⇒ \(\frac{d y}{d x}=\frac{\log x \frac{d}{d x}(\cos x)-\cos x \times \frac{d}{d x}(\log x)}{(\log x)^2}\)

=\(\frac{-\sin x \log x-\cos x \times \frac{1}{x}}{(\log x)^2}\)

=\(\frac{-[x \log x \cdot \sin x+\cos x]}{x(\log x)^2}\), x>0

Question 10. \(\cos \left(\log x+e^x\right)\), x>0

Solution:

Let y=\(\cos \left(\log x+e^x\right)\) (By using chain rule, we obtain)

⇒ \(\frac{d y}{d x} =\frac{d}{d x}\left\{\cos \left(\log x+e^x\right)\right\}\)

⇒ \(\frac{d y}{d x} =-\sin \left(\log x+e^x\right) \cdot \frac{d}{d x}\left(\log x+e^x\right)\)

=-\(\sin \left(\log x+e^x\right) \cdot\left(\frac{1}{x}+e^x\right)\)

=-\(\left(\frac{1}{x}+e^x\right) \sin \left(\log x+e^x\right)\), x>0

Continuity And Differentiability Exercise 5.5

Question 1. \(\cos \mathrm{x} \cdot \cos 2 \mathrm{x} \cdot \cos 3 \mathrm{x}\)

Solution:

Let y=\(\cos \mathrm{x} \cdot \cos 2 \mathrm{x} \cdot \cos 3 \mathrm{x}\)

Taking logarithm on both the sides \(\log y=\log (\cos x \cdot \cos 2 x \cdot \cos 3 x)\)

⇒ \(\log y=\log (\cos x)+\log (\cos 2 x)+\log (\cos 3 x)\)

Differentiating both sides with respect to x, we obtain

⇒ \(\frac{1}{y d x}=\frac{1}{\cos x} \cdot \frac{d}{d x}(\cos x)+\frac{1}{\cos 2 x} \cdot \frac{d}{d x}(\cos 2 x)+\frac{1}{\cos 3 x} \cdot \frac{d}{d x}(\cos 3 x)\)

⇒ \(\frac{d y}{d x}=y\left[-\frac{\sin x}{\cos x}-\frac{\sin 2 x}{\cos 2 x} \cdot \frac{d}{d x}(2 x)-\frac{\sin 3 x}{\cos 3 x} \cdot \frac{d}{d x}(3 x)\right]\)

⇒ \(\frac{d y}{d x}=-\cos x \cdot \cos 2 x \cdot \cos 3 x[\tan x+2 \tan 2 x+3 \tan 3 x]\)

Question 2. \(\sqrt{\frac{(x-1)(x-2)}{(x-3)(x-4)(x-5)}}\)

Solution:

Let y=\(\sqrt{\frac{(x-1)(x-2)}{(x-3)(x-4)(x-5)}}\)

Taking logarithm on both the sides \(\log y=\log \sqrt{\frac{(x-1)(x-2)}{(x-3)(x-4)(x-5)}}\)

⇒ \(\log y=\frac{1}{2} \log \left[\frac{(x-1)(x-2)}{(x-3)(x-4)(x-5)}\right]\)

⇒ \(\log y=\frac{1}{2}[\log \{(x-1)(x-2)\}-\log \{(x-3)(x-4)(x-5)\}]\)

⇒ \(\log y=\frac{1}{2}[\log (x-1)+\log (x-2)-\log (x-3)-\log (x-4)-\log (x-5)]\)

Differentiating both sides with respect to x, we obtain

⇒ \(\frac{1}{y} \frac{d y}{d x}=\frac{1}{2}\left[\begin{array}{l}
\frac{1}{x-1} \cdot \frac{d}{d x}(x-1)+\frac{1}{x-2} \cdot \frac{d}{d x}(x-2)-\frac{1}{x-3} \cdot \frac{d}{d x}(x-3) \\
-\frac{1}{x-4} \cdot \frac{d}{d x}(x-4)-\frac{1}{x-5} \cdot \frac{d}{d x}(x-5)
\end{array}\right]\)

⇒ \(\frac{d y}{d x}=\frac{y}{2}\left(\frac{1}{x-1}+\frac{1}{x-2}-\frac{1}{x-3}-\frac{1}{x-4}-\frac{1}{x-5}\right)\)

⇒ \(\frac{d y}{d x}=\frac{1}{2} \sqrt{(x-1)(x-2)}\left[\frac{1}{(x-3)(x-4)(x-5)}+\frac{1}{x-2}-\frac{1}{x-3}-\frac{1}{x-4}-\frac{1}{x-5}\right]\)

Question 3. \((\log x)^{\cos x}\)

Solution:

Let y=\((\log x)^{\cos x}\)

Taking logarithm on both the sides \(\log y=\cos x \cdot \log (\log x)\)

Differentiating both sides with respect to x, we obtain

⇒ \(\frac{1}{y} \cdot \frac{d y}{d x}=\frac{d}{d x}(\cos x) \times \log (\log x)+\cos x \times \frac{d}{d x}[\log (\log x)]\)

⇒ \(\frac{1}{y} \cdot \frac{d y}{d x}=-\sin x \log (\log x)+\cos x \times \frac{1}{\log x} \cdot \frac{d}{d x}(\log x)\)

⇒ \(\frac{d y}{d x}=y\left[-\sin x \log (\log x)+\frac{\cos x}{\log x} \times \frac{1}{x}\right]\)

∴ \(\frac{d y}{d x}=(\log x)^{\cos x}\left[\frac{\cos x}{x \log x}-\sin x \log (\log x)\right]\)

Question 4. \(x^x-2^{\sin x}\)

Solution:

Let y=\(x^x-2^{\sin x}\)

Also, let \(\mathrm{x}^{\mathrm{x}}=\mathrm{u} and 2^{\sin \mathrm{x}}=\mathrm{v}\)

y=\(\mathrm{u}-\mathrm{v}\)

⇒ \(\frac{\mathrm{dy}}{\mathrm{dx}}=\frac{\mathrm{du}}{\mathrm{dx}}-\frac{\mathrm{dv}}{\mathrm{dx}}\) →  Equation 1

⇒ \(\mathrm{u}=\mathrm{x}^{\mathrm{x}}\)

Taking logarithm on both sides log u = x log x

Differentiating both sides with respect to x, we obtain

⇒ \(\frac{1}{\mathrm{u}} \frac{\mathrm{du}}{\mathrm{dx}}=\left[\frac{\mathrm{d}}{\mathrm{dx}}(\mathrm{x}) \times \log \mathrm{x}+\mathrm{x} \times \frac{\mathrm{d}}{\mathrm{dx}}(\log x)\right]\)

⇒ \(\frac{\mathrm{du}}{\mathrm{dx}}=\mathrm{u}\left[1 \times \log \mathrm{x}+\mathrm{x} \times \frac{1}{\mathrm{x}}\right]\)

⇒ \(\frac{\mathrm{du}}{\mathrm{dx}}=\mathrm{x}^{\mathrm{x}}(1+\log \mathrm{x})\) →  Equation 2

⇒ \(\mathrm{v}=2^{\sin x}\)

⇒ \(\frac{1}{\mathrm{u} d \mathrm{du}}=\left[\frac{\mathrm{d}}{\mathrm{dx}}(\mathrm{x}) \times \log \mathrm{x}+\mathrm{x} \times \frac{\mathrm{d}}{\mathrm{dx}}(\log x)\right]\)

⇒ \(\frac{\mathrm{du}}{\mathrm{dx}}=\mathrm{u}\left[1 \times \log \mathrm{x}+\mathrm{x} \times \frac{1}{\mathrm{x}}\right]\)

⇒ \(\frac{\mathrm{du}}{\mathrm{dx}}=\mathrm{x}^{\mathrm{x}}(1+\log \mathrm{x})\)

⇒ \(\mathrm{v}=2^{\sin x}\)

Taking logarithm on both the sides \(\log v=\sin x \cdot \log 2\)

Differentiating both sides with respect to x, we obtain

⇒ \(\frac{1}{v} \cdot \frac{d v}{d x}=\log 2 \cdot \frac{d}{d x}(\sin x)\)

⇒ \(\frac{d v}{d x}=v(\log 2 \cos x) \Rightarrow \frac{d v}{d x}=2^{\sin x}(\cos x \log 2)\) Equation 3

Therefore, from equations (1), (2) and (3), we obtain

⇒ \(\frac{\mathrm{dy}}{\mathrm{dx}}=\mathrm{x}^{\mathrm{x}}(1+\log \mathrm{x})-2^{\sin \mathrm{x}} \cos \mathrm{x} \log 2\)

Question 5. \((x+3)^2 \cdot(x+4)^3 \cdot(x+5)^4\)

Solution:

Let \(y=(x+3)^2 \cdot(x+4)^3 \cdot(x+5)^4\)

Taking logarithms on both sides

⇒ \(\log y=\log (x+3)^2+\log (x+4)^3+\log (x+5)^4\)

∴ \(\log y=2 \log (x+3)+3 \log (x+4)+4 \log (x+5)\)

Differentiating both sides with respect to x, we obtain

⇒ \(\frac{1}{y} \cdot \frac{d y}{d x}=2 \cdot \frac{1}{x+3} \cdot \frac{d}{d x}(x+3)+3 \cdot \frac{1}{x+4} \cdot \frac{d}{d x}(x+4)+4 \cdot \frac{1}{x+5} \cdot \frac{d}{d x}(x+5)\)

⇒ \(\frac{d y}{d x}=y\left[\frac{2}{x+3}+\frac{3}{x+4}+\frac{4}{x+5}\right]\)

⇒ \(\frac{d y}{d x}=(x+3)^2(x+4)^3(x+5)^4 \cdot\left[\frac{2}{x+3}+\frac{3}{x+4}+\frac{4}{x+5}\right]\)

⇒ \(\frac{d y}{d x}=(x+3)^2(x+4)^3(x+5)^4 \cdot\left[\frac{2(x+4)(x+5)+3(x+3)(x+5)+4(x+3)(x+4)}{(x+3)(x+4)(x+5)}\right]\)

⇒ \(\frac{d y}{d x}=(x+3)(x+4)^2(x+5)^3 \cdot\left[2\left(x^2+9 x+20\right)+3\left(x^2+8 x+15\right)+4\left(x^2+7 x+12\right)\right]\)

⇒ \(\frac{d y}{d x}=(x+3)(x+4)^2(x+5)^3\left(9 x^2+70 x+133\right)\)

∴ \(\left.\left(x+\frac{1}{x}\right)^x+x^x+\frac{1}{x}\right)\)

Question 6. \(\left(x+\frac{1}{x}\right)^x+x^{\left(1+\frac{1}{x}\right)}\)

Solution:

Let y=\(\left(x+\frac{1}{x}\right)^x+x^{\left(1+\frac{1}{x}\right)}\)

Also, let \(mathrm{u}=\left(\mathrm{x}+\frac{1}{\mathrm{x}}\right)^{\mathrm{x}} and \mathrm{v}=\mathrm{x}^{\left(1+\frac{1}{\mathrm{x}}\right)}\)

y=u+v

⇒ \(\frac{d y}{d x} =\frac{d u}{d x}+\frac{d v}{d x}\)

u =\(\left(x+\frac{1}{x}\right)^x\)

Taking logarithms on both sides

⇒ \(\log u=\log \left(x+\frac{1}{x}\right)^x \Rightarrow \log u=x \log \left(x+\frac{1}{x}\right)\)

Differentiating both sides with respect to x, we obtain

⇒ \(\frac{1}{u} \cdot \frac{d u}{d x}=\frac{d}{d x}(x) \times \log \left(x+\frac{1}{x}\right)+x \times \frac{d}{d x}\left[\log \left(x+\frac{1}{x}\right)\right]\)

⇒ \(\frac{1}{u} \frac{d u}{d x}=1 \times \log \left(x+\frac{1}{x}\right)+x \times \frac{1}{\left(x+\frac{1}{x}\right)} \cdot \frac{d}{d x}\left(x+\frac{1}{x}\right)\)

⇒ \(\frac{d u}{d x}=u\left[\log \left(x+\frac{1}{x}\right)+\frac{x}{\left(x+\frac{1}{x}\right)} \times\left(1-\frac{1}{x^2}\right)\right]\)

⇒ \(\frac{d u}{d x}=\left(x+\frac{1}{x}\right)^x\left[\log \left(x+\frac{1}{x}\right)+\frac{\left(x-\frac{1}{x}\right)}{\left(x+\frac{1}{x}\right)}\right]\)

⇒ \(\frac{d u}{d x}=\left(x+\frac{1}{x}\right)^x\left[\log \left(x+\frac{1}{x}\right)+\frac{x^2-1}{x^2+1}\right]\)

⇒ \(\frac{d u}{d x}=\left(x+\frac{1}{x}\right)^x\left[\frac{x^2-1}{x^2+1}+\log \left(x+\frac{1}{x}\right)\right]\)

⇒ \(x^{\left(1+\frac{1}{x}\right)}\)

Taking logarithms on both the sides \(\log v=\log \left[x^{\left(1+\frac{1}{x}\right)}\right]\)

⇒ \(\log v=\left(1+\frac{1}{x}\right) \log x\) Differentiating both the sides with respect to x, we obtain

⇒ \(\frac{1}{v} \cdot \frac{d v}{d x}=\left[\frac{d}{d x}\left(1+\frac{1}{x}\right)\right] \times \log x+\left(1+\frac{1}{x}\right) \cdot \frac{d}{d x}(\log x)\)

⇒ \(\frac{1}{v} \frac{d v}{d x}=\left(-\frac{1}{x^2}\right) \log x+\left(1+\frac{1}{x}\right) \cdot \frac{1}{x} \Rightarrow \frac{1}{v} \frac{d v}{d x}=-\frac{\log x}{x^2}+\frac{1}{x}+\frac{1}{x^2}\)

⇒ \(\frac{d v}{d x}=v\left[\frac{-\log x+x+1}{x^2}\right] \Rightarrow \frac{d v}{d x}=x^{\left(1+\frac{1}{v}\right)}\left(\frac{x+1-\log x}{x^2}\right)\)

Therefore, from (1), (2), and (3), we obtain

∴ \(\frac{d y}{d x}=\left(x+\frac{1}{x}\right)^x\left[\frac{x^2-1}{x^2+1}+\log \left(x+\frac{1}{x}\right)\right]+x^{\left(1+\frac{1}{x}\right)}\left(\frac{x+1-\log x}{x^2}\right)\)

Question 7. \((\log x)^x+x^{\log x}\)

Solution:

Let y=\((\log x)^x+x^{\log x}\)

Also, let u=\((\log x)^x and v=x^{\log x}\)

y=u+v

⇒ \(\frac{d y}{d x}=\frac{d u}{d x}+\frac{d v}{d x}\)

u=\((\log x)^x\)

Taking logarithm on both the sides \(\log u=\log \left[(\log x)^x\right]\)

⇒ \(\log u=x \log (\log x)\) Differentiating both sides with respect to x, we obtain

⇒ \(\frac{1}{u} \frac{d u}{d x}=\frac{d}{d x}(x) \times \log (\log x)+x \cdot \frac{d}{d x}[\log (\log x)]\)

⇒ \(\frac{d u}{d x}=u\left[1 \times \log (\log x)+x \cdot \frac{1}{\log x} \cdot \frac{d}{d x}(\log x)\right]\)

⇒ \(\frac{d u}{d x}=(\log x)^x\left[\log (\log x)+\frac{x}{\log x} \cdot \frac{1}{x}\right]=(\log x)^x\left[\log (\log x)+\frac{1}{\log x}\right]\)

⇒ \(\frac{d u}{d x}=(\log x)^x\left[\frac{\log (\log x) \cdot \log x+1}{\log x}\right]=(\log x)^{x-1}[1+\log x \cdot \log (\log x)]\)

v=\(x^{\log x}\)

Taking logarithm on both the sides \(\log v=\log \left(x^{\log x}\right)\)

⇒ \(\log v=\log x \log x=(\log x)^2\)

Differentiating both sides w.r.t.x, we obtain

⇒ \(\frac{1}{v} \cdot \frac{d v}{d x}=\frac{d}{d x}\left[(\log x)^2\right]\)

⇒ \(\frac{1}{v} \cdot \frac{d v}{d x}=2(\log x) \cdot \frac{d}{d x}(\log x) \Rightarrow \frac{d v}{d x}=2 v(\log  x) \cdot \frac{1}{x}\)

⇒ \(\frac{d v}{d x}=2 x^{\log x} \frac{\log x}{x} \Rightarrow \frac{d v}{d x}=2 x^{\log x-1} \cdot \log x\)

Therefore, from (1), (2), and (3), we obtain

∴ \(\frac{d y}{d x}=(\log x)^{x-1}[1+\log x \cdot \log (\log x)]+2 x^{\log x-1} \cdot \log x\)

Question 8. \((\sin x)^x+\sin ^{-1} \sqrt{x}\)

Solution:

Let y=\((\sin x)^x+\sin ^{-1} \sqrt{x}\)

⇒Also, let u=\((\sin x)^x and v=\sin ^{-1} \sqrt{x}\)

\(\mathrm{y}=\mathrm{u}+\mathrm{v}\)

⇒ \(\frac{\mathrm{dy}}{\mathrm{dx}}=\frac{\mathrm{du}}{\mathrm{dx}}+\frac{\mathrm{dv}}{\mathrm{dx}}\)  → Equation 1

⇒ \(\mathrm{u}=(\sin \mathrm{x})^{\mathrm{x}}\)

Taking logarithm on both the sides \(\log \mathrm{u}=\log (\sin \mathrm{x})^{\mathrm{x}} \Rightarrow \log \mathrm{u}=\mathrm{x} \log (\sin \mathrm{x})\)

Differentiating both sides with respect to x, we obtain

⇒ \(\frac{1}{u} \frac{d u}{d x}=\frac{d}{d x}(x) \times \log (\sin x)+x \times \frac{d}{d x}[\log (\sin x)]\)

⇒ \(\frac{d u}{d x}=u\left[1 \cdot \log (\sin x)+x \cdot \frac{1}{\sin x} \cdot \frac{d}{d x}(\sin x)\right]\)

⇒ \(\frac{d u}{d x}=(\sin x)^x\left[\log (\sin x)+\frac{x}{\sin x} \cdot \cos x\right]=(\sin x)^x(x \cot x+\log \sin x)\)  → Equation 2

v=\(\sin ^{-1} \sqrt{x}\)

Differentiating both sides with respect to x, we obtain

⇒ \(\frac{\mathrm{dv}}{\mathrm{dx}}=\frac{1}{\sqrt{1-(\sqrt{\mathrm{x}})^2}} \cdot \frac{\mathrm{d}}{\mathrm{dx}}(\sqrt{\mathrm{x}})\)

=\(\frac{1}{\sqrt{1-\mathrm{x}}} \cdot \frac{1}{2 \sqrt{\mathrm{x}}}=\frac{1}{2 \sqrt{\mathrm{x}-\mathrm{x}^2}}\) ⇒ Equation 3

Therefore, from (1), (2), and (3), we obtain \(\frac{d y}{d x}=(\sin x)^x(x \cot x+\log \sin x)+\frac{1}{2 \sqrt{x-x^2}}\)

Question 9. \(x^{\sin x}+(\sin x)^{\cos x}\)

Solution:

Let y=\(x^{\sin x}+(\sin x)^{\cos x}\)

Also, let \(\mathrm{u}=\mathrm{x}^{\sin \mathrm{x}}\) and \(\mathrm{v}=(\sin \mathrm{x})^{\cos \mathrm{x}}\)

⇒ \(\mathrm{y}=\mathrm{u}+\mathrm{v}\)

⇒ \(\frac{d y}{d x}=\frac{d u}{d x}+\frac{d v}{d x}\) →  Equation 1

⇒ \(u=x^{\sin x}\)

Taking logarithm on both the sides \(\log \mathrm{u}=\log \left(\mathrm{x}^{\sin \mathrm{x}}\right)\)

⇒ \(\log \mathrm{u}=\sin \mathrm{x} \log \mathrm{x}\) Differentiating both sides with respect to x, we obtain

⇒ \(\frac{1}{\mathrm{u} d \mathrm{du}}=\frac{\mathrm{d}}{\mathrm{dx}}(\sin \mathrm{x}) \cdot \log \mathrm{x}+\sin x \cdot \frac{\mathrm{d}}{\mathrm{dx}}(\log x)\)

⇒ \(\frac{\mathrm{du}}{\mathrm{dx}}=\mathrm{u}\left[\cos x \log x+\sin x \cdot \frac{1}{x}\right]\)

⇒ \(\frac{\mathrm{du}}{\mathrm{dx}}=x^{\sin x}\left[\cos x \log x+\frac{\sin x}{x}\right]\) . Equation 2

⇒ \(\mathrm{v}=(\sin x)^{\cos x}\)

Taking logarithm on both the sides \(\log \mathrm{v}=\log (\sin x)^{\cos x} \log \mathrm{y}=\cos x \log (\sin x)\)

Differentiating both sides with respect to x, we obtain

⇒ \(\frac{1}{v d v}=\frac{d}{d x}(\cos x) \times \log (\sin x)+\cos x \times \frac{d}{d x}[\log (\sin x)]\)

⇒ \(\frac{d v}{d x}=v\left[-\sin x \cdot \log (\sin x)+\cos x \cdot \frac{1}{\sin x} \cdot \frac{d}{d x}(\sin x)\right]\)

⇒ \(\frac{d v}{d x}=(\sin x)^{\cos x}\left[-\sin x \log \sin x+\frac{\cos x}{\sin x} \cos x\right]=(\sin x)^{\cos x}[-\sin x \log \sin x+\cot x \cos x]\) →  Equation 3

⇒ \(\frac{d v}{d x}=(\sin x)^{\cos x}[\cot x \cos x-\sin x \log \sin x]]\)

From (1), (2), and (3), we obtain

∴ \(\frac{d y}{d x}=x^{\sin x}\left(\cos x \log x+\frac{\sin x}{x}\right)+(\sin x)^{\cos x}[\cos x \cot x-\sin x \log \sin x]\)

Question 10. \(x^{x \cos x}+\frac{x^2+1}{x^2-1}\)

Solution:

Let y=\(x^{x \cos x}+\frac{x^2+1}{x^2-1}\)

Also, let \(\mathrm{u}=\mathrm{x}^{\mathrm{x} \cos \mathrm{x}} and \mathrm{v}=\frac{\mathrm{x}^2+1}{\mathrm{x}^2-1}\)  → Equation 1

⇒ \(\mathrm{y}=\mathrm{u}+\mathrm{v}\)

⇒ \(\frac{\mathrm{dy}}{\mathrm{dx}}=\frac{\mathrm{du}}{\mathrm{dx}}+\frac{\mathrm{dv}}{\mathrm{dx}}\)

⇒ \(\mathrm{u}=\mathrm{x}^{\mathrm{xec} x}\)

Taking logarithm on both the sides \(\log \mathrm{u}=\log \left(\mathrm{x}^{\mathrm{x} {cosx}}\right)\)

⇒ \(\log \mathrm{u}=\mathrm{x} \cos \mathrm{x} \log \mathrm{x} \)

Differentiating both sides with respect to x, we obtain

⇒ \(\frac{1}{u} \frac{d u}{d x}=\frac{d}{d x}(x) \cdot \cos x \cdot \log x+x \cdot \frac{d}{d x}(\cos x) \cdot \log x+x \cos x \cdot \frac{d}{d x}(\log x)\)

⇒ \(\frac{d u}{d x}=u\left[1 \cdot \cos x \cdot \log x+x \cdot(-\sin x) \log x+x \cos x \cdot \frac{1}{x}\right]\)

⇒ \(\frac{d u}{d x}=x^{x \cos x}(\cos x \log x-x \sin x \log x+\cos x)\)  → Equation 2

⇒ \(\frac{d u}{d x}=x^{x \cos x}[\cos x(1+\log x)-x \sin x \log x]\)

v =\(\frac{x^2+1}{x^2-1}\)

Taking logarithm on both the sides \(\log v=\log \left(x^2+1\right)-\log \left(x^2-1\right)\)

Differentiating both sides with respect to x, we obtain

⇒ \(\frac{1}{v} \frac{d v}{d x}=\frac{2 x}{x^2+1}-\frac{2 x}{x^2-1}\)

⇒ \(\frac{d v}{d x}=v\left[\frac{2 x\left(x^2-1\right)-2 x\left(x^2+1\right)}{\left(x^2+1\right)\left(x^2-1\right)}\right]\)

⇒ \(\frac{d v}{d x}=\frac{x^2+1}{x^2-1} \times\left[\frac{-4 x}{\left(x^2+1\right)\left(x^2-1\right)}\right]\)

⇒ \(\frac{d v}{d x}=\frac{-4 x}{\left(x^2-1\right)^2}\)

Therefore, from (1), (2), and (3), we obtain

⇒ \(\frac{d y}{d x}=x^{x \cos x}[\cos x(1+\log x)-x \sin x \log x]-\frac{4 x}{\left(x^2-1\right)^2}\)

Question 11. \((x \cos x)^x+(x \sin x)^{\frac{1}{x}}\)

Solution:

Let y=\((x \cos x)^x+(x \sin x)^{\frac{1}{x}}\)

Also, let u=\((x \cos x)^x and v=(x \sin x)^{\frac{1}{x}}\)

⇒ \(\mathrm{y}=\mathrm{u}+\mathrm{v}\)

⇒ \(\frac{\mathrm{dy}}{\mathrm{dx}}=\frac{\mathrm{du}}{\mathrm{dx}}+\frac{\mathrm{dv}}{\mathrm{dx}}\)

⇒ \(\mathrm{u}=(\mathrm{x} \cos \mathrm{x})^{\mathrm{x}}\)(Differentiating both sides w.r.t. x, we obtain)

Taking logarithms on both sides

⇒ \(\log u =\log (x \cos x)^x\)

⇒ \(\log u =x \log (x \cos x) \Rightarrow \log u=x[\log x+\log \cos x] \log u=x \log x+x \log \cos x\)

Differentiating both sides with respect to x, we obtain

⇒ \(\frac{1}{\mathrm{u}}\)

⇒ \(\frac{\mathrm{du}}{\mathrm{dx}}=\frac{\mathrm{d}}{\mathrm{dx}}(\mathrm{x} \log \mathrm{x})+\frac{\mathrm{d}}{\mathrm{dx}}(\mathrm{x} \log \cos \mathrm{x})\) .

⇒ \(\frac{\mathrm{du}}{\mathrm{dx}}=\mathrm{u}\left[\left\{\log \mathrm{x} \cdot \frac{\mathrm{d}}{\mathrm{dx}}(\mathrm{x})+\mathrm{x} \cdot \frac{\mathrm{d}}{\mathrm{dx}}(\log \mathrm{x})\right\}+\left\{\log \cos \mathrm{x} \cdot \frac{\mathrm{d}}{\mathrm{dx}}(\mathrm{x})+\mathrm{x} \cdot \frac{\mathrm{d}}{\mathrm{dx}}(\log \cos \mathrm{x})\right\}\right]\)

⇒ \(\frac{\mathrm{du}}{\mathrm{dx}}=(\mathrm{x} \cos \mathrm{x})^{\mathrm{x}}\left[\left(\log \mathrm{x} \cdot 1+\mathrm{x} \cdot \frac{1}{\mathrm{x}}\right)+\left\{\log \cos \mathrm{x} \cdot 1+\mathrm{x} \cdot \frac{1}{\cos \mathrm{x}} \cdot \frac{\mathrm{d}}{\mathrm{dx}}(\cos \mathrm{x})\right\}\right]\)

⇒ \(\frac{\mathrm{du}}{\mathrm{dx}}=(\mathrm{x} \cos \mathrm{x})^{\mathrm{x}}\left[(\log \mathrm{x}+1)+\left\{\log \cos \mathrm{x}+\frac{\mathrm{x}}{\cos \mathrm{x}} \cdot(-\sin \mathrm{x})\right\}\right]\)

⇒ \(\frac{\mathrm{du}}{\mathrm{dx}}=(\mathrm{x} \cos \mathrm{x})^{\mathrm{x}}[(1+\log \mathrm{x})+(\log \cos \mathrm{x}-\mathrm{x} \tan \mathrm{x})]\)

⇒ \(\frac{\mathrm{du}}{\mathrm{dx}}=(\mathrm{x} \cos \mathrm{x})^{\mathrm{x}}[1-\mathrm{x} \tan \mathrm{x}+(\log \mathrm{x}+\log \cos \mathrm{x})]\)

⇒ \(\frac{d u}{d x}=(x \cos x)^x[1-x \tan x+\log (x \cos x)]\)

⇒ \(\mathrm{v}=(\mathrm{x} \sin \mathrm{x})^{\frac{1}{\mathrm{x}}}\)

Taking logarithms on both sides

⇒ \(\log \mathrm{v}=\log (\mathrm{x} \sin \mathrm{x})^{\frac{1}{\mathrm{x}}}\)

⇒ \(\log \mathrm{v}=\frac{1}{\mathrm{x}} \log (\mathrm{x} \sin \mathrm{x}) \Rightarrow \log \mathrm{v}\)

=\(\frac{1}{\mathrm{x}}(\log \mathrm{x}+\log \sin \mathrm{x})\)

⇒ \(\log \mathrm{v}=\frac{1}{\mathrm{x}} \log \mathrm{x}+\frac{1}{\mathrm{x}} \log \sin \mathrm{x}\)

Differentiating both sides with respect to x

⇒ \(\frac{1}{v} \frac{d v}{d x}=\frac{d}{d x}\left(\frac{1}{x} \log x\right)+\frac{d}{d x}\left[\frac{1}{x} \log (\sin x)\right]\)

⇒ \(\frac{1}{v} \frac{d v}{d x}=\left[\log x \cdot \frac{d}{d x}\left(\frac{1}{x}\right)+\frac{1}{x} \cdot \frac{d}{d x}(\log x)\right]+\left[\log (\sin x) \cdot \frac{d}{d x}\left(\frac{1}{x}\right)+\frac{1}{x} \cdot \frac{d}{d x}\{\log (\sin x)\}\right]\)

⇒ \(\frac{1}{v} \frac{d v}{d x}=\left[\log x \cdot\left(-\frac{1}{x^2}\right)+\frac{1}{x} \cdot \frac{1}{x}\right]+\left[\log (\sin x) \cdot\left(-\frac{1}{x^2}\right)+\frac{1}{x} \cdot \frac{1}{\sin x} \cdot \frac{d}{d x}(\sin x)\right]\)

⇒ \(\frac{1}{v} \frac{d v}{d x}=\frac{1}{x^2}(1-\log x)+\left[-\frac{\log (\sin x)}{x^2}+\frac{1}{x \sin x} \cdot \cos x\right]\)

⇒ \(\frac{d v}{d x}=(x \sin x)^{\frac{1}{x}}\left[\frac{1-\log x}{x^2}+\frac{-\log (\sin x)+x \cot x}{x^2}\right]\)

⇒ \(\frac{d v}{d x}=(x \sin x)^{\frac{1}{x}}\left[\frac{1-\log x-\log (\sin x)+x \cot x}{x^2}\right]\)

⇒ \(\frac{d v}{d x}=(x \sin x)^{\frac{1}{x}}\left[\frac{1-\log (x \sin x)+x \cot x}{x^2}\right]\)

From (1), (2), and (3), we obtain

∴ \(\frac{d y}{d x}=(x \cos x)^x[1-x \tan x+\log (x \cos x)]+(x \sin x)^{\frac{1}{x}}\left[\frac{x \cot x+1-\log (x \sin x)}{x^2}\right]\)

Question 12. \(x^y+y^x=1\)

Solution:

The given function is \(\mathrm{x}^y+\mathrm{y}^{\mathrm{x}}\)=1.

Let \(\mathrm{x}^{\mathrm{y}}=\mathrm{u} and\mathrm{y}^{\mathrm{x}}=\mathrm{v}\)

Then, the function becomes \mathrm{u}+\mathrm{v}=1

⇒ \(\frac{\mathrm{du}}{\mathrm{dx}}+\frac{\mathrm{dv}}{\mathrm{dx}}\)=0

⇒ \(\mathrm{u} =\mathrm{x}^{\mathrm{y}}\)

Taking logarithm on both the sides \(\log u=\log \left(x^y\right) \Rightarrow \log u=y \log x \)

Differentiating both sides with respect to x, we obtain

⇒ \(\mathrm{v}=\mathrm{y}^{\mathrm{x}}\)

⇒ \(\mathrm{v}=\mathrm{y}^{\mathrm{x}}\)

Taking logarithm on both the sides \(\log v=\log \left(y^x\right) \Rightarrow \log v=x \log y\)

Differentiating both sides with respect to x, we obtain

⇒ \(\frac{1 \mathrm{du}}{\mathrm{u} x}=\log x \frac{\mathrm{dy}}{\mathrm{dx}}+\mathrm{y} \cdot \frac{\mathrm{d}}{\mathrm{dx}}(\log \mathrm{x}) \Rightarrow \frac{\mathrm{du}}{\mathrm{dx}}\)

= \(\mathrm{u}\left[\log \mathrm{x} \frac{\mathrm{dy}}{\mathrm{dx}}+\mathrm{y} \cdot \frac{1}{\mathrm{x}}\right] \Rightarrow \frac{\mathrm{du}}{\mathrm{dx}}\)

= \(\mathrm{x}^y\left(\log \mathrm{x} \frac{\mathrm{dy}}{\mathrm{dx}}+\frac{\mathrm{y}}{\mathrm{x}}\right)\)

⇒ \(\mathrm{v}=\mathrm{y}^{\mathrm{x}}\)

Taking logarithm on both the sides \(\log v=\log \left(y^x\right) \Rightarrow \log v=x \log y\)

Differentiating both sides with respect to x, we obtain

⇒ \(\frac{1}{v} \cdot \frac{d v}{d x}=\log y \cdot \frac{d}{d x}(x)+x \cdot \frac{d}{d x}(\log y)\)

⇒ \(\frac{d v}{d x}=v\left(\log y \cdot 1+x \cdot \frac{1}{y} \cdot \frac{d y}{d x}\right)\)

⇒ \(\frac{d v}{d x}=y^x\left(\log y+\frac{x d y}{y}\right)\)

From (1), (2), and (3), we obtain

⇒ \(x^y\left(\log x \frac{d y}{d x}+\frac{y}{x}\right)+y^x\left(\log y+\frac{x d y}{y d x}\right)\)=0

⇒ \(\left(x^y \log x+x y^{x-1}\right) \frac{d y}{d x}=-\left(y x^{y-1}+y^x \log y\right)\)

∴ \(\frac{d y}{d x}=-\frac{y x^{y-1}+y^x \log y}{x^y \log x+x y^{x-1}}\)

Question 13. \(\mathrm{y}^{\mathrm{x}}=\mathrm{x}^y\)

Solution:

The given function is \(\mathrm{y}^{\mathrm{x}}=\mathrm{x}^y\)

Taking logarithm on both sides, we obtain \(\log y^\pi=\log x^5 \Rightarrow x \log y=y \log x\)

Differentiating both sides with respect to x, we obtain

⇒ \(\log y \cdot \frac{d}{d x}(x)+x \cdot \frac{d}{d x}(\log y)=\log x \cdot \frac{d}{d x}(y)+y \cdot \frac{d}{d x}(\log x)\)

⇒ \(\log y \cdot 1+x \cdot \frac{1}{y} \cdot \frac{d y}{d x}=\log x \cdot \frac{d y}{d x}+y \cdot \frac{1}{x}\)

⇒ \(\log y+\frac{x}{y} \frac{d y}{d x}=\log x \frac{d y}{d x}+\frac{y}{x}\)

⇒ \(\left(\frac{x}{y}-\log x\right) \frac{d y}{d x}=\frac{y}{x}-\log y \Rightarrow\left(\frac{x-y \log x}{y}\right) \frac{d y}{d x}=\frac{y-x \log y}{x}\)

∴ \(\frac{d y}{d x}=\frac{y}{x}\left(\frac{y-x \log y}{x-y \log x}\right)\)

Question 14. \((\cos x)^y=(\cos y)^x\)

Solution:

The given function is \((\cos x)^y=(\cos y)^x\)

Taking logarithms on both sides, we obtain

y \(\log \cos x=x \log \cos y\)

⇒ \(\log \cos x \cdot \frac{d y}{d x}+y \cdot \frac{d}{d x}(\log \cos x)=\log \cos y \cdot \frac{d}{d x}(x)+x \cdot \frac{d}{d x}(\log \cos y)\)

⇒ \(\log \cos x \frac{d y}{d x}+y \cdot \frac{1}{\cos x} \cdot \frac{d}{d x}(\cos x)=\log \cos y \cdot 1+x \cdot \frac{1}{\cos y} \cdot \frac{d}{d x}(\cos y)\)

⇒ \(\log \cos x \frac{d y}{d x}+\frac{y}{\cos x} \cdot(-\sin x)=\log \cos y+\frac{x}{\cos y}(-\sin y) \cdot \frac{d y}{d x}\)

⇒ \(\log \cos x \frac{d y}{d x}-y \tan x=\log \cos y-x \tan y \frac{d y}{d x}\)

⇒ \((\log \cos x+x \tan y) \frac{d y}{d x}=y \tan x+\log \cos y\)

∴ \(\frac{d y}{d x}=\frac{y \tan x+\log \cos y}{x \tan y+\log \cos x}\)

Question 15. \(x y=e^{(x-y)}\)

Solution:

The given function is x y=\(e^{(x-y)}\)

⇒ Taking logarithm on both the sides \(\log (x y)=\log \left(e^{x-y}\right)\)

⇒ \(\log x+\log y=(x-y) \log e \Rightarrow \log x+\log y=(x-y) \times 1\)

⇒ \(\log x+\log y=x-y\)

Differentiating both sides with respect to x, we obtain

⇒ \(\frac{d}{d x}(\log x)+\frac{d}{d x}(\log y)=\frac{d}{d x}(x)-\frac{d y}{d x}\)

⇒ \(\frac{1}{x}+\frac{1}{y} \frac{d y}{d x}=1-\frac{d y}{d x}\)

⇒ \(\left(1+\frac{1}{y}\right) \frac{d y}{d x}=1-\frac{1}{x}\)

⇒ \(\left(\frac{y+1}{y}\right) \frac{d y}{d x}=\frac{x-1}{x}\)

∴ \(\frac{d y}{d x}=\frac{y(x-1)}{x(y+1)}\)

Equation 16. Find the derivative of the function given by \(f(x)=(1+x)\left(1+x^2\right)\left(1+x^4\right)\left(1+x^8\right)\) and hence find \(\mathrm{f}^{\prime}(1)\).

Solution:

The given relationship is \(f(x)=(1+x)\left(1+x^2\right)\left(1+x^4\right)\left(1+x^3\right)\)

Taking logarithms on both sides

⇒ \(\log f(x)=\log (1+x)+\log \left(1+x^2\right)+\log \left(1+x^4\right)+\log \left(1+x^x\right)\)

Differentiating both sides with respect to x, we obtain

⇒ \(\frac{1}{f(x)} \cdot \frac{d}{d x}[f(x)]=\frac{d}{d x} \log (1+x)+\frac{d}{d x} \log \left(1+x^2\right)+\frac{d}{d x} \log \left(1+x^4\right)+\frac{d}{d x} \log \left(1+x^8\right)\)

⇒ \(\frac{1}{f(x)} \cdot f^{\prime}(x)=\frac{1}{1+x} \cdot \frac{d}{d x}(1+x)+\frac{1}{1+x^2} \cdot \frac{d}{d x}\left(1+x^2\right)+\frac{1}{1+x^4} \cdot \frac{d}{d x}\left(1+x^4\right)+\frac{1}{1+x^8} \cdot \frac{d}{d x}\left(1+x^3\right)\)

⇒ \(f^{\prime}(x)=f(x)\left[\frac{1}{1+x}+\frac{1}{1+x^2} \cdot 2 x+\frac{1}{1+x^4} \cdot 4 x^3+\frac{1}{1+x^3} \cdot 8 x^7\right]\)

⇒ \(f^{\prime}(x)=(1+x)\left(1+x^2\right)\left(1+x^4\right)\left(1+x^8\right)\left[\frac{1}{1+x}+\frac{2 x}{1+x^7}+\frac{4 x^3}{1+x^4}+\frac{8 x^7}{1+x^5}\right]\)

Hence,\(f^{\prime}(1)=(1+1)\left(1+1^2\right)\left(1+1^4\right)\left(1+1^3\right)\left[\frac{1}{1+1}+\frac{2 \times 1}{1+1^2}+\frac{4 \times 1^3}{1+1^4}+\frac{8 \times 1^7}{1+1^1}\right]\)

=\(2 \times 2 \times 2 \times 2\left[\frac{1}{2}+\frac{2}{2}+\frac{4}{2}+\frac{8}{2}\right]=16 \times\left(\frac{1+2+4+8}{2}\right)=16 \times \frac{15}{2}=120\)

Question 17. If u, v and w are functions of x, then show that \(\frac{d}{d x}(u, v, w)=\frac{d u}{d x} v \cdot w+u, \frac{d v}{d x}, w+u, v, \frac{d w}{d x}\) in two ways-first by repeated application of product rule, second by logarithmic differentiation.

Solution:

Let y= u.v.w. =\(\mathrm{u} .(\mathrm{v} . \mathrm{w})\)

By applying the product rule, we obtain

⇒ \(\frac{d y}{d x}=\frac{d u}{d x} \cdot(v \cdot w)+u \cdot \frac{d}{d x}(v, w)\)

⇒ \(\frac{d y}{d x}=\frac{d u}{d x} v \cdot w+u\left[\frac{d v}{d x} \cdot w+v \cdot \frac{d w}{d x}\right]\) (Again applying product rule)

⇒ \(\frac{d y}{d x}=\frac{d u}{d x} \cdot v \cdot w+u \cdot \frac{d v}{d x} \cdot w+u \cdot v \cdot \frac{d w}{d x}\)

By taking logarithm on both sides of the equation y = u.v.w, we obtain

⇒ \(\log y=\log u+\log v+\log w\)

Differentiating both sides with respect to x, we obtain

⇒ \(\frac{1}{y} \cdot \frac{d y}{d x}=\frac{d}{d x}(\log u)+\frac{d}{d x}(\log v)+\frac{d}{d x}(\log w)\)

⇒ \(\frac{1}{y} \cdot \frac{d y}{d x}=\frac{1}{u} \frac{d u}{d x}+\frac{1}{v} \frac{d v}{d x}+\frac{1}{w} \frac{d w}{d x}\)

⇒ \(\frac{d y}{d x}=y\left(\frac{1}{u} \frac{d u}{d x}+\frac{1}{v} \frac{d v}{d x}+\frac{1}{w} \frac{d w}{d x}\right)\)

⇒ \(\frac{d y}{d x}=u \cdot v \cdot w \cdot\left(\frac{1}{u d x}+\frac{1 d v}{v d x}+\frac{1}{w} \frac{d w}{d x}\right)\)

∴ \(\frac{d y}{d x}=\frac{d u}{d x} \cdot v \cdot w+u \cdot \frac{d v}{d x} \cdot w+u \cdot v \cdot \frac{d w}{d x}\)

Continuity And Differentiability Exercise 5.6

Question 1. \(\mathrm{x}=2 \mathrm{at} \mathrm{t}^2, \mathrm{y}=\mathrm{at}^4\)

Solution:

The given equations are \(\mathrm{x}=2 \mathrm{at}^2\) and \(\mathrm{y}=a \mathrm{t}^4\)

Then, \(\frac{\mathrm{dx}}{\mathrm{dt}}=\frac{\mathrm{d}}{\mathrm{dt}}\left(2 \mathrm{at}^2\right)=2 \mathrm{a} \cdot \frac{\mathrm{d}}{\mathrm{dt}}\left(\mathrm{t}^2\right)\)

=\(2 \mathrm{a} \cdot 2 \mathrm{t}=4 \mathrm{at}\) and \(\frac{\mathrm{dy}}{\mathrm{dt}}=\frac{\mathrm{d}}{\mathrm{dt}}\left(\mathrm{at}^4\right)\)

=\(\mathrm{a} \cdot \frac{\mathrm{d}}{\mathrm{dt}}\left(\mathrm{t}^4\right)=\mathrm{a} \cdot 4 \cdot \mathrm{t}^3=4 \mathrm{at}^5\)

∴ \(\frac{d y}{d x}=\frac{\left(\frac{d y}{d t}\right)}{\left(\frac{d x}{d t}\right)}=\frac{4 a t^3}{4 a t}=t^2\)

Question 2. \(x=a \cos \theta, y=b \cos \theta\)

Solution:

The given equations are x=a \(\cos \theta and y=b \cos \theta\)

Then, \(\frac{d x}{d \theta}=\frac{d}{d \theta}(a \cos \theta)=a(-\sin \theta)=-a \sin \theta and \frac{d y}{d \theta}\)

=\(\frac{d}{d \theta}(b \cos \theta)=b(-\sin \theta)=-b \sin \theta\)

∴ \(\frac{d y}{d x}=\frac{\left(\frac{d y}{d \theta}\right)}{\left(\frac{d x}{d \theta}\right)}=\frac{-b \sin \theta}{-a \sin \theta}=\frac{b}{a}\)

Question 3. x=\(\sin t, y=\cos\) 2 t

Solution:

The given equations are x=\(\sin t and y=\cos\) 2 t

Then, \(\frac{\mathrm{dx}}{\mathrm{dt}}=\frac{\mathrm{d}}{\mathrm{dt}}(\sin \mathrm{t})=\cos \mathrm{t}\) and

⇒ \(\frac{\mathrm{dy}}{\mathrm{dt}}=\frac{\mathrm{d}}{\mathrm{dt}}(\cos 2 \mathrm{t})=-\sin 2 t \cdot \frac{\mathrm{d}}{\mathrm{dt}}(2 \mathrm{t})=-2 \sin 2 \mathrm{t}\)

∴ \(\frac{d y}{d x}=\frac{\left(\frac{d y}{d t}\right)}{\left(\frac{d x}{d t}\right)}=\frac{-2 \sin 2 t}{\cos t}=\frac{-2 \cdot 2 \sin t \cos t}{\cos t}=-4 \sin t\)

Question 4. x=4 t, y=\(\frac{4}{t}\)

Solution:

The given equations are x=4 t and y=\(\frac{4}{t}\)

⇒ \(\frac{d x}{d t}=\frac{d}{d t}(4 t)=4 and \frac{d y}{d t}=\frac{d}{d t}\left(\frac{4}{t}\right)\)

=\(4 \cdot \frac{d}{d t}\left(\frac{1}{t}\right)=4 \cdot\left(\frac{-1}{t^2}\right)=\frac{-4}{t^2}\)

∴ \(\frac{d y}{d x}=\frac{\left(\frac{d y}{d t}\right)}{\left(\frac{d x}{d t}\right)}=\frac{\left(\frac{-4}{t^2}\right)}{4}=\frac{-1}{t^2}\)

Question 5. x=\(\cos \theta-\cos 2 \theta, y=\sin \theta-\sin 2 \theta\)

Solution:

The given equations are x=\(\cos \theta-\cos 2 \theta and y=\sin \theta-\sin 2 \theta\)

Then,\(\frac{d x}{d \theta}=\frac{d}{d \theta}(\cos \theta-\cos 2 \theta)=\frac{d}{d \theta}(\cos \theta)-\frac{d}{d \theta}(\cos 2 \theta)\)

=-\(\sin \theta-(-2 \sin 2 \theta)=2 \sin 2 \theta-\sin \theta\)

and \(\frac{d y}{d \theta} =\frac{d}{d \theta}(\sin \theta-\sin 2 \theta)=\frac{d}{d \theta}(\sin \theta)-\frac{d}{d \theta}(\sin 2 \theta)\)

=\(\cos \theta-2 \cos 2 \theta\)

\(\frac{d y}{d x} =\frac{\left(\frac{d y}{d \theta}\right)}{\left(\frac{d x}{d \theta}\right)}\)

= \(\frac{\cos \theta-2 \cos 2 \theta}{2 \sin 2 \theta-\sin \theta}\)

Question 6. x=a\((\theta-\sin \theta), y=a(1+\cos \theta)\)

Solution:

The given equations are x=a\((\theta-\sin \theta) and y=a(1+\cos \theta)\)

Then, \(\frac{d x}{d \theta}=a\left[\frac{d}{d \theta}(\theta)-\frac{d}{d \theta}(\sin \theta)\right]=a(1-\cos \theta)\)

and \(\left.\frac{d y}{d \theta}=a\left[\frac{d}{d \theta}(1)+\frac{d}{d \theta}(\cos \theta)\right]=a[0+(-\sin \theta)]\right]=-a \sin \theta\)

⇒ \(\frac{d y}{d x}=\frac{\left(\frac{d y}{d \theta}\right.}{\left(\frac{d x}{d \theta}\right)}\)

=\(\frac{-a \sin \theta}{a(1-\cos \theta)}=\frac{-2 \sin \frac{\theta}{2} \cos \frac{\theta}{2}}{2 \sin ^2 \frac{\theta}{2}}\)

= \(\frac{-\cos \frac{\theta}{2}}{\sin \frac{\theta}{2}}=-\cot \frac{\theta}{2}\)

Question 7. x=\(\frac{\sin ^3 t}{\sqrt{\cos 2 t}}, y=\frac{\cos ^3 t}{\sqrt{\cos 2 t}}\)

Solution:

The given equations are x=\(\frac{\sin ^3 t}{\sqrt{\cos 2 t}} and y=\frac{\cos ^3 t}{\sqrt{\cos 2 t}}\)

Then,\(\frac{d x}{d t}=\frac{d}{d t}\left[\frac{\sin ^3 t}{\sqrt{\cos 2 t}}\right]\)

=\(\frac{\sqrt{\cos 2 t} \cdot \frac{d}{d t}\left(\sin ^3 t\right)-\sin ^3 t \cdot \frac{d}{d t} \sqrt{\cos 2 t}}{(\sqrt{\cos 2 t})^2}\)

=\(\frac{\sqrt{\cos 2 t} \cdot 3 \sin ^2 t \cdot \frac{d}{d t}(\sin t)-\sin ^3 t \times \frac{1}{2 \sqrt{\cos 2 t}} \cdot \frac{d}{d t}(\cos 2 t)}{\cos 2 t}\)

=\(\frac{3 \sqrt{\cos 2 t} \cdot \sin ^2 t \cos t-\frac{\sin ^3 t}{2 \sqrt{\cos 2 t}} \cdot(-2 \sin 2 t)}{\cos 2 t}\)

= \(\frac{3 \cos 2 t \sin ^2 t \cos t+\sin ^3 t \sin 2 t}{\cos ^2 t \sqrt{\cos 2 t}}\)

and \(\frac{d y}{d t} =\frac{d}{d t}\left[\frac{\cos ^3 t}{\sqrt{\cos 2 t}}\right]=\frac{\sqrt{\cos 2 t} \cdot \frac{d}{d t}\left(\cos ^3 t\right)-\cos ^3 t \cdot \frac{d}{d t}(\sqrt{\cos 2 t})}{\cos 2 t}\)

= \(\frac{\sqrt{\cos 2 t} \cdot 3 \cos ^2 t \cdot \frac{d}{d t}(\cos t)-\cos ^3 t \cdot \frac{1}{2 \sqrt{\cos 2 t}} \cdot \frac{d}{d t}(\cos 2 t)}{\cos 2 t}\)

=\(\frac{3 \sqrt{\cos 2 t} \cdot \cos ^2 t(-\sin t)-\cos ^3 t \cdot \frac{1}{2 \sqrt{\cos 2 t}} \cdot(-2 \sin 2 t)}{\cos 2 t}\)

=\(\frac{-3 \cos 2 t \cdot \cos ^2 t \cdot \sin t+\cos ^3 t \sin 2 t}{\cos ^2 t \cdot \sqrt{\cos 2 t}}\)

\(\frac{d y}{d x} =\frac{\left(\frac{d y}{d t}\right)}{\left(\frac{d x}{d t}\right)}=\frac{-3 \cos 2 t \cdot \cos ^2 t \cdot \sin t+\cos ^3 t \sin 2 t}{3 \cos 2 t \sin ^2 t \cos t+\sin ^3 t \sin 2 t}\)

=\(\frac{-3 \cos 2 t \cdot \cos ^2 t \cdot \sin t+\cos ^2 t(2 \sin t \cos t)}{3 \cos 2 t \sin ^2 t \cos t+\sin ^3 t(2 \sin t \cos t)}\)

=\(\frac{\sin t \cos t\left[-3 \cos 2 t \cdot \cos t+2 \cos ^3 t\right]}{\sin t \cos t\left[3 \cos 2 t \sin t+2 \sin ^3 t\right]}\)

=\(\frac{\left[-3\left(2 \cos ^2 t-1\right) \cos t+2 \cos ^3 t\right]}{\left[3\left(1-2 \sin ^2 t\right) \sin t+2 \sin ^3 t\right]}\)

⇒ \(\left[\begin{array}{l}
\cos 2 t=\left(2 \cos ^2 t-1\right)
\cos 2 t=\left(1-2 \sin ^2 t\right)
\end{array}\right]\)

=\(\frac{-4 \cos ^3 t+3 \cos t}{3 \sin t-4 \sin ^3 t}\)

∴ \(\frac{d y}{d x}=\frac{-\cos 3 t}{\sin 3 t}\left[\begin{array}{l}
\cos 3 t=4 \cos ^3 t-3 \cos t \sin 3 t=3 \sin t-4 \sin ^3 t
\end{array}\right]=-\cot 3 t\)

Question 8. x=a\(\left(\cos t+\log \tan \frac{t}{2}\right), y=a \sin t\)

Solution:

The given equations are x=a\(\left(\cos t+\log \tan \frac{t}{2}\right) and y=a \sin t\)

Then, \(\frac{\mathrm{dx}}{\mathrm{dt}}=\mathrm{a} \cdot\left[\frac{\mathrm{d}}{\mathrm{dt}}(\cos \mathrm{t})+\frac{\mathrm{d}}{\mathrm{dt}}\left(\log \tan \frac{\mathrm{t}}{2}\right)\right]\)

=\(\mathrm{a}\left[-\sin \mathrm{t}+\frac{\mathrm{t}}{\tan \frac{\mathrm{t}}{2}} \cdot \frac{\mathrm{d}}{\mathrm{dt}}\left(\tan \frac{\mathrm{t}}{2}\right)\right]\)

=a\(\left[-\sin t+\cot \frac{t}{2} \cdot \sec ^2 \frac{t}{2} \cdot \frac{d}{d t}\left(\frac{t}{2}\right)\right]=a\left[-\sin t+\frac{\cos \frac{t}{2}}{\sin \frac{t}{2}} \times \frac{1}{\cos ^2 \frac{t}{2}} \times \frac{1}{2}\right]\)

=a\(\left[-\sin t+\frac{1}{2 \sin \frac{t}{2} \cos \frac{t}{2}}\right]\)

=a\(\left(-\sin t+\frac{1}{\sin t}\right)=a\left(\frac{-\sin ^2 t+1}{\sin t}\right)\)

=a\(\left(\frac{\cos ^2 t}{\sin t}\right)\)

and \(\frac{d y}{d t}=a \frac{d}{d t}(\sin t)=a \cos t\)

∴ \(\frac{d y}{d x}=\frac{\left(\frac{d y}{d t}\right)}{\left(\frac{d x}{d t}\right)}=\frac{a \cos t}{\left(a \frac{\cos ^2 t}{\sin t}\right)}=\frac{\sin t}{\cos t}=\tan t\)

Question 9. x=a \(\sec \theta, y=b \tan \theta\)

Solution:

The given equations are x=a \(\sec \theta and y=b \tan \theta\)

Then, \(\frac{d x}{d \theta}=a \cdot \frac{d}{d \theta}(\sec \theta)=a \sec \theta \tan \theta\)

⇒ \(\frac{d y}{d \theta}=b \cdot \frac{d}{d \theta}(\tan \theta)=b \sec ^2 \theta\)

⇒ \(\frac{d y}{d x}=\frac{\left(\frac{d y}{d \theta}\right)}{\left(\frac{d x}{d \theta}\right)}\)

= \(\frac{b \sec ^2 \theta}{{asec} \theta \tan \theta}=\frac{b}{a} \sec \theta \cot \theta\)

= \(\frac{b \cos \theta}{a \cos \theta \sin \theta}=\frac{b}{a} \times \frac{1}{\sin \theta}=\frac{b}{a} {cosec} \theta\)

Question 10. x=a\((\cos \theta+\theta \sin \theta), y=a(\sin \theta-\theta \cos \theta)\)

Solution:

The given equations are x=a\((\cos \theta+\theta \sin \theta) and y=a(\sin \theta-\theta \cos \theta)\)

Then, \(\frac{d x}{d \theta} =a\left[\frac{d}{d \theta}(\cos \theta)+\frac{d}{d \theta}(\theta \sin \theta)\right]\)

=a\(\left[-\sin \theta+\theta \frac{d}{d \theta}(\sin \theta)+\sin \theta \frac{d}{d \theta}(\theta)\right]\)

=\(a(-\sin \theta+\theta \cos \theta+\sin \theta)=a \theta \cos \theta \)

and \(\frac{d y}{d \theta} =a\left[\frac{d}{d \theta}(\sin \theta)-\frac{d}{d \theta}(\theta \cos \theta)\right]\)

=\(a\left[\cos \theta-\left\{\theta \frac{d}{d \theta}(\cos \theta)+\cos \theta \cdot \frac{d}{d \theta}(\theta)\right\}\right]\)

=\(a[\cos \theta+\theta \sin \theta-\cos \theta]=a \theta \sin \theta \)

∴ \(\frac{d y}{d x} =\frac{\left(\frac{d y}{d \theta}\right)}{\left(\frac{d x}{d \theta}\right)}=\frac{a \theta \sin \theta}{a \theta \cos \theta}=\tan \theta\)

Question 11. If x=\(\sqrt{a^{\sin ^{-1} 1}}\), y=\(\sqrt{a^{\alpha x^{-1} t}}\), show that \(\frac{d y}{d x}=-\frac{y}{x}\) ?

Solution:

The given equations are x=\(\sqrt{a^{\sin ^{-1} t}}\) and y=\(\sqrt{a^{\cos ^{-1} t}}\)

x=\(\left(a^{\sin ^{-1} t}\right)^{\frac{1}{2}}\) and y=\(\left(a^{\cos ^{-1} t}\right)^{\frac{1}{2}}\)

x=\(a^{\frac{1}{2} {tin}^{-1} t}\) and y=\(a^{\frac{1}{2} \cos ^{-1} t}\)

Consider \(\mathrm{x}=\mathrm{a}^{\frac{1}{2} \sin ^{-1} \mathrm{t}}\)

Taking logarithms on both sides, we obtain

⇒ \(\log x=\frac{1}{2} \sin ^{-1} t \log a\)

⇒ \(\frac{1}{x} \cdot \frac{d x}{d t}=\frac{1}{2} \log a \cdot \frac{d}{d t}\left(\sin ^{-1} t\right)\)

⇒ \(\frac{d x}{d t}=\frac{x}{2} \log a \cdot \frac{1}{\sqrt{1-t^2}}\)

⇒ \(\frac{d x}{d t}=\frac{x \log a}{2 \sqrt{1-t^2}}\)

Then, consider y=\(a^{\frac{1}{2} \cos ^{-1} t}\)

Taking logarithms on both sides, we obtain

⇒ \(\log y=\frac{1}{2} \cos ^{-1} t \log a\)

⇒ \(\frac{1}{y} \cdot \frac{d y}{d t}=\frac{1}{2} \log a \cdot \frac{d}{d t}\left(\cos ^{-1} t\right)\)

⇒ \(\frac{d y}{d t}=\frac{y \log a}{2} \cdot\left(\frac{-1}{\sqrt{1-t^2}}\right)\)

⇒ \(\frac{d y}{d t}=\frac{-y \log a}{2 \sqrt{1-t^2}}\)

⇒ \(\frac{d y}{d x}=\frac{\left(\frac{d y}{d t}\right)}{\left(\frac{d x}{d t}\right)}\)

=\(\frac{\left(\frac{-y \log a}{2 \sqrt{1-t^2}}\right)}{\left(\frac{x \log a}{2 \sqrt{1-t^2}}\right)}=-\frac{y}{x}\)

Hence, proved.

Continuity And Differentiability Exercise 5.7

Question 1. \(x^2+3 x+2\)

Solution:

Let y=\(x^2+3 x+2\)

Differentiating w.r.t. x on both the sides

Then,\(\frac{\mathrm{dy}}{\mathrm{dx}}=\frac{\mathrm{d}}{\mathrm{dx}}\left(\mathrm{x}^2\right)+\frac{\mathrm{d}}{\mathrm{dx}}(3 \mathrm{x})+\frac{\mathrm{d}}{\mathrm{dx}}(2)\)

=2 \(\mathrm{x}+3+0=2 \mathrm{x}+3\)

Again differentiating with respect to x on both sides

⇒ \(\frac{d^2 y}{d x^2}=\frac{d}{d x}(2 x+3)=\frac{d}{d x}(2 x)+\frac{d}{d x}(3)=2+0=2\)

Question 2. \(\mathrm{x}^{20}\)

Solution:

Let \(\mathrm{y}=\mathrm{x}^{20}\)

Then,\(\frac{\mathrm{dy}}{\mathrm{dx}}=\frac{\mathrm{d}}{\mathrm{dx}}\left(\mathrm{x}^{20}\right)=20 \mathrm{x}^{19}\)

Again differentiating with respect to x on both sides

∴ \(\frac{d^2 y}{d x^2}=\frac{d}{d x}\left(20 x^{19}\right)=20 \frac{d}{d x}\left(x^{19}\right)=20 \cdot 19 \cdot x^{18}=380 x^{18}\)

Question 3. x \(\cdot \cos x\)

Solution:

Let y=x \(\cdot \cos x\)

Then, \(\frac{d y}{d x}=\frac{d}{d x}(x \cdot \cos x)=\cos x \cdot \frac{d}{d x}(x)+x \frac{d}{d x}(\cos x)=\cos x \cdot 1+x(-\sin x)=\cos x-x \sin x\)

Again differentiating with respect to x on both sides

⇒ \(\frac{d^2 y}{d x^2} =\frac{d}{d x}[\cos x-x \sin x]=\frac{d}{d x}(\cos x)-\frac{d}{d x}(x \sin x)\)

=-\(\sin x-\left[\sin x \cdot \frac{d}{d x}(x)+x \cdot \frac{d}{d x}(\sin x)\right]=-\sin x-(\sin x+x \cos x)=-(x \cos x+2 \sin x)\)

Question 4. log x

Solution:

Let y=\(\log \mathrm{x}\)

Then, \(\frac{d y}{d x}=\frac{d}{d x}(\log x)=\frac{1}{x}\)

Again differentiating with respect to x on both sides

⇒ \(\frac{d^2 y}{d x^2}=\frac{d}{d x}\left(\frac{1}{x}\right)=\frac{-1}{x^2}\)

Question 5. \(x^3 \log x\)

Solution:

Let y=\(x^3 \log x\)

Then,\(\frac{d y}{d x} =\frac{d}{d x}\left[x^3 \log x\right]=\log x \cdot \frac{d}{d x}\left(x^3\right)+x^3 \cdot \frac{d}{d x}(\log x)\)

=\(\log x \cdot 3 x^2+x^3 \cdot \frac{1}{x}=\log x \cdot 3 x^2+x^2=x^2(1+3 \log x)\)

Again differentiating with respect to x on both sides

⇒ \(\frac{d^2 y}{d x^2} =\frac{d}{d x}\left[x^2(1+3 \log x)\right]=(1+3 \log x) \cdot \frac{d}{d x}\left(x^2\right)+x^2 \frac{d}{d x}(1+3 \log x)\)

= \((1+3 \log x) \cdot 2 x+x^2 \cdot \frac{3}{x}\)

= 2 x+6 x \(\log x+3 x=5 x+6 x \log x=x(5+6 \log x)\)

Question 6. \(e^x \sin 5 x\)

Solution:

Let y=\(e^x \sin 5 x\)

⇒ \(\frac{d y}{d x}= \frac{d}{d x}\left(e^x \sin 5 x\right)\)

=\(\sin 5 x \cdot \frac{d}{d x}\left(e^x\right)+e^x \frac{d}{d x}(\sin 5 x)\)

=\(\sin 5 x \cdot e^x+e^x \cdot \cos 5 x \cdot \frac{d}{d x}(5 x)\)

=\(e^x \sin 5 x+e^x \cos 5 x \cdot 5=e^x(\sin 5 x+5 \cos 5 x)\)

Again differentiating with respect to x on both sides

⇒ \(\frac{d^2 y}{d x^2} =\frac{d}{d x}\left[e^x(\sin 5 x+5 \cos 5 x)\right] \)

=\((\sin 5 x+5 \cos 5 x) \cdot \frac{d}{d x}\left(e^x\right)+e^x \cdot \frac{d}{d x}(\sin 5 x+5 \cos 5 x)\)

= \((\sin 5 x+5 \cos 5 x) e^x+e^x\left[\cos 5 x \cdot \frac{d}{d x}(5 x)+5(-\sin 5 x) \cdot \frac{d}{d x}(5 x)\right]\)

=\(e^x(\sin 5 x+5 \cos 5 x)+e^x(5 \cos 5 x-25 \sin 5 x)\)

=\(e^x(10 \cos 5 x-24 \sin 5 x)=2 e^x(5 \cos 5 x-12 \sin 5 x)\)

Question 7. \(e^{4 x} \cos 3 x\)

Solution:

Let y=\(e^{6 x} \cos 3 x\)

Then, \(\frac{d y}{d x}=\frac{d}{d x}\left(e^{6 x} \cdot \cos 3 x\right)\)

=\(\cos 3 x \cdot \frac{d}{d x}\left(e^{6 x}\right)+e^{4 x} \cdot \frac{d}{d x}(\cos 3 x)\)

=\(\cos 3 x \cdot e^{6 x} \cdot \frac{d}{d x}(6 x)+e^{6 x} \cdot(-\sin 3 x) \cdot \frac{d}{d x}(3 x)\)

=\(6 e^{6 x} \cos 3 x-3 e^{6 x} \sin 3 x\)

Again differentiating with respect to x on both sides

⇒ \(\frac{d^2 y}{d x^2} =\frac{d}{d x}\left(6 e^{6 x} \cos 3 x-3 e^{6 x} \sin 3 x\right)\)

= \(6 \cdot \frac{d}{d x}\left(e^{6 x} \cos 3 x\right)-3 \cdot \frac{d}{d x}\left(e^{6 x} \sin 3 x\right)\)

=\(6 \cdot\left[6 e^{6 x} \cos 3 x-3 e^{6 x} \sin 3 x\right]-3 \cdot\left[\sin 3 x \cdot \frac{d}{d x}\left(e^{6 x}\right)+e^{6 x} \cdot \frac{d}{d x}(\sin 3 x)\right] [U {sing}(1)]\)

=\(36 e^{6 x} \cos 3 x-18 e^{6 x} \sin 3 x-3\left[\sin 3 x \cdot e^{6 x} \cdot 6+e^{6 x} \cdot \cos 3 x \cdot 3\right]\)

=\(36 e^{6 x} \cos 3 x-18 e^{6 x} \sin 3 x-18 e^{6 x} \sin 3 x-9 e^{6 x} \cos 3 x\)

=\(27 e^{6 x} \cos 3 x-36 e^{6 x} \sin 3 x=9 e^{6 x}(3 \cos 3 x-4 \sin 3 x)\)

Question 8. \(\tan ^{-1} x\)

Solution:

Let y=\(\tan ^{-1} x\)

Then, \(\frac{d y}{d x}=\frac{d}{d x}\left(\tan ^{-1} x\right)=\frac{1}{1+x^2}\)

Again differentiating with respect to x on both sides, use obtain

⇒ \(\frac{d^2 y}{d x^2}=\frac{d}{d x}\left(\frac{1}{1+x^2}\right)=\frac{d}{d x}\left(1+x^2\right)^{-1}\)

= \((-1) \cdot\left(1+x^2\right)^{-2} \cdot \frac{d}{d x}\left(1+x^2\right)\)

= \(\frac{-1}{\left(1+x^2\right)^2} \times 2 x=\frac{-2 x}{\left(1+x^2\right)^2}\)

Question 9. \(\log (\log x)\)

Solution:

Let y=\(\log (\log x)\)

Then, \(\frac{d y}{d x}=\frac{d}{d x}[\log (\log x)]=\frac{1}{\log x} \cdot \frac{d}{d x}(\log x)\)

=\(\frac{1}{x \log x}=(x \log x)^{-1}\)

Again differentiating with respect to x on both sides

⇒ \(\frac{d^2 y}{d x^2}=\frac{d}{d x}\left[(x \log x)^{-1}\right]=(-1) \cdot(x \log x)^{-2} \cdot \frac{d}{d x}(x \log x)\)

=-\(\frac{1}{(x \log x)^2}\left[\log x \frac{d(x)}{d x}+x \cdot \frac{d}{d x}(\log x)\right]\)

=-\(\frac{1}{(x \log x)^2} \cdot\left[\log x \cdot 1+x \cdot \frac{1}{x}\right]=\frac{-(1+\log x)}{(x \log x)^2}\)

Question 10. \(\sin (\log x)\)

Solution:

Let \(\mathrm{y}=\sin (\log \mathrm{x})\)

Then, \(\frac{d y}{d x}=\frac{d}{d x}[\sin (\log x)]=\cos (\log x) \cdot \frac{d}{d x}(\log x)=\frac{\cos (\log x)}{x}\)

Again differentiating with respect to x on both sides

⇒ \(\frac{d^2 y}{d x^2} =\frac{d}{d x}\left[\frac{\cos (\log x)}{x}\right]\)

= \(\frac{x \cdot \frac{d}{d x}[\cos (\log x)]-\cos (\log x) \frac{d}{d x}(x)}{x^2}\)

=  \(\frac{x \cdot\left[-\sin (\log x) \cdot \frac{d}{d x}(\log x)\right]-\cos (\log x) \cdot 1}{x^2}\)

=\(\frac{-x \sin (\log x) \cdot \frac{1}{x}-\cos (\log x)}{x^2}\)

=\(\frac{-[\sin (\log x)+\cos (\log x)]}{x^2}\)

Question 11. If y=5 \(\cos x-3 \sin x, prove that \frac{d^2 y}{d x^2}+y=0\)

Solution:

It is given that, y=5 \(\cos x-3 \sin x\)

Then,\(\frac{d y}{d x}=\frac{d}{d x}(5 \cos x)-\frac{d}{d x}(3 \sin x)=5 \frac{d}{d x}(\cos x)-3 \frac{d}{d x}(\sin x)\)

=\(5(-\sin x)-3 \cos x=-(5 \sin x+3 \cos x)\)

Again differentiating with respect to \(\mathrm{x}\) on both the sides

⇒ \(\frac{d^2 y}{d x^2} =\frac{d}{d x}[-(5 \sin x+3 \cos x)]\)

=-\(\left[5 \cdot \frac{d}{d x}(\sin x)+3 \cdot \frac{d}{d x}(\cos x)\right]\)

=-\([5 \cos x+3(-\sin x)]=-[5 \cos x-3 \sin x]\)=-y

⇒ \(\frac{d^2 y}{d x^2}+y\) =0

Hence, proved.

Question 12. If y=\(\cos ^{-1}\) x, find \(\frac{d^2 y}{d x^2}\) in terms of y alone?

Solution:

It is given that, y=\(\cos ^{-1} x \Rightarrow x=\cos y\)

⇒ \(\frac{d y}{d x}=-\frac{1}{\sqrt{1-x^2}}=-\frac{1}{\sqrt{1-\cos ^2 y}}=-{cosec} y \)

⇒ \(\frac{d^2 y}{d x^2}=\frac{d}{d x}(-{cosec} y)\)

⇒ \(\frac{d^2 y}{d x^2}={cosec} y \cot y \frac{d y}{d x}\)

=-\({cosec} y \cot y\)

Alternate Method:

It is given that, y=\(\cos ^{-1} x\)

Then, \(\frac{d y}{d x}=\frac{d}{d x}\left(\cos ^{-1} x\right)=\frac{-1}{\sqrt{1-x^2}}=-\left(1-x^2\right)^{\frac{-1}{2}}\)

Again differentiating with respect to x on both sides

⇒ \(\frac{d^2 y}{d x^2}=\frac{d}{d x}\left[-\left(1-x^2\right)^{\frac{-1}{2}}\right]\)

=-\(\left(-\frac{1}{2}\right) \cdot\left(1-x^2\right)^{\frac{-3}{2}} \cdot \frac{d}{d x}\left(1-x^2\right)\)

=\(\frac{1}{2 \sqrt{\left(1-x^2\right)^3}} \times(-2 x)\)

⇒ \(\frac{d^2 y}{d x^2}=\frac{-x}{\sqrt{\left(1-x^2\right)^2}}\)

⇒ \(y=\cos ^{-1} x \Rightarrow x=\cos y\)

Putting x=\(\cos y\) in equation (1), we obtain

⇒ \(\frac{d^2 y}{d x^2}=\frac{-\cos y}{\sqrt{\left(1-\cos ^2 y\right)^3}}=\frac{-\cos y}{\sqrt{\left(\sin ^2 y\right)^3}}\)

=\(\frac{-\cos y}{\sin ^3 y}=\frac{-\cos y}{\sin y} \times \frac{1}{\sin ^2 y}\)

∴ \(\frac{d^2 y}{d x^2}=-\cot y \cdot {cosec}^2 y\)

Question 13. If y=3 \(\cos (\log x)+4 \sin (\log x)\), show that \(x^2 y_2+x y_1+y\)=0

Solution:

It is given that, y=3 \(\cos (\log x)+4 \sin (\log x)\)

Then, \(y_1 =3 \cdot \frac{d}{d x}[\cos (\log x)]+4 \cdot \frac{d}{d x}[\sin (\log x)]\)

=3 \(\cdot\left[-\sin (\log x) \cdot \frac{d}{d x}(\log x)\right]+4 \cdot\left[\cos (\log x) \cdot \frac{d}{d x}(\log x)\right]\)

⇒ \(y_1 =\frac{-3 \sin (\log x)}{x}+\frac{4 \cos (\log x)}{x}=\frac{4 \cos (\log x)-3 \sin (\log x)}{x}\)

⇒ \(y_1 =4 \cos (\log x)-3 \sin (\log x)\)

Again differentiating with respect to x on both sides

x \(\frac{d\left(y_1\right)}{d x}+y_1 \frac{d(x)}{d x}=4(-\sin (\log x)) \frac{d}{d x}(\log x)-3 \cos (\log x) ⇒ \frac{d}{d x}(\log x)\)

⇒ \([latex]x y_2+y_1=\frac{-4 \sin (\log x)}{x}-\frac{3 \cos (\log x)}{x} \)

⇒ \(x^2 y_2+x y_1=-(4 \sin (\log x)+3 \cos (\log x))\)

⇒ \(x^2 y_2+x y_1=-y or x^2 y_2+x y_1+y=0\) [From eq. (1)]

Hence, proved.

Question 14. If y=500 \(e^{7 x}+600 e^{-7 x}\), show that \(\frac{d^2 y}{d x^2}\)=49 y

Solution:

It is given that, y=500 \(e^{1 x}+600 e^{-3 x}\)

Then, \(\frac{d y}{d x} =500 \cdot \frac{d}{d x}\left(e^{7 x}\right)+600 \cdot \frac{d}{d x}\left(e^{-7 x}\right)\)

=500 \(\cdot e^{7 x} \cdot \frac{d}{d x}(7 x)+600 \cdot e^{-7 x} \cdot \frac{d}{d x}(-7 x)=3500 e^{7 x}-4200 e^{-7 x}\)

Again differentiating with respect to x on both sides

⇒ \(\frac{d^2 y}{d x^2} =3500 \cdot \frac{d}{d x}\left(e^{7 x}\right)-4200 \cdot \frac{d}{d x}\left(e^{-7 x}\right)\)

=3500 \(\cdot e^{7 x} \cdot \frac{d}{d x}(7 x)-4200 \cdot e^{-7 x} \cdot \frac{d}{d x}(-7 x)\)

=7 \(\times 3500 \cdot e^{7 x}+7 \times 4200 \cdot e^{-7 x}=49 \times 500 e^{7 x}+49 \times 600 e^{-7 x} \)

=49\(\left(500 e^{7 x}+600 e^{-7 x}\right)\)=49 y

Hence, proved.

Question 15. If \(\mathrm{e}^y(\mathrm{x}+1)=1\), show that \(\frac{\mathrm{d}^2 \mathrm{y}}{\mathrm{dx}^2}=\left(\frac{\mathrm{dy}}{\mathrm{dx}}\right)^2\)

Solution:

The given relationship is \(e^y(x+1)=1 \Rightarrow e^y=\frac{1}{x+1}\)

Taking logarithm on both the sides \(\log e^y=\log \frac{1}{(x+1)} \Rightarrow y=\log \frac{1}{(x+1)}\)

Differentiating this relationship with respect to x

⇒ \(\frac{\mathrm{dy}}{\mathrm{dx}}=(\mathrm{x}+1) \frac{\mathrm{d}}{\mathrm{dx}}\left(\frac{1}{\mathrm{x}+1}\right)\)

=\((\mathrm{x}+1) \cdot \frac{-1}{(\mathrm{x}+1)^2}=\frac{-1}{\mathrm{x}+1}\)

Again differentiating with respect to x on both sides

⇒ \(\frac{d^2 y}{d x^2}=-\frac{d}{d x}\left(\frac{1}{x+1}\right)=-\left(\frac{-1}{(x+1)^2}\right)\)

=\(\frac{1}{(x+1)^2}=\left(\frac{-1}{x+1}\right)^2=\left(\frac{d y}{d x}\right)^2\)

Hence, proved.

Question 16. If y=\(\left(\tan ^{-1} x\right)^2\), show that \(\left(x^2+1\right)^2 y_2+2 ⇒ x\left(x^2+1\right) y_1\)=2

Solution:

The given relationship is y=\(\left(\tan ^{-1} x\right)^2\).

Then, \(y_1=2 \tan ^{-1} x \frac{d}{d x}\left(\tan ^{-1} x\right)\)

⇒ \(y_1=2 \tan ^{-1} x \cdot \frac{1}{1+x^2}\)

⇒ \(\left(1+x^2\right) y_1=2 \tan ^{-1} x\)

Again differentiating with respect to x on both sides

⇒ \(\left(1+x^2\right) y_2+2 x y_1=2\left(\frac{1}{1+x^2}\right)\)

⇒\(\left(1+x^2\right)^2 y_2+2 x\left(1+x^2\right) y_1=2\)

Hence, proved.

Continuity And Differentiability Miscellaneous Exercise

Question 1. \(\left(3 x^2-9 x+5\right)^6\)

Solution:

Let y=\(\left(3 x^2-9 x+5\right)^{\prime}\)

Using the chain rule, we obtain

⇒ \(\frac{d y}{d x} =\frac{d}{d x}\left(3 x^2-9 x+5\right)^9=9\left(3 x^2-9 x+5\right)^1 \cdot \frac{d}{d x}\left(3 x^2-9 x+5\right)\)

=\(9\left(3 x^2-9 x+5\right)^3 \cdot(6 x-9)\)

=9\(\left(3 x^2-9 x+5\right)^3 \cdot 3(2 x-3)=27\left(3 x^2-9 x+5\right)^3(2 x-3)\)

Question 2. \(\sin ^3 x+\cos ^6 x\)

Solution:

Let y=\(\sin ^3 x+\cos ^4 x \)

⇒ \(\frac{d y}{d x} =\frac{d}{d x}\left(\sin ^3 x\right)+\frac{d}{d x}\left(\cos ^6 x\right)\)

=\(3 \sin ^2 x \cdot \frac{d}{d x}(\sin x)+6 \cos ^5 x \cdot \frac{d}{d x}(\cos x)\)

=3 \(\sin ^2 x \cdot \cos x+6 \cos ^5 x \cdot(-\sin x)\)

=3 \(\sin x \cos x\left(\sin x-2 \cos ^4 x\right)\)

Question 3. \((5 x)^{3 \mathrm{mon} t}\)

Solution: 

Let y=\((5 x)^{2 \cos 2 x}\)

Taking logarithm on both sides, we obtain, \(\log y=3 \cos 2 x \log 5 x\)

Differentiating both sides with respect to x, we obtain

⇒ \(\frac{1}{y} \frac{d y}{d x}=3\left[\log 5 x \cdot \frac{d}{d x}(\cos 2 x)+\cos 2 x \cdot \frac{d}{d x}(\log 5 x)\right]\)

⇒ \(\frac{d y}{d x}=3 y\left[\log 5 x(-\sin 2 x) \cdot \frac{d}{d x}(2 x)+\cos 2 x-\frac{1}{5 x} \cdot \frac{d}{d x}(5 x)\right]\)

⇒ \(\frac{d y}{d x}=3 y\left[-2 \sin 2 x \log 5 x+\frac{\cos 2 x}{x}\right]\)

⇒ \(\frac{d y}{d x}=y\left[\frac{3 \cos 2 x}{x}-6 \sin 2 x \log 5 x\right]\)

∴ \(\frac{d y}{d x}=(5 x)^{3 \cos 2 x}\left[\frac{3 \cos 2 x}{x}-6 \sin 2 x \log 5 x\right]\)

Question 4. \(\sin ^{-1}(x \sqrt{x}), 0 \leq x \leq 1\)

Solution:

Let \(y=\sin ^{-1}(x \sqrt{x})\)

Using the chain rule, we obtain

⇒ \(\frac{d y}{d x} =\frac{d}{d x} \sin ^{-1}(x \sqrt{x})=\frac{1}{\sqrt{1-(x \sqrt{x})^2}} \times \frac{d}{d x}(x \sqrt{x})\)

=\(\frac{1}{\sqrt{1-x^3}} \cdot \frac{d}{d x}\left(x^{\frac{3}{2}}\right)\)

=\(\frac{1}{\sqrt{1-x^2}} \times \frac{3}{2} \cdot x^{\frac{1}{2}}=\frac{3 \sqrt{x}}{2 \sqrt{1-x^3}}\)

=\(\frac{3}{2} \sqrt{\frac{x}{1-x^3}}\)

Question 5. \(\frac{\cos ^{-1} \frac{x}{2}}{\sqrt{2 x+7}},-2<x<2\)

Solution:

Let \(\mathrm{y}=\frac{\cos ^{-1} \frac{\mathrm{x}}{2}}{\sqrt{2 \mathrm{x}+7}}\)

By the quotient rule, we obtain,

⇒ \(\frac{d y}{d x} =\frac{\sqrt{2 x+7} \frac{d}{d x}\left(\cos ^{-1} \frac{x}{2}\right)-\left(\cos ^{-1} \frac{x}{2}\right) \frac{d}{d x}(\sqrt{2 x+7})}{(\sqrt{2 x+7})^2}\)

=\(\frac{\sqrt{2 x+7}\left[\frac{-1}{\sqrt{1-\left(\frac{x}{2}\right)^2}} \cdot \frac{d}{d x}\left(\frac{x}{2}\right)\right]-\left(\cos ^{-1} \frac{x}{2}\right) \frac{1}{2 \sqrt{2 x+7}} \cdot \frac{d}{d x}(2 x+7)}{2 x+7}\),

=\(\frac{\sqrt{2 x+7} \frac{-1}{\sqrt{4-x^2}}-\left(\cos ^{-1} \frac{x}{2}\right) \frac{2}{2 \sqrt{2 x+7}}}{2 x+7}\)

=-\(\frac{\sqrt{2 x+7}}{\sqrt{4-x^2} \times(2 x+7)}-\frac{\cos ^{-1} \frac{x}{2}}{(\sqrt{2 x+7})(2 x+7)}\)

=-\(\left[\frac{1}{\sqrt{4-x^2} \sqrt{2 x+7}}+\frac{\cos ^{-1} \frac{x}{2}}{(2 x+7)^{\frac{3}{2}}}\right]\)

Question 6. \(\cot ^{-1}\left[\frac{\sqrt{1+\sin x}+\sqrt{1-\sin x}}{\sqrt{1+\sin x}-\sqrt{1-\sin x}}\right], 0<x<\frac{\pi}{2}\)

Solution:

Let y=\(\cot ^{-1}\left[\frac{\sqrt{1+\sin x}+\sqrt{1-\sin x}}{\sqrt{1+\sin x}-\sqrt{1-\sin x}}\right]\)

Then,\(\left[\frac{\sqrt{1+\sin x}+\sqrt{1-\sin x}}{\sqrt{1+\sin x}-\sqrt{1-\sin x}}\right]\)

=\(\frac{(\sqrt{1+\sin x}+\sqrt{1-\sin x})^2}{(\sqrt{1+\sin x}-\sqrt{1-\sin x})(\sqrt{1+\sin x}+\sqrt{1-\sin x})}\)

=\(\frac{(1+\sin x)+(1-\sin x)+2 \sqrt{(1-\sin x)(1+\sin x)}}{(1+\sin x)-(1-\sin x)}\)

=\(\frac{2+2 \sqrt{1-\sin ^2 x}}{2 \sin x}=\frac{1+\cos x}{\sin x}\)

=\(\frac{2 \cos ^2 \frac{x}{2}}{2 \sin \frac{x}{2} \cos \frac{x}{2}}=\cot \frac{x}{2}\)

Therefore, equation (1) becomes, y=\(\cot ^{-1}\left(\cot \frac{x}{2}\right)\)

⇒ \(y=\frac{x}{2}\)

⇒ \(\frac{d y}{d x}=\frac{1}{2} \frac{d}{d x}(x)\)

∴ \(\frac{d y}{d x}=\frac{1}{2}\)

Question 7. \((\log x)^{\log x}, x>1\)

Solution:

Let y=\((\log x)^{\log x}\)

Taking logarithms on both sides, we obtain

⇒ \(\log y=\log (\log x)^{\log x}\)

⇒ \(\log y=\log x \cdot \log (\log x)\)

Differentiating both sides with respect to x, we obtain

⇒ \(\frac{1}{y d y}=\frac{d}{d x}[\log x \cdot \log (\log x)]\)

⇒ \(\frac{1}{y d y}=\log (\log x) \cdot \frac{d}{d x}(\log x)+\log x \cdot \frac{d}{d x}[\log (\log x)]\)

⇒ \(\frac{d y}{d x}=y\left[\log (\log x) \cdot \frac{1}{x}+\log x \cdot \frac{1}{\log x} \cdot \frac{d}{d x}(\log x)\right]\)

⇒ \(\frac{d y}{d x}=y\left[\frac{1}{x} \cdot \log (\log x)+\frac{1}{x}\right] \Rightarrow \frac{d y}{d x}\)

=\((\log x)^{\log x}\left[\frac{1}{x}+\frac{\log (\log x)}{x}\right]\)

Question 8. \(\cos (a \cos x+b \sin x)\), for some constant a and b.

Solution:

Let y=\(\cos (a \cos x+b \sin x)\)

By using the chain rule, we obtain

⇒ \(\frac{d y}{d x} =\frac{d}{d x} \cos (a \cos x+b \sin x)\)

⇒ \(\frac{d y}{d x} =-\sin (a \cos x+b \sin x) \cdot \frac{d}{d x}(a \cos x+b \sin x)\)

=-\(\sin (a \cos x+b \sin x) \cdot[a(-\sin x)+b \cos x]=(a \sin x-b \cos x) \cdot \sin [a \cos x+b \sin x]\)

Question 9. \((\sin x-\cos x)^{(\sin x-\cos x)}, \frac{\pi}{4}<x<\frac{3 \pi}{4}\)

Solution:

Let y=\((\sin x-\cos x)^{(\sin x-606 x)}\)

Taking logarithms on both sides, we obtain

⇒ \(\log y=\log \left[(\sin x-\cos x)^{\sin x-\cos x)}\right]\)

⇒ \(\log y=(\sin x-\cos x) \cdot \log (\sin x-\cos x)\)

Differentiating both sides with respect to x, we obtain

⇒ \(\frac{1}{y d y}=\frac{d}{d x}[(\sin x-\cos x) \log (\sin x-\cos x)]\)

⇒ \(\frac{1 d y}{y d x}=\log (\sin x-\cos x) \cdot \frac{d}{d x}(\sin x-\cos x)+(\sin x-\cos x) \cdot \frac{d}{d x} \log (\sin x-\cos x)\)

⇒ \(\frac{1}{y} \frac{d y}{d x}=\log (\sin x-\cos x) \cdot(\cos x+\sin x)+(\sin x-\cos x) \cdot \frac{1}{(\sin x-\cos x)} \cdot \frac{d}{d x}(\sin x-\cos x)\)

⇒ \(\frac{d y}{d x}=y[(\cos x+\sin x) \cdot \log (\sin x-\cos x)+(\cos x+\sin x)]\)

∴ \(\frac{d y}{d x}=(\sin x-\cos x)^{(\sin x-\cos x)}(\cos x+\sin x)[1+\log (\sin x-\cos x)]\)

Question 10. Find \(\frac{d y}{d x}, if y=12(1-\cos t), x=10(t-\sin t),-\frac{\pi}{2}<t<\frac{\pi}{2}\)

Solution:

It is given that, y=12\((1-\cos t), x=10(t-\sin t)\)

⇒ \(\frac{\mathrm{dx}}{\mathrm{dt}}=\frac{\mathrm{d}}{\mathrm{dt}}[10(\mathrm{t}-\sin t)]=10 \cdot \frac{\mathrm{d}}{\mathrm{dt}}(\mathrm{t}-\sin t)=10(1-\cos \mathrm{t})\)

and\(\frac{\mathrm{dy}}{\mathrm{dt}}=\frac{\mathrm{d}}{\mathrm{dt}}[12(1-\cos \mathrm{t})]\)

=12 \(\cdot \frac{\mathrm{d}}{\mathrm{dt}}(1-\cos \mathrm{t})=12 \cdot[0-(-\sin t)]=12 \sin t\)

⇒ \(\frac{\mathrm{dy}}{\mathrm{dx}}=\frac{\left(\frac{\mathrm{dy}}{\mathrm{dt}}\right)}{\left(\frac{\mathrm{dx}}{\mathrm{dt}}\right)}\)

=\(\frac{12 \sin \mathrm{t}}{10(1-\cos \mathrm{t})}=\frac{12 \cdot 2 \sin \frac{\mathrm{t}}{2} \cdot \cos \frac{\mathrm{t}}{2}}{10 \cdot 2 \sin ^2 \frac{\mathrm{t}}{2}}\)

= \(\frac{6}{5} \cot \frac{\mathrm{t}}{2}\)

Question 11. If x \(\sqrt{1+y}+y \sqrt{1+x}=0, for -1<x<1\), prove that \(\frac{d y}{d x}=-\frac{1}{(1+x)^2}\) It is given that, x \(\sqrt{1+y}+y \sqrt{1+x}=0 \Rightarrow x \sqrt{1+y}=-y \sqrt{1+x}\)

Solution:

Squaring both sides, we obtain

⇒ \(x^2(1+y)=y^2(1+x) \Rightarrow x^2+x^2 y=y^2+x y^2\)

⇒ \(x^2-y^2=x y^2-x^2 y\)

⇒ \(x^2-y^2=x y(y-x)\)

(x+y)(x-y)=x y(y-x)

⇒ \(x+y=-x y \)

(1+x) y=-x

⇒ \(y=\frac{-x}{(1+x)}\)

Differentiating both sides with respect to x, we obtain

⇒ \(\frac{d y}{d x}=-\frac{(1+x) \frac{d}{d x}(x)-x \frac{d}{d x}(1+x)}{(1+x)^2}=-\frac{(1+x)-x}{(1+x)^2}=-\frac{1}{(1+x)^2}\)

Hence, proved.

Question 12. If \(\cos y=x \cos (a+y) with \cos a \neq \pm 1\), prove that \(\frac{d y}{d x}=\frac{\cos ^2(a+y)}{\sin a}\).

Solution:

⇒ \(\cos y=x \cos (a+y) \Rightarrow x=\frac{\cos y}{\cos (a+y)}\)

Differentiating both sides with respect to y. we obtain

⇒ \(\frac{d x}{d y}=\frac{\cos (a+y) \frac{d}{d y}(\cos y)-\cos y \frac{d}{d y}(\cos (a+y))}{\cos ^2(a+y)}\)

=\(\frac{-\cos (a+y) \sin y+\cos y \sin (a+y)}{\cos ^2(a+y)} \)

⇒ \(\frac{d x}{d y}=\frac{\sin (a+y-y)}{\cos ^2(a+y)}=\frac{\sin a}{\cos ^2(a+y)} (\sin (A-B)=\sin A \cos B-\cos A \sin B)\)

∴ \(\frac{d y}{d x}=\frac{\cos ^2(a+y)}{\sin a}\)

Question 13. If x=a\((\cos t+t \sin t)\) and y=a\((\sin t-t \cos t)\), find \(\frac{d^2 y}{d x^2}\)

Solution:

It is given that, x=a\((\cos t+t \sin t) and y=a(\sin t-t \cos t)\)

⇒ \(\frac{\mathrm{dx}}{\mathrm{dt}}=\mathrm{a} \cdot \frac{\mathrm{d}}{\mathrm{dt}}(\cos \mathrm{t}+\mathrm{t} \sin \mathrm{t})\)

=a\(\left[-\sin t+\sin t \cdot \frac{d}{d t}(t)+t \cdot \frac{d}{d t}(\sin t)\right]=a[-\sin t+\sin t+t \cos t]=a t \cos t \)

and \(\frac{d y}{d t}=a \cdot \frac{d}{d t}(\sin t-t \cos t)\)

=a\(\left[\cos t-\left\{\cos t \cdot \frac{d}{d t}(t)+t \cdot \frac{d}{d t}(\cos t)\right\}\right]=a[\cos t-\{\cos t-t \sin t\}]=a t \sin t \)

⇒ \(\frac{d y}{d x}=\frac{\left(\frac{d y}{d t}\right)}{\left(\frac{d x}{d t}\right)}=\frac{a t \sin t}{a t \cos t}=\tan t\)

Then, \(\frac{d^2 y}{d x^2}=\frac{d}{d x}\left(\frac{d y}{d x}\right)=\frac{d}{d x}(\tan t)\)

=\(\sec ^2 t \cdot \frac{d t}{d x}=\sec ^2 t \cdot \frac{1}{a t \cos t}[\frac{d x}{d t}=a t \cos t\)

⇒ \(\frac{d t}{d x}=\frac{1}{a t \cos t}]\)

=\(\frac{\sec ^3 \mathrm{t}}{\mathrm{at}}, 0<\mathrm{t}<\frac{\pi}{2}\)

Question 14. If f(x)=\(|x|^3\), show that \(f^{\prime \prime}(x)\) exists for all real x, and find it.

Solution:

It is known that, \(|\mathrm{x}|=\begin{cases}{cl}\mathrm{x}, & \text { if } \mathrm{x} \geq 0 \\ -\mathrm{x}, & \text { if } \mathrm{x}<0\end{cases}\).

Therefore when x \(\geq 0, f(x)=|x|^3=x^3\).

In this case, \(f(x)=3 x^2\) and hence, \(f^{\prime}(x)=6 x\)

When x<0, \(f(x)=|x|^3=(-x)^2=-x^3\)

In this case, \(f(x)=-3 x^2 and hence, f^{\prime \prime}(x)=-6 x\)

Thus, for \(f(x)=|x|^3, f^{\prime}(x)\) exists for all real x and is given by, \(f^{\prime \prime}(x)=\begin{cases}{cl}6 x, & \text { if } x \geq 0 \\ -6 x, & \text { if } & x<0\end{cases}\).

Question 15. Using the fact that \(\sin (A+B)=\sin A \cos B+\cos A \sin B\) and the differentiation, obtain the sum formula for cosines.

Solution:

⇒ \(\sin (A+B)=\sin A \cos B+\cos A \sin B\)

Differentiating both sides with respect to X, we obtain

⇒ \(\frac{d}{d x}[\sin (A+B)]=\frac{d}{d x}(\sin A \cos B)+\frac{d}{d x}(\cos A \sin B)\)

⇒ \(\cos (A+B) \cdot \frac{d}{d x}(A+B)\)

= \(\cos B \cdot \frac{d}{d x}(\sin A)+\sin A \cdot \frac{d}{d x}(\cos B)+\sin B \cdot \frac{d}{d x}(\cos A)+\cos A \cdot \frac{d}{d x}(\sin B)\)

⇒ \(\cos (A+B) \cdot \frac{d}{d x}(A+B)\)

=\(\cos B \cdot \cos A \frac{d A}{d x}+\sin A(-\sin B) \frac{d B}{d x}+\sin B(-\sin A) \cdot \frac{d A}{d x}+\cos A \cos B \frac{d B}{d x}\)

⇒ \(\cos (A+B) \cdot\left[\frac{d A}{d x}+\frac{d B}{d x}\right]=(\cos A \cos B-\sin A \sin B) \cdot\left[\frac{d A}{d x}+\frac{d B}{d x}\right]\)

⇒ \(\cos (A+B)=\cos A \cos B-\sin A \sin B\)

Question 16. If y=\(\left|\begin{array}{ccc}f(x) & g(x) & h(x) \\ \ell & m & n \\ a & b & c\end{array}\right|\), prove that \(\frac{d y}{d x}=\left|\begin{array}{ccc}f^{\prime}(x) & g^{\prime}(x) & h^{\prime}(x) \\ \ell & m & n \\ a & b & c\end{array}\right|\)

Solution:

y=\(\left|\begin{array}{ccc}
f(x) & g(x) & h(x) \\
\ell & m & n \\
a & b & c
\end{array}\right|\)

y=\((m c-n b) f(x)-(\ell c-n a) g(x)+(c b-m a) h(x)\)

Then, \(\frac{d y}{d x} =\frac{d}{d x}[(m c-n b) f(x)]-\frac{d}{d x}[(c-n a) g(x)]+\frac{d}{d x}[(f b-m a) h(x)]\)

=\((m c-n b) f^{\prime}(x)-(f c-n a) g^{\prime}(x)+((b-m a) h^{\prime}(x)\).

=\(\left|\begin{array}{ccc}
f^{\prime}(x) & g^{\prime}(x) & h^{\prime}(x) \\
f & m & n \\
a & b & c
\end{array}\right|\)

Thus, \(\frac{d y}{d x}=\left|\begin{array}{ccc}
f^{\prime}(x) & g^{\prime}(x) & h^{\prime}(x) \\
c & m & n \\
a & b & c
\end{array}\right|\)

Question 17. If y=\(e^{3 \cos ^{-1} x},-1 \leq x \leq 1\), show that \(\left(1-x^2\right) \frac{d^2 y}{d x^2}-x \frac{d y}{d x}-a^2 y\)=0.

Solution:

It is given that, y=\(e^{0.06 E^{-1} x}\)

Taking logarithms on both sides, we obtain.

⇒ \(\log y=\log e^{a \cos ^{-1} x}\)

⇒ \(\log y=a \cos ^{-1} x \log e \Rightarrow \log y=a \cos ^{-1} x\)

Differentiating both sides with respect to x, we obtain

⇒ \(\frac{1}{y} \frac{d y}{d x}=a \times \frac{-1}{\sqrt{1-x^2}}\)

⇒ \(\frac{d y}{d x}=\frac{-a y}{\sqrt{1-x^2}}\)

By squaring both the sides, we obtain

⇒ ⇒ \(\left(\frac{d y}{d x}\right)^2=\frac{a^2 y^2}{1-x^2}\)

⇒ \(\left(1-x^2\right)\left(\frac{d y}{d x}\right)^2=a^2 y^2\)

Again differentiating both sides with respect to x, we obtain

⇒ \(\left(\frac{d y}{d x}\right)^2 \frac{d}{d x}\left(1-x^2\right)+\left(1-x^2\right) \times \frac{d}{d x}\left[\left(\frac{d y}{d x}\right)^2\right]\)

=\(a^2 \frac{d}{d x}\left(y^2\right)\)

⇒ \(\left(\frac{d y}{d x}\right)^2(-2 x)+\left(1-x^2\right) \times 2 \frac{d y}{d x} \cdot \frac{d^2 y}{d x^2}=a^2 \cdot 2 y \cdot \frac{d y}{d x}\)

-x\( \frac{d y}{d x}+\left(1-x^2\right) \frac{d^2 y}{d x^2}=a^2 \cdot y \left[\frac{d y}{d x} \neq 0\right]\)

⇒ \(\left(1-x^2\right) \frac{d^2 y}{d x^2}-x \frac{d y}{d x}-a^2 y\) = 0

Hence Proved